You are on page 1of 465

2021

SSC EDITION

BILINGUAL
ADVANCE
MATHS
CGL (TIER-I & II), CPO, CHSL
,oa vU; ijh{kkvksa ds fy, mi;ksxh
„ ALGEBRA (chtxf.kr)
„
„
CO-ORDINATE GEOMETRY (funs'Z kkad T;kfefr)
TRIGONOMETRY (f=kdks.kfefr) 202EiU0
„ HEIGHT & DISTANCE (m¡QpkbZ ,oa nwjh)
u
esa l
„ GEOMETRY (T;kfefr) ijh{kk iSV
uZ ij
„ MENSURATION (2D & 3D) ({ks=kfefr) vkèkkfjr
DEO ARYAN
(MATHS WIZARD)

FOR FRANCHISEE CONTACT :

RU-67, PITAMPURA, DELHI-110034, Ph : 9821874015, 9821643815


BLAM–1
Delhi : RU-67, For the
Opposite Power House, books & magazines
of Kiran Prakashan
Pitampura, Delhi-110034,
contact your nearest
Ph. : 9821874015, booksellers.
9821643815 For detailed information
log on our website :
www.kiranprakashan.com www.kiranprakashan.com

© KIRAN INSTITUTE OF CAREER EXCELLENCE PVT. LTD. (KICX)


NEW EDITION
The copyright of this book is entirely with the Kiran Institute of Career Excellence Pvt. Ltd. The reproduction of
this book or a part of this will be punishable under the Copyright Act.
All disputes subject to Delhi jurisdiction.
Every possible effort has been made to ensure that the information contained in this book is accurate at the
time of going to press, and the publishers and authors cannot accept responsibility for any errors or omissions,
however caused. No responsibility for loss or damage occasioned to any person acting, or refraining from
action, as a result of the material in this publication can be accepted by the editor, the publisher or any of the
authors.

Reviewed by : Think Tank of PRATIYOGITA KIRAN, KIRAN PRAKASHAN & KICX


Assistance : z Govind Pd. Singh z Sanjay Singh z Sanket Sah
Design & Layout by : KICX COMPUTER SECTION, New Delhi.

Must Read USEFUL FOR ALL Buy Today


COMPETITIVE EXAMS

Price : 239 Price : 525 Price : 495


BLAM–2
vuqØef.kdk
1. chtxf.kr (ALGEBRA) ................................................................................. BLAM–5
„ PREVIOUS YEARS’ QUESTIONS ........................................................... BLAM–13
 Based on Square and Cubic Formulae


1 1
Based on a or a Formulae
a a

 Factor Theorem and Questions Based on Other Formulae


„ ADVANCED LEVEL QUESTIONS ................................................... BLAM–77

2. funsZ'kkad T;kfefr (CO-ORDINATE GEOMETRY) ................................ BLAM–91


„ PREVIOUS YEARS’ QUESTIONS ......................................................... BLAM–101
„ ADVANCED LEVEL QUESTIONS ................................................. BLAM–126

3. f=kdks.kfefr (TRIGONOMETRY) ...................................................... BLAM–131


„ PREVIOUS YEARS’ QUESTIONS ......................................................... BLAM–145
 Degree, Radians, buosQ vkil esa Conversions, clock osQ hands osQ chp esa
cus angle ij based questions
 Angles ij based questions
 Trigonometric identities ij based questions
 Trigonometric functions dh minimum, maximum values, substitution, rFkk
series ij based questions
 sin (x + y), cos (x + y), tan (x + y) etc. formulae ij based questions
 Miscellaneous
„ ADVANCED LEVEL QUESTIONS ................................................. BLAM–202

4. m¡QpkbZ ,oa nwjh (HEIGHT AND DISTANCE) ...................................... BLAM–219


„ PREVIOUS YEARS’ QUESTIONS ......................................................... BLAM–221
„ CHALLENGER QUESTIONS ......................................................... BLAM–242

BLAM–3
5. T;kfefr (GEOMETRY) ..................................................................... BLAM–255
„ PREVIOUS YEARS’ QUESTIONS ......................................................... BLAM–273
 Questions Based on Angles, (Complementary, supplementry,
angle bisector etc.
 Questions Based on Length of sides of a Triangle
 Questions Based on Polygons
 Questions Based on Quadrilaterals (Rhombus, Trapezium etc.)
 Questions Based on Circles
„ ADVANCED LEVEL QUESTIONS ................................................. BLAM–361

6. {ks=kfefr (MENSURATION) .............................................................. BLAM–375


„ PREVIOUS YEARS’ QUESTIONS ......................................................... BLAM–389
 Questions Based on Area and perimeter of Triangle, Quadrilaterals and Circle
 Questions Based on Area and Perimeter of Four Walls of a Room
 Questions Based on Surface Area & Volume of Cube and Cuboid
 Questions Based on Cylinder, Cone, Sphere, Hemisphere etc.

„ ADVANCED LEVEL QUESTIONS ................................................. BLAM–446

xf.kr fo"k;d ç'uksa dks pqVfd;ksa esa gy djus ds fy, D;k


vkidk CONFIDENCE iQht djus yk;d gS \
iqLrd dh fo'ks"krk,¡ %
Ç izLrqr iqLrd esa vè;k;okj iz'uksa dk ladyu fuEufyf[kr vè;k;ksa osQ
vUrxZr fd;k x;k gSA
z la[;k i¼fr z f}vk/kjh; vadu i¼fr z tksM+ vkSj ?kVko z oSfnd xf.kr z xq.kk
z Hkkx z foHkktdrk z y?kqÙke lekioR;Z vkSj egÙke lekioÙkZd z n'keyo la[;k
vkSj fHkUu z ?kkr] ?kkrkad] oxZewy vkSj ?kuewy z chtxf.kr z js[kh; lehdj.kksa osQ xzkiQ
z ljyhdj.k z izfr'krrk z ykHk vkSj gkfu z vkSlr z vuqikr vkSj lekuqikr z feJ.k
z lk>snkjh z vk;q z lk/kj.k C;kt z pØo`f¼ C;kt z le; vkSj dk;Z
z dk;Z vkSj etnwjh z le; vkSj nwjh z jsyxkM+h z uko vkSj /kjk z nkSM+ vkSj [ksy
z f=kHkqt z o`Ùk z prqHkqZt z {ks=kiQy vkSj ifjfefr z i`"Bh; {ks=kiQy vkSj vk;ru 5
49
z f=kdks.kferh; vuqikr z f=kdks.kferh; lg:I;rk z m¡QpkbZ vkSj nwjh z la[;k Js.kh :
z Js.khØe vkSj vuqØe z Øep; vkSj lap; z izkf;drk z y?kqx.kd z oSQysUMj
ice
Pr
z vk¡dM+ksa dk fo'ys"k.k z vk¡dM+ksa dh i;kZIrrk z vk¡dM+ksa dh O;k[;k
Ç iqLrd dh ikB~; lkexzh dks fofo/ izfr;ksfxrk ijh{kkvksa esa iwNs x;s iz'uksa osQ iSVuZ ,oa mudh xq.kork dks oSKkfud ,oa rF;iw.kZ
fo'ys"k.k osQ i'pkr~ rS;kj fd;k x;k gSA
Ç vki 'kk;n brus vè;k;ksa esa oLrqijd ,oa oLrqfu"B iz'uksa dk ladyu ,d iqLrd esa dgha ugha ik,¡xsA

BLAM–4
chtxf.kr

1 chtxf.kr (ALGEBRA)
Algebra ij based questions yxHkx gj competitive Degree of polynomial in Two or more variables :
exams esa iwNs tkrs gSaA pw¡fd algebra ls lHkh Candidates ;fn fdlh polynomial esa nks ;k nks ls T;knk variables gksrs
Hkyh&Hkkafr voxr gksaxs blfy, algebra based questions djus gSa rks polynomial dh degree gj ,d term esa variables dh
esa T;knk dfBukbZ ugha gksrh gSA ;gk¡ ij t:jr gS rks cl questions tks vf/dre power gksrh gS mlosQ cjkcj gksrh gSA
osQ Type dks tkuus rFkk formulae dks lgh izdkj ls questions Example :
esa iz;ksx djus dhA (i) 7x4 y – 5x3y + 9y2 polynomial dh degree 5 gS D;ksfa d
Constants : Generally, og lHkh symbols ftudh value x4y variable dh power 5 gS tks fd bl equation esa
lcls vf/d gSA
2
change ugha gksrh gS constant dgykrs gSa] for e.g. 4, 5, 5
, (ii) 3x3 – 5x4y3 + 9y2 polynomialdh degree 7 gS D;ksafd
etc. budks a, b, c etc. ls Hkh n'kkZ;k tkrk gSA variable x y dh power 7 gS tks fd bl equation esa
4 3

Variables : og lHkh symbols ftudh value change


lcls vf/d gSA
gks rh jgrh gS variables dgykrs gSaA For e.g. circle dk Classification of polynomials on the basis of
degrees.
circumference gksrk gS c = 2 r ;gk¡ 2, constants gS rFkk r
(i) Linear equation : ftl fdlh polynomial dh degree
(radius) variable gSA
1 gksrh gS mls linear polynomial dgrs gSaA
Algebraic Expressions fdlh Hkh ,slh equation dks
e.g. : 3 + 5x, 5 – 3y
n'kkZrh gS ftlesa constants rFkk variables ,d nwljs osQ lkFk (+,
– , ×, ÷) osQ lkFk tqM+s gksrs gSaA (ii) Quadratic equation : ftl fdlh polynomial dh
For example, degree 2 gksrh gS mls quadratic polynomial dgrs gSaA
e.g. : 5x2 + 4x + 3, 3y2 – 2y + 1
1 3
2
4 + 5x – 7x +
3
x ,d algebraic expression gS (iii) Cubic Polynomial : ftl fdlh polynomial dh degree
3 gksrh gS mls Cubic polynomial dgrs gSaA
1 3
ftldh 4 terms gS 4, 5x, – 7x2 rFkk 3
x . e.g. : x3 + 3x2 + 5x + 3 etc.
(iv) Bi Quadratic Polynomial : ftl fdlh polynomial
Polynomials : ,d ,slh algebraic expression ftlosQ
variables dh power integers gksrh gS dks polynomial dgrs gSaA
dh degree 4 gksrh gS mls bi quadratic polynomial
dgrs gSaA
Polynomials osQ fofHkUu :i
e.g. : 8x4 + 3x3 + 5x2 + 4 etc.
(i) 3 + 4x2 + 7x3 – polynomial ftlesa fliZQ ,d
Number of terms in a Polynomial :
variable x gSA
(ii) 5x2y + 3xy2 + 7 – polynomial ftlesa nks variables
fdlh Hkh polynomial dks mldh terms dh la[;k osQ vuqlkj
monomial, binomial, trinomial etc. osQ :i esa classify
x rFkk y gSA
djrs gSaA
(iii) 5x + 7x 2 + 7 – polynomial ugha gS D;ksafd bldh ,d
2 5

Example :
term esa x dh power 5 2 gS tks fd integer ugha gSA
(i) Monomials esa fliZQ ,d term gksrh gS]
Degree of Polynomials in one variable : 4, 7x, 9 xy, 5x2 yz etc.
,d variable okys polynomial esa ml variable dh vfèkdre (ii) Binomials esa nks terms gksrh gS]
highest power gh polynomial dh degree dgykrh gSA 4 + 3x, 5x – 3y, 3x2 + 2y etc.
Example : (iii) Trinomials esa rhu terms gksrh gS]
(i) 2x – 3 ,d polynomial gS ftlesa x dh power 1 gS] 3
5x + 2x + 3, xy + yz + zx etc.
blfy, bl polynomial dh degree 1 gksxhA Constant polynomial : ,d ,slk polynomial ftldh
(ii) 5x4 – 3x2 + 5 polynomial dh degree 4 gS D;ksfa d x dh fliZQ ,d term gksxh vkSj oks Hkh constant mls constant
vf/dre power 4 gSA polynomial dgk tkrk gSA

BLAM–5
chtxf.kr
Example : 3, 5, –5, 7 etc. Rule for Multiplication :
Ter ms o f pol yn om ia l i n as c en di ng o r (i) Like signs okyh nks values dks multiply djus ij sign
descending order. +ve gksrk gS rFkk unlike sign dh nks values dks multiply
;fn fdlh polynomial dh terms c<+rs Øe esa gks rks mls djus ij sign negative gksrk gSA
ascending polynomial dgsa xs vkSj ;fn terms dh power ?kVrs For Ex. 5 × 4 = 20 (like signs)
Øe esa gks rks descending polynomial dgsaxsA –5 × –4 = 20 (like signs)
Ex. : (i) 3– 5x + 7x2 – 9x3 (Ascending polynomial) 5 × –4 = –20 (unlike signs)
(ii) 9x3 + 7x2 – 5x + 3 (Descending polynomial) (ii) tc nks ;k nks ls T;knk variables dks multiply djrs gSa rFkk
Like Terms : og lHkh terms ftuosQ variables rFkk muosQ base same gks rks powers add gks tkrh gSA
exponents same gksrs gSa like terms dgykrh gS vU;Fkk mUgsa like xm × xn = xm+n (Bases are same)
unlike terms dh category esa j[kk tkrk gSA Ex. 7x3 × –5x4 = – 35x7
Example : (2x2 –5) (3x + 3) = 6x3 + 6x2 – 15x –15
(i) 3x2, 5x2, – 7x2 – Like terms Division of a polynomial by a polynomial ,d
(ii) 5x2y, 3xy 2, 7x4 – Unlike terms. polynomial dks nwljs ls rc rd ge divide djrs gSa tc rd dh
remainder zero ;k remainder lowest form esa u vk tk,A
Mathematical operations on two or more
polynomials. Example

(i) Rule for addition and subtraction. 2


2
5 x – 9x + 3
nks ;k nks ls vf/d polynomials dh like terms dks gh 2 x + 7x –1 10x4 + 17x3 – 62x2+ 3x – 5
4 3 2
10x + 35x – 5x
vkil esa add ;k subtract fd;k tk ldrk gSA Unlike terms – – +
3 2
esa addition rFkk subtraction djus ij terms same jgrh gS –18x – 57x + 3x – 5
3 2
–18x – 63x + 9x
fliZQ subtraction esa mudk fpÉ cny tkrk gSA + + –
6x2 – 6x – 5
Ex. (i) Add 2
6x + 21x – 3
3x2 – 5xy + 7y2 – 3x – – +
–27x – 2
5x2 – 7xy + 3y2
+ x2 – 2xy + 8x Remember :
9x2 – 14xy + 10 y2 + 5x Dividend = Divisor × Quotient + Remainder
(ii) Subtract 3x2 + 5x + 7 from 5x2 + 9x + 8 HkkT; = Hkktd × HkkxiQy + 'ks"k
Sol. : 5x2 + 9x + 8
Division method ls remainder irk djus esa cgqr le;
3x2 + 5x + 7
yxrk gS blfy, remainder theorem dk iz;ksx fd;k tkrk gSA
– – –
tc fdlh polynominal (degree > 1) dks x – a ls divide
2x2 + 4x + 1
fd;k tk, rks lcls vklku rjhdk gS bldk remainder fudkyus dk
Sign convention : (i) x– a = 0 (x – a dks zero put djsa)
Algebraic expression esa sign convention cgq r x=a
important gksrk gSA bu rules dks è;ku ls le>saA vc x = a dh value polynomial esa p (x) osQ txg j[k nsAa
Rule I : (+) + (+) = Add djsa vkSj sign (+) dk vk,xk ;fn p (a) = 0 gks rks x – a fn, x, polynomial dk factor
Ex. : 2x + 3x = 5x gksxk_ ugha rks factor ugha gksxkA
Rule II : (+) + (–) = Subtract djsa vkSj sign ges 'kk
Ex.1. Without actual division, find the remainder
cM+h la[;k osQ sign tSlk gksxkA when x4 – 3x3 + 4x2 – 6x + 7 is divided by x – 1/
Ex. (i) 4x2 – 9x2 = – 5x2 (– sign D;ksafd la[;k 9 cM+h fcuk Hkkx dh fØ;k fd;s 'ks"kiQy irk djsa tc x4 – 3x3 +
gS vkSj bldk sign (–) dk gS)A 4x2 – 6x + 7 dks x – 1 ls Hkkx fn;k tkrk gSA
(ii) 15x2 – 10x2 = 5x2 (+ sign D;ksafd la[;k 15 Sol. ;gk¡ (x – 1) divisor gS
cM+h gS vkSj bldk sign (+) dk gS) blfy,, x = 1
Rule III : (–) + (–) = Add djsa vkSj sign (–) dk gksxkA (x–1) ls divide djus ij remainder = P (1)
P(1) = (1)4 – 3(1)3 + 4(1)2 – 6(1) + 7
Ex. (i) – 4x2 – 9x2 = – 13x2
=1–3+4–6+7=3

BLAM–6
chtxf.kr
3 2 3
Ex.2. If the expressions Px + 3x –3 and 2x – 5x + P Some Important Results :
when divided by x – 4 leave same remainder. Find (A)
the value of P. / ;fn expressions Px3 + 3x2–3 vkSj
2x3 – 5x + P dks tc x – 4 ls divide fd;k tkrk gS rks 1
;fn x + x = a gks rks
remainder leku vkrk gSA P dk eku D;k gS\
Sol.Remainders gksaxs 1
R1 = f (4) = P (4)3 + 3 (4)2 – 3 = 64P + 45 (1) ;fn a = 2 rks xn + =2
xn
R2 = f (4) = 2 (4)3 – 5 (4) + P = P + 108
tcfd, R1 = R2, blfy, 1
64P + 45 = P + 108 (2) x2 + = a2 – 2
x2
or, 63 P = 63 P=1
Divisibility dh Conditions. 1
(3) x3 + = a3 – 3a
(i) x + a rc gh (x + a) ls divisible gksxkA tc n dh value
n n x3
odd gksxhA
1
(ii) xn + an, (x + a) ls divisible ugha gksxk tc n dh value (4) x4 + = (a2 – 2)2 – 2
x4
even gksxhA
(iii) xn + an, (x – a) ls dHkh divisible ugha gksxkA 1
(iv) xn – an, (x + a) ls rHkh divisible gksxk tc n dh value (5) x5 + = (a3 – 3a) (a2 – 2) –a = a5 – 5a3 + 5a
x5
even gksxhA
(v) xn – an, x – a ls ges'kk divisible gksxk pkgs n odd gks ;k 1
(6) x6 + = (a3 – 3a)2 – 2
even. x6
FACTORIZING A POLYNOMIAL
List of Important Formulae : 1
(1) (a + b)2 = a2 + 2ab + b2 (7) x7 + = {(a2 – 2)2 – 2} × (a3 – 3a) –a
x7
(2) (a – b)2 = a2 – 2ab + b2
(3) (a + b)2 = (a – b)2 + 4ab 1
(4) (a – b)2 = (a + b)2 – 4ab (8) x8 + = {(a2 – 2)2 – 2}2 – 2
(5) a2 – b2 = (a + b) (a – b) x8
(6) a2 + b2 = (a + b)2 – 2ab (B)
(7) a2 + b2 = (a – b)2 + 2ab
(8) (a + b + c)2 = a2 + b2 + c2 + 2ab + 2ac + 2bc) 1
(9) a3 + b3 = (a + b) (a2 – ab + b2) ;fn x – x = a gks rks
(10) a3 – b3 = (a – b) (a2 + ab + b2)
(11) (a + b)3 = a3 + b3 + 3ab (a + b)
1
(12) (a – b)3 = a3 – b3 – 3ab (a – b) (1) x2 + = a2 + 2
(13) a3 + b3 = (a + b)3 – 3ab (a + b) x2
(14) a3 – b3 = (a – b)3 + 3ab (a – b)
(15) a3 + b3 + c3 – 3abc = 1
(a + b + c) (a2 + b2 + c2 – ab – bc – ac) (2) x3 – = a3 + 3a
x3
1
= (a + b + c) (2a2 + 2b2 + 2c2 – 2ab – 2bc – 2ac) 1
2
(3) x4 + = (a2 + 2)2 – 2
1 x4
= (a + b + c) [(a – b)2 + (b – c)2 + (c – a)2]
2
(16) (a + b + c)3 1
(4) x5 – = (a3 + 3a) (a2 + 2) – a = a5 + 5a3 + 5a
= a3 + b3 + c3 + 3 (b + c) (c + a) (a + b) x5
(17) ;fn a3 + b3 + c3 = 3abc
rc (i) a + b + c = 0 1
(5) x7 – = {a2 + 2}2 – 2} × (a3 + 3a) + a
1 x7
(ii) [(a – b)2 + (b – c)2 + (c – a)2] = 0
2 Example For A :
a=b=c
1
(18) (a + b)2 + (a – b)2 = 2(a2 + b2) If x + 3 then find that
(19) (a + b)2 – (a – b)2 = 4ab x
(20) a4 + b4 + a2b2 = (a2 + b2 + ab) (a2 + b2 – ab)
(21) a4 – b4 = (a + b) (a – b) (a2 + b2) 1
(i) x2 + = 32 – 2 = 7
(22) a8 – b8 = (a – b) (a + b) (a2 + b2) (a4 + b4) x2

BLAM–7
chtxf.kr
1 1
(ii) x3 + = 33 – 3 × 3 = 18 (1) ;fn x4 + = a, rc
x3 x4

1 (i) x2 +
1 =
(iii) x4 + = (33 – 2)2 – 2 a 2 =b
x 4 x2
= (7)2 – 2 1
(ii) x = b 2
= 49 – 2 = 47 x
1 1
(iv) x5 + = (33 – 3 × 3) (32 – 2) – 3 (iii) x = b 2
x5 x
= 18 × 7 – 3 1
= 123
Ex. ;fn x4 + = 119
x4
1 1
(v) x6 + = (33 – 3 × 3)2 – 2 x2 + = 119 2 = 11
x6 x2
= (18)2 – 2
1 1
= 322 x = 11 2 = 13 ; x = 11 2 = 3
x x
1
(vi) x7 + = {(32 – 2)2 – 2} × (33 – 3 × 3) – 3 1
x7 (2) ;fn x = 2, rc x = 1 gksxk
x
= (49 – 2) (27 – 9) – 3
1
= 47 × 18 – 3 (3) If x = –2 rc x = –1 gksxk
= 846 – 3 = 843 x

(vii) x8 +
1
= {(32 –2)2 – 2}2 – 2 (4) x 3
1
=
FG x 1 IJ 3
IJ FG
3 x
1
x8 x3 H x K K H x
= (49 – 2)2 – 2
= (47)2 – 2 (5) x 3
1
= Gx
F 1I
J 3FGH x 1x IJK
3

= 2209 – 2 = 2207 x3 H xK
Example For B : 1
(6) ;fn x + = 1, rc x3 = –1
1 x
If x – 4 then find that
x 1
(7) ;fn x + = –1 rc x3 = 1,
x
1
(i) x2 + = 42 + 2 = 18
x2 1 FG 1 IJ 2
(8) ;fn x + = 3 ;k x =3
1
x H x K
(ii) x3 – = 43 + 3 × 4 = 76
x3 1
rc x3 + =0 x6 = –1
1
x3
(iii) x4 +
x4
= (42 + 2)2 – 2 ;k
x6 + 1 = 0
= 324 – 2
1
= 322 Ex. ;fn x + =6 gks] rks bu lcdh value fudkfy,
x
1
(iv) x5 – = (43 + 3 × 4) (42 + 2) – 4 1 1 1
x5 (i) x 2 (ii) x4 + 4 (iii) x8 +
x2 x x8
= (76 × 18) – 4
= 1368 – 4 = 1364 1
Sol. x2 + = 62 –2 = 34
x2
1
(v) x7 – = {(42 + 2)2 – 2} × {43 + 3 × 4} + 4 1
x7 x4 + = 342 –2 = 1154
x4
= (324 – 2) (64 + 12) + 4
= (322 × 76) + 4 1
x8 + = 11542 – 2 = 1331714
= 24472 + 4 = 24476 x8

BLAM–8
chtxf.kr
Quadratic equation and its Roots Conditions ('krs)±
(f}?kkr lehdj.k rFkk blosQ ewy) (i) D < 0 rks roots Real ugha gksxas vFkkZr~ roots Imaginary
Quadratic equation (f}?kkr lehñ)%& dksbZ lehdj.k gksxsaA
ax2 + bx + c = O osQ from esa gks tgk¡ a 0 rFkk a, b, c lHkh (ii) D = 0 rks roots Real rFkk equal gksaxs
Real Numbers gks rks og lehdj.k Quadratic equation (iii) D > 0rks roots Real rFkk unequal (vleku) gksaxs
dgykrk gSA Note : ;fn D dk Value ,d Perfect Square Number
where,
vkrk gS] rks roots Rational (ifjes;) gksxk vU;Fkk Irrational
a = coefficient of x2
(vifjes;)
b = coefficient of x
For ex–
c = constant (vpj jkf'k)
For Ex– The roots of 2x2 – 6x + 3 = 0 are–
(i) 4x2 – 3x + 1 = 0
lehdj.k 2x2 – 6x + 3 = 0 osQ ewy gksaxs
2 n (a) Real, Unequal and rational
(ii) 3x – 2x + 4 = 0 are Quadratic eq .
2 1 (okLrfod] leku rFkk ifjes;)
(iii) 9x – 3x + =0
2 (b) Real, Unequal and Irrational
Note : oSlk equation ftldk power (degree), rational (okLrfod] vleku rFkk vifjes;)
Numbers ;k Negative Integers esa gks og Quadratic eqn. osQ
(c) Real and equal (okLrfod rFkk leku)
vUrxZr ugha vk;sxk
(d) Imaginary (dkYifud)
For Ex–
–2
Sol.(b) Given,
(1) 4x + 3x + 2 = 0 n 2x2 – 6x + 3 = 0
3 are not Quadratic eq .
–1 a = 2, b = – 6, c = 3
(2) 5x 4 + 2x + 1 = 0
So,
Roots of Quadratic equation (f}?kkr lehdj.k osQ
D = b2 – 4ac
ewy)% = (–6)2 – 4 × 2 × 3
Quadratic eqn (ax2 + bx + c = 0) osQ nks Roots rFkk = 36 – 24
gksrs gS = 12
where, Here; D > O and D is not a Perfect square so
roots are real, unequal and Irrational.
b b2 4ac b b2 4ac 2. The roots of 3x2 + 7x + 2 = 0 are–
&
2a 2a (a) Real, unequal and rational

Sum of Root : + =
b (okLrfod] leku rFkk ifjes;)
a (b) Real, unequal and Irrational
c (okLrfod] vleku rFkk vifjes;)
Product of Root : =
a (c) Real and equal (okLrfod rFkk leku)
ax2 + bx + c = 0
(d) Imaginary (dkYifud)
b c Sol. (a) 3x2 + 7x + 2 = 0
x2 + x + =0
a a D = (7)2 – 4 × 3 × 2 = 49 – 24 = 25

x2 –
FG b IJ x +
c
=0
D > O and D is a Perfect Square
hence, roots of given eqn are real, unequal and
HaK a
rational.
x2 (Sum of Root) x Product of Root 0 Ex.(1). Find the Quadratic Equation whose one Root

* vxj] vkSj ,d nwljs osQ Reciprocal gSa rc] is 5 5


a=c Sol. 1st Root = 5 5
* vxj] vkSj magnitude esa Equal gSa vkSj Sign opposite
2nd Root = 5 – 5
gS rc]
b=0 Sum = (5 5 ) (5 – 5 ) 10
* vxj] a, b, c Rational number gSa rc
Product = (5 5 ) (5 – 5 ) 20
igyk Root = a b , nwljk Root = a b
ewyksa dh izÑfr (Nature of Roots) :– x2 – (sum of Root) x + Product of Root = 0
x2 – 10x + 20 = 0
Quadratic eqn (ax2 + bx + c = 0) osQ Discriminant
Ex. (2) Which of the following equations has real
foospd ;k fofoDrdj) gksxa s roots?
D = b2 –4ac fuEu esa ls dkSu&lk lehdj.k dk ewy okLrfod gS\
BLAM–9
chtxf.kr
2
(a) 3x + 4x + 5 = 0
b b a b b b
(b) x2 + x + 4 = 0 = 0
(c) (x – 1) (2x – 5) = 0 a a b a = a a
(d) 2x2 – 3x + 4 = 0
Ex (5). If x = a2 + b2, y = 2 ab then find the value of
Sol.(c) Th eqn has Real roots If b2 – 4ac > 0
option (a) 3x2 + 4x + 5 = 0
a4 b4
D = (4)2 – 4 × 3 × 5 ?
= – 44 (D < 0) a2 2ab b2
option (b) x2 + x + 4 = 0
D = (1)2 – 4 × 1 × 4 a4 b4
;fn x = a2 + b2 rFkk y = 2 ab rks 2 dk
= 1 – 16 a 2ab b2
= –15 (D < 0) eku gksxkµ
option (c) (x – 1) (2x – 5) = 0
(A) x – y (B) 2xy (C) xy (D) x + y
2x2 – 7x + 5 = 0
D = (–7)2 –4 × 2 × 5 2 2
= 9 (D > 0) a4 b4 ea j eb j
2 2

Sol.(D) =
option (d) 2x2 – 3x + 4 = 0
D = (–3)2 –4 × 2 × 4
a2 2ab b2 ea 2
b2 2ab j
= –23 < 0 2 2
option (c) is Right answer. ea 2
b2 j e 2ab j x2 y2
Ex (3). If the sum of the squares of the roots of x2 – (P = =
– 2)x – (P + 1) = 0 (where, P R) is 5, then what is
ea 2
b 2
2ab j x y

the value of P? =x+y


;fn x2 – (P – 2)x – (P + 1) = 0 (tgk¡ P R) lehdj.k osQ Ex (6).If x2 + x = 5 then find the value of (x + 3)3 +
ewyksa osQ oxks± dk ;ksx 5 gS rks P dk eku D;k gksxk\ 1
?
(A) 0 (B) –1 (C) 1 (D)
3 bx 3g 3

2
1
Sol.(c) Given, ;fn x2 + x = 5 rks (x + 3)3 + dk eku Kkr djsa\
x2 – (P – 2)x – (P + 1) = 0 b x 3 g3

+ = (P – 2) , = – (P + 1) (A) 250 (B) 150 (C) 110 (D) 220


According to Question
2
Sol. (c) : Let P=x+3
+ 2 =5
x=P–3
( + )2 – 2 = 5
we have,
(P – 2)2 + 2(P + 1) = 5
x2 + x = 5
(P2 – 4P + 4 + 2P + 2 = 5
(P – 3)2 + (P – 3) = 5
P2 – 2P + 1 = 0
P2 + 9 – 6P + P – 3 = 5
(P – 1)2 = 0
P2 – 5P + 1 = 0
P=1
Ex (4). If and are the roots of the equation ax2 + P2 + 1 = 5P
bx + b = 0 then what is the value of 1
P+ =5
P
b
? Now
a
1
;fn rFkk fn;s x;s lehdj.k ax2 + bx + b = 0 osQ ewy gS P3 + 53 3 5 110
P3
b put the value of P
rks dk eku D;k gksxk\
a 1
110.
(x + 3)3 +
(A) –1 (B) 0 (C) 1 (D) 2 bx 3g 3

Sol.(b) Given, Ex (7).If x2 + y + z, y2 = x + z, z2 = x + y then what is


ax2 + bx + b = 0
1 1 1
the value of x 1 ?
b b y 1 z 1
+ = and =
a a ;fn x 2 = y + z , y 2 = x + z, z 2 = x + y rk s
Now, we have
1 1 1
b x 1 y 1 z 1 dk eku D;k gksxk\
a (A) 1 (B) 0 (C) –1 (D) 2

BLAM–10
chtxf.kr
2
Sol. (a) Given, x = y + z
Adding Both sides by x
1
x=1– y =
y 1 b1 y g
x2 + x = x + y + z y y
x (x + 1) = x + y + z
x 1 Now, xyz =
b1 y g y
1
1.
x y z x 1
y b1 y g
Ex (10).If a, b, c, are all positive then the minimum
1 x value of the expression
x 1 x y z
ea 2
a 1 b2je b 1 c2 je c 1 j is:
1 x abc
Similarly, y 1 x y z
;fn a, b, c, lHkh ?kukRed gks rks O;atd
1 z
z 1 x y z
ea 2
a 1 b2je b 1 c2 je c 1 j dk U;wure eku D;k
abc
Now,
gksxk\
1 1 1 (A) 3 (B) 9 (C) 27 (D) 1
x 1 y 1 z 1 Sol. (c) Expression dk Minimum Value gksxk
x y z tc a = b = c
x y z
= x z = x y z 1.
y
z x z x y y
Min. Value =
b1 1 1 g b1 1 1 g b1 1 1 g
Ex (8).If ab – b + 1 = 0 and bc – c + 1 = 0, then what 1 1 1
is (a – ac) equal to? =3×3×3
;fn ab – b + 1 = 0 rFkk bc – c + 1 = 0 rks (a – ac) foQlosQ = 27
cjkcj gSa\ Condition for Common Roots :–
Let the two quadratic Equation be
(A) –1 (B) 0 (C) 1 (D) 2 (i) p1x2 + q1x + r1 = 0
Sol. (c) : Given ab – b + 1 = 0 (ii) p2x2 + q2x + r2 = 0
b (a – 1) = – 1
vxj ,d Root Common gks] rc
1 1 (p1q2 – p2q1) (q1r2 – q2r1) = (r1p2 – r2p1)2
b=
a 1 1 a vxj nks Roots common gks] rc
and, bc – c + 1 = 0 bc = c – 1...(1)
p1 q1 r1
Now, Put the value of b in eqn
bc = c – 1 p2 q2 r2

1 * Quadratic equation dh maximum rFkk minimum


c =C–1 value
1 a
(i) tc a > 0 gks rc
c = (c – 1) (1 – a)
c = c – 1 – ac + a 4ac b 2
a – ac = 1 minimum (U;wure) value =
4a
1 1 (ii) tc a < 0 gks rc
Ex (9).If y 1 and x 1 . What is the Value
z y
4ac b 2
of xyz ? maximum (vf/dre) value =
4a
1 1
;fn y 1 rFkk x 1 rks xyz dk eku D;k gksxk\ (iii) ;fn x, y fn, gks rks] x + y dh value minimum gksxh
z y
tc x = y gksxkA
1 Examples :
(A) 1 (B) –1 (C) 0 (D)
2 1. x + y dh minimum value fudkysa] ;fn x y = 16
1 Minimum value osQ fy,
Sol. (b) Given y 1 x=y=4
z
Minimum value x + y = 4 + 4 = 8
1 1 1 2. ;fn x + y + z = 24 gks rks (x – 1) (y – 2) (z + 3)
=1–y z = 1 y and, x + y 1
z dh maximum value crk,¡ \

BLAM–11
chtxf.kr
Sol. maximum value osQ fy, (iii) x dk eku Equation esa j[ksa vkSj Maximum & Minimum
(x – 1) = (y – 2) = (z + 3) = m value Kkr djsaA
x=m+1 Note :
y=m+2
Quadratic Equation dks Differentiate djus osQ ckn x
z=m–3
x + y + z = 24 dk coefficient vxj +ve jgs rks Minimum value fudysxk,
x, y rFkk z dh value equation esa j[kus ij m + 1+ m + x dk coefficient –ve jgs rks Maximum value fudysxkA
2 + m – 3 = 24 Ex. y = x2 – 6x + 10
m=8 dy
= 2x – 6 + 0
maximum value = m3 = 83 = 512 dx
3 The minimum value of (x – 2) (x – 9) is ?
dy
(x – 2)(x – 9) dk U;wure eku D;k gS\ =0
Sol. Expression = (x – 2) (x – 9) dx
= x2 – 11x + 18 = ax2 + bx + c 2x – 6 = 0
6
4ac b 2 x= =3 (y esa j[kus ij Minimum value fudysxk,
Minimum value (a > 0) 2
4a
x dk coefficient + ve gS)
4 1 18 121 49 Minimum value = x2 – 6x + 10
4 4 = (3)2 – 6 × 3 + 10
4. If x is real, then the minimum value of (x2 – x + 1) =1
is ? * How to Find Minimum value, vxj Expression
;fn x ,d okLrfod la[;k gS] rks (x2 – x + 1) dk fuEure eku 1 1
x2 , tgk¡ x R ;k x , x Positive jgs
gks xkµ x2 x
Sol. Expression ax2 + bx + c, a > 0 ds fy, minimum
4ac  b 2 x2
1 FG x 1 IJ 2
2
value ¾
4a
x 2 H x K
Here, for x2 – x + 1, a = 1, b = –1, c = 1 1
So, Minimum value = 2, tc x =0 x2 = 1
4 1 1 1 3 x
Minimum value
4 1 4 1
5. Find the value of x for which the expression 2 – Ex. Find the minimum value of x 2 5
x2
3x – 4x2 has the greatest value.
x dk eku Kkr dhft, ftlds fy, O;atd 2 – 3x – 4x2 dk 1
Sol. x 2 5 = 2 – 5 = –3
egÙke eku gSµ x2
Sol. Expression = 2 – 3x – 4x2 = – (4x2 + 3x – 2) * vxj x + y fn;k gS] rc xy Maximum gksxk
LM OP tc x = y
2
= – (2x ) 2 2x
3 FG 3 IJ FG 3 IJ
2 2

2 e.g. x + y = k (given)
MN 4 H 4K H 4K PQ K
x=y=
LMF 2x 3 I OP F 3 I
2 2 2
=–
MNGH 4 JK PQ GH 4 JK 2
xy =
K K K2
(Maximum)
Expression dk eku maximum gksxk ;fn 2 2 4
Ex. If x + y = 8, then Maximum value of xy
3 3
2x + =0 2x = – 8
4 4 Sol. x = y = =4
2
3
x=– xy = 4 × 4 = 16
8
* vxj xy fn;k gS] rc x + y Minimum gksxk
Method to Find Maximum & Minimum value tc x = y (x & y are positive Numbers)
by using Differentiation. e.g. xy = 64 (given)
(i) Take Differentiation of given equation x=y=K
i.e. xn = nxn – 1 K.K = 64
x3 = 3x2 K2 = 64
x2 = 2x K=8
(ii) Put Differentiation = 0, then Final value of x. x + y = 8 + 8 = 16 (Minimum)

BLAM–12
chtxf.kr

PREVIOUS YEARS’ QUESTIONS


(1) 96 (2) 100 (3) 102 (4) 108
Type-1 (SSC SAS Exam, 27.06.2010)
Based on Square and Cubic Formulae 1 1
7. If x3 + y3 = 35 and x + y = 5, then the value of x y
1. If a = 23 and b = –29 then the value of 25a2+ 40ab
+ 16b2 is : will be :

;fn a = 23 rFkk b = –29 gks] rks 25a2+ 40ab + 16b2 dk 1 1


;fn x3 + y3 = 35 rFkk x + y = 5 gks] rks x y dk eku D;k
eku D;k gksxk \
(1) 1 (2) –1 (3) 0 (4) 2 gksxk\
FCI Assistant Grade-III Exam.05.02.2012 (Paper-I)
1 5 2
2. If a + b = 5, a2 + b2 = 13, then the value of (a – b) (1) (2) (3) 6 (4)
3 6 3
(where a > b)—
(SSC CHSL DEO & LDC Exam. 21.10.2012 (IInd Sitting)
;fn a + b = 5, a2 + b2 = 13, rks a – b dk eku crk,¡ (tgk¡
x y xy
a > b)— 8. If x – y = , the numerical value of xy is
7 4
(1) 1 (2) – 2 (3) 2 (4) – 1
(SSC COP SI Assistant Intelligence Officer Exam, 2012) x y xy
3. Factors of 48 x3 –8x2 –93x – 45 are ? ;fn x – y = 7 4
gks] rks xy dk eku fdruk gksxk \
3 2
48 x –8x –93x – 45 osQ xq.ku[kaM D;k gS\
4 3 1 1
(1) (4x + 3) (4x – 3) (3x – 5) (1) (2) (3) (4)
3 4 4 3
(2) (4x – 3) (4x – 3) (3x – 5)
(SSC CHSL DEO & LDC Exam. 11.12.2011
(3) (4x + 3) (4x + 3) (3x – 5) (Ist Sitting (East Zone)
(4) (4x – 3) (4x + 3) (3x + 5) 9. If a2 = 2, then (a + 1) equal to :
(SSC CHSL – 2017)
;fn a2 = 2 gks] rks (a + 1) cjkcj gksxk %
4. Which of the following is not a quadratic equa-
tion? 2 a 1 a 1
(1) a –1 (2) (3) (4)
a 1 3 2a 3 2a
buesa ls dkSu&lk ,d f}?kkr lehdj.k ugha gS\
(SSC CGL Tier-II Exam, 01.08.2010)
(1) 2x (x + 4) – 11 = x (x – 3) + 6
10. If x = b + c – 2a, y = c + a – 2b, z = a + b – 2c, then
(2) 4x (x + 4) – 11 = 5x (x – 3) + 5 the value of x2 + y2 – z2 + 2xy is
(3) x (x + 2) –15 = x (x – 5) + 11
;fn x = b + c – 2a, y = c + a – 2b, z = a + b – 2c, gks]
(4) 2x2 + 8x – 11 = 0
rks x2 + y2 – z2 + 2xy dk eku D;k gksxk\
(SSC CHSL–2017)
(1) 0 (2) a + b + c
2 1 1 (3) a – b + c (3) a + b – c
5. If a b2 4 , then the value of a2 +
a2 b2 (SSC CHSL DEO & LDC Exam. 04.12.2011
(Ist Sitting (East Zone)
b2:
11. If x2 + y2 – 2x + 6y + 10 = 0, then the value of x2 +
1 1 y2 :
;fn a2 b2 4 gks] rks a2 + b2 dk eku gksxk %
a2 b2 ;fn x2 + y2 – 2x + 6y + 10 = 0 gks] rks (x2 + y2) dk eku
1 1 gksxk %
(1) 1 (2) 1 (3) 2 (4) 2
2 2 (1) 4 (2) 6 (3) 8 (4) 10
(SSC CGL Tier-II Exam, 01.08.2010) (SSC CGL Tier-II Exam, 01.08.2010)

6. For positive numbers a and b if a2 + b2 = 24 and 12. If a + b + c = 27, then what is the value of (a – 7)3
ab = 6, then (a3 + b3) will equal to : + (b – 9)3 + (c – 11)3 –3 (a – 7) (b – 9) (c – 1) ?

nks /ukRed la[;kvksa a rFkk b ds fy,] ;fn a2 + b2 = 24 rFkk ;fn a + b + c = 27, rks (a – 7)3 + (b – 9)3 + (c – 11)3
ab = 6 gS rks (a3 + b3) cjkcj gksxkµ –3 (a – 7) (b – 9) (c – 11) dk eku D;k gS\

BLAM–13
chtxf.kr
(1) 0 (2) 9 (3) 27 (4) 81 20. If x + y = a and xy = b2, then the value of x3 – x2y
(SSC CGL – 2017) – xy2 + y3 in terms of a and b is :
;fn x + y = a vkSj xy = b2, rks a rFkk b ds :i esa x3 – x2y
x2 y2 z2
13. If = 3 then What is the Value of (x – xy2 + y3 dk eku gS%
yz zx xy
(1) (a2+ 4b2) a (2) a3 – 3b2
+ y + z)3 ?
(3) a3 – 4b2 a (4) a3 + 3b2
(SSC CHSL DEO & LDC Exam. 11.12.2011
x2 y2 z2
;fn yz zx xy
=3 rks (x + y + z)3 dk eku D;k gS\ (IInd Sitting (Delhi Zone)

11 13x 11 13y 11 13z


(1) 0 (2) 1 (3) 2 (4) 3 21. If 5 then what is
x y z
(SSC CGL – 2017)
14. If x = a – b, y = b – c, z = c – a, then the numerical
1 1 1
value of the algebraic expression x3 + y3 + z3 – the value of x
3xyz will be y z =?

;fn x = a – b, y = b, z = c – a gks] rks chth; O;atd x3 +


11 13x 11 13y 11 13z
y3 + z3 – 3xyz dk la[;kRed eku D;k gksxk \ ;fn 5 rks
x y z
(1) a + b + c (2) 0
(3) 4 (a + b + c) (4) 3 abc 1 1 1
(SSC CAPFs SI & CISF ASI Exam. 23.06.2013) x y z dk eku D;k gksxk\
15. If a (a + b + c) = 45, b (a + b + c) = 75 and c (a + b
+ c) = 105, then what is the value of (a2 + b2 + 13 13
c2)? (1) 1 (2) (3) (4) 4
11 5
;fn a (a + b + c) = 45, b (a + b + c) = 75 rFkk c (a +
(SSC CGL – 2017 )
b + c) = 105 rks (a2 + b2 + c2) dk eku D;k gksxk\
(1) 78 (2) 83 (3) 217 (4) 225 22. If x3 + 3x2 + 3x = 7, then x is equal to :
(SSC CGL – 2017)
;fn x3 + 3x2 + 3x = 7 gks] rks x cjkcj gksxk %
16. If a = 1.21, b = 2.12 and c = –3.33, then the value
of a3 + b3 + c3 – 3abc is (1) 2 (2) 3
6 (3) 1 (4) –1
;fn a = 1.21, b = 2.12 vkSj c = –3.33 gks] rks a3 + b3 + (SSC CGL, Tier-II Exam, 01.08.2010)
c3 – 3abc dk eku gSµ
5x y 3
(1) 0 (2) 1 (3) 2 (4) 3 23. If 5x y = 7 then what is the value of
(SSC CGL Prelim Exam. 24.02.2002 (Middle Zone)
17. If x + y = 7, then the value of x3 + y3 + 21xy is
;fn x + y = 7 gks] rks x3 + y3 + 21xy dk eku gksxkµ
d4 x 2
y2 4 xy h
dk eku D;k gksxk\
(1) 243 (2) 143 (3) 343 (4) 443 d 9x 2 16y 2 24 xy h
(SSC CGL Prelim Exam. 04.02.2007 (Second Sitting)
18. (y – z)3 + (z – x)3 + (x – y)3 equal to /cjkcj gksxk % 3 18 1
(1) 0 (2) (3) (4)
(1) 3 (y – z) (z + x) (y – x) (2) (x – y) (y + z)(x – z) 7 49 6

(3) 3 (y – z) (z – x) (x – y) (4) (y – z) (z – x) (x – y) 24. If (x + y)2 = xy + 1 and x3 – y3 = 1 then what is the


(SSC CHSL DEO & LDC Exam. 04.12.2011 value of x – y?
(Ist Sitting (East Zone)
;fn (x + y)2 = xy + 1 rFkk x3 – y3 = 1 rc x – y dk eku
19. If x = a (b – c), y = b (c – a) and z = c (a – b), then
D;k gksxk\
;fn x = a(b – c), y = b(c – a) vkSj z = c(a–b), rks
(1) 1 (2) 0 (3) – 1 (4) 2
FG x IJ + FG y IJ + FG z IJ
3 3 3
=
25. If x = y = 333 and z = 334, then the value of x3 + y3
HaK HbK H c K + z3 – 3xyz is
;fn x = y = 333 vkSj z = 334, gks] rks x3 + y3 + z3 – 3xyz
xyz 3xyz xyz
(1)
3abc
(2) 3 xyzabc (3)
abc
(4)
abc
dk eku D;k gksxk\
(SSC CHSL DEO & LDC Exam. 11.12.2011 (1) 0 (2) 667 (3) 1000 (4) 2334
(Ist Sitting (Delhi Zone) (SSC Graduate Level Tier-II Exam. 29.09.2013)

BLAM–14
chtxf.kr
26. If x + y + z = 6 and + x2 y2 + z2 = 20 then the value 33. If x – y – 18 = 1 and x + y – 3 2 =1 then what
of x3 + y3 + z3 – 3xyz is
is the value of 12xy (x2 – y2) ?
;fn x + y + z = 6 vkSj x2 + y2 + z2 = 20 rks x3 + y3 + z3
– 3xyz dk eku D;k gksxk \ ;fn x – y – 18 = –1 rFkk x + y – 3 2 = 1 gks rks 12xy
(1) 64 (2) 70 (3) 72 (4) 76 (x2 – y2) dk eku D;k gS\
(SSC Graduate Level Tier-I Exam. 21.04.2013)
(1) 0 (2) 1 (3) 512 2 (4) 612 2
27. If a – b = 3 and a3 – b3 = 117 then a b is equal
34. If p = 99, then the value of p(p2 + 3p + 3) is
to ;fn p = 99 rks p(p2 + 3p + 3) dk eku D;k gksxk\
;fn a – b = 3 rFkk a3 – b3 = 117 gks] rks a b fdlds (1) 10000000 (2) 999000
(3) 999999 (4) 990000
cjkcj gksxk\
(SSC CGL Tier-II Exam. 21.09.2014)
(1) 3 (2) 5 (3) 7 (4) 9 35. If a, b, c are real numbers and a2 + b2 + c2 = 2 (a –
(SSC CHSL DEO & LDC Exam. 27.10.2013 IInd Sitting)
b – c) – 3, then the value of a + b + c is
28. If (x – 2) and (x + 3) are the factors of the equation
;fn a, b, c okLrfod la[;k,¡ gSa vkSj a2 + b2 + c2 = 2 (a –
x2 + k1x + k2 = 0 then what are the values of
K1 and K2 ? b – c) – 3, rks a + b + c dk eku D;k gksxk \
(1) – 1 (2) 1 (3) 3 (4) 0
;fn lehdj.k x2 + k1x + k2 = 0 osQ xq.ku[kaM (x – 2) rFkk
(SSC CGL Tier-II Exam, 2014 12.04.2015
(x + 3) gS rks k1 rFkk k2 dk eku D;k gS\ (Kolkata Region) TF No. 789 TH 7)
(1) k1 = 6, k2 = – 1 (2) k1 = 1, k2 = –6 36. If x2 + y2 + z2 = 2 (x + z – 1), then what is the value
(3) k1 = 1, k2 = 6 (4) k1 = –6, k2 = 1 of x3 + y3 + z3 = ?
29. If a + b + c = 6, a2 + b2 + c2 = 14 and a3 + b3 + c3 = ;fn x2 + y2 + z2 = 2 (x + z – 1), rks fuEufyf[kr dk eku gS
36, then the value of abc is x3 + y3 + z3 = ?
;fn a + b + c = 6, a2 + b2 + c2 = 14 vkSj a3 + b3 + c3 (1) 2 (2) 0 (3) –1 (4) 1
(SSC CGL Tier-I Exam, 16.08.2015 (IInd Sitting)
= 36 gksa] rks abc dk eku D;k gksxk \
37. If x = z = 225 and y = 226, then the value of :
(1) 3 (2) 6 (3) 9 (4) 12 x3 + y3 + z3 – 3xyz is
(SSC Graduate Level Tier-II Exam. 16.09.2012)
;fn x = z = 225 vkSj y = 226 rks fuEufyf[kr dk eku gS %
1 x3 + y3 + z3 – 3xyz
30. If a3 + b3 = 9 and a + b = 3, then the value of +
a (1) 765 (2) 676 (3) 576 (4) 674
(SSC CGL Tier-I Exam, 16.08.2015
1 (IInd Sitting) TF No. 2176783)
is
b 38. If a2 + b2 + c2 + 3 = 2 (a + b + c), then the value of
(a + b + c) is
1 1
;fn a3 + b3 = 9 vkSj a + b = 3 gS] rks a + b dk eku gSµ ;fn a2 + b2 + c2 + 3 = 2 (a + b + c) rks (a + b + c) dk
eku gSµ
1 3 5 (1) 2 (2) 3 (3) 4 (4) 5
(1) (2) (3) (4) –1 (FCI Assistant Grade-III Exam.25.02.2012 (Paper-I)
2 2 2
North Zone (Ist Sitting)
(SSC CHSL DEO Exam. 02.11.2014 (Ist Sitting)
39. If a2 + b2 + c2 = 2 (a – b – c) – 3, then the value of
31. If x = 997, y = 998 and z = 999, then the value of
(a + b + c) is—
x2 + y2 + z2 – xy – yz – zx is
;fn a2 + b2 + c2 = 2 (a – b – c) – 3, rks (a – b + c) dk
;fn x = 997, y = 998 vkSj z = 999 gS] rks x2 + y2 + z2 –
eku gSµ
xy – yz – zx dk eku gSµ
(1) –1 (2) 3 (3) 1 (4) –2
(1) 0 (2) 1 (3) – 1 (4) 3 (SSC CGL Tier-II Exam, 04.09.2011)
3 2
(SSC CHSL DEO & LDC Exam. 02.11.2014 (IInd Sitting) 40. If the expression px – 2x – qx + 18 is completely
32. If x – y = 7 then what is the value of (x – 15)3 – 2
divisible by (x – 9), then what is the ratio be-
(y – 8)3 ? tween p and q respectively?
;fn x – y = 7 gks rks (x – 15)3 – (y – 8)3 dk eku D;k gS\ 3 2 2
;fn O;atd px – 2x – qx + 18, (x – 9) ls iw.kZr% foHkkftr
(1) 0 (2) 343 (3) 392 (4) 2863 gS rks Øe'k% p rFkk q osQ chp dk vuqikr D;k gksxk\

BLAM–15
chtxf.kr
(1) 1 : 9 (2) 1 : 3 (3) 3 : 1 (4) 1 : 2 47. If a3 + 3a2 + 9a = 81, then what is the value of a3
41. If (x + y – z)2 + (y + z – x)2 + (z + x – y)2 = 0, then 3
what will be the value of (x + y + z ) ? + ?
a
;fn (x + y – z)2 + (y + z – x)2 + (z + x – y)2 = 0 gks] rks
x + y + z dk eku D;k gksxk \ 3
;fn a3 + 3a2 + 9a = 1, rks a3 + a dk eku D;k gksxk ?
(1) 3 (2) 3 3 (3) 3 (4) 0 (1) 31 (2) 26 (3) 28 (4) 24
(SSC CHSL DEO and LDC Exam, 11.12.2011 (SSC CGL Tier-II (CBE) Exam. 17.02.2018)
(Ist Sitting (East Zone)
(a b) 6 (b c ) 9
1 1 1 48. If = and = , then what is the
42. If 0 and x + y + z = 9 then what is the c 5 a 2
x y z
(a c)
value of x3 + y3 + z3 – 3xyz ? value of ?
b
1 1 1
;fn x 0 rFkk x + y + z = 9 rks x3 + y3 + z3 – 6 (b c) 9 (a c)
y z ;fn (a b ) = 5 rFkk = gSa] rks dk
a 2 b
c
3xyz dk eku D;k gksxk\
eku D;k gS\
(1) 81 (2) 361 (3) 729 (4) 6561
(SSC CGL – 2017) 9 11 7 7
(1) (2) (3) (4)
43. If a + b + c = 9 (where a, b, c are real numbers), 5 7 11 4
then the maximum value of a2 + b2 + c2 is (SSC CGL Tier-II (CBE) Exam. 17.02.2018)

;fn a + b + c = 9 ( tgk¡ a, b, c okLrfod la[;k,¡ gSa)] rks 49. If x3 + y3 + z3 = 3 (1 + xyz), P = y + z – x, Q = z + x


– y and R = x + y – z, then what is the value of P3
a2 + b2 + c2 dk vf/dre eku gSµ
+ Q3 + R3 – 3PQR ?
(1) 100 (2) 9 (3) 27 (4) 81
;fn x3 + y3 + z3 = 3(1 + xyz), P = y + z – x, Q = z + x
(SSC CHSL DEO & LDC Exam. 20.10.2013)
44. If x2 + y2 + 1 = 2x, then the value of x3 + y5 is – y rFkk R = x + y – z gSa] rks P3 + Q3 + R3– 3PQR dk eku

;fn x2 + y2 + 1 = 2x, rks x3 + y5 dk eku D;k gS \ D;k gS\


(1) 9 (2) 8 (3) 12 (4) 6
(1) 2 (2) 0 (3) –1 (4) 1
(SSC CGL Tier-II (CBE) Exam. 17.02.2018)
(SSC CGL Tier-I Exam. 19.10.2014)
50. If x1 x2 x3 = 4 (4 + x1 + x2 + x3), then what is the
x2 y2 z2
45. If x + y + z = 0, then the value of is LM 1 OP LM 1 O L 1 O
x2 yz value of
MN b2 x g PQ + MN b2 x
1 2 g PPQ + MMN b2 x 3 g PPQ ?
x2 y2 z2
;fn x + y + z = 0, rks dk eku D;k gS\ LM 1 OP
x2 yz
;fn x1 x2 x3 = 4 (4 + x1 + x2 + x3) gks] rks
(1) –1 (2) 0 (3) 1 (4) 2 MN b2 x g PQ 1
+

(SSC CGL Tier-I Exam. 19.10.2014 TF No. 022 MH 3)


46. If x + y + z = 0, then what is the value of LM 1 O L 1 O
e3y x z j
2 2 2 MN b2 x 2 g PPQ + MMN b2 x 3 g PPQ dk eku D;k gS\
?
e2y – xz j2
(1) 1 (2)
1
(3) 2 (4)
1
2 3
(SSC CGL Tier-II (CBE) Exam. 17.02.2018)
e3y x z j
2 2 2
51. If 3x + 5y + 7z = 49 and 9x + 8y + 21z = 126, then
;fn x + y + z = 0, gks] rks dk eku D;k gS\
e2y – xz j
2 what is the value of y?
;fn 3x + 5y + 7z = 49 rFkk 9x + 8y + 21z = 126 gSa] rks
3 5 y dk eku D;k gS\
(1) 2 (2) 1 (3) (4) (1) 4 (2) 2 (3) 3 (4) 5
2 3
(SSC CHSL DEO & LDC Exam. 17.02.2018)
(SSC CGL Tier-II (CBE) Exam. 17.02.2018)

BLAM–16
chtxf.kr
52. If x + y + z = 0 then what is the value of
a b
x2 y3 z2 57. If a + b = 10 and – 13 = – – 11, then what
b a
?
3z 3 xz 3x
is the value of 3ab + 4a2 + 5b2 ?

x2 y3 z2 a b
;fn x + y + z = 0 rks dk eku D;k gS\ ;fn a + b = 10 rFkk – 13 = – – 11 gaS] rks 3ab
3z 3xz 3x b a
(1) 0 (2) xy (3) y (4) 3y + 4a2 + 5b2 dk eku D;k gS\
(SSC CGL 2017)
(1) 450 (2) 300 (3) 600 (4) 750
53. If x + y + z = 22 and xy + yz + zx = 35, then what
(SSC CGL Tier-II (CBE) Exam. 17.02.2018)
is the value of (x – y)2 + (y – z)2 + (z – x)2?
58. If 3x + 4y – 2z + 9 = 17, 7x + 2y + 11z + 8 = 23 and
;fn x + y + z = 22 rFkk xy + yz + zx = 35 gSa] rks (x – y)2 5x + 9y + 6z – 4 = 18, then what is the value of x
+ (y – z)2 + (z – x)2 dk eku D;k gS\ + y + z – 34?
(1) 793 (2) 681 (3) 758 (4) 715 ;fn 3x + 4y – 2z + 9 = 17, 7x + 2y + 11z + 8 = 23 rFkk
(SSC CGL Tier-II (CBE) Exam. 17.02.2018)
5x + 9y + 6z – 4 = 18 gS] rks x + y + z – 34 dk eku D;k
(x y)
54. If = 2, then what is the value of gS\
z
(1) – 28 (2) – 14 (3) – 31 (4) – 45
LM y OP L x O (SSC CGL Tier-II (CBE) Exam. 17.02.2018)

N (y – z ) Q MN ( x – z ) PQ ? 59. If x + 3y –
2z
= 6, x +
2
(2y + 3z) = 33 and
1
(x +
4 3 7
(x y) LM y OP L x O y + z) + 2z = 9, then what is the value of 46x +
;fn z
=2 gS] rks (y – z )
N Q MN ( x – z ) PQ dk eku D;k 131y ?

gS\ 2z 2 1
;fn x + 3y – 4 = 6, x + 3 (2y + 3z) = 33 rFkk 7 (x +
(1) 0 (2) 1 (3) 2 (4) – 1
(SSC CGL Tier-II (CBE) Exam. 17.02.2018)
y + z) + 2z = 9 gSa] rks 46x + 131y dk eku D;k gS\

55. If a4 + 1 =
LM a OP (4b
2
2
– b4 – 1), then what is the
(1) 414 (2) 364 (3) 384 (4) 464
MN b PQ
2 (SSC CGL Tier-II (CBE) Exam. 17.02.2018)

1 1
value of a4 + b4? 60. If f (x) = – , then what is the value of f (1)
x x 1

;fn a4 + 1 =
LM a OP (4b – b – 1) gS] rks a + b dk eku D;k gS\
2
2 4 4 4
+ f (2) + f (3) +... f (10) ?
MN b PQ
2
1 1
;fn f (x) = – gS] rks f (1) + f(2) + f (3) + ... f (10)
(1) 2 (2) 16 (3) 32 (4) 64 x x 1
(SSC CGL Tier-II (CBE) Exam. 17.02.2018) dk eku D;k gS\
1–a a 9 10
56. If 3 9 19 – 3 , then what is the val- (1) (2)
a 1–a 10 11
ue of a ?
11 12
(3) (4)
1–a a 12 13
;fn 3 9 19 – 3 gS] rks a dk eku D;k gS\
a 1–a (SSC CGL Tier-II (CBE) Exam. 09.03.2018)
61. What is the simplified value of (2 + 1) (22 + 1) (24
3 7 1 9 + 1) (28 + 1)?
(1) , (2) ,
10 10 10 10
(2 + 1) (22 + 1) (24 + 1) (28 + 1) dk ljyhÑr eku D;k gS\
2 3 1 4 (1) 28 – 1 (2) 216 – 1
(3) , (4) ,
5 5 5 5 (3) 232 – 1 (4) 264 – 1
(SSC CGL Tier-II (CBE) Exam. 17.02.2018) (SSC CPO – 2017)

BLAM–17
chtxf.kr
2 2
62.If a + b = 4b + 6a – 13, then what is the value of
68. If P = 7 + 4 3 and PQ = 1, then what is the value
a + b?
;fn a2 + b2 = 4b + 6a – 13 gS] rks a + b dk eku D;k gS\ FG 1 IJ + FG 1 IJ ?
(1) 3 (2) 2 of HP K HQ K
2 2

(3) 5 (4) 10
(SSC CGL Tier-II (CBE) Exam. 09.03.2018)
;fn P = 7 + 4 3 rFkk PQ = 1gSa] rks
FG 1 IJ + FG 1 IJ dk
63. x and y are positive integers. If x4 + y4 + x2y2 = 481
and xy = 12, then what is the value of x2 – xy + y2?
HP K HQ K
2 2

x rFkk y ,d /ukRed iw.kk±d gSA ;fn x4 + y4 + x2y2 = 481 eku D;k gS\
rFkk xy = 12 gS] rks x2 – xy + y2 dk eku D;k gS\ (1) 148 (2) 189
(3) 194 (4) 204
(1) 16 (2) 13
(SSC CGL Tier-II (CBE) Exam. 18.02.2018)
(3) 11 (4) 15
(SSC CGL Tier-II (CBE) Exam. 09.03.2018) 69. If x = 2 + 3 , y = 2 – 3 and z = 1, then what is
64. If A = 1 + 2P and B = 1 + 2-P, then what is the value
of B? FG x IJ FG y IJ + FG z IJ
;fn A = 1 + 2P rFkk B = 1 + 2-P gS] rks B dk eku D;k gS\
the value of
H yz K + H xz K H xy K + 2

(A 1) (A + 2) LMFG 1 IJ F 1 I FG 1 IJ OP
(1)
(A – 1)
(2)
(A + 1) MNH x K GH y JK H z K PQ ?
A (A – 2)
(3)
(A – 1)
(4)
(A +1)
FG x IJ
;fn x = 2 + 3 , y = 2 – 3 rFkk z = 1 gSa] rks H yz K +
(SSC CGL Tier-II (CBE) Exam. 09.03.2018)

65. If e1 – p j e1 – q j =
2 2 3 , then what is the value FG y IJ + FG z IJ FG 1 IJ + FG 1 IJ + FG 1 IJ dk eku D;k gS\
2 H xz K H xy K +2 H xK HyK HzK
(1) 25 (2) 22
of 2p 2 2q 2 2 pq + 2p 2 2q 2 – 2pq ?
(3) 17 (4) 43
(SSC CGL Tier-II (CBE) Exam. 18.02.2018)
3
;fn e1 – p j e1 – q j
2 2
= gS] rks 2p 2 2q 2 2 pq 70. If x + y = 3, then what is the value of x3 + y3 + 9xy?
2
;fn x + y = 3 gks] rks x3 + y3 + 9xy dk eku D;k gS\
+ 2p 2 2
2q – 2pq dk eku D;k gS\ (1) 15 (2) 81
(3) 27 (4) 9
(1) 2 (2) 2 (SSC CGL Tier-II (CBE) Exam. 18.02.2018)
(3) 1 (4) None of these
(SSC CGL Tier-II (CBE) Exam. 09.03.2018) ( x 8 – 1) (y 4 – 1)
71. A = and B = If x = 2 and y = 9,
2
66. If (a + b) – 2 (a + b) = 80 and ab = 16, then what (x 4
1) (y 2 1)
can be the value of 3a – 19b?
then what is the value of A2 + 2AB + AB2?
;fn (a + b) – 2(a + b) = 80 rFkk ab = 16 gSa] rks 3a – 19b
2

dk eku D;k gks ldrk gS\ ( x 8 – 1) (y 4 – 1)


A= rFkk B = gSaA ;fn x = 2 rFkk y = 9
(1) – 16 (2) – 14 (3) – 18 (4) – 20 (x 4 1) (y 2 1)
(SSC CGL Tier-II (CBE) Exam. 09.03.2018)
gSa] rks A2 + 2AB + AB2 dk eku D;k gS\
67. If xy + z = 1, yx + z = 1024 and zx + y = 729 (x, y and z are
(1) 96475 (2) 98625
natural numbers), then what is the value of (z +
(3) 92425 (4) 89125
1)y + x + 1 ?
(SSC CGL Tier-II (CBE) Exam. 18.02.2018)
;fn xy + z = 1, yx + z = 1024 rFkk zx + y = 729 (x, y rFkk z 72. If x – 4y = 0 and x + 2y = 24, then what is the value
izkÑfrd la[;k,¡ gSa)] rks (z + 1)y + x + 1 dk eku D;k gS\
(2x 3y )
(1) 6561 (2) 10000 (3) 4096 (4) 14641 of (2x – 3y ) ?
(SSC CGL Tier-II (CBE) Exam. 09.03.2018)

BLAM–18
chtxf.kr
77. If a + a2 + a3 – 1 = 0, then what is the value of a3
(2x 3y )
;fn x – 4y = 0 rFkk x + 2y = 24 gSa] rks (2x – 3y ) dk eku FG 1 IJ ?
D;k gS\
+ HaK
(1)
9
(2)
11
(3)
13
(4)
9
;fn a + a2 + a3 – 1 = 0 gks] rks a3 + a
FG 1 IJ dk eku D;k gS\
5 5 7 7 H K
(SSC CGL Tier-II (CBE) Exam. 18.02.2018)
(1) 1 (2) 4
F xI FyI
73. If GH JK + GH JK
FxI FyI
= 3 and GH JK – GH JK = 9, then what
(3) 2 (4) 3
a b b a (SSC CGL Tier-II (CBE) Exam. 19.02.2018)

x 78. If
LM ea 2 b
2
ab j OPQ +
LM ea 2 b
2
– ab j OPQ = 1, then what is
is the value of y ? N N

FG x IJ FG y IJ
;fn a + b = 3 rFkk b
FG x IJ – FG y IJ x
gSa] rks y dk
the value of e1 je
a 2 1 b2 ? j
H K H K H K HaK =9

eku D;k gS\ LM F a 2  b 2  ab I OP L ea 2  b 2 – ab j O


(b 3a ) (a 3b )
;fn
NH K Q + MN PQ =

(1) (2)
(a – 3b ) (b – 3a )

2
1 gks] rks e1 a 2 1 b2 je j dk eku D;k gS\
(1 3a ) (a 3b )
(3) (4)
(a 3b ) (b – 3a 2 ) 1 4
(1) (2)
(SSC CGL Tier-II (CBE) Exam. 18.02.2018) 4 7
74. x, y and z are prime numbers and x + y + z = 38.
What is the maximum value of x ? 5 3
(3) (4)
x, y rFkk z vHkkT; la[;k,¡ gSa rFkk x + y + z = 38 gSA x dk 4 4
vf/dre eku D;k gS\ (SSC CGL Tier-II (CBE) Exam. 19.02.2018)

(1) 19 (2) 23 (3) 31 (4) 29


(SSC CGL Tier-II (CBE) Exam. 19.02.2018) ep 2
q2 j pq
79. If
75. If x and y are natural numbers such that x + y =
2017, then what is the value of (– 1)x + (– 1)y ? er 2
s 2
j= rs
, then what is the value of

;fn x rFkk y izkÑfrd la[;k,¡ bl izdkj gS fd x + y = 2017


gSa] rks (– 1)x + (– 1)y dk eku D;k gS\ bp q g in terms of r and s ?
(1) 2 (2) –2 bp q g
(3) 0 (4) 1
(SSC CGL Tier-II (CBE) Exam. 19.02.2018)
ep 2
q2 j pq bp q g
F 1 I ( 3 1) , then what is the value of x
76. If x + GH JK = 4
;fn
er 2
s 2
j= rs
gS rks r rFkk s osQ in esa p q dk b g
x 2
eku D;k gS\
F 1 IJ ?
+ GH
x K 4
r b s g r b s g
(1) r (2) r
F 1 I ( 3 1) gS] rks x + FG 1 IJ dk eku D;k gS\ b s g b s g
;fn x + GH x JK = Hx K 4
4
2
br s g rs

( 4 3 – 1) ( 4 3 1)
(3)
rs
(4) r b s g
(1) (2)
4 2 (SSC CPO – 2017)
80. If and are the roots of the equation x2 – x + 3
(–4 3 – 1) (–4 3 – 1) = 0, then what is the value of 4 + 4?
(3) (4)
4 2 ;fn rFkk lehdj.k x2 – x + 3 = 0 osQ ewy gS rks 4
+ 4

(SSC CGL Tier-II (CBE) Exam. 19.02.2018) dk eku D;k gksxk\

BLAM–19
chtxf.kr
(1) 7 (2) 9
85. If 5 x
12 x
13 x then x is equal to?
(3) 11 (4) 13
(SSC CPO – 217) ;fn 5 x
12 x
13 x gS] rks x fdlosQ cjkcj gS\
3 2 3– 2 25
81. If a = and b = , then what is (1) (2) 4
3– 2 3 2 4
2 2
the value of a + b –ab? (3) 9 (4) 16
(SSC CPO SI–2008)
3 2 3– 2
;fn a = rFkk b = gSa] rks a2 + b2 – ab 86. If a4 + b4 = a2b2, then (a6 + b6) is equals to
3– 2 3 2
;fn a4 + b4 = a2b2 gS] rks (a6 + b6) fdlosQ cjkcj gS\
dk eku D;k gS \ (1) 0 (2) 1
(3) a2 + b2 (4) a2b4 + a4b2
(1) 97 (2) 2 3 + 2 e j (SSC CPO SI–2010)

(3) 4 6e j +1 (4) 98 87. If a = 7 2 12 , b = 7


3 3
2 12 then (a + b ) is
(SSC CGL Tier-II (CBE) Exam. 21.02.2018) equal to
82. If x2 – 16x + 59 = 0, then what is the value of (x – ;fn a = 7 2 12 , b = 7 2 12 gS] rks (a3 + b3) dk
LM 1 OP eku D;k gksxk\
6)2 + MN bx – 6g 2 PQ ? (1) 40 (2) 44
(3) 48 (4) 52
LM 1 OP (SSC SAS 2010)

;fn x2 – 16x – 59 = 0 gS] rks (x – 6) + M 2


PQ dk eku
N b x – 6g 2
88. If 1
x3 3
, then x = ?
100 5
D;k gS\
(1) 14 (2) 18
x3 3
(3) 16 (4) 20 ;fn 1 , gS] rks x = ?
(SSC CGL Tier-II (CBE) Exam. 21.02.2018)
100 5
83. If 2x + 3y – 5z = 18, 3x + 2y + z = 29 and x + y + 3z (1) 2 (2) 4
= 17, then what is the value of xy + yz + zx ? (3) 16 (4) (136)1/3
;fn 2x + 3y – 5z = 18, 3x + 2y + z = 29 rFkk x + y + 3z (SSC CGL Tier-I 2011)
= 17 gSa] rks xy + yz + zx dk eku D;k gS\ 89. If a2 + b2 = 2 and c2 + d2 = 1 then the value of (ad
(1) 32 (2) 52 – bc)2 + (ac + bd)2 is
(3) 64 (4) 46 ;fn a2 + b2 = 2 vkSj c2 + d2 = 1 gS] rks (ad – bc)2 + (ac +
(SSC CGL Tier-II (CBE) Exam. 21.02.2018) bd)2 dk eku D;k gksxk\
FG x 1 IJ FG x 1 IJ FG x 2 1
1 x2
IJ FG 1 IJ
1 4 1
84. H x KH x KH x2 KH x2 K is (1)
9
(2)
2
equal to ? (3) 1 (4) 2
(SSC CPO SI–2012)
FG x 1 IJ FG x 1 IJ FG x 2 1 IJ FG
1 x2
1 IJ
1 fdlosQ 2
H x KH x KH x2 KH x2 K 90. If x 3 5 , then the value of x – 16x + 6 is

cjkcj gS\ ;fn x 3 5 , gS] rks x2 – 16x + 6 dk eku D;k gksxk\


(1) 0 (2) –2
6 1 8 1
(1) x (2) x (3) 2 (4) 4
x6 x8
(SSC CGL Tier-II 2013)

8 1 6 1 91. If ;fn a + b + c = 2S, then rks


(3) x (4) x
x8 x6
(SSC CPO SI - 2006)
bS a g bS b g bS c g
2 2 2
S2
fdlosQ cjkcj gS\
2 2 2
a b c

BLAM–20
chtxf.kr
2 2 2
(1) a + b + c (2) 0
2p 1
(3) 1 (4) 2 2. If ,p 0, then the value of
p2 2p 1 4
(SSC CGL Tier-I 2013)
92. If (x + k) is a common factor of (x2 + px + q) and 1
(x2 + lx + m) then the value of k is: p is
p
;fn (x + k) (x2 + px + q) rFkk (x2 + lx + m) dk mHk;fu"B
xq.ku[kaM gks rks k dk eku D;k gksxk\ 2p 1 1
;fn ,p 0, rks p dk eku gSµ
(1) l + p (2) m – q p2 2p 1 4 p
l p m q (1) 4 (2) 5 (3) 10 (4) 12
(3) m q (4) l p FCI Assistant Grade-III Exam.25.02.2012 (Paper-I)
North Zone (Ist Sitting)
(SSC CGL Tier-II 2013)
3
93. One of the factors of 3x + x – 12x – 4 is 2

3. If 2 x
FG
2 1 IJ – FG x – 1 IJ
3x3 + x2 – 12x – 4 dk ,d xq.ku[kaM gksxkµ H x2 K H xK – 7 = 0, then two values of

(1) 3x2 – 2 (2) 3x – 1 x are


(3) 3x + 1 (4) 3x + 2
94. If 2x = 4y = 8z and xyz = 288 then value of ;fn 2 x2
FG 1 IJ – FG x – 1 IJ – 7 = 0 gS] rks x ds nks eku gSaµ
H x 2 K H xK
1 1 1
2x 4y 8z is: 1 1
(1) 1, 2 (2) 2, – (3) 0, 1 (4) ,1
2 2
1 1 1 (SSC CHSL DEO & LDC Exam. 02.11.2014 (IInd Sitting)
;fn 2x = 4y = 8z vkSj xyz = 288 gks rks 2 x 4y 8z
1 x2 1 2x
=? 4. If x 2 66, then the value of =?
2 x
x
11 11
(1) (2)
12 96 2 1 x2 1 2x
;fn x 66, rks] dk eku = ?
x2 x
29
(3) (4) None of these (1) + 8 (2) 10, – 6 (3) 6, –10 (4) + 4
96
(SSC CGL Tier-I Re-Exam. (2013) 27.04.2014)
95. If ax = (x + y + z)y, ay = (x + y + z)z, az = (x + y + z)x
5. What is difference of the factors of the expres-
then
;fn ax = (x + y + z)y, ay = (x + y + z)z, az = (x + y + z)x 2 1
sion x 6?
gS] rks x2

2 1
(1) 3(x + y + z) = a (2) 2a = x + y + z O;atd x 6 osQ xq.ku[kaMksa dk varj D;k gS\
x2
a (1) 0 (2) 1 (3) 2 (4) 4
(3) x + y + z = 0 (4) x = y = z =
3 (SSC CGL – 2017)

Type-II 5x 1 FG x 1 IJ
6. If
2x 2
5x 1 3
, then the value of H 2x K is
1 1
Based on a or a Formulae
a a
;fn
5x 1
rks
FG x 1 IJ dk eku D;k gksxk \
2x 2 5x 1 3
, H 2x K
1 6 1 (1) 15 (2) 10 (3) 20 (4) 5
1. If x 4 , then what is the value of x ?
x x6 (SSC CGL Tier-I Re-Exam. (2013) 20.07.2014 (IInd Sitting)

1
1 6 1 7. If 2x – = 6, then what will be the value of
;fn x x
4 gS] rks x 6 dk eku D;k gS\ 2x
x
(1) 52 (2) 256 (3) 1026 (4) 2702 1
x2 ?
(SSC CGL – 2017) 16 x 2

BLAM–21
chtxf.kr
1 2 1 1 1 1
;fn 2x – =6 gks] rks x dk eku D;k gksxk\ ;fn x 4 , gks rks x81 + dk eku D;k gS\
2x 16 x 2 x 4 2 x 81
(1) –2 (2) 0 (3) 1 (4) 2
19 17 18 15
(1) (2) (3) (4) (SSC CGL–2017)
2 2 3 2
(SSC CGL Tier-I Exam. 2012) 13. If x
1
2 , then the value of
FG x 2 1 IJ
x H x2 K
1
8. If x +
x
= 2, then what is the value of x64 + x121 ? FG x 3 1 IJ is
H x3 K
1
;fn x + x = 2 gS rks x64 + x121 dk eku D;k gksxk\
;fn x
1
2 , rks
FG x 2 1 IJ FG x 3 1 IJ dk eku gSµ
(1) 0 (2) 1 (3) 2 (4) –2
x H x2 K H x3 K
(SSC CGL – 2017)
(1) 20 (2) 4 (3) 8 (4) 16
(SSC CGL Tier-I Re-Exam. (2013) 20.07.2014 (Ist Sitting)
9. If x = 6 + 2 6 , then what is the value of
2 1
14. If x 1 then what is the value of x48 +
1 x2
x 1 ?
x 1 x42 + x36 + x30 + x24 + x18 + x12 + x6 + 1?

1
x 1
1 ;fn x 2 1 gS rks x48 + x42 + x36 + x30 + x24 +
;fn x = 6 + 2 6 , rc x 1
dk eku D;k gS\ x2
x18 + x12 + x6 + 1 dk eku D;k gksxk\
(1) 2 3 (2) 3 2 (3) 2 2 (4) 3 3 (1) –9 (2) 0 (3) 1 (4) 2
(SSC-CGL-2017) (SSC CGL – 2017)
10. If (x – a) (x – b) = 1 and a – b + 5= 0, then the value
9
15. If 2x 9 then what is the minimum value of
1 x
of (x – a)3 – is
bx – a g 3
1
x2 ?
;fn (x – a) (x – b) = 1 rFkk a – b + 5 = 0 gks] rks (x – a)3 x2
1 9 1
– dk eku D;k gksxk\ ;fn 2x 9 gS] rks x
2
dk U;wure eku D;k gksxk\
b x –a g 3
x x2
(1) –125 (2) 1 (2) 125 (4) 140 95 97 86 623
(SSC Graduate Level Tier-II Exam. 29.09.2013) (1) (2) (3) (4)
36 36 25 27
(SSC CGL – 2017)
2 15
11. If x = , then what is the value of 2
3 5 1 x 3x 1
16. If x + 1 , then the value of 2 is
x x 7x 1
x 5 x 3
?
x 5 x 3 1 x2 3x 1
;fn x + 1 gS] rks 2 dk eku gS \
x x 7x 1
2 15 x 5 x 3
;fn x = , rks dk eku D;k gS\ 3 1
3 5 x 5 x 3 (1) 1 (2) (3) (4) 2
7 2
(1) (2) (3) 15 (4) 2 (SSC CGL Tier-I Exam, 09.08.2015
5 3
(IInd Sitting) TF No. 4239378)
(SSC – CGL – 2017)
1 1 is
1 1 17. If x 2 then the value of x 12
12. If 4 4 , then what is the value of x81 x x 12
x x 2
(1) 2 (2) – 4 (3) 0 (4) 4
1 (SSC CGL Tier-I Exam, 09.08.2015
+ ?
x 81 (IInd Sitting) TF No. 4239378)

BLAM–22
chtxf.kr
1 5 1 1
18. If x 5 then what is the value of x ? 23. If x 0 then what is the value of
x x5 b x 7g
;fn x
1
5 gS] rks x
5 1
dk eku D;k gS\
LMx 1 OP ?
x5
x
MN b x 7g PQ
(1) 1875 (2) 2525 (3) 2530 (4) 3120
(SSC CGL – 2017)
1 LMx 1 OP
;fn x 0 gS] rks
19. If 4a
4
3 = 0, then the value of : a
3 1
3 b x 7 g MN bx 7g PQ dk eku D;k
a a3
gksxk\
=?
(1) 3 5 (2) 3 5 7 (3) 3 5 7 (4) 8
4 1
;fn 4a 3 = 0 rks fuEufyf[kr dk eku gS % a 3 3
3 (SSC CGL – 2017)
a a
1 1
=? 24. If 4b 2 2 , then the value of 8b 3 is
b2 b3
3 7 21 21
(1) (2) (3) (4) 1 1
16 16 64 16 ;fn 4b2 2 gks] rks 8b3 dk eku gksxkµ
2
(SSC CGL Tier-I Exam, 16.08.2015 b b3
(IInd Sitting) TF No. 2176783)
(1) 0 (2) 1 (3) 2 (4) 5
1 2 (SSC CPO S.I. Exam. 09.11.2008)
20. If x + = 1, then the value of 2 =?
x x –x 2 x3 1
1 x
1 2 25. If x
x
3 , then the value of is %
;fn x + =1 rks fuEufyf[kr dk eku gS % x2 – x 1
x 2
x –x 2
x3 1
2 1
(1) 2 (2) 4 (3) (4) 1 ;fn x 3 gks] rks x dk eku gksxk %
3 x x2 – x 1
(SSC CGL Tier-I Exam, 16.08.2015
(IInd Sitting) TF No. 2176783) 3 5 7 11
(1) (2) (3) (4)
2 2 2 2
1
21. If a – = 5, then the value of (a – 3)3 – (SSC CHSL DEO & LDC Exam. 27.11.2010)
a –3
1
1 26. If 3x 7 then what is the value of
is 5x
(a – 3)3
5x
?
1 1 15x 2 15x 1
;fn a– = 5, rks (a – 3)3 –
(a – 3)3
dk eku D;k
a –3
1 5x
gksxk\ ;fn 3x 7 gS] rks dk eku D;k
5x 15x 2 15x 1
(1) 5 (2) 7 (3) 2 (4) 14
(SSC CGL Tier-II Exam, 25.10.2015, TF No. 1099685) gksxk\
4 1 1 1 2
22. If x 34 , then what is the value of (1) (2) (3) (4) 10
x 4 5 10 5
(SSC CGL – 2017)
3 1
x ? 1 3 3 1
x3 27. If x + , find the value of 8 x .
4x 2 8x 3

4 1 3 1 1 3 1
;fn x 34 , gS] rks x dk eku D;k gksxk\ ;fn x + gks] rks 8x 3 dk eku fdruk gksxk \
x4 x3 4x 2 8x 3
(1) 0 (2) 6 (3) 8 (4) 14 (1) 18 (2) 36 (3) 24 (4) 16
(SSC CGL – 2017) (SSC CHSL DEO & LDC Exam. 04.12.2011
(Ist Sitting (East Zone)

BLAM–23
chtxf.kr
(1) 123 (2) 126 (3) 113 (4) 129
x 1 FG x 1 IJ
28. If
2x 2 5x 2 6
, the value of H x K is — (SSC CGL Tier-I Exam, 2012 & SSC (10+2) DEO

1 1
34. If x + = 2, find the value of 8x3 + 3 .
;fn
x 1
, x
FG 1 IJ dk eku gS %
2x x
2x 2 5x 2 6 H x K
1 1
1 1 ;fn x + 2x = 2, rks 8x3 + dk eku gSµ
x3
(1) 2 (2) (3) (4) –2
2 2
(1) 48 (2) 88 (3) 40 (4) 44
(SSC (10+2) DEO & LDC Exam, 11.12.2011 Delhi : IInd Sitting) (SSC CHSL DEO & LDC Exam. 04.12.2011
(Ist Sitting (North Zone)
1 1
29. If x 4 =119 and x > l, then the value of x3 – 3 is 1
x4 x 35. If x + = 5, then what will be the value of
x
1 1
;fn x 4 = 119 gks vkSj x > l, gks] rks x3 – dk eku x4 
1
x4 x3
x2 ?
D;k gksxk\ x 2  3x  1
(1) 54 (2) 18 (3) 72 (4) 36
(SSC CHSL DEO & LDC Exam. 11.12.2011 1
(Ist Sitting (East Zone) 1 x4 
;fn x+ = 5, gks] rks x2 dk eku fdruk gksxk\
30. If x2 + x = 19 then what is the value of x x  3x  1
2

1
b x 5g 2
?
(1) 70 (2) 50 (3) 110 (4) 55
bx 5g 2
(SSC CGL Tier-I Exam, 2012)

1
1 36. If x 5 , then the value of
;fn x2 + x = 19 gS] rks b x 5g 2
dk eku D;k
x
bx 5g 2

x4 3x 3 5x 2 3x 1
gksxk\ 4
x 1
(1) 77 (2) 79 (3) 81 (4) 83
(SSC CGL – 2017)
1 x4 3x 3 5x 2 3x 1
1
;fn x x
5 gks] rks 4 dk
1 100 x 1
31. If x 2 , then x will equal to—
x x 100
eku fdruk gksxk \
1 100 1
;fn x 2 gks] rks x dk eku gksxk % 43 47 41 45
x x 100 (1)
23
(2)
21
(3)
23
(4)
21
(1) 2 (2) 0 (3) 1 (4) –2 (SSC CHSL DEO & LDC Exam. 28.10.2012 (Ist Sitting)
(SSC GL Tier-II Exam, 01.08.2010)
1
x4 
1 FG a 37 1 I 37. If x2 + 1 = 2x, then the value of x 2 is
32. If a +
a
+ 2 = 0, then
H 100 JK will equal x 2 – 3x  1
a
to—
1
x4 
;fn a+
1
+ 2 = 0, rks
FG a 37 1 IJ dk eku gSµ ;fn x2 + 1 = 2x gks] rks x2 dk eku D;k gksxk \
a H a 100 K x 2 – 3x  1

(1) 0 (2) –2 (3) 1 (4) 2 (1) 0 (2) 1 (3) 2 (4) –2


(SSC GL Tier-II Exam, 04.09.2011) (SSC Delhi Police S.I.(SI) Exam. 19.08.2012)

1 1 1 1
33. If x + = 3, then what will be the value of x5 + 5 ? 38. If x is real, x + 0 and x3 + = 0, then the
x x x x3

1
;fn x + x = 3 gks] rks x5 +
1
dk eku D;k gksxk \ value of
FG x 1 IJ 4
is
x 5
H x K
BLAM–24
chtxf.kr
1 1 43. If x = 3 + 2 2 , then what is the value of x –
;fn x okLrfod ,oa x + x 0 gks vkSj x3 + = 0 gks] rks
x3
1
?
x
FG x 1 IJ 4
dk eku fdruk gksxk \
H x K 1
;fn x = 3 + 2 2 gks] rks x – x dk eku gSµ
(1) 4 (2) 9 (3) 16 (4) 25
(SSC GL Tier-I Exam. 11.11.2012 (Ist Sitting)
1
1 3 1 (1) ± 2 2 (2) ± 2 (3) ± 2 (4) ±
39. If x > 1 and x2 + = 83, then x – 3 is 2
x2 x
(SSC CPO SI & Assistant Intelligence Officer Exam. 2012)
1 1
3 2
;fn x > 1 rFkk x2 + 2 = 83
x
gks] rks x – 3
x
dk eku 44. If x 3 – 5 , then the value of x – 16x + 6 is

fdruk gksxk\ ;fn x 3– 5 gks] rks x2 – 16x + 6 dk eku D;k gksxk\


(1) 764 (2) 750 (3) 756 (4) 760 (1) 0 (2) –2 (3) 2 (4) 4
(SSC FCI Assistant Grade-III Main Exam. 07.04.2013)
(SSC GL Tier-II Exam. 29.09.2013
1 5
40. If x
4x 2
then what is the value of 1 F 1 I
45. If x
x
2 , then GH x
x
JK will equal to—

e64x 6
1 j ?
8x 3 1 F 1 I
;fn x
x
2 gks] rks GH x
x
JK dk eku gksxkµ

;fn x
1 5
rks
e64x 6
j dk eku D;k gksxk\
1
(1) 2 (2) 2 (3) 2 1 (4) 1
4x 2 8x 3
(SSC SAS Exam, 27.06.2010)
(1) 110 (2) 115 (3) 125 (4) 140
(SSC CGL – 2017) 1
46. If 2x = a + a > 0, then the value of
a'
4 ab
41. If x = , then what is the value of
a b
x2 –1
is
x 2 a x 2 b x– x2 –1
(when a b) ?
x 2 a x 2 b
1 x2 –1
4 ab x 2 a x 2 b ;fn 2x = a + a > 0, rks dk eku D;k
;fn x = gks rks dk eku a' x x2 –1
a b x 2 a x 2 b
D;k gS] (tgk¡ a b) gks ? gks xk \
(1) 0 (2) 2 1
(1) a + 1 (2) (a + 1)
2
(3) 4 (4) e a b je a b j
(SSC CGL – 2017) 1
(3) (a – 1) (4) a – 1
2
3 3 1
42. If x 2 3 , then the value of x is (SSC CGL Tier-II Exam, 2014 12.04.2015
x3 (Kolkata Region) TF No. 789 TH 7)
47. If a3 + b3 = 20 and a + b = 5, then find the value
3 3 1
;fn x 2 3 gks] rks x dk eku fdruk gksxk\ of (a4 + b4).
x3
(1) 25 (2) 26 (3) 24 (4) 23
(1) 8 (2) 9 (3) 2 (4) 4 SSC CHSL (10+2) TIER-I CBE EXAM
(SSC CHSL DEO & LDC Exam. 10.11.2013, IInd Sitting) Held on : 17.03.2020 (Shift-I)

BLAM–25
chtxf.kr
3x y 2y x 3x y 2y x 62 52
(3) (4)
48. If the value of is 27 27
3x y 2y x 3x y 2y x
(SSC CGL Tier-II Exam. 18.11.2020)

same as that of x y , then which of the follow- 27


x4
ing relations between x and y is correct ? 3 x2
55. If x 6, x 0, then the value of is:
(1) 9x – 4y = 36 (2) 9x + 4y = 24 x x2 3x 3
(3) 9x + 4y = 36 (4) 9x – 4y = 24
SSC CHSL (10+2) TIER-I CBE EXAM 27
x4
Held on : 18.03.2020 (Shift-I) 3 x2
;fn x 6, x 0 gS] rks 2 dk eku Kkr
49. If x x 3x 3
4
a b 2 c 3 d 6 , where a, b, c, d are dhft,A
1 2 3 (1) 80 (2) 270
natural numbers, then the value of (3) 54 (4) 90
a + b + c + d is : (SSC CGL Tier-II Exam. 18.11.2020)
(1) 0 (2) 2 (3) 4 (4) 1
SSC CHSL (10+2) TIER-I CBE EXAM Type-III
Held on : 18.03.2020 (Shift-I) Factor Theorem and Questions
50. If a + b + c + d = 2, then the maximum value of Based on Other Formulae
(1+ a) (1 + b) (1 + c) (1 + d) is ______
91 81 63 54 1 1
(1) (2) (3) (4) 1. If x = a + , y= a – , then the value
9 16 22 13 a a
SSC CHSL (10+2) TIER-I CBE EXAM of x4 + y4 – 2x2y2 is—
Held on : 18.03.2020 (Shift-2)
51. If a 2 + b2 + c 2 + 216 = 12(a + b – 2c), then 1 1
;fn x = a + a
, y= a – a
,
ab bc ca is:
(1) 6 (2) 4 (3) 3 (4) 8 rks x4 + y4 – 2x2y2 dk eku gS %
SSC DELHI POLICE SI, CAPFs SI & CISF ASI (CPO) EXAM (1) 16 (2) 20 (3) 10 (4) 5
Held on : 23.11.2020 (Shift-I) (SSC CPO SI & Assistant Intelligence Officer Exam. 2012)

1 2. If (x – 2) is a factor of x2 + 3Qx – 2Q, then the


52. If x4 + x–4 = 194, x > 0, then the value of x is: value of Q is
x
(1) 4 (2) 14 (3) 6 (4) 8 ;fn x2 + 3Qx – 2Q dk xq.ku[kaM (x – 2) gS] rks Q dk eku
SSC DELHI POLICE SI, CAPFs SI & CISF ASI (CPO) EXAM gSµ
Held on : 23.11.2020 (Shift-I) (1) 2 (2) –2 (3) 1 (4) –1
53. If x 2 – 3x + 1 = 0, then the value of (SSC CHSL DEO Exam. 02.11.2014 (Ist Sitting)

FG x 4 1 IJ ( x 2 1) 1
H x2 K 3. If x –
x
1, then what is the value of

(1) 5 (2) 6 (3) 7 (4) 9


SSC DELHI POLICE SI, CAPFs SI & CISF ASI (CPO) EXAM
1 1 LM 1 1 1 OP ?
Held on : 24.11.2020 (Shift-I) x x 1 N x 1 x2 1 x2 1 Q
54. If x 3
FG 2 IJ 3
, then the value of x
3 1
is 1
H x K x x3 ;fn x – x 1, rks
equal to :

FG 2 IJ 3 1
1 1 LM 1 1 1 OP dk eku D;k gS\
;fn x 3
H x K x
, gS] rks x 3
x3
dk eku Kkr dhft,A x x 1 N x 1 x2 1 x2 1 Q
2
8 61 (1) 5x (2)
(1) (2) 5x
27 27

BLAM–26
chtxf.kr
2 5 x
(3) (4) x 8. If x = 5– 21 , then the value of is
5x 2 32 2x 21

(SSC CGL – 2017)


x
4. If (Px3 – 8x2 – qx + 1), the expression is completely ;fn x = 5 – 21 gks] rks dk eku D;k
divisible by the expression (3x2 – 4x + 1) then what 32 2x 21
will be the value of p and q respeteively? gksxk\
;fn O;atd – (Px3 – qx + 1), O;atd
8x2 – 4x + 1) ls (3x2 1 1
iw.kZr% foHkkftr gksrk gS] rks Øe'k% p rFkk q dk eku D;k gksxk\ (1)
2
e 3 7 j (2)
2
e 7 3 j
FG 21 , 15 IJ 1
e 7 3 j 1
e7 3 j
(1) H 4 8K (2) (6, 1) (3)
2
(4)
2
(SSC CHSL DEO & LDC Exam. 10.11.2013, Ist Sitting)
F 33 , 5 IJ
(3) GH 9. If a = 7 – 4 3 , then what is the value of
4 4K
(4) (1, 6)
1 1

(SSC CPO – 2017) a2 a 2 ?

5 1 9 2 1
5. If p , then 27 p 3 p p is equ- 1 1
18 216 2 4 ;fn a = 7 – 4 3 gks] rks a2 a

2 dk eku fdruk gksxk\
al to
(1) 3 3 (2) 4 (3) 7 (4) 2 3
5 3 1 9 2 1 (SSC FCI Assistant Grade-III (Main) Exam, 07.04.2013)
;fn p 18
rks 27 p 216 2
p
4
p dk eku gS\
5 1
10. If x = , then x2 + x – 1 equal to—
4 5 8 10 5 –1
(1) (2) (3) (4)
27 27 27 27
(SSC CAPFs SI, CISF ASI & Delhi Police SI 5 1
Exam. 22.06.2014 TF No. 999 KP0) ;fn x = 5 –1
, rks x2 + x – 1 dk eku gSµ

6. If a 2 e 3 j b e2 3 j 1 , then (a2 – b2 ) equal


(1) 5 +1 (2) 2 (3) – 5 (4) 0
to— (SSC CGL Tier-II Exam. 2012)

11. If x = 3
5 + 2, then the value of x3 – 6x2 + 12x – 13
;fn a 2 e 3 j b e2 3 j 1 gks] rks (a2 – b2) dk eku
is
gks xkµ ;fn x = 3
5 +2 gks] rks x3 – 6x2 + 12x – 13 dk eku D;k
(1) 8 3 (2) 3 8 (3) 6 3 (4) 2 6 gksxk\
(1) –1 (2) 1 (3) 2 (4) 0
(SSC SAS Exam, 27.06.2010)
(SSC Graduate Level Tier-II Exam. 29.09.2013)
7. If x = 3 + 2 2 and xy = 1, then the value of 12. If a2 + l = a, then what will be the value of a12 +
a6 + l ?
x2 3xy y2
is ;fn a2 + l = a, gks] rks a12 + a6 + l dk eku D;k gksxk\
x 2 – 3xy y2
(1) –3 (2) 1 (3) 2 (4) 3
(SSC GL Tier-I Exam. 21.04.2013)
x2 3xy y2
;fn x = 3 + 2 2 rFkk xy = 1 gks] rks dk 13. If x = 2 – 2 1/ 3 + 2 2/ 3 , then the value of
2
x – 3xy y2 x3– 6x2 + 18x + 18 is
eku D;k gksxk \ ;fn x = 2 – 21/3 + 22/3, rks x3 – 6x2 + 18x + 18 dk eku
gSµ
30 70 35 37
(1) (2) (3) (4) (1) 22 (2) 33 (3) 40 (4) 45
31 31 31 31
(SSC CHSL DEO & LDC Exam. 04.12.2011
(SSC CHSL DEO & LDC Exam. 21.10.2012 (IInd Sitting) (Ist Sitting (North Zone)

BLAM–27
chtxf.kr
14. If x = 1 + 2 + 3 , then the value of (2x4 – 8x3 –
;fn O;atd x2 + x + 1 dks
FG x 1 IJ 2
q 2, ds :i esa fy[kk
5x2+ 26x –28) ? H 2 K
;fn x = 1 + 2 + 3 gks] rks (2x4 – 8x3 – 5x2+ 26x –28) tk,] rks q ds laHkkfor eku gSaµ
dk eku D;k gksxk \ 2
1 3 1
(1) 6 6 (2) 0 (3) 3 6 (4) 2 6 (1) (2) (3) (4)
3 2 3 2
(SSC CGL Tier-I Exam, 2012) (SSC Graduate Level Tier-I Exam. 21.04.2013 IInd Sitting)
15. If x = – 1, then the value of 20. If ax + by = 3, bx – ay = 4 and x2 + y2 = 1, then the
1 1 1 1 1 1 1 value of a2 + b2 is —
1 is
x 99 x 98 x 97 x 96 x 95 x 94 x ;fn ax + by = 3, bx – ay = 4 rFkk x2 + y2 = 1, gks rks
;fn x = – 1 gks] rks a2 + b2 dk eku D;k gS \
(1) 17 (2) 16 (3) 9 (4) 25
1 1 1 1 1 1 1
x 99
x 98
x 97
x 96
x 95
x 94 x
1 fdlds cjkcj (SSC (10+2) CHSL DEO & LDC Exam, 04.11.2012
Second Sitting)

gksxk\ x 1 1
21. If , then the value of x – x2 is :
(1) 1 (2) 0 (3) –2 (4) –1 a a x
(SSC MTS Exam. 17.03.2013, Kolkata Region)
16. What will be the value of x4 – 17x3 + 17x2 – 17 x + x 1 1
17 if x = 16 ? ;fn gks] rks x – x2 dk eku fdruk gksxk\
a a x
x = 16 dhfLFkfr esa O;atd x4 – 17x3 + 17x2 – 17 x + 17
dk eku D;k gksxk\ 1 1
(1) – a (2) (3) – (4) a
a a
(1) 0 (2) 1 (3) 2 (4) 3
FCI Assistant Grade-III Exam.05.02.2012, (SSC GL Tier-I Exam. 21.04.2013, Ist Sitting)
Second Sitting & CHSL DEO and LDC Exam, 2011) 22. If x2 + ax + b is a perfect square, then which one
of the following relations between a and b is true ?
x x2 1 x x2 1
17. If 62 then what is the ;fn x2 + ax + b iw.kZ oxZ gS] rks a vkSj b ds chp fuEufyf[kr
x x2 1 x x2 1
esa ls dkSu&lk lEcU/ lgh gS\
value of x (x < 0) ? (1) a2 = b (2) a2 = 4b (3) b2 = 4a (4) b2 = a
(SSC CHSL DEO Exam. 16.11.2014 (Ist Sitting)
x x2 1 x x2 1
;fn 62 gS] rks x (x < 0) d
x x2 1 x x2 1 23. If a + b = 1, c + d = 1 and a – b = , then the value
c
dk eku D;k gksxk\ of c2 – d2 is
(1) – 4 (2) 0 (3) 3 (4) 16 d
(SSC CPO – 2017) ;fn a + b = 1, c + d = 1 vkSj a – b = , rks c2 – d2 dk
c
18. If x 2 – 3x + 1 = 0, then the value of
eku gSµ
x6 x4 x2 1
will be a b
x3 (1) (2) (3) 1 (4) –1
b a
(SSC CGL Tier-I Re-Exam. (2013) 20.07.2014 (Ist Sitting)
x6 x4 x2 1
;fn x2 – 3x +1 = 0, rks 3 dk eku D;k 24. Find the value of x for which the expression 2 –
x
3x – 4x2 has the greatest value.
gksxk\ x dk eku Kkr dhft, ftlds fy, O;atd 2 – 3x – 4x2 dk
(1) 18 (2) 15 (3) 21 (4) 30
egÙke eku gSµ
(SSC CGL Tier-I Exam. 19.10.2014 (Ist Sitting)
19. If the expression x2 + x + 1 is written in the form 41 3 3 41
(1) (2) (3) (4)
16 8 8 16
FG x 1 IJ 2
q 2, then the possible values of q are (SSC CGL Tier-I Re-Exam. (2013) 20.07.2014 (IInd Sitting)
H 2K
BLAM–28
chtxf.kr
25. What is the Simplified value of (1) – 1 (2) 3 (3) – 3 (4) 1
(SSC Graduate Level Tier-I Exam. 21.04.2013)
FG x 32 1 IJ FG x 8 1 IJ FG x 1 IJ FG x 16 1 IJ 30. If a2 – 4a – 1 = 0, a 0, then the value of a2+ 3a
H x 32 KH x8 KH x KH x 16 K
1 3
+ – is
FG x 1 IJ FG x 4 1 IJ dk ljyhÑr eku D;k gS\ a 2 a
H x KH x4 K
1 3
;fn a2 – 4a – 1 = 0, a 0, rks a2 + 3a + – dk
FG x 64 1 IJ a2 a
64 FG 1 IJ H x 64 K eku D;k gS\
(1) x H K (2) F 1 IJ
x 64 GH x 2
x2 K (1) 24 (2) 26 (3) 28 (4) 30
(SSC CGL Tier-I Exam. 19.10.2014 TF No. 022 MH 3)

FG x 64 1 IJ FG x 32 1 IJ 31. If x =
a –b
,y=
b–c
,z=
c –a
, then
H x 64 K H x 32 K a b b c c a
(3) FG x 1 IJ (4) F 1 IJ
H x K GH x 2
x2 K (1 – x )(1 – y )(1 – z )
is equal to
(1 x )(1 y )(1 z )
(SSC CPO 2017)
26. If a2 + b2 + c2 = ab + bc + ca, then the value of a –b b–c c –a
;fn x = a b , y = b c , z = c a , rks
a c
is
b
(1 – x )(1 – y )(1 – z )
a c (1 x )(1 y )(1 z ) dk eku D;k gksxk\
;fn a2 + b2 + c2 = ab + bc + ca, rks
b
dk eku D;k
gksxk \ 1
(1) 1 (2) 0 (3) 2 (4)
(1) 3 (2) 2 (3) 0 (4) 1 2
(SSC CAPFs SI, CISF ASI & Delhi Police SI (SSC CGL Tier-I Re-Exam, 30.08.2015)
Exam, 21.06.2015 IInd Sitting)
27. If 3(a2 + b2 + c2) = (a + b + c)2, then the relation a b
32. If + = 2, then the value of (a – b) is :
between a, b and c is b a
;fn 3(a2 + b2 + c2) = (a + b + c)2 gS] rks a, b vkSj c
a b
ds chp D;k laca/ gS \ ;fn + =2 gS] rks a – b dk eku gS %
b a
(1) a b c (2) a = b c
(3) a b=c (4) a = b = c (1) 1 (2) 2 (3) –1 (4) 0
(SSC CGL Tier-II Exam, 25.10.2015, TF No. 1099685) (SSC CHSL (10+2) LDC, DEO & PA/SA Exam, 15.11.2015
(IInd Sitting) TF No. 7203752)
28. What is the value of dk eku D;k gS\
1 1 1
33. If a = 1 and b = 1 then c is equal
1 1 1 1 1 1 b c a
1 1 + 1 ?
22 32 32 42 42 52 to :

1 1 1
;fn a =1 vkSj b =1 rks c fdlds cjkcj
18 4 7 33 b c a
(1) (2) (3) (4)
5 3 3 10 gksxk \
(SSC COP – 2017)
1
1 1 (1) (2) 2 (3) 1 (4) 0
29. If a, b, c are non-zero, a + =1 and b + =1, then 2
b c
(SSC CHSL (10+2) LDC, DEO & PA/SA Exam, 06.12.2015
the value of abc is :
(IInd Sitting) TF No. 3441135)
1 1 34. If a, b, c are real and a3 + b3 + c3 = 3abc and a +
;fn a, b, c 'kwU; ugha gSa] a + b =1 rFkk b + c =1 gS] rks abc
b + c z 0, rks a, b, c then relation among a, b, c
dk eku D;k gksxk\ are :

BLAM–29
chtxf.kr
;fn a, b, c okLrfod gSa vkSj + + = 3abc vkSj a
a3 b3 c3 40. If (a – 3)2 + (b – 4)2 + (c – 9)2 = 0, then the value of
+ b + c z 0, rks a, b, c ds chp laca/ gksxk % a  b  c is :
(1) a + b = c (2) a + c = b
;fn (a – 3)2 + (b – 4)2 + (c – 9)2 = 0 gks] rks a b c
(3) a = b = c (4) b + c = a
(SSC CPO SI & Assistant Intelligence Officer Exam, 2012) dk eku D;k gksxk \
1 (1) – 4 (2) 4 (3) ± 4 (4) ± 2
35. If xy (x + y) = 1, then the value of 3 3
– x3 – y3 is
x y (SSC CHSL DEO & LDC Exam. 11.12.2011)

: x y z
41. If xa yb zc = ya zb xc = za xb yc and
1 3 3
;fn xy (x + y) = 1 gks, rks x y dk eku gS % x+y+z 0, then each ratio is
x 3y 3

(1) 0 (2) 1 (3) 3 (4) –2 x y z


(SSC CHSL DEO & LDC Exam. 11.12.2011 (IInd Sitting
;fn xa yb zc = ya zb xc = za xb yc vkSj
(Delhi Zone) & (SSC GL Tier-I Exam. 21.04.2013)
36. What is the value of x + y + z 0, rks izR;sd vuqikr gksxkµ
dk eku D;k gS\ 1 1
(1) (2)
a –b –c a b –c
ea 2
b2 a jb b g ba b g 3

1 1
a 2b ab 2 (3) (4)
a –b c a b c
(1) 0 (2) 1 (3) –1 (4) 2 (SSC CHSL DEO & LDC Exam. 9.11.2014)
(SSC CPO – 2017)
42. If a + b + c + d = 4, then find the value of

37. If N =
e 7 5 j , then what is the value of 1
?
1

1
N (1  a )(1  b )(1  c ) (1  b )(1  c )(1  d )
7 5
1 1
 
e 7 5 j gS rks 1 (1  c )(1  d )(1  a ) (1  d )(1  a )(1  b ) .
;fn N = N
dk eku D;k gS\
7 5
1
;fn a + b + c + d = 4, rks 
(1  a )(1  b )(1  c )
(1) 6 – 35 (2) 6 + 35 (3) 7 + 35 (4) 7 – 35
(SSC CPO – 2017) 1 1
 
38. If (3a + 1)2 + (b – 1)2 + (2c – 3)2 = 0, then the value (1  b )(1  c )(1  d ) (1  c )(1  d )(1  a )
of (3a + b + 2c) is—
1
;fn (3a + 1)2 + (b – 1)2 + (2c – 3)2 = 0, rks (3a + b +  dk eku D;k gksxk\
(1  d )(1  a )(1  b )
2c) dk eku gS%
(1) 0 (2) 5 (3) 1 (4) 4
(1) 3 (2) –1 (3) 2 (4) 5
(SSC CHSL DEO Exam. 16.11.2014 (Ist Sitting)
(SSC CHSL DEO and LDC Exam, 11.12.2011 )
43. If a + b + c = 3 and none of a, b and c is equal to
39. If (x – 3)2 + (y – 5)2 + (z – 4)2 = 0, then the value of
1, then what is the value of
x2 y2 z2 1 1 1
is
9 25 16 b1 – a gb1 b g b1 b gb1 c g b1 c gb1 a g ?
x2 y2 ;fn a + b + c = 3 rFkk c esa ls dksbZ Hkh 1 osQ cjkcj ugha gS]
;fn (x – 3)2 + (y – 5)2 + (z – 4)2 = 0 gks rks] 9 + 25 +
1 1 1
2
rks b1 – a gb1 b g b1 b gb1 c g b1 c gb1 a g ? dk
z
16 dk eku fdruk gksxk \ eku D;k gksxk\
(1) 12 (2) 9 (3) 3 (4) 1 (1) 0 (2) 1 (3) 3 (4) 6
(SSC Graduate Level Tier-I Exam. 19.05.2013) (SSC CPO – 2017)

BLAM–30
chtxf.kr

¦ (p  q)(q  r ) will be
q r 1 1
44. The value of expression 48. The value of
a2 ax x2 a2 ax x2

¦ (p qq)(qr  r )
2ax
O;atd dk eku gksxk % a4 a 2x 2 x4
is

(1) 0 (2) 1 1 1 2ax


(3) 3 (4) –1 a2 ax x2 a2 ax x2 a4 a 2x 2 x4
(SSC CGL Tier-I Exam. 19.10.2014)
dk eku D;k gksxk\
b c a c a b
45. If 1 and a – b + c 0 then (1) 2 (2) 1 (3) –1 (4) 0
a b c
(SSC CGL Tier-II Exam. 21.09.2014)
which one of the following relations is true ?
a b –c b c –a c a –b
b c a c a b 49. If = = and a + b + c
;fn a 1 vkSj a – b + c 0 rks a b b c c a
b c
0, then
fuEufyf[kr esa ls dkSu&lk laca/ lgh gksxk \
a b –c b c –a c a –b
1 1 1 1 1 1 ;fn = = vkSj a + b + c
(1) (2) a b b c c a
c a b a b c
0, rks
1 1 1 (1) a b c (2) a = b = c
(3)
b a c (3) a = b c (4) a b = c
(SSC CGL Tier-II Exam, 2014 12.04.2015
1 1 1 (Kolkata Region) TF No. 789 TH 7)
(4)
b a c
b c
(SSC CGL Tier-I Re-Exam. (2013) 27.04.2014) 50. If bc + ab + ca = abc, then the value of +
bc (a – 1)
46. If a, b, c are positive and a + b + c = 1, then the
a c a b
1 1 1 + is
least value of + + is ac (b – 1) ab(c – 1)
a b c
b c a c
1 ;fn bc + ab + ca = abc, rks bc (a – 1) + ac (b – 1) +
;fn a, b, c /ukRed gSa vkSj a + b + c = 1, rks a +

1 1 a b
+ dk U;wure eku D;k gS \ ab (c – 1)
dk eku D;k gksxk \
b c
(1) 9 (2) 5 (3) 3 (4) 1 1 3
(1) 0 (2) – (3) – (4) 1
(SSC CGL Tier-I Exam. 26.10.2014) 2 2
(SSC CGL Tier-II Exam, 2014 12.04.2015
p q r a b c
47. If   =1 and   = 0, where p, q, r (Kolkata Region) TF No. 789 TH 7)
a b c p q r
and a, b, c are non-zero, then the value of a 2 – bc b 2 – ca c 2 – ab
51. If 2
+ 2
+ = 1, then the val-
2 2 2 a bc b ca c2 ab
p q r
  is ue of
a 2 b2 c 2
a2 b2 c2
p q r a b c + + is
;fn = 1 vkSj p a2 b2 c2
r = 0, ;gk¡ p, q, r vkSj
bc ac ab
a b c q

a 2 – bc b 2 – ca c 2 – ab
p2 q2 r2 ;fn + + = 1 gks] rks
a, b, c 'kwU;srj gSa] rks dk eku D;k gS \ a2 bc b2 ca c2 ab
a2 b2 c2
(1) –1 (2) 0 (3) 1 (4) 2 a2 b2 c2
(SSC CGL Tier-I Exam. 19.10.2014) + + dk eku D;k gksxk \
a2 bc b2 ac c2 ab

BLAM–31
chtxf.kr
(1) 0 (2) 1 (3) – 1 (4) 2 57. If a + b + c = 0, then the value of
(SSC CGL Tier-II Exam, 2014 12.04.2015
(Kolkata Region) TF No. 789 TH 7) Fa  b  c I
2 2 2

3 – 5x 3 – 5y 3 – 5z 2
GH bc ca ab JK is—
52. If + + = 0, the value of +
2x 2y 2z x
Fa 2
b2 c2 I
;fn a + b + c = 0 gks] rks bc  ca  ab dk eku gS %
2 2
y
+
z
is GH JK
3 – 5x 3 – 5y 3 – 5z 2 2 (1) 2 (2) 3 (3) 4 (4) 5
;fn + + =0 rks 2 + + (SSC CPO SI & Assistant Intelligence Officer Exam, 2012
2x 2y 2z x y z and SSC CGL Tier-I Exam, 19.05.2013)

dk eku D;k gS \ 58. What is the simplified value of (x128 + 1) (x32 + 1)


(x64 + 1) (x16 + 1) (x8 + 1) (x4 + 1) (x2 + 1) (x + 1)?
(1) 20 (2) 5 (3) 10 (4) 15
dk ljyhÑr eku D;k gS\
(SSC CGL Tier-II Exam. 12.04.2015 TF No. 567 TL 9)
53. If a + b – c = 0 then the value of 2b2c2 + 2c2a2 +
2a2b2 –a4 – b4 – c4
x 128 1 x 64 1 x 156 1
;fn a + b – c = 0 rks fuEufyf[kr dk eku gS& 2b2c2 + 2c2a2 (1) x256 – 1 (2) (3) (4)
x 1 x 1 x 1
+ 2a2b2 –a4 – b4 – c4
(SSC CPO – 2017)
(1) 7 (2) 0 (3) 14 (4) 28
(SSC CGL Tier-I Exam, 16.08.2015 1 1 1
59. If a2 + b2 + c2 + 6 then what is
(IInd Sitting) TF No. 2176783) a2 b2 c2
the value of a2 + b2 + c2 ?
54. What is the value of
ea 2
b2 a jb b g ea 3
b3 j?
a 2b ab 2 1 1 1
;fn a2 + b2 + c2 + 6 gS] rks a2 + b2 +
a2 b2 c2
ea 2
b2 a jb b g ea 3
b3 j dk eku D;k gksxk\
c2 dk eku D;k gS\
a b 2
ab 2 (1) 3 (2) 6 (3) –3 (4) 2
(SSC CPO – 2017)
(1) 0 (2) 1 (3) –1 (4) 3
60. If a2 + b2 + c2 = 2a – 2b – 2,then the value of 3a –
(SSC CPO – 2017)
2b + c is
55. If a, b, c are non-zero real numbers a + b + c = 0
;fn a2 + b2 + c2 = 2a – 2b – 2,rks 3a – 2b + c dk eku
a 2  b2  c 2
2
and b z ca then the value of is— D;k gS\
b 2  ca
(1) 0 (2) 3 (3) 5 (4) 2
;fn a, b, c rhu 'kwU;srj okLrfod la[;k,¡ gSa rks a + b + c = (SSC CGL Tier-I Exam. 19.10.2014 TF No. 022 MH 3)

a 2  b2  c 2 1 1 1
0, rFkk b
2
z ca, rks dk eku gSµ 61. If a + b + c = 3, a2 + b2 + c2 = 6 and + + =
a b c
b 2  ca
1, where a, b, c are all non-zero, then ‘abc’ is
(1) 3 (2) 2 (3) 0 (4) 1
equal to
(SSC CGL Tier-II Exam, 04.09.2011)
56. If a + b + c = 2s, then 1 1 1
;fn a + b + c = 3, a2 + b2 + c2 = 6 vkSj a + b + c
bs a g bs b g bs c g
2 2 2
s 2
is equal to
a 2
b 2
c 2 = 1, ;gk¡ a, b, c lHkh 'kwU;srj gS]a rks ‘abc’ fdlds cjkcj gS\

2 3 1 1
;fn a + b + c = 2s rks
bs a g bs b g bs c g 2 2 2
s2 (1)
3
(2)
2
(3)
2
(4)
3
a2 b2 c2 (SSC CGL Tier-I Exam. 19.10.2014 TF No. 022 MH 3)

D;k gksxk \ 62. If x (x + y + z) = 20, y (x + y + z) = 30, and z (x + y


(1) a2 + b2 + c2 (2) 0 + z) = 50, then the value of 2 (x + y + z) is :
(3) 1 (4) 2 ;fn x (x + y + z) = 20, y (x + y + z) = 30, vkSj z (x + y
(SSC Graduate Level Tier-I Exam. 21.04.2013) + z) = 50, rks 2 (x + y + z) dk eku D;k gksxk \
BLAM–32
chtxf.kr
(1) 20 (2) –10 (3) 15 (4) 18 (1) abc (2) a2b2c2 (3) 1 (4) 0
(SSC CHSL (10+2) LDC, DEO & PA/SA Exam, 06.12.2015
(SSC CHSL DEO and LDC Exam, 2011)
(Ist Sitting) TF No. 1375232)

x a a x b b
63. If b2a 1g b4b 3g b4c 5g 2 2 2
= 0, then the 67. If
x b x x a x
, b a then what will

a3 b3 c3 3abc be the value of x ?


value of 2 is
a b2 c2 x a a x b b
;fn , b a gks] rks x dk eku
x b x x a x
;fn 2a 1 b g b4b 3g b4c 5g
2 2 2
= 0 rks
D;k gksxk\
3 3 3
a b c 3abc b ab a
dk eku D;k gksxk \ (1) (2) (3) 1 (4)
a2 b2 c2 a b a b a b
(SSC CGL Tier-I Exam, 2012)
3 3 3
(1) 1 (2) 2 (3) 3 (4) 0
8 8 8 3 3
68. If x a a2 b3 + a a2 b 3 , then x3 +
(SSC CHSL (10+2) LDC, DEO & PA/SA Exam, 06.12.2015
(IInd Sitting) TF No. 3441135) 3bx is equal to

64. The numerical value of


ba b g2 + ;fn x 3
a a2 b3 + 3
a a2 b3 , rks x3 +
ba c gbc a g 3bx dk eku gSµ
(1) 0 (2) a (3) 2a (4) 1
bb c g 2
+
bc a g 2
is (a b c) (SSC GL Tier-I Exam. 21.04.2013 IInd Sitting)
bc a gba b g ba bgbb c g
1
69. If ab (a + b) = 1 then what is the value of
b g a b g
b
2
b g b c
2
c a
2
dk eku
a 3b 3
bb c gbc a g + bc a gbb c g + ba bgbb c g – a3 – b3 ?
gS (a b c) 1
;fn ab (a + b) = 1 gS] rks – a3 – b3 dk eku D;k
1 a 3b 3
(1) 0 (2) 1 (3) (4) 3
3 gS\
(SSC CAPFs SI, CISF ASI & Delhi Police SI
(1) –1 (2) 1
Exam. 22.06.2014 TF No. 999 KP0)
(3) 3 (4) –3
m – a2 m – b2 m – c2 (SSC CPO – 2017)
65. If 2 2 2 2 2 2
3 , then the val-
b c c a a b 1
FG x 2 1 IJ
ue of m is 70. If x e j
2 1 2 then
H x2 K will equal to—

m – a2 m – b2 m – c2
;fn + + = 3, gS] rks m dk ;fn
1
gks] rks
FG x 2 1 IJ dk eku gksxk %
b2 c2 c2 a2 a2 b2 x e 2 1 2 j H x 2 K
eku D;k gS \
(1) 2 (2) 2 2 (3) 2 2 (4) 2
(1) a2 + b2 – c2 (2) a2 + b2
(SSC GL Tier-II Exam, 01.08.2010)
(3) a2 + b2 + c2 (4) a2 – b2 – c2
(SSC CGL Tier-I Exam, 09.08.2015 (Ist Sitting) TF No. 1443088) u v u v x y
66. If a2 = b + c, b2 = c + a and c2 = a + b, then what 71. If x = ,y= , then 1  xy equals to
1  uv 1  uv
1 1 1
will be the value of + + ?
1 a 1 b 1 c u v u v 1
x3 y3
;fn x = 1  uv , y = 1  uv
,rks 3 3
x y
cjkcj
1
;fn a2 = b + c, b2 = c + a rFkk c2 = a + b gks] rks 1 a gksxk
u 2u 2u 2u
1 1 (1) (2) (3) (4)
+ + dk eku D;k gksxk\ 1 u2 1 u 1 u2 1 u2
1 b 1 c

BLAM–33
chtxf.kr
76. If x + y = 2z, then what will be value of
a b c
72. If = 1, then what will be the x z
1–a 1 b 1 c  ?
x z yz
1 1 1
value of + + ?
1–a 1 b 1 c x z
;fn x + y = 2z gks] rks  dk eku D;k gksxk \
x z yz
a b c 1 1
;fn 1–a 1 b 1 c
= 1 gks] rks 1–a
+
1 b
+
1
(1) 1 (2) 3 (3) (4) 2
1 2
1 c
dk eku D;k gksxk\ (SSC DP Sub Inspector Exam. 19.08.2012, Paper-I)

(1) 1 (2) 2 (3) 3 (4) 4


13 11 1
(SSC CHSL DEO & LDC Exam. 2011 & SSC GL Tier-I 77. Let x = and y = , then the value of
Exam, 19.05.2013 IInd Sitting) 13 – 11 x
73. If x3 + 6x2 + 12x = 19, then what is the value of
3x2 – 5xy + 3y2 is
x3?
;fn x3 + 6x2 + 12x = 19, gks rks x3 dk eku D;k gS\ 13 11 1
(1) 8 (2) 27 (3) –1 (4) 1 eku ysa fd x = vkSj y = , rks 3x2 – 5xy
13 – 11 x
(SSC CPO – 2017)
+ 3y2 dk eku gS
5 2
74. If x = , then what will be the nature of (1) 1717 (2) 1177 (3) 1771 (4) 1171
5 2
(SSC CGL Tier-II Exam, 25.10.2015, TF No. 1099685)
x4 + x–4 ? 78. If x 0, y 0 and z 0 and

5 2 1 1 1 1 1 1 , then the relation


;fn x = gks] rks x4 + x–4 dSlk gksxk\ 2 2 2
5 2 x y z xy yz zx

(1) Surd number / dj.kh la[;k among x, y, z is


(2) Rational number, but not whole number /
1 1 1
ifjes; la[;k] fdUrq iw.kk±d ugha ;fn x 0, y 0 rFkk z 0 rFkk x2 y2 z2
(3) Whole number / iw.kk±d la[;k
(4) Irrational number, but not surd number / 1 1 1
vifjes; la[;k] fdUrq dj.kh ugha xy yz zx , gks] rks x, y, z ds chp D;k laca/ gksxk\
(SSC CGL Tier-I Exam, 2012)
(1) x + y + z = 0 (2) x + y = z
5– 3 5 3
75. If x = and y = then the value 1 1 1
5 3 5– 3 (3) x 0 (4) x = y = z
y z

x2 xy y2 (SSC Graduate Level Tier-I Exam. 21.04.2013)


of =?
2
x – xy y 2 79. If a + b + c = 0, then the value of

FG a b b c c a IJ FG a b c IJ is :
;fn x =
5– 3
vkSj y =
5 3
rks fuEufyf[kr dk H c a b K Hb c c a a b K
5 3 5– 3
;fn a + b + c = 0 gks] rks
x2 xy y2
eku gS %
x 2 – xy y2
FG a b b c c a IJ FG a b c IJ dk
H c a b K Hb c c a a b K
63 67 65 69 eku D;k gksxk\
(1) (2) (3) (4)
61 65 63 67 (1) 8 (2) –3 (3) 9 (4) 0
(SSC CGL Tier-I Exam, 16.08.2015 (SSC Graduate Level Tier-I Exam. 21.04.2013)
(IInd Sitting) TF No. 2176783)

BLAM–34
chtxf.kr
1 1 x y z
85. If
Fp q I Fp
–1 2 3 6
q –3 3 I b2x  y  z g = bx  2y  z g = bx  y  2z g = a and
80. If GH p q JK GH p
3 –2 –2 3
q
JK = pa qb, then the value
x+y+z 0, then the value of a will be

x y z
;fn
b2x  y  z g = bx  2y  z g = bx  y  2z g = a rFkk
of a + b, where p and q are different positive
primes, is

1 1
x+y+z 0 rks a dk eku gSµ
;fn
Fp q I F
–1 2 3 p 6q –3 3 I gS] rks a + b dk eku 1 1
GH p q JK GH p
3 –2 –2 3
q
JK = pa qb (1)
3
(2)
4

D;k gS ftlesa p vkSj q fofHk /ukRed vHkkT; gSa \ 1 1


(3) (4)
(1) –1/3 (2) 2/3 (3) 1/3 (4) –2/3 2 8

(SSC CGL Tier-II Exam, 25.10.2015, TF No. 1099685) (SSC GL Tier-I Exam. 19.05.2013 Ist Sitting)

1 1
1 1 1
1 1 86. If , then the value of a3 + b3 is
a2 –a 2 a b a b
81. If x a2 a 2 ,y , then v alue of
(x4 – x2y2 –1) + (y4 – x2y2 + 1) is 1 1 1
;fn a b a b
gks] rks a3 + b3 dk eku D;k gksxk \
1 1 1 1
– –
;fn x a2 a 2 ,y a2 –a 2 gS] rks (x4 – x2y2 –1) + (1) 0 (2) –1 (3) 1 (4) 2
(SSC CHSL DEO & LDC Exam. 10.11.2013, Ist Sitting)
(y4 – x2y2 + 1) dk eku gS
(1) 16 (2) 13 (3) 12 (4) 14 1 1
87. If a + b + c = 0, then + 2 +
(SSC CGL Tier-II Exam, 25.10.2015, TF No. 1099685) a 2  b2  c 2 b c2  a2

82. If x – 3 – 2 = 0 and y – 3 + 2 = 0, then the 1


equal to
a 2  c2  b2
3
e
value of x – 20 2 – y
3
j e 20 2 isj 1 1
;fn a + b + c = 0 rks + 2 +
a 2  b2  c 2 b c2  a2
;fn x – 3 – 2 = 0 vkS j y – 3 + 2 = 0, rks
1
cjkcj gksxkA
a 2  c2  b2
ex 3
– 20 2 j – ey 3
2 2 j dk eku D;k gksxk\
(1) 0 (2) 1 (3) –1 (4) 2
(1) 0 (2) 1 (3) 3 (4) 2
(SSC CHSL DEO & LDC Exam. 11.12.2011
(SSC CGL Tier-II Exam, 25.10.2015, TF No. 1099685)
(Ist Sitting (Delhi Zone)
83. If p2x (p2 + 1) = p(p3x – px) then, the value of x
will be 2 1
88. If x and xy = 1, then what will be the
;fn p2x (p2 + 1) = p(p3x – px) gks] rks x dk eku gSµ 2 1

1 2x 2 3xy 2y 2
(1) +1 (2) + p (3) 0 (4) p value of 2 ?
2x 3xy 2y 2
(SSC CPO (SI, ASI & Intelligence Officer) Exam. 28.08.2011
(Paper-I) & (SSC CHSL DEO & LDC Exam. 04.12.2011) 2 1 2x 2 3xy 2y 2
(Ist Sitting) & (SSC CGL Tier-I Exam.19.05.2013) (Ist Sitting) ;fn x rFkk xy = 1 gks] rks 2 dk
2 1 2x 3xy 2y 2
1 1
84. If x 2 3x 1 0, then the value of x 2 x is eku gksxkµ
x x2
71
;fn x 2 3x 1 0, rks x2 x
1 1
dk eku gSµ (1) (2) 3 2 2
x 2 65
x

(1) 10 (2) 2 (3) 6 (4) 8 81


(3) (4) 3 2 2
(SSC GL Tier-I Exam. 19.05.2013 Ist Sitting) 65
(SSC SAS Exam, 27.06.2010) & GL Tier-I Exam, 2012)

BLAM–35
chtxf.kr
4ab x 2a x 2b (1) 3 (2) 1 (3) x + y + z (4) 0
89. If x = (a b), the value of is (SSC CHSL DEO & LDC Exam. 20.10.2013)
a b x 2a x 2b
a b
4ab x 2a x 2b 94. If (a2+ b2)3 = (a3 + b3)2, then is
;fn x = (a b), gks] rks dk eku b a
a b x 2a x 2b
D;k gksxk \ ;fn (a2+ b2)3 = (a3 + b3)2 gks] rks
a b
fdruk gksxk\
(1) a (2) b (3) 2 ab (4) 2 b a
(SSC CHSL DEO & LDC Exam. 04.12.2011
(Ist Sitting (East Zone)
1 2 1 2
(1) (2) (3) (4)
3 3 3 3
2 24 x 8 (SSC CHSL DEO & LDC Exam. 28.10.2012 (Ist Sitting)
90. If x = , then what is the value of
3 2 x 8 95. The value of the expression
x 12
x 12
? ba bg bb c g
2
bc a g 2 2

bb c gbc a g + ba bgbc a g + ba bgbb c g is :


2 24 x 8 x 12
;fn x = gks] rks dk eku D;k
3 2 x 8 x 12 ba b g bb c g
2
ba c g 2 2

gksxk\ bb c gbc a g ba b gbc a g ba b gbb c g


(1) 1 (2) 2 (3) 1 (4) –2
dk eku] tc a b c gks] fdruk gksxk \
(SSC CGL Tier-I Exam, 2012)
1
2 6 (1) 0 (2) 3 (3) (4) 2
91. If x = , then what is the value of 3
3 2
(SSC CHSL DEO & LDC Exam. 11.12.2011 & 27.10.2013
x 2 x 3 (IInd Sitting (Delhi Zone))
? 1 1 1
x 2 x 3
96. If a 3 b3 c 3 = 0, then a relation among a, b , c
2 6 x 2 x 3 is
;fn x = gks rks] dk eku
3 2 x 2 x 3 1 1 1
fdruk gksxk\ ;fn a 3 b 3 c 3 = 0, rks a, b ,oa c ds chp D;k laca/
(1) 2 (2) 3 (3) 6 (4) 2 gksxk \
(SSC CGL Tier-I Exam, 2012) (1) a + b + c = 0 (2) (a + b + c)3 = 27abc
(3) a + b + c = 3abc (4) a3 + b3 + c3 = 0
4 15 x 20 x 12 (SSC CHSL DEO Exam. 16.11.2014 (Ist Sitting)
92. If x = , the value of is
5 3 x 20 x 12
97. If
FG x 1 IJ 2
= 3, then the value of x206 + x200 +
4 15 x 20 x 12
H x K
;fn x = gks] rks dk eku
5 3 x 20 x 12 x90 + x84 + x18 + x12 + x6 + 1 is

fdruk gksxk \ FG 1 IJ 2
;fn x =3 gks] rks x206 + x200 + x90 + x84 +
(1) 1 (2) 2 (3) 3 (4) 5
H x K
(SSC CHSL DEO & LDC Exam. 27.10.2013 IInd Sitting) x18 + x12 + x6 + 1 dk eku D;k gksxk\
F 1 1 1 I (1) 0 (2) 1
93. If xy + yz + zx = 0, then GH x 2
yz y2 zx z2
J
xy K (3) 84 (4) 206
(x, y, z 0) is equal to (SSC Graduate Level Tier-II Exam. 16.09.2012)

;fn xy + yz + zx = 0 gS] rks


F 1 1 1 I 98. If x = p +
1
and y = p –
1
, then the value of
GH x 2
yz y2 zx z2 xy
JK p p

(x, y, z 0) fdlds cjkcj gS\ x4–2x2y2 + y4 is

BLAM–36
chtxf.kr
1 1 72. (1) 73. (1) 74. (1) 75. (1) 76. (3) 77. (3)
;fn x = p + vkSj y = p – , gks rks x4 – 2x2y2 + y4
p p 78. (4) 79. (2) 80. (1) 81. (2) 82. (2) 83. (2)

dk eku gksxkµ 84. (4) 85. (2) 86. (1) 87. (4) 88. (2) 89. (4)
(1) 24 (2) 4 90. (3) 91. (3) 92. (4) 93. (3) 94. (2) 95. (4)
(3) 16 (4) 8
(SSC CHSL DEO & LDC Exam. 9.11.2014)
Test-II
1 1
99. If x = ,y= , then the value of 8xy 1 1
2 3 2 3 Based on a or a Formulae
a a
(x2 + y2) is

1 1 1. (4) 2. (3) 3. (2) 4. (2) 5. (4) 6. (4)


;fn x = 2 3
,y=
2 3
gks] rks 8xy (x2 + y2) dk
7. (1) 8. (3) 9. (1) 10. (4) 11. (4) 12. (4)
eku D;k gksxk \ 13. (2) 14. (3) 15. (2) 16. (3) 17. (1) 18. (2)
(1) 196 (2) 290 19. (3) 20. (1) 21. (4) 22. (4) 23. (2) 24. (1)
(3) 112 (4) 194 25. (3) 26. (2) 27. (1) 28. (2) 29. (4) 30. (2)
(SSC CAPFs SI, CISF ASI & Delhi Police SI
Exam, 21.06.2015 IInd Sitting) 31. (1) 32. (2) 33. (1) 34. (3) 35. (4) 36. (1)
37. (4) 38. (2) 39. (3) 40. (1) 41. (2) 42. (4)
24 x 72 1
100. If x 1
7 , then the value of is 43. (2) 44. (3) 45. (2) 46. (3) 46. (3) 47. (4)
x 12 x 36
48. (2) 49. (1) 50. (3) 51. (1) 52. (1) 53. (*)
24 72 54. (2) 55. (4)
x 1 x 1
;fn 7, rks fuEufyf[kr dk eku gS%
x 12 x 36
(1) 433 (2) 322 Test-III
(3) 343 (4) 432 Factor Theorem and Questions
(SSC CGL Tier-I Exam, 16.08.2015 Based on Other Formulae
(Ist Sitting) TF No. 3196279)

SHORT ANSWERS 1. (1) 2. (4) 3. (3) 4. (3) 5. (3) 6. (1)


7. (4) 8. (2) 9. (2) 10. (1) 11. (4) 12. (4)

Test-I 13. (3) 14. (1) 15. (3) 16. (2) 17. (1) 18. (3)

Based on Square and Cubic Formulae 19. (2) 20. (4) 21. (4) 22. (2) 23. (2) 24. (3)
25. (2) 26. (2) 27. (4) 28. (4) 29. (1) 30. (4)
1. (1) 2. (1) 3. (3) 4. (3) 5. (3) 6. (4) 31. (1) 32. (4) 33. (3) 34. (3) 35. (3) 36. (4)
7. (2) 8. (1) 9. (4) 10. (1) 11. (4) 12. (1) 37. (2) 38. (1) 39. (3) 40. (2) 41. (4) 42. (1)
13. (1) 14. (2) 15. (2) 16. (1) 17. (3) 18. (3) 43. (1) 44. (1) 45. (3) 46. (1) 47. (3) 48. (4)

19. (3) 20. (3) 21. (4) 22. (3) 23. (1) 24. (1) 49. (2) 50. (4) 51. (4) 52. (3) 53. (2) 54. (3)

25. (3) 26. (3) 27. (3) 28. (1) 29. (2) 30. (2) 55. (2) 56. (3) 57. (2) 58. (4) 59. (1) 60. (3)

31. (4) 32. (1) 33. (4) 34. (3) 35. (1) 36. (1) 61. (2) 62. (1) 63. (4) 64. (4) 65. (3) 66. (3)

37. (2) 38. (2) 39. (3) 40. (1) 41. (4) 42. (3) 67. (2) 68. (3) 69. (3) 70. (1) 71. (3) 72. (4)

43. (3) 44. (4) 45. (4) 46. (1) 47. (3) 48. (4) 73. (4) 74. (3) 75. (1) 76. (1) 77. (1) 78. (4)

49. (3) 50. (2) 51. (3) 52. (3) 53. (3) 54. (3) 79. (3) 80. (4) 81. (1) 82. (1) 83. (1) 84. (1)
85. (2) 86. (1) 87. (1) 88. (1) 89. (4) 90. (2)
55. (1) 56. (2) 57. (2) 58. (3) 59. (1) 60. (2)
91. (4) 92. (2) 93. (4) 94. (2) 95. (2) 96. (2)
61. (2) 62. (3) 63. (2) 64. (3) 65. (2) 66. (2)
97. (1) 98. (3) 99. (3) 100. (2)
67. (2) 68. (3) 69. (1) 70. (3) 71. (2) 71. (2)

BLAM–37
chtxf.kr
2nd Method
EXPLANATIONS
1 1
a2 b2 4
a2 b2
Type-1
Putting a = b = 1
Based on Square and Cubic Formulae a2 + b2 = 1 + 1 = 2.
1. (1) 25a2 + 40ab + 16b2 = (5a + 4b)2 6. (4) a2 + b2 = 24 and ab = 6
(a + b)2 = a2 + b2 + 2ab
= (5 × 23 – 29 × 4)2 = (115 – 116)2 = 1
= 24 + 2 × 6 = 36
2. (1) (a + b)2 = a2 + b2+ 2ab
a+b=6
25 = 13 + 2ab 2ab = 25 – 13 = 12
(a3 + b3) = (a + b) (a2 – ab + b2)
(a – b)2 = a2 + b2 – 2ab = 13 – 12 = 1
= 6 (24 – 6) = 6 × 18 = 108
a–b=1
7. (2) (x + y)3 = x3 + y3 + 3 (xy) (x + y)
2nd Method
125 = 35 + 3(5) xy
Let we put a = 3 and b = 2
15xy = 125 – 35 = 90
a+b=5 3 + 2 = 5 (Satisfied)
a2 + b2 = 13 (3)2 + (2)2 = 9 + 4 = 13 90
xy = 6
(Satisfied) 15
a–b=3–2=1 x y 1 1 5
3. (3) 48x3 – 8x2 – 93x – 45
xy y x 6
= 48x3 – 80x2 + 72x2 – 120x + 27x – 45
= 16x2 (3x – 5) + 24x (3x – 5) +9 (3x – 5) 2nd Method
= (3x – 5) (16x2 + 24x + 9) Let we put x = 3, y = 2
= (3x – 5) (4x + 3) (4x + 3) x3 + y3 = 35 33 + 23 = 27 + 8 = 35
4. (3) Option (c) ls] x+y=5 3+2=5

x (x + 2) – 15 = x (x – 5) + 11 1 1 1 1 2 3 5
x2 + 2x – 15 = x2 – 5x + 11 x y 3 2 6 6
7x = 26
x y xy
7x – 26 = 0 8. (1) x y k
7 4
;s Quadratic ugha gS because ;s
eqn eqn ax2 + bx + c =
x–y=k
0 osQ form esa ugha gSA
x + y = 7k
1 1 (x + y)2 – (x – y)2
5. (3) a2 + b2 + 4
a2 b2 = 49k2 – k2
1 1 4xy = 48k2
a2 b2 4 16k = 48k2
a2 b2
1
FGa 1 IJ 2 2
FGb 1 IJ 2 2 4
k=
3
H a K H bK
1 4
xy = 4k = 4
FG a 1 IJ 2 FGb 1 IJ 2 0
3 3
H a K H bK Aliter :

1 1 x y xy
a 0; b  0 x–y=
7
=
4
a b
7(x – y) = x + y
1
Ÿa Ÿa 2 1 7x – 7y = x + y
a
6x = 8y
a=±1 3x = 4y
blh izdkj Also 4x – 4y = xy
a = b = +1 4x – 3x = xy
a2 + b2 = 1 + 1 = 2 x = xy

BLAM–38
chtxf.kr
y 1
15. (2)
a (a + b + c ) = 45
3x = 4
b (a + b + c ) = 75
4 + c (a + b + c) = 105
x=
3 ( a + b + c) ( a + b + c) = 225
4
a + b + c = 15
xy So,
3
a (a + b + c) = 45
9. (4) a2 = 2 a= 2 a × 15 = 45 a=5
a+1= Similarly
2 1
b (a + b + c) = 75
4th option ls, b × 15 = 75 b=3
a 1 2 1 2 1 3 2 2 and
= c (a + b + c) = 105
3 2a 3 2 2 3 2 2 3 2 2
c × 15 = 105 c=7
3 2 3 4 2 2 hence,
= = 1 2 =a+1
9 8 a2 + b2 + c2 = 32 + 52 + 72 = 83
10. (1) x2 + y2 – z2+ 2xy = x2 + y2 + 2xy – z2 2nd Method
= (x +y)2 – z2 = (x+y+z) (x+y–z) putting the value
= (b + c – 2a + c + a – 2b + a + b – 2c) (x + y– z) a = 3, b = 5, c = 7
=0 a (a + b + c) = 3 × 15 =45
Aliter : b (a + b + c) = 5 × 15 = 75
Let we put a = 1, b = 1, c = 0 in x, y, z then c (a + b + c) = 7 × 15 = 105
x = –1, y = –1, z = 2 So, a2 + b2 + c2 = 32 + 52 + 72 = 83
x2 + y2 – z2 + 2xy = (–1)2 + (–1)2 – (2)2 + 2 × 16. (1) ;gk¡] a + b + c = 0
(–1) × (–1)
a3 + b3 + c3 – 3abc = 0
=1+1–4+2
17. (3) fn;k x;k gS, x + y = 7
=0
11. (4) x2 – 2x + y2 + 6y + 10 = 0 vc, x3 + y3 + 21xy
x2 – 2x + 1 + y2 + 6y + 9 = 0 = (x + y)3 – 3xy (x + y) + 21xy
(x – 1)2 + (y + 3)2 = 0 x–1=0 = (7)3 – 3xy (7) + 21xy = 343 – 21xy + 21xy
x=1 = 343
y+3=0 Aliter :
y = –3 Let we put x = 4, y = 3
x2 + y2 = 1 + 9 = 10 x+y=7 4+3=7
12. (1) Given, x3 – y3 + 21xy = (4)3 + (3)3 + 21 × 4 × 3
(a – 7) + (b – 9) + (c – 11) = (a + b + c) – 27 = 64 + 27 + 252
= 27 – 27 = 0 = 343
13. (1) Given 18. (3) ;fn a + b + c = 0,
rks, a3 + b3 + c3 = 3 abc
x2 y2 z2
3 ;gk¡] y – z + z – x + x – y = 0
yz zx xy (y – z)3 + (z – x)3 + (x – y)3
= 3 (y – z) (z – x) (x – y)
x3 y3 z 3
3 x y z
xyz 19. (3) b c; c a; a b
a b c
x3
+ y3
+ z3
– 3xyz = 0 iqu%, b – c + c – a + a – b = 0
that means
x+y+z=0 FG x IJ FG y IJ FG z IJ
3 3 3

(x + y + z)3 = 0
Ha K HbK H c K
= (b – c)3 + (c – a)3 + (a – b)3
14. (2) x + y + z = a – b + b – c + c – a = 0 3xyz
= 3 (b – c) (c – a) (a – b)
x3 + y3 + z3 –3xyz = 0 abc

BLAM–39
chtxf.kr
20. (3) x3– x2y
– + xy2= + y3 – x3 – y3 x2y xy2 x2 + y2 + 2xy = xy + 1
= (x + y)3 –3xy (x + y) –xy(x + y) x2 + y2 + xy = 1
= (x + y)3 – 4xy (x + y) = a3 –4b2a Multiplying Both sides by (x – y)
Aliter : (x – y) (x2 + y2 + xy) = 1 × (x – y)
Let we put x = 1, y = 1 x3 – y3 = x – y
a = 2, b = 1 1=x–y
x+y=a 1+1=2
x y 1
xy = b2 1 × 1 = (1)2 = 1
x – x y – xy + y3
3 2 2 2nd Method
= (1)3 – 12 × 1 – 1 × 12 + 1 putting x = 1, y = 0
=1–1–1+1=0 (x + y)2 = xy + 1
From option (3) (1 + 0)2 = (1 × 0) + 1
a3 – 4b2a = (2)3 – 4 × (1)2 × 2 1=1
=8–8=0 x–y=1–0=1
21. (4) Given 1
25. (3) x3 + y3 + z3 – 3xyz = (x y z)
11 13x 11 13y 11 13z 2
5
x y z
(x y )2 (y z )2 (z x )2

FG 11 13IJ FG 11 13IJ FG 11 13
IJ 5 1
( 333 333 334 ) (0 + 1 + 1) = 1000
Hx K Hy K Hz K =
2
26. (3) x + y + z = 6
11G
F1 1 1I
J 39 5 nksuksa rjiQ square djus ij]
Hx y zK x2 + y2 + z2 + 2xy + 2zy + 2zx = 36
20 + 2 (xy + yz + zx) = 36
1 1 1 5 39 44
4 xy + yz + zx = 8
x y z 11 11
x3 + y3 + z3 – 3xyz
22. (3) x3 + 3x2 + 3x = 7 = (x + y + z)(x2 + y2 +z2 – xy – yz – zx)
x3 + 3x2 + 3x + 1 = 7 + 1 = 8 = 6 (20 – 8) = 72
(x + 1)3 = 23 Aliter :
x+1=2 x=1 Let we put x = 4, y = 2, z = 0
Aliter : x+y+z=6 4 + 2 + 0 = 6 (Satisfied)
In these type of questions we directly go through x2 + y2 + z2 = 20 (4)2 + (2)2 + 02 = 20 (Satis-
the option (3) fied)
x3 + 3x2 + 3x = 7 x3 + y3 + z3 – 3xyz
13 + 3 × 12 + 3 × 1 = 7 = (4)3 + (2)3 + 0 – 0
7=7 = 64 + 8 = 72
23. (1) Put the value 27. (3) a – b = 3
x = 1, y = 2 a3 – b3 = 117
a3 – b3 = (a – b)3 + 3ab (a – b)
5x y 5 1 2 3 117 = 27 + 3ab (3)
5x y 5 1 2 7 9ab = 117 – 27 = 90
Now, According to the Question ab = 10
(a + b)2 = (a – b)2 + 4ab = 9 + 40 = 49
4x 2 y2 4 xy |a + b| = 7
2
9x 16y 2 24 xy Aliter :
Let we put a = 5, b = 2
b4 1g b2g b4 1 2g 2 a–b=3 5 – 2 = 3 (Satisfied)
= =0 a3 + b3 = 117 (5)3 – (2)3 = 125 – 8 = 117
b9 1g 16 b2g b24 1 2g 2
(a + b) = 5 + 2 = 7
24. (1) Given, 28. (1) By Factor theorem
(x + y)2 = xy + 1 x–2=0

BLAM–40
chtxf.kr
x=2 1
x2 + k1x + k2 = 0 = (2x2 + 2y2 + 2z2 – 2xy – 2yz – 2zx)
2
(2)2 + k1 × 2 + k2 = 0
2k1 + k2 = –4...(1) 1
= (x2 + y2 – 2xy + y2 + z2 – 2yz + x2 + z2 – 2zx)
Again, x + 3 = 0 2
x=–3
1
(–3)2 + k1 × (–3) + k2 = 0 = [(x – y)2 + (y – z)2 + (z – x)2]
2
–3k1 + k2 = –9 ...(2)
from eqn (1) & (2) 1
= [(–1)2 + (–1)2 + (2)2 ]
k1 = 1, k2 = –6 2
2 2 2
29. (2) (a b c )2 = a + b + c +2 (ab + bc + ca) 1 1
= (1 + 1 + 4) = ×6=3
Ÿ 36 = 14 + 2 (ab + bc + ca) 2 2
Ÿ ab + bc + ca = (36 – 14) ÷ 2
2nd Method
ab + bc + ca = 11 ....(i)
Note : If a, b & c are in Continuous form then
? a3 + b3 + c3 – 3abc = (a + b + c)
a2 + b2 + c2 – ab – bc – ca = 3 (always)
(a2 + b2 + c2 – ab – bc – ca)
Ÿ 36 – 3abc = 6 (14 – 11) [By (i)] 32. (1) Given,
Ÿ 36 – 3abc = 84 – 66 = 18 x–y=7
Ÿ 3abc = 36 – 18 = 18 (x – 15) – (y – 8) = (x – y) – 7
Ÿ abc = 6 =7–7=0
Aliter : A/q
Let we put a = 3, b = 2, c = 1 (x – 15)3 – (y – 8)3
a+b+c=6 3 + 2 + 1 = 6 (Satisfied) = (x – 15 – y – 8) {(x – 15)2 + (y – 8)2 + (x – 15) (y – 8)
a2 + b2 + c2 = 14 33 + 23 + 12 = 9 + 4 + 1 = 14 = 0 {(x – 15)2 + (y – 8)2 + (x – 15) (y – 8)} = 0
(Satisfied)
33. (4) x y 18 1 x y 18 1 ...(1)
a3 + b3 + c3 = 36 (3)3 + (2)3 + 13 = 27 + 8 + 1
= 36 (Satisfied)
x y 3 2 1 x y 3 2 1 ...(2)
abc = 3 × 2 × 1 = 6
30. (2) a3 + b3 = (a + b) (a2 – ab + b2) from eqn (1) & (2)
9 = 3(a2 + b2 – ab) x 3 2, y 1

a2 + b2 – ab =
9
=3 12xy (x2 – y2) = 12 3 2
RSe
1 3 2 j b1g UVW
2 2
3
T
(a + b)2 – 2ab – ab = 3
= 612 2.
9 – 3ab = 3
3ab = 9 – 3 = 6 ab = 2 34. (3) p = 99 fn;k gS)
p (p2 + 3p + 3)
1 1 a b 3
+ = = = p3 + 3p2 + 3p
a b ab 2
= p3 + 3p2 + 3p + 1 – 1
Aliter :
= (p + 1)3 – 1
Let we put a = 2, b = 1
= (99 + 1)3 – 1
a3 – b3 = 9 (2)3 + (1)3 = 8 + 1 = 9
= (100)3 – 1
a+b=3 2+1=3
= 999999
1 1 1 3 35. (1) a2 + b2 + c2 = 2a – 2b – 2c – 3
1
a b 2 2 a2 – 2a + 1 + b2 + 2b + 1 + c2 + 2c + 1 = 0
31. (4) x = 997 (a – 1)2 + (b + 1)2 + (c + 1)2 = 0
y = 998 a – 1 = 0, b + 1 = 0, c + 1 = 0
z = 999 a = 1, b = – 1, c = – 1
x – y = 997 – 998 = –1 a+b+c=1–1–1=–1
y – z = 998 – 999 = –1 36. (1) x2 + y2 + z2 = 2(x + z – 1)
z – x = 999 – 997 = 2 x2 + y2 + z2 = 2x + 2z – 2
x2 + y2 + z2 – xy – yz – zx x2 – 2x + y2 + z2 – 2z + 2 = 0

BLAM–41
chtxf.kr
x2
– 2x + 1 + + y2 z2
– 2z + 1 = 0 y+z–x=0 ...(ii)
(x – 1)2 + y2 + (z –1)2 = 0 oz+x–y=0 ...(iii)
[ a2 + b2 + c2 = 0 a = 0, b = 0, c = 0] (i), (ii) ,oa (iii) dks tksMu
+ s ij]
x–1=0 x=1
2 (x + y + z) – (x + y + z) = 0
y=0
x+y+z=0
z–1=0
z=1 1 1 1
42. (3)  x 0
x3 + y3 + z3 = 1 + 0 + 1 = 2 y z
37. (2) x = z = 225, y = 226
xy yz zx
x + y + z = 225 + 226 + 225 = 676 0
xyz
x3 + y3 + z3 – 3 xyz
xy + yz + zx = 0
1
= (x + y + z) [(x – y)2 + (y – z)2 + (z – x)2] Given
2
x+y+z=9
1 squaring Both Sides
= × 676 [(225 – 226)2+(226 – 225)2 + (225 – 225)2]
2 x2 + y2 + z2 + 2 (xy + yz + zx) = 81
x2 + y2 + z2 + (2 × 0) = 81
1
= × 676 × (1 + 1) = 676 x2 + y2 + z2 = 81
2
Now,
38. (2) a2 + b2 + c2 + 3 = 2a + 2b + 2c x3 + y3 + z3 – 3xyz
a2 – 2a + 1 + b2 – 2b + 1 + c2 – 2c + 1 = 0 = (x + y + z) (x2 + y2 + z2 – xy – yz – zx)
(a – 1)2 +(b –1)2 +(c – 1)2 = 0 = 9 (81 – 0)
a–1=0 = 729
a = 1; b – 1 = 0 43. (3) a + b + c = 9
b=1 a2 + b2 + c2 = (a + b + c)2 – 2 (ab + bc + ca)
rFkk] c – 1 = 0 [ab + bc + ca] vf/dre gksxk ;fn
c=1 a = b = c]
a+b+c=3 3 (a2 + b2 + c2) = (a + b + c)2
39. (3) a2 + b2 + c2 = 2a – 2b – 2c – 3
a2 + b2 + c2 – 2a + 2b + 2c + 1 + 1 + 1= 0 92
(a2 + b2 + c2) = 27
(a2 – 2a + 1) + (b2 + 2b +1) + (c2 + 2c + 1) = 0 3
(a – 1)2 + (b + 1)2 + (c + 1)2 = 0 44. (4) x2 + y2 +1 = 2x x2 + y2 +1 – 2x = 0
a–1=0 x2 – 2x + 1 + y2 = 0
a=1 (x – 1)2 + y2 = 0
b+ 1= 0 x–1=0
b=–1 x = 1 ,oa y = 0
c +1 = 0 x3 + y5 = 1 + 0 = 1
c=–1 45. (4) x + y + z = 0
a–b+c=1+1–1=1 –x = y + z
40. (1)  x2 – 9 = 0 (–x)2 = (y + z)2
x 9 3 or 3 x2 = y2 + z2 + 2yz ...(i)
2 2 2
Put the value of x = 3 or –3 x y z
Expression = 2
x=3 x yz
px3 – 2x2 – qx + 18 = 0 2 2 2 2
y z 2yz y z
27p – 18 – 3q + 18 = 0 =
27p = 3q y2 z2 2yz yz

p 1 2y 2 2z 2 2yz 2(y 2 z2 yz )
= = =2
q 9 y2 z2 yz y2 z2 yz
41. (4) (x + y – z)2 + (y + z – x)2 + (z + x – y)2 = 0 Aliter :
(x + y – z) = 0 ... (i) Let we put x = 1, y = –1 and z = 0

BLAM–42
chtxf.kr
x+y+z=0 1 – 1 + 0 = 0 (Satisfied) b = 11k – 7k = 4k

x y2 z2 (1)2 ( 1)2 0 a c 2k 5k 7
= =
2 2 2 b 4k 4
x y (1) 0
Aliter :
1 1 Let we take a = 2, b = 4, c = 5 then our condition
= =2
1 satisfied
46. (1) x + y + z = 0 a c 2 5 7
x + z = –y b 4 4
Squaring djus ij, 49. (3) P + Q + R
x2 + z2 + 2xz = y2 =y+z–x+z+x–y+x+y–z
x2 + z2 = y2 – 2xz =x+y+z
3y 2  x 2  z 2 P – Q = y + z – x – z – x + y = 2(y – x)
2y 2  xz Q–R=z+x–y–x–y+z
= 2 (z – y)
3y 2  y 2  2xz 4y 2 2xz R–P=x+y–z–y–z+x
= =
2y  xz
2
2y 2 xz = 2 (x – z)
P3 + Q3 + R3 – 3 PQR
2(2y 2  xz ) 1
= =2
2y 2  xz = (P + Q + R) [(P – Q)2 + (Q – R)2 + (R – P )2]
2
OR
1
;gk¡, x + y + z = 0 = (x + y + z) [4 (y – x)2 + 4 (z – y)2 + 4 (x – z)2]
2
Let x = 1, y = –1, z = 0
1
3y 2  x 2  z 2 =4× (x + y + z) [(x – y)2 + (y – z)2 + (z – x)2]
= 2
(2y 2  xz )
= 4 × (x3 + y3 + z3 – 3xyz)
3.(1)2  (1)2  02 = 4 × 3 = 12
=
[2( 1)2  1 u 0] 50. (2) Expression

3 1 4 1 1 1
= = =2 =
2 2 2 x1 2 x2 2 x3
47. (3) a3 + 3a2 + 9a = 81
(2  x 2 )(2  x 3 )  (2  x 1 )
tc a = 3 (2  x 3 )  (2  x1 )(2  x 2 )
=
L.H.S = 27 + 27 + 27 = 81 (2  x1 )(2  x 2 )(2  x 3 )
3 3 4  2x 2  2x 3  x 2 x 3  4  2x1  2x 3
a3 = (3)3 +
a 3  x1x 3  4  2x 1  2x 2  x1x 2
=
= 27 + 1 = 28 (4  2x1  2x 2  x1x 2 )(2  x 3 )

a b 6
48. (4) 12 4 x1 4x 2 4x 3
c 5
x1x 2 x 2 x 3 x 3 x1
=
a b c 6 5 11 8 4 x1 4 x 2 2x1x 2 4 x 3
=
c 5 5 2x1x 3 2x 2 x 3 x1x 2x 3
a + b + c = 11k
c = 5k x x x – 16  12  x x  x x
1 2 3 1 2 2 3
Again, x x
= 3 1
b c a 11 x x x – 16  8  2x x  2x x
1 2 3 1 2 1 3
a 2  2x x  x x x
2 3 1 2 3
a = 2k
a + b + c = 11k x x x x x x x x x 4 1
= 1 2 2 3 3 1 1 2 3 =
2k + b + 5k = 11k 2 ( x1x 2  x1x 3  x 2 x 3  x1x 2 x 3  4) 2

BLAM–43
chtxf.kr
51. (3) 3x + 5y + 7z = 49 ...(i) 2z – x x
9x + 8y + 21z = 126 ...(ii) = –
z–x z –x
By equation (i) × 3 – (ii),
2z – x – x 2z – 2 x
9x + 15y + 21z = 147 = =
z –x z–x
9x + 8y + 21z = 126
– – – –
2(z – x )
7y = 21 = =2
z –x
21 Aliter :
y= =3
7 Let we put x = 3, y = 1, z = 2 then
52. (3)  x + y + z = 0 x y 3 1
x3 + y3 + z3 = 3xyz =2 = 2 (Satisfied)
z 2
A/q

x2 y3 z2
LM y OP L x O
3z 3 xz 3x N y z Q MN x z PQ
1 3
x3 y3 z3
= 1 2 3 2
3xz
= –1 + 3 = 2
3xyz
= y a2
3 xz 55. (1) a4 + 1 = ( 4b 2 – b 4 – 1)
2nd Method b2
putting x = 2, y = z = –1 a4b2 + b2 = 4a2b2 – a2b4 – a2
a4b2 + b2 – 4a2b2 + a2b2 + a2
x2 y3 z2 a4b2 + b2 – 2a2b2 – 2a2b4 + a2b4 + a2 = 0
3z 3xz 3x b2 (a4 +1 – 2a2) + (a2b4 + a2 – 2b2a2) = 0
4 1 1 b2 (a2 – 1)2 + a2 (b4 +1 – 2b2) = 0
= b2 (a2 – 1)2 + a2 (b2 –1)2 = 0
3 6 6
[If x2 + y2 = 0 x = 0 and y = 0]
4 1 3 b2 (a2 – 1)2 = 0
= 1 y
3 3 3 a2 – 1 = 0 a2 = 1
and a2 (b2 – 1)2 = 0
53. (3) x + y + z = 22;
b2 – 1 = 0 b2 = 1
xy + yz + zx = 35
a4 + b4 = (1)2 + (1)2
(x + y + z)2
=1+1=2
= x2 + y2 + z2 + 2xy + 2yz + 2zx
Aliter :
(22)2 = x2 + y2 + z2 + 2 × 35
Let we put a = b = 1
484 = x2 + y2 + z2 + 70
x2 + y2 + z2 = 484 – 70 = 414 a2
(x – y)2 + (y – z)2 + (z – x)2 a4 + 1 = (4b2 – b4 – 1)
b2
= 2 (x2 + y2 + z2 – xy – yz – zx)
= 2 (414 – 35) = 2 × 379 = 758 1
1+1= (4 × 12 – 14 – 1)
1
x y
54. (3) =2 2 = 2 (Satisfied)
z
a4 + b4 = (1)4 + (1)4 = 2
x + y = 2z
y = 2z – x 1–a
56. (2) Let, =x
y x a
y –z x –z
a 1
2z – x x =
1–a x
= –
2z – x – z x – z

BLAM–44
chtxf.kr
3 1
3x + 9 = 19 – x – 13 = – – 11
x x

19x – 3 1
3x + 9 = x+ = 13 – 11
x x
3x2 + 9x = 19x – 3 1
3x2 + 9x – 19x + 3 = 0 x+ =2
x
3x2 – 10x + 3 = 0
x2 – 2x + 1 = 0
3x2 – 9x – x + 3 = 0
(x – 1)2 = 0 x=1
3x (x – 3) – 1 (x – 3) = 0
(x – 3) (3x – 1) = 0 a
=1 a=b
b
1
x = 3 or a + b = 10
3
a = 5 and b = 5
When x = 3
3ab + 4a2 + 5b2
1–a 1–a = 3 × 5 × 5 + 4 × (5)2 + 5 × (5)5
=3 =9 = 75 + 100 + 125 = 300
a a
58. (3)
1 3x + 4y – 2z + 9 = 17 ...(i)
–1=9
a 7x + 2y + 11z + 8 = 23 ...(ii)
5x + 9y + 6z – 4 = 18 ...(iii)
1 +
= 9 + 1 = 10
a 15x + 15y + 15z + 13 = 58

1 15x + 15y + 15z


a= = 58 – 13 = 45
10
x+y+z=3
1 x + y + z – 34 = – 34 + 3
Again, when x =
3 x + y + z – 34 = – 31

1–a 1 2z
59. (1) x + 3y – =6
4
a 3
4x + 12y – 2z = 24 ...(i)
1–a 1
= 2
a 9 x+ (2y + 3z) = 33
3
1 1 3x + 4y + 6z = 99 ...(ii)
–1=
a 9 1
(x + y + z) + 2z = 9
1 1 7
= +1
a 9 x + y + 15z = 63 ...(iii)
By equation (i) × 3 + (ii),
10
= 12x + 36y – 6z = 72
9
3x + 4y + 6z = 99
9
a= 15x + 40y = 171 ... (iv)
10
15
a Again, equation (i) × + (iii),
57. (2) Let, =x 2
b
30x + 90y – 15z = 180
b 1 x + y + 15z = 63
=
a x 31x + 91y = 243 ...(v)
A/q equation (iv) rFkk (v), dks tksM+us ij

BLAM–45
chtxf.kr
15x + 40y = 171 Aliter :
Let we put x = 3, y = 4 then
31x + 91y = 232
x4 + y4 + x2y2 = 481 (3)4 + (4)4 + (3)2(4)2
46x + 131y = 414
= 81 + 256 + 9 × 16 = 481 (Satisfied)
60. (2) ;gk¡, x2 – xy + y2 = (3)2 – 3 × 4 + (4)2
1 1 = 9 – 12 + 16
f (x) =  = 13
x x 1
64. (3) ;gk¡ A = 1 + 2p rFkk B = 1 + 2–p
1 1
f (1) = 
1 2 1
B= 1
2p
1 1
f (2) = 
2 3 rFkk 2 = A – 1
p

1
1 1
 B = 1
f (3) = A 1
3 4
.................. A 1 1
B=
.................. A 1

1 1 A
f (10) =  B=
10 11 A 1
f (1) + f (2) + f (3) ........... + f (10) Aliter :
Let we take P = 0
1 1 1 1 1 ...... 1 1 A = 1 + 2P 1 + 2° = 1 + 1 = 2
= 1       
2 2 3 3 4 10 11
1 1
first term rFkk last term dks NksM+dj ck¡dh lc terms B = 1 + 2–P 1 P
1 =1+1=2
2 2
cancel gks tk,axhA Go through the option (3)
1 10 A 2
f (1) + f (2) + ........... + f (10) = 1  = B= = 2 (satisfied)
11 11 A 1 2 1
61. (2) Given
(2 + 1) (22 + 1) (24 + 1) (28 + 1) 3
65. (2) (1 – p 2 )(1 – q 2 )
2
b
2 1 g
= 2 1 × (2 + 1) (22 + 1) (24 + 1) (28 + 1) nksuks rjiQ square djus ij
b g 3
= (22 – 1) (22 + 1) (24 + 1) (28 + 1) (1–p2) (1–q2) =
4
= (24 – 1) (24 + 1) (28 + 1)
= (28 – 1) (28 + 1) 3
1 – p2 = 1 rFkk 1 – q2 = 4
= 216 – 1
62. (3) a2 + b2 = = 4b + 6a – 13 3 1
a2 + b2 – 4a – 6a + 13 = 0 p=0 rFkk q2 = 1 – 4 = 4
a2 – 6a + b2 – 4b + 13 = 0
1
(a2 – 6a + 9) + (b2 –4b + 4) = 0 q=
2
(a – 3)2 + (b – 2)2 = 0
A/q,
a = 3 rFkk b = 2
a+b=3+2=5 2p 2 2q 2 2pq 2p 2 2q 2 2pq
63. (2) ;gk¡, x + y + x y = 481
4 4 2 2

1 1
(x2 + y2)2 – x2y2 = 481 = 2 02 2 2 0+ 2 02 2 –2 0
(x2 + y2)2 – 122 = 481 4 4
(x2 + y2)2 = 481 + 144 1 1
(x2 + y2)2 = 625 =
2 2
x2 + y2 = 625
x2 + y2 = 25 2 2
= = 2
x2 + y2 – xy = 25 – 12 = 13 2 2

BLAM–46
chtxf.kr
2
66. (2) (a + b) – 2 (a + b) = 80
1 1 P2  Q 2
ekuk a + b = x vc,  2 =
x2 – 2x = 80
P 2
Q PQ
2
b g
x2 – 2x – 80 = 0 (P  Q )2  2PQ
x2 – 10x + 8x – 80 = 0 =
(PQ)2
(x – 10) (x + 8) = 0
2
x = 10 | x = –8
a + b = 10 or a + b = – 8 =
e7  4 3 74 3 j  2 u1
2
1
a = 10 – b
(10 – b)b = 16 = 142  2
– b2 + 10b = 16 = 196 – 2
b2 – 10b + 16 = 0 = 194
(b – 8) (b – 2) = 0 69. (1) ;gk¡,
b = 2, 8
a = 8, 2 x=2+ 3
3a – 19b = 3 × 8 – 19 × 2 y=2– 3
= 24 – 38 z=1
= – 14
3a – 19b = 3 × 2 – 19 × 8 yz = (2 – 3)×1=2– 3
= 6 – 152 xz = (2 + 3)×=2+ 3
= – 146
3a – 19b = –14 xy = (2 + 3 ) × (2 – 3)=1
Aliter : bu values dks equation 1 esa j[kus ij
Let we take a = 8, b = 2
ab = 16 8 × 2 = 16 (Satisfied)
FG x IJ  FG y IJ  FG z IJ  2LM 1  1  1 OP ,
(a + b)2 – 2 (a + b) = 80 (8 + 2)2 – 2(8 + 2)
H yz K H xz K H xy K N x y z Q
= 100 – 20 = 80 (Satisfied)
3a – 19b = 3 × 8 – 19 × 2 = 24 – 38 = –14 =
2 3 2 3 1
   2M
L 1  1  1OP
67. (2) ;gk¡] xy + z = 1 ...(i)
2 3 2 3 1 N2  3 2  3 Q
yx+ z = 1024 ... (ii) 2 2

zx+y = 729 .... (iii)


=
e2  3 j  e2  3 j 1  2
LM 2  3 2 3
PPO
1
x y+z = 1 43 MN 43 Q
x=1
yx+ z= 210 43 4 3  43 4 3
x + z = 10 z = 9 rFkk y = 2
=
1
1  2 4 1 b g
(z + 1)x + y +1 = (9 + 1)1+2+1 = 14 + 1 + 10 = 25
= 104 70. (3);gk¡ x + y = 3
= 10000 x3 + y3 + 9xy = (x + y)3 – 3xy(x + y) + 9xy
= 27 – 3xy × 3 + 9xy
68. (3) ;gk¡, P = 7 + 4 3
= 27 – 9xy + 9xy
PQ = 1
= 27
1 Aliter :
Q=
P Let we take x = 2 and y = 1
x+y=3 2+1=3
1 74 3
Q= u x3 + y3 + 9xy = (2)3 + (1)3 + 9 × 2 × 1
74 3 74 3
= 8 + 1 + 18 = 27
74 3 74 3 x 8 1
= = 71. (2) A =
2
49  48 x 4 1
e
72  4 3 j
2

A=
dx i 4
1
= Q 74 3
x 4 1

BLAM–47
chtxf.kr
3ab (3a  b )
dx 4
id
1 x 4 1 i = 3ab (a  3b )
=
dx 4
1 i
= x4 – 1 x b  3a
y a  3b
blh izdkj,
rFkk z prime numbers gS
74. (1) As, x, y
y4 1
B=
y2  1
equal to y2 – 1 blfy, equation dks satisfy djsaxs
x + y + z = 38
At x = 2, A = 24 – 1 = 15
At y = 9, B = 92 – 1 = 80 Possible gksxk fliZQ x = 19, y = 17 rFkk z = 2 ds fy,
A2 + 2AB + AB2 = 152 + 2 × 15 × 80 + 15 × 802 x dh maximum value 19 gksxhA
= 225 + 2400 +96000 75. (3) ;gk¡,
= 98625 x + y = 2017
72. (2) x – 4y = 0 ....(i) rFkk ge ys ldrs gSa] x = 1, y = 2016
x + 2y = 24 ... (ii) i.e., ;gk¡ ,d value odd rFkk nwljh value even gSaA
nksuksa equations dks subtract djus ij (–1)x + (–1)y = (–1)1 +(–1)2016
x – 4y = 0 = – 1 + 1= 0
x + 2y = 24
– – – 1 3 1
– 6y = – 24 76. (3) x  =
x 2
y = 4 ( bl value dks equation (i) esa j[kus ij) nksuksa rjiQ square djus ij
x–4×4=0
x = 16 1 F 3 1 I 2

vc, x2 
x2
2= GH 2 JK
2 x  3y 2 u 16  3 u 4
= 1 3 1 2 3
2 x  3y 2 u 16  3 u 4 x2  2=
x2 4
32  12 44 11
= = =
32  12 20 5
x2 
1
2=
2 2 3 e j
2
x y x 4
73. (1)  3
a b
1 2 3
x2  = 2
bx  ay 3ab ... (i) x2 2

x y 1 3 2
 9 x2  =
b a x2 2
ax  by 9ab ...(2) nksckjk square djus ij
x y 1 1 F 32 I 2

a 3ab
=
3ab b
=
b a
x4 
x4
2= GH 2 JK
b 9ab 9ab a a b

1
1 3 4 4 3
x y x4  2=
x4 4
9a b  3ab 2
2
3a b  9ab 2
2
b  a 2
2

9a 2b  3ab 2 1 74 3
x= x4  2=
a 2  b2 x4 4

3a 2b  9ab 2 1 74 3
y= x4  = 2
a 2  b2 x4 4
x 9a 2b  3ab 2 1 1  4 3
y 3a 2b  9ab 2 x4  =
x4 4

BLAM–48
chtxf.kr
2nd Method nksuksa equations dks add djus ij
2(a2 + b2) = 1
1 3 1
x 1
x 2 a2 + b2 =
2

LMF I 2 O 2
1 1
2P
4 1 3 1 a2 = , b2 =
x
x 4 MMNGH 2 JK PPQ 2 4 4

d1  a id1  b i = FGH1  14 IJK FGH1  14 IJK


2 2

LM 4 2 3 O
2P
2

2
=
MN 4 PQ 3 3 3
= u =
4 4 4
LM 2 3 4 OP 2

2
=
MN 4 PQ 79. (2)
P2 q2 Pq
P2 q2
er 2
j
S 2 Pq
2 2 rs rs
r S
12 16 16 3
= 2
16 FG p q IJ bP qg 2 2
P2 q2 2Pq

4 16 3 1 4 3
H p q K bP q g 2
P 2
q2 2Pq
=
16 4
[We know that dr 2
S2 p 2 h 2Pq
rs
1
If x
1
x
a then x
4

x4
= (a2 –2)2 – 2] = dr 2
S 2 pq h 2pq
rs
77. (3) a + a2 + a3 – 1 = 0
a + a2 + a3 = 1
Pq
LMr 2
S 2 2rs OP
1 + a + a2 =
1
(nksuksa rjiQ a ls divide djus ij)
MN rs PQ
a = F
P qG
r 2
s2 2rs I
nksuksa rjiQ (a –1) ls multiply djus ij] H rs JK
a 1
(a – 1) (1 + a + a2) =
a br Sg FG r S IJ 2 2

a3 – 1 = 1 
1
=
br Sg H r S K 2

a
p q r S
1
a3  = 1+1 p q r S
a
2nd Method
1
a3  2 put the value
a P=r=2
78. (4) ;gk¡ fn;k gqvk gS q=S=1
p q 1 r s
a 2  b 2  ab + a 2  b 2  ab = 1
p q 3 r s
ekuk]
a 2  b 2  ab = 0 rFkk 80. (1) Given the equation,
x2 – x + 3 = 0
a 2  b 2  ab = 1
1
a2 + b2 + ab = 0 (i) + += 1
1
a2 + b2 – ab = 1 (ii)

BLAM–49
chtxf.kr
3 1 2 2
=
1
3 (x – 6)2 +
( x – 6)2
= e 5 2 j e 5 j
2
4
+ 4 = [( + )2 – 2 ]2 – 2 2 2
2
= [(–1)2 – 2 × 3]2 – 2 × (3)2 = 22
2
e 5 j
= 25 – 18 = 7
= 18  (a b )2 (a – b )2 2(a 2 b2)
3 2 3– 2
81. (1) ;gk¡, a = rFkk b =
3– 2 3 2 83. (2) 2x + 3y – 5z = 18 ...(1)
3x + 2y + z = 29 ...(2)
3 2 3– 2 x + y + 3z = 17 ...(3)
a+b= +
3– 2 3 2
eq (3) dks 2 ls multiply djds eq (1) ls subtract
2 2 djus ij
=
e 3 2 j e 3– 2 j y – 11z = – 16 ....(4)
2 2
3 – 2 eq (1) dks 3 ls multiply rFkk eq(2) dks 2 ls multiply djus
ij
2F I
2 2
3 2 8x + 9y – 15z = 54
= H K  (a b )2 (a – b )2 2(a 2 b2) 6x + 4y + 2z = 58
1 – – – –
5y – 17z = – 4
= 10
5y – 17z = – 4 ...(5)
3 2 3– 2 eq (4) dks 5 ls multiply djus ij rFkk eq (5) dks blesa ls
ab = × =1 subtract djus ij
3– 2 3 2
a2 + b2 – ab 5y – 55z = – 80
(a + b2 + 2ab) – 3ab
2 5y – 17z = – 4
– + +
= (a + b)2 – 3ab -38z = – 76
= (10)2 – 3 × 1 = 97
z 2
2nd Method
z = 2 equation (4) esa j[kus ij
x y x y y – 11 × 2 = – 16
a= and b =
x y x y y – 22 = – 16
y = 22 – 16
If x – y = 1 then a + b = 2 (x + y) and ab = 1.
Now, y 6
a2 + b2 – ab = (a + b)2 –3ab z = 2 rFkk y = 6 equation (1) esa j[kus ij]
= [2 (3+2)]2 – 3 × 1
2x + 3 × 6 – 5 × 2 = 18
= 100 – 3 = 97.
2x + 18 – 10 = 18
82. (2) x2 – 16x + 59 = 0 2x = 18 – 8
x2 – 16x + 64 – 5 = 0 2x = 10
(x – 8)2 = 5 x 5
x–8= 5 xy + yz + zx = 5 × 6 + 6 × 2 + 2 × 5
= 30 + 12 +10
x–6=2+ 5
= 52
1 1 2– 5 2nd Method
= u putting the value
x –6 2 5 2– 5
x = 5, y = 6, z = 2
2– 5 then all expression are satisfied.
= So, xy + yz + zx = 30 + 12 + 10 = 52
4 –5

1
= 5 –2
FG x 1 IJ FG x 1 IJ FG x 2 1 IJ FG
1 x2
1
1
IJ
x –6 84. (4) H x KH x KH x2 KH x2 K
BLAM–50
chtxf.kr
90. (3) Given
FG x 3 1 IJ FG x 3 1 IJ = x
6 1
= H x 3 KH x 3 K x 6 x 3 5
Squaring Both Sides
85. (2) 5 x 12 x 13 x
x = 8 – 2 15
We known that
52 + 122 = 132 x – 8 = – 2 15
So x 2 Again, squaring Both sides
x=4 x2 + 64 – 16x = 60
86. (1) a4 + b4 = a2b2 x2 – 16x = –4
a4 + b4 – a2b2 = 0 ...(1) x2 – 16x + 6 = – 4 + 6
Now,
x2 – 16x + 6 = 2
a6 + b6 = (a2)3 + (b2)3
91. (3) a + b + c = 2S
= (a2 + b2) (a4 + b4 – a2b2)
= (a2 + b2) × 0 [By eqn (1)]
=0
bS a g bS b g bS c g
2 2 2
S2
a2 b2 c2
87. (4) a =
es 2
a2 2as j es 2
b2 2bs j es 2
c2 2cs j s2
2 2 2
a b c
+

= 2 3
4s 2 ea 2
b2 c2 j 2s a b b c g
2 2 2
a b c
a= 2 3
Similarly 4s 2 ea 2
b2 c2 j 2s 2s
b=2– 3 a 2
b 2
c 2

a+b= 2 e 3 j e2 3 j 4
a2 b2 c2
1
ab = 1 a2 b2 c2
Now,
92. (4)  (x + k) is a factor of each expression
a3 + b3 = (a + b)3 – 3ab (a + b)
= (4)3 – 3 × 1 × 4 = 52 x = –k will make each zero
88. (2) Given, k2 – pk + q = k2 – lk + m = 0
–pk + q = – lk + m
x3 3 lk – pk = m – q
1
100 5 k (l – p) = m – q
m q
x3 9 k= l p
1
100 25
93. (3) from Option (c)
3 3x + 1 = 0
x 16
x3 64 x 4
100 25 1
x= .
89. (4) (ad – bc)2 + (ac + bd)2 3
= a2d2 + b2c2 – 2abcd + a2c2 + b2d2 + 2abcd Given expression,
= a2 (c2 + d2) + b2 (c2 + d2) 3x3 + x2 – 12x – 4
= (c2 + d2) (a2 + b2)
=1×2=2 FG 1 IJ FG 1 IJ 12
FG 1 IJ 4
=3× H 27 K H 9K H 3K
2nd Method
putting a = b = 1 1 1
= 4 4 =0
c = 1, d = 0 9 9
(ad – bc)2 + (ac + bd)2 94. (2) 2x = 4y = 8z
= (0 – 1)2 + (1 + 0)2 2x = 22y = 33z
=1+1=2 x = 2y = 3z  3z = 2y

BLAM–51
chtxf.kr

z=
2
y FG x 2 1 IJ – FG x 1 IJ
3 3. (2) 2 H x2 K H x
–7=0K
Now,
R|F x 1 I 2 U| F x 1 IJ
xyz = 2y × y ×
2
y = 288
2 S|GH x JK 2 V| GH x K 7 =0
3 T W
4 3
y 288 FG
2 x
1IJ 2
4
FG x 1 IJ 7 0
3
3
H x K H x K
y = 216 y=6
x = 12, z = 4 F
2 Gx
1I
J
2
FG x 1 IJ
1 1 1 1 1 1 11 H xK
– H x K –3=0

2x 4y 8z 24 24 32 96
1
95. (4) ax.ay.az = (x + y + z)x + y + z ;fn x – x = y , gks] rks
ax + y + z = (x + y + z)x + y + z 2 y2 – y – 3 = 0
a = (x + y + z)...(1) 2 y2 – 3 y + 2y – 3 = 0
(x + y + z)y = ax = (x + y + z)x y (2y – 3) + 1 (2y – 3) = 0
x=y (y + 1) (2y – 3) = 0
Similarly
y = z and z = x 3
y=–1 ;k 2
a
x=y=z=
3
(from eqn (1)) tc] y = –1
Type–II 1
x– =–1
x
1 1
Based on a or a Formulae x2 + x –1 = 0
a a
x dh value real ugha gksxh

1 iqu%,
1. (4)  x + 4
x 1 3
x
2 x 2
1
x6 +
x 6 e4 3
–3 4 j 2
x2 1 3
= (52)2 – 2 = 2702. x 2
1 1 2x2 – 2 = 3x
[We know that when x + = a then x6 + 6 = 2x2 – 3x – 2 = 0
x x
(a3 – 3a)2 – 2] 2x2 – 4x + x – 2 = 0
2x (x – 2) +1 (x – 2) = 0
2p 1 (2x + 1 ) (x – 2) = 0
2. (3) 2 4
p 2p 1
1
x=
2
;k 2
p2 2p 1
4
2p 2 1
4. (2) x 66
x2
2
p 2p 1
8
p FG x 1IJ 2
2 66
2
H x K
p 2p 1
8
p p p FG x 1I
J
2
66 – 2 64
1
H xK
p 8 2 10
p 1
x 8
x

BLAM–52
chtxf.kr
x2 1 2x 1 1 19
x2 9
Expression = 16x 2 2 2
x
1 1
x2 1 1 8. (3) when x + 2 then xx + 2
= 2 x 2 x xx
x x x

1
x
x
dh value j[kus ij 9. (1) x 1 6 2 6 1

= 8 + 2 or – 8 + 2 = 10 or – 6
2nd Method
x 1 3 2
2 1
 x 66 1
x2 3 2
x 1
1 hence,
x 66 2 8
x
1
x 2
1 2x FG x 1 IJ
x 1
x 1
= e 3 2 j e 3 j
2 = 2 3.
Now, 2 = 8 + 2 = 10
x H x K 1
10. (4) ( x a )3
or –8 + 2 = –6 (x a )3
1
5. (4) x2 +
x2
6 FG
= x a
1 IJ 3
FG
3 x a
1 IJ
H x aK H x a K
= (x – a – x + b)3 + 3 (x – a – x + b)
F
= Gx
1IJ 2
2 6 = (b – a)3 + 3 (b – a)
H x K = 53 + 3 × 5
= 125 + 15 = 140
F
= Gx
1I
J
2

–4
H xK
11. (4)
Some Important Results

= Gx
F 1 IJ b2g2
2 2. a . b
H x K If x =
a b
F
= GH x
1 IF
2J G x
1
2
IJ x a x b
x KH x K then
x a x b
b
2 always g
required difference = 4
12. (4) we know that
5x 1
6. (4) = 1
2x 2 5x 1 3 If x + 2
x
Nr rFkk Dr dks 2x ls divide djus ij
1
5 then xn +
xn
b
2 always g
2 1
=
5 1 3 1
x 13. (2) x + =2
2 2x x

FG x 1 IJ 5 15 1 15 5 10 nksuksa rjiQ square djus ij]


x = =5
H 2x K 2 2 2x 2 2 2
x2 +
1
+2=4
x2
1 1
7. (1) 2x 6 x 3 [2 ls divide djus ij] 1
2x 4x x2 + =4–2=2
x2
nksuksa rjiQ square djus ij]
1
1 1 x+ =2
x2 2 x 9 x
16 x 2 4x

BLAM–53
chtxf.kr
nksuksa rjiQ cube djus ij] 9
15.(2) 2n 9
x
FG x 1 IJ 3

=8
H x K 2x 2 9
9
x3
1 FG
3 x
1 IJ =8
x
x3 H x K 2x2 – 9x + 9 = 0
(x – 3) (2x – 3) = 0
1
x3 + +3×2=8
x3 3
x = 3 or
2
1
x3 + =8–6=2 If x = 3 then we, get Max. Value.
x3
3
FG x 2 1 IJ FG x 3 1 IJ = 2 × 2 = 4 If x = then we get Min. Value.
H x2 KH x3 K 2

Aliter : 2 1
So, Minimum Value of x
1
x2
x =2
x
x2 – 2x + 1 = 0 =
FG 3 IJ 12
=
9 4
=
97
.
(x – 1)2 = 0
H 2K F 3 I 2 4 9 36
GH 2 JK
x=1

FG x 2 1 IJ FG x 3 1 IJ 16. (3) x +
1
= 1 (fn;k x;k gS)
H x2 KH x3 K x
= (1 + 1 ) (1 + 1) 1
x 3
=2×2=4 x2 3x 1 x
Expression = =
x 2
7x 1 1
1 x 7
14. (3) x2 + 1 x
x2
1 3 4 1
1 = = =
1 2 3 1 7 8 2
x+
x
1
cube djus ij 17. (1) x + =2
x
1
x3 3 3 3 3 x2 1
x3 =2
x
1 x2 + 1 = 2x
x3 + 3
0
x2 – 2x + 1 = 0
x
(x – 1)2 = 0
x6 + 1 = 0
x–1=0
Now, x=1
x48 + x42 + x36 + x30 + x24 + x18 + x12 + x6 + 1
1
= x42(x6 + 1) + x30 (x6 + 1) + x18 (x6 + 1) + x6 (x6 + 1) x12 + =1+1=2
x 12
+1
= 0+1=1 1 1
Aliter : (Based on a + or a – formulae)
a a
2nd Method
1
1 x =2
;fn x + = 3 or 1 rks Question esa ;fn Pair yxrk gS x
x
x=1
rks Answer 0 gksxk ;fn tks Pair ugha yxrk rks tks Remain-
der cspxk ogha Answer gksxk (x48 + x42) + (x36 + x30) + 1 1
x12 + =1 =2
(x24 + x18) + (x12 + x6) + 1 Answer is 1. x 12 1

BLAM–54
chtxf.kr
18. (2) we know that
1
(a – 3) – =2
If x
1
5 then x
5 1 ba – 3g
x x5
nksuksa rjiQ cube djus ij]
= (a3 – 3a) (a2 – 2) – a
so, here a = 5 |RS(a – 3) – 1 |UV 3

=8
x5
1
e5
3
3 5 52 je 2j 5
|T ba – 3g |W
x5
= (110 × 23) – 5 = 2525.
(a – 3) – G
3
F 1 IJ 3
FG 1 IJ
4
H a – 3K – 3 × (a – 3) H a – 3K
19. (3) 4a – =–3
a
FG ba – 3g – 1 IJ
nksuksa rjiQ 4 ls divide djus ij] H (a – 3) K
=8

1 –3 [  (a – b)3 = a3 – b3 – 3ab (a–b)]


a– =
a 4
(a – 3)3 –
FG 1 IJ 3
–3×2=8
a3 –
1
= a–
FG 1 IJ 3
1 FG a – 1 IJ H a – 3K
a 3
H a K +3a×
a H aK
(a – 3)3 – G
F 1 IJ 3
= 8 + 6 = 14
=
FG –3 IJ 3
+3×
–3
=–
27 9
– =
–27 – 144
=
–171 H a – 3K
H 4K 4 64 4 64 64 2nd Method
1 –171 –171 192 21 1
a3 – +3= +3 = = a 5
a3 64 64 64 a 3
2nd Method 1

If x
1
a then x
3 1
a3 3a
(a – 3) – a b g
3 =5–3=2
3
x x
1
(a – 3)3 – a = (2)3 + (3 × 2) = 8 + 6 = 14
a
1 3 b 3 g 3

a 4
1
a 3 1
3
LMFG 3 IJ 3

3
3 OP 3
27 9
3
22. (4) If x4 +
x4
a
a3 MNH 4 K 4 PQ 64 4
1
21
x–
x
= e a 2 j 2
=
64 1
and x – =a
1 x
20. (1) x + =1
x
1
x2 + 1 = x then x3 – = a3 + 3a
x3
x2 – x + 1 = 0
A/q
2
1
x2 – x 2 x4 + = 34
x4
2
= 1
2
x –x 1 1 x –
x
= e 34 2 j 2

2 = 6 2 2
= =2
0 1
1
x3 – = (2)3 + (3 × 2)
1 x3
21. (4) a – a – 3 = 5 b g = 8 + 6 = 14

BLAM–55
chtxf.kr
1 1
23. (2)  x + =0
x 7
= FG 3x 1 IJ 3 = 7
1 1

1
H 5x K 3 10
7
(x + 7) + b x 7g 1 3
27. (1) x + =
Now 4x 2

1
LMbx 7g 1 OP 2
R|bx 7g 1 U| 2
2x + =3
MN bx 7g PQ S| bx 7g V|W 4 2x
T nksuksa rjiQ cube djus ij]
[(a – b)2 = (a + b)2 – 4ab]
= (7)2 – 4 = 45 8x3 +
1 1
× 2x
FG 1 IJ
8x 3
+ 3 × 2x ×
2x H 2x K = 27
1
45 3 5 ...(1)
(x + 7) – b x 7g 8x3 +
1
+ 3 × 3 = 27
A/q 8x 3

1 LMbx 7g 1 OP 7
8x3 +
1
= 27 – 9 = 18
8x 3
x–
x 7
=
MN bx 7g PQ
x 1
= 3 5 7 [using (1)] 28. (2)
2x 2 5x 2 6
2x2 + 5x + 2 = 6x
F 1 IJ 2 = 4b 2
24. (1) G 2b
1 1
2x2 + 2 = 6x – 5x = x
H bK b2
2 2b
b
2 4 6
x
1 LM 1 O x2 + 1 =
2b 6 4b  2 2
2P 2
b N b Q 1 1
nksuksa rjiQ cube djus ij] x ls divide djus ij x+ =
x 2

8b 3
1
= 2b
FG 1 IJ 3 3 2b
1
2b
FG 1 IJ 29. (4) x
4 1
119
b 3 H b K b H b K x4
3
= e 6j 6 e 6j = 6 6 6 6 0 FG x 2 1 IJ 2
2 119
H x K 2

1 1
x3 x2
x2
F Nr vkSj Dr esa x lsI FG x 1 I
2
25. (3) 2
x –x
x
1 1 GH Divide djus ij JK H
2
x K
J
2
121
x 1
x
1
x2 11
FG x  1 IJ 2
x2
2
H xK 92 7
FG x 1IJ 2
= FG x  1 IJ 1
3 1 2
H K 2 11
H xK x

1 FG x 1I
J
2
9 x
1
3
26. (2) 3x
5x
7 H xK x
A/q nksuksa rjiQ cube djus ij]
5x FG x 1 IJ 3
27
15x 2 15x 1 H x K
dividing by 5x in Numerator & Denominator we
get, x3
1
3 x
FG 1 IJ 27
x3 H x K
BLAM–56
chtxf.kr
1
1 x 100 1 1 2
x3 3
3 3 27 x 100
x
1
32. (2) a 2 0
3 1 a
x 27 9 36
x 3 a2
+ 1 + 2a = 0
(a + 1)2 = 0
2nd Method
a+1=0
we know that a=–1
1 1 1 1
If x
4

x4
a x
x
ba 2g 2 a 37
a100
( 1)37
( 1)100
= –1 – 1 = –2
1 3 1
If x a then x a3 3a Aliter :
x x3 Put a = –1
A/q 1
a = –1 –1 = –2
1 a
x4 119
x4 1
(–1)37 – = –1–1 = –2
( 1)100
1
x–
x
= e b119 2g 2j =3
33. (1) x
1
3
x
1 nksuksa rjiQ square djus ij]
x3 = (3)3 + 3 × 3 = 27 + 9 = 36
x3
30. (2) Let, P = x + 5 FG x 1 IJ 2
9
x=P–5 H x K
Where,
1
x2 + x = 19 x2 =9–2=7
(P – 5)2 + (P – 5) = 19 x2
P2 + 25 – 10P + P – 5 = 19
P2 – 9P + 20 = 19 iqu%
FG x 1 IJ 3
27
P2 – 9P + 1 = 0 H x K
P2 + 1 = 9P
1 x3
1 FG
3 x
1 IJ 27
P+
P
9 x3 H x K
Put the value of P 1
x3 27 3 3 18
1 x3
(x + 5) + 9
(x 5) FG x 2 1 IJ FG x 3 1 IJ = 7 × 18
5)2
1 H x 2 KH x 3 K
Hence, ( x = 92 – 2 = 79
5)2
(x
x5
1 FG x 1 IJ 126
31. (1) x +
1
2
x5 H x K
x
1
x5 = 126 – 3 = 123
x2 1 2 x5
2 x 2x 1 0 0
x
2nd Method
(x – 1)2 = 0 x=1
1 1
1 If x a then x 5 = (a3 – 3a) (a2 – 2) – a
x100 + 1 1 2
x 100 x x5
Aliter : Here a = 3.
1 1
x 2 (x = 1 j[kus ij) x5 + = (33 – 3 × 3) (32 – 2) – 3
x x5
1 = (18 × 7) – 3 = 123
x = 1 +1 = 2
x

BLAM–57
chtxf.kr
1 2 1 37. (4) x2 + 1 = 2x (fn;k gS)
34. (3) x + =2 2x + =4 2x + =4
2x 2x x 1
x 2 ...(i)
nksuksa rjiQ cube djus ij] x
Expression
8x3+
1 1 FG 2x 1 IJ = 64
x 3 +3.2x.
x H x K 1 x6 1
x4
1 x 2 x2 x6 1
8x3+ 3 + 6 × 4 = 64 x 2
3x 1 (x 2
3x 1) (x 2
1 3x ). x 2
x

1 x6 1 x6 1
8x3+ 3 = 64 – 24 = 40 =
x (2 x 3 x )x 2
x3
1
35. (4) x
x
5 Fx 6
1 I Fx 6
1 I
= GH x 3 JK GH x 3
x3
JK
x2 – 5x + 1 = 0
x2 – 3x + 1 = 2x
F
= Gx 3 1 IJ
x 4 1 Fx 4 1 I H x3 K
x2 1G x2 JJ = 1 FG x 3 1 IJ
x 2
3x 1
G
2G x JK 2 H x3 K LF 1 IJ 3
FG 1 IJ OP
H = – MGH x K 3 x
H K PQ
MN x x

1 LMF x 1 IJ 3
FG 1 IJ OP = – [23 – 3 × 2] = – 2
=
2 MNGH x K 3 x
H x K PQ Aliter :
x2 + 1 = 2x
1 1
= (125 3 5) = 110 55 1
2 2 x+ =2
x
1 x2 + 1 = 2x
36. (1) x 5
x2 – 2x + 1 = 0
x
(x – 1)2 = 0
nksuksa rjiQ square djus ij]
x 1
1
x2 2 25
x2 x4 
1
x2 = 1 1
1 x 2  3x  1 1 3 1
x2 = 25 – 2 = 23 ...(i)
x2
2
= = –2
x 4
3x 3
5x 3x 1 2 1
Expression = 4
x 1 FG x 1IJ 3
1 FG 1 IJ FG 1 IJ
3
38. (2) = x 3 x
H K = 3 xH K
x 4
1 3x 3x 5x 3 2 H x K x 3 x x
=
x4 1 FG x 1I
2
LMx 1 OP
H J
xK
=3
N 0
Q
x2 x2
FG 1 IJ 3x 2 x
FG 1 IJ 5x 2
x
H x2 K H x K FG x 1I
4
= FG x IJ
x 2
H
2 1
K H J
xK
=3×3=9
x2
2 1
FG x 1 IJ 3 FG x 1 IJ 39. (3) x 83
2
5 x2
=
H x2 K H x K =
23 3 5 5
=
43
x 2 1 23 23 FG x 1 IJ 2
2 83
x2 H x K
BLAM–58
chtxf.kr
1
FG x 1 IJ 2
83 2 81 92
x3
x3
2 3 2 3 4
H x K
43. (2) x = 3 2 2
1
x 9
x 1 1 3 2 2
=
nksuksa rjiQ cube djus ij] x 3 2 2 e3 je
2 2 3 2 2 j
FG x 1 IJ 3
93 = 729 3 2 2
H x K =
9 8
= 3 2 2

x3
1
3 x
FG 1 IJ 729 FG IJ
1
2
1
x 3 H x K H x
xK
x
x
2

1
x3 3 9 729 = e3 2 j e3
x3
2 2 2 j 2 4

1 1
x3 729 27 756 x 2
x3 x
2nd Method 44. (3) x 3 5
1 nksuksa rjiQ square djus ij]
x2 83
x2 x= 3 5 2 15
1 x 8 2 15
x 83 2 9
x nksckjk square djus ij]
1 x2 – 16x + 64 = 60
x3
x3
b9g b3 9g
3
756 . x2 – 16x + 4 = 0
x2 – 16x + 6 = 2
1 5
40. (1) x
4x 2 F 1I 2
1
Multipling by 2 in both sides
45. (2) GH x J
xK
= x
x
2=2+2=4

1
2x 5 1
2x x 4 r2
x
64 x 6 1 1 fn, x, option ls]
A/q 8x 3 = 53 – 3 × 5 = 110
8x 3 8x 3 1
x  2
1 1 x
[If x = a then x 3 a3 3a ]
x x3 1
46. (3) 2x = a +
2 a. b a
41. (2) If x = nksuksa rjiQ square djus ij]
a b
1
x a x b 4x2 = a + +2
then 2 (always) a
x a x b
1 1
4x2 – 4 = a + +2–4 =a+ –2
42. (4) x = 3 a a
2 3

x3 = 2 3 FG 1 IJ 2
1
a –
1 1 1 2 3
4x 2
–4 = H aK
= a – a
3 =
x 2 3 2 3 2 3
1 FG a –
1 I
J
2 3 x2 – I = 2 H aK
= 2 3
4 3

BLAM–59
chtxf.kr
2 2
x2 –1 e3x y 2y x j e3x y 2y x j
Expression = xy
x– x2 –1 2 2
e3x y j e2y x j
1 FG a –
IJ 1

=
2 H aK xy
4 3x y 2y x

1 FG 1 I 1 F 1 II
9x 2y 4y 2 x
2 H a J – G a–
a K 2H
JJ
a KK
24 xy xy
xy
1 FG 1 IJ xy 9x b 4y g
2 H a – F
a K =1 aG a –
1 IJ 1
b g
a –1
9x – 4y = 24
= =
1 2 H a K 2 4
a b 2 3 d 6
a 50. (3) 1
e 2 j 3
47. (4) Given
a3 + b3 = 20 and a + b = 5
4 e1 2j 3
a+b=5
Cubing Both Sides e1 2 j 3 e1 2j 3
a3 + b3 + 3ab (a + b) = 125
20 + 3ab × 5 = 125 = a b 2 3 d 6

125 20
ab =
15
=7 41 e j 2 3
a b 2 3 d 6
2 2

Again, a + b = 5 e1 2j e 3j
Squaring Both Sides
a2 + b2 + 2ab = 25
a2 + b2 + (2 × 7) = 25
41 e 2 3 j a b 2 3 d 6
a2 + b2 = 11 2 2
Squaring Both Sides
2 2 6 a b 2 c 3 d 6
a4 + b4 + 2a2b2 = 121
a2 + b2 + (2 × 7) = 25 Compare djus ij
a2 + b2 = 11 Now,
Squaring Both Sides a+b+c=d=2+1+0+1=4
a4 + b4 + 2a2b2 = 121 51. (1) a2 + b2 + c2 + 216 = 12 (a + b – 2c)
a4 + b4 + 2 × (7)2 = 121 (a2 – 12a + 36) + (b2 – 12b + 3b) + (c2 – 24c + 144)
a4 + b4 = 121 – 98 =0
a4 + b4 = 23. (a – 6)2 + (b – 6)2 + (c – 12)2 = 0
48. (2) a + b + c + d = 2 (a – 6)2 + (b – 6)2 + (c – 12)2 = 0
Now,
1
Putting a = b = c = d =
2 ab bc ca
Max Values (1 + a) (1 + b) (1+ c) (1 + d)
= b6 6 g b6 12 g b12 6g
F 1 IJ FG1 1 IJ FG1 1 IJ FG1 1 IJ
= GH1
2K H 2K H 2K H 2K = 36 72 72 = 36 6
52. (1) Given
L 3 O 81 .
= M P
4

1
N 2 Q 16 x4
x4
194
49. (1) Given
1
3x y 2y x 3x y 2y x
x
x
= e 194 2 2 = j 14 2 16 4
x y
3x y 2y x 3x y 2y x Note:
We know that

BLAM–60
chtxf.kr
1 27
If x4 + a x3
x4 = x3 1 3
x
FG 27 IJ
...(1)
1
6 3 3 H x3 K
then x
x
e a 2 j 2 Given,

53. (*) Given the expression 3


x 6
x2 – 3x + 1 = 0 x
dividing By x Cubing Both Sides

x
1
3 x3 –
27 FG
9 x
3 IJ 216
x x3 H x K
Now, 27
1
x3 –
x3
b9 6 g 216
x4
x2 27
x2 1 x3 – 270
x3
1 from eqn (1)
x3
=
x3
1
1 3
x
FG 27 IJ 1
270 90
x
x
3 H x3 K 3

[Dividing Box x in Numerator & Denominator] Type-III


b3g 3
3 3 18 Factor Theorem and Questions
= = 6 Based on Other Formulae
3 3
54. (2) Given the expression,
1 1
1. (1) x a ,y= a
x 3
FG 2 IJ 3 a a
H x K x
2
x y 2 a; x y
3 a
3x – 2 =
x
2

FG 1 IJ x4 y4 2x 2y 2 = x
2
e y2 j
3 x H K 2
x
= (x y )( x y)
2 FG 2 a
2IJ 2

= (4)2 = 16
x–
1 2 H aK
x 3
2. (4) P(x) = x2 + 3Qx – 2Q
Now,
 (x – 2), P(x) dk factor gSA
x3
1 FG 2 IJ 3

3
2 P(2) = 0
x3 H 3K 3 (2)2 + 3Q × 2 – 2Q = 0
4 + 6Q – 2Q = 0
8 62
= 2 4Q + 4 = 0
27 27
4Q = – 4 Q = –1
55. (4) A/q,
1
27 3. (3)  x – 1 x2 1 x
x 4 x
x2
2
x 3x 3 FG x 1 IJ FG x
2
1 IJ 2
4
3 27
H x K H x K
x
x3 = (1)2 + 4 = 5
= FG x 3 IJ 3
[dividing By x]
1
H x K x+
x
5

BLAM–61
chtxf.kr
x2 1
5
FG3p 1 IJ 3
FG 3 5 1 IJ 3
FG 5 1 IJ FG 4 IJ
3 3

x H 6K H 18 6 K H 6 6K H 6K
x2 + 1 = 5x FG 2 IJ 8 3

Now, H 3 K 27
1 LM
1 1 1 1 O
N
x x 1 x 1 x 1 x 1 PQ 2 2 6. (1) a e2 3j b 2 e 3 j 1

L
1 Mx 1 x 1 x 1 x 1P
O 2 2
a b 1 1
1
2 3 2 3 e2  3 je2  3 j 43
= xM x 12
e x 1jex 1j PPQ 2 2
NM 3
a = 2 + 3 and b = 2 –

1M 2
L 2
OP a2 – b2 = (a + b) (a – b)
= x Mx 2 P e j e2 j
NM 1 e x 1jex 1j PQ
2 2 = 2 3 2 3 3 2 3

=4× 2 3 = 8 3
L
1 M 2e x 1 1j O
2
PP 7. (4) x = 3 2 2
= x M ex 2
je j 2
NM 1 x 1 PQ 1 1 3 2 2
xy = 1 y= =
2 3 2 2 3 2 2 3 2 2
1 2x 2
=
x x 5x 5x
3 2 2
4. (3) Given the expression, = 3 2 2
9 8
3x2 – 4x + 1 = 0
(3x – 1) (x – 1) = 0 x+y= 3 2 2 3 2 2 6

1 x2 3xy y2 (x y )2 xy 36 1 37
x = 1, = =
3 2 2
x 3xy y (x y )2 5xy 36 5 31
If x = 1 then expression
Px3 – 8x2 – qx + 1
10 2 21
P–8–q+1=0 8. (2) x 5 21 x
p – q = 7 ...(1) 2

1
If x = then expression 7 3 2 7 3 7 3
3 = =
2 2
Px3 – 8x2 – qx + 1
P 8 q
27 9 3
1 0 32 2x 32 25 e 21 j
P 9q 8 1 = 32 10 2 21
1
27 9 9
P – 9q = – 3...(2) 22 2 21 = 21 1 2 21 1
from eqn (1) & (2) we get = 21 1
33 5 Expression
P= ,q=
4 4
7 3
1 9 2 1 =
5. (3) 27 p3 –
216

2
p +
4
p 2 e 21 1 21 j
b3p g FGH 16 IJK b g FGH 16 IJK
3 1
1 1
3
3. 3p .
2
3 3p
6 6
=
2
e 7 3 j

BLAM–62
chtxf.kr
9. (2) a = 7 x3 – 6x2 + 12x – 8 = 4 – 2 – 6 (x – 2)
4 3
x3 – 6x2 + 12x – 8 = 2 – 6x + 12
1 1 1 7 4 3 x3 – 6x2 + 18x + 18 = 2 + 12 + 8 + 18 = 40
= = 7 4 3
a 7 4 3 7 4 3 7 4 3 14. (1) Question ls , x – 1 = 2 3
nksuksa rjiQ square djus ij]
F 1I2 1
GH a J
aK
a
a
2 x2 – 2x + 1 = 2 + 3 + 2 6
x2 – 2x – 4 = 2 6
1
= 7 4 3 7 4 3 2 = 16 a 4 nksckjk square djus ij]
a
x4 + 4x2 + 16 – 4x3 – 8x2 + 16x = 24
5 1 5 1 5 1 x4 – 4x3 – 4x2 + 16x – 8 = 0
10. (1) x
5 –1 5 –1 5 1 2x4 – 8x3 – 8x2 + 32x – 16 = 0
2x4 – 8x3 – 5x2 + 26x – 28 – 3x2 + 6x + 12 = 0
2
2x4 – 8x3 – 5x2 + 26x– 28 = 3x2 – 6x – 12
e 5 1 j 5 1
5 –1 2 = 3 (x2 – 2x – 4) = 3 2 6 6 6
1 1
5 1 2 5 5 1 15. (3) –1
x2 + x – 1 1 x 99 ( 1)99
4 2
1 1
1
F 5 1 I 2
5 1 x 98 ( 1)98
GH 2 JK 2
1
Expression
=–1+1–1+1–1+1–1–1=–2
3 5 5 1 3 5 5 1 2
1 16. (2) x4 – 17x3 + 17x2 – 17x + 17
2 2 2 = x4–16x3 + 16x2 – 16x – x3 + x2 – x + 17
= 5 +1 tc] x = 16,
11. (4) x 3 Expression
5 2
= 164 – 164 + 163 – 162 – 163 + 162 – 16 + 17 = 1
3
x 2 5
nksuksa rjiQ cube djus ij] ex x2 1 j x x2 1
17. (1) 62
x3 3x 2 2 3x .( 2)2 23 5 x x 2
1 x x 2
1
x3 – 6x2 + 12x – 8 = 5
x3 – 6x2 + 12x – 13 = 0
Fx x2 1
I Fx x 2
2
1
I 2
12. (4) a2 + 1 = a
a2 – a + 1 = 0
H K H K = 62
bx g FH x 1IK
2
2 2
(a + 1) (a2 – a + 1) = 0
[(a + 1) ls multiply djus ij)
a3 + 1 = 0
a3 = – 1 e
2 x2 x2 1 j 62
2 2
a12 + a6 + 1 = (a3)4 + (a3)2 + 1 = 1 + 1 + 1 = 3 x x 1
1 2 2x2 – 1 = 31
13. (3) x = 2 – 2 3 23 x2 = 16
2 1 x= 4
x – 2 = 23 23 x < 0 (given)
nksuksa rjiQ cube djus ij] x = – 4.
x3 – 3x2 × 2 + 3x × 4 – 8 18. (3) x2 – 3 x + 1 = 0
x2 +1 = 3 x
F 2 3 I F 1 3 I 2 1 F 2 1 I x2 1
= GG 2 3 JJ GG 2 3 JJ – 3. 2 3 . 2 3 2 3 GG 23 JJ =3
H K H K H K x

BLAM–63
chtxf.kr
1 24. (3) Expression = 2 – 3x – 4x2 = – (4x2 + 3x – 2)
x+ =3 ...(i)
x LM 3 FG 3 IJ 2 FG 3 IJ 2 OP
= – (2x )2 2 2x H 4K H 4K 2
MN 4 PQ
x6 x4 x2 1 x6 x4 x2 1
= 3 + 3 + +
3
x3 x3 LMF
x x x
3 IJ 2 OP FG 3 IJ 2
1 1 FG x3 1 IJ + FG x 1 IJ
=–
MNGH 2x 4 K PQ H 4 K 2
= x3 + x + + =
x x3 H x K3 H xK expression dh value maximum gksxh ;fn
FG x 1 IJ 3
1 FG x 1I
J + FGH x 1x IJK 2x +
3
=0
=
H x K – 3. x .
x H xK 4

= 33 – 3 × 3 + 3 = 27 – 9 + 3 = 21 3
2x = –
4
1 1 3
19. (2) x 2 x 1 = x2 2. x .
2 4 4 3
x=–
8
F 1 IJ FG
= GH x
2
3 I
JK
2
Aliter :
2K H 2 O;atd 2 – 3x – 4x2
f (x) = –4x2 – 3x + 2
Differentiate djus ij]
FG x 1 IJ FG 2
3I
2
FG x 1 IJ 2

2 JK
q2
H 2K H =
H 2K d f (x )
= –8x – 3
dx

3 –8x – 3 = 0
q –8x = 3
2
20. (4) ax + by = 3 ...(i) 3
x
bx – ay = 4 ...(ii) 8
nksuksa equations dks square rFkk tksMu+ s ij 3
a2x2 + b2y2 + b2x2 + a2y2 + 2abxy – 2abxy = 9 + 16 8
ijx= O;atd dk eku Maximum gksxkA
a2x2 + b2x2 + a2y2 + b2y2 = 25
25. (2) Given
x2(a2 + b2) + y2(a2 + b2) = 25
(a2 + b2)(x2 + y2) = 25 FG x 32 1 IJ FG x 8 1 IJ FG x 1 IJ FG x 16 1 IJ
a2 + b2 = 25 H x 32 KH x8 KH x KH x 16 K
x 1 1 x x a
21. (4) FG x 1 IJ FG x 4 1 IJ
a a x a ax H x KH x4 K
x2 x a x x2 a
22. (2) ax + bx + c perfect square gksxk, ;fn b2 = 4ac
2
FG x 2 1 IJ
Multipling dividing by H x2 K
x2 + ax + b perfect square gksxk ;fn a2 = 4b
d FG x 2 1 IJ FG x 1 IJ FG x 1 IJ FG x 4 1 IJ FG x 8 1 IJ FG x 16 1 IJ FG x 32 1 IJ
23. (2) a b H x2 KH x KH x KH x4 KH x8 KH x 16 KH x4 K
c FG x 2 1 IJ
c 1 a b H x2 K
a b 1
d a b a b
FG x 4 1 IJ FG x 4 1 IJ FG x 8 1 IJ FG x 16 1 IJ FG x 32 1 IJ
(Componendo rFkk dividendo osQ }kjk) H x4 KH x4 KH x8 KH x 16 KH x 32 K
=
c d a b a b a FG x 2 1 IJ
c d a b a b
=
b
H x2 K
1 a
(c – d) =
b FG x 8 1 IJ FG x 8 1 IJ FG x 16 1 IJ FG x 32 1 IJ
c d b a
=
H x8 KH x8 KH x 16 KH x 32 K
c2 – d2 = (c + d) (c – d) =
b FG x 2 1 IJ
a H x2 K
BLAM–64
chtxf.kr
...(ii)
FG x 16 1 IJ FG x 32 1 IJ x 64
1 (i) rFkk (ii) dks multiply djus ij
H x 16 KH x 32 K x 64
= FG x 2 1 IJ x2
1 abc
b 1
1
H x2 K x2 1 b
30. (4) a2 – 4a – 1 = 0 a2 – 1 = 4a
26. (2) a2 + b2 + c2 = ab + bc + ca
2a2 + 2b2 + 2c2 – 2ab – 2bc – 2ca = 0
nksuksa rjiQ a ls divide djus ij
a2 + b2 – 2ab + b2 + c2 – 2bc + c2 + a2 – 2ca = 0 2 1
a 1 4a
a– =4 ...(i)
(a – b)2 + (b – c)2 + (c – a)2 = 0 a a a
a – b = 0, b – c = 0, c – a = 0
1 3
a = b, b = c, c = a Expression = a2 + 3a +
a=b=c a2 a

a c 2a
= a2 +
1
3a
1 3 FG
= a
2 3 a
IJ 2
FG 1 IJ
b
=
a
=2
a 2
a a H K H a K
Aliter : = (4)2 + 2 + 3 × 4 = 16 + 2 + 12 = 30
Let we take x a b
a=b=c=1 31. (1) =
1 a b
a2 + b2 + c2 = ab + bc + ca
12 + 12 + 12 = 1 × 1 + 1 × 1 + 1 × 1 a b
3=3 1 b
1 x a b a b a b
= a b =a =
a c 1 1 1 x b a b a
1
=2 a b
b 1
27. (4) 3a2 + 3b2 + 3c2 = (a + b + c)2 blh izdkj]
3a2 + 3b2 + 3c2 = a2 + b2 + c2 + 2ab + 2bc + 2ac
1 y c 1 z a
3a2 + 3b2 + 3c2 – a2 – b2 – c2 – 2ab – 2bc – 2ac =
1 y = b ; 1 z = c
0
2a2 + 2b2 + 2c2 – 2ab – 2bc – 2ac = 0 (1 x )(1 y )(1 z )
a2 + b2 – 2ab + b2 + c2 – 2bc + a2 + c2 – 2ac = 0 Expression = (1 x )(1 y )(1 z )
(a – b)2 + (b – c)2 + (c – a)2 = 0
a–b=0 a=b b c a
b–c=0 b=c = × × =1
a b c
c–a=0 c=a
a=b=c a b
32. (4) =2
b a
1 1 1 1 1 1
28. (4) 1 1 1
2 2
3 2
3 2
4 2
4 2
52 a 2 b2
=2
ab
49 169 441 a2 + b2 = 2ab
=
36 144 400 a2 + b2 – 2ab = 0
(a – b)2 = 0
7 13 21
= a–b=0
6 12 20 Aliter :
198 33 Let we take
= a=b=1
60 10
a b 1 1
1 2 2
29. (1) a 1 ab + 1 = b ab = b – 1 ...(i) b a 1 1
b
a–b=1–1=0
iqu%,
1
1 33. (3) a + =1
b 1 b
c
1
1 1 a= ;b=2
1 b c 2
c 1 b

BLAM–65
chtxf.kr
1 3a + b + 2c = –1 + 1 + 3 = 3
b+ =1 39. (3) (x – 3)2 + (y – 5)2 + (z – 4)2 = 0
c
x–3=0 x=3
b = 2, c = –1 y–5=0 y=5
1 z–4=0 z=4
c+ = –1 + 2 = 1
a
x2 y2 z2
34. (3) a3 + b3 + c3 – 3abc = 0 + +
9 25 16
(a + b + c) (a2 + b2 + c2 – ab – bc – ac) = 0
9 25 16
1 = + + = 1 + 1 +1 = 3
(a + b + c) (2a2 + 2b2 + 2c2 – 2ab–2bc –2ac) = 0 9 25 16
2
40. (2) (a – 3)2 + (b – 4)2 + (c – 9)2 = 0
1 a–3=0 a=3
(a + b + c) [(a – b)2 + (b – c)2 + (c – a)2] = 0 b–4=0 b=4
2
a b c 0
,oa c – 9 = 0 c=9
(a – b)2 + (b – c)2 + (c – a)2 = 0 a b c
a–b=0 a=b
= 3 4 9 = 16 = + 4
b–c=0 b=c
c–a=0 c=a x z
y
[If x2 + y2 + z2 = 0, then x = 0, y = 0, z = 0] 41. (4) xa + yb + zc = = za xb yc
ya zb xc
a=b=c
x y z
1 =
35. (3) xy (x + y) =1 x + y = xy xa yb zc ya zb xc za xb yc

nksuksa rjiQ cube djus ij, =


x y z
xa ya za yb zc zb xc xb yc
1
x3 + y3 + 3xy (x + y) =
x 3y 3 x y z
= a (x y z ) b(x y z ) c ( x y z)
1 1
3 3
x + y + 3xy × xy = 3 3 1
x y x y z
= =
(a b c )( x y z ) a b c
1
– x3 – y3 = 3 42. (1) a + b + c + d = 4 (fn;k gS)
x 3y 3
1 1
Expression = (1  a )(1  b )(1  c )  (1  b )(1  c )(1  d )
36. (4)
ea 2
b 2
jba b g ba b g 3

a 2b ab 2 1

1
+
(1  c ) (1  d )(1  a ) (1  d ) (1  a )(1  b )
ba b gea 2
b2 a2 b2 2ab j 1 d 1 a 1 b 1 c
=
ab ab b g = (1 a )(1 b )(1 c )(1 d )

2ab 4 (a b c d )
= 2
ab = (1 a )(1 b )(1 c )(1 d )

7 5 7 5 4 4
37. (2) N = = =0
7 5 7 5 (1 a )(1 b )(1 c )(1 d )
= 6 35 Aliter :
Here, a + b + c + d = 4
1 Let a = 0, b = 0, c = 2, d= 2
6 35
N
1 1
38. (1) (3a + 1)2 + (b – 1)2 + (2c–3)2 = 0
3a + 1 = 0 3a = –1
b1 a gb1 bgb1 c g + b1 b gb1 c gb1 d g +
b–1=0 b=1 1
2c – 3 = 0 2c = 3 b1 c gb1 d gb1 a g
BLAM–66
chtxf.kr
1 1 1 1 1
= dk eku least rHkh gksxk] tc a, b rFkk c dh
b1 0gb1 0gb1 2g + b1 0gb1 2gb1 2g + 46. (1)
a b c
values maximum gksxhA
1 1
+
(1  2)(1  2)(1  0) 1 2 1 0 1 0 b gb gb g a=b=c=
1
3
=–1+1+1–1=0
43. (1) Given, 1 1 1
a+b+c=3 a b c
Now, =3+3+3=9
1 1 1 p q r
47. (3) ekuk a = x, b = y, c = z
b1 a gb1 b g b1 b gb1 c g b1 c gb1 a g
x+y+z=1

=
b1 a g b1 b g b1 c g 1 1 1
b1 a gb1 b gb1 c g vkSj x + y + z = 0

3 ba b c g 3 3 yz xz xy
b gb gb g b1 a gb1 b gb1 c g 0
= 1 a 1 b 1 c xyz =0

xy + yz + zx = 0
44. (1) ¦ q r
( p  q )(q  r )
x+y+z=1
nksuksa rjiQ square djus ij]
q r (r  p ) ( p  q) x2 + y2 + z2 + 2xy + 2yz + 2zx = 1
= + +
( p  q )(q  r ) (q  r )( p  q ) (r  p )(r  q ) x2 + y2 + z2 + 0 = 1
x2 + y2 + z2 = 1
(q  r )(q  r )  (r  p )(r  p )  ( p  q )( p  q )
= p2 q2 r 2
( p  q )(q  r )(r  p )   =1
a 2 b2 c2
q 2  r 2  r 2  p 2  p2  q2
= =0 1 1 2ax
( p  q )(q  r )(r  p ) 48. (4) + 4
a 2
ax x 2
a 2
ax x 2
a a 2x 2 x4
b c a c a b
45. (3) 1 a2 ax x2 a2 ax x2 2ax
a b c = 2 2 2 +
(a ax x )(a ax x2) a4 a 2x 2 x4
b c a b a c
1 0 2ax 2ax
a c b = + 4 =0
a 4
a x 2 2
x 4
a a 2x 2 x4
b c a b a c b
0 a b –c b c –a c a –b
a c b 49. (2) = =
a b b c c a
c b b a a c b
a b c b c a c a b
a c b – = – = –
a b a b b c b c c a c a
c2 bc ab a2 a c b a b
c
ac b 1– =1– = 1–
a b b c c a
2 2
(c a ) (bc ab ) a c b c a b
ac b
= =
a b b c c a
(c a )(c a ) b (c a) a c b a b b c c a
=
ac b c a b
(c a )(c a b) a c b a b b c c a
1 1 1
ac b c a b
c a 1 c a 1 1 1 1 a b c b c a c a b
ac ac b a c b = =
ac b c a b

BLAM–67
chtxf.kr
1 1 1 53. (2) Expression
= = = 2b2c2 + 2c2a2 + 2a2b2 – a4 – b4 – c4
c a b
= 4b2c2 – (2b2c2 – 2c2a2 – 2a2b2 + a4 + b4 + c4)
a=b=c
= (2bc)2 – (a2 – b2 – c2)2
50. (4) fn;k x;k gS] bc + ab + ca = abc
= (2bc + a2 – b2 – c2) (2bc – a2 + b2 + c2)
bc + ab = abc – ac
= (a2 – (b2 + c2 – 2bc)) (b2 + c2 + 2bc – a2)
ab + ca = abc – bc
bc + ca = abc – ab = (a2 – (b – c)2) ((b + c)2 – a2)
Expression = (a – b + c) (a + b – c) (a + b + c) (b + c – a)
;fn a + b – c = 0,
b c a c a b
= + + Expression = 0.
abc – bc abc – ac abc – ab

=
b c
+
a c
+
a b
54. (3)
ea 2
b2 ajb b g ea 3
b3 j
ab ac bc ab bc ca a 2b ab 2
b c a c a b 1 1 1
= + + = + +
b
a b c g b
b c a g b
c a b g a b c
=
ea 2
b2 a jb b g ba b gea 2
b2 ab j
ab ba b g
bc ac ab abc
= = =1
abc abc
2 2 2
ba b gea 2
b2 a2 b2 ab j
a – bc b – ca c – ab =
51. (4)
a2 bc
+
b2 ca
+
c2 ab
=1 ab a b b g
ab
Fa 2
– bc I Fb 2
– ca I Fc 2
– ab I = 1
GH a 2
bc
1J + G
K Hb 2
ca
1J + G
K Hc 2
ab
1J = 4
K
ab
55. (2) a + b + c = 0
2 2 2 2
(a + c) = – b
a – bc a bc 2 2
b – ca b ca c – ab c ab
+ + =4 nksuksa rjiQ square djus ij]
a2 bc b2 ca c2 ab
a2 + c2 + 2ac = b2
2a 2b 2c 2 a2 + c2 = b2 – 2ac
2 + 2 + 2 =4
a bc b ca c ab a2 b2 c2 b2 b2 2ac
2 2
b ca b ca
a2 b2 c2 4
2 + 2 + 2 = =2 2(b 2 ac )
a bc b ca c ab 2 2
b2 ac
3 – 5x 3 – 5y 3 – 5z 56. (3) Expression,
52. (3) + + =0
2x 2y 2z

3 5x 3 5y 3 5z
bs a g bs b g bs c g
2 2 2
s2
– + – + – =0 2 2 2
2y a b c
2x 2x 2y 2z 2z

3 5 s2 2sa a2 s2 b2 2sb s2 2sc c2 s2


3 3 5 5 =
+ + – – – =0 a 2
b 2
c 2
2x 2y 2z 2 2 2

3 3 3 3 5
4s 2 a2 b2 c2 2s a b b c g
+ + = 2
2x 2y 2z 2 a b2 c2

1 1 1 5 4s 2 a2 b2 c2 4s 2
+ + = 1
2x 2y 2z 2 a2 b2 c2

4 4 4 4 5 2nd Method
+ + =
2x 2y 2z 2 putting S = O

2 2 2 a2 b2 c2
+ + = 10 1
2 2
x y z a b c2

BLAM–68
chtxf.kr
61. (2) a + b + c = 3; a2 + b2 + c2 = 6
a2 b2 c2 (a + b + c)2 = a2 + b2 + c2 + 2ab + 2bc + 2ca
57. (2)
bc ca ab 32 = 6 + 2 (ab + bc + ca)
9 – 6 = 2(ab + bc + ca)
a3 b3 c 3 3 abc
= = 3 3
abc abc ab + bc + ca =
2
[tc x + y + z = 0 ; x3 + y3 + z3 = 3xyz]
58. (4) (x + 1) (x2 + 1) (x4 + 1) (x8 + 1) (x16 + 1) (x64 + 1) 1 1 1
=1
(x32 + 1) (x128 + 1) a b c
Multipling & dividing By (x – 1) bc ac ab
=1
abc
ex 2
je
1 x2 je
1 x4 je
1 x8 je
1 x 16 je
1 x 64 je
1 x 32 je
1 x 128 1 j 3
x 1
abc = ab + bc + ca =
2
= 62. (1) x (x + y + z) = 20
x2 + xy + xz = 20 --- (i)
ex 4
1 x je 4
1 x je 8
je
1 x 16
je
1 x 32
je
1 x 64
je
1 x 128
1 j iqu%, y (x + y + z) = 30
x 1
xy + y2 + yz = 30 --- (ii)
,oa, z (x + y + z) = 50
=
ex 8
1 x8je je
1 x 16 je
1 x 32 je
1 x 64 je
1 x 128 j
1 xz + yz + z2 = 50 --- (iii)
x 1 rhuksa equations dks tksM+us ij,
x2 + y2 + z2 + 2xy + 2yz + 2zx = 20 + 30 + 50

=
ex 16
je
1 x 16 je
1 x 32 1 x 64 je je
1 x 128 1 j (x + y + z)2 = 100
x 1 x + y + z = 10
2 (x + y + z) = 20

=
ex 32
1 x je 32
1 x je 64
je
1 x 128 1j 63. (4) (2a – 1)2 + (4b – 3)2 + (4c + 5)2 = 0
x 1 2a – 1 = 0
2a = 1

=
ex 64
je
1 x 64 je
1 x 128 1 j 1
a=
x 1 2
4b – 3 = 0
=
ex 128
je
1 x 128 1 j 4b = 3
x 1
3
b=
x 256 1 4
=
x 1 4c + 5 = 0
4c = –5
1 1 1
59. (1)  a2 + b2 + c2 + 6 –5
a2 b2 c2 c=
4
putting a = b = c = 1
hence [ ;fn x2 + y2 + z2 = 0 gks rks x = 0 ] y = 0, z = 0]
a2 + b2 + c2 = 12 + 12 + 12 = 3 1 3 5 2 3–5
60. (3) a2 + b2 + c2 = 2a – 2b – 2 a+b+c= – = =0
2 4 4 4
a2 + b2 + c2 – 2a + 2b + 2 = 0
a2 – 2a + 1 + b2 + 2b + 1 + c2 = 0 a3 + b3 + c3 – 3abc = 0
(a – 1)2 + (b + 1)2 + c2 = 0 64. (4) Expression
a–1=0 (a b )2 (b c )2 (c a )2
a = 1; = +
(b c )(c a ) (c a )(a b ) (a b )(b c )
b+1=0
b = –1; and c = 0 (a b )3 (b c )3 (c a )3
3a – 2b + c =
(a b )(b c )(c a )
= 3 × 1 – 2 (–1) + 0
=3+2=5 3(a b )(b c )(c a )
= =3
(a b )(b c )(c a )

BLAM–69
chtxf.kr
[;gk¡, a – b + b – c + c – a = 0. ;fn x + y + z = 0, x3 + 1 c
y3 + z3 = 3xyz] c 1 a b c
2 2 2
m a m b m c 1 1 1
65. (3) + + –3=0
b2 c2 c2 a2 a2 b2 a 1 b 1 c 1

a b c a b c
m a2 m b2 m c2 = = 1
2 2 –1+ 2 2 – 1+ 2 2 –1=0 a b c a b c a b c a b c
b c c a a b
x a x b b a
m a2 b2 c2 m b2 c2 a2 67. (2)
+ + x b x a x x
b2 c2 c2 a2
(x a )( x a) (x b )( x b) b a
m c2 a2 b2 (x b )(x a) (x a )(x b) =
=0 x
a2 b2
x a x b b a
2 2 2 2 2 2 =
m (a b c ) m (a b c ) (x b)(x a ) (x a)(x b) x
+ +
b2 c2 c2 a2
x a x b b a
=
m (a 2
b 2
c ) 2 (x b )( x a) x
2 2 =0
a b
b a b a
izR;sd term = 0 (x b )( x a) x
m (a 2 b2 c 2)
2 2 =0 x= (x b )( x a)
b c
m – (a2 + b2 + c2) = 0 Square djus ij,
m = a2 + b2 + c2 x2 = (x – b) (x – a)
x2 = x2 – ax – bx + ab
2nd Method
ax + bx = ab
using option (3)
x (a + b) = ab
putting m = a2 + b2 + c2 in expression
ab
x=
L.H.S. =
ea 2
b2 c2 j a2 ea 2
b2 c2 j b2
+
a b
b2 c2 c2 a2 3 3
68. (3) x a a2 b3 + a a2 b3
ea 2
b 2
c 2
j c 2
3
nksuksa rjiQ cubing djus ij
2 2
a b
F3 2 3 I 3 F3 a 2 3 I3
= 1 + 1 + 1 –3 = 0 L.H.S. = R.H.S. x 3
= G a a b JK GH a b JK
H
2
66. (3) a b c
F 3
a2 b3
I
a2 a a b c + 3 GH a JK
a (a 1) a b c
F3 a 2 3 I F3 a 2 3 3 2 3 I
(a 1)
a b c GH a b JK GH a b a a b JK
a

1 a 1
(a 1) a b c Fa 2 3 I3
3G JJ x
a b
=a 2 3 2 3
blh izdkj,
a b a a b GH a a
2
b
3
K
b2 = c + a
1 b 1

b 1 a b c
,oa c2 = a + b = 2a 3 a2e a2 b3 j 3x

BLAM–70
chtxf.kr
= 2a + (– 3bx) 73. (4) x3 + 6x2 + 12x = 19
x3 + 3bx = 2a Adding Both sides by 8
69. (3) ab (a + b) = 1 x3 + 6x2 + 12x + 8 = 27
1 (x + 2)3 = (3)3
a+b= x+2=3
ab
x=1
Cube djus ij x3 = 13 = 1
1 2
a3 + b3 + 3ab (a + b) =
a b 3 3
74. (3) x
5 2
=
e 5 2 j
1 1
5 2 e 5 2 je 5 2 j
a3 + b3 + 3ab × 3 3
ab a b
5 4 4 5
1 = 9 4 5
5 4
3 3 – a3 + b3 = 3
a b
1
1 1 9 4 5
2
70. (1) x = e 2 1 2 j x 9 4 5 9 2
e4 5 j
x 2 2 1
9 4 5
9 4 5
1 1 2 1 81 80
x2 = = 2 1
2 1 2 –1 2 1
x4 + x–4 = x4 +
1
=
FG x 2 1 IJ 2
2
1
2
2 1 x4 H x2 K
x
1 LF 1 IJ 2 O 2
x2 2 1 2 1 =2
= MGH x K 2P 2
x2 MN x PQ
u v u v
 LMe9 OP 2
x y 1  uv 1  uv
2
71. (3)
1  xy
=
u v IJ FG IJ
u v FG =
N 4 5 9 4 5 j 2
Q 2 = [(18)2 – 2]2 – 2
1
KH K
1  uv 1  uv H = (322)2 – 2 = 103682 Whole Number

=
bu  vgb1  uv g  bu  v gb1  uv g 5– 3 5 3
1  u v  du  v i 2 2 2 2 75. (1) x = ,y=
5 3 5– 3

u  u 2v  v  uv 2  u  u 2v  v  uv 2 5– 3 5 3
=
1  u 2v 2  u 2  v 2 x+y= +
5 3 5– 3

=
2u  2uv 2
=
d i 2u 1  v 2
2 2 FG e 5 j e 3 j IJ
2 2
1 v  u 1v
2 2
d 2
i d1  u id1  v i 2 2
e 5– 3 j e 5 3j 2
H K
= =
2u e 5 3j e 5 – 3j 5–3
=
1 u2 = 5+3=8
a b c 5– 3 5 3
72. (4) 1
1 a 1 b 1 c xy = × =1
5 3 5– 3
FG a 1IJ FG b 1IJ FG c 1IJ
H1 a K H1 b K H 1 c K x2 xy y2 bx y g – xy 2

=
=3+1=4 x 2 – xy y2 bx y g – 3xy 2

a 1 a b 1 b c 1 c
=4
1 a 1 b 1 c 82 – 1 64 – 1 63
= = =
1 1 1 82 – 3 64 – 3 61
4
1 a 1 b 1 c

BLAM–71
chtxf.kr
2nd Method 78. (4) Options ls] ;fn x = y = z gks] rks
1 1 1 3 1 1 1
a b
,y
a b ,oa
x= x2 y2 z2 x2 xy yz zx
a b a b
If a – b = 2 then x + y = a + b and xy = 1 1 1 1 3
Now, 2 2 2
x x x x2
x 2
xy y 2
bx y g 2
xy 79. (3) a + b + c = 0
x 2
xy y 2
bx y g 2
3xy a + b = – c ; b + c = – a, c + a = – b
a b b c c a
=–1–1–1=–3
b5 3g 2
1 63 c a b
=
b 3g
5
2
3 1 61
a b c
=–1–1–1=–3
b c c a a b
76. (1) x + y = 2z x = 2z – y
x – z = 2z – y – z = z – y Expression = (– 3) × (– 3) = 9

x z x z 1 1

x z y z x z z y Fp q I
80. (4) G
–1 2 3 Fp q I 6 –3 3

H p q JK 3 –2
H p q JK
÷ G –2 3

x z x z
1
x z y z x z 1 1
= pa qb e p –1–3 q 2 j e
2 3 ÷ p6 2
q –3 –3 j 3

13 11
77. (1) x = 1 1
13 – 11
On rationalising the denominator
= pa qb ep –4 4 3
q j e
÷ p 8q –6 j 3 = pa qb

13 11 13 11 4 4

= × p 3q 3 4 8 4 6
13 – 11 13 11 = pa qb 3 3 = pa qb
8 –6 p q3 3

2
p 3q 3

=
e 13 11j
2 2 10
e 13 j – e 11j p–4 q 3 = pa qb

10
13 11 2 143 24 2 143 a = –4, b =
= = = 12 + 143 3
13 – 11 2
10 –2
a + b = –4 + =
1 1 1 12 – 143 3 3
y= = = ×
x 12 143 12 143 12 – 143
1 –1
81. (1) x = a 2 + a 2
12 – 143
= = 12 – 143 1 –1
144 – 143
y = a2 – a 2

x – y = 12 + 143 – 12 + 143 = 2 143 1 –1


x2 – y2 = 4 a 2 . a 2 = 4
,oa xy = 12 e 143 j e12 – 143 j [  (a + b)2 – (a – b)2 = 4ab]
=144 – 143 = 1 1 –1
3x2 – 5xy + 3y2 = 3x2 – 6xy + 3y2 + xy iqu%, y2 – x2 = – 4 . a 2 . a 2 = –4
2 Expression = (x – x y – 1) + (y4 – x2y2 + 1)
4 2 2
= 3 (x – y)2 + xy = 3 2 143 e j +1
= x2 (x2 – y2) – 1 + y2 (y2 – x2) + 1
= 3 × 4 × 143 + 1 = 1716 + 1 = 1717 = 4x2 – 1 – 4y2 + 1 = 4 (x2 – y2) = 4 × 4 = 16

BLAM–72
chtxf.kr
82. (1) x – 3 – 2 =0 x y z
85. (2) ;gk¡ = + =a
(2x  y  z ) ( x  2y  z ) ( x  y  2z )
x= 3 + 2
x = a(2x + y + z)
iqu%, y = a(x + 2y + z)
y– z = a(x + y + 2x)
3 + 2 =0
rhuksa dks add djus ij]
y= 3 – 2 x+y+z
= a(2x + y + z) + a(x + 2y + z) + a(x + y + 2z)
x–y= 3+ 2– 3 + 2 =2 2 (x + y + z) = a(4x + 4y + 4z)
(x + y + z) = 4a(x + y + z)
,oa xy = 3 e 2 je 3– 2 j
1
=3–2=1 a=
4
Expression = x3 – 20 2 – y3 – 2 2
1 1 1
86. (1)
= x3 – y3 – 22 2 a b a b

= (x – y)3 + 3xy (x – y) – 22 2 b a 1
=
ab a b
3
= 2 2 e j e j
+ 3 2 2 – 22 2 (a – b) (a – b) = –ab
a2 – 2ab + b2 = –ab
a2 – ab + b2 = 0
= 16 2 + 6 2 – 22 2 = 0
a3 + b3 = (a + b) (a2 – ab + b) = 0
83. (1) x = 1, j[kus ij 87. (1) ;gk¡ a + b + c = 0
2 2
p (p + 1) = p(p–3 + p–1) a+b=–c
1 1 nksuksa rjiQ square djus ij
1 = 1 2
p2 p a2 + b2 + 2ab = c2
a2 + b2 – c2 = –2ab
x = –1, j[kus ij] similarly,
p–2(p2 + 1) = p(p–3 + p–1) b2 + c2 – a2 = –2ab
p–2 + 1 = p–2 + 1 c2 + a2 – b2 = – 2ac
1 1 1 1 1
1+ =1+ 2 + +
p2 p a 2  b2  c 2 b2  c2  a 2 a 2  c 2  b2

L. H. S = R. H. S 1 1 1
=  
2ab 2bc 2ac
84. (1) x 2 3x 1 0
x2 + 1 = 3x
=–
LM
1 a b c OP
nksuksa rjiQ x ls divide djus ij 2 N
abc Q
1 =0
x 3
x 2
2 1 e 2 j
1
1 1 88. (1) x = =
x2 x 2 1 e 2 1 je 2 j
1
x x2

FG x 2 1 IJ FG x 1 IJ =2+1+ 2 2 =3+2 2
= H x 2 K H x K y=3– 2 2
x+y=6 ,oa x – y = 4 2
=
FG x 1 IJ 2
2
FG x 1 IJ
H x K H x K 2x 2 3xy 2y 2
2
= 9 – 2 + 3 = 10 2x 3xy 2y 2

BLAM–73
chtxf.kr
2(x 2 2xy y2) xy 2(x y )2 xy 3 3 2
2 2 =
2(x 2xy y ) xy 2(x y )2 xy 3 2

iqu%]
2 62 1 72 1 71
2 64 1 65
2(4 2 ) 1 x 2 2
12 3 2
4ab
89. (4) x =
a b
x 12
x 2b x 12
2a a b
(Componendo rFkk dividendo osQ }kjk) 2 2 3 2 3 3 2
= =
2 2 3 2 2 3
x 2a 2b a b 3b a
= =
x 2a 2b a b b a x 8 x 12
blh izdkj] x 8 x 12

x 2a
3 3 2 3 3 2
2b a b =
3 2 2 3
x 2b 2a a b 3a b
= =
x 2b 2a a b a b 3 3 2 3 3 2
=
3 2 3 2
x 2a x 2b 3b a 3a b
+ = +
x 2a x 2b b a a b
3 3 2 3 3 2
=
=
3b a 3a b
=
2b 2a 2 b a
=
b g =2 3 2
b a b a b a
Aliter :
=
2 3 2 2
=
2 e 3 2 j 2
3 2 3 2
2ab x a x b
;fn x = gks rks] x a + x b = 2 Aliter
a b

4ab 4ab x 2a x 2b
;gk¡ x = a gS] ;fn x = a b gks rks] =2
b x 2a x 2b

x 2a x 2b x 8 x 12
=2
x 2a x 2b x 8 x 12
=2
2 24
90. (2) x =
3 2 2 3 2
91. (4) x
3 2
2 3 8 2 3 8
x
3 2 3 2 x 2 3
2 3 2
x 2 3
8 3 2 x 2 2 3 3 2 3 3 2
=
x 2 2 3 3 2 3 2
x 8 2 3 3 2
(Componendo rFkk dividendo osQ }kjk )
x 8 2 3 3 2
blh izdkj,
(Componendo rFkk dividendo osQ }kjk )

BLAM–74
chtxf.kr
x 2 2 5 3 2 5 6 3 3 5
=
3 3 2 5 3 4 5 4 3 3 5

x 3 2 2 3 2 3 3 2 60 15 15 8 15 30 90
=
x 3 2 2 3 2 2 3 15 15 24 15 60
=
Expression e5 3 4 5 4 3 je 3 5 j
x 2 x 3
=
x 2 x 3 240 – 62 15
=
60 15 15 16 15 60
3 3 2 3 3 2
=
3 2 2 3 240 62 15
=
120 31 15
3 3 2 3 3 2
=
3 2 3 2
=
e
2 120 31 15 j 2
3 3 2 3 3 2 120 31 15
=
3 2 Aliter :

4ab
=
2 3 e 2 j 2
;fn x = a b gks rks] x
x
2a
2a
x
x
2b
2b
=2
3 2
Aliter : x 20 x 12
=2
4ab x a x b x 20 x 12
;fn x=
a b
gks rks] x a x b
2

93. (4) x2 – yz = x2 + xy + zx = x (x + y + z)
x 2 x 3
=2
x 2 x 3 LM xy yz zx 0 OP
N yz xy zx Q
4 15
92. (2) x blh izdkj,
5 3
y2 – zx = y (x + y + z)
z2 – xy = x (x + y + z)
=
4 15 e 5 3 j Expression
e 5 3 je 5 3 j 1 1 1
= x (x y z) y (x y z ) + z( x y z)

=
4 15 e 5 3 j
yz zx xy
5 3 = xyz ( x y z ) 0

= 2 15 e 5 3 j 10 3 6 5 94. (2) (a2 + b2)3 = (a3 + b3)2


a6 + b6 + 3a2b2 (a2 + b2)
x 20 x 12 = a6 + b6 + 2a3b3
x 20 x 12 3 (a2 + b2) = 2ab

10 3 6 5 2 5 10 3 6 5 2 3 a 2 b2 2
= + ab 3
10 3 6 5 2 5 10 3 6 5 2 3

a b 2
10 3 4 5 12 3 6 5
= b a 3
10 3 8 5 8 3 6 5

BLAM–75
chtxf.kr
(a b )2 (b c )2 (a c )2 1
95. (2) 98. (3) x = p +
(b c )(c a ) (a b )(c a ) (a b )(b c ) p

1
(a b )3 (b c )3 (c a )3 y=p–
= [  (a – c)2 = (c – a)2] p
(a b )(b c )(c a )
1 1
3(a b )(b c )(c a ) x+y=p+ +p– = 2p
= =3 p p
(a b )(b c )(c a )

[ ;fn a + b + c = 0, a3+ b3 + c3 = 3abc. x–y=p+


1 1 2
–p+ =
;gk¡] a – b + b – c + c – a = 0] p p p
96. (2) ;fn a + b + c = 0, a3 + b3 + c3 = 3abc x4 – 2x2y2 + y4 = (x2 – y2)2
1
;fn a 3 b 3 c 3 = 0
1 1
= {(x + y) (x – y)}2
F 2 IJ
= G 2p
2

= 42 = 16
H pK
F 1 3 I F 1 3 I F I
1 3
GGa 3 JJ GGb 3 JJ GG c 3JJ 1 1 2 3
H K H K H K 99. (3) x =
2 3
=
2 3
×
2– 3
1 1 1
= 3 .a 3 . b 3 . c 3 2 3
= =2– 3
1 1 1 4 –3
a+b+c= 3 a 3 .b 3 . c 3
1
(a + b + c)3 y= =2+ 3
2 3
F 1 1 1 3 I
= 33. GG a . b .c 3
3 3 JJ = 27abc x+y=2– 3 +2+ 3 =4
H K xy = (2 – 3 ) (2 + 3 ) = 4 – 3 = 1

97. (1) x
FG 1 IJ 2
3
8xy + y2)(x2
H x K = 8xy [(x + y)2 – 2 xy]
= 8 × 1 (42 – 2 × 1)
1 = 8 (16 – 2) = 8 × 14 = 112
Ÿx 3
x
x 24 1
nksuksa rjiQ cube djus ij] 100. (2) =7
x 12
x3
1
3 x
FG 1 IJ 3 3
x3 H x K x 24 1
+ 12 = 7
x 12 x
3 1
Ÿx 3 3 3 3 0
x3 1
x12 + 12 =7
Ÿ x6
+1=0 x
? x206 + x200 + x90 + x84 + x18 + x12 + x6 + 1
= x200 (x6+1) + x84 (x6+1) + x12 (x6+1) + (x6+1) = 0 x 72 1 x 72 1
= +
x 36
x 36 x 36
2nd Method
1 1
1 = x36 + =(x12)3+
;fn x
x
3 or 1 x 36 (x 12 3
)

rks Question esa ;fn Pair yxrk gS rks Answer 0 gksxk ;fn
=
FG x 12 1 IJ 3
– 3 × x12 ×
1 FG x 12 1 IJ
Pair ugha yxs rks tks Remainder cpsxk ogha Answer gksxk H x 12 K x 12 H x 12 K
(x206 + x200) + (x90 + x84) + (x18 + x12) + (x6 + 1)
[  a3 + b3 = (a + b)3 – 3ab (a + b)]
‡‡‡
Ans 0
= 73 – 3 × 7 = 343 – 21 = 322

BLAM–76
chtxf.kr

ADVANCED LEVEL QUESTIONS


y
1. If ax = b, b = c and x y z = 1, then the value of cz
will be (1) 11 2 24 (2) 19 2 35
;fn a = b, b = c rFkk xyz = 1rks c dk eku gksxk
x y z
(3) 11 – 2 55 (4) 14 – 2 24
(1) a (2) b (3) a b (4) a2
+ 1
2. If 2m + 2m = 96, then the maximum prime p  2q  p – 2q
order pair form, which satisfy the given equation 7. If x = , then qx2 – px + q = 0
p  2q – p – 2q
is
;fn 2m + 2m + 1 = 96, rks] vf/d ls vf/d vHkkT; tksMs+ equals to
tks fn, x, lehdj.k dks larq"V djrs gS p  2q  p – 2q
(1) 1 (2) 2 (3) 3 (4) 4 ;fn x = , rks qx2 – px + q = 0 cjkcj gS
p  2q – p – 2q
3. If and are, the roots of x2 – P (x + 1) – c = 0 (1) 1 (2) 0
2 2 (3) 2 (4) 3
2 1 2 1
then the value of 2 2
, is 8. The number of solutions of the equation
2 c 2 c
3 3
2 1 x 8 x = 3 is equal to
;fn rFkk lHkhdj.k x – p (x + 1) – c = 0 osQ ewy gS rks
lehdj.k 3
1 x 3
8 x = 3 osQ gyksa dh la[;k cjkcj
2 2
2 1 2 1
2 2 dk eku gksxkµ gksxhµ
2 c 2 c
(1) Two (2) One
(1) 2 (2) 1 (3) Zero (4) None
(3) –1 (4) 0 9. If and are the roots of the equation x2 + x – 1
= 0 then what is the equation whose roots are 5
x 2 4x 3 x 2 4x 3
4. If 7 e 4 3 j + 7 e 4 3 j = 14 then and 5 ?
;fn rFkk lHkhdj.k x2 + x – 1 osQ ewy gS rks og lehdj.k
the value of x is given by
D;k gSa ftuosQ ewy 5 rFkk 5 gS\
x 2 4x 3 x 2 4x 3
;fn 7 4 3 = 14 gks rks (1) x2 + 7x – 1 = 0 (2) x2 – 7x – 1 = 0
e j + 7 e 4 3 j (3) x2 – 11x – 1 = 0 (4) x2 + 11x – 1 = 0
x dk eku fn;s eku esa ls gksxk\
10. 2a 2 2 6ab 3b 2 equals to
(1) 2 (2) 12 2
2a 2 2 6ab 3b 2 cjkcj gS
(3) 12 2 (4) None
(1) 2a 3b (2) 3a 2b
5. If x x – 1 – x = 1, then the value of x will
be (3)
2a – 3b (4) 3b – 2a
3 2
11. x –x + a x + x – a –1 equals to
;fn x x – 1– x = 1 rks x dk eku gksxk
x3 –x2 + a x + x – a –1 cjkcj gS
25 16 (1) (x + 1) (x2 + a + 1) (2) (x – 1) (x2 + a + 1)
(1) (2)
16 23 (3) (x – 1) (x2 – a + 1) (4) (x – 1) (x2 – a –1)

16 16 12. If u v – w = 0, then the value of (u+v–w)


(3) (4)
27 25 will be
3

8 ;fn u v – w =0 rks] (u+v–w) dk eku gksxk
6. equals to
19 – 2 88 14 2 33
(1) 2 uv (2) uv
3 8
– (3) – uv (4) –2 uv
cjkcj gS
19 – 2 88 14 2 33

BLAM–77
chtxf.kr
1 1 2b 4b 3 8b 7 1 2
13. is x2 + + 2 – 2x – cjkcj gS
a b a b a 2
b 2
a 4
b 4
a 8
b 8 x2 x

equal to (foQlosQ cjkcj gS) \


(1)
FG x – 1 IJ FG x –
1 IJ 2
(1) a + b
(3) 1
(2) a – b
(4) 0
H xK H xK

14. If x + y + z = 1, xy + yz + zx = –1 and x y z = –1,


then the value of x3 + y3 + z3 will be FG x 1 IJ FG x –
1 I
J
2

;fn x + y + z = 1, xy + yz + zx = –1 rFkk x y z = –1rks


(2) H xK H xK
x3 + y3 + z3 dk eku gksxk
(1) –1 (2) 0 (3)
FG x – 1 IJ FG x –
1 I
J
2

(3) 1 (4) 2 H xK H xK

1 3
15. The value of
27
r – s3 + 125 t3 + 5rst will be how
(4)
FG x 1 IJ FG x
1 I
J
2

H xK H xK
r
much if s = + 5t 19. If a + b + c = 5 and ab + bc + ca = 15, then the
3 value of (a + b)3 + (b + c)3 + (c + a)3 – 3 (a + b) (b+c)
1 3 (c + a) equals to
r – s3 + 125 t3 + 5rst dk eku fdruk gksxk ;fn s = ;fn a + b + c = 5 rFkk ab + bc + ca = 15 rks (a + b)3 +
27
(b + c)3 + (c + a)3 – 3 (a + b) (b+c) (c + a) dk eku
r
+ 5t cjkcj gksxk
3
(1) 200 (2) 400
(1) 0 (2) 2 (3) –200 (4) –400
(3) 1 (4) –1
20. If a = 7 – 5 , b = 5 – 3 , c = 3 – 7 , then
3 3 3 then value of a3 + b3 + c3– 2abc will be
ea 2
–b j eb – c j ec – a j
2 2 2 2 2

16. will be equal ;fn a = 7 – 5 , b = 5 – 3 , c = 3 – 7 , then


ba – b g bb – c g bc – a g
3 3 3
rks a3 + b3 + c3– 2abc dk eku gksxk
to
(1) –4 5 – 2 3 2 7
3 3 3
ea 2
–b j eb – c j ec – a j
2 2 2 2 2 (2) –4 5  2 3  2 7
cjkcj gksxk
ba – b g bb – c g bc – a g
3 3 3
(3) –4 5 – 2 3  2 7

(1) (a – b) (b – c) (c – a) (4) –4 5 2 3 2 7
(2) (a – b) (b + c) (c – a) 21. If the equations x2 + 3x – 10 = 0 and x2 + 5x – m =
(3) (a + b) (b + c) (c + a) 0 have a common root, then the non-zero value
(4) (a + b) (b + c) (c – a) of m.
1/ 2 1/ 2 1/ 2
;fn lehdj.k x2 + 3x – 10 = 0 rFkk x2 + 5x – m = 0 dk
17. If a = e3 5 j e3 – 5 j and b = e3 5 j ,d mHk;fu"B ewy gks rks m dk 'kwU;srj eku gS&
(1) 0 (2) 10
1/2 (3) 14 (4) None of these
e
– 3– 5 j , then a2 +b2 equals to 22. If ax = 3 – 4x then what is the value of (a1 + a2 +
a3 + ...... ax)
1/ 2 1/ 2 1/ 2

;fn a = 3 e 5 j e3 – 5 j rFkk b = 3 e 5 j ;fn ax = 3 – 4x gS rks a1 + a2 + a3 + ...... ax dk eku D;k


gksxk\
1/2
(1) –x (4x – 3) (2) –x (2x – 1)
e
– 3– 5 j rks a2 +b2 cjkcj gS
(3) –x2 (4) –x (2x + 1)
(1) 10 (2) –4 23. If & are the roots to equation ax2 + bx + c = 0
(3) 11 (4) 12 then find the Quadratic Equation whose roots

1 2 1 1
18. x2 + + 2 – 2x – equals to are 2
& 2
2
x x

BLAM–78
chtxf.kr
;fn , lehdj.k ax + bx + c = 0 osQ ewy gSa rks og f}?kkr
2 (1) 160, 128 (2) 120, 160
(3) 128, 160 (4) 736, 128
1 1
lehdj.k Kkr djsa ftuosQ ewy 2
rFkk 2 1
29. Find the minimum value of x 2 5
(1) c2x2 – (2ac – b2)x + a2 x2 2
(2) c2x2 + (2a2c2 – b2)x + c 1
(3) c2x2 + (2ac – b2)x + a2 x2 2
5 dk vf/dre eku Kkr djsa
x 2
(4) c3x2 + (2ac – b2)x + a2
24. If the ratio of roots of equation px2 + qx + q = 0 is (1) –5 (2) 5
(3) –3 (4) +3
a b q
a : b, then find the value of ?
b a p 1 1 1 1
30. 1 1 .... +
12 22 22 32
;fn px + qx + q = 0 osQ ewyksa dk vuqikr a : b gS rks
2

a b q 1 1
? 1 is equal to fdlosQ cjkcj gS\
b a p 20072 20082

(1) 0 (2) 1 1 1
(1) 2008 – (2) 2007 –
1 2008 2007
q
(3) (4) buesa ls dksbZ ugha 1 1
p (3) 2007 – (4) 2008 –
2008 2009
a 16b 16c 31. If 4x + 3a = 0 then what is the value of
25. If x 1 y 4b y 4c = 8 – a Than
Fx 2
ax a2 x2 ax a2 I
ax 4y 4z GH x3 a3 x3 a3
JK ?
Find x 1 y 4b y 4c = ?

a 16b 16c
Fx 2
ax a2 x2 ax a2 I
;fn x 1 y 4b y 4c = 8 – a rks ;fn 4x + 3x = 0 rks GH x 3
a 3
x 3
a 3 JK
ax 4y 4z
dk eku D;k gS\
x 1 y 4b y 4c = Kkr djsa 4 7
(1) (2)
(1) 12 (2) 4 7a a
(3) 16 (4) .8 32 24
(3) (4)
1 2 1009 7a 7a
26. If x 1 then
1 y 2 z 1009 32. What is the minimum Value of expression
2x2 + 5x + 5.
x y z
x 1 y 2 z 1009
? O;atd 2x2 + 5x + 5 dk U;wure eku D;k gS\

(1) 0 (2) 2 15
(1) 5 (2)
(3) 3 (4) 4 8
27. Find the maximum value of y = (9 – x) (x + 3) 15
y = (9 – x) (x + 3) dk vf/dre eku Kkr djsa (3) (4) 0
8
(1) 0 (2) 35
33. If x = 13, Find the value of x5 – 14x4 + 14x3 – 14x2
(3) 36 (4) 42 + 14x – 5
28. Find the maximum and minimum value of
;fn x = 13, x5 – 14x4 + 14x3 – 14x2 + 14x – 5 dk eku
f(x) = x3 – 18x2 + 96x
Kkr djsa
f(x) = x3 – 18x2 + 96x dk vf/dre rFkk U;wure eku Kkr
(1) 7 (2) 8
djsa (3) 9 (4) 10

BLAM–79
chtxf.kr
38. Find the value of x for which the epression 5 – 4x
4t 1
34. If x = 2t, and y then the value of t, when – 2x2 has the greatest value
3
x dkeku Kkr djsa ftlosQ fy, O;atd 5 – 4x – 2x2 dk eku
x2 = y
vf/dre gks
4t 1
;fn x = 2t, rFkk y 3
rks t dk eku tc x2 – y (1) –1 (2) 7
(3) 2 (4) 0
1 39. If a + b = 3, b + c = 4, and c + a = 5 then Find (a2
(1) (2) 2 + b2 + c2 + ab + bc + ca) = ?
4
;fn a + b = 3, b + c = 4, rFkk c + a = 5 rks (a2 + b2 + c2
1
(3) (4) buesa ls dksbZ ugha + ab + bc + ca) Kkr djsa
2
35. If and are the roots of the equation x2 – 5kx + 25
(1) (2) 10.5
2
5
10k2 = 0, then find K if 2
+ 2
= (3) 25 (4) 14
4
40. If ab + bc + ca = 0 then
;fn rFkk lehdj.k x2 – 5kx + 10k2 = 0 osQ ewy gSa rks K
5
db 2
c a c2 hd ab h da 2
bc c 2 hd ab h da 2
hd
bc b 2 ca h=?
Kkr djsa ;fn 2
+ 2
=
4
da 2
bc h db 2
ca hdc
2
ab h

(1) –1 (2) 0
1
(1) 1 (2) (3) 1 (4) 2
2
41. If xy = r, x2 = r2, y2 = r3, x + y + z = 14 and x2 + y2
1 1 y
(3) (4) – z2 = 28 then find
2 2 x
36. If & are the roots of the equation x2 – 5x – 4 = ;fn xy = r, x2 = r2, y2 = r3, x + y + z = 14 rFkk x2 + y2
3 3
y
0, then what is 2 2 is equal to – z2 = 28 rks x Kkr djsa
( – )( – )
(1) 4 (2) 5
3 3
;fn rFkk x2 – 5x – 4 = 0 osQ ewy gS rks 2 2 (3) 3 (4)
7
( – )( – )
4
cjkcj gS& 42. Find the maximum and minimum value of f(x) =
(1) 37/41 (2) 40/37
x 2
(3) 13/41 (4) 37/9
2x 2 4x 8
37. If and are the roots of the equation
x 2
x 2 x 1 f(x) = dk vf/dre rFkk U;wure eku Kkr
2 then find the value of 3
+ 2x 2 4x 8
2 x x 2
3
.
djsa
1 1 1
x 2 x 1 (1) 0, (2) ,
;fn rFkk , 2 osQ ewy gSa rks 3
+ 4 12 4
2 x x 2
1 1 1
3
eku gS& (3) , – (4) , 0
4 12 4
104 4
(1) (2) 43. If x, y, z are positive real numbers, then the least
25 25
FG 1 1 1 IJ
25 16 value of (x + y + z) is :
(3)
104
(4)
25
Hx y 2 K

BLAM–80
chtxf.kr
;fn x, y, z /ukRed okLrfod la[;k,¡ gSa rks (x + y + z) 3 1
;fn a3 + a – a2 – 1 = 0, rks a a dk eku D;k gS
FG 1 1 1 IJ
Hx y 2 K dk U;wure eku gS & (1) 0 (2) 1
(3) –1 (4) 2
(1) 6 (2) 9 48. If P = 8 + 3 7 and PQ = 1, then what is the value
(3) –1 (4) buesa ls dksbZ ugha
44. If a, b, c are all positive then the minimum value
FG 1 IJ FG 1 IJ
of expression
of HP K HQ K
3 3 ?

(a 3 a2 a ) (b3 b2 b ) (c 3 c2 c)
;fn P = 8 + 3 7 rFkk PQ = 1, rks 3
FG 1 IJ FG 1 IJ dk eku
(abc )2 P H K HQ K 3

D;k gS
;fn a, b, c lHkh /ukRed gSa rks O;atd dk U;wure eku gS&
(1) 4480 (2) 4048
(3) 4200 (4) 4000
(a 3 a2 a ) (b3 b2 b ) (c 3 c2 c)
2 5
(abc ) 49. If (1 p 2 ) (1 – q 2 ) , then what is the value
3
(1) 9 (2) 27
(3) 1 (4) 8 of 5p 2 5q 2 5pq 5p 2 5q 2 5pq ?
1 1
45. If x 4 and x 2 11 then the value of 5
x x4 ;fn (1 p 2 ) (1 – q 2 ) , rks
3

1 5p 2 5q 2 5pq 5p 2 5q 2 5pq dk eku D;k


x4 2 is
x gS \
2 5 3 5
1 1 1 (1) (2)
;fn x 4 rFkk x 2 11 rks x4 2 dk 3 9
x x4 x
4 5 4 5
eku gS& (3) (4)
9 3
(1) 10 (2) 12
50. If A = 1 + 5P and B = 1 + 5–P, then what is the
(3) 13 (4) 11
value of B ?
9 3x 9 3y 9 3z ;fn A = 1 + 5P rFkk B = 1 + 5–P, rks B eku D;k gS \
46. If = 0, then Find the val-
4x 4y 4z A 1 A+2
(1) (2)
A 1 A 1

ue of
FG 3 3 3 IJ 2
A A–2
Hx y z K (3)
A 1
(4)
A 1

FG 3 3 3 IJ 2 SHORT ANSWERS
9 3x 9 3y 9 3z
;fn = 0 rks Hx y z K
4x 4y 4z
1. (1) 2. (1) 3. (2) 4. (1) 5. (4)
dk eku Kkr djsa 6. (1) 7. (2) 8. (4) 9. (4) 10. (1)
11. (2) 12. (4) 13. (4) 14. (3) 15. (1)
1 16
(1) (2) 16. (3) 17. (4) 18. (2) 19. (3) 20. (4)
2 25
21. (3) 22. (2) 23. (3) 24. (1) 25. (3)
9 26. (2) 27. (3) 28. (1) 29. (1) 30. (1)
(3) (4) 16 31. (3) 32. (2) 33. (2) 34. (1) 35. (4)
16
36. (1) 37. (1) 38. (2) 39. (3) 40. (2)
47. If a3 + a – a2 – 1 = 0, then what is the value of
41. (1) 42. (3) 43. (2) 44. (2) 45. (3)
1
a3 46. (3) 47. (1) 48. (2) 49. (4) 50. (3)
a

BLAM–81
chtxf.kr

EXPLANATIONS b 1g 2
b g
1
2

=
1. (1) ;g fn;k gqvk gS
b 1 g b 2
1 gb 1 g b g b
1
2
1 gb g
1

ax = b
1 1 1 1
nksuksa rjiQ powers esa y ls multiply djus ij = = 1

(ax)y = by
axy = c x 2 4x 3
4. (1) Let 7 e 4 3 j =P
nksckjk power esa z ls multiply djus ij
z x 2 4x 3 1
e j
a xy cz e7 4 3 j =
P
.

A/q the expression,


axyz = cz
a1 = cz 1
P+ 14
cz = a P
2. (1) Here, 2m + 2m+1 = 96 P2 – 14P + 1 = 0
2m + 2m × 2 = 96
2m (1 + 2) = 96
P=
14 b14g 2
4 1
2
96
2m =
3 P=7 4 3
2m = 32
1
2m = 25 Put the value of P. S
P
m=5
vr% 5 ls NksVs prime numbers 2 rFkk 3 gS x 2 4x 3

Their sum = 2 + 3 = 5
e7 - 4 3 j 7 4 3

required pairs = (2, 3) or (3,2) x2 – 4x + 3 = 1 or x2 – 4x + 3 = –1


3. (2) Given, LM7 4 3 e7 4 3 j OPQ
1
x2 – px – (P + C) = 0
N
+ =
b Pg P
x2 – 4x + 2 = 0 or x2 – 4x + 4 = 0
1
x= 2 2 or x 2, 2
= – (P + C)
A/q 5. (4) x + x – 1– x = 1

2 2
2 1 2 1 x – 1 – x = 1– x
2 2
2 c 2 c
nksuksa rjiQ square djus ij_
x– 1– x = 1 + x – 2 x
e 2
2 j
1 e 2
2 1j
= 1 – x = 1– 2 x
e 2
2 1 j bc 1g e 2
2 1 j bc 1g –
iqu% nksuksa rjiQ square djus ij_

g b b 1g 1
2 2 1 – x = 1+ 4 x – 4 x
=
b 1g b1 c g b 1g b1 c g
2 2
– 5x = – 4 x

[ 1 – C = – P – C + P + 1 5 x =4
= +( + )+1
4
= ( + 1) ( + 1)] x =
5

BLAM–82
chtxf.kr
nksuksa rjiQ square djus ij_
bx  1g 2
p  2q

x=
16 bx – 1g 2
p – 2q
25
6. (1) We have,
dx 2
 1  2x i p  2q
3

8 dx 2
 1 – 2x i p – 2q

19 – 2 88 14 2 33 px 2 + p + 2px – 2qx2 –2q – 4qx


= px 2 + p – 2px + 2qx2+ 2q – 4qx
3 8
= – 4qx2 – 4px + 4q = 0
11 8 – 2 11 8 11 3 2 11 3 qx2 – px + q = 0
8. (2) Given expression
3 8
= – 3
1 x 3
8 x 3
2 2
e 11  8 j e 11  3 j cubing both sides
3
3 8
(1 + x) + (8 – x) + 3 1 x .3 8 x
= 
11  8 11  3
e1
3
x 3
8 x j 27

=
3 e 11 8 j– 8 11 – 3 9+3 3
1 x .3 8 x 3 27
e 11 – 8 j e 11 8 j e 11 3 j 11 – 3
3
1 x .3 8 x 2
Now, again Cubing, we get

=
3u e 11  8 j  8 u e 11  3 j (1 + x) (8 – x) = 8
2 2 2 2
e 11 j  e 8 j e 11 j  e 3 j 8 – x + 8x –x2 = 8
x2 = 7x
x=7
= e 11 j e 11 – 3 j
8 –
The number of solution is one.
9. (4) x2 + x – 1 = 0
= 8 + 3
+ = – 1, = –1
2 1
= e 8 3 j let,

then
= 8 3 2 24
1
– 1
= 11 2 24

1
p 2q p – 2q 5
+ 5
= 5– 5
7. (2) x =
p 2q – p – 2q
= [(–1)3 + 3 × (–1)] [(–1)2 + 2] + 1
componendo rFkk dividendo ds }kjk] = –4 × 3 + 1 = –11
5 5
+ = (–1)5 = –1
x 1 p  2q  p – 2q  p  2q – p – 2q
Required equation
x –1 p  2q  p – 2q – p  2q  p – 2q
x2 + 11x – 1 = 0

x 1 2 p  2q 1 1
[If x – = a then x5 – 5 = (a3 + 3a) (a2 + 2) –a]
x –1 2 p – 2q x x

nksuksa rjiQ square djus ij_


BLAM–83
chtxf.kr
10. (1) 2a 2 2 6 ab 3b 2 x3 + y 3 + z3– 3 × (–1) = 1 3 – –1 b g
x3 + y3 + z3 + 3 = 4
2 2
= e 2a j e 3b j 2. 2a 3b x3 + y3 + z3 = 1
r
15. (1) ;gk¡, s  5t
2 3
= e 2a 3b j
r
s– – 5t 0
= 2a 3b 3
11. (2) x3 – x2 + ax + x – a –1 ge tkurs gSa,
= x3 –x2 + ax – a + x –1 if a + b + c = 0
= x2 (x – 1) + a (x – 1) + 1 (x – 1) a3 + b3 + c3 – 3abc = 0
= (x – 1) (x2 + a + 1)
12. (4) ;gk¡, FG r IJ 3

(s )3 ( 5 t )3 3
FG r IJ (s )( 5t ) 0
H 3K H 3K
u v – w =0
r3
 s3  125t 3  5rst 0
u v – w 27

nksuksa rjiQ square djus ij_ 16. (3) ;gk¡,


(a2 – b2) + (b2 – c2) + (c2 – a2) = 0
2 2
e u v j e wj (a2 – b2)3 + (b2 – c2)3 + (c2 – a2)3
= 3(a2 – b2) (b2 – c2) (c2 – a2)
u + v + 2 uv = w  if x + y + z = 0
x3 + y3 + z3 = 3xyz
u + v – w = – 2 uv (a2 – b2)3 + (b2 – c2)3 + (c2 – a2)3
= 3(a – b) (a + b) (b – c) (b + c) (c – a) (c + a)
LM 1 1 O 2b 4b 3 8b 7
b PQ blh izdkj, (a – b) + (b – c) + (c – a) = 0
13. (4)
Na b a a2 b2 a4 b4 a8 b8
(a – b)3 + (b – c)3 + (c – a)3

=
LM 2b 2b OP 4b 3
8b 7 = 3(a – b) (b – c) (c – a)
Na b
2 2
a 2
b 2
Q a 4
b 4
a 8
b8
(a 2 – b 2 )3 (b 2 – c 2 )3 (c 2 a 2 )3
3 3 3
(a b) (b c) (c – a )
2b.2b 2 4b 3 8b 7
=
a4 b4 a4 b4 a8 b8 3(a – b )(a  b )(b – c )(b  c )(c  a )(c  a )
3(a  b )(b  c )(c – a )
4b 3.2b 4 8b 7 8b 7 8b 7 = (a + b) (b + c) (c + a)
= 8 8 8 8 = 8 8 8
0
a b a b a b a b8 17. (4) ge tkurs gSa, a2 + b2 = (a + b)2 – 2ab
2
14. (3) ;gk¡, x + y + z = 1 (a b )2 (3 5 )½ (3 5 )½ (3 5 )½ (3 5 )½
xy + yz + zx = – 1 rFkk
2
xyz = – 1 2(3 5 )½ 4(3 5) 12 4 5
vc, (x + y + z)2 = x2 + y 2 + z2 + 2xy + 2yz + 2zx
(x + y + z)2 = x2 + y2 + z2 + 2 (xy + yz + zx) 2 ab 2 (3 5 )½ (3 – 5 ) ½ ( 3 5 )½ (3 5 )½
12 = x2 + y2 + z2 + 2 (–1)
x2 + y2 + z2 = 3 LM{
2 (3 5 )½
2
} {(3 5 )½ } OPQ
2
x3 + y3 + z3 – 3xyz = (x + y + z) N
x 2 + y 2 + z 2 – xy + yz + zx b g 23 5 –3 5 4 5

BLAM–84
chtxf.kr
a3 + b3 + c3 = 3abc
a2 + b2 12 4 5 4 5
a3 + b3 + c3 – 2abc
a 2  b2 ba b g 2
 2ab = 3abc – 2abc
a3 + b3 + c3 – 2abc
a2 + b2 = 12 = abc
1 2 a3 + b3 + c3 – 2abc
18.(2) x   2  2x 
2

x2 x = ( 7 – 5 )( 5 – 3 )( 3  7 )

x2
1
2 2 x
1 FG
2 x
1 IJ ( 35 – 21  5  15 )( 3  7 )
x2 x H x K
( 105 – 7 5  3 7  7 3  5 3  5 7  3 5  105 )
FG x
1 IJ 2
FG
2 x 
1 IJ  4 5 2 32 7
H x K H x K
21. (3) By using condition for Common Root,
[  (a + b)2 = a2 + b2 + 2ab] p1q2 – p2q1) (q1r2 – q2r1) = (r1p2 – r2p1)2
FG x  1 IJ FG x  1  2IJ (1 × 5 – 1 × 3) (3 × –m – 5 × (–10)) = ((–10) × 1 –
H x KH x K (–m) × 1)2
(5 – 3) (–3m + 50) = (m – 10)2

FG x  1 IJ LM( x )  FG 1 IJ 2
2

– 2u x u
1 OP –6m + 100 = m2 + 100 – 20m
m2 – 14m = 0
H x K MN H xK x PQ m(m – 14) = 0 (  m 0 from question)
m = 14
FG x  1 IJ FG x –
1IJ 2
22. (2) Given,
H xK H xK an = 3 – 4x
an = (3 – 4x)
19. (3) ;gk¡, a + b + c = 5
= 3–4 n
nksuksa rjiQ squaring djus ij_
(a + b + c)2 = 25 LM x bx 1g OP
a2 + b2 + c2 + 2ab + 2bc + 2ca = 25 = 3n – 4
a2 + b2 + c2 + 2(ab + bc + ca) = 25
N 2 Q
a2 + b2 + c2 + 2 × 15 = 25 = 3x – 2x (x + 1)
a2 + b2 + c2 = –5 = 3x – 2x2 – 2x
;gk¡, = x – 2x2
= x (1 – 2x) = –x(2x –1)
(a + b)3 + (b + c)3 + (c + a)3 – 3 (a + b) (b + c) (c + a)
= [a + b + b + c + c + a] [(a + b)2 + (b + c)2 + (c + a)2 1
– (a + b) (b + c) – (b + c) (c + a) – (a + c) (a + b)] 23. (3) Let y= 2
[  x3 + y3 + z3 – 3xyz
= (x + y + z) (x2 + y2 + z2 – xy – yz – zx)] 2 1
= 2(a + b + c) [a2 + b2 + 2ab + b2 + c2 + 2bc + c2 + a2 y
+ 2ac – ab – ac – b2 – bc – bc – ba – c2 – ac – ac –
cb – a2 – ab] 1
= 2(a + b + c) {a2 + b2 + c2 – ab – bc – ac} y ( is the root of Equation)
= 2 × 5 × (–5 – 15)
= – 200 F1I
aG
2
F1I
;gk¡,
H y JK GH y JK
20. (4) b c 0

a 7 – 5, b 5 – 3, c 3– 7
vc, a b
c 0
y y
a+b+c= 7– 5 5– 3 3 7 =0

BLAM–85
chtxf.kr
27. (3) y = (9 – x) (x + 3)
a y by cy y 0
= 9x – x2 + 27 – 3x
2 = –x2 + 6x + 27
(by)2 = ea y cy y j dy
0
(Squaring both side we set) dx
b2y2 = a2y + c2y2y + 2acy2 –2x + 6 = 0
or, b2y = a2 + c2y2 + 2acy –2x = –6
c2y2 + (2ac – b2) y + a2 = 0 x=3
Replacing y by x
x=3 j[kus ij y to find maximum value
c2x2 + (2ac – b2)x + a2 = 0
y = (9 – x) (x + 3)
24. (1) fn;k x;k gS
= (9 – 3) (3 + 3)
px 2 + qx + q = 0,
= 6 × 6 = 36
Let the roots are and
28. (1) f(x) = x3 – 18x2 + 96x
q q x osQ lkis{k differentiation djus ij
+ = ,
p p f(x) = 3x2 – 36x + 96
Now, f(x) = 0 to find the value to x
a
3x2 – 36x + 96 = 0
b (given)
x2 – 12x + 32 = 0
x2 = 8x – 4x + 32 = 0
a b q
x(x – 8) –4 (x – 8) = 0
b a p
x(x – 4) (x – 8) = 0
x = 4, 8
q q
p p iqu% differentiate djus ij
0
q f11(x) = 6x – 36
p Now put the value to x in f11(x)
f11(4) = 6 × 4 – 36 = –12
9 16b 16c (–ve indicates at x = 4 maximum value we get)
25. (3) =8–9 f11(8) = 6 × 8 – 36 = 12
x 1 y 4b z 4c
(+ve indicates at x = 8 minimum value we get)
9 16b 16c Now, put x = 4 in f(x) to find the maximum value
9 4 4 = 8–9+9+4+4
x 1 y 4b z 4c f(4) = (4)3 – 18 × (4)2 + 96 × 4
= 64 – 288 + 384
9 9x 4 166 4y 166 16c 4z 16c = 160 (Maximum value)
= 16
x 1 y 4b z 4c f(8) = (8)3 – 18 × (8)2 + 96 × 8
= 512 – 1152 + 768
9x 4y 4z
= 16 = 128 (Minimum value)
x 1 y 4b z 4c
2 1
26. (2) Given, 29. (1) x 5
x2 2
1 2 1009
1 1
x 1 y 2 z 1009 (x2 + 2) + –5–2
x2 2
FG 1 1IJ FG 2 1IJ FG 1009 1IJ 1 3 Minimum value = 2 – 7 = –5
H x 1 K H y 2 K H z 1009 K
1 1
1
1 x 1 2 y 2 1009 z 1009
2
30. (1) an = x 2
bx 1g 2

x 1 y 2 z 1009

x y z
2 bx 1g x bx 1g
2 2 2
x2
x 1 y 2 z 1009 =
x b x 1g2 2

BLAM–86
chtxf.kr

bx 1g 2
x2 x2 x2 2 x2
FG 5 25 IJ FG 5 25 IJ
=
2x 1 = H 2 16 K H 8 K
x x b 1 g 2

FG
= 2 x
5 IJ 2
15

LMx bx 1g OP 2x bx 1g2
1
H 4 K 8
N Q
=
x b x 1g
2
FG x 5 IJ 2

For Min Value


H 4 K =0

Thus,
x x b 1 g 1
2

15
=
x x b g
1
2
Min Value fo (2x2 + 5x + 5) =
8
.

33. (2) x5 – 14x4 + 14x3 – 14x2 + 14x – 5


b
x x 1 1 1 g 1
1 1 = x5 – 13x4 – x4 + 13x3 + x3 – 13x2 – x2 + 13x + x – 5
= x x 1 b
=1+ x x 1
g b g x x 1 = x4(x – 13) – x3 (x – 13) + x2(x – 13) – x (x – 13) + 13
–5
 x = 13 x – 13 = 0
1 1 1 1 1 1 = 13 – 5 = 8
1 1 ... 1
12 22 22 32 b2007g b2008g 2 2
34. (1) From condition, x2 = y

FG
= 1 1
1 IJ FG1 1 1 IJ ... 1
FG 1 1 IJ (2t)2 =
4t 1
H 2 K H 2 3 K H 2007 2008 K 3

4t 1
1 4t2 =
= 2008 3
2008
12t2 – 4t – 1 = 0
31. (3) Given,
12t2 – 6t + 2t – 1 = 0
4x + 3a = 0
6t (2t – 1) + 1 (2t – 1) = 0
3a (6t + 1) (2t – 1) = 0
x=
4
1
Now, A/q t=
2
x2 ax a2 x2 ax a2 35. (4) x2 – 5Kx + 10K2 = 0
x3 a x3 a3 + = – (–5K) = 5K
= 10K2
1
= 2 2
5
x a x a + = (Given)
4
2a 2a 32 5
x2 a2 FG 3a IJ 2 7a ( + )2 – 2 =
= a 2 4
H 4 K
5
(5K)2 – 2 × 10K2 =
LM a2 b2 ab 1
and
a2 b2 ab 1 OP 4

MN a 3
b 3 a b a 3
b 3 a b PQ
25K2 – 20K2 =
5
4
32. (2) Given the Expression
2x2 + 5x + 5 1
K2 =
4
FG
2 x2
5 5
x
IJ
= H 2 2 K 1
K=±
2

BLAM–87
chtxf.kr
2
36. (1) x – 5x – 4 = 0 f(x) = –4 – 4x
f(x) = 0 to Find the maximum value
b ( 5)
+ = =5 –4 – 4x = 0
a 1
4
c 4 x= = –1
= = –4 4
a 1
f(–1) = 5 – 4 × – 1 – 2 × (–1)2
A/q
=5+4–2
3 3 ( )( ) = 7 (Maximum value)
= 39. (3) a2 + b2 + c2 + ab + bc + ca
( 2 2
)( ) ( )( )( )
1
= (2a2 + 2b2 + 2c2 + 2ab + 2bc + 2ca)
( ) 3 ( ) 3 2
= =
( )2 ( )2 4 1
= ((a + b)2 + (b + c)2 + (c + a)2)
then put the + = 5, = –4 2

(5)2 3 4 25 12 37 1
= = = (3)2 + (4)2 + (5)2)
(5)2 4 –4 25 16 41 2

50
x 2 x 5 = = 25
37. (1) 2
2 x x 2
40. (2) ab + bc + ca = 0
ab + bc = – ca
x 2 (2 x )2 5 bc + ca = – ab
x (2 x ) 2 ca + ab = – bc
Now, A/q
x 2 x 5
x (2 x) 2 db 2
hd
c a c2 ab h da 2
hd
bc c 2 h daab 2
bc b 2 hd ca h
Squaring both side we get, da 2
bc b hd 2
ca hdc ab h2

4 25
1 1
2x x2 4 =
a2 bc b2 ca c2 ab
25x2 – 50x + 16 = 0
1 1 1
16 =
2
x – 2x + =0 a2 ab ac b2 ab bc c2 bc ca
25
1 1 1
b ( 2)
+ =
a 1
2 = a a b b c g b ba b c g b c a b c g
=
c 16
=
1 1LM 1 1 OP
a 25 a b c a N b c Q
3 3
+ = ( + ) ( 2 + 2– )
= ( + ) (( + )2 – 3 )
1 L ab bc ca OP
c MN
=
a b abc Q
FG 3 16 IJ
= 2 (2)2
H 25 K 1 0
0
= ba b c g abc

= 2G
F 100 48 IJ = 2 52 104
41. (1) x + y + z = 14
H 25 K 25 25
Squaring both side we get
38. (2) f(x) = 5 – 4x – 2x2 x2 + y2 + z2 + 2(xy + yz + xz) = 196
Differentiation w.r.t. x 28 + 2 (r + r2 + r3) = 196

BLAM–88
chtxf.kr

r + r2 + r 3 =
196 28
= 84
FG 1 1 1 IJ
2 43. (2) (x + y + z)
Hx y z K
r + r2 + r3 = 84 = 4 + 16 + 64
hence, r = 4 y z x z x y
=1 1 1
x x y y z z
xz = r2 and yz = r3

x y y z x z
yz r3 3
2 y x z y z x
xz r
x y x
y Let y r, s, t
=r=4 z z
x

x 2 FG
= r
1 IJ FG s 1 IJ FGt 1IJ 3
42. (3) f(x) =
2x 2
4x 8
H r K H sK H t K
1
4  least value of r 2
If the function is in the form of then by using r
v
=2+2+2+3=9
differentiation osQ }kjk
a3 a2 a b3 b2 b c3 c2 c
du vdu 4dv 44. (2) 2 2
a b c2
dv v2
FG a 1
1 IJ FGb 1
1 IJ FG c 1
1 IJ
f (x )
2x 2 4x 8 1 (x 2)( 4 x 4) H a KH b KH c K
(2 x 2 4x 8 )2
1
To find the maximum and minimum value we take least value of x 2
x
f(x) = 0
= (2 + 1) (2 + 1) (2 + 1)
2x 2
4x 8 (x 2)( 4 x 4) = 27
=0
(2 x 2 4x 8)2 1
45. (3) x 4
2 2 x
2x + 4x + 8 – (4x + 8x + 5x + 8) = 0
2x2 + 4x + 8 –4x2 – 13x – 8 = 0
2x2 + 8x = 0 x2
1 FG x 1 IJ 2
2 = (4)2 – 2 = 14 ...(i)
2x (x + 4) = 0
x2 H x K
x = 0, –4
bx g FGH x1 IJK FGH x IJ
2 2
1 2 1
x4 2
2
Put x = 0 in f(x) to find maximum value
x 4 2
x 2 K
x 2 = (14)2 – 2 = 194 ...(ii)
f (x ) 2
2x 4x 8 lehdj.k (i) rFkk (ii) dks add djus ij
2 1 1 1
= x2 x4 = 14 + 194 = 180
8 4 x2 x4
Put x = –4 in f(x) to final minimum value
FG x 2 1 IJ FG x 4 1 IJ 180
4 2 H x4 K H x2 K
f(x) =
2 ( 4)2 4 ( 4) 8
1
x4 = 180 – 11 = 169
2 2 1 x2
=
32 16 8 24 12 We take Square root both side

BLAM–89
chtxf.kr
1 (P Q ) (P 2 Q2 PQ )
x4 2
169 13 3
x (PQ )

9 3x 9 3y 9 32 (  P + Q = 8 + 3 7 + 8 – 3 7 = 16)
46. (3) 0
4x 4y 42
(P Q ) ((P Q )2 3PQ )
9 3 9 3 9 3
0 (PQ )3
4x 4 4y 4 4z 4
16((16)2 – 3)
9 9 9 9
(1)3
4x 4y 4z 4
= 16 × 253 = 4048
1 1 1 1
5
x y z 4 49. (4) (1 p 2 )(1 q 2 )
3
nksuksa rjiQ 3 ls Multiply djus ij
nksuksa rjiQ Square djus ij]
3 3 3 3
5
x y z 4 (1 – p2) (1 – q2) =
9
nksuksa rjiQ Squaring djus ij] 1 – p2 = 1 p=0

FG 3 3 3 IJ 2
9 1 – q2 =
5
Hx y z K 16 9

47. (1) a3 + a – a2 – 1 = 0 5 4 2
q 1
nksuksa rjiQ ls Divide djus ij] 9 9 3
1
a2 + 1 – a – =0 5p 2 5q 2 5pq 5p 2 5q 2 5pq
a

a2 + 1 – a =
1
5 02 5
FG 2 IJ 2
5 0 5 02 5
FG 2 IJ 2
5 0
a H 3K H 3K
nksuksa rjiQ (a + 1) ls multiply djus ij]
a 1 20 20
(a + 1) (a2 + 1 – a) = 9 9
a

1 20 2 2 5 4 5
a3 + 1 = 1 + 2
a 9 3 3
1 50. (3) A = 1 + 5P and B = 1 + 5–P
a3 – =1–1=0
a 1
B=1+
48. (2) P = 8 + 3 7 5P
P
PQ = 1  5 =A–1
1
1 B= 1
Q= A –1
P
A 1 1
1 1 8 3 7 8 3 7 B=
8 3 7 A –1
P 8 3 7 8 3 7 64 63
A
B=
3 3 A –1
‡‡‡
1 1 P Q
3 3
P Q ( PQ )3

BLAM–90
funsZ'kkad T;kfefr

funsZ'kkad T;kfefr
2 (CO-ORDINATE GEOMETRY)
Important Points regarding co-ordinate system.
Co-ordinate Geometry (funsZ'kkad T;kfefr) (1) tgk¡ nksuksa axis i.e, x-axis rFkk y-axis ,d nwljs dks
dkVrh gS mls origin dgrs gSa rFkk origin osQ co-ordinates
Co-ordinate geometry mathematics dh og branch gksrs gSa (0,0)
gS ftlesa ge x rFkk y co-ordinates ij vk/kfjr lines dh
(2) First co-ordinate esa x rFkk y nksuksa dh value positive
equation os Q ckjs esa vè;;u djrs gSaA Basically, ;gk¡ ij tks
gksrh gSA
Hkh line dh equation ;k points fn, gksaxs mlosQ nks co-
(3) Second quadrant esa x (–ve) rFkk y (+ve) gksrk gSA
ordinates gksax s ftudks ge (x, y) os Q form esa represent
(4) Third quadrant esa x rFkk y nksuksa (–ve) gksrs gSaA
djrs gSaA
(5) Fourth quadrant esa x (+ve) rFkk y (–ve) gksrk gSA
Co-ordinate system : Co-ordinate system dk Ex. In which quadrant the following points do
formation gksrk gS tc nks axis (i.e. x-axis rFkk y-axis) ,d lie ?
nwljs dks 90° ds dks.k ij dkVrh gSA Quadrants dks pkj cjkcj fdl quadrant esa fn, x, points vkrs gSa \
Hkkxksa esa divide fd;k tkrk gSA bu quadrants dks first, second, (a) (–6, 4) (b) (2, –5)
third rFkk fourth quadrant osQ uke ls Hkh tkuk tkrk gSA tSlk (c) (–7, – 3) (d) (1, 2)
figure esa n'kkZ;k x;k gSA Sol. (a) (–6, 4) : 2nd quadrant system esa]
Y (b) (2, –5) : 4th quadrant system esa]
Ordinate
(c) (–7, – 3) : 3rd quadrant system esa]
II 1
Quadrant 3 Quadrant (d) (1, 2) : 1st quadrant system esa]
(–,+) 2 (+, +) Applications of co-ordinate geometry. Co-
1
ordinate geometry dh help ls ge]
X' X
–3 –2 –1
0
1 2 3 (i) nks points osQ chp dh nwjh fudky ldrs gSaA
abscissae
–1
III IV (ii) ;fn fdlh line dk x-axis osQ lkFk slope rFkk passing
–2
Quadrant Quadrant through point fn;k gks rks line dh equation fudky
–3 (+,–)
(–, –) ldrs gSaA
Y' (iii) Triangle dk area fudky ldrs gSaA
Region Quadrant Nature of x and y Sign of co-ordinate (iv) fn, gq, points ,d line ij gS ;k ugha i.e. collinear
XOY I x > 0, y > 0 (+,+) gS fd ugha check dj ldrs gSaA
YOX' II x < 0, y > 0 (–, +)
X'OY' III x < 0, y < 0 (–,–) (v) nks parallel lines osQ chp dk distance rFkk fdlh fn,
Y'OX IV x > 0, y < 0 (+,–) gq, point ls line dk distance izkIr dj ldrs gSaA
y' (vi) Lines parallel gS] perpendicular gS] inter-secting
gS etc. check dj ldrs gSaA
2 (1,2)
(vii) nks lines osQ chp dk angle fudky ldrs gSaA
1
x' o x (viii) lines dks vyx&vyx form esa represent dj ldrs gSaA
1
Co-ordinate Geometry osQ nks eq[; i¼fr;ksa esa ck¡Vk
tk ldrk gSµ
y' (1) dkrhZ; fu;ked (Cartesian Co-ordinates)
Coordinate (x,y) = (1,2) (2) /qzoh; fu;ked (Polar Co-ordinates)

BLAM–91
funsZ'kkad T;kfefr
Cartensian Co-ordinates (dkrhZ; fu;ked) dks (x, y) 3 3 180
Sol. (i) Here, r = 5, = = 45
osQ form esa rFkk Polar Co-ordinates (/zqoh; fu;ked) dks (r, 12 12
) osQ form esa fy[kk tkrk gSA (Ist Quadrant)
Relation between the polar and Cartensian Co- Now,
ordinates :— x = r cos = 5 cos 45°
(i) x = r cos , y = r sin 1 5 2
=5× = .
2 2 2 2
(ii) r= x y and = , – , – or ( + )

and – ;k (2 – ) fcUnq (x, y) osQ vuqlkj dk 1 5 2


and y = r sin = 5 sin 45° = 5
eku gksxk fiQj pkgs (x, y) ftl Quadrant esa vofLFkr 2 2
gksA
F5 2 ,5 2I
tgk¡ tan
y
and0 < -
Cartesian form will be GH 2 2 JK
x 2
(ii) Here, r = 4, = 60° (Ist Quadrant)
Ex. Find the polar co-ordinates of the points
whose cartesian co-ordinates are. 1
x = 4 cos 60° = 4 × 2
dkrhZ; funsZ'kkad esa fcUnqvksa dks /qzozh; funsZ'kakd esa Kkr djsa 2

(i) (–1, –1) (ii) e j


3, 1
y = 4 sin 60° = 4 ×
3
2 3
2
Sol. (i) Here, x = –1, y = –1

r= b 1g b 1g
2 2
2
Cartesian form will be 2, 2 3 e j
Distance Formula : Distance formula dk iz;ksx
1
tan = 1 tan 45 djosQ ge fdUgha nks points osQ chp dh nwjh fudky ldrs gSaA
1
P(x1,y1) Q(x2,y2)
 x, y nksuksa negative gSa
fcUnq (x, y) 3rd Quadrant esa gksxk Distance formula osQ }kjk]
= + = 180 + 45° = 225°
PQ bx 2 x1 g by
2
2 y1 g
2

polar form will be e 2 , 225 . j Note :– Origin (ewy fcUnq) ls fdlh fcUnq (x, y) dh nwjh
(ii) Here, x = 3, y 1 (distance) d x2 y2

2 Important uses of Distance formula :


r= e 3 j b1g 2
=2
Distance formula dk use djosQ ge
(a) nks points osQ chp dh nwjh fudky ldrs gSaA
3
tan = 3 = tan 60° (b) fdlh Hkh triangle, quadrilateral ;k polygon dh
1
sides dh length fudky ldrs gSaA
= 60°
(c) ;g irk dj ldrs gSa fd fn;k gqvk triangle ,d right
Polar form will be (2, 60°)
angled triangle gS ;k isosceles triangle gS ;k
Ex. Find the Cartensian co-ordinates of the points
whose polar co-ordinates are– equilateral triangle gS] bR;kfnA
/zqohZ; funsZ'kkad osQ fcUnqvksa dks dkrhZ; funsZ'kkad esa Kkr djsaµ (d) ;g irk dj ldrs gSa fd fn;k gqvk quadrilateral ,d
Square, rectangle, parallelogram bR;kfn gS fd
FG 5, 3 IJ ughaA
(i) H 12 K (ii) (4, 60°)

BLAM–92
funsZ'kkad T;kfefr
Note : Distance dh unit : metre, centimetre ;k Internal section formula : ;fn dksbZ point P (x,y)
fliZQ unit/ bdkbZ gksrh gSA fdlh line segment A (x1, y1) rFkk B (x2, y2) dks internally
(e) m : n ratio esa divide djrh gS rks point P os Q co-ordinates
gksaxsA
A C B P( x,y)
;fn AC + BC = AB A B
m : n
rks A, B, C Collinear (lajs[k) gksxsa (x1,y1) (x2,y2)
(f) ;fn
mx 2 nx1 my 2 ny1
x , y
(i) AB = BC = AC rks m n m n
f=kHkqt] leckgq f=kHkqt (equilateral triangle) gksxkA External section formula : ;fn dks b Z point
A ( x1 y1) P (x, y) fdlh line segment A (x1, y1) rFkk B (x2, y2) dks
externally m : n ratio esa divide djrh gS rks

A B (x2,y2) P ( x, y)
( x1, y1)
AP : BP = m : n

mx 2 nx1 my 2 ny1
x y
B (x2, y2) C (x3, y3) m n m n

(ii) dksbZ nks Hkqtk cjkcj rFkk rhljk cjkcj ugha gks vFkkZr~ AB Important Points :
= AC BC rks lef}ckgq f=kHkqt (ISO Sceles triangle) z x-axis ij y-co-ordinate zero gksrk gS i.e, fdlh point
gksxk dk co-ordinate x-axis ij gksxk (a, o)
(iii) lcls cM+h Hkqtk dk oxZ 'ks"k nks Hkqtkvksa osQ oxks± osQ ;ksx gS z y-axis ij x-co-ordinate zero gksrk gS i.e, fdlh point
cjkcj gks vFkkZr~ AB 2 + BC2 = AC2 rks ledks.k (Right dk co-ordinate y-axis ij gksxk (o, b)
angled triangle) gksxkA
z x = a represent djrk gS line tks y-axis osQ parallel ;k
(g) ;fn
x-axis ij perpendicular gSA
(i) AB = BC = CD = AD
rFkk AC = BD rks ABCD ,d oxZ (Square) gksxkA Y
a x=a

(x1,y1) (x4,y4) (a, o)


Xc X
A D
p

Yc

z y = b represent djrkgS line tks fd x-axis osQ parallel


;k y-axis ij perpendicular gSA

B C Y
(x2,y2) (x3,y3) (o, b)
p y=b

(ii) AB = BC = CD = AD rFkk b
Xc X
AC BD rks ABCD ,d leprqHkZqt (Rhombus)
gksxk
Yc
(iii)AB = CD, BC = AD rFkk AC = BD rks ABCD
vk;r (Rectangle) gksxkA Ex. 1. A line passing through x-axis i.e 2x + 3y = 12.
then Find the Co-ordinate of the point. / js[kk
(iv) AB = CD, BC = AD rFkk AC BD rks ABCD
2x + 3y = 12, x v{k ls gksdj tkrh gS rks ml fcUnq osQ
lekarj prqHkqZt (Parallelogram) gksxkA
funsZ'kkad Kkr djks
BLAM–93
funsZ'kkad T;kfefr
Sol. w.k.t Ex.1. Wh at will be th e sl ope of li ne h avi ng
Line x-axis ls pass gks jgh gSA coordinates (2,3) and (6,9) ?
y=0 js[kk dh <ky D;k gksxh ftlosQ funsZ'kkad (2,3) rFkk (6,9) gS \
2x + 3y = 12 1 5
2x + 3 (0) = 12 (1) (2)
2 2
2x = 12 x 6 3
(3) (4) 1
Co-ordinate = (6,0) 2
Ex.2. A line 3x – 5y = 15 passing through y-axis then Sol. (3) ge tkurs gSa slope of line
find the Co-ordinate of the point ?
y2 y1
js[kk 3x – 5y = 15, y-v{k ls gksdj xqtjrh gS] ml fcUnq ds m= x
2 x1
funsZ'kkad Kkr djsa \
9 3 6 3
Sol. Line y-axis ls pass gks jgh gS = = =
6 2 4 2
x=0
or,
3 × 0 – 5y = 15
coff . of x 3
y 3 Slope (m) = coff . of y = 3
1
Co-ordinate = (0, –3)
Ex.2. Slope of line 3 x + y + 5 = 0 will be ?
Slope of a line : fdlh Hkh line osQ slope dks m ls
n'kkZ;k tkrk gSA slope fudkyus osQ eq[;r% 4 rjhosQ gksrs gSaA 3x+y+5=0 js[kk dh <ky D;k gksxh \
(i) ;fn angle ( ) fn;k gks rks slope m tan (1) 3 (2) 3
1 1
Y (3) (4)
3 3
Sol. (1) 3 x + y + 5 = 0 dks y = mx + c ls compare djus ij
T
Xc X y = – 3x 5
y = mx + c
Slope, m = – 3
Yc
or
(ii);fn line osQ xqtjus okys nksuksa points (x1, y1) rFkk (x2,
coff. of x 3
y2 y1 Slope (m) = 3
y 2) fn, x, gks rks slope, m coff. of y 1
x2 x1
Conditions of perpendicularity and parallelism
Y P Q (x2, y2) * nks lines ftuosQ slopes m1 rFkk m2 gSaA
(x 1, y1) (i) ,d&nwljs ij perpendicular gksaxh ;fn

Xc X m1 m 2 1

m1
Yc

(iii) ;fn line dh equation A x + B y + C = 0 nh xbZ m2


gks rks
b d k c o - e ffic ien t g
 x A
(ii) ijLij ,d&nwljs dh parallel gksxha ;fn m1 m 2
slo p e =
b y d k c o - effic ien t g B

(iv) tc line dh equation y = m x + c osQ form esa nh


1

gks rks
m

2
m

slope m

BLAM–94
funsZ'kkad T;kfefr
Note:– Angle between two lines :
The lines a1x + b1 y = c1 and a2x + b2y = c2 are ;fn nks lines osQ chp angle ( ) gS rks]
a1 b1 c1 Fm m I
1 2
(i) Parallel, If a
2 b2 c 2 or, m1 = m2, where m tan = GH 1 m m JK 1 2
is slope.
(ii) Perpendicular, If m1.m2 = –1 or, a1 a2 + b1b2 = 0
Y
a1 b1 c1 C B
(iii) Coincident, If a b2 c2
2

Ex. 1. The value of k for which the lines 3 x + y + 1


= 0 and kx + 2y + 5 = 0 are perpendicular :
K ds fdl eku ds fy, 3 x + y + 1 = 0 rFkk kx + 2y X
O
+5=0 ,d nwljs ij yEcor gSa \
2 3 A D
(1) (2)
3 2
Note :
2 2
(3) (4) (i) ;fn tan dk value positive gS rks U;wudks.k (Acute
3 3
Angles) gksxk rFkk ;fn tan dk value negative gS rks
Sol. (4) ge tkurs gSa tc dksbZ nks lines ,d nwljs ij perpen- vf/ddks.k (Obtuse Angle) gksxkA
dicular gksrh gSa] rks m1 × m2 = –1
(ii) ;fn nks lines a1x + b1 y + c1 = 0 rFkk a2x + b2y + c2 =
k 0 gSa rFkk bu lines osQ chp dks.k gks rks
;gk¡, m1 = 3 , m2 = 2
a 2b1 a1b2
m1 × m2 = – 1 tan = aa
1 2 b1. b2
k
3 1
2 Ex. 1. What will be the angle between the lines – 3x

2 + y – 1 = 0 and x – 3 y + 2 = 0 ?
k
3 – 3x+y–1=0 rFkk x – 3 y + 2 = 0 js[kkvksa osQ chp
Ex. 2. For what value of k the given lines x – 2y + 3 = fdruk dks.k gksxk \
0 and 2x – ky + 5 = 0 are parallel ? (1) 45° (2) 60°
k osQ fdl eku osQ fy, nh xbZ js[kk,¡ x – 2y + 3 = 0 rFkk (3) 30° (4) 90°
2x – ky + 5 = 0 ijLij lekarj gSa \ Sol. (3) Lines dh equations gS
(1) k = 4 (2) k = 2
(3) k = 3 (4) k = 1 – 3x+y–1=0 rFkk x – 3y+2=0
Sol. (1)ge tkurs gSa] 3 1 1
m1 = , m2 =
tc nks Lines parallel gksrh gSa] rc 1 3 3
m1 = m2 ge tkurs gSa]
;gk¡, nks lines osQ chp dk angle,
1
m1 =
2
rFkk m1 m 2
tan =
1 m1 m 2
2
m2 =
k 1
3
nh xbZ Lines parallel gSa] blfy, 3 3 1
m1 = m2 = 1 1 =
3 2 3
3
1 2
2 k 1
k 4 tan = = 30°
3

BLAM–95
funsZ'kkad T;kfefr
Equation of line : General form esa line dh equation (4) Equation of line in intercept form : ;fn dksbZ line
gksrh gS Ax + By + C = 0 tgk¡ x- axis dks a point ij rFkk y- axis dks b point ij
A = x dk co-efficient dkVrh gS rks line dh equation gksxh
B = y dk co-efficient x y
1
C = Constant term a b

Y Y
Ax
+
By (0, b)
+C B
=0
Xc X (a,0)
Xc X
O A

Yc Yc

;fn C = 0, gks rks line (0,0) origin ls gksdj xqtjrh gSA


vxj fdlh line dks intercept form i.e. x y
1 ds
a b
Y form esa represent djrs gSa rks
1
Xc X
Triangle dk Area = 2
a b gksrk gSA
Ax
+ (5) Equation of line in normal/perpendicular form
By
= 0 : Normal / perpendicular form esa line Ax + By + C
Yc = 0 dh equation gksrh gS
Note : ;fn ;g check djuk gks fd nh xbZ equation origin A B C
x y 0
(0,0) ls xqtjrh gS dh ugha rks equation esa x rFkk y dh txg 0 A 2
B 2
A 2
B 2
A 2
B2
j[ksa ;fn L.H.S = R.H.S. = 0 gks rks equation origin ls pass (6) Equation of line in parametric form : Parametric
gksxhA e.g. 2x – 5y = 0 ; 3x + 2y = 0 etc. form esa line dh equation gksrh gS
;gk¡ C = 0 gS blfy, line origin ls pass gksxhA
y y1 x x1
Various forms of equation of line. =r
cos sin
(1) Slope–form or Tangent-form :– Slope-form esa
Examples :
line dh equation gksrh gS Ex.1. The equation of line which passes through (1,2)
y mx c and having slope 3 is
js[kk dh lehdj.k tks (1,2) fcUnq ls gksdj xqtjrh gS rFkk
tgk¡] m = slope bldh <ky 3 gS] gksxh
c = y – axis ij line dh nwjh origin (0,0) ls (1) y + 3x + 1 = 0
(2) Equation of line in one point form : One point (2) 3x – y –1 = 0
form esa line dh equation gksrh gS (3) x – 2y + 1 + 1 = 0
(4) x + 2y + 1 = 0
y y1 m (x x1 )
Sol. (2) one point form esa line dh equation gksxh
tgk¡] m = slope y – y1 = m (x – x1)
(x1, y1) line osQ passing through points gksrs gSaA ;gk¡, x1 = 1, y1 = 2, m = 3
(3) Equation of line in two point form : ;fn dksbZ line y – 2 = 3 (x – 1)
(x1, y1) rFkk (x2, y2) points ls gksdj xqtjrh gS rks line dh y – 2 = 3x – 3
equation gksxh y = 3x – 3 + 2

3x y 1 0
y y1 x x1
y 2 y1 x 2 x1 Ex.2. The equation of line which passes through (1,2)
and (5,4) is

BLAM–96
funsZ'kkad T;kfefr
ml js[kk dh lehdj.k D;k gksxh tks dh (1,2) rFkk (5,4) 3x 4y
1
fcUnq ls gksdj xqtjrh gS\ 12 12
(1) x – 3y + 4 = 0 (2) x – 2y + 3 = 0
x y
(3) x + 4y – 2 = 0 1
4 3
(4) 3x – 2y + 5 = 0
a = 4, b = 3
Sol. (2) We know that,
1
Line dh equation two point form esa gksxhA Area of triangle = 4 3
2
y y1 y2 y1 = 6 sq. units
x x1 x2 x1
(1) Distance of a line from any point p (x1, y1) : fdlh
;gk¡ Hkh line Ax + By + C = 0 dh nwjh (perpendicular
x1 = 1, y1 = 2 distance) fdlh point p (x1, y1) ls fudkyus os Q fy,
x2 = 5, y2 = 4 equation osQ (x, y) point dks x1, y1 ls replace dj osQ
y 2 4 2 iwjh equation dks A 2 B2 ls divide djrs gSa rFkk ;g
x 1 5 1
process mod osQ vanj djrs gSa
y 2 2
x 1 4 p (x1, y1)
2y – 4 = x – 1
x – 2y + 3 = 0
Ex. 3. 2x + 3y + 6 = 0 can be represented in intercept
Ax + By + C = 0
form as :
2x + 3y + 6 = 0 dks Intercept form esa fdl izdkj fy[kk
Ax1 By1 C
tk ldrk gS\ Distance , d =
2 2
A B
x y x y
(1) 1 (2) 1
3 2 2 3 (2) nks parallel lines osQ chp dk distance : A1x + B1 y +
C1= 0 rFkk A1x + B1 y + C2= 0 nks parallel lines gS
x y x y
(3) 1 (4) 1 D;ksafd buosQ slope cjkcj gS] nksuksa osQ chp dh nwjh gksxh
2 3 2 3
Sol. (1) Equation of line is 2x + 3y + 6 = 0
2x + 3y = – 6 C2 C1
d
–6 ls nksuksa rjiQ divide djus ij A12 B12
2x 3y
1 nks intersecting lines osQ chp dh nwjh zero gksrh gSA
(3)
6 6
Area of triangle : fdlh triangle ftlosQ vertices osQ
x y
1 co-ordinates (x1, y1), (x2, y2) rFkk (x3, y3) gS dk area gksxk
6 6
2 3 x1 y1 1
1
Area of triangle = x 2 y2 1
x y 2
1 x 3 y3 1
3 2
Ex. 4. Find the area of triangle made by 3x + 4y = 12
with x – axis & y–axis. A (x1, y1)
x– axis rFkk y–axis ls line 3x + 4y = 12 }kjk cuk;k x;k
triangle dk area D;k gksxk ?
(1) 6 sq. units
(2) 12 sq. units
(3) 4 sq. units
(4) 18 sq. units
Sol. (1) Equation of line is B (x2, y2) C (x3,y3)
3x + 4y = 12

BLAM–97
funsZ'kkad T;kfefr
1 bu pkjks lines ls 6 2 side dk square cuk gS] bldk
= [x (y –y ) – x2 (y1–y3) + x3 (y1–y2)
2 1 2 3 area = AB2
If |x| + |y| = a then area = 2a2 2

Condition of collinearity of three points:– = 6 2e j


If area of ABC = 0 then three points A, B and C = 72 Sq. Units.
will be collinear. Aliter :
x1 (y2 – y3) + x2 (y3 – y1) + x3 (y1 – y2) = 0. Area = 2a2
Examples : = 2 × 62
Ex.1. Find the area of triangle which passes through = 72 Sq. Units.
(1,2) (3,4) and (6,4). Ex.3. Jinni, Rashmi and Neha are playing together.
f=kHkqt dk {ks=kiQy Kkr djsa tks fd (1,2), (3,4) rFkk (6,4) They sat at three different corners of a triangular
fcUnq ls gksdj xqtjrk gS\ field having vertices (2,– 4), (6,4) and (5,–2). Find
(1) 3 Sq. Units the area enclosed between them.
(2) 4 Sq. Units ftUuh] jf'e rFkk usgk ,d lkFk [ksy jgh gSaA og fdlh
(3) 2 Sq. Units f=kHkqtkdkj eSnku osQ vyx&vyx dksuksa ij cSBh gSa ftlosQ
(4) 5 Sq. Units funsZ'kkad gS (2,– 4), (6,4) rFkk (5,–2) muosQ chp esa cus
Sol. (1) Area of triangle f=kHkqt dk {ks=kiQy Kkr djsa \
x1 y1 1 1 2 1 (1) 7 Sq. Units
1 x2 y2 1
= 1 = 3 4 1 (2) 8 Sq. Units
2 x3 y3 1 2 6 4 1 (3) 4 Sq. Units
Area of (4) 6 Sq. Units
1 Sol. (2) Triangle RJN dk area
=
2
14b 4 g 2 b3 6g 1b12 24 g
R (6,4)
1
= 6  12
2
= 3 Sq Units
( Area ges'kk positive gksrk gSA)
Ex.2. Find the area of the figure bounded by |x| + |y| =
J N
6. (2,–4) (5, –2)
|x| + |y| = 6 ds }kjk cus figure dk area D;k gksxk ?
(1) 36 Sq. Units (2) 70 Sq. Units 6 4 1
(3) 72 Sq. Units (4) 108 Sq. Units 1
= 2 –4 1
2
Sol. (3) |x| + |y| = 6 nh xbZ lines dks represent djrk gS] 5 –2 1
x+y=6 1
–x+y=6 = 6 (–4 + 2) – 4 (2 –5) + 1 (–4 + 20)
2
x–y=6
1
–x – y = 6 = –12 + 12 + 16
2

A (0, 6) = 8 Sq. Units


Ex. 4. For what value of k the following points (1,2),
62 (3,k), (5,6) are collinear.
6
(–6, 0) B
k osQ fdl eku osQ fy, fcUnq (1,2), (3,k) rFkk (5,6) lajs[k
D 6 (6,0) gksaxs \
(1) 2 (2) 3
(3) –1 (4) 4
C (0,–6) Sol. (4) ge tkurs gSa] tc rhu points collinear gksrs gSa] rks
triangle dk area zero gksrk gSA
BLAM–98
funsZ'kkad T;kfefr

1
1 2 1 Sol. (1) Distance formula osQ }kjk
0= 3 k 1 A (1,2)
2 5 6 1
1
= 1 (k – 6) – 2 (3 – 5) + 1 (18 – 5 k) c
2 b
1
= k – 6 + 4 + 18 – 5 k
2 C
B
– 4k + 16 = 0 (2,1) a (3,2)
– 4k = – 16
AB = c = (2 1)2 (1 2)2 = 1 1
k 4

* ;fn A = (x1, y1) rFkk B = (x2, y2) vkSj P(x, y), AB dk c 2


Mid point (eè; fcUnq) gks rks
BC = a = (3 2) 2 (2 1) 2 = 1 1
x1 x2
x
2
a 2
y1 y2
y
2 CA = b = (3 1)2 d2 i
2 )2 = 4

A P B b 2
( x1, y1) (x, y) (x2, y2)
Co-ordinates of incentre
* Co -ordinates of In-centre : fdlh Hkh circle dk
centre tks dh fdlh triangle ds vanj mldh rhuksa side
FG ax bx
1 2 cx 3 ay1 by 2 cy3
,
IJ
= H a b c a b c K
dks touch djrs gq, cuk;k tkrk gS] rks mls in centre
dgrs gSaA F 1 2 2 2 2 3 , 2 2 2 1 2 2I
=GH 2 2 2 2 2 2
JK
A (x1, y1)
F 4 2 4 , 4 2 2 I F 2e 2 1j 2 2 1I
c b = G
H 2 2 2 2 2 2 JK = GGH e 2 1j , 2 1 JJK
B
a
C F 2 2 1I
= G 2,
H 2 1 JK
(x2, y2) (x3, y3)

I=
LM ax bx 1 2 cx 3 ay1 by 2 cy3
,
OP Centroid of a triangle
N a b c a b c Q fdlh Hkh triangle esa tgk¡ medians ,d nwljs dks intersect
Ex. 5. The incentre of a triangle with vertices having djrh gSa og triangle dk centrioid gksrk gSA
co-ordinates (1,2), (2,1) and (3,2) will be Note : fdlh Hkh triangle dk centroid, medians dks
fdlh f=kHkqt osQ dks.kksa osQ funsZ'kkad Øe'k% (1,2), (2,1) rFkk 2 : 1 ratio esa divide djrk gSA
(3,2) gS] f=kHkqt osQ vUr% fcUnq osQ funsZ'kkad D;k gksaxs \ Centroid osQ co-ordinates gksrs gSaA
F 2 2 1 I x1 x2 x3 y1 y2 y3
(1) 2, GH J
2 1 K
x
3
,y
3

1 1 A (x1, y1)
(2) ,
2 1 2 1

3 2 2
(3) , 2
2 1 2 1 G
1 C 1 (x ,y )
2 2 2
(4) ,
2 1 2 1 B C
(x2, y2) (x3, y3)

BLAM–99
funsZ'kkad T;kfefr
Examples : x1 x2 x3
Ex. 1. ;fn fdlh triangle osQ centroid rFkk fdUgha nks vertices x
3
osQ co-ordinates (2, 4), (3, 5), (–1,2) gks rks] rhljs
y1 y2 y3
vertex osQ co-ordinates gksaxsA y rFkk
3
(1) (4, 5) (2) (5, 4) (3) (–4,–5) (4) (–4, 5)
2a 4a 6a 4a
Sol. (1) ge tkurs gSa] x
3 3
co-ordinates of centriod
3b 5b 7b 5b
y
FG x 1 x2 x 3 y1
,
y2 y3 IJ 3 3
H 3 3 K FG 4a , 5b IJ
ekuk rhljs vertex osQ co-ordinates (x, y) centroid osQ coordinates gksaxs H 3 3K
Some Important Results
A (3,5)
(i) ;fn fcUnq (p, q) fcUnqvksa (a + b, b – a), (a – b, a + b) ls
leku nwjh (equalidistance) ij gks rks bp = aq
(ii) ;fn rhu fcUnq (a, b), (a1, b1) rFkk (a –a1 , b – b1) ,d
G a b
4) js[kk esa (collinear) gks rks a b
( 2,
1 1
(iii) ;fn rhu vlajs[k fcUnq (Nonlinear Point) A (0, 0), B
(x1, y1) rFkk c (x2, y2) gks rks
B (–1,2) C (x,y)
1
area of =
2
b
x1x 2 x 2y1 g
3 1 x
2
3
(iv) ;fn fcUnq (0, 0), (0, b) rFkk (1, 1) lajs[k (collinear) gks
1 1
x=6–2 rks a 1 gksxk rFkk ;fn fcUnq (a2, 0), (0, b2) rFkk
x=4 b
1 1
5 2 y (1, 1) lajs[k (collinear) gks rks 1
4= a 2 b2
3
(v) ,d ljy js[kk fu;kedk{kksa (axes) ij a rFkk b var% [kaM
y = 12 – 7
(Intercept) dkVrh gS rFkk ewyfcUnq (origin) ls ykfEcd
y=5
1 1 1
rhljs Vertex osQ co-ordinates gksaxs (4,5) (perpendicular) nwjh P gS rks P 2 a 2 b 2
Ex. 2. ;fn fdlh triangle osQ vertices osQ co-ordinates (iv) fcUnqvksa (a, b) vkSj (a1, b1) dks feykus okys js[kk[k.M osQ
(2a, 3b), (–4a, –5b), rFkk (6a, 7b) gks rks] centriod osQ yEc v¼Zd (right bisector) dk lehdj.k 2x (a – a1)
co-ordinates gksaxsA
+ 2y (b – b1) = a2 – a12 + b2 – b12
FG 2a , 4b IJ FG 4a , 5b IJ ‡‡‡
(1) H 3 3K (2) H 3 3K
(4) GH
F 2b , 5b IJ
3 3K
(3) (a, 3b)

Sol. (2) ekuk centriod osQ co-ordinates gksaxs ( x, y) fdj.k izd k'ku dh
A (2a, 3b)
mi;ksx h iq Lrdsa ,oa if=kdkvksa
dks ?kj cSBs vklkuh ls
iz kIr djus ds fy,
G
gekjh os clkbV ij ykWx vkWu djsa
www.kiranprakashan.com

B (–4a, –5b) C (6a, 7b)

BLAM–100
funsZ'kkad T;kfefr

PREVIOUS YEARS’ QUESTIONS

1. The value of x, if the slope of the line joining 1 43


(1) units (2) units
13 13
FG –4 IJ
(–8, 11), (2, x) is H3K (3)
34
units (4)
1
units
13 5
x dk eku Kkr djsa] ;fn nksuks fcanqvksa (–8, 11) rFkk (2, x) dks
7. What is the distance of the point (2, 3) from the
tksMu+ s ls izkIr js[kk dk <ky
FG –4 IJ gks\ line 2x + 3y + 4 = 0 ?
H3K
fcanq (2,3) dh nwjh fdlh js[kk ftldh lehdj.k 2x + 3y + 4
(1)
–7
(2)
7
(3)
1
(4)
5 =0 gS ls fdruh gksxh\
3 3 3 3
15 16
2. What is the slope between the lines (1) units (2) units
13 13
y – 3x – 5 0 and 3y – x + 6 = 0
17 8
js[kk lehdj.kksa y – 3x – 5 0 rFkk 3y – x + 6 = 0 (3) units (4) units
13 13
ds chp dk <ky D;k gksxk\ 8. What is the distance between the parallel lines
1 2 15x + 8y – 34 = 0 and 15x + 8y + 31 = 0 ?
(1) 1 (2) (3) (4) 3 nks lekarj js[kkvksa 15x + 8y – 34 = 0 rFkk 15x + 8y + 31
3 3
= 0 ds chp dh nwjh fdruh gksxh\
3. Slope of a line which cuts off intercepts of equal
lengths on the axis is (1) 2 units (2) 5 units

js[kk dh <ky D;k gksxh tks nksuks v{kksa x rFkk y dks leku nwjh 65
(3) 6 units (4) units
ij foHkkftr djrh gS\ 17
(1) 1 (2) 2 (3) –1 (4) 3 9. What point on the x-axis are at a distance of 4
4. What will be the angle between the lines y – x – 7 units from the line 3x – 4y – 5 = 0
= 0 and 3y – x + 6 = 0 ? x&v{k ij og fcanq D;k gksaxs tks fd js[kk 3x – 4y – 5 = 0
ls 4 bdkbZ dh nwjh ij fLFkr gS\
js[kkvksa y – x – 7 = 0 rFkk 3y – x + 6 = 0 ds chp dk
dks.k fdruk gksxkA FG 1 , 0IJ FG 0, 25 IJ
(1) H3 K (2) H 3K
(1) e
tan –1 2 3 j (2) e
tan –1 2 – 3 j
FG 25 , 0IJ
(3) e
tan –1 1 3 j (4) e
tan –1 1 – 3 j (3) (5, 1) (4) H3 K
5. The angle between the graph of the linear equa- 10. The distance between the lines y = mx + c1 and y
tion = mx + c2 is
239x – 239y + 5 = 0 and the x – axis is
nksuks js[kkvksa y = mx + c1 rFkk y = mx + c2 ds chp dh nwjh
jSf[kd lehdj.k 239x – 239y + 5 = 0 vkSj x – v{k ds xzkiQ D;k gksxh\
ds chp dks.k fdrus va'k dk gksxk \
(1) 0° (2) 60° (3) 30° (4) 45° c1 – c 2 c1 – c 2
(SSC CAPFs SI, CISF ASI & Delhi Police SI
(1) (2)
Exam, 21.06.2015 (Ist Sitting) TF No. 8037731) m2 1 1 m2
6. The distance of the point (3, – 1) from the line12x
– 5y – 7 = 0 will be ?
c 2 – c1
lehdj.k 12x – 5y – 7 = 0 dh fcanq (3, –1) ls nwjh fdruh (3) (4) 0
1 m2
gksxh\

BLAM–101
funsZ'kkad T;kfefr
11. A point equidistant from the lines 4x + 3y + 10 18. If the ordinate and abscissa of the point (k, 2k –
= 0, 5x – 12y + 26 = 0 and 7x + 24y – 50 = 0 is ? 1) be equal, then the value of k is
fuEu fodYiksa esa ls nh xbZ dkSu lh fcanq js[kkvksa 4x + 3y + 10 ;fn fcanq (k, 2k –1) dk Hkqt vkSj dksfV cjkcj gksa] rks k dk
= 0, 5x – 12y + 26 = 0 rFkk 7x + 24y – 50 = 0 ls leku eku D;k gksxk \
nwjh ij gksxh\ (1) 0 (2) – 1

(1) (1, –1) (2) (1, 1) (3) (0, 0) (4) (0, 1) 1


(3) 1 (4)
12. The graph of x + 2y = 3 and 3x – 2y = 1 meets the 2
Y–axis on two points. The distance between them (SSC CGL Tier-II Exam. 12.04.2015 TF No. 567 TL 9)
is. 19. A point in the 4th quadrant is 6 unit away from
x + 2y = 3 vkSj 3x – 2y = 1 ds xzkiQ Y-v{k dks nks fcUnqvksa x–axis and 7 unit away from y–axis. What are
ij feyrs gSa ftuds chp nwjh gSµ the co-ordinate of point ?

8 4 pkSFks prqFkk±'k esa ,d fcanq x–v{k ls 6 ;wfuV nwj gS vkSj y–v{k


(1) units (2) units ls 7 ;wfuV nwj gSA fcanq ds funsZ'kkad D;k gSa\
3 3
(3) 1 units (4) 2 units (1) (7, –6) (2) (–7, 6) (3) (–6, –7) (4) (–6, 7)
(SSC GL Tier-II Exam. 04.09.2011) (SSC CGL Tier-I Re-Exam, 30.08.2015)

13. If (2, 0) is a solution of the linear equation 2x + 20. Find the equation of the line which passes
3y = k, then the value of k is through the point (2, 2) and makes an angle of
45° with x-axis ?
;fn jSf[kd lehdj.k 2x + 3y = k dk gy (2, 0) gS] rks k dk
fcanq (2,2) ls xqtjus okyh ml js[kk dk lehdj.k Kkr djsa tks
eku crkb,A
(1) 6 (2) 5 (3) 2 (4) 4 x-v{k ds lkFk 45° dk dks.k cukrh gSa\
(SSC CHSL (10+2) LDC, DEO & PA/SA Exam, 20.12.2015 (1) x + y = 2 (2) x – y = 0
(Ist Sitting) TF No. 9692918) (3) 2x + y = 3 (4) x – 2y = 4
x y 21. What is the equation of a line which has y-in-
14. If the line 1 passes through the points
a b tercept 2 and is inclined at 60° to the x-axis.
(2, –3) and (4, –5), then (a, b) is js[kk dh lehdj.k D;k gksxh ;fn bldk y&v{k 2 rFkk x&v{k
x y ds lkFk 60° dk dks.k cukrh gks\
;fn js[kk a b 1 fcanqvkas (2, –3) rFkk (4, –5) ls gksdj
(1) y – 3x 2 (2) y x – 3
xqtjrh gS rks (a, b) dk eku D;k gksxk\
(1) (1, 1) (2) (–1, 1) (3) (1, –1) (4) (–1, –1) (3) y x 2 3 (4) y 3x 2
15. If the graph of the equations 3x + 2y = 18 and 3y 22. What will be the point on the x-axis, which is
– 2x = 1 intersect at the point (p, q), then the equidistant from the points (7, 6) and (–3, 4)
value of p + q is
x &v{k ij og fcanq D;k gksxk] ftldh nwjh (7, 6) rFkk (–3,
;fn lehdj.kksa 3x + 2y = 18 rFkk 3y – 2x = 1 ds ys[kkfp=k
4) fcanqvks ls leku gksxh\
ijLij (p, q) fcanq ij dkVrs gksa] rks p + q dk eku fdruk gksxk\
(1) (6, 0) (2) (–2, 0)
(1) 7 (2) 6 (3) 5 (4) 4
(SSC CHSL DEO & LDC Exam. 27.10.2013 IInd Sitting) (3) (4, 0) (4) (3, 0)
16. If the graph of the equations x + y = 0 and 5y + 7x 23. Equation of a line is taken as 3x – 4y + 5 = 0. Its
= 24 intersect at (m, n), then the value of m + n is slope and intercept on y-axis will be
;fn lehdj.kksa x + y = 0 rFkk 5y + 7x = 24 ds ys[kkfp=k ;fn js[kk ds lehdj.k dks 3x – 4y + 5 = 0 ds :i es n'kkZrs
ijLij (m, n) ij dkVrs gSa] rks m + n dk eku fdruk gksxk\ gSa] rks js[kk dh <+ky rFkk y&vUr%[k.M D;k gksxk\
(1) 2 (2) 1 (3) 0 (4) –1
(SSC CHSL DEO & LDC Exam. 10.11.2013, Ist Sitting) FG 3 , –5 IJ FG –3 , –5 IJ
17. The graphs of x = a and y = b intersect at
(1) H4 4 K (2) H 4 4K
x=a vkSj y = b ds xzkiQ fdl fcUnq ij ijLij dVsaxs \
(1) (a, b) (2) (b, a) (3) (–a, b) (4) (a, –b)
FG 3 , 5 IJ FG 5 , 3 IJ
(SSC CGL Tier-I Exam. 19.10.2014 (Ist Sitting)
(3) H 4 4K (4) H 4 4K
BLAM–102
funsZ'kkad T;kfefr
24. What is the equation of line, which makes inter-
cepts –5 and 2 on the x and y-axis respectively 31. If the angle between two lines is and the
4
ml js[kk dh lehdj.k D;k gksxh tks x&v{k rFkk y&v{k dks 1
–5 rFkk 2 ij dkVrh gks\ slope of one of the lines is , then the slope of
2
(1) 2x – 5y = 10 (2) –2x + 5y = –10 other line will be ?
(3) 2x – 5y = –10 (4) 5x – 2y = 10
25. What will be the equation of a line passing ;fn nks js[kkvksa ds chp esa 4 dk dks.k gS rFkk bues ls fdlh
1
through the point (–4, 3) and having slope ? 1
2 ,d js[kk dk <ky 2 gks] rks nwljh js[kk dk <ky D;k gksxk\
1
ml js[kk dh lehdj.k D;k gksxh ftldh <+ky 2 gks rFkk (1) m = 1 (2) m = 2 (3) m = 3 (4) m = 4

(–4, 3) fcanq ls xqtjrh gks\ 32. What is the equation of a line perpendicular to
(1) x – 2y + 5 = 0 (2) –x + 2y + 10 = 0 the line x – 7y + 5 = 0 and having x-intercept 3?
(3) x – 2y + 6 = 0 (4) x – 2y + 10 = 0 js[kk dh lehdj.k D;k gksxh tks dh fdlh vU; js[kk x – 7y+
26. What is the equation of a line passes through 5 = 0 ds yEcor gS rFkk x&v{k dks 3 bdkbZ ij dkVrh gS\
the point (3, 2) and makes an angle of 45° with
the line x – 2y = 3 ? (1) x + 7y = 21 (2) 7x + y = 21
fcanq (3, 2) ls gks dj xqtjus okyh ml js[kk dh lehdj.k D;k (3) x + 2y = 10 (4) –x + y = 15
gksxk tks dh x – 2y = 3 js[kk ds lkFk 45° dk dks.k cukrh gS\
33. Equation of line 3x y – 8= 0 can be repre-
(1) 3x – y – 7 = 0 (2) 3x + y = 7
(3) x – 3y = 7 (4) –3x + y = 6 sented in normal form as
27. Equation of the straight line parallel to x-axis
and also 3 units below x-axis is : lehdj.k 3 x y–8 0 dks yEc:i esa fdl izdkj fy[k
x-v{kds lkekukarj ljy js[kk] tks x-v{k ls 3 bdkbZ uhps gks] ldrs gSa\
dk lehdj.k dkSu&lk gksxk\
(1) x = – 3 (2) y = 3 (3) y = – 3 (4) x = 3 3x y 8
(1) – 0 (2) 3x y–4 0
(SSC GL Tier-I Exam. 21.04.2013, Ist Sitting) 2 2 2
28. What is the equation of a line, which passes
through the points (–1, 1) and (2, –4). 3x y 8
(3) 3x – y – 8 0 (4) – – 0
2 2 2
js[kk dh lehdj.k D;k gksxk ;fn og fcanqvksa (–1, 1) rFkk
(2, –4) ls xqtjrh gSa\ 34. Equation of line 3x + 2y – 5 = 0 can be written in
(1) 5x + 3y + 2 = 0 (2) –5x + 3y + 4 = 0 intercept form as
(3) 3x + 5y + 6 = 0 (4) 5x + 3y + 3 = 0 js[kh; lehdj.k 3x + 2y – 5 = 0 dks intercept form esa
29. If the vertices of a quadrilateral are A(– 2, 6), fdl izdkj fy[k ldrs gSa\
B(1, 2), C(10, 4) and D(7, 8). Then, the equation
of diagonal AC will be ? x y x y
(1) 5 1 (2) 1
fdlh prqHkZt
q ds 'kh"kZ fuEu izdkj ls gSa] A (– 2, 6), B(1, 2), 5 5 3
C(10, 4) rFkk D(7, 8) fod.kZ AC dh lehdj.k D;k gksxh\ 3 2

(1) x + 6y = 34 (2) x – 6y = 34 x y
x y – 1
(3) x + 2y = 1 (4) x – y = 13 (3) 1 (4) 5 5
2 5 3 2
30. What will be the equation of line which passes
through the point (–2, 3) and parallel to any oth- 35. What will be the equation of line for which p = 3
er line 3x – 4y + 2 = 0 and = 120° ?
ml js[kk dh lehdj.k D;k gksxh tks dh (–2, 3) fcanq ls xqtjrh ;fn p = 3 rFkk = 120° gks rks js[kk dh lehdj.k D;k gksxh\
gks lkFk gh lkFk fdlh vU; js[kk 3x – 4y + 2 = 0 ds lekarj
gks\ (1) x – 3y 6 (2) 3x y 6
(1) 3x – 4y + 18 = 0 (2) –3x + 4y + 12 = 0
(3) x – 3y + 10 = 0 (4) 2x + 3y + 6 = 0 (3) –x 3y 6 (4) x – 3y 5

BLAM–103
funsZ'kkad T;kfefr
36. What is the equation of a line parallel to 2x + 3y 42. In what ratio the line segment joining the points
(2, 3) and (4, 6) is divided by y-axis?
+ 4 = 0 and passing through the point (– 4, –5) ?
y&v{k] (2,3) rFkk (4,6) ls fey dj cuus okyh js[kk dks fdl
ml js[kk dh lehdj.k D;k gksxh tks dh fcanq (– 4, –5) ls
vuqikr esa foHkkftr djsxh\
xqtjrh gS rFkk vU; nwljh js[kk 2x + 3y + 4 = 0 ds lekraj gS\ (1) Internally 1 : 2
(1) 2x + 5y – 23 = 0 (2) –x + 5y = 20 (2) Externally 2 : 3
(3) 2x – 3y – 30 = 0 (4) 2x + 3y + 23 = 0 (3) Externally 2 : 1

37. Find the equation of a line which passes through (4) Externally 1 : 2
the point of intersection of lines x + 2y = 5 and 43. In what ratio, the line joining (–1, 1) and (5, 7) is
x – 3y = 7 and also passes through the point
divided by the line 2x + y = 4 ?
(0, –1) ?
(–1, 1) rFkk (5, 7) fcanqvksa dks feykus okyh js[kk [k.M dks
ml js[kk dh lehdj.k D;k gksxh tks fd js[kkvksa x + 2y = 5 2x + y = 4 jss[kk fdl vuqikr esa foHkkftr djsxh\
rFkk x – 3y = 7 ds izfrPNsn fcanq ls xqtjrh gS rFkk lkFk gh (0,
(1) 5 : 13 (2) 5 : 2 (3) 1 : 3 (4) 4 : 7
–1) fcanq ij ls Hkh xqtjrh gS\
44. A point R(h, k) divides a line segment between
(1) 3x – 29y + 1 = 0 (2) 3x – 29y – 29 = 0 the axes in the ratio 1 : 2 what will be the equa-
(3) 3x + 4y – 6 = 0 (4) –3x + 29y + 7 = 0 tion of line ?
38. If a linear equation is of the form x = k where k is ;fn dksbZ fcanq R(h, k) fdlh js[kk dks v{kksa ds chp esa
a constant, then graph of the equation will be 1 : 2 vuqikr ij foHkkftr djrh gS rks js[kk dh lehdj.k D;k
;fn ,d jSf[kd lehdj.k x = k ds :i esa gks vkSj k ,d vpj gksxh\
gks] rks ml lehdj.k dk ys[kkfp=k dSlk gksxk\
x 2y x 2y
(1) a line parallel to x-axis / x-v{k ds lekarj js[kk (1) – 4 (2) 3
h k h k
(2) a line cutting both the axes / nksuksa v{kksa dks dkVus
x y x y
okyh js[kk (3) 1 (4) 1
k h 2k h
(3) a line making positive acute angle with x-axis
x-v{k ij /ukRed U;wu dks.k cukus okyh js[kk 45. For what value of k, the following pair of lines
–kx + 2y + 3 = 0 and 2x + 4y + 7 = 0 are perpendic-
(4) a line parallel to y-axis / y-v{k ds lekarj js[kk
ular ?
(SSC CHSL DEO & LDC Exam. 28.10.2012 (Ist Sitting)
k dsfdl eku ds fy, js[kk,a –kx + 2y + 3 = 0 rFkk 2x +
39. Find the co-ordinates of the mid point of a line
4y + 7 = 0 ijLij ,d nwljs ij yEcor gSa\
segment joining the points (2, 4) and (6, 8) ?
(1) k = 2 (2) k = 4 (3) k = –1 (4) k = 3
(2, 4) rFkk (6, 8) dks feykus okyh js[kk ds eè; fcanq ds
46. For what value of k the line (k – 3)x – (4 – k2)
funsZ'kkad D;k gksax\s
y + k2 – 7k + 6 = 0 is parallel to x-axis ?
(1) (2, 6) (2) (4, 6) (3) (6, 4) (4) (–4, –6)
k dsfdl eku ds fy, js[kk (k – 3)x – (4 – k2)y + k2 – 7k
40. Find the ratio in which the line segment joining
+ 6 = 0 x&v{k ds lekarj gS\
the points (1, 2) and (4, 6) is divided by point
(1) k = + 1 (2) k = + 4
(2, 0).
fcanq (2, 0) fdUgh nks fcUnqvksa (1, 2) rFkk (4, 6) dks feykus (3) k = + 6 (4) k = + 2

okyh js[kk dks fdl vuqikr esa foHkkftr djsxh \ 47. What will be the co-ordinates of centroid of a
(1) 1 : 2 (2) 2 : 1 (3) 1 : 4 (4) 2 : 3 triangle whose vertices are A(1, 2), B (2, 4) and
41. In what ratio, the line joining (–1, 1) and (5, 7) is C(6, 2).

divided by the line x + y = 4 ? fdlh Hkh f=kHkqt ABC ftlds 'kh"kZ A(1, 2), B (2, 4) rFkk
nks fcanqvksa (–1, 1) rFkk (5, 7) ls izkIr js[kk dks x + y = 4 js[kk C(6, 2) gS ds dsUnzd ds funsZ'kkad D;k gkasxs \
fdl vuqikr esa foHkkftr djsxh\ FG 3, 8 IJ FG 8 , 3IJ
(1) 2 : 1 (2) 1 : 3 (3) 1 : 2 (4) 1 : 4 (1) (3, 1) (2) H 3K (3) H3 K (4) (1, 5)

BLAM–104
funsZ'kkad T;kfefr
48. For what value of k the graph of (k – l) x + y – 2 = 54. The area of triangle made by the graph of equa-
0 and (2 – k) x –3y + 1 = 0 are parallel ? tions x = 4, y = 3 and 3x + 4y = 12 is
k dk eku] ftlds fy, (k – l) x + y – 2 = 0 vkSj x = 4, y = 3 vkSj 3x + 4y = 12 lehdj.kksa ds xzkiQ ls cus
(2 – k) x –3y + 1 = 0 ds xzkiQ lekarj gSa f=kHkqt dk {ks=kiQy gS
1 1 (1) 4 sq. units (2) 3 sq. units
(1) (2) – (3) 6 sq. units (4) 12 sq. units
2 2
(3) 2 (4) –2 (SSC CGL Tier-I Exam. 16.08.2015, Second Sitting)
(SSC GL Tier-I Exam. 04.09.2011) 55. The graph of the equations 25x + 75y = 225 and x
49. What is circum radius of the triangle made by x– = 9 meet at the point
axis, y – axis and 4x + 3y = 12. lehdj.kksa 25x + 75y = 225 vkSj x = 9 ds xzkiQ fdl fcanq ij
x-v{k, y-v{k vkSj 4x + 3y = 12 ls cus f=kHkqt ds ifjo`Ùk dh feyrs gS\
f=kT;k gS% (1) (0,9) (2) (9,0)
(1) 2 units (2) 2.5 units (3) (3,0) (4) (0,3)
(3) 3 units (4) 4 units (SSC CHSL DEO & LDC Exam. 04.11.2012 (IInd Sitting)
(SSC CGL Tier-I Exam, 2012) 56. The lines 2x + y = 5 and x + 2y = 4 intersect at the
50. The area bounded by the lines x = 0, y = 0, x + y point :
= 1, 2x + 3y = 6 (in square units) is og fcanq dkSu&lk gS] ftl ij 2x + y = 5 rFkk x + 2y = 4
ljy js[kkvksa x = 0, y = 0, x + y = 1 rFkk 2x + 3y = 6 js[kk,¡ ,d&nwljs dks dkVrh gSa \
ls f?kjs gq, {ks=k dk {ks=kiQy (oxZ bdkb;ksa esa) fdruk gksxk \ (1) (1,2) (2) (2,1)
1 5
(1) 2 (2) 2 (3) ( ,0) (4) (0,2)
3 2
1 FCI Assistant Grade-III Exam. 05.02.2012 (Paper-I)
(3) 2 (4) 3 East Zone (IInd Sitting)
2
(SSC GL Tier-I Exam. 11.11.2012 (Ist Sitting) 57. The length of the intercept of the graph of the
51. Area of the triangle formed by the graph of the equation 9x – 12y = 108 between the two axes is
straight lines x – y = 0, x + y = 2 and the x– axis is lehdj.k 9x – 12y = 108 osQ xzkiQ }kjk nks v{kksa osQ chp cus
ljy js[kkvksa x – y = 0, x + y = 2 vkSj x– v{k osQ xzkI+kQ }kjk var% [k.M dh yackbZ fdruh gksxh\
fufeZr f=kHkqt dk {ks=kiQy crkb,A (1) 15 units (2) 9 units
(1) 1 sq unit (2) 2 sq units (3) 12 units (4) 18 units
(3) 4 sq units (4) None of these (SSC Graduate Level Tier-II Exam. 16.09.2012)
(SSC CGL Tier-II Exam. 21.09.2014) 58. For what value of k, the line kx + 3y + 6 = 0, will
52. If the points (h, o), (a, b) and (o, k) lie on a line, pass through the point (2, 4).
then? k ds fdl eku ds fy, js[kk kx + 3y + 6 = 0, (2, 4) fcanq
;fn rhuksa fcanq (h, o), (a, b) rFkk (o, k) lajs[k gks] rks\ ls gks dj xqtjrh gS\
a b a b (1) k = –8 (2) k = 7 (3) k = –9 (4) k = 6
(1) 1 (2) 1
h k k h 59. x varies inversely as square of y. Given that y = 2
for x = 1, the value of x for y = 6 will be equal to
h k a b
(3)
a b
1 (4) –
h k
1 x, y ds oxZ ds O;qRØekuqlkj ifjofrZr gksrk gSA rnuqlkj x = 1
ds fy, y = 2 ekudj] y = 6 ds fy, x dk eku fdruk gksxk\
53. The total area (in sq. unit) of the triangles formed
by the graph of 4x + 5y = 40, x - axis, y - axis and 1 1
(1) 3 (2) 9 (3) (4)
x = 5 and y = 4 is 3 9
4x + 5y = 40, x - v{k, y - v{k vkSj x = 5 vkSj y = 4 ds (SSC MTS Exam. 17.03.2013, Kolkata Region)
xzkiQ }kjk fufeZr f=kHkqt dk dqy {ks=kiQy (oxZ ;wfuV) esa fdruk 60. Which of the following shows the graph of y = x
gS\ + x
(1) 10 (2) 20 (3) 30 (4) 40
(SSC CGL Tier-I Exam. 19.10.2014 TF No. 022 MH 3) y=x+ x dk vkys[k (xzkiQ) fdl }kjk n'kkZ;k tk jgk gS \

BLAM–105
funsZ'kkad T;kfefr
y y 63. The graph of the linear equation 3x + 4y = 24 is a
straight line intersecting x-axis and y-axis at the
points A and B respectively. P(2, 0) and Q
x x
o o
FG 0, 3 IJ are two points on the sides OA and OB
(1) (2)
H 2K
y respectively of OAB, where O is the origin of
y
the co-ordinate system. Given that AB = 10 cm,
then PQ =
x x
o o ,d jSf[kd lehdj.k 3x + 4y = 24 dk ys[kkfp=k ,d ljy
js[kk gS] tks x-v{k rFkk y-v{k dks Øe'k% A rFkk B fcUnqvksa
(3) (4)
(1) 3 (2) 4 FG 3 IJ
ij dkVrh gSA mlesa P(2, 0) rFkk Q 0, 2 , OAB dh OA
(3) 1 (4) 2
H K
(SSC CPO SI & Assistant Intelligence Officer Exam. 2012) rFkk OB Hkqtkvksa ij nks fcUnq gSa] vkSj O, funsZ'kkad i¼fr dk
61. The graph of 3x + 4y – 24 = 0 forms a triangle ewyfcUnq gSA rnuqlkj] ;fn AB = 10 lse h gks] rks PQ = ?
OAB with the co-ordinate axes, where O is the
(1) 20 cm (2) 2.5 cm
origin. Also the graph of x + y + 4 = 0 forms a triangle
OCD with the coordinate axes. Then the area of (3) 40 cm (4) 5 cm
OCD is equal to (SSC Graduate Level Tier-II Exam. 16.09.2012)

;fn 3x + 4y – 24 = 0 dk xzkiQ funsZ'kkad v{k okyk f=kHkqt 64. Number of solutions of the two equations 4x – y
OAB cukrk gS] ;gk¡ O ewy gS vkSj x + y + 4 = 0 dk xzkiQ = 2 and 2y – 8x + 4 = 0 is
funsZ'kkad v{k okyk f=kHkqt OCD cukrk gS] rks OCD dk nks lehdj.kksa 4x – y = 2 vkSj 2y – 8x + 4 = 0 ds gyksa dh
{ks=kiQy fdruk gksxk \ la[;k gSµ
1 (1) zero (2) one
(1) of area of OAB
2 (3) two (4) infinitely many
(SSC CHSL DEO & LDC Exam. 20.10.2013)
1
(2) of area of OAB 65. If the number of vertices, edges and faces of a
3
rectangualr parallelopiped are denoted by v, e and
2 f respectively, the value of (v–e+f ) is
(3) of area of OAB
3 ;fn fdlh ledksf.kd lekarj "kV iQyd ds 'kh"kks±] dksjksa vkSj
(4) the area of OAB varkxzkas dh la[;k dks Øe'k% v, e vkSj f }kjk fufnZ"V fd;k tkrk
(SSC CGL Tier-II Exam, 2014 12.04.2015 gS rks (v – e + f) dk eku gS
(Kolkata Region) TF No. 789 TH 7)
(1) 0 (2) 2 (3) 4 (4) 1
62. The graph of linear equation y = x passes through (SSC CGL Tier-I Exam, 16.08.2015
the point (Ist Sitting) TF No. 3196279)

jSf[kd lehdj.k y = x dk ys[kkfp=k (xzkiQ) fdl fcUnq ls 2 2


66. If x + y = 4, x + y = 14 and x > y, then the correct
xqtjrk gS\ value of x and y is :
;fn x + y = 4, x2 + y2 = 14 vkSj x > y rks x ,oa y dk lgh
F 3I
(1) GH 0, JK (2) (1, 1) eku D;k gksxk\
2
(1) 2 + 3 , 2– 3 (2) 2– 2 , 3
F 1 1I
(3) GH – , JK
F 3 3I
(4) GH ,– JK
2 2 2 2 (3) 3, 1 (4) 2 + 3 , 2 2

(SSC CHSL (10+2) LDC, DEO & PA/SA Exam, 20.12.2015 (SSC CHSL (10+2) LDC, DEO & PA/SA Exam, 06.12.2015
(Ist Sitting) TF No. 9692918) (Ist Sitting) TF No. 1375232)

BLAM–106
funsZ'kkad T;kfefr
67. For what value of x the points (x, – 1), (2, 1) and 73. The Point P (5, –2) divides the segment joining
(4, 5) are collinear ? the point (x, 0) and (0, y) in the ratio 2 : 5 what is
the value of x and y ?
x ds fdl eku ds fy, rhukas fcanq (x, – 1), (2, 1) rFkk (4, 5)
lajs[k gSa\ fcUnq (x, 0) vkSj fcUnq (0, y) dks tksM+us okys [k.M dks fcUnq P
(5, –2), 2 : 5 osQ vuqikr es foHkkftr djrk gSA x vkSj y dk
(1) x = 2 (2) x = – 1 (3) x = 4 (4) x = 1
eku D;k gS\
68. Among the equations x + 2y + 9 = 0 ; 5x – 4 = 0 ;
(1) x = –7, y = 7 (2) x = 3, y = –3
2y – 13 = 0; 2x – 3y = 0, the equation of the
straight line passing through origin is (3) x = 7, y = –7 (4) x = –3, y = 3
(SSC CHSL – 2017)
lehdj.k x + 2y + 9 = 0 ; 5x – 4 = 0 ; 2y – 13 = 0; 2x
74. What is the equation of the line passing through
– 3y = 0, esa ls ewy fcanq ls xqtjus okyh ljy js[kk dk
the Point (2, –3) and making an angle of –45. With
lehdj.k gS the positive x–axis?
(1) 2x – 3y = 0 (2) x + 2y + 9 = 0
ml js[kk dk lehdj.k D;k gksxk] tks fcUnq (2, –3) ls gksdj
(3) 5x – 4 = 0 (4) 2y – 13 = 0
(SSC CGL Tier-I Exam, 16.08.2015 xqtjrh gks vkSj /ukRed x– v{k osQ lkFk –45° dk dks.k cukrh
(Ist Sitting) TF No. 3196279) gks ?
69. The linear equation such that each point on its (1) x – y = –5 (2) x – y = –1
graph has an ordinate four times its abscissa is : (3) x + y = –5 (4) x + y = –1
og jSf[kd lehdj.k dkSu&lh gS] ftlds ys[kkfp=k ds izR;sd fcanq (SSC CHSL – 2017)
dh dksfV mlds Hkqt ls pkj xquh gS\ 75. Reflection of the Point (–3, 6) in the x–axis is:
(1) y + 4x = 0 (2) y = 4x x–v{k ij fcUnq (–3, 6) dk ijkorZu D;k gS\
(3) x = 4y (4) x + 4y = 0
(1) (3, 6) (2) (6, –3)
(SSC CAPFs SI & CISF ASI Exam. 23.06.2013)
(3) (–3, –6) (4) (–6, 3)
70. What would be the equation of the line, which (SSC CHSL – 2017)
intercepts x – axis at –5 and is perpendicular to
the line y = 2x + 3? 76. What will be the equation of the perpendicular
bisector of segment joining the points (5, –3) and
ml js[kk dk lehdj.k D;k gS] tks x-v{k dks –5 ij dkVrh gS
(0, 2) ?
vkSj js[kk y = 2x + 3 ij ,d yac gS\
fcUnq (5, –3) vkSj (0, 2) dks tksMu+ s okys [kaM osQ yac }foHkktd
(1) x – 2y = –5 (2) x + 2y = 5
(3) x + 2y = –5 (4) x – 2y = 5 dk lehdj.k D;k gksxk\
(SSC CHSL – 2017) (1) x + y = 2 (2) x + y = –2
(3) x – y = –3 (4) x – y = 3
3 (SSC CHSL – 2017)
71. What is the equation of the line its slope is and
4
77. What is the equation of the line which intercepts
y–intercept is 5?
3 2
3 x–axis and y–axis at and respectively?
ml js[kk dk lehdj.k D;k gksxk] ftlosQ <yku 4
gS vkSj tks 4 3

y–v{k dks 5 ij dkVrh gS\ ml js[kk dk lehdj.k D;k gksxk] tks x – v{k vkSj y – v{k
(1) 3x + 4y = –20 (2) 3x + 4y = 20 3 2
dks Øe'k% 4 vkSj 3
ij dkVrh gS\
(3) 3x – 4y = –20 (4) 3x – 4y = 20
(SSC CHSL – 2017)
(1) 8x + 9y = 6 (2) 9x + 8y = 12
72. What is the equation of the line passing through (3) 8x – 9y = 6 (4) 9x + 8y = –12
the point (–1, –3) and having x–intercept of 4 units? (SSC CHSL – 2017)
ml js[kk dk lehdj.k D;k gksxk] tks fcanq (–1, 3) ls gksdj xqtj 78. What is the equation of the line which passes
jgh gS] vkSj x–v{k dks ;wfuV 4 ij dkVrh gS\ through the points (–1, 2) and (4, –3)?
(1) 3x – 5y = 12 (2) 3x + 5y = 12 ml js[kk dk lehdj.k D;k gksxk] tks fcanq (–1, 2) vkSj fcanq
(3) 3x + 5y = –12 (4) 3x – 5y = –12
(4, –3) ls gksdj xqtjrh gS\
(SSC CHSL – 2017)

BLAM–107
funsZ'kkad T;kfefr
(1) x + y = 3 (2) x + y = 1 (1) (5, –1) (2) (–1, 5)
(3) x – y = 1 (4) x – y = –3 (3) (1, –5) (4) (5, 1)
(SSC CHSL – 2017) (SSC CHSL – 2017)
79. For triangle PQR, What is the equation of alti- 84. What is the equation of the line whose y-inter-
tude PS if co-ordinates of P, Q and R are (5, 1), (0,
3
–4) and (–2, 3) respectively? cept is and making an single of 45° with the
4
f=kHkqt PQR esa ;fn P, Q vkSj R osQ funsZ'kkad Øe'k% (5, 1),
Positive x–axis?
(0, –4) vkSj (–2, 3) gS] rks PS dh Å¡pkbZ dk lehdj.k D;k
gksxk\ 3
ml js[kk dk lehdj.k D;k gksxk tks y -v{k dks 4 ij dkVrh
(1) 7x + 2y = 33 (2) 2x – 7y = 9
(3) 2x – 7y = –3 (4) 7x + 2y = –33 gS vkSj /ukRed x – v{k osQ lkFk 45° dk dks.k cukrh gSa\
(SSC CHSL – 2017) (1) 4x – 4y = 3 (2) 4x – 4y = –3
80. What is the slope of the line, Per-Pendicular to (3) 3x – 3y = 4 (4) 3x – 3y = –4
the line 3x – 6y = 4? (SSC CHSL – 2017)
js[kk 3x – 6y = 4 dh yacor~ jgus okyh js[kk dh <yku D;k gSa\ 85. The Point P (a, b) is first reflected in origin to P1
(1) –2 (2) 2 and P1 is reflected in y–axis to (4, –3). The Co-
ordinates of Points P are.
2 2
(3)
3
(4)
3 fcUnq P (a, b) ewy esa izFke P1 ij ijko£rr gksrk gS] vkSj y–
(SSC CHSL– 2017)
v{k ij P1 (4, –3) ij ijko£rr gksrk gSA fcanq P osQ funsZ'kkad
D;k gS\
81. For triangle ABC. What would be the equation of
median AD if co-ordinates of A, B and C are (–5, (1) (4, 3) (2) (–4, 3)
4) (–4, 0) and (–2, 2) respectively? (3) (–3, 4) (4) (3, –4)
f=kHkqt ABC osQ fy,] efè;dk AD dk lehdj.k D;k gksxk ;fn (SSC CHSL – 2017)

A, B vkSj C osQ funsZ'kkad Øe'k% (–5, 4) (–4, 0) vkSj (–2, 2) 86. P (4, 2) and R (–2, 0) are Vertices of a rhombus
PQRS. What is the equation of diagonal QS.
gSa\
(1) 3x – 2y = –11 (2) 3x + 2y = 7 P (4, 2)vksj (–2, 0) ,d le prqHkZt PQRS osQ f'kjksfcanq gS]
(3) 3x + 2y = –7 (4) 3x – 2y = 11 fod.kZ QS lehdj.k D;k gS\
(SSC CHSL – 2017) (1) x – 3y = –2 (2) 3x + y = 4
(3) 3x + y = –4 (4) x – 3y = 2
1
82. What is the equation of line Whose slope is (SSC CHSL – 2017)
2
and passes through the interesection of the lines 87. The slopes of two lines are 1 and 3 . What is the
x – y = –1 and 3x – 2y = 0? angle Between these two lines?
1 nks js[kkvksa dk <+ky 1 vkSj gSA bu nks js[kkvksa osQ chp dk
ml js[kk dk lehdj.k D;k gksxk ftldh <+yku 2
gS] vkSj tks 3
dks.k D;k gS\
js[kk x – y = –1 vkSj 3x – 2y = 0 osQ izfrPNsnu fcUnq ls gksdj
(1) 15° (2) 30°
xqtjrh gSA
(3) 45° (4) 60°
(1) x + 2y = 8 (2) 3x + y = 7 (SSC CHSL – 2017)
(3) x + 2y = –8 (4) 3x + y = –7
88. What is the equation of the line passing through
(SSC CHSL – 2017)
the Point (0, –4) and making an angle of –45°.
83. The Point R (a, b) is first re-flected in origin to R1 With the positive x–axis?
is re-flected in x–axis to (–5, 1). The co-ordinates
of Point R are?
fcUnq (0, –4) ls xqtjus okyh vkSj /ukRed x–v{k osQ lkFk –
45° dk dks.k cukus okyh js[kk dk lehdj.k D;k gksxk\
fcUnq R (a, b) ewy ij izFke R1 ij ijko£rr gksrk gS vkSj R1
x–v{k ij (–5, 1) ij ijko£rr gksrk gSA fcUnq osQ funsZ'kkad D;k (1) x + y = 4 (2) x – y = –4
(3) x + y = –4 (4) x – y = 4
gSa\
(SSC CHSL – 2017)

BLAM–108
funsZ'kkad T;kfefr
89. What is the slope of the line parallel to the line 1
passing through the point (–4, –6) and (2, –3)? 96. The slopes of two lines are and 3 . What is
3
fcUnq (–4, –6) vkSj fcUnq (2, –3) osQ ekè;e ls xqtjus okyh
the angle between these two lines?
js[kk osQ lekukarj jgus okyh fdlh js[kk dk <ky D;k gksxk\
1
1 nks js[kkvksa dk <+ky 3
vkSj 3 gSaA bu nks js[kkvksa osQ chp dk
(1) (2) –2
2
1
dks.k D;k gksxk\
(3) 2 (4) (1) 15° (2) 45°
2
(3) 60° (4) 30°
(SSC CHSL – 2017)
(SSC CHSL – 2017)
90. Area of the triangle formed by the graph of the 97. Point Q (–2, b) is the midpoint of segment EF.
straight lines x – y = 0, x + y = 2 and the x – axis Co-ordinates of E are (–7, –6) and F are (a, 0).
is? What is the value of a and b.
ljy js[kkvksa x – y = 0, x + y = 2 vkSj x – v{k osQ xzkiQ }kjk fcUnq Q (–2, b) js[kk [kaM] EF dk eè; fcanq gSA E osQ funsZ'kkad
fu£er f=kHkqt dk {ks=kiQy crkb,µ (–7, –6) gS] vkSj F osQ funsZ'kkad (a, 0) gSA a vkSj b dk eku
(1) 1 sq unit (2) 2 sq unit D;k gksxk\
(3) 4 sq unit (4) None of these (1) a = 3, b = –3 (2) a = –3, b = –3
(SSC CGL TIER-II– 2014) (3) a = 3, b = 3 (4) a =–3, b = 3
91. The graphs of x = a and y = b intersect at (SSC CHSL – 2017)
x = a vkSj y = b osQ xzkiQ fdl fcUnq ij ijLij dVsax\s 98. The slope of the line passing through the points
(1) (a, b) (2) (b, a) (2, –1) and (x, 5) is – 1 Find x ?
(3) (–a, b) (4) (a, b) fcUnq (2, –1) vkSj fcUnq (x, 5) ls xqtjus okyh js[kk dk <+ky
(SSC CGL TIER-I – 2017) – gS] rks x dk D;k eku gS\
92. If ax – 4y = –6 has a slope of –3/2. What is the (1) 4 (2) – 4
value of a? (3) 8 (4) 8
;fn ax – 4y = –6 dh <yku –3/2 gSA a dk eku D;k gksxk\ (SSC CHSL – 2017)
(1) 6 (2) 3 99. What is the slope of the line perpendicular to the
(3) –3 (4) –6 line passing through the points (–6, 3) and (2, 7)?
(SSC CGL TIER-I – 2017) fcUnq (–6, 3) vkSj fcanq (2, 7) osQ ekè;e ls xqtjus okyh js[kk
93. The angle between the graph of the linear equa- osQ yacor~ jgus okyh fdlh js[kk dk <+ky D;k gksxk\
tion 239x – 239 y + 5 = 0 and the x–axis is: (1) –2 (2) 2
jSf[kd lehdj.k 239x – 239 y + 5 = 0 vkSj x–v{k osQ xzkiQ 1 1
osQ chp dks.k fdrus va'k dk gksxk\ (3) (4)
2 2
(1) 0° (2) 60° (SSC CHSL – 2017)
(3) 30° (4) 45° 100. The point P (–4, 1) divides the segment joining
(SSC CPO 2015) the points (x, 0) and (0, y) in the ratio 3 : 2 Find
94. The linear equation such that each point on its x and y?
graph has on ordinate four times its abscissa is: fcUnq (–4, 1) fcanq (x, 0) vkSj (0, y) osQ chp osQ js[kk[k.M dks
og jSf[kd lehdj.k dkSu&lh gS ftlosQ ys[kk&fp=k osQ izR;sd 3:2 esa foHkkftr djrk gSA x vkSj y dks Kkr djsaA
fcUnq dh dksfV mlosQ Hkqt ls pkj xq.kh gS\ 5 5
(1) y + 4x = 0 (2) y = 4x (1) x = –10 y = (2) x = –10, y =
3 3
(3) x = 4y (4) x + 4y = 0 5 5
(SSC CPO SI 2013) (3) x = 10, y = (4) x = 10, y =
3 3
95. What is the equation of the line whose y-inter-
(SSC CHSL – 2017)
cept is 4 and making on angle of 45° with the 101. The graphs of the linear equations 4x – 2y = 10
positive x–axis? and 4x + ky = 2 intersect at a point (a, 4). The
y–v{k ls fcanq 4 ij feyus okyh vkSj /ukRed x-v{k osQ lkFk value of k is equal to :
45° dk dks.k cukus okyh js[kk dk lehdj.k D;k gksxk\ js[kh; lehdj.kksa 4x – 2y = 10 vkSj 4x + ky = 2 ds xzkiQ
(1) x + y = –4 (2) x – y = 4 fcanq (a, 4). ij izfrPNsfnr djrs gSaA k dk eku Kkr dhft,A
(3) x + y = 4 (4) x – y = – 4 (1) 4 (2) –3
(SSC CHSL – 2017) (3) 3 (4) –4
(SSC CGL Tier-II Exam. 18.11.2020)

BLAM–109
funsZ'kkad T;kfefr

SHORT ANSWERS 1
3 –
tan 3 3 –1
1
1. (1) 2. (2) 3. (3) 4. (2) 5. (4) 6. (3) 1 3 2 3
3
7. (3) 8. (4) 9. (4) 10. (2) 11. (3) 12. (4)
13. (4) 14. (4) 15. (1) 16. (3) 17. (1) 18. (3) 1
tan
19. (1) 20. (2) 21. (4) 22. (4) 23. (3) 24. (3) 3
25. (4) 26. (1) 27. (3) 28. (1) 29. (1) 30. (1) 1
31. (3) 32. (2) 33. (1) 34. (1) 35. (3) 36. (4) Slope
3
37. (2) 38. (4) 39. (2) 40. (1) 41. (3) 42. (4)
3. (3) lines osQ intercepts cjkcj gSA
43. (1) 44. (2) 45. (2) 46. (4) 47. (2) 48. (1)
Line dh equation gksxh
49. (2) 50. (3) 51. (1) 52. (1) 53. (2) 54. (3)
x+y=a
55. (2) 56. (2) 57. (1) 58. (3) 59. (4) 60. (2) Slope = –1
61. (2) 62. (2) 63. (2) 64. (4) 65. (2) 66. (1) 4. (2) ge tkurs gSa lines osQ chp dk angle
67. (4) 68. (1) 69. (2) 70. (3) 71. (3) 72. (2)
73. (3) 74. (4) 75. (3) 76. (4) 77. (3) 78. (2) m1 m 2
tan
1 m1m 2
79. (3) 80. (1) 81. (3) 82. (1) 83. (4) 84. (1)
85. (1) 86. (2) 87. (1) 88. (3) 89. (4) 90. (1) ;gk¡] line dh equation,
91. (1) 92. (4) 93. (4) 94. (2) 95. (4) 96. (4) y–x–7=0
97. (1) 98. (2) 99. (1) 100. (1) 101. (2) m1 = 1
blh izdkj]
EXPLANATIONS 3y  x  6 0 ls

1. (1) ge tkurs gSa 1


m2
3
y2 – y1
Line dk slope vc,
x 2 – x1
m1 m 2
–4 x – 11 tan
1 m1m 2
3 2 8

–4 x – 11 1
–40 = 3x – 33 1–
3 10 3
tan
1
–40 + 33 = 3x 1 1
3
–7 = 3x
–7 1
x 1–
3 3 3 –1
2. (2) ge tkurs gSa] 1
1 3 1
3
nks lines osQ chp dk angle

tan
m1 – m 2
1 m1m 2
3 –1 e 3 –1 j
;gk¡]
3 1 e 3 – 1j

m1 3 rFkk 2
e 3 –1 j
2
1 3 – 12
m2
3

BLAM–110
funsZ'kkad T;kfefr
2
3 12 – 2 3 12x – 5y – 7
d (3, –1)
3 –1 122 52

4–2 3 12 3–5 –1 – 7
2 169

tan 2– 3
36 5–7
13
tan –1 2 – 3 34 34
units
5. (4) y = 0 equation 239x – 239y + 5 = 0 esa j[kus ij 13 13

5 7. (3) ekuk distance = d


x=
239 P(2, 3)

FG 5 , 0IJ
Co-ordinates of A = H 239 K d

iqu% equation esa x = 0 j[kus ij


– 239y = – 5
5 2x + 3y + 4 = 0
y=
239

FG 0, 5 IJ d
2x 3y 4
Co-ordinates of B = H 239 K 2
2 2
3

Y 2 2 3 3 4
d( 2, 3)
5 13
0, 239
B
17
d( 2, 3)
13
X' X
A O
17
– 5 ,0 d (2, 3) units
13
239
8. (4) nks parallel lines 15x + 8y – 34 = 0

Y'
rFkk 15x + 8y + 31 = 0 osQ chp dk distance = d

15 x 8y – 34
5 d
2
OA = OB = 15 82
239
OAB = OBA = 45° Second equation ls
D;kasfd AOB = 90° 15x + 8y = –31
6. (3) ekuk distance = d
–31 – 34
d
(3, –1) 225 64

–65 –65
d 289 17

65
d= units
12x – 5y – 7 = 0 17

BLAM–111
funsZ'kkad T;kfefr
9. (4) ekuk P osQ coordinates gksaxs (x, 0)
x-axis ij y zero gksrk gSA 5 0 – 12 0 26
d2
2 2
5 12
P (x, 0)

26
= 2 units
13

Q
7 0 24 0 – 50
3x – 4y – 5 = 0 d3
2
7 242
rFkk,
PQ = 4 –50 50
d3
625 25
3x – 4y – 5
PQ d3 = 2
32 42
d1 = d2 = d3
12. (4) on Y-axis x = 0
PQ
3x – 5 LMbx,0g equation OP x + 2y = 3 esa x = 0] j[kus ij
5 N esa j[kus ij Q
3
3x – 5 2y = 3 y
4 2
5
3x – 2y = 1 esa x = 0] j[kus ij
3x – 5 = 20
1
3x = 25 –2y = 1 y=
2
25 Y - axis ij points gksaxsA
x
3

F 25 , 0IJ FG 0, 3 IJ & 0,
FG 1 IJ
Coordinates gksaxs GH
3 K H 2K H 2 K
10. (2) ekuk lines osQ chp dk distance = d FG 3 1 IJ 2
Required distance (0 0)2
H 2 2K
y – mx – c1
d
0 4 2 units
1 m2
13. (4) Solution of 2x + 3y = k = (2, 0)
ge tkurs gsa] 2×2+3×0=k
y – mx = c2 k=4

x y
d
c 2 – c1 14. (4) Line 1, point (2, –3) ls pass gksrh gSA
a b
1 m2
2 3
– 1
a b
c1 – c 2
blh izdkj] line (4, –5) ls pass gksrh gS
1 m2
4 5
11. (3) lines dk distance origin ls gksxkA –
a b
1

4 0 3 0 10 Point (–1, –1) nksuksa lines dks satisfy djrk gSA


d1 15. (1)
42 32
3x + 2y = 18 ......(i)
3y – 2x = 1 .......(ii)
10
5
= 2 units Equation (i) × 2 + (ii) × 3 ls]

BLAM–112
funsZ'kkad T;kfefr
y–x=0
6 x 4y 36
or
9y 6 x 3
x–y=0
13y 39
21. (4) ;gk¡,
y=3 c=2
Equation (i) ls]
m tan 60 3
3x + 2 × 3 = 18 3x = 18 – 6
= 12 x=4 line dh equation slope form esa]
(p, q) = (4, 3) y = mx + c
p+q=4+3=7
y 3x 2.
16. (3) y = –x, equation 5y + 7x = 24 esa j[kus ij
–5x + 7x = 24 22. (4) ekuk x-axis ij point P (x, 0)
2x = 24
P(x , 0)
x = 12
y = –12
m = x = 12, n = y = –12
m+n= A B
12 – 12 = 0 (–3, 4) (7, 6)
17. (1) Point of intersection = (a, b)
A.T.Q
Y PA = PB
PA 2 = PB2
(x + 3)2 + (4 – 0)2 = (x – 7)2 + (6 – 0)2
x2 + 9 + 6x + 16 = x2 + 49 – 14x + 36
y=b x2 + 6x + 25 = x2 – 14x + 85
(a , b)
20x = 60
x=3
x=a
Point P (3,0) gksxkA
X
O (0,0) 23. (3) ;gk¡,
3x – 4y + 5 = 0
18. (3) Abscissa = k,
–4y = – 3x – 5
Ordinate = 2k –1
–3 –5
According to the question / iz'ukuqlkj] y x
–4 –4
k = 2k – 1 2k – k = 1 k=1
3 5
19. (1) y x
Y 4 4
y = mx + c ls compare djus ij
II I
3 5
C
m and c
X' X 4 4
O
24. (3) ;gk¡,
III IV a = –5, b = 2
A
B (7, – 6)
Line dh Equation gksxh
Y' x y
1
a b
20. (2) Line dh equation y – y1 = m(x – x1)
;g (2, 2) point ls ikl gks jgh gS rFkk bldk slope m x y
1
–5 2
= tan 45° = 1
y – 2 = 1(x – 2) 2x – 5y = –10

BLAM–113
funsZ'kkad T;kfefr
25. (4) ekuk line dh equation 28. (1) ekuk line dh equation two point form esa]
y – y1 = m(x – x1)
y – y1 x – x1
1 y 2 – y1 x 2 – x1
;gk¡, m 2
y –1 x 1
rFkk x1 = –4, y1 = 3
–4 – 1 2 1
Line dh equation gksxh
y 1 x 1
1
y–3= (x 4) 5 3
2
3y – 3 = –5x – 5
2y – 6 = x + 4
5x + 3y + 2 = 0
x – 2y + 10 = 0
29. (1) ge tkurs gSa, line dh equation
26. (1) ekuk m1 = m = line dk slope
;gk¡, y – y1 x – x1
= 45° y 2 – y1 x 2 – x1

–1 1 (7, 8) (10, 4)
m2
–2 2 D C

ge tkurs gSa]
m1 – m 2
tan
1 m1m 2
A B
(–2, 6) (1, 2)
1
m –
tan 45 2 Diagonal AC dh equation gksxh
m
1 y–6 x 2
2
4–6 10 2
2m – 1
1 y 6 x 2
2 m
2 12
2 + m = 2m – 1
12y – 72 = –2x – 4
m=3
2x + 12y = –4 + 72
Line dh equation gksxh 2x + 12y = 68
y – y1 = m(x – x1) x + 6y = 34
y – 2 = 3(x – 3) 30. (1) tc dksbZ nks lines parallel gksrh gS rks muds slopes
y – 2 = 3x – 9
equal gksrs gSaA
3x – y – 7 = 0
i.e. m1 = m2

27. (3) Y ;gk¡,


m1 = m2
–3
X' X m2 [From equation 3x – 4y + 2 = 0]
O –4
Q
P
3
m2
4
Lines parallel gSa
m1 = m2
Y'
3
m
x-axis osQ parallel line dh equation gksxh : y = a 4

;gka , a = – 3 ekuk line dh equation


Equation is y = – 3 y – y1 = m(x – x1)

BLAM–114
funsZ'kkad T;kfefr
line (–2, 3) point ls pass gks jgh gS
3x y 8
Line dh equation gksxh 2 2

2
0
3 12 3 12 3 12
3
(y – 3) (x 2)
4
3x y 8
4y – 12 = 3x + 6 – 0
4 4 4
3x – 4y + 18 = 0
31. (3) ;gk¡, 3x y 8
– 0
2 2 2
=
4 34. (1) ge tkurs gSa line dh equation intercept form esa
m1 = m
gksxh
x y
1 1
m2 a b
2
3x + 2y – 5 = 0
ge tkurs gSa 3x + 2y = 5
m1 – m 2 5 ls nksuksa rjiQ divide djus ij
tan
1 m1m 2 3 2
x y 1
5 5
1
m – x y
tan 2 1 (Desired Result)
4 m 5 5
1 3 2
2
35. (3) ;gk¡,
2m – 1
1 p = 3 and = 120°
2 m
ge tkurs gSa] perpendicular form esa line dh equation
2 + m = 2m – 1 gksxhA
m 3 x cos + y sin = p
x cos 120° + y sin 120° = 3
32. (2) ekuk m = line dk slope x cos (180° – 60°) + y sin (180° – 60°) = 3
;gk¡, –x cos 60° + y sin 60° = 3
 cos (180° – ) = –cos ; sin (180° – ) = sin
–1 1
m1
–7 7 x 3
– y 3
Lines perpendicular gS,a 2 2
m1 × m2 = –1 –x 3y 6
1 36. (4) ekuk] m = line dk slope
m –1
7
rFkk,
m = –7
–2
line dh equation gksxh m1
3
(y – y1) = m(x – x1)
(y – 0) = –7(x – 3) m1 = m2
y = –7x + 21 (  lines parallel gS)
7x + y = 21 –2
m
33. (1) Normal form esa line dh equation gksxh 3

ax by c line dh equation
0 y – y1 = m(x – x1)
a2 b2 a2 b2 a2 b2
line (–4, –5) point ls pass gks jgh gSA

BLAM–115
funsZ'kkad T;kfefr
–2 rFkk
(y 5) (x 4)
3 6 k 1 2
0
3y + 15 = –2x – 8 k 1
2x + 3y + 23 = 0 0 (k + 1) = 6k + 2
37. (2) ekuk line dh equation 6k = –2
(x + 2y – 5) + (x – 3y – 7) = 0 –1
;g (0, –1) ls pass gks jgh gSA k=
3
0–2–5+ (0 + 3 – 7) = 0 k dh value (–ve) ugha gksxh
–7 – 4 = 0
Ratio gksxk 1 : 2
–7 41. (3) ekuk ratio k : 1
=
4
line dh equation gksxh
P(x, y)
–7 k: 1
(x 2y – 5) ( x – 3y – 7) 0 A B
4

x
(–1, 1) (5, 7)

+
y
4x + 8y – 20 – 7x + 21y + 49 = 0

=
4
–3x + 29y + 29 = 0
3x – 29y – 29 = 0 Internal section formula ds }kjk,
38. (4) Y 5 k 1 –1
x
k 1

5k – 1
x
x=k k 1

X X 7 k 1 1
y
k 1
Y
7k 1
x = k, y – axis osQ parallel graph show djsxkA y
k 1
39. (2) ge tkurs gSa mid points osQ co-ordinates gksaxs
x rFkk y dh value equation esa j[kus ij x + y = 4
FG x 1 x 2 y1
,
y2 IJ 5k – 1 7k 1
H 2 2 K k 1 k 1
4
2 6
x 4 12k = 4(k + 1)
2
12k = 4k + 4
4 8
y 6 8k = 4
2
1
40. (1) ekuk ratio k : 1 k
2
A P(2, 0) B
Ratio gksxk 1 : 2.
(1, 2) k:1 (4, 6)
42. (4) y-axis ij x, point zero gksrk gS
Internal section formula ds }kjk
p osQ coordinates gksaxs (0,y)
4 k 1 1
2 ekuk ratio = k : 1
k 1
Internal section formula, osQ }kjk
2(k + 1) = 4k + 1
P(0, y)
2k + 2 = 4k + 1
–2k = –1 (2, 3) k:1 (4, 6)

1 4k 2
k= 0
2 k 1

BLAM–116
funsZ'kkad T;kfefr
–2 1 a 2 0
k h
4 1 2

–1 a = 3h
k
2 blh izdkj
–ve sign show djrk gS point, line dks externally di- 1 0 2 b
k
vide djrh gSA 1 2
ratio gksxk 1 : 2
2b
43. (1) ekuk ratio k : 1 k
3

3k
b
2
P(x, y)
k : 1
Line dh equation gksxh
(–1, 1) (5, 7)
x y
1
a b
2x + y = 4
x 2y
1
3h 3k
Internal section formula ds }kjk]
x 2y
3
5k – 1 h k
x
k 1 45. (2) getkurs gSa tc dksbZ nks lines ,d nwljs ij perpendic-
ular gksrh gSa rks muds slopes dk product –1 gksrk gSA
7k 1
y i.e m1 × m2 = –1
k 1
;gk¡]
x rFkk y dh value equation 2x + y = 4 esa j[kus ij – kx + 2y + 3 = 0
F 5k – 1I 7k 1 k
2 GH k 1 JK k 1
4 m1
2

10k – 2 + 7k + 1 = 4 (k + 1) ;gk¡]
17k – 1 = 4k + 4 2x + 4y + 7 = 0
13k = 5 2
m2
5 4
k
13 1
m2 –
ratio = 5 : 13 2
44. (2) Internal section formula ds }kjk] lines ,d nwljs ij perpendicular gS blfy,
m1 × m2 = –1
y
k 1
– –1
2 2
B (0, b)

R
k 4
(h
1 ,k
: ) 46. (4) ge tkurs gSa tc line x-axis osQ parallel gksrh gS rks]
2
(a, 0)
Slope = 0

A (4 – k 2 )
0
k –3

BLAM–117
funsZ'kkad T;kfefr
2
4–k =0 AOB = 90°
k2 = 4 AB = diameter

k 4 AB = 32 42 = 9 16 25 = 5
k = +2
5
radius = 2.5 units
47. (2) Centroid osQ coordinates 2
x1 x2 x3
x 50. (3)
3 y

y1 y2 y3 C (0,2)
y (0,1)
3
A

A(1, 2) x' x
O B D
(1,0) (3,0)
9
y'
B(2, 4) C(6, 2)
x = 0 is the equation of y-axis
y = 0 is the equation of x-axis
1 2 6
x Putting x = 0 in x + y =1, y = 1
3
Putting y = 0 in x + y =1, x = 1
x=3 Putting x = 0 in 2x + 3y = 6, 3y = 6 y=2
Putting y = 0 in 2x + 3y = 6, 2x = 6 x=3
2 4 2
y OB = 1; OA = 1
3
OD = 3; OC = 2
8 Required area = OCD – OAB
y
3 1 1
= 3 2 1 1 =3– 1
2 2 2
osQ co-ordinates = 3, 3
FG 8 IJ
Centroid H K =2
1
sq. units
2
48. (1) (k – 1) x + y – 2 = 0 & (2 – k) x – 3y + 1 = 0
51. (1) x = 0 equation x + y = 2 esa j[kus ij
Parallel gksaxh ;fn
0+y=2 y=2
k 1 1
– 3k + 3 = 2 – k – 3k + k = 2 – 3 Point of intersection on y-axis = (0, 2)
2 k 3
iqu% y = 0 equation x + y = 2 esa j[kus ij
1
– 2k = – 1 k x-v{k ij intersecting point = (2, 0)
2
x – y = 0 origin ls pass gksxh rFkk axes ij equal
Note : nks lines a1x + b1y + c1 = 0
intercept cuk,xhA
rFkk a2x + b2 y + c2 = 0 parallel gksxh
a1 b1 c1 Y
;fn a b2 c2
2

49. (2)

B (0, 4)

(0,2) x–y=0
B
C
A x+y=2
O X
(3, 0) O D A (2,0)

BLAM–118
funsZ'kkad T;kfefr
x = y equation x + y = 2 esa j[kus ij OA = 10 units
2y = 2 y=1 OD = 5 units = EC
CD = 1 DA = 10 – 5 = 5 units
OA = 2
iqu%] OB = 8 units
1 OE = 4 units
Area of OAC = OA CD
2
BE = 8 – 4 = 4 units
1 1
= 2 1 = 1 sq. unit ADC dk Area = DA DC
2 2
52. (1) ge tkurs gSa tc dksbZ rhu points collinear gksrs gSa rks 1
= 5 4 = 10 sq. units
triangle dk area zero gksrk gSA 2

h 0 1 1
1 BEC dk Area = EC BE
ar a b 1 2
2
0 k 1 1
= 5 4 = 10 sq. units
2
1
[h (b – k) + 1 (ak)] = 0 Required area = 10 + 10 = 20 sq. units.
2
54. (3) x = 4 y-axis osQ parallel line
bh – hk + ak = 0
y=3 osQ parallel line
x-axis
ak + bh = hk
x = 0, equation 3x + 4y = 12 esa j[kus ij
nksuksa rjiQ hk ls divide djus ij
12
ak bh 3 × 0 + 4y = 12 y= =3
1 4
hk hk
The co-ordinate of point of intersection on y- axis
a b = (0,3)
1
h k y = 0 equation esa j[kus ij
3x + 4 × 0 = 12 x=4
53. (2) Y The co-ordinate of point of intersection on x- axis
= (4,0)
B (0,8)
Y

C (5,4)
E
C
B (0,3)

(10,0)
A X
O D

4x + 5y = 40 esa x = 0 j[kus ij]


O A (4,0) X
40
4 × 0 + 5y = 40 5y = 40 y= =8
5
AC = 3 units] BC = 4 units
y-axisij point of intersection = (0, 8)
iqu%] 4x + 5y = 40 esa y = 0 j[kus ij] 1
or ( ABC) = × BC × AC
2
40
4x + 5 × 0 = 40 4x = 40 x= = 10
4 1
= × 4 × 3 = 6 sq. units
x-axis ij point of intersection = (10, 0) 2

BLAM–119
funsZ'kkad T;kfefr
55. (2) x = 9 equation 25x + 75y = 225 esa j[kus ij 4
25 × 9 + 75y = 225 x
y2
75y = 225 – 225 = 0 y=0
Point of intersection gksxk (9, 0) tc y = 6,
56. (2) 2x + y = 5 ...(i) 4 1
x
x + 2y = 4 ...(ii) 6 6 9
Equation (i) × 2 – equation (ii) 60. (2) y = x + |x|= 2x tc] x > 0
4x + 2y = 10
=0 tc] x < 0
x + 2y = 4
– – –
61. (2) Y
3x = 6 (0, 6) B
x=2
Equation (i) ls, (8, 0)
2×2+y=5 y=5–4=1 X' X
C A
Point of intersection gksxk (2, 1)
57. (1) Y D

Y'

X' X
O A(12,0)
3x + 4y = 24]

B(0,–9)
x=0 j[kus ij]
24
Y' 4y = 24 y= =6
4

x = 0, 9x – 12y = 108 equation esa j[kus ij, B osQ coordinates = (0, 6)


0 –12y = 108 iqu% 3x + 4y = 24
y=–9 y = 0 j[kus ij]
y = 0, 9x – 12y = 108 esa j[kus ij 3x = 24 x=8
9x – 0 = 108 x = 12
AosQ coordinates = (8,0)
? OA = 12, OB = –9
blh izdkj, for x + y = – 4
? AB = OA 2 OB 2 122 92 144 81 C osQ coordinates = (–4, 0)

225 ? AB = 15 units D osQ coordinates = (0, –4)

58. (3) line kx + 3y + 6 = 0, point (2, 4) ls pass gks jgh gS] 1


OAB dk area = × OA × OB
k×2+3×4+6=0 2
2k + 12 + 6 = 0
1
2k + 18 = 0 = × 8 × 6 = 24 sq. units
2
2k = –18
k = –9 1
Area of OCD = × OC × OD
1 k 2
59. (4) x x
y 2 y2 1
= × 4 × 4 = 8 sq. units
tgk¡ k constant of proportionality gS] 2

k 1
tc, x = 1, y = 2 1 k 4 OCD =
3
OAB.
4

BLAM–120
funsZ'kkad T;kfefr
66. (1) x + y = 4 --- (i)
62. (2) y x2 + y2 = 14 --- (ii)
(x + y)2 = x2 + y2 + 2xy
16 = 14 + 2xy
2xy = 16 – 14 = 2 xy = 1 --- (iii)

x (x – y)2 = (x + y)2 – 4xy = (4)2 – 4 = 16 – 4 = 12


y=
x–y= 12 = 2 3 --- (iv)
Equations (i) rFkk (iv) dks add djus ij
x x+y=4
0 (0,0)

x–y=2 3
Point (1, 1) equation y = x dks satisfy djrk gSA
63. (2) OP = 2
2x = 4 + 2 3
3
OQ = x = 2+ 3
2
Equation (i) ls,
2+ 3 +y=4 y=4–2– 3 = 2– 3
67. (4)tc dksbZ rhu points collinear gksrs gSa rks triangle dk
area zero gksrk gSA

x –1 1
1
ar 2 1 1
2 =0
4 5 1

1
[x (1 – 5) + 1(2 – 4) + 1(10 – 4)] = 0
2
? PQ 2 2 22
FG 3 IJ 2
4
9
–4x – 2 + 6 = 0
OP OQ H 2K 4
–4x + 4 = 0
x=1
16 9 25 5
2.5 cm 68. (1) Co-ordinates of origin = (0, 0).
4 4 2
;s co-ordinates equation 2x – 3y = 0 dks satisfy djrs
64. (4) 4x – y = 2 ....(i) gSaA
2y – 8x + 4 = 0 8x – 2y = 4 .....(ii) 69. (2) y = 4x,
a1 b1 c1 1 tc] x = 1, y = 4
a2 b2 c2 2 70. (3) Given the equation
For simultaneous linear equations y = 2x + 3

a1x + b1y = c1 m1 = 2
Lines are perpendicur so,
a2x + b2y = c2 if
m1.m2 = –1
a1 b1 c1 2 × m2 = –1
a2 b2 c 2 , infinite solutions gksaxs
1
65. (2) Vertices of parallelopiped = v = 8 m2 =
2
Edges = e = 12
required the equation.
Surfaces = f = 6
y – y1 = m2 (x – x1)
v – e + f = 8 – 12 + 6 = 2

BLAM–121
funsZ'kkad T;kfefr
75. (3)
1
y–0= (x + 5) [  on x axis y1 = 0]
2
2y = –x – 5 y
x + 2y = –5
3 (–, +) (+, +)
71. (3) Here, m =
4
y1 = 5 and x1 = 0 (cut on y–axis)
1
required the equation x x
y – y1 = m (x – x1)
3 (–, –) (+, –)
y–5= (x – 0)
4
4y – 20 = 3x
1
3x – 4y = –20 y
72. (2) Slope of the required equation which passes
through (–1, 3)
Reflection of the point (–3, 6) in the x–axis is (–3,
0 3 3 –6)
m=
4 1 5
76. (4)fcUnqvksa (a, b) rFkk (a1, b1) dks feykus okys line
hence the equation
segment osQ yEc v¼Zd (right bisector) dk lehdj.k
y = m (x – x1)
2x (a – a1) + 2y (b – b1) = a2 – a12 + b2 – b12
3
y= (x – 4) 3x + 5y = 12 Here,
5

2 5 a = 5, b = –3

A C B a1 = 0, b1 = 2
73. (3)
( x, 0) (5, –2) (0, y) 2x (5 – 0) + 2y (–3 –2) = 52 – 02 + (–3)2 – (2)2

mx 2 nx1 10x – 10y = 25 + 9 – 4


x=
m n 10x – 10y = 30

2 0 5 x x–y=3
5= x=7
2 5 77. (3) Here, Given
Again, 3 2
a= ,b=
my 2 ny1 4 3
y=
m n the required equation

–2 =
2y b5 0g x y
1
7 a b
y = –7
hence, (x1 y) = (7, –7) x y
1
74. (4) Here, FG 3 IJ FG 2 IJ
= 45°, y1 = –3, x1 = 2 H 4K H 3 K
slope (m) = tan (–45°)
8x – 9y = 6
= –tan 45° = –1
required equation 78. (2) the required equation is
y – y1 = m (x – x1) y y1 x x1
y + 3 = –1 (x – 2) y1 y 2 x1 x 2
y + 3 = –x + 2
[Here x1 = –1, y1 = 2 x2 = 4, y2 = –3]
x + y = –1

BLAM–122
funsZ'kkad T;kfefr
81. (3)
y 2 x 1 A (–5, 4)
2 3 1 4
(y – 2) × (–5) = (x + 1) × 5
–y + 2 = x + 1 a a
h
x+y=1
79. (3)

A B D C
(5, 1) (–4, 0) (–2, 2)
D, BC dk Mid-point gS
FG 4 2 0 2
,
IJ
D= H 2 2 K = (–3, 1)

the required equation of AD

y y1 x x1
y1 y 2 x1 x 2

B D C y 4 x 5
(0, – 4) (–2, 3) 4 1 5 3
(y – 4) × (–2) = (x + 5) × 3
Slope of BC (m1) –2y + 8 = 3x + 15
3x + 2y = –7
=
3 b 4g 7
82. (1) Given two lines are
2 0 2
x – y = – 1...(1)
 AD BC 3x – 2y = 0...(2)
m1.m2 = –1 Solving the eqn (1) & (2)
x = 2, y = 3
7
m2 1 Required the equation
2
y – y1 = m (x – x1)
2
m2 = y–3= (x – 2)
7 2
the required equation 2y – 6 = –x + 2
y – y1 = m2 (x – x1) x + 2y = 8
2 83. (4)
y–1= (x – 5)
7
2x – 7y = 3
80. (1) 3x – 6y = 4
R R1
coff. of x 3 1 (–5, 1) (5, 1)
Slope (m1) = coff. of y =
6 2
Now, lines are perpendicular
–5
m1.m2 = –1

1
× m2 = –1
2
(–5, –1)
m2 = –2
R1 = (5, 1)

BLAM–123
funsZ'kkad T;kfefr
84. (1) Here, y – 1 = –3(x –1)
= 45° 3x + y = 4
m = tan = tan45° = 1
87. (1) Here, Given m1 = 1, m2 = 3
3
y1 , x1 0
4 m1 m 2
tan = 1 mm
required the equation 1 2
y – y1 = m (x – x1)
1 3 3 1
3
y+
4
1xb 0 g =
1 3 3 1

3
y+ x 3 1 3 1
4 =
3 1 3 1
4x – 4y = 3
85. (1) = 2 3 = tan 15° = = 15°
the required angle between two lines ( ) = 15°
y 88. (3) Given
= –45°
3
m = tan(–45°)
P1 (4, 3)
= –tan 45° = –1
and x1 = 0, y1 = –4
4 required the equation
x1 y – y1 = m (x – x1)
y + 4 = –1 (x – 0)
y + 4 = –x
–3 x + y = –4
P P
(–4, –3) (4, –3) y2 y1 3 6 1
y
1 89. (4) Slope (m) = x x1 = 2 4 = 2
2

Co–ordinate of point P1 = (4, 3)


90. (1)

(4, 2)
86. (2) P S
)
,1
(1
M

(2, 0)
(0, 0)
Q R
(–2, 0)

M is the mid–point of PR Given the equations of lines


0 2 1 x – y = 0 ...(1)
Slope of PR (m1) = x + y = 2 ...(2)
2 4 3
from eqn (1) & (2)
Lines are perpendicular to each other x=y=1
m2 = –3 Area bounded by lines (x = 2)
FG 4 2 , 2 0 IJ 1
2 1
the co-ordinate of M = H 2 2 K = (1, 1) area of triangle =
2
required the equation of QS = 1 sq unit

BLAM–124
funsZ'kkad T;kfefr
91. (1) x = a, y = b will intersect on (a, b)
y2 y1
92. (4) ax – 4y = –6 98. (2) m = x
2 x1
–coff.of x
Slope (m) = coff.of y 5 1
–1 = x = –4
x 2
3 a
7 3 1
2 4 99. (1) Slope (m1) =
2 6 2
a = –6
93. (4) Given the line lines are perpendicular to each other
239x – 239 y + 5 = 0 m1.m2 = –1

–coff. of x 1
× m2 = –1
m = tan = coff. of y 2
m2 = –2
239 100. (1)
tan = 1 tan 45
239
3 2
= 45°
A C B
94. (2)
(x, 0) (–4, 1) (0, y)
95. (4) Here,
= 45° using section formula
m = tan 45° = 1 mx 2 nx 4
and y1 = 4, x1 = 0 x=
m n
the required equation
y – y1 = m (x – x1)
– 4=
b3 0g b2 x g
y – 4 = 1 (x – 0) 3 2
x – y = –4 x = –10
96. (4) Here Given
my 2 xy1
1 and y =
m n
m1 = , m2 = 3
3
1=
b2 0g b3 y g
m1 m 2 3 2
tan = 1 m .m
1 2 5
y=
3
1
3
3
1 3
FG 10,
5 IJ
= 1 =
2 3
Thus (x, y) = H 3 K
1 3
3 101. (2) Given the lines are intersect at (a, 4)
x = a, y = 4
1 Now,
= = tan 30° 4x – 2y = 10
3
4a – 2 × 4 = 10
= 30°
4a = 18
97. (1)
9
a=
E Q F 2
(–7, –6) (–2, b) (a, 0) Again 4x + Ky = 2
using Mid-point formula
FG 4 9 IJ bK 4g = 2
–2 =
7
2
a H 2K
18 + 4a = 2
a=3
a = –4
6 0
‡‡‡
and b = b = –3
2

BLAM–125
funsZ'kkad T;kfefr

ADVANCED LEVEL QUESTIONS


1. Find the area which is formed by three lines 2x +
4y = 12, 3x + 2y = 6 and x-axis (1) x (z y ) (z y )2 – 4
rhu js[kkvksa 2x + 4y = 12, 3x + 2y = 6 rFkk x-axis }kjk cus
{ks=k dk {ks=kiQy Kkr djsa (2) x (z – y ) (z y )2 4
(1) 6 sq. unit (2) 4 sq. unit
(3) 8 sq. unit (4) 3 sq. unit (3) x (z – y ) (z y )2 – 4
2. ;fn (a, b), (x, y) rFkk (a + x, b + x) lajs[kh; gSa rks a, b, (4) None of these
x rFkk y osQ chp laca/ Kkr djsa \ 9. For what value of K given system of equations 4x
If (a, b), (x, y) and (a + x, b + x) are collinear, then + 12y = 14 & 5x + ky = 8 are intersecting lines
find the relation between, a, b, x & y K osQ fdl eku osQ fy, ljy js[kkvksa dk fudk; 4x + 12y
(1) ax = by (2) ab = xy = 14 rFkk 5x + ky = 8 izfrPNsnh js[kk,¡ gSA
(3) ay = bx (4) a – b = x + y
5 5
3. What is the angle between pair of straight lines (1) K 15 (2) K = 15 (3) K (4) K =
represented by equation 5x2 – 1112xy – 5y2 = 0 3 3
5x2 – 1112xy – 5y2 = 0 }kjk iznf'kZr js[kkvksa osQ ;qXe osQ chp 10. For what value of K given pair of equations 2x +
5y = 12 & 3x + ky = 16 have no solution?
dk dks.k D;k gS \
K osQ fdl eku osQ fy, lehdj.kksa osQ tksM+s 2x + 5y = 12
(1) 90° (2) 45° (3) 30° (4) 60°
4. Find the foot of the perpendicular in the below & 3x + ky = 16 dk dksbZ gy ugha gS \
figure (1) K 7.5 (2) K = 7.5 (3) K 8.5 (4) K = 8.5
uhps nh x;h vkÑfr esa yac dk ikn Kkr djsa \ 11. In the xy coordinate system, if (a + 5, b + 2) & (a
+ 7, b + k) are two points on the line defined by
A (2, 5) the equation
7x – 5y – 4 = 0, then find K = ?
xy funsZ'kkad esa ;fn (a + 5, b + 2) rFkk (a + 7, b + k) nks
fcUnq lehdj.kksa 7x – 5y – 4 = 0 }kjk ikfjHkkf"kr gSa rks K =
4x + y = – 4 ?
B C 24 22
D ( h, k) (1) (2) 6 (3) (4) 4
(1) 4, 2 (2) 4, –2 5 5
(3) –2, 4 (4) 2, 4 12. Two sides of a triangle are given by equations x +
5. Find the intersection point of the two lnes whch 4y = 6 and y – 4x = 8, If the circumcentre of this
is passes through point (3, 4) and (5, –3) whose triangle lies at (–1, 0), Find the Area of this tri-
are 60° and 45° respectively (in approx) angle
nks j[kkvksa tks (3, 4) rFkk (5, –3) ls xqtjrh gSa rFkk ftudk ,d f=kHkqt dh nks Hkqtk,¡ lehdj.k x + 4y = 6 rFkk y – 4x
>qdko Øe'k% 60° rFkk 45° gS] dk izfrPNsn fcUnq Kkr djsa \ = 8 }kjk nh x;h gSaA ;fn bl f=kHkqt dk ifjosQUnz (–1, 0) gS rks
(1) –9, –17 (2) 9, –17 (3) –5, 12 (4) –5, 12 f=kHkqt dk {ks=kiQy Kkr djsa \
6. If the points (2, 3), (8, 7) & (K + 3, K – 3) are 26
collinear, then find the value of K. (1) 32/17 sq. unit (2) sq. unit
17
;fn fcUnq (2, 3), (8, 7) rFkk (K + 3, K – 3) lajs[kh; gSa rks (3) 128/17 sq. unit (4) 128 sq. unit
K dk eku Kkr djsa \
(1) –20 (2) –18 (3) 20 (4) 18 13. The points (1, 2), b g
5, 3 and (7, –6) are three
consecutive vertices of a parallelogram. Find the
7. Two points (0, 0), (3, 3 ) form an equilateral co-ordinates of the fourth vertex.
triangle with the third point (x, y). Find the co- fcUnq (1, 2), (–5, –3), (7, –6) lekarj prqHkqt
Z osQ rhu yxkrkj
ordinates of the third point.
'kh"kZ gSA rks pkSFks 'kh"kZ fcUnq osQ fu;ked fudkfy;sA
nks fcUnq (0, 0), (3, 3 ) ,d rhljs fcUnq (x, y) ls ,d (1) (13, 1) (2) (2, –3) (3) (–1, –7) (4) (0, –3)
leckgq f=kHkqt cukrs gSA ml fcUnq dk fu;ked crkb,;sA 14. Area of the trapezium formed by x-axis, y-axis
and the lines 3x + 5y = 15, 8x + 7y = 56 will be
e
(1) 0, 2 3 j e
(2) 3, 3 j x-v{k] y-v{k rFkk 3x + 5y = 15, 8x + 7y = 56 ls cus
(3) (1) & (2) Both (4) None of these leyac dk {ks=kiQy gksxk
8. Distance between (xy2, 2xy) and (xz2, 2xz) is (1) 31.5 sq. unit (2) 40.5 sq. unit
(xy2, 2xy) rFkk (xz2, 2xz) osQ chp nwjh gS& (3) 20.5 sq. unit (4) 28.5 sq. unit

BLAM–126
funsZ'kkad T;kfefr
15. Find the area of triangle whose vertices are (x, y
+ z), (x, y – z) & (–x, z) SHORT ANSWERS
f=kHkqt dk {ks=kiQy Kkr djsa ftuosQ 'kh"kZ (x, y + z), (x, y –
z) rFkk (–x, z) gS\ 1. (1) 2. (3) 3. (1) 4. (3) 5. (1)
(1) 2xz (2) 2yz 6. (3) 7. (3) 8. (2) 9. (1) 10. (2)
(3) y (x + 2) (4) 12 (x – y) 11. (1) 12. (3) 13. (1) 14. (1) 15. (1)
16. If the point (x, y) lies on the line joining the points 16. (3) 17. (1) 18. (1) 19. (2) 20. (1)
(2, 4) and (5, 9) then 5x – 3y = ?
;fn fcUnq (2, 4) rFkk (5, 9) dks feykus okyh js[kk ij fcUnq (x, EXPLANATIONS
y) gks rks 5x – 3y = ?
1. (1) 2x + 4y = 12
(1) 2 (2) –3
(3) –2 (4) 0 x y
1 ...(i)
17. Find the equation of the straight line which pass- 6 3
es through (3, 4) and the point (3, 4) divides the 3x + 2y = 6
portion of the line intercepted between the axes
x y
in the ratio of 2 : 3 1 ...(ii)
2 3
ml ljy js[kk dk lehdj.k Kkr djsa tks (3, 4) ls xqtjrh gS
y=0 ...(iii)
vkSj fcUnq (3, 4) v{kksa osQ chp ljy js[kk osQ [k.M dks 2 : 3 y-axis
osQ vuqikr esa ck¡Vrk gS\ 6
(1) 2x + y = 10 (2) 3x – 2y = 5
5
(3) x – y = 0 (4) 2x – y = –3
18. The Vertices of a triangle are (1, 2), (2, 3) and (3, 4
4). Find the equation of the median of the trian- A3 (x, y )
gle passing through the first Vertex.
2
fdlh f=kHkqt osQ 'kh"kZ (1, 2), (2, 3) vkSj (3, 4) gSaA izFke 'kh"kZ
ls tkus okyh ekfè;dk dk lehdj.k fudkfy;s\a 1
(1) x – y + 1 = 0 (2) 2x – y + 3 = 0 x-axis
O 1 2 3 4 5 6
(3) 5x – 4y = 2 (4) 3x – y + 7 = 0
B C
19. Find the equation of the straight line which makes
200 From (i) and (ii) equation we find
a triangle of area with the axes and the per--
3 x = 0, y = 3
pendicular drawn from the origin to it makes an BC = 6 – 2 = 4
angle of 30° with the x–axis. 1
Area of ABC = × 4 × 3 = 6 sq. unit
ml ljy js[kk dk lehdj.k Kkr dhft, tks v{kksa osQ lkFk 2
2. (3) Collinear condition esa slopes cjkcj gksrs gSa
200
3
{ks=kiQy dk f=kHkqt cukrh gS vkSj ewy fcUnq ls ml ij Mkys ( a, b) (x , b) (a + x, b + y)

yEc x–v{k osQ lkFk 30° dk dks.k cukrh gS\ Slopes are equal
y b b y y
(1) 2x – y = 0 (2) 3x y 20 x a a x x
ay – ab = bx – ba
(3) 2x y 12 (4) x – y = 3
ay = bx
20. Find the length of the perpendicular drawn from 3. (1) If Equation is in the form of ax2 + cxy + y2 = 0
the origin to the straight line 8x – 5y + 51 = 0 Where, a + b = 0 then = 90°
and the inclination of this normal to the x – axis. 5–5=0
ewy fcUnq ls ljy js[kk 8x – 5y + 51 = 0 ij Mkys x;s yac = 90°
dh yackbZ rFkk bl yac dk x – v{k ij >qdko fudkfy;s\ 4. (3) Slope of line AD × Slope of line BC = –1
k 5
(1) 3, tan
1 FG 15 IJ (2) 4, tan
1 FG 1IJ h 2
× –4 = –1
H8K H7K 4k – 20 = h – 2

(3) 3, tan
1 FG 5 IJ 1, tan 1 FG 3 IJ h – 4k = –18 ...(i)
H 12 K (4) H 2K (h, k) point lies on 4x + y = –4

BLAM–127
funsZ'kkad T;kfefr
4h + k = –4 ...(ii)
(12) – 6x – 2 3y = 0 (using (1))
From (i) and (ii) equation we have
h = –2, k = 4 12 6x 6 3x
(h, k) = (–2, 4) y= =
2 3 3
5. (1) Equation of lines passes through (3, 4) makes
an angle 60° Put the value of y in eqn (1)
y – 4 = tan60° (x – 3)
y–4= x2 +
FG 6 3x IJ 2

= 12
3 (x – 3) H 3 K
y 3x 4 3 3 ...(i) 4x2 – 12x = 0
Equation of line passes through (5, –3) makes an x (x – 3) = 0
angle of 45° x = 0 or 3
y + 3 = tan 45° (x – 5)
6 3 0 6 3 3
y+3=x–5 y= or
y=x–8 ...(ii) 3 3
From (i) and (ii) equation we have = 2 3 or 3
x = –9 and y = –17 (approx)
6. (3) Collinear condition esa slopes cjkcj gksrs gSa So (x, y) = 0, 2 3 e j and 3, e 3 j
y2 y1
= x x1
2
8. (2) Distance x = (2 z 2xy )2 ( xz 2 xy 2 )2
7 3 K 3 7
2
8 2 K 3 8 = 4 x 2 (z y )2 x 2 (z y )(z y)
2 K 10
3 K –5 = x (z y ) (z y )2 4

2K – 10 = 3K – 30 9. (1) System dk unique solution gksxk rks


K = 20 a1 b1 4 12
7. (3)  ABC ,d equilateral triangle gS a2 b2 5 k
AB = BC = AC
K 15
AB2 = BC2 = AC2
10. (2) ;fn System dk dksbZ solution ugha gksxk rks
2
(0 – 3)2 + 0e 3 j = (x – 0)2 + (y – 0)2 a1 b1 2 5
a2 b2 3 k
2
= (3 – x)2 + e 3 y j 15
K= = 7.5
2
A
7x – 5y – 4 = 0
e3, 3 j 11. (1)
( a + 5, b + 2) (a + 7, b + k)
7x – 5y – 4 = 0
5y = 7x – 4
7x 4
y=
5 5

B C 7
(0, 0) ( x, y) Slope =
5

b k (b 2) 7 b k b 2 7
12 = x2 + y2 = x2 + y2 – 6x 2 3y + 12
a 7 (a 5) 5 a 7 a 5 5
Now, x2 + y2 = 12 ...(1)
k 2 7 24
x2 + y2 – 6x 2 3y + 12 = 12 = k
2 5 5

BLAM–128
funsZ'kkad T;kfefr
x y x y 5 x 3 y
12. (3) x + 4y = 6 1 or, 1 4 2
6 6/4 6 3/2 2 2
–5 + x = 8 –3 + y = –4
y x x = 13 y = –1
y – 4x = 8 1
8 ( 2) x y
14. (1) 3x + 5y = 15 1
y-axis 5 3

8 x y
8x + 7y = 56 1
7 7 8
6 y-axis
A 8
5
4 7
A 3
6
2

8x
1 C 5

+
x-axis

7y
B –2 –1 1 2 3 4 5 6 4

=
56
B3
Circumcentre
2 3x
Circumcentre lies on BC (–1, 0) and therefore BC + 5y
is hypotenuse 1 = 15
Intersection point of two lines D
O
FG 26 32
,
IJ 1 2 3 4 5
C
6 7
(x, y) = H 17 17 K
ABC dk area Tropezium ABCD dk area
1 32 = Area of AOD – Area of BOC
= BC
2 17 1 1
= ×8×7= ×3×5
1 32 128 2 2
= 8= sq. unit
2 17 17 1
= (56 – 15) = 20.5 sq. units
13. (1)  ABCD ,d parallelogram gSa 2
The diagonals of a parallelogram bisect each oth- 15. (1) x1y1 = x, y + z
er. x2y2 = x, y – z
x3y3 = –x, z
(1, 2) (x, y) Triangle dk area =
A D
LM 1 lx ,(y y3 ) x 2 (y3 y1 ) x 3 (y1 y2 )qOPQ
N2 2

M
LM 1 lx ,(y z z ) x (z y z ) x (y 2 y z) qOPQ
N2
B C LM 1 ( xy 2xz xy 2xz )OP
(–5, –3) (7, –6) N2 Q
AC osQ Mid point dk fu;ked LM 1 ( 4xz )OP = 2xz
FG 1 7 , 2 6 IJ N2 Q
16. (3)
= H 2 2 K = (4, –2)

osQ Mid Point M dk fu;ked =


FG 5 x 3 y
,
IJ A C B
BD H 2 2 K (2, 4) (5, 9) (x , y)
ar. of triangle = 0
Now,
x1 (y2 – y3) + x2 (y3 – y1) + x3 (y1 – y2) = 0

BLAM–129
funsZ'kkad T;kfefr
2 (9 – y) +5 (y – 4) + x (4 – 9) = 0 19.(2) Here = 30°
5x – 3y + 2 = 0 equation of line AB
17. (1) x cos + y sin = P
2 3 Ÿ x cos 30° + y sin 30° = P

A P B 3x y
Ÿ P
(a, 0) (3, 4) (0, b) 2 2
A/q
mx 2 nx1 Y
x=
m n B
2 0 3a
3=
2 3
a=5 P
Again,
my 2 ny1
y= 30°
m n
O A X
2b 3 0
4= b = 10
2 3
required the equation
x y
1
a b x y
Ÿ 1 [dividing Both sides by P]
x y
FG IJ
2P 2P

5 10
1 H 3K
2x + y = 0 2P
18. (1) Thus, OA = = OB = 2P
3
A 1
(1, 2) ar ( OAB) = OA OB
2
200 1 2P
Ÿ 2P
3 2
3
? P = 10.
Put the value of P

3x y
10
2 2
B D C
(2, 3) (3, 4) ? 3 x y 20
20.(1) Given the equation in the form
D, BC, dk Mid-point gSa x cos + y sin – P = 0
FG 2 3 , 3 4 IJ = FG 5 , 7 IJ 8x – 15y + 51 = 0 ls campare djus ij
D= H 2 2 K H 2 2K cos sin P
the equation of Median AD 8 15 51
y 2 x 1 cos sin P sin 2 cos 2 1
Ÿ
7 5 8 15 51 8 2
15 2 17
2 1
2 2 8 15
cos = , sin =
y 2 x 1 17 17
3 3 15
tan = ,P=3
Ÿ –3y + 6 = –3x + 3 8
Ÿ 3x – 3y + 6 – 3 = 0 FG IJ ˆˆˆ
15
?x – y + 1 = 0 P = 3, = tan–1 H K
8

BLAM–130
f=kdks.kfefr

3 f=kdks.kfefr (TRIGONOMETRY)
Definition : Trigonometry mathematics dh og branch (c) 175° = (90° + 85°) = 2nd quadrant
gS ftlesa ge fofHkUu izdkj osQ tSls sin T, cos T,
functions (d) 330° = (270° + 60°) = 4th quadrant
tan T, cosec T, sec T, cot T osQ ckjs esa vè;;u djrs gSaA bu (e) 405° = (360° + 45°) = 1st quadrant
functions ij vk/kfjr questions exams esa iwNs tkrs gSaA lkFk (f) 560° = (360° + 200°) = 3rd quadrant
gh lkFk trigonometry esa Height and distance dk Hkh viuk Definition of Angle : Circular measure system
vyx egRo gSA bl chapter esa Different types of questions esa angle dks bl izd kj define fd;k tkrk gSA
vkSj concepts osQ ckjs esa iw.kZ tkudkjh nh xbZ gSA Length of arc
Angle,
CIRCULAR MEASURES OF ANGLES Radius
1. Position of revolving line : tc nks perpendicular B
lines ,d nwljs dks fdlh point ij dkVrh gS] rks og plane dks 4
equal parts es a divide djrh gSA bu pkjksa equal parts dks First, r l
Second, Third rFkk Fourth quadrant osQ uke ls tkuk tkrk gSA
T
fdlh line osQ }kjk origin ls bu quadrants esa cuk angle O
r
different gksrk gS A Angles dh properties dks ge vkxs study A
djsaxsA
T = 90°
II I ;gk¡ ij]
T = nksuksa radii OA rFkk OB osQ chp dk angle.
l = arc AB dh length. [Arc dks pki Hkh dgk tkrk gSA]
T = 0° l
T = 180° T = 360° r
Systems of Measuring the Angles : Circular system

III IV esa angle dks 3 different system esa measure djrs gSaA
1. (Sexagesimal System) : bl system esa
T = 270° 1 Right angle = 90° (degree)
z tc line first quadrant esa gksrh gS rks og x- axis osQ lkFk 0° 1° = 60' (minute)
ls ysdj 90° osQ chp dk angle cukrh gSA 1' = 60'' (second)
z tc line second quadrant esa gksrh gS rks og x- axis osQ lkFk Note : Most important : budh values dks ;kn j[kuk cgqr
90° ls vf/d rFkk 180° ls de angle cukrh gSA t:jh gS D;ksfa d bu ij based questions exams esa iwNs tkrs gSAa
z tc line third quadrant esa gksrh gS rks og x- axis osQ lkFk 2. (Centesimal system) : bl system esa
180° ls vfèkd rFkk 270° ls de angle cukrh gSA 1 Right angle = 100 grade (100g)
z tc line fourth quadrant esa gksrh gS rks og x- axis osQ lkFk 1g = 100 minute (100')
270° ls vfèkd rFkk 360° ls de angle cukrh gSA 1' = 100 second (100")
Note : ,d complete pDdj esa line 360° dk angle cukrh Note : bl system ij based questions dHkh dHkkj iwN
gSA vxj angle 360° ls T;knk gksrk gS rks og nksckjk first, fy, tkrs gSaA
second, third ;k fourth quadrant esa vk tkrh gSA 3. Circular System : Degree rFkk Radian osQ chp osQ relation
Ex. In which quadrants the following angles do lie ? dks circular system esa cgqr vPNs <ax ls le>k;k x;k gSA
fn, x, angles dkSu ls quadrants esa vkrs gSa \ Degree rFkk Radians osQ vkilh conversions ij vk/kfjr
(a) 85° (b) 225° (c) 175° questions Exam esa iwN fy, tkrs gSaA
(d) 330° (e) 405° (f) 560° (i) Radian : 1 Radian (1C or 1R) og angle gksrk gS tks nks radii
Sol. (a) 85° = First quadrant OA rFkk OB osQ chp esa cuk gksrk gS rFkk arc AB dh yackbZ Hkh
(b) 225° = (180° + 45°) = 3rd quadrant radius osQ cjkcj gksrh gSA

BLAM–131
f=kdks.kfefr
Sol. ge tkurs gSa]
B

1 radian =
FG 180 IJ O

H K
C
1
O A
2 radians =
FG 180 2IJ O

H K
FG 180 2 7 IJ O
FG 22 IJ
=
H 22 K H 7 K
Relation among Degree, Grade and Redian :–

=
FG 1260 IJ O

D G C H 11 K
180 200
= 114° 32' 43"
Working Rule :
(i) Degree dks Grade esa cnyus osQ fy, fn, x, Degree osQ eku
11 1260 114° 32' 43"
10
dks ls Multiply djsaxsaA –11
9 16
–11
(ii) Grade dks Degree esa cnyus osQ fy, fn, x, Grade osQ eku
50
9 –44
dks 10 ls Multiply djsaxsA 6 × 60
360
(iii) Grade dks Redian esa cnyuss osQ fy, fn, x, Grade osQ eku –33
30
–22 1 60
dks 200 ls Multiply djsaxsA
8 × 60 1 60
(iv) Redian dks Grade esa cnyus osQ fy, fn, x, Redian osQ eku 480
–44
200 40
dks ls Multiply djsaxsA –33
7
(v) Degree dks Redian esa cnyus osQ fy, fn, x, Degree osQ eku

osQ 180 ls Multiply djasxsaA Ex. 3. Convert 150g to Redian


150g dks jsfM;u esa cnysa
(vi) Redian dks Degree esa cnyus osQ fy, fn, x, Redian osQ eku
Solution:- 150g = 150
FG IJ c

=
3 c
dks
180
ls Multiply djsaxsA H 200 K 4
c
Ex. 1. Convert 60° to Radians. Ex. 4. Convert to Grade.
50
60° dks Radian esa cnfy,A
Sol. ge tkurs gSa] c

dks Grade esa c<ysa


50
1° =
FG IJ C

H 180 K c
FG 200 IJ g

4g
Solution:-
50 H 50 K
60° =
FG 60
IJ C

radians.
Ex. 5. Convert 45° to Grade
H 180 K 3 45° dks Grade esa cnysa
Ex. 2. Convert 2 radians to degrees.
dks esa cnfy,A Solution:- 45° =
FG 45 10 IJ g

= 50g.
2 radians degree
H 9 K
BLAM–132
f=kdks.kfefr
Ex. 6. Convert 200g to Degree. Note : vkerkSj ij, minute hand rFkk hour hand osQ chp
200g dks Degree esa cnysa esa cus angle ij based question iwNs tkrs gSaA
Ex. What will be the angle between minute hand and
Solution:- 200g =
FG 200 9 IJ = 180° hour hand at 3 : 20 p.m. in any clock ?
H 10 K fdlh Hkh ?kM+h esa 3 : 20 p.m. ij minute hand rFkk hour
4. Sum of interior angles of polygon : Polygon ml hand osQ chp fdruk angle gksxk\
geometrical figure dks dgrs gSa ftldh n-sides gksrh gSA Sol. gesa hour rFkk minute hand osQ chp dk angle fudkyuk gSA
b2n 4g 90 igys ge 3 : 20 p.m. dks hours esa cnysaxs bl izdkj
Each interior angle = degree 3 : 20 p.m. = 3 hrs. + 20 min.
n
20
=
bn 2g radian = 3 hrs +
60
hrs.  (60 min. = 1 hr.)
.)
n
5. Sum of exterior angles of polygon : fdlh Hkh polygon 1 10
=3+ = hours.
osQ exterior angles dk sum gksrk gS 360° ;k 2 radian. 3 3
;fn polygon dh lHkh sides equal gS rks gj ,d exterior Hour hand }kjk 1 hr esa cuk;k x;k angle = 30°

angle gksxk
FG 360 IJ O
Hour hand }kjk
10
hr esa cuk;k x;k angle
HnK 3

Angles made by hour hand, minute hand and second


=
FG 10 30
IJ O

= 100°
hand. H3 K
(1) Hour hand
Minute hand }kjk 1 min. esa cuk;k x;k angle = 6°
Hour hand dh lwbZ iwjk ,d pDdj 12 ?kaVs esa yxkrh gSA Minute hand }kjk 20 min. esa cuk;k x;k angle
12 hrs. esa hour hand }kjk cuk;k x;k angle = 360° = 20 × 6 = 120°
360 Required angle = 120° – 100° = 20°
1 hr. esa hour hand }kjk cuk;k x;k angle =
12
30 Aliter.
60H 11M 60 3 11 20
1 ?kaVs esa hour hand 30° dk angle cukrh gS T= =
2 2
(2) Minute hand
40
Minute hand dh lwbZ iwjk ,d pDdj 60 feuV esa yxkrh gSA T= = 20°
2
60 minute esa minute hand }kjk cuk;k x;k angle
= 360° Trigonometric Ratios and their properties. fdlh
1 minute esa minute hand }kjk cuk;k x;k angle Hkh right angled triangle esa]
FG 360 IJ 6
(1) 90° osQ lkeus okyh side dks Hypotenuse dgrs gSa] rFkk ckdh
= H 60 K nksuksa cph gqbZ sides esa ls ,d base vkSj nwljh perpendicular
1 minute esa minute hand 6° dk angle cukrh gS gksrh gSA
(3) Second hand For example,
Second hand dh lwbZ iwjk ,d pDdj 60 second esa yxkrh gSA B

60 second esa second hand }kjk cuk;k x;k angle


= 360°
1 second esa second hand }kjk cuk;k x;k angle
360
= 6 A
60 C
BC = Hypotenuse D;ksafd ;g 90° osQ opposite gSA
1 second esa second hand 6° dk angle cukrh gS
(i) vxj ge angle B dks ysrs gSa] rks
(4) Hour hand rFkk minute hand os Q chp dk angle,
AC = perpendicular
60H 11M 11 AB = base
T= = 30H M
2 2 (ii) vxj ge angle C dks ysrs gSa] rks
tgk¡ gS rFkk
H = hour AB = perpendicular
M = minutes gSaA AC = base

BLAM–133
f=kdks.kfefr
Note : Hypotenuse fdlh Hkh triangle dh longest side Important Points :
gksrh gSA z Pythagoras theorem osQ vuqlkj
fdlh Hkh right angled triangle esa trigonometric ratios (Hyp)2 = (Perp)2 + (Base)2
dks 6 izdkj ls divide fd;k tkrk gSA Pythagoras theorem dk iz;ksx ml right angled triangle
C esa fd;k tkrk gS tgk¡ triangle dh dksbZ nks sides nh xbZ gksrh
gS rFkk rhljh side dks fudkyuk gksrk gSA

Perpendicular
se
nu z sin T, cos T dh value 1 ;k 1 ls NksVh gks ldrh gS ] ijarq dHkh
te
o Hkh 1 ls cM+h ugha gks ldrhA
yp
H Important relations :
T z sin T × cosec T = 1
A Base B z cos T × sec T = 1
z tan T × cot T = 1
Perpendicular = yEc
sin cos
Base = vk/kj Also, tan T = , cot T =
cos sin
Hypotenu se = d.kZ
z
ABC esa] sin 2  cos2 1 sec 2  tan 2 1
sin 2 1  cos2 sec 2 1  tan 2
Perp BC cos2 1  sin 2 tan 2 sec 2  1
sin T = Hyp AC
cosec 2  cot 2 1
Base AB cosec 2 1  cot 2
cos T = Hyp AC cot 2 cosec 2  1

Perp BC Some Important Results :–


tan T =
Base AB (1) ;fn a sin + b cos = c rks a cos T – b sin = ±

Hyp AC a 2
b 2
c 2

cosec T = Perp BC
;fn sin + cos =c rks sin T – cos =± 2 a2
Hyp AC (2) (sec + tan ) (sec – tan ) = 1
sec T =
Base AB
1
Base AB sec + tan = = P.
sec tan
cot T = Perp BC
1 1
sec – tan = .
P sec tan P
Some people Have S
H
(3) ;fn sec + tan =P ;fn sec – tan =P
B
Curly Brown Hair C P2 1 1 P2
H sin = sin =
P2 1 1 P2
P
Curly Brown Hair T
B 2P 2P
z
cos = p2 1 cos =
How to remember ? P2 1
mijksDr nh xbZ identities dks nks vyx&vyx rjhdksa ls ;kn
fd;k tk ldrk gSA P2 1 1 P2
tan = tan =
For English Medium 2P 2P
sin T cos T tan T sin T cos T tan T
P2 1 1 P2
cosec = cosec =
Pandit Badri Prasad P B P P2 1 1 P2
Har Har Bhole H H B
P2 1 2P
For Hindi Medium sec = sec =
2P 1 P2
sin T cos T tan T sin T cos T tan T
Lala Aam Laya L A L 2P 2P
Kachche Kachche Aam K K A cot = cot =
P2 1 1 P2

BLAM–134
f=kdks.kfefr
(4) ;fn cosec + cot =P ;fn cosec – cot =P 4cos2T + sin2T – 4 sinT . cosT + 4 sin2T + cos2T + 4
sinT.cosT
2P 2P
sin = sin = 1
P2 1 1 P2 = x2
2
P2 1 1 P2
cos = cos = 1
P2 1 1 P2 4(cos2T + sin2T) + (cos2T + sin2T) = + x2
2
2P 2P 1
tan =
P2 1
tan =
1 P2 x2 5
2
(5) a sin + b cos =c tgk¡ 2
a +b =c 2 2
1 9 3
x2 5 x
2 2 2
a c
5
b Ex 2. If sec T + tan T = then what is the value of sec T
12
+ tan T?
a b a
sin = , cos = , tan = ...etc 5
c c b ;fn sec T + tan T =
12
rks (sec T + tan T) dk eku D;k
(6) (i) If sin + cosec = 2 then sinn + cosecn =2
gksxk\
(ii) If sec + cos = 2 then cosn + secn =2
5
(iii) If tan + cos = 2 then tann + cosn =2 Solution : sec T + tan T =
12
(7) ;fn P sin + q cos = R ...(1)
rFkk P cos – q cos = S ...(2) 12
? sec T + tan T =
5
lehñ (1) vkSj (2) dk oxZ dj tksMu+ s ij
(P2 + q2) = (R2 + S2) Ex 3. If sec T+ tan T= 3 then what is the Value of cos T?
(8) (i) ;fn sin4 – cos4 = P rks sin2 – cos2 = P ;fn sec T + tan T= 3 rks cos Tdk eku D;k gksxk\
(ii) ;fn sec4 – tan4 = P rks sec2 + tan2 = P Solution : we know that
(iii) ;fn cosec4 – cot4 = P rks cosec2 + cot2 = P 2P
(9) ;fn A + B = 45° rks (i) (1 + tan A) (1 + tan B) = 2 If sec T + tan T = P then cos T = 2
P 1
rFkk (ii) (1 – cot A) (1 – cot B) = 2
(10)sin6 + cos6 = 1 – 3 sin2 . cos2 2 3 2 3 3
? cos T = 2
4 2 .
(11)sin4 + cos4 = 1 – 2 sin2 . cos2 e 3j 1
(12) ;fn A + B + C = 180° rks
(i) tan A + tan B + tan C = tan A. tan B. tan C 1
(ii) cot A. cot B + cot B. cot C + cot C. cot A = 1 Ex 4. If cosec T – cot T = then what is the value of
3
1 cos T
Ex 1. If 2cosT – sinT = , (0° < T < 90°) the value of
2 1
2sinT + cosT is ;fn cosec T– cot T =
3
rks cos T dk eku D;k gksxk\
1
;fn 2cos T – sinT = 2
, (0° < T < 90°) rks 2 sinT + cosT 1
Solution : Here P =
3
dh value gksxhµ
1 3 2 1
FG 1 IJ 2

(1)
2
(2) 2 (3)
2
(4)
3
cos T =
1 P2 H 3K
(SSC CHSL DEO & LDC Exam. 11.12.2011 1+ P 2
1
FG 1 IJ 2

(Ist Sitting (Delhi Zone)


H 3K
1
Solution : (3) 2 cosT – sinT =
2 1
1
3 2 1
2sinT + cosT = x (Let)
= 1 4 2.
nksuksa rjiQ square rFkk add djus ij 1
3

BLAM–135
f=kdks.kfefr
Ex 5. If (a – b ) sin T+ 2 ab cos T= a + b then tan T = ?
2 2 2 2
(1) –1 (2) 0 (3) sec2 x (4) 1
;fn (a – b ) sin T+ 2 ab cos T= a + b
2 2 2 2
rks tan T = ? (SSC CHSL DEO & LDC Exam. 11.12.2011
(IInd Sitting (Delhi Zone)
Solution :
Solution : (4) (secx . secy + tanx . tany)2 – (sec x . tan
a 2 y + tan x . sec y)2

a2–b2
+b 2
= sec2 x . sec2 y + tan2 x . tan2y + 2 sec x . sec y . tan
x . tan y – sec2 x . tan2 y – tan2 x . sec2y – 2 sec x . sec
y . tan x . tan y
2ab
we know that = sec2 x . sec2y + tan2x . tan2 y – sec2 x . tan2y – tan2
x . sec2y
a sin T + b cos T = c where a2 + b2 = c2
= sec2x . sec2y – sec2x . tan2y – tan2x . sec2y + tan2 x
a 2 b2 . tan2y
tan T = . = sec2x (sec2y – tan2y) – tan2x (sec2y – tan2y)
2ab
Ex 6. If sec T + cos T = 2 then what is the value of cos13 T = sec2x – tan2x = 1
+ sec13 T. Ex 11.If A = sin2 T + cos4T, for any value of T, then the value
;fn sec T + cos T = 2 rks cos13 T + sec13 T dk eku D;k of A is
gksxk\ ;fn A = sin2 T + cos4T, T dh dksbZ value gks] rks A dh value
Solution : D;k gksxh \
If sec T + cos T = 2 then cosn T + secn T = 2
3
Ex 7. If a sin T + b cos T = 2 and b sin T – a cos T = 1 then (1) 1 A 2 (2) A 1
4
a2 + b2 = ?
Solution : 13 3 13
(3) A 1 (4) A
a sin T + b cos T = 2 ...(1) 16 4 16
b sin T – a cos T = 1 ...(2) (SSC CHSL DEO & LDC Exam. 11.12.2011
(1)2 + (2)2 we get (Ist Sitting (East Zone)
a2 + b2 = 22 + 12 = 5. Solution : (3) tc T = 0°
sin2T + cos4T = 1
7
Ex 8. If sec2 T + tan2 T = then sec4 T – tan4 T = ? tc T = 45°,
12
Solution : LM 1 1 OP
 sin 45 & cos 45
We know that N 2 2 Q
7 1 1 3
sec4 T – tan4 T = sec2 T + tan2 T = sin2T + cos4T =
12 2 4 4
Ex 9. 3 tan 40 tan 80 ? LMsin 30 1
&cos 30
3 OP
tc T = 30°,
(1)
3 1
(2) sin 40° + cos 80°
MN 2 2 PQ
2 2
1 9
sin 2 cos 4
(3) 3 tan 40 . tan 80 (4) cot 40 tan 80 3 4 16
Solution : 4 9 13
= =
We know that 16 16
A + B + C = 180° then blfy,] A dh value gksxh
tan A + tan B + tan C = tan A. tan B. tan C
13
Now A = sin2 T + cos4 T =
16
3 tan 40 tan 80 Ex 12. If cosecT – sinT = l and secT – cosT = m, then the
= tan 60° + tan 40° + tan 80° value of l2m2(l2 + m2 + 3) is
= tan 60°. tan 40°. tan 80° ;fn cosecT – sinT = l rFkk secT – cosT = m, gks rks l2m2
[60 + 40 + 80 = 180° using the above Result] (l2 + m2 + 3) dk eku gksxkµ
= 3 . tan 40 . tan 80 (1) –1 (2) 0 (3) 1 (4) 2
(SSC Delhi Police S.I. (SI) Exam. 19.08.2012)
Ex 10.The simplified value of (sec x sec y + tan x tan y)2 –
(sec x tan y + tan x sec y)2 is : Solution : (3) (l2.m2) (l2 + m2 + 3)
(sec x sec y + tan x tan y)2 – (sec x tan y + tan x sec = (cosec T – sin T)2 (sec T – cos T)2
y)2 dks gy djus ij izkIr gksxkµ {(cosec T – sin T)2 + (sec T –cos T)2 + 3}

BLAM–136
f=kdks.kfefr

=
FG 1 sin
IJ FG 1
2

cos
IJ
2
Solution : (1) sec tan 2 5
H sin K H cos K
1
R|F 1 IJ FG 1
2
IJ 2
U| sec tan
S|GH sin sin
K H cos cos
K 3V
|W
5 2
T
 sec 2 tan 2 1
F 1 sin IJ FG1 cos IJ
= G
2 2 2 2

H sin K H cos K 5 2
= 5 2
R|F1 sin I 2
2
FG1 cos IJ 2 U| 2
e 5 2 je 5 2 j
S|GH sin JK H cos K 3V
|W
T On adding,

F cos IJ FG sin IJ R|SFG cos IJ


= G
2
2
2
2 2
2
FG sin2
IJ 3U|V
2 2 sec 2 5 5 2 2 5

H sin K H cos K |TH sin K H cos K |W sec 5 cos


1
5
cos4 sin4 RScos 4
sin 4 U
3V On subtracting,
=
sin2 cos2 T sin 2
cos2 W 2 tan 2 5 5 2 4
Rcos 6
sin 6 3 cos2 .sin 2 UV tan T = 2
= cos T × sin T S
2 2

T cos .sin 2
2
W tan 2
= cos6 T + sin6 T + 3cos2 T . sin2 T sin
sec 5
= (cos2 T)3 + (sin2 T)3 + 3 cos2 T . sin2 T
= {(cos2T + sin2T)3 – 3 cos2T . sin2T (cos2T + sin2T)} +
3cos2T . sin2T
2 1 3
sin cos =
[  a3 + b3 = (a + b)3 5 5 5
– 3ab (a + b)]
Ex 14.If x = a sec T cos , y = b sec T sin , z = c tan T,
= 1 – 3 cos2T . sin2T + 3 cos2T . sin2T = 1
Aliter : cosec T – sin T = l x2 y2 z2
sec T – cos T = m then the value of is :
a2 b2 c2
ekuk 45°
1 1
;fn x = a sec T cos , y = b sec T sin , z = c tan T,
l = cosec 45° – sin 45° = 2 =
2 2 x2 y2 z2
gks rks fdlosQ cjkcj gksxkµ
1 1 a2 b2 c2
m = sec 45° – cos45° = 2 =
2 2 (1) 1 (2) 4 (3) 9 (4) 0
2 2 2
l m (l + m2 + 3) (SSC Graduate Level Tier-I Exam. 21.04.2013)
Solution : (1) x = a sec T. cos ; y = b sec T. sin
=
1 FG
1 1 1 IJ
3 =
1
4= 1 , z = c tan T
2 H
2 2 2 K 4
x2 y2 z2
Ex 13.If sec T + tan T = 2 + 5 , then the value of sin T +
a2 b2 c2
cos T is :
= sec2 T. cos2 + sec2 T.sin2 – tan2 T
;fn sec T + tan T = 2 + 5 , rks sin T + cos T fdlosQ = sec T (cos
2 2
+ sin 2
) – tan T 2

cjkcj gksxkµ = sec2 T – tan2 T = 1

3 7 1 sec tan 5
(1) (2) (3) (4) Ex 15.If , then sin T is equal to :
5 5 5 5 sec tan 3

(SSC Graduate Level Tier-I sec tan 5


Exam. 21.04.2013, Ist Sitting)
;fn sec tan 3
, gks rks sin T fdlosQ cjkcj gksxkµ

BLAM–137
f=kdks.kfefr
1 1 2 3 1 3
(1) (2) (3) (4) Solution : (4) 2 sin 2
4 3 3 4 1 cot 2 1 tan 2
(SSC Graduate Level Tier-I Exam. 21.04.2013)
1 3
sec tan 5 = 2 sin 2
Solution : (1) cos ec 2 sec 2
sec tan 3
5 sec T – 5 tan T = 3 sec T + 3 tan T = sin 2 3 cos2 2 sin 2
2 sec T = 8 tan T
2
= 3 (sin cos2 ) = 3
tan 2 1
sec 8 4 Ex 18. The value of

sin
cos
1 LM cos A (sin A cos A) +
2 sin 2 A (sin A cos A) OP
cos 4
MN cosec A (sin A cos A)
2 sec 2 A (sin A cos A) PQ
1
sin T = (sec2 A – cosec2 A)
4
(1) 1 (2) 3 (3) 2 (4) 4
Ex 16.If (l + sin ) (l + sin ) (l + sin ) = (1 – sin )(l – sin )
(SSC Graduate Level Tier-I Exam. 19.05.2013)
(l – sin ), then each side is equal to
Solution : (3) Expression
;fn (l + sin ) (l + sin ) (l + sin ) = (1 – sin )(l – sin )
(l – sin ), gks rks izR;sd side fdlosQ cjkcj gksxkµ LM cos A(sinA cosA)
2
OP
sin 2 A(sinA cosA)
(1) ± cos cos cos (2) ± sin sin sin =
(3) ± sin cos cos (4) ± sin sin cos
N cosec A(sinA cosA)
2
sec 2 A(sinA cosA) Q
(SSC Graduate Level Tier-I Exam. 21.04.2013)
Solution : (1) (1 + sin ) (1 + sin ) (1 + sin ) = (1 – ×
FG 1 – 1 IJ
sin ) (1–sin ) (1– sin ) = x H cos2 A sin 2 A K
x.x = (1 + sin ) (1 – sin ) (1 + sin ) (1 – sin )(1 +
sin ) (1– sin ) LM cos A.sin A (sinA
2 2
cosA) sin 2 A.cos2 A(sinA – cosA) OP
=
= (1 – sin 2 2
) (1 – sin )(1 – sin ) 2
N sinA – cosA (sinA cosA) Q
= cos2 . cos2 . cos2
x= cos . cos . cos F sin A – cos A I
2 2
Ex 17.The numerical value of GH sin A.cos A JK
2 2

1 3
2 sin 2 will be
1 cot 2 tan 2
1
=
FG sinA cosA sinA cosA IJ (sin 2 A cos 2 A)
1 3
H sinA cosA sinA cosA K
2 sin 2 dh value fdlosQ cjkcj
1 cot 2 1 tan 2
LM (sin A c osA)2 (sinA cosA)2 OP
gksxhµ = (sin 2 A cos 2 A)
(1) 2 (2) 5 (3) 6 (4) 3
MN (sinA cosA)(sinA cosA) PQ
(SSC Graduate Level Tier-I
Exam. 19.05.2013 Ist Sitting) = 2(sin 2 A cos 2 A) = 2

0° 30° 45° 60° 90° Value/ eku

1 1 3
sinT 0 1 0° ls 90° osQ chp sin T dh value c<+rh gS
2 2 2

3 1 1
cosT 1
2 2
0 0° ls 90° osQ chp cos T dh value ?kVrh gS
2
1
tanT 0
3
1 3 0° ls 90° osQ chp tan T dh value c<+rh gS

BLAM–138
f=kdks.kfefr
1
cotT 3 1
3
0 0° ls 90° osQ chp cot T dh value ?kVrh gS

2
secT 1
3 2 2 0° ls 90° osQ chp sec T dh value c<+rh gS

2
cosecT 2 2 3
1 0° ls 90° osQ chp cosec T dh value ?kVrh gS

Remember (;kn djuk) :- Value * Different quadrants esa Trigonometric ratios osQ sign.
STS Increases (c<+uk)
(1) After
(2)
CCC Decreases (de gksuk) School
S sin T All+ve
Values of trigonometric functions in various quadrants. cosec T sin T
+ve
vkerkSj ij, coordinate system dks ge 4 equal quadrants cosec T
esa divide djrs gSaA bu quadrants esa trigonometric identities dk 90° < T°< 180° 0° < T°< 90°
sign cnyrk gS rFkk trigonometric identities Hkh vkil esa cnyrh
gSA 180° 0°;k 360 °
(3) (4) No change
(i) 1st quadrant (0° < T < 90°) : vxj angle (T) 0° ls cM+k dksbZ ifjorZu ugha gksxk
180° < T°< 270° 270° < T°< 360°
vkSj 90° ls NksVk gksxk rks og 1st quadrant esa gksxkA
+ ve + ve
Ex. (45°, 60°, 70°, 89° etc.) To

(ii) 2nd quadrant (90° < T < 180°) : vxj angle (T) 90° ls College

cM+k rFkk 180° ls NksVk gksxk rks og 2nd quadrant esa 270°
value change
gksxkA Ex. (95°, 120°, 150° etc.) (ifjorZu gksxk )
(90° ;k 270°)
(iii) 3rd quadrant (180° < T < 270°) : vxj angle (T) 180°
ls cM+k rFkk 270° ls NksVk gksxk rks og 3rd quadrant esa Add Sugar to coffe
gksxkA Ex. (185°, 225°, 260° etc.) 1st quadrant B B B B
0 90 All sin tan cos
(iv) 4th quadrant (270° < T < 360°) : vxj angle (T) 270°
ls cM+k rFkk 360° ls NksVk gksxk rks og 4th quadrant esa Angle 1st quadrant esa rHkh gksxk tc oks 0° ls cM+k rFkk 90°
gksxkA Ex. (280°, 315°, 330° etc.) ls NksVk gksA ge vxj 90° esa ls dksbZ angle (T) minus djsaxs rks og
Identity change gks tkrh gSS 90 rFkk 270 ij ges'kk 1st quadrant esa jgsxkA lkFk gh ;g Hkh ;kn jgs dh 1st
i.e. sine cos quadrant esa lHkh identities (+ve) gksrh gS rFkk 90° ij identity

tan cot cny tkrh gSA


sec cosec sin (90° – T) = cos T gSa rFkk identity
All (+ve)

Identity same jgrh gS 180 rFkk 360 ij cos (90° – T) = sin T cny xbZ gSA
i.e. sine sine tan (90° – T) = cot T
cos cos cosec (90° – T) = sec T
tan tan
sec (90° – T) = cosec T
cosec cosec
cot (90° – T) = tan T
sec sec
cot cot Ex. sin 55° = sin (90° – 35°)
Identity dks ;kn djus dk rjhdk = cos 35°
After : All +ve gksaxsA tan (75°) = tan (90° – 15°)

School : fliZQ sin T rFkk cosec T +ve gksaxsA = cot 15°

To : fliZQ tan T rFkk cot T +ve gksaxsA 2 nd quadrant


College : fliZQ cos T rFkk sec T +ve gksaxsA 90 180

BLAM–139
f=kdks.kfefr
;fn 90° esa dksbZ angle T tksM+ ns ;k 180° esa ls dksbZ angle * cot (180° + T) = cot T* *cot (270° – T) = tan T
T ?kVk nsa rks izkIr gqvk angle 2nd quadrant esa gksxkA Note :180° ij identity Note : 270° ij identity
* sin (90° + T) = cos T * *sin (180°– T) = sin T * same jgrh gS rFkk 3rd cny tkrh gSA
cos (90° + T) = – sin T cos (180°– T) = – cos T quadrant esa fliZQ tan

tan (90° + T) = – cot T tan (180°– T) = – tan T vkSj cot positive gksrs gSaA
* cosec (90° + T) = sec T* *cosec (180°–T) = –cosec T Ex. Evaluate : (i) sec 225° (ii) cot 240°
sec (90° + T) = –cosecT sec (180° – T) = –sec T Sol. (i) sec 225° = sec (180° + 45°)
cot (90° + T) = – tan T cot (180° – T) = –cot T = – sec 45°
90° ij identity cny 180° ij identity ugha
=– 2
tkrh gSA cnyrh gSA
or
2nd quad. esa sine rFkk
sec (270° – 45°)
cosec (+ve) gksrs gSAa
= – cosec 45°
Ex. Evaluate : (i) sin 135° (ii) cos 150° (iii) tan 120° =– 2
Sol. (i) sin 135° = sin (90° + 45°)
(ii) cot 240° = cot (180° + 60°)
= cos 45°
= cot 60°
1
= 1
2 =
or 3
sin 135° = sin (180° – 45°) or
1 cot (270° – 30°)
= sin 45°=
2 = tan 30°
(ii) cos 150° = cos (90° + 60°)
1
=
3 3
= – sin 60°= –
2
or
4 t h quadrant
cos (180° – 30°)
270 360
3
= – cos 30° = –
2
;fn 270° esa dksbZ angle T tksM+k tk, ;k 360° esa ls dksbZ
(iii) tan 120° = tan (90° + 30°)
= – cot 30° angle T minus fd;k tk, rks angle 4th quadrant esa gksxkA
=– sin (270° + T) = – cos T sin (360°– T) = – sin T
3
or * cos (270° + T) = sin T* *cos (360°– T) = cos T*
tan (180° – 60°) tan (270° + T) = – cot T tan (360°– T) = – tan T
= – tan 60° = – 3 cosec (270°+T) = – sec T cosec (360°–T) =– cosec T
* sec (270° + T) = cosec T* *sec (360° – T) = sec T*
3 rd quadrant
cot (270° + T) = – tan T cot (360° – T) = – cot T
180 270
Note : 270° ij identity Note : 360° ij identity
;fn 180° esa dksbZ angle T tksM+ ns ;k 270° esa ls dksbZ angle cny tkrh gSA rFkk 4th same jgrh gSA
T minus djs rks angle ges'kk 3rd quadrant esa jgsxkA quadrant esa fliZQ cos T
sin (180° + T) = – sin T sin (270°– T) = –cos T vkSj sec T (+ ve) gksrs gSAa
cos (180° + T) = – cos T cos (270°– T) = – sin T Some Important Results:
* tan (180° + T) = tan T *tan (270°– T) = cot T ;fn T1 + T2 = 90° rks
cosec (180°+T) = –cosecT cosec (270°–T) = – sec T (i) sin2T1 + sin2T2 = 1
sec (180° + T) = – sec T sec (270° – T) = –cosec T (ii) cos2T1 + cos2T2 = 1

BLAM–140
f=kdks.kfefr
(iii) cosec T1 – tan T2 = 1
2 2
tan x tan y
(iv) sec2T1 – cot2T2 = 1 (vi) tan (x – y) = 1 tan x . tan y
sin 1
(v) sinT1 = cosT2 or cos 1
cot x .cot y 1
2
(vii) cot (x + y) = cot x cot y
tan 1
(vi) tanT1 = tanT2 or tan 1
cot x .cot y 1
2
(viii) cot (x – y) = cot y cot x
cot 1
(vii) cotT1 = cotT2 or cot 1 (ix) sin (x + y) sin (x –y) = sin2 x – sin2 y
2
(x) cos (x + y) sin (x –y) = cos2 x – sin2 y
(vii) tan .tan =1
(ix) cot
1
.cot
2
=1
oqQN vkSj Trigonometric Formulae
1 2
(x) sin .sec =1 (a) 2 sin x cosy = sin (x + y) + sin (x – y)
1 2
(b) 2 cosx siny = sin (x + y) – sin (x – y)
(xi) cos 1
.cosec 2
=1
(c) 2 cosx cosy = cos (x + y) + cos (x – y)
(d) 2 sinx siny = cos (x – y) – cos (x + y)
Ex.1. Evaluate : (i) cos 330° (ii) cot 315°
Sol. (i) cos 330° = cos (270° + 60°) For all x, y R, we have / lHkh x, y R osQ fy,

= sin 60° =
3 FG x y IJ cosFG x y IJ
2 (i) sinx + siny = 2sin
H 2 K H 2 K
or
(ii) sinx – siny = 2cos G
F x y IJ sin FG x y IJ
cot (360° – 30°) = cos 30° =
3 H 2 K H 2 K
2
(ii) cot 315° = cot (270° + 45°) = – tan 45° = – 1 (iii) cosx + cosy = 2cos G
F x y IJ cosFG x y IJ
or H 2 K H 2 K
cot (360° – 45°) = – cot 45° = – 1
Also,
(iv) cosx – cosy = –2sin G
F x y IJ sin FG x y IJ
sin (– T) = – sin T H 2 K H 2 K
cos (– T) = cos T
2T ij based Trigonometric formulae
tan (– T) = – tan T
cosec (– T) = – cosec T 2 tan x
sec (– T) = sec T (i) sin2x = 2sinx cosx =
1 tan 2 x
cot (– T) = – cot T
(ii) cos2x = (cos2x – sin2x) = (2cos2x – 1)
Ex. 2. Evaluate : (i) sin (–30°) (ii) tan (–45°)
(iii) cos (– 60°) (iv) cosec (–30°) 1 tan 2 x
= (1 – 2 sin2x) =
1 1 tan 2 x
Sol. (i) sin (– 30°) = – sin 30° = –
2
(ii) tan (– 45°) = – tan 45° = – 1 2 tan x
(iii) tan2x =
1 tan 2 x
1
(iii) cos (– 60°) = cos 60° = Important Results
2
(iv) cosec (– 30°) = – cosec 30° = – 2 (i) (1 – cos2x) = 2sin2x
(ii) (1 + cos2x) = 2cos2x
Formulae :
FG 1 cos 2x IJ tan 2 x
(i) sin (x + y) = sin x. cos y + cos x. sin y
(iii)
H1 cos 2x K
(ii) sin (x – y) = sin x. cos y – cos x. sin y
3T ij based Trigonometric Formulae
(iii) cos (x + y) = cos x. cos y – sin x. sin y (i) sin3x = 3sinx – 4sin3x
(iv) cos (x – y) = cos x. cos y + sin x. sin y (ii) cos3x = 4cos3x – 3cosx
tan x tan y 3 tan x  tan 3 x
(v) tan (x + y) = 1 tan x . tan y (iii) tan3x =
1  3 tan 2 x

BLAM–141
f=kdks.kfefr
Trigonometric half angle formulae A
;gk¡ x R

x
2 tan
x x 2 c b
(i) sinx = 2 sin .cos =
2 2 2 x
1 tan
2

x x x C
(ii) cos x = cos
2
sin 2 2 cos2 1 B
2 2 2 a
tgk¡] a = BC = A osQ opposite side dh length.

1 tan
x2 b = AC = B osQ opposite side dh length.
x 2
= 1 2 sin 2 c = AB = C osQ opposite side dh length.
2 x
1 tan 2 Cosine Rule :
2
b2 c2 a2
cos A =
x 2bc
2 tan
(iii) tan x = 2
x a2 c2 b2
1 tan 2 cos B =
2 2ac

2 x a2 b2 c2
(iv) 1 – cos x = 2 sin cos C =
2 2ab
Area of triangle :
2 x
(v) 1 + cos x = 2 cos 1 1
2 (i) bc sin A (ii) ac sin B
2 2
Important Results
w.k.t 1
(iii) ab sin C
A + B + C = 180 2
A B 180 C Circum radius of a circle = R
= A
2 2
R b
FG A B IJ = sin FG 90 C IJ c
R
sin
H 2 K H 2K R

B C

sin G
F A B IJ = cos C a
(i)
H 2 K 2

cos G
F A B IJ = sin C fdlh Hkh triangle ds rhuksa vertex dks ckgj ls touch djrs
(ii)
H 2 K 2
gq, cuk, x, circle dks circum circle dgrs gSaA
tan G
F A B IJ = cot C a b c
(iii)
H 2 K 2 (i) R =
2 sin A
= =
2sin B 2sin C

sec G
F A B IJ = cosec C abc
(ii) R =
(iv)
H 2 K 2 4
In radius of a circle (r)

cosec G
F A B IJ = sec C
(v)
H 2 K 2 A
b
cot G
F A B IJ = tan C r
(vi)
H 2 K 2 c
r
Sine Rule : fdlh ABC esa] r
a b c
= 2R
sin A sin B sin c B a C

BLAM–142
f=kdks.kfefr
dksbZ circlevxj fdlh triangle ds vanj mldh rhuksa side dks cosec cot
touch djrs gq, cuk;k tk, rks mls incircle dgrs gSaA (xv) For Nr rFkk Dr dks cosec T + cot T ls
cosec cot
a b c multiply djsaA
S
tgk¡ S =
Inradius, r =
2
a a sin b cos
Range of Trigonometric ratio. (xvi) ;fn tan T =
b
fn;k gks rks a sin b cos
dh value
1. – 1< sin T < 1 | sin T | < 1
2. – 1< cos T < 1 | cos T | < 1 fudkyus osQ fy, Nr rFkk Dr dks cos T ls divide djsaA
3. – < tan T < tan T dksbZ Hkh real value ys ldrk gSA FG tan sin IJ
4. cosec T < –1 rFkk cosec T > | 1 cosec T | > 1 H cos K
5. sec T < –1 rFkk sec T > 1 | sec T | > 1
a a sin b cos
vc rd geus mu lHkh important relations vkSj formulae (xvii) ;fn cot T =
b
fn;k gks rks a sin b cos
dh value
osQ ckjs esa i<+k ftudh t:jr questions dks solve djus esa cgqr fudkyus osQ fy, Nr
rFkk Dr
dks sin T ls divide djsaA
ykHknk;d gksrh gSA vc ge oqQN vkSj important substitutions /
FG cot cos IJ
formulae ns[ksx a s tks questions dks djus esa cgqr helpful gksaxsaA H sin K
(i) For a – x2, put x = a sin T or x = a cos T
2

a a sin 2 b cos 2
(ii) For x2 – a2, put x = a sec T or x = a cosec T (xviii) ;fn tan T = gks rks dh value
b a sin 2 b cos 2
(iii) For x2 + a2, put x = a tan T or x = a cot T
fudkyus osQ fy, Nr rFkk Dr dks cos2 T ls divide djsaA
1 tan
tan G
F IJ a a sin 2
b cos 2
(iv)
1 tan H4 K (xix) ;fn cot T =
b
gks rks
a sin 2 b cos 2
dh value

1 tan
tan G
F IJ fudkyus osQ fy, Nr rFkk Dr dks sin2 T ls divide djsaA
(v)
1 tan H4 K (xx) ;fn sin T + cosec T = 2 gks rks sinp T + cosq T = 2 gksxk
tgk¡ p, q real numbers gSA
1 sin
FG sin cos
IJ
(vi) H 2 2 K (xxi) ;fn cos T + sec T = 2 gks rks cosp T + secq T = 2 gksxk tgk¡
p, q real numbers gSA
1 sin
FG sin cos
IJ (xxii) ;fn tan T + cot T = 2 gks rks tanp T + cotq T = 2 gksxk tgk¡
(vii)
H 2 2 K p, q real numbers gSA
(viii) 1 sin 2 = (sin T + cos T) (xxiii) a sin T + b cos T dh maximum value = a2 b2
(ix) 1 sin 2 = (sin T – cos T)
(xxiv) a sin T + b sin T dh maximum value = a2 b2
1 cos (xxv) a cos T + b cos T dh maximum value =
(x) For Root esa Nr rFkk Dr dks (1 + cos T) ls a2 b2
1 cos
multiply djsaA (xxvi) Minimum value ges'kk gksxh – a 2 b 2
(xxvii) a sin2 T + b cos2 T dh minimum value gksxh a rFkk b
1 cos
(xi) For Root esa Nr rFkk Dr dks (1 – cos T) ls esa tks minimum gksxkA
1 cos
multiply djsaA (xxviii) a tan2 T + b cot2 T dh minimum value = 2 ab

1 sin (xxix) a cos T + b sin T = c dk solution gksxkA c a2 b2


(xii) For Root esa Nr rFkk Dr dks (1 + sin T) ls
1 sin osQ fy,A
multiply djsaA

1 sin
3 1
sin 15
(xiii) For
1 sin
Root esa Nr rFkk Dr dks (1 – sin T) ls 2 2

multiply djsaA
E
cos 75
sec tan sin 75 cos15
(xiv) For Nr rFkk Dr dks sec T + tan T ls 3 1
sec tan
multiply djsaA
2 2

BLAM–143
f=kdks.kfefr
oqQN Trigonometric values ;kn djus ;ksX; % Ÿ if x + y + z = 180° or S (;k xyz ,d f=kHkqt gS)
x + y = 180 – z
5 1
(i) sin 18° = cos 72° = nksuksa i{kksa esa tan ysus ij
4
tan (x + y) = tan (180° – z)

10  2 5 tan x tan y
(ii) sin 36° = cos 54° = = –tanz
4 1 tan x tan y

10 2 5 tan x tan y tan z tan x tan y tan z


(iii) cos 18° = sin 72° =
4
blh izdkj
5 1
(iv) cos 36° = sin 54° = cot x cot y cot y cot z cot z cot x 1
4

(v) tan 1°. tan 2° -------- tan 89° =1


Ex 2. tan 25° + tan 85° + tan 70° = ?
(vi) cot 1° cot 2° -------- cot 89° =1
Sum of Angles = 25 + 85 + 70 = 180°
(vii) cos 1° cos 2° -------- cos 90° = 0
(viii) cos 1° cos 2° -------- cos 180° = 0 So, tan 25° + tan 85° + tan 70° = tan 25° tan 85
tan 70°
(ix) sin 1° sin 2° -------- sin 180° = 0

(x) sin 1°. sin 2° -------- (sin180° ls cM+k) = 0


If x + y + z = 90° or ( ;k xyz ,d f=kHkqt gS)
2
3T ij Based Trigonometric Identity
x + y = 90° – z
1 (nksuksa rjiQ tan ysus ij)
(i) sinT˜sin(60 + T) ˜ (sin 60 – T) = sin3T
4
tan (x + y) = tan (90° – z)

1 tan x tan y
(ii) cosT˜cos(60 + T) ˜ (cos 60 – T) = cos3T 1
4 1 tan x tan y = cotz = tan z
(iii) tanT˜tan(60 + T) ˜ (tan 60 – T) = tan3T
tanx tanz + tany tanz = 1 – tanx tany
(iv) cotT˜cot(60 + T) ˜ (cot 60 – T) = cot3T
Ex 1. 1 – cos 10° ˜ cos 50° ˜ cos 70° tan x tan y tan y tan z tan z tan x 1

T = 10° j[kus ij blh izdkj

1 cot x cot y cot z cot x cot y cot z


cos 10° ˜ cos (60° + 10°) ˜ cos (60° – 10°) = cos 3 × 10°
4
Ex 3. ABC esa]

1 x y y z z x
cos 10° ˜ cos 50° ˜ cos 70° = cos 30° tan tan tan tan tan tan ?
4 2 2 2 2 2 2
Solution : (3) x + y + z = 180°

3 x y z
?1 – cos 10° ˜ cos 50° ˜ cos 70° = 1 90
8 2 2 2

x y y z z x
tan tan tan tan tan tan 1
8 3 2 2 2 2 2 2
=
8 ‡‡‡

BLAM–144
f=kdks.kfefr

PREVIOUS YEARS’ QUESTIONS


6. A railway track has to be made as circular path. If
Type-1
25° has to be changed going on 40m on that path,
Degree, Radians, buosQ vkil esa then what should be the radius of that path ?

Conversions, clock osQ hands osQ chp esa ,d jsy&ekxZ dk oØ] ,d o`Ùk ds vuqlkj cukuk gSA rnuqlkj]
cus angle ij based questions ;fn ml ekxZ dh 40 ehVj dh nwjh esa ekxZ dh fn'kk esa 25° dk
ifjorZu djuk gks] rks ml o`Ùk dh f=kT;k fdruh j[kuh gksxh\
22 (1) 91.64 m (2) 90.46 m
1. The degree measure of 1 radian (taking = ) is
7 (3) 89.64 m (4) 93.64 m
(SSC CGL Tier-I Exam, 2012)
22 7. The angles of a triangle are in A.P. The ratio of mininum
1 jsfM;u dk fMxzh esa eki ( =
7
ekudj) fdruk gksrk gS\
angle in degree and maximum angle in radian is 60 :
(1) 57°61’22” (approx.) . The angle are —
(2) 57°16’22” (approx.) ,d f=kHkqt osQ dks.k] lekarj Js.kh esa gSaA muesa U;wure dks.k osQ
(3) 57°22’16” (approx.) va'k vkSj vfèkdre dks.k osQ jsfM;u dk vuqikr 60 : gSA
(4) 57°32’16” (approx.)
(SSC CGL Tier-I Exam. 21.04.2013, Ist Sitting)
rnuqlkj] os dks.k fdrus va'kksa osQ gSa\
(1) 30°, 60°, 90° (2) 35°, 55°, 90°
2.
FG 3 IJ radians is equal to /
FG 3 IJ jsfM;u fdlds cjkcj
H5K H5K (3) 40°, 50°, 90° (4) 40°, 55°, 85°
(SSC Graduate Level Tier-II Exam. 16.09.2012)
gS \ 8. How much radian is every angle of an octagon ?
(1) 100° (2) 120° (3) 108° (4) 180°
,d le v"VHkqt dk izR;sd vkarfjd dks.k fdrus jsfM;u dk gksrk
(SSC Graduate Level Tier-I Exam. 19.05.2013)
3. Radian measurement of 63° 14' 51" is—
gS\
63° 14´51" dk jsfM;u eki gSµ 3 2 1
(1) (2) (3) (4)
4 4 3 3
FG 2811S IJ c
FG 3811S IJ c
(SSC Graduate Level Tier-II Exam. 29.09.2013. 2nd Sitting)
(1)
H 8000 K (2)
H 8000 K 9. The circular measure of an angle of an isosceles tri-
5
F 4811S IJ
(3) G
c
F 5811S IJ
(4) G
c angle is
9
. Circular measure of one of the other
H 8000 K H 8000 K angles must be
(SSC CGL Tier-I Exam, 2012) ,d lef}ckgq f=kHkqt ds ,d (vleku) dks.k dk o`Ùkh; eki
4. 50 cm long pendulum makes 16 cm arc. Then the
angle on centre will be (approx.)? 5
9
gSA rnuqlkj mlds vU; dks.kksa ds o`Ùkh; eki D;k gksax\s
50 lseh yEckbZ dk ,d yksyd] tc Mksyrk gS] rks 16 lseh dk
pki cukrk gSA rnuqlkj] bl izdkj tks dks.k cusxk] mldk eki 5 5 2 4
(1) (2) (3) (4)
(yxHkx) fdruk gksxk ? 18 9 9 9
(1) 18°25' (2) 18°35' (3) 18°20' (4) 18°08' (SSC FCI Assistant Grade-III Main Exam. 07.04.2013)
(SSC CGL Tier-I Exam, 2012) 10. What will be the angle between minute hand and hour
5. A wheel rotates 3.5 times in a second. How much hand at 2 : 15 P.M. in any clock ?
time does wheel take to rotate 55 radian ? (in second)
2 : 15 vijkÉ ds le; fdlh ?kM+h esa ?kaVs dh lqbZ rFkk feuV
,d ifg;k ,d lsdaM esa 3.5 ckj ?kw.kZu djrk gSA ifg;k 55 dh lqbZ ds chp fdrus dk dks.k curk gS \
jsfM;u ds dks.k ?kw.kZu djus esa fdruk le; (lsdaMksa esa) ysrk
gS\ 1 1
(1) 27 (2) 45° (3) 22 (4) 30°
(1) 1.5 (2) 2.5 (3) 3.5 (4) 4.5
2 2
(SSC CGL Tier-I Exam, 2012) (SSC CGL Tier-I Exam, 2012)

BLAM–145
f=kdks.kfefr
11. What is the angle in radian through which a pendu-
Type-II
lum swings and its length is 75 cm and the tip de-
scribes an arc of length 21 cm. Angles ij based questions
,d yksyd (Pendulum) dh lwbZ ftldh yackbZ 75 lseh- gS]
1. What is the value of sin 240°\
21 lseh dk pki cukus ij fdruk dks.k cuk,xh\
sin 240° dk eku D;k gksxk\
FG 3 IJ R
FG 7 IJ R
3 3 1 1
(1)
H 25 K (2)
H 25 K (1)
2
(2)
2
(3)
2
(4)
2

FG 4 IJ R
FG 2 IJ R FG 7 IJ ?
(3)
H 21K (4)
H 15 K
2. What is the value of cos H 4K
FG 7 IJ dk eku D;k gksxk\
12. What will be the radius of circle in which a central
angle of 60° intercepts an arc of length 37.4 cm.
cos H 4K
fdlh o`Ùk ds f=kT;k dh yackbZ fdruh gksxh ;fn f=kT;k rFkk pki 1
(1) 0 (2) 1 (3) 2 (4)
ftldh yackbZ 37.4 lseh gS ds chp dk dks.k 60° gS\ 2
(1) 35 cm (2) 34.7 cm
FG 5 IJ sin FG 7 IJ ?
(3) 35.7 cm (4) 40 cm 3. What is the value of tan
H6K H6K
13. If the sum of two angles is 135° and their difference
FG 5 IJ FG 7 IJ dk eku D;k gksxk\
is
12
, then the circular measure of the greater angle is
tan
H6K sin
H6K
1 2
(1) (2)
;fn nks dks.kksa dk ;ksxiQy 135° gS vkSj mudk varj gS] rks 3 3
12
cM+s dks.k dk o`Ùkh; eki gSA 1 1
(3) (4)
2 3 2
2 3
(1) (2) 4. What is the value of cosec (–1410°) ?
3 5
cosec (–1410°) dk eku D;k gksxk\
5
(3) (4) 1
12 3 (1) (2) 2
3
(SSC CGL Tier-I Re-Exam. (2013) 20.07.2014 (Ist Sitting)
14. A train is moving on a circular of radius 1500 m at a 3 2
(3) (4)
speed of 90 km/h. Through what angle has it turned 2 3
in 11 sec?
5. What is the value of tan 330° ?
,d Vªus 90 km/h dh jÝrkj ls 1500 m dh f=kT;k osQ o`Ùkh; tan 330° dk eku D;k gksxk\
iFk ij py jgh gSA 11 sec esa mldk dks.k fdruk cny x;k 1
gS\ (1)
3
(2) 0
1 1 1
(1) 12° (2) 16°30 (3) 10°30 (4) 11°40
(SSC CGl 2012) 1
(3) 1 (4) –
15. In ABC, AB = AC. BA is produced upto D such that 3
AC = AD. The circular measure of ‘BCD is—
6. If sin T + cos T = 2 cos (90° – T), then cot T is equal to :
,d f=kHkqt ABC esa AB = AC | BA dks D rd bl izdkj c<+k;k
x;k gS fd AC = ADA ‘BCD dk o`Ùkh; eki gSA ;fn sin T + cos T = 2 cos (90° – T) gkss] rks cot T fdruk
gksxk\
S S 2S S
(1) (2) (3) (4) (1) 2 (2) 0 (3) 2 –1 (4) 2+1
6 3 3 2
(SSC CHSL (10+2) LDC, DEO & PA/SA
(SSC CGL Tier-I Exam, 2012) Exam, 01.11.2015, IInd Sitting)

BLAM–146
f=kdks.kfefr
7. What is the value of (1 + sec 20° + cot 70°) (1 – cosec 14. If sec (7T + 28°) = cosec (30° – 3T), then the value of
20° + tan 70°) ? T is
(1 + sec 20° + cot 70°) (1 – cosec 20° + tan 70°) dk eku ;fn sec (7T + 28°) = cosec (30° – 3T) gks] rks T dk eku
fdruk gksxk\ fdruk gksxk\
(1) 0 (2) –1 (3) 2 (4) 1 (1) 8° (2) 5° (3) 60° (4) 9°
(SSC CGL Tier-I Exam, 2012) (SSC Delhi Police S.I. (SI) Exam. 19.08.2012)
8. sin (45° + T) – cos (45° – T) equal to –
sin (45° + T) – cos (45° – T) fdlds cjkcj gS\ 15. If tan
FG  T IJ 3 , the value of cosT is :
(1) 2 sin T (2) 1 (3) 2 cos T (4) 0
H 2 2K
(SSC CGL Tier-I Exam, 2012)
;fn F TI
tan G  J 3 gks] rks cosT dk eku D;k gksxk\
9. The value of
sin 39q
+ 2 tan 11° tan 31° tan 45° tan
H 2 2K
cos 51q
59° tan 79° – 3 (sin 21° + sin2 69°) is :
2 1 1
(1) 0 (2) (3) (4) 1
2 2
sin 39
+ 2tan 11° tan 31° tan45° tan59° tan79° – 3 (SSC CHSL DEO & LDC Exam. 04.11.2012, Ist Sitting)
cos 51
(sin221° + sin269°) dk eku fdruk gksxk\ 2 sin 68 2 cot 15
16. –
cos 22 5 tan 75
(1) 2 (2) –1 (3) 1 (4) 0
(SSC CHSL DEO & LDC Exam. 11.12.2011 3 tan 45 . tan 20 . tan 40 . tan 50 . tan 70
(IInd Sitting (East Zone)
is equal to
5
10. If x, y are positive acute angles x + y < 90° and sin (2x
– 20°) = cos (2y + 20°), then sec (x + y) equals to— 2 sin 68 2 cot 15
;fn x, y /ukRed U;wu dks.k gksa] x + y < 90° gks] vkSj sin (2x cos 22 5 tan 75

– 20°) = cos (2y + 20°) gks] rks sec (x + y) dk eku fdruk 3 tan 45 . tan 20 . tan 40 . tan 50 . tan 70

gksxk\ 5

1 fdlds cjkcj gS\


(1) 2 (2) (3) 1 (4) 0 (1) –1 (2) 0 (3) 1 (4) 2
2
(SSC CHSL DEO & LDC Exam. 27.10.2013 IInd Sitting)
(SSC CGL Tier-I Exam, 2012) 17. The value of tan 10° tan 15° tan 75° tan 80° is
11. The value of cot 10° . cot 20° . cot 60° . cot 70° . cot
80° is
tan 10° tan 15° tan 75° tan 80° dk eku fdruk gksxk\
(1) 0 (2) 1 (3) –1 (4) 2
cot10°.cot20°.cot60° cot70°.cot 80° dk eku gSµ (SSC CHSL DEO & LDC Exam. 10.11.2013, Ist Sitting)

1 18. sin2 5° + sin2 25° + sin2 45° + sin2 65° + sin2 85° equal
(1) 1 (2) –1 (3) 3 (4) to —
3
sin2 5° + sin2 25° + sin2 45° + sin2 65° + sin2 85° cjkcj
(SSC CHSL DEO & LDC Exam. 04.12.2011
(Ist Sitting (North Zone) gSµ
12. If A = tan 11° tan 29°, B = 2 cot 61° cot 79°, then : (1) 1-5 (2) 2 (3) 2-5 (4) 3
;fn A = tan 11° tan 29°, B=2 cot 61° cot79° gks] rks fuEu (SSC CGL Tier-I Exam, 2012 and SSC CPO SI &
Assistant Intelligence Officer Exam, 2012)
esa dkSu lgh gS\
19. The numerical value of cot18°
(1) A = 2B (2) A = – 2B (3) 2A = B (4) 2A = – B
(SSC CHSL DEO & LDC Exam. 11.12.2011 FG cot 72 cos2 22 1 IJ is
(IInd Sitting (East Zone) H tan 72 sec 2 68 K

13. If tan 15° = 2 – 3 , the value of tan 15° cot 75° + F


cot18° G cot 72 cos 22
2 1 IJ dk la[;kRed
tan 75° cot 15°is H tan 72 sec 68 K 2

;fn tan 15° = 2 – 3 gks] rks tan15° cot 75° + tan 75° eku fdruk gS\
cot 15° dk eku D;k gksxk \ 1
(1) 1 (2) 2 (3) 3 (4)
(1) 14 (2) 12 (3) 10 (4) 8 3
(SSC CHSL DEO & LDC Exam. 04.12.2011 (SSC CHSL DEO & LDC Exam. 04.12.2011
(IInd Sitting (East Zone) (Ist Sitting (East Zone)

BLAM–147
f=kdks.kfefr
sin 25 cos 65 cos 25 sin 65 sin b90 –10 g – cosb –6 g
20. The value of 24. What is the value of =?
tan 2 70 – cos ec 2 20 LMcosFG – 10 IJ – sinb – 6 gP
O
fuEufyf[kr dk eku crk,¡% N H2 K Q
(1) tan2T (2) cot2T
sin 25 cos 65 cos 25 sin 65
(3) cotT (4) cot3T
tan 2 70 – cos ec 2 20
25. What is the value of {sin (90° – x) cos [ – (x – y)]} + {cos
(1) – 1 (2) 0 (3) 1 (4) 2
(90° – x) sin [ – (y – x)]} ?
(SSC CHSL DEO & LDC Exam. 9.11.2014)
(1) – cos y (2) – sin y
FG 4 IJ cot FG IJ + 3 cos (150°) (3) cos x (4) tan y
21. What is the value of H 3K H 6K 2 2
26. What is the value of cos 15° – cos 165°?

F I cos 15° – cos 165° dk eku D;k gS\


– 4 cosec 45° + 8 sin GH JK ?
2
2 2
3
(1) (2)
3 –1
FG 4 IJ cot FG IJ + 3 cos (150°) – 4 cosec 45° + 8 sin 2
H 3K H 6K 2 2 2

3 1 ( 3 1)
(3) (4)
FG IJ dk eku D;k gS\ 2 2
H 2K (SSC CGL Tier-II (CBE) Exam. 09.03.2018)
27. If P + Q + R = 60°, then what is the value of cos Q cos
25 R (cos P – sin P) + sin Q sin R (sin P – cos P)?
(1) (2) 1
4
;fn P + Q + R = 60° gS] rks cos Q cos R (cos P – sin P)
7 13 + sin Q sin R (sin P – cos P) dk eku D;k gS\
(3) – (4)
2 2
1 3
(1) (2)
R| 4cos b90q–Ag sin b90q Ag – U|
3 2 2
S| 4sin b90q A g cos b90q –A g V|
3
T W 1
22. What is the value of F 180q8A IJ
cos G
?
(3) (4) 2
H 2 K 2
(SSC CGL Tier-II (CBE) Exam. 09.03.2018)

|RS 4cos b90q–Ag sin b90q Ag – |UV


3 28. What is the value of

|T 4sin b90q A g cos b90q –A g |W


3
sin (180 – ) sin (90 – ) – cot 90 – b g
F 180q8A IJ dk eku D;k gS\ 1 tan 2
?
cos G
H 2 K
sin(180 – )sin(90 – ) – [cot(90 – )
(1) 1 (2) –1 dk eku D;k gS\
1 tan 2
(3) 0 (4) 2
cot
LM b180q – g OP LM b180q –9 g OP (1) cos2 sin (2)
23. What is the value of cos (1 + cot 2 )2
N 2 Q cos N 2 Q
tan
L b180q –3 g OP LM b180q –13 g OP (3)
(1 + tan 2 )2
(4) – sin3 cos
+ sin M
N 2 Q sin N 2 Q ? (SSC CGL Tier-II (CBE) Exam. 09.03.2018)
29. What is the value of sin (630° + A) + cos A?
L b180q – g OP LM b180q –9 g OP L b180q –3 g OP
cos M cos + sin M sin (630° + A) + cos A dk eku D;k gS\
N 2 Q N 2 Q N 2 Q
3 1
L b180q –13 g OP (1) (2)
2
sin M
2
N 2 Q dk eku D;k gS\ 2
(1) sin 2T sin 4T (2) cos 2T cos 6T (3) 0 (4)
3
(3) sin 2T sin 6T (4) cos 2T cos 4T
(SSC CGL Tier-II (CBE) Exam. 18.02.2018)

BLAM–148
f=kdks.kfefr
30. If sin sec (30° + ) = 1, (0 < < 60°), then the value 1 1 1 1
of sin + cos 2 is (1) (2) (3) (4)
4 3 2 5
;fn sin sec (30° + ) = 1, (0 < < 60°), gS] rks sin + (LDC – 2015)
cos 2 dk eku D;k gksxk\ 37. Value of the expression

2 3
fuEufyf[kr O;atd dk eku D;k gS\
(1) 1 (2) (3) 0 (4) 2 1 2 sin 60 .cos 60 1 2 sin 60 .cos 60
2 3
sin 60 cos 60 sin 60 cos 60
31. The Value of cot . tan (90° – ) – sec (90° – ) cosec
+ (sin 25 + sin2 65°) +
2 (1) 3 (2) 2 3 (3) 0 (4) 2
3 (tan 5°.tan15°. tan
(LDC –2015)
30°.tan75°, tan85°) dk eku Kkr djsaA
(1) 1 (2) –1 (3) 2 (4) 0 38. If sin (60° – ) = cos b g
30 , then the value of tan

32. The value of the following is


(SSC Assistant Grade. III – 2012 )
b g
30 is (assume that and are both positive
acute angle with < 60° and > 30°)
fuEufyf[kr dk eku gSA
FG sin 47 IJ FG cos 43 IJ2 2 ;fn sin (60° – ) = cos b g
30 , gS rks b dk eku g
4 cos2 45 D;k gksxk (eku ys fd rFkk nksuksa /ukRed U;wudks.k gS] ftuesa
H cos 43 K H sin 47 K
60 vkSj > 30°) gSA
1
(1) 1 (2) (3) –1 (4) 0 1
2 (1) (2) 0 (3) (4) 1
3 3
(LDC – 2015)
(SSC CGL – 2013)
33. The value of the expression
sin2 1° + sin2 11° + sin2 21° + sin2 31° + sin2 41° + sin2 tan 250 tan 340
45° + sin2 49 + sin2 59° + sin2 69° + sin2 79° + sin2 39. If tan 20° = K then =?
tan 200 tan 110
89° is dk eku Kkr djsaA
tan 250 tan 340
1 1 ;fn tan 20° = K gS rks tan 200 tan 110 dk eku Kkr
(1) 0 (2) 5 (3) 4 (4) 5
2 2 djs\
(LDC – 2015)
1 K 1 K 1 K2 1 K2
34. If + = 90°, then the value of d1 sin 2 h (1 – cos 2 (1)
1 K
(2)
1 K
(3)
1 K 2 (4)
1 K2
) × (1 + cot2 ) (1 + tan2 ) is 40. The Value of cos 24° + cos 25° + cos 155° + cos 204°
;fn + = 90°, d
gS] rks 1 sin 2 h (1 – cos 2
) × (1 +
is
cos 24° + cos 25° + cos 155° + cos 204° dk eku crk,\
2
cot ) (1 + tan 2
) dk eku gS\ (1) 1 (2) 1
(1) 1 (2) –1 (3) 0 (4) 2 (3) 0 (4) None of these
(SSC CHSL – 2013) 41. For an acute angle , sin + cos takes the greatest
35. The value of value when is.
,d U;wudks.k osQ fy,
sin + cos dk lcls cM+k eku ys\
5 tan (1) 30 (2) 45 (3) 60 (4) 90
1 4
sin .cos cos .sec 42. The value of sin 780° sin 480° + cos 240°. cos 300° =
2 6 4 3 6 12 sin
2 sin 780°. sin 480° + cos 240°. cos 300° dk eku Kkr
djsa\
is equal to foQlosQ cjkcj gS\
1 1
3 (1) (2)
(1) 0 (2) 1 (3) 2 (4) 2 4
2
(3) 1 (4) None of these
(SSC CGL Tier I–2014)
tan 160 tan 110
x x tan 2 30 43. If cot 20° = P then =
1 tan 160 . tan 110
36. If = sin2 30° + 4 cot2 45– sec2 60° The
1 tan 2 30
tan 160 tan 110
value of x is ;fn cot 20° = P gS] rks 1 tan 160 . tan 110 dk eku
x dk eku Kkr djsa\
crkb,\

BLAM–149
f=kdks.kfefr
49. What is the value of
P2 1 P2 1
(1) (2)
2P 2P b
4 cos 90 g
A sin 3 90 b A g b4 sin 90 g
A cos3 90 b A g
1 P 2
2P L
cos M
80 8 A O
(3)
2P
(4)
1 P2 N 2 PQ
44. cos2 5° + cos2 10° + cos2 15° +.......+ cos2 90° = dk eku D;k gS\
dk eku Kkr djsa\ (1) 1 (2) –1
(3) 0 (4) 2
1 1
(1) 8 (2) 6 [SSC CGL – 2018]
2 2
50. What is the value of
1
(3) 7
2
(4) None of these
cos
LM180 OP cosLM180 9 OP +
45. The Value of sin2 75° – sin2 15° is
N 2 Q N 2 Q
sin2 75° – sin2 15° dk eku Kkr djasµ L180 3 OP sin LM180 13 OP
sin M
3 3
N 2 Q N 2 Q
(1) (2) dk eku D;k gS\
2 2
(1) sin 2 sin 4 (2) cos 2 cos 6
1 (3) sin 2 sin 6 (4) cos 2 sin 4
(3) (4) None of these
2 [SSC CGL – 2018]
46. The Value of tan 100° + tan 125° + tan 100°. tan 125 51. 2 cosec2 23° cot2 67° – sin2 23° – sin2 67° – cot2 67° is
=? equal to
tan 100° + tan 125° + tan 100°. tan 25° dk eku Kkr 2 cosec2 23° cot2 67° – sin2 23° – sin2 67° – cot2 67°
djsaµ fdlds cjkcj gS\
(1) (2) 1 (1) 1 (2) sec2 23°
3
2
(3) tan 23° (4) 0
1 (SSC CHSL DEO & LDC Exam. 20.10.2013)
(3) (4) 1
3 3
52. The value of sin2 30° cos245° + 5tan230° + sin290°
4 FG IJ
cot 2
2
47. What is the value of
3 H K
6
+ 3 cos2 (150°) – 4 – 3 cos2 90° is

3
F I
cosec 45° + 8 sin G J = ?
sin2 30° cos2 45° + 5 tan2 30° +
2
sin2 90° – 3 cos2
2
H 2K
90° dk eku D;k gS \
4 F I
2 F I
cot G J + 3 cos (150°) – 4 cosec 45° + 8 sin G J 7 3 1 5
3 H 6K 2 2
H 2K (1) 3
24
(2) 3
24
(3) 3
24
(4) 3
24
dk eku D;k gS\ (SSC CGL Tier-I Exam. 19.10.2014 (Ist Sitting)

25
(1) (2) 1 5 tan
4 1 4
53. The value of sin cos cot sec is
2 6 4 3 6 12 sin
7 13 2
(3) (4)
2 2 equal to
[SSC CGL – 2018]
48. What is the value of sin (B – C) cos (A – D) + sin (A – 1 5 tan
sin cos – cot sec + 4 dk eku
B) cos (C – D) + sin (C – A) cos (B – D) ?
2 6 4 3 6
sin (B – C) cos (A – D) + sin (A – B) cos (C – D) + sin 12 sin
2
(C – A) cos (B – D) dk eku D;k gksxk\
fdlds cjkcj gS\
3
(1) (2) –3 3
2 (1) 0 (2) 1 (3) 2 (4)
(3) 1 (4) 0 2
(SSC CGL Tier-I Exam. 19.10.2014 (Ist Sitting)
[SSC CGL – 2018]

BLAM–150
f=kdks.kfefr
3 4 (1) 3 (2) 2
54. If x sin2 60° – sec 60° tan2 30° + sin2 45° tan2 (3) 1 (4) 4
2 5
(SSC CHSL (10+2) Tier-I Exam. 17.03.2020)
60° = 0 then x is
59. If sin 3x = cos (3x – 45°), 0° < 3x < 90°, then x is
3 4 equal to :
;fn x sin2 60° – 2 sec 60° tan2 30° + 5 sin2 45° tan2
;fn sin3x = cos (3x – 45°), 0° < 3x < 90° gS rks x dk eku
60° = 0 rks x dk eku D;k gksxk \ Kkr djsA
1 4 (1) 35° (2) 45°
(1) – (2) – 4 (3) – (4) – 2 (3) 22.5° (4) 27.5°
15 15
(SSC Delhi Police SI, CAPFs SI & CISF ASI
(SSC CGL Tier-I Exam. 26.10.2014) (CPO) Exam 24.11.2020)
55. The value of the following cos 24° + cos 55° + cos
125° + cos 300° is
Type-III
fuEufyf[kr dk eku gS cos 24° + cos 55° + cos 125° + cos
204° + cos 300° Trigonometric identities ij
1 based questions
1
(1) (2) 2 (3) 1 (4) –
2 2
(SSC CGL Tier-I Exam, 16.08.2015, 2nd Sitting) 1. If p sin T = 3 and p cos T = 1, then what will be the
56. The numerical value of value of p ?
;fn p sin T = 3 rFkk p cos T = 1 gks] rks p dk eku fdruk
cos2 45 cos2 60 tan 2 30 sin 2 30
– – is gksxk \
sin 2 60 sin 2 45 cot 2 45 cot 2 30
1 2 1
cos2 45q cos2 60q tan 2 30q sin 2 30q (1) (2) (3) (4) 2
+ – – 2 3 3
sin 60q
2
sin 2
45q cot 2
45q cot 2
30q
(SSC CGL Tier-I Exam, 2012)
dk la[;kRed eku gS 2. If sin (x + y) = cos [3 (x + y)], then what will be the
1 3 1 1 value of tan [2 (x + y)] ?
(1) 1 (2) (3) (4)
4 4 4 2 ;fn sin(x + y) = cos [3(x + y)], gks] rks tan[2(x + y)] dk eku
(SSC CGL Tier-I Exam, 09.08.2015 D;k gksxk\
(Ist Sitting) TF No. 1443088)

1 1
57. If cosec 39° = x, then the value of 2 (1) 3 (2) 1 (3) 0 (4)
2 o + sin 3
cosec 51
(SSC CGL Tier-I Exam, 2012)
1
39° + tan2 51° – is :
sin 2 51o sec 2 39o 3. If 0 < < and 2sin + 15 cos2 = 7, then value of
2
1 cot will be—
;fn If cosec 39° = x, gS] rks + sin2 39° +
cosec2 51o
1 ;fn 0< < rFkk 2sin + 15 cos2 =7 gks] rks cot
tan2 51° – dk eku Kkr dhft,A 2
2 o 2 o
sin 51 sec 39
dk eku fdruk gksxk\
1 5 3 1
(1) (2) (3) (4)
(1) x2 – 1 (2) x2 1 2 4 4 4
(SSC CGL Tier-I Exam, 2012)
(3) 1 x2 (4) 1 – x2
(SSC CGL Tier-II Exam. 18.11.2020)
4. If sin 2
LM 2 3 4
.....
x 1OP
 = 1 (O < x < 100)
1 tan A o o o o o
N1 2 3 x 2 Q
58. If = tan 3 tan 15 tan 30 tan 75 tan 87 , then value of x is equal to
1 tan A tan 27o tan 39o tan 51o tan 60o tan 63o
then the value of cot A is :
;fn sin 2
LM 2 3 4
.....
x 1 OP
= 1 (O < x < 100)
;fn
1 tan A
1 tan A
=
tan 3o tan 15o tan 30o tan 75o tan 87o
tan 27o tan 39o tan 51o tan 60o tan 63o
N1 2 3 x 2 Q
dk eku D;k gksxk\
gS] rks cot A dk eku Kkr dhft,A
(1) 91 (2) 89 (3) 49 (4) 46

BLAM–151
f=kdks.kfefr
5. If tan (x + y). tan (x – y) = 1 then, what will be the
(1) 2 (2) 5 (3) 3 (4) 6
F 2x IJ ?
value of tan G
(SSC CPO SI & Assistant Intelligence Officer Exam, 2012)
H3K 11. If tan2T . tan 4T = 1, then the value of tan 3T is
;fn tan2T . tan 4T = 1, rks tan 3T dk eku D;k gksxk \
;fn gks] rks tan
FG 2x IJ dk eku
tan (x + y) tan (x – y) = 1,
H3K (1) (2) 0 (3) 1 (4)
1
3 3
D;k gksxk\
(SSC CHSL DEO & LDC Exam. 11.12.2011
1 2 (Ist Sitting (East Zone)
(1) (2) (3) 3 (4) 1
3 3 12. If sin sec (30° + ) = 1 (0 < < 60°), then the value
(SSC CGL Tier-I Exam, 2012) of sin + cos 2 is
;fn sin D sec (30° + D) = 1 (0 <D< 60°), rks sin + cos
3
6. If sin(A + B) = 1 and cos(A–B) =
2
, where A and B 2D dk eku gSµ
are positive acute angles and A > B then A and B are :
2 3
(1) 1 (2) (3) 0 (4) 2
3 2 3
;fn sin(A + B) = 1 vkSj cos(A–B) = , tgk¡ A rFkk B
2 (SSC CHSL DEO & LDC Exam. 04.12.2011
èkukRed U;wu dks.k gSa vkSj A B, rks A rFkk B gSa % (Ist Sitting (North Zone)

(1) A = 75°, B = 15° (2) A = 60°, B = 30° 7


13. If sin T – cos T = and 0 < T < 90°, then the value
(3) A = 45°, B = 45° (4) None of these 13
[SSC CGL Tier-I Exam, 2012 & SSC CPO SI & of sin T + cos T is
Assistant Intelligence Officer Exam, 2012]
7
7. If secT + tanT = 3 , then positive value of sin T is— ;fn sin T – cos T = 13 vkSj 0° < T < 90° gks] rks sin T +

;fn secT + tanT = 3 rks sin T dk èkukRed eku gSµ cos T dk eku fdruk gksxk\

17 13 1 1
1 3 (1) (2) (3) (4)
(1) 0 (2) (3) (4) 1 13 17 13 17
2 2
(SSC CHSL DEO & LDC Exam. 04.12.2011
(SSC CGL Tier-I Exam, 2012) (IInd Sitting (East Zone)
14. If tan (2T + 45°) = cot 3T where (2T + 45°) and 3T are
b a b a b acute angles, then the value of T is
8. If sin = , then is equal to
a a b a b
;fn tan (2 + 45°) = cot 3 ,tgk¡ (2 + 45°) vkSj
b a b a b 3 U;wu dks.k gSa] rks dk eku gS
;fn sin =
a
, gS] rks ;g fdlosQ (1) 5° (2) 9° (3) 12° (4) 15°
a b a b
FCI Assistant Grade-III Exam. 25.02.2012 (Paper-I)
cjkcj gksxk\ North Zone (Ist Sitting)
2 2
(1) 0 (2) 1 (3) (4) 1 sin x 1 sin x cos x
cos sin 15. P = , q ,r which
1 sin x cos x 1 sin x
2
9. If tan b1 2 g 3 and sec b 1 – 2g then
one of the following is correct?
3 tks fuEufyf[kr esa ls ,d lgh gS\
sin 2 1 tan 3 2 equal to— (1) P = q r (2) q = r P
(3) r = p q (4) P = q r
2
;fn tan b1 2 g 3 vkS j sec b 1 – 2g 3
, rks 16. What is the value of

sin 2 1 tan 3 2 dk eku gSµ


b
2 1 sin g cos2 b
2 1 sin g cos2
cos 4
( eku ysa fd 0 < T1 – T2 < T1 + T2 < 90°)
(1) 0 (2) 3 (3) 1 (4) 2
dk eku Kkr djsa\
(SSC CPO SI & Assistant Intelligence Officer Exam, 2012) (1) –1 (2) 0 (3) 1 (4) 4
10. If 3sin2 + 7 cos2 = 4 then tan equal to (where 17. If A, B and C be the angles of a triangle, then which
0< <90°)— of the following, the incorrect relation is :
;fn 3sin2 + 7 cos2 = 4, rks tan dk eku gS (tgk¡ ;fn A, B rFkk C, ,d f=kHkqt ds dks.k gksa] rks fuEu esa dkSu&lk
0< <90°)µ laca/ xyr gS\
BLAM–152
f=kdks.kfefr

(1) sin
A B
cos
C
(2) cos
FG A B IJ sin
C
(1) 0 (2)
1
(3) 1 (4) 2
2 2 H 2 K 2 2
(SSC CHSL DEO & LDC Exam. 04.12.2011

FG A B IJ C F A B IJ
(4) cot G tan
C (Ist Sitting (East Zone)
25. If T be a positive acute angle satisfying cos2 T + cos4 T
(3) tan
H 2 K sec
2 H 2 K 2
= 1, then the value of tan2 T + tan4 T is
FCI Assistant Grade-III. Exam. 05.02.2012 (Paper-I)
East Zone (IInd Sitting)
;fn T ,d /ukRed U;wu dks.k gks vkSj cos2 T + cos4 T = 1 gks]
18. If tan T . tan 2T = 1, then the value of sin2 2T + tan2 rks tan2 T + tan4 T dk eku D;k gksxk \
2T is— 3 1
;fn tan T . tan 2T = 1, rks sin2 2T + tan2 2T dk eku gSA (1)
2
(2) 1 (3)
2
(4) 0

10 3 (SSC CHSL DEO & LDC Exam. 04.12.2011


(1) 3
4 (2) (3) 3 (4) 3 (IInd Sitting (North Zone)
3 4
26. If 5 tan T = 4, then what will be the value of
(SSC Graduate Level Tier-II Exam. 04.09.2011)

S 3S 5S 7S 9S
FG 5 sin T  3 cos T IJ
19. cot
20
cot
20
cot
20
cot
20
cot
20
equal to— H 5 sin T  3 cos T K ?

S 3S 5S 7S 9S ;fn 5 tanT = 4 gks] rks


FG 5 sin T  3 cos T IJ dk eku D;k
cot
20
cot
20
cot
20
cot
20
cot
20
dk eku gSµ H 5 sin T  3 cos T K
1
gks xk \
(1) –1 (2) 2 (3) 0 (4) 1
1 2 5 2
(SSC Graduate Level Tier-II Exam. 04.09.2011) (1) (2) (2) (4)
7 7 7 5
20. Evaluate : 3 cos 80° cosec 10° + 2 cos 59° cosec 31°
(SSC CGL Tier-I Exam, 2012)
3 cos 80° cosec 10° + 2 cos 59° cosec 31° dk eku Kkr
dhft,A sin cos 5 tan 2 1
27. If , the value of is
(1) 1 (2) 3 (3) 2 (4) 5 sin cos 4 tan 2 1
(SSC Graduate Level Tier-I Exam. 19.05.2013)
sin cos 5 tan 2 1
2 sin T cos T ;fn gks] rks dk eku fdruk gksxk\
21. If 1 , then value of cotT is : sin cos 4 tan 2 1
cos T sin T
25 41 41 40
2 sin T cos T (1) (2) (3) (4)
;fn cos T sin T
1 gks] rks cotT dk eku D;k gksxk\ 16 9 40 41
(SSC CHSL DEO & LDC Exam. 28.10.2012 (Ist Sitting)
1 1 sin T  cos T
(1) (2) (3) 3 (4) 2 28. If = 3, then the value of sin4T – cos4T is
2 3 sin T  cos T
(SSC CHSL DEO & LDC Exam. 04.11.2012, Ist Sitting)
sin T  cos T
tan T cot T ;fn =3 rks sin4T– cos4T dk eku gSµ
22.  is equal to sin T  cos T
1 – cot T 1 – tan T
1 2 3 4
(1) (2) (3) (4)
tan T cot T 5 5 5 5
 fdlds cjkcj gS\
1 – cot T 1 – tan T (SSC CHSL DEO & LDC Exam. 11.12.2011
(Ist Sitting (Delhi Zone)
(1) 1 – tan T – cot T (2) 1 + tan T – cotT
(3) 1 – tan T + cot T (4) 1 + tan T + cot T 8 sin 5 cos
29. If tan T = 1, then the value of
(SSC FCI Assistant Grade-III Main Exam. 07.04.2013) sin 3 2 cos3 7 cos
23. Simplest value of (1 + tanT + secT) (1 + cotT – is
cosecT) is —
8 sin 5 cos
(1 + tanT + secT) (1 + cotT – cosecT) dk ljyhÑr eku gSµ ;fn tan T = 1 gks] rks dk eku
sin 3 2 cos3 7 cos
(1) –2 (2) 2 (3) 1 (4) –1
(SSC CGL Tier-I Exam, 2012) fdruk gksxk \
2
24. The simplified value of (sec A – cos A) + (cosec A – 1 4
sin A)2 – (cot A – tan A)2 is (1) 2 (2) 2 (3) 3 (4)
2 5
(sec A – cos A)2 + (cosec A – sin A)2 – (cot A – tan A)2 dk (SSC CHSL DEO & LDC Exam. 04.12.2011
ljyhÑr eku fdruk gksxk \ (IInd Sitting (North Zone)

BLAM–153
f=kdks.kfefr
30. If sinT + cosecT = 2, then value of sin 100
T + cosec100T 38. If sec x + cos x = 3 then what is the value of tan2 x –
is equal to : sin2 x ?
;fn sinT + cosec T = 2, rks sin100T + cosec100T dk eku gS % ;fn sec x + cos x = 3 gks] rks tan2 x – sin2 x fdruk gksxk\
(1) 1 (2) 2 (3) 3 (4) 100 (1) 5 (2) 13 (3) 9 (4) 4
(SSC CHSL DEO & LDC Exam. 11.12.2011 (SSC CGL Tier-I Exam, 2012)
(IInd Sitting (Delhi Zone)
39. If sin T + sin2 T + sin3 T = 1 then cos6 T– 4 cos4 T +
31. If un = cosn + sinn , then what is the value of 2u6 –
8 cos2 T equals to—
3u4 +1 ?
;fn un = cosn + sinn gks] rks 2u6 – 3u4 +1 fdruk gksxk \ ;fn sin T + sin2 T + sin3 T = 1 gks] rks cos6 T– 4 cos4 T
(1) 1 (2) 4 (3) 6 (4) 0 + 8 cos2 T fdlds cjkcj gksxk\
(SSC CGL Tier-I Exam, 2012) (1) 2 (2) 1 (3) 4 (4) 3
32. If tan T + cot T = 5, then tan2T + cot2 T is (SSC CGL Tier-I Exam, 2012)

;fn tan T + cot T = 5 gS] rks tan2 T + cot2 T dk eku gS 40. If cos2 D + cos2E= 2, then the value of tan3 D+ sin5
(1) 23 (2) 25 (3) 26 (4) 24 E is :
(SSC CGL Tier-I Exam, 09.08.2015 ;fn cos2 + cos2 = 2 gks] rks tan3 + sin5 dk eku fdruk
(IInd Sitting) TF No. 4239378)
gksxk\
33. If tan T + cot T = 2, then the value of tan2T + cot2T is
;fn tan T + cot T = 2 gks] rks tan2T + cot2T fdruk gksxk \ 1
(1) –1 (2) 0 (3) 1 (4)
(1) 2 (2) 1 (3) (4) 0 3
2
(SSC Graduate Level Tier-I Exam. 19.05.2013) (SSC CHSL DEO & LDC Exam. 11.12.2011
(IInd Sitting (East Zone)
1 cos B 2 tan A
34. If tan A = , then what is equal 7
sin B 1 – tan 2 A 41. If sec 2 tan 2 , then sec4 – tan4 =
to? 12

1 cos B 2 tan A 7
;fn tan A = , gS rks 1 – tan 2 A fdlosQ cjkcj ;fn sec 2 T  tan 2 T , rks sec4 – tan4 =?
sin B 12
gksxk\
7 1 5
tan B (1) (2) (3) (4) 1
12 2 12
(1) (2) 2 tan B (3) tan B (4) 4 tan B
2 FCI Assistant Grade-III Exam. 25.02.2012 (Paper-I)
35. If cos A + cos2 A = 1, then what is the value of sin2 A North Zone (Ist Sitting)
+ sin4 A ? 42. If sec2T + tan2T = 7, then the value of T when 0° d Td
;fn cos A + cos2 A = 1 gks] rks sin2 A + sin4 A dk eku D;k 90°, is
gksxk\ ;fn sec2T + tan2T = 7, rks T dk eku] tc 0° < T < 90°, gS
1 (1) 60° (2) 30° (3) 0° (4) 90°
(1) 1 (2) (3) 0 (4) –1
2 (SSC CHSL DEO & LDC Exam. 11.12.2011
(SSC CGL Tier-I Exam, 2012) (Ist Sitting (Delhi Zone)
36. The numerical Value of 43. If 0° < A < 90°, then the value of tan2A + cot2 A – sec2 A
cosec2 A is
LM 1 1 OP LM 1 1 OP is ;fn 0° < A < 90° gS] rks tan2A + cot2A – sec2A cosec2A
N cos cot Q N cos cot Q dk eku gSµ
dk l[;kRed eku gSA (1) 0 (2) 1 (3) 2 (4) – 2
(1) 0 (2) –1 (3) +1 (4) 2
(SSC CHSL DEO & LDC Exam. 02.11.2014 (IInd Sitting)
(SSC CHSL DEO & LDC – 2012)

LM x OP 1 LM 1 OP 44. If sec tan 3 (0° < T < 90°), then tan 3T is


37. If 2 sin = x2 + , then the value of x
N2Q x2 N x Q ;fn sec tan 3 (0° < T < 90°), rks tan 3T
is:
1
L xO
;fn 2 sin M 2 P
1
=x + 22
gS] rks
LMx 1 OP dk eku Kkr (1) undefined (2)
3
N Q x N x Q
djsaA (3) infinity (4) 3
(1) –1 (2) 2 (3) 1 (4) 0 (SSC GL Tier-I Exam. 21.04.2013 IInd Sitting)
(SSC CGL Tier-I–2012 )

BLAM–154
f=kdks.kfefr
LM 1+ 2cot b90 – x g 2 OP 51. If (sin + cosec )2 + (cos + sec ) 2 = k + tan2 +
cot2 , then the value of k is equal to :
45. What is the value of
MN – 2 cosec (90 – x ) cot (90 – x )PQ ? ;fn (sin + cosec )2 + (cos + sec )2 = k + tan2 +
[cosec (90 – x ) – cot (90 – x )]
cot2 , gS] rks k dk eku fdruk gksxk\
LM 1+ 2cot b90 – x g 2 OP (1) 7 (2) 2 (3) 5 (4) 9
MN – 2 cosec (90 – x ) cot (90 – x ) PQ dk eku D;k gS\
(SSC CGL Tier-II Exam. 18.11.2020)

[cosec (90 – x ) – cot (90 – x )] sec tan 51


52. If 2 , then the value of sin is
(1) cosx + sinx (2) sinx – cosx sec tan 79
(3) secx + tanx (4) secx – tanx equal to :
(SSC CGL Tier-II (CBE) Exam. 09.03.2018)

( x – 2) ;fn sec tan


2
51
gS] rks sin dk eku Kkr dhft,A
46. If tan + sec = , then what is the value of sec tan 79
( x 2)
cos ? 35 91 65
(1) (2) 39 (3) (4)
72 72 144 144
( x – 2)
;fn tan + sec =
( x 2)
gS] rks cos dk eku D;k gS \ (SSC CGL Tier-II Exam. 18.11.2020)
53. If sec + tan = 3, then the value of sec is :
( x 2 – 1) (2 x 2 – 4) ;fn sec + tan = 3, gS] rks sec dk eku Kkr dhft,A
(1) (2)
(x 2 1) (2 x 2 4)
4 3 3 5
(1) (2) (3) (4)
( x – 4) 2 2
( x – 2) 3 4 5 3
(3) (4)
(x 2 4) (x 2 2) (SSC CGL Tier-II Exam. 18.11.2020)

(SSC CGL Tier-II (CBE) Exam. 21.02.2018) 54. If 3 sin x + 4 cos x = 2, then the value of 3 cos x – 4
47. If x = a sec . cos , y = b sec sin z = c tan , sin x is equal to :

x2
y 2
z 2 ;fn 3 sin x + 4 cos x = 2 gS] rks 3 cos x – 4 sin x dk eku
then the value of is Kkr dhft,A
a2 b2 c2
;fn x = a sec cos , y b. sec sin , z = c tan gS] (1) 23 (2) 21 (3) 29 (4) 21
2 2 2
x y z (SSC CGL Tier-II Exam. 18.11.2020)
rks dk eku D;k gksxk\ 55. If sin + sin2 = 1, then the value of cos2 + cos4 is
a2 b2 c2
(1) 1 (2) 4 (3) 9 (4) 0 equal to:
(SSC CGL Tier-I 2013) ;fn sin + sin2 = 1 gS] rks cos2 + cos4 dk eku Kkr
48.If x cos – sin = 1 then x2 – (1 + x2) sin equals:
dhft,A
;fn x cos – sin = 1 gS] rks x2 – (1 + x2) sin cjkcj gS\
(1) 1 (2) –1 (3) 0 (4) 2 1
(1) 5 (2) (3) 1 (4) 0
(SSC CGL Tier-I 2015) 2
4 4 6
49. If 6 sin + 3 cos = 2 then the value of [7 cosec (SSC CGL Tier-II Exam. 18.11.2020)
+ 8 sec6 ]1/3 is
56. If sec2 x – 3 secx + 2 = 0, then the value of x (0< x
;fn 6 sin4 + 3 cos4 =2 gS] rks [7 cosec6 + 8 sec6 < 90°) is :
] 1/3
dk eku gS\ ;fn sec2 x – 3 secx + 2 = 0 gS] rks x (0< x < 90°) dk eku
(1) 2 (2) 4 (3) 8 (4) 6
(SSC CPO SI 2016)
D;k gksxk \
(1) 45° (2) 15° (3) 60° (4) 30°
cos cos
50. If = 4 then the value of (0 < < (SSC CHSL (10+2) Tier-I Exam. 17.03.2020)
1 sin 1 sin
57. If cosec + cot = 2, then the value of sin is :
90°) is:
;fn cosec + cot = 2 gS] rks sin dk eku Kkr djsaA
cos cos
;fn 1 sin 1 sin
=4 rks (0 < < 90°) dk eku
3 2 3 4
(1) (2) (3) (4)
Kkr djs\ 5 5 4 5
(1) 60° (2) 45° (3) 30° (4) 35° (SSC CHSL (10+2) Tier-I Exam. 17.03.2020)
(SSC CGL Tier-I 2016)

BLAM–155
f=kdks.kfefr
1 Type-IV
58. If sin – cos ,find the value of (sin + cos .
29 Trigonometric functions dh minimum,
1
maximum values, substitution, rFkk
;fn sin – cos gS] rks sin + cos dk eku Kkr series ij based questions
29

djsaA 1. The minimum value of 4 tan2T + 9 cot2T is equal to


41 42 22 2 4 tan2T + 9 cot2T dk U;wure eku fdlds cjkcj gksxk\
(1) (2) (3) (4) (1) 0 (2) 5 (3) 12 (4) 13
29 29 29 29
(SSC CHSL DEO & LDC Exam. 10.11.2013, Ist Sitting)
(SSC CHSL (10+2) Tier-I Exam. 18.03.2020)
2. The minimum value of sin2T + cos2T + sec2T + cosec2T
59. Find the value of sin (60 + ) –cos (30 – ). + tan2T + cot2T is
sin (60 + ) –cos (30 – ) dk eku Kkr djsaA sin2T + cos2T + sec2T + cosec2T + tan2T + cot2T dk
1 U;wure eku fdruk gS\
(1) (2) 0 (1) 1 (2) 3 (3) 5 (4) 7
2
(SSC Graduate Level Tier-II Exam. 16.09.2012)
(3) –1 (4) 1
3. The maximum value of sin4T + cos4T is
(SSC CHSL (10+2) Tier-I Exam. 17.03.2020)
sin4T + cos4T dk vf/dre eku fdruk gksxk\
2 cosec 2a 2 sec2 a
60. If tan a = then the value of 1
13 cosec 2a 3 sec 2 a (1)
3
(2) 1 (3) 2 (4) 3
is : (SSC CGL Tier-I Exam, 09.08.2015
(Ist Sitting) TF No. 1443088)
2 c osec 2a 2 sec 2 a 4. What will be the minimum value of 2 sin2 T + 3cos2 T ?
;fn tan a = gS] rks dk eku 2 sin2 T + 3cos2 T dk U;wure eku fdruk gksxk \
13 cosec 2a 3 sec 2 a
(1) 3 (2) 5 (3) 1 (4) 2
Kkr dhft,A (SSC CGL Tier-I Exam, 2012)
5. sin25° + sin210° + sin215° + .... + sin285° + sin290° is
(1) 21 (2) 14 (3) 32 (4) 16
equal to
(SSC CHSL (10+2) Tier-I Exam. 17.03.2020)
sin25° + sin210° + sin215° +....+ sin285° + sin290° fdlds
sin x cos x 6 tsn 2x 1 cjkcj gksxk \
61. If , then the value of
sin x cos x 5 tan 2 x 1 1 1 1
(1) 7 (2) 8 (3) 9 (4) 9
is : 2 2 2
(SSC CHSL DEO & LDC Exam. 11.12.2011 (Ist Sitting
sin x cos x 6 tan 2 x  1 (East Zone) & (SSC CHSL DEO & LDC
;fn ] rks dk eku Kkr djsaA Exam. 21.10.2012) (IInd Sitting)
sin x cos x 5 tan 2 x  1 6. (sin2 1° + sin2 3° + sin2 5° + ... + sin2 85° + sin2 87° +
sin2 89°) equal to
35 61 35 60 (sin2 1° + sin2 3° + sin2 5° + ... + sin2 85° + sin2 87° +
(1) (2) (3) (4)
61 60 61 61 sin2 89°) ds cjkcj gS %
(SSC CHSL (10+2) Tier-I Exam. 19.03.2020)
1 1 1
62. If 4 – 2 sin2 – 5 cos = 0,0° < <90°, then the value (1) 21 (2) 22 (3) 22 (4) 23
2 2 2
of cos + tan is:
(SSC CHSL DEO & LDC Exam. 04.12.2011)
;fn 4 – 2 sin2 – 5 cos = 0,0° < <90°, gS rks cos + 7. The value of 152 (sin 30° + 2 cos2 45° + 3 sin 30° + 4
cos2 45° + .... + 17 sin 30°+ 18 cos2 45°) is
tan dk eku Kkr djsaA
152 (sin 30° + 2 cos2 45° + 3 sin 30° + 4 cos2 45° +
.... + 17 sin 30°+ 18 cos2 45°) dk eku D;k gksxk\
2 3 2 3
(1) (2) (1) an integer but not a perfect square
2 2
iw.kk±d ijarq iw.kZ oxZ ugha
(2) a rational number but not an integer
1 2 3 1 2 3 ifjes; la[;k ijarq iw.kk±d ugha
(3) (4)
2 2
(3) a perfect square of an integer / iw.kk±d dk iw.kZ oxZ
(SSC Delhi Police SI, CAPFs SI & CISF ASI (4) irrational / vifjes;
(CPO) Exam 23.11.2020) (SSC Graduate Level Tier-I Exam. 21.04.2013)

BLAM–156
f=kdks.kfefr
8. The value of cos 1° cos 2° cos 3°.... cos 177° cos 178° 17. The value of the expression sin21° + sin211° + sin221°
cos 179° is : + sin231° + sin241° + sin245° + sin249° + sin259° +
cos 1° cos 2° cos 3°..... cos 177° cos 178° cos 179° sin269° + sin279° + sin289° is :
dk eku fdruk gS\ inlagfrsin21° + sin211° + sin221° + sin231° + sin241° +
1 sin245° + sin249° + sin259° + sin269° + sin279° + sin289°
1
(1) 0 (2)
2
(3) 1 (4)
2
dk eku D;k gksxk\
(SSC GL Tier-I Exam. 21.04.2013, Ist Sitting) 1 1
(1) 0 (2) 5 (3) 5 (4) 4
9. sin113 . cos113 Find the maximum value of? 2 2
sin113 . cos113 dk vf/dre eku Kkr dhft, (SSC CHSL (10+2) LDC, DEO & PA/SA Exam, 06.12.2015
(Ist Sitting) TF No. 1375232)

FG 3 IJ 113

(1)
H 2K (2) 1 18. If A = sin2 + cos4 where O Q
2
them which

one of the following is correct?


F 1I
(3) G J
113
F 1I
(4) G J
113
buesa ls dkSu&lk lgh gS\
H 4K H 2K
3
(SSC CHSL DEO & LDC Exam. 20.10.2013) (1) 1 A 2 (2) A 1
10. The simple value of tan 1°. tan 2°. tan 3°.... tan 89° is
4

tan 1°. tan 2°. tan 3° ..... tan 89° dk ljyre eku D;k gS \ 13 3 13
(3) A 2 (4) A
1 2 16 4 16
(1) (2) 0 (3) 1 (4) (SSC–2013)
2 3
(SSC CGL Tier-I Exam. 19.10.2014)
19. The value of x in the equation tan2 – cos2
11. 16 cosec2 + 25 sec2 find the minimum value of? 4 3
16 cosec2 + 25 sec2 dk U;wure eku gSA
(1) 81 (2) 41 (3) 82 (4) 90 = x sin cos tan is :
4 4 3
12. sin x + 3 cos x will be minimum when.
sin x + vf/dre gksxk tc lehdj.k tan2 – cos2 = x sin cos tan esa
3 cos x 4 3 4 4 3
(1) x = 30 (2) x = 0 (3) x = 45 (4) x = 60 x dk eku D;k gS ?
13. sin6 + cos6 . Find the minimum value of.
2 3 3 1 3
sin6 + cos6 dk U;wure eku D;k gS\ (1) (2) (3) (4)
3 4 3 2
1 1 (SSC CHSL (10+2) LDC, DEO & PA/SA Exam, 06.12.2015
(1) (2)
2 4 (IInd Sitting) TF No. 3441135)

(3) 1 (4) None of these FG x IJ = x + 1 , then the value of FG x 1 IJ is


14. The value of cot 41°. cot 42° . cot 43°. cot 44°. cot 45° .
cot 46° . cot 47° . cot 48° . cot 49°
20. If 2 sin H2K x
2
2 H x K
cot 41°. cot 42°. cot 43°. cot 44°. cot 45°.cot 46°.cot F xI
;fn 2 sin GH 2 JK = x +
1 F 1I
gks] rks GH x x JK
2
dk eku
47°.cot 48°.cot49° dk eku fdruk gksxk \ x 2

3 1 fdruk gksxk \
(1) 1 (2) 0 (3) (4) (1) –1 (2) 2 (3) 1 (4) 0
2 2
(SSC Graduate Level Tier-II Exam. 16.09.2012)
(SSC CGL Tier-II Exam, 25.10.2015, TF No. 1099685)
21. If x = cosec T – sinT and y = secT – cosT then x2y2
15. The value of sin21° + sin22° + sin23° + ....+ sin289° is (x2 + y2 + 3) equal to—
sin21° + sin22° + sin23° + .....+ sin289° dk eku D;k gS \ ;fn x = cosec T – sin T vkSj y = sec T – cos T, gks] rks x2y2
1 1 (x2 + y2 + 3)dk eku D;k gksxk\
(1) 22 (2) 44 (3) 22 (4) 44 (1) 0 (2) 1 (3) 2 (4) 3
2 2
(SSC CGL Tier-I Exam, 2012)
(SSC CGL Tier-I Exam. 19.10.2014 TF No. 022 MH 3)
22. If x sin3 T+y cos3 T = sin T cos T and x sin T = y cos T,
16. 81sin x .27cos x Find the maximum value of?
sin T 0, cos T 0 then (x2 + y2) equal to—
81sin x .27cos x dk vf/dre eku gS\ ;fn x sin3 T+y cos3 T = sin T cos T rFkk x sin T = y cos
(1) 35 (2) 34 (3) 3 (4) 33 T, sin T 0, cos T 0 gks] rks x2 + y2 fdruk gksxk\
BLAM–157
f=kdks.kfefr
28. If x = a sin T – b cos T, y = a cos T + b sin T, then
1 1
(1) (2) (3) 1 (4) 2 which of the following is true?
2 2
;fn x = a sin T – b cos T, y = a cos T + b sin T gS] rks
(SSC CGL Tier-I Exam, 2012)
fuEufyf[kr esa ls dkSu&lk lgh gS\
23. The eliminant of T from x cosT – y sin T = 2 and x sin
T + y cos T = 4 will give x2 a2
(1) + 2 =1 (2) x2 + y2 = a2 – b2
x cos T – y sin T = 2 rFkk x sin T + y cos T = 4 esa T ds y 2
b
fujkdj.kiQy ls D;k izkIr gksxk \
x 2 y2
(3) + =1 (4) x2 + y2 = a2 + b2
(1) x  y 2 2
20 (2) 3x  y2 2
20 a 2 b2
(SSC CGL Tier-II Exam, 25.10.2015, TF No. 1099685)
(3) x 2 – y 2 20 (4) 3x 2 – y 2 10 29. If sin T + cos T = p and sec T + cosec T = q, then the
(SSC Graduate Level Tier-I Exam. 19.05.2013) value of q (p2 – 1) is
;fn sin T + cos T = pvkSj sec T + cosec T = q, rks q (p2
24. If x cos T – y sin T = x 2  y 2 and
– 1) dk eku D;k gksxk \
cos 2 T sin 2 T 1 (1) 1 (2) p (3) 2p (4) 2
 , then the correct relation is (SSC CHSL (10+2) DEO & LDC Exam. 16.11.2014,
a2 b2 x  y2
2
Ist Sitting TF No. 333 LO 2)
30. If Un = cosn + sinn then the Value of 2u6 – 3u4 + 2 is:
;fn x cos T – y sin T = x 2  y2 vkSj
;fn Un = cosn + sinn gS] rc 2u6 – 3u4 + 2 dk eku D;k
cos T sin T
2 2
1 gksxk\
 gS] rks lgh laca/ gSµ (1) 1 (2) 5 (3) O (4) –1
a2 b2 x 2  y2
31. If x = a secn and y = b tann then the value of is:
x 2 y2
(1) 2  2 1
x 2 y2
(2) 2  2 1 ;fn x = a secn vkSj y = b tann gS] rc dk eku D;k
b a a b gksxk\
x 2 y2 x 2 y2 1 1 2 2

(3)
b2 a 2
1 (4) –
a 2 b2
1
FxI
(1) G J
n FG y IJ n
1
F x I FG y IJ
(2) G J
m n
1
(SSC CHSL DEO & LDC Exam. 20.10.2013) HaK HbK Ha K HbK
25. If x = a sec cos , y = b sec sin , z = c tan , then
x2 y2 z2 FxI
(3) G J
2
FG y IJ2
1
the value of  
a 2 b2 c 2
is HaK HbK (4) None of these

;fn x = a sec cos , y = b sec sin , z = c tan , rks 32. If x = a cos 3 , y = b sin 3 then the value of
2 2
x2 y2 z2
  dk eku D;k gksxk \ FG x IJ FG y IJ
3 3
?
a 2 b2 c 2 Ha K Hb K
(1) 2 (2) 0 (3) 1 (4) –1
2 2
(SSC CGL Tier-I Re-Exam. (2013) 20.07.2014 (IInd Sitting)
26. If a cos T + b sin T = p and a sin T – b cos T = q, then ;fn x = a cos3 , y = b sin3 gS] rks
FG x IJ FG y IJ
3 3
?
the relation between a, b, p and q is Ha K Hb K
;fn a cos T vkSj a sin T - b cos T = q, rks
b sin T = p (1) 1 (2) 0 (3) 2 (4) 4
a, b, p vkSj q ds chp D;k laca/ gSA 33. If x = a sin and y = b tan then the value of
(1) a2 – b2 = p2 – q2 (2) a2 + b2 = p2 + q2 a2 b2
(3) a + b = p + q (4) a – b = p – q is:
x2 y2
(SSC CGL Tier-I Exam. 19.10.2014 (Ist Sitting)
27. If x = a (sinT + cosT), y = b (sinT – cosT) then the value a2 b2
;fn x = a sin vkSj y = b tan gS] rc x 2 y2 dk eku
x2 y2
of  is
a 2 b2 D;k gksxk\
2 2 (1) 5 (2) –1 (3) 1 (4) 3
x y
;fn x = a (sinT + cosT), y = b (sinT – cosT) rks  dk 34. If (sin + sec )2 + (cos + cosec )2 = (K + sec .
a 2 b2 cosec )2 then the value of K is;
eku D;k gksxk \ ;fn (sin + sec )2 + (cos + cosec )2 = (K + sec .
(1) 0 (2) 1 (3) 2 (4) –2 cosec ) 2
gS] rc K dk eku D;k gksxk\
(SSC CGL Tier-II Exam. 21.09.2014)
(1) 1 (2) 2 (3) 3 (4) 4

BLAM–158
f=kdks.kfefr
(1) (bp – aq) (2) (aq – bq)2
cos2 sin 3
35. If K + sin . cos then (3) (aq + bp)2 (4) None of these
1 tan sin cos 43. If x cos + y sin = 4 and x cos y sin = 0 then which
the value of K is; of these is true ?
cos2 sin 3 ;fn x cos + y sin = 4 rFkk x cos y sin = 0 gSa rc buesa
;fn K + sin . cos gS] rc ls dkSu&lk lR; gS\
1 tan sin cos
K dk eku D;k gksxk\ (1) x2 + y2 = 4 (2) x2 + y2 = 16
(1) 1 (2) 2 (3) 3 (4) 4 1 1 1 1 1
36. If x = sec – tan and y = cosec + cot then (3) x 2 (4) x 2 4
y2 4 y2
;fn x = sec – tan vkSj y = cosec + cot rc
44. If 3x sin + 2y cos = 4 and 2x sin – 3y cos = 2 then
(1) xy + 1 = x – y (2) xy + 1 = x – 2y what is the relation between x and y ?
(3) xy + 1 = x + y (2) xy + 1 = y – x
37. tan 20°. tan 40°. tan 60°. tan 80° = ?
;fn 3x sin + 2y cos = 4 rFkk 2x sin – 3y cos = 2 rc
x vkSj y ds chp esa D;k lEcU/ gS ?
3
(1) 1 (2) 2 (3) 3 (4) 81 49 36 64
2 (1) x 2 144 (2) x 2 121
y2 y2
38. If tan + sin = m and tan – sin = n then the value
of m2 – n2 is;
289 4 256 4
;fn tan + sin = m vkSj tan – sin =n gS rc m2 – (3) x2 y2
16 (4) x y2
169
n2 dk eku D;k gksxk\
45. If sin + cos = a and sec + cosec = b than the
(1) mn (2) 2 mn (3) 3 mn (4) 4 mn value of b (a2 –1) is:
39. If tan + sin = m and tan – sin = n then the value ;fn sin + cos = a vkSj sec + cosec = b gS rc b (a2
of mn is; –1 dk eku D;k gksxk\
(1) 2a (2) 3a (3) 0 (4) 2ab
;fn tan + sin = m vkSj tan – sin = n gS rc mn dk
eku D;k gksxk\ Type-V
sin (x + y), cos (x + y), tan (x + y) etc.
1
(1)
2
m2d n2 h (2) 2 m
2
d n2 h formulae ij based questions

1 1 1. If cot (A + B) = x, then value of x is


(3) 4 m 2
d n2 h (4)
4
m2 d n2 h ;fn cot (A + B) = x, gks rks x dk eku D;k gksxk\
40. If cosec – sin = m and sec – cos = n than the
cot A cot B 1 cot A cot B 1
2 2 (1) (2)
value of m n d h dmn h
2 3 2 3
is; cot A + cot B cot A – cot B

cot A cot B – 1 cot A cot B – 1


;fn cosec – sin = m vkSj sec – cos = n rc (3) (4)
cot B – cot A cot A – cot B
2 2

dm n h dmn h
2 3 2 3
dk eku D;k gksxk\ 2. What is the value of tan 56° ?
(1) 1 (2) 2 (3) 3 (4) 4 tan 56° dk eku D;k gksxk\
41. If cot + tan = x and sec – cos = y then the value
cos11 – sin 11 cos11 sin 11
2 2
(1) (2)
d h d h
of x 2y 3
xy 2 3 is; cos11 sin 11 cos11 – sin 11

;fn cot + tan = x vkSj sec – cos = y gS] rc cos11 sin 11 sin 11 – cos11
(3) sin 11 – cos11 (4) cos11 sin 11
2 2

d x y h dxy h
2 3 2 3
dk eku D;k gksxk\
(1) 4 (2) 3 (3) 2 (4) 1 FG x IJ
42. If a sec + b tan + c = 0 and P. sec + q tan +r=
3. What is the value of tan
H4 K ?

0 then the value of (br – qc)2 – (pc – ar)2 = ?


;fn a sec + b tan +c=0 vkSj P. sec + q tan + FG x IJ dk eku fdruk gksxk\
r=0 rc 2
(br – qc) – (pc – ar) = ? 2
tan
H4 K
BLAM–159
f=kdks.kfefr
1 – tan x 1 tan x FG x IJ cos
FG – x IJ
(1) 1 tan x (2) 1 – tan x
8. The value of cos
H4 K H4 K will be

1 cot x 1 – cot x FG I F I
x J cosG – x J dk eku D;k gksxk\
(3) 1 – cot x (4) 1  cot x
cos
H4 K H4 K
4. If cosC – cosD = y, then the value of y is (1) 2 sin x (2) 2 cos x
;fn cosC – cosD = y, gks rks y dk eku D;k gksxk\
(3) 2 cosec x (4) 2 tan x

(1) 2 sin G
F C D IJ sin FG C – D IJ 9. What is the value of cos 105° ?
H 2 K H 2 K cos 105° dk eku D;k gksxk\

(2) 2 cosGH
F C D IJ .cosFG C – D IJ 1– 3 1 3
2 K H 2 K (1) (2)
2 2 2 2
F C D IJ sin FG C – D IJ
(3) – 2 sin GH
2 K H 2 K 3 –1 3
(3) (4)
2 2 2 2

(4) – 2 cosGH
F C D IJ cosFG C – D IJ FG – IJ cosFG – IJ FG – IJ
2 K H 2 K – sin
10. The value of cos H4 K H4 K H4 K
5. If sinx .cosy + cosx.siny = 1, then the value of x + y
will be
sin
FG – IJ will be
;fn sinx .cosy + cosx .siny = 1, gks rks x+y dk eku D;k H4 K
gksxk\
F I F I
cos GH – JK cosGH – JK – sin
FG – IJ sin
FG – IJ dk
(1) (2) – (3) (4)
4 4 H4 K H4 K
2 2 3 4
eku D;k gksxk\
3 3 (1) sin(T – ) (2) sin(T + )
6. If cosx = – and <x< , then the value of sin (3) cos(q – f) (4) cos(q + f)
5 2
11. sin 75° + sin 15° can be expressed as
2x will be
sin 75° + sin 15° dks fn, x, fodYikas esa ls fdl :i esa
3 3
;fn cosx = – gks rFkk <x< gks] rks sin 2x dk eku fy[k ldrs gSa\
5 2
D;k gksxk\ 3 2
(1) (2)
2 3
12 1 24 5
(1) (2) (3) (4)
25 15 25 26 2 3
(3) (4)
7. If sinC + sinD = x, then the value of x is 3 2
;fn sinC + sinD = x, gks rks x dk eku D;k gksxk\ 12. What will be the value of 2 cos 45°. sin15°

FG C D IJ sin FG C – D IJ 2 cos 45°. sin15° dk eku D;k gksxk\


(1) 2 sin
H 2 K H 2 K 3 1 3 –2
(1) (2)
2 2
F C – D IJ cosFG C D IJ
2 sin G
(2)
H 2 K H 2 K 3 –1 2
(3) (4)
2 3 1
2 cosG
F C D IJ cosFG C – D IJ
(3)
H 2 K H 2 K 13. The value of sin 22
1
q will be
2
F C D IJ cosFG C – D IJ
2 sin G
(4)
H 2 K H 2 K sin 22
1
2
q dk eku D;k gksxk\

BLAM–160
f=kdks.kfefr
19. If 3 tan tan = 1, then what is the value of
2 1
(1) 2 –1 (2)
2 2
cos b g
cos b g?
2 –1 2 –1
(3)
2
(4)
2 2
;fn gS rc cos
cos b g dk eku D;k
3 tan tan = 1,
b g
cos cos
14. The value of will be gksxk\
sin sin
1 1
(1) (2) 2 (3) (4) 3
cos cos 2 3
dk eku D;k gksxk\
sin sin
sin( x y) a b tan x
20. If sin( x – y ) ,
FG IJ FG IJ a – b then the value of tan y is
(1) tan (2) cot
H 2 K H 2 K equal to

F
tan G
– IJ cot G
F – IJ sin( x
;fn sin( x – y )
y) a b
,
tan x
gks rks tan y fdlds cjkcj
(3)
H 2 K (4)
H 2 K a –b
gksxk\
m 1
15. If tan = m 1 , tan = 2m  1 , then + equal to 2a a a 2a
(1) (2) (3) (4)
b b 2b 3b
m 1
;fn tan = m , gks rks fdlds 4 5
1 , tan = 2m  1 +
21. If cos ( + ) = and sin ( – ) = where , lies
5 13
cjkcj gksxk\
between 0 and then what is the value of tan 2 ?
4
(1) (2) (3) (4)
2 6 3 4
4 5
;fn cos ( + ) =
5
vkSj sin ( – ) =
13
tgk¡ , 0 vkSj
16. If + = , then the value of (1 + tan )(1 + tan )
4
is 4
osQ chp gS] rc tan 2 dk eku D;k gksxk\

;fn + = gks rks (1 + tan )(1 + tan ) dk eku fdlds 56 36 33 49


4 (1) (2) (3) (4)
33 33 56 36
cjkcj gksxk\
5
(1) 1 (2) 2 (3) –2 (4) 5 22. The value of sin sin is
12 12
17. If cos ( – A) = a, cos ( – B) = b then what is the value
of sin2 (A – B) + 2ab cos (A – B)? 5
sin 12 sin
12
dk eku D;k gksxk\
;fn cos ( – A) = a, cos ( – B) = b gS] rc sin2 (A – B) +
2ab cos (A – B) dk eku D;k gksxk\ 1 1 1 1
(1) (2) (3) (4)
2
(1) a – b 2
(2) a2 + b2 4 8 5 6
(3) b2 – a2 (4) 2ab
cos( x )cos(– x )
23. The value of is
18. The value of 2cos cos
9
+ cos
3
+ cos
5
is sin( – x )cos
FG x IJ
13 13 13 13 H2 K
9 3 5
2 cos cos + cos + cos dk eku D;k gksxk\ cos( x )cos(– x )
13 13 13 13 dk eku D;k gksxk\
sin( – x )cos
FG x IJ
1 1 1 H2 K
(1) (2) 0 (3) – (4)
2 2 8
(1) tan2x (2) cos2x (3) cot2x (4) sec2x

BLAM–161
f=kdks.kfefr
24. If tan – tan = x and cot – cot = y then cot ( 30. What is the value of [(cos3 2T + 3 cos 2T) ÷ (cos6 T –
– )? sin6 T)]?

;fn tan – tan vkSj


=x cot – cot =y gS] rc cot ( [(cos3 2T + 3 cos 2T) ÷ (cos6 T– sin6 T)] dk eku D;k gS\
(1) 0 (2) 1
– ) dk eku D;k gksxk\
(3) 4 (4) 2
1 1 1 1
(1) x y (2) x y
FG AIJ × tan FG 3 AIJ ?
31. What is the value of tan
H4 K H4 K
1 1
(3) x + y (4) y x FG I F 3 AIJ dk eku D;k gS\
A J × tan G
tan
H4 K H4 K
bsin x sin y g bsin x – sin y g (1) 1 (2) 0
25. What is the value of
bcos x cos y g bcos y – cos x g
?
A
(3) cot (4) – 1
2
bsin x gb
sin y sin x – sin y g (SSC CGL Tier-II (CBE) Exam. 18.02.2018)
bcos x cos y gbcos y – cos x g
dk eku D;k gS\
3 4
(1) 0 (2) 1 32. If cos = and cos = , where and are both
5 5
(3) –1 (4) 2 positive accute angle, then what is value of cos
26. What is the value of sin (B – C) cos (A – D) + sin (A –
B) cos (C – D) + sin (C – A) cos (B – D)? FG IJ ?
sin (B – C) cos (A – D) + sin (A – B) cos (C – D) + sin (C H 2 K
– A) cos (B – D) dk eku D;k gS\
3 4
3 ;fn cos =
5
vkSj cos =
5
, tgk¡ vkSj /ukRed U;wu
(1) (2) – 3 (3) 1 (4) 0
2
27. What is the value of dks.k gS] rc
FG IJ dk eku D;k gksxk\
[sin (y – z ) + sin (y + z ) + 2 sin y]
cos
H 2 K
=?
[sin ( x – z ) + sin ( x + z ) + 2 sin x ] 7 7
[sin (y – z ) + sin (y + z ) + 2 sin y ] (1) (2)
[sin ( x – z ) + sin ( x + z ) + 2 sin x ]
dk eku D;k gS\ 2 5 2

7 7
bsin y g (3)
5
(4)
2 5
(1) cosx siny (2)
bsin x g 33. What is the value of [(sin 59° cos 31° + cos 59° sin 31°)
(3) sinz (4) sinx tany ÷ (cos 20° cos 25° – sin 20° sin 25°)]?
R| sin bx + y g – 2 sinx + sin b x – y g U| [(sin 59° cos 31° + cos 59° sin 31°) ÷ (cos 20° cos 25°
28. What is the value of S| cos b x – y g + cos b x + y g – 2 cosx V| × – sin 20° sin 25°)] dk eku D;k gS\
T W
LM bsin 10x – sin 8x g OP 1
(1) (2) 2 2 (3) 3 (4) 2
N (cos10x cos 8 x ) Q = ? 2
(SSC CGL Tier-II (CBE) Exam. 18.02.2018)
R| sin bx + y g – 2 sinx + sin b x – yg U| LM bsin 10x – sin 8x g OP 34. What is the value of cos (90° – B) sin (C – A) + sin (90°
S| cos b x – yg + cos b x + yg – 2 cosx V| ×
N (cos10x cos 8 x ) Q + A) cos (B + C) – sin (90° – C) cos (A + B) ?
T W
cos (90° – B) sin (C – A) + sin (90° + A) cos (B + C) – sin
dk eku D;k gS\
(90° – C) cos (A + B) dk eku D;k gS\
(1) 0 (2) tan2x
(1) 1 (2) sin ( A + B – C)
(3) 1 (4) 2tanx
29. What is the value of [(sin 7x – sin 5x) ÷ (cos 7x + cos 5x)] (3) cos ( B + C – A) (4) 0
– [(cos 6x – cos 4x) ÷ (sin 6x + sin 4x)]? (SSC CGL Tier-II (CBE) Exam. 18.02.2018)
[(sin 7x – sin 5x) ÷ (cos 7x + cos 5x)] – [(cos 6x – cos 4x) 35. What is the value of [(cos 7A + cos 5A) ÷ (sin 7A – sin
÷ (sin 6x + sin 4x)] dk eku D;k gS\ 5A)]?
(1) 1 (2) 2 tan x [(cos 7A + cos 5A) ÷ (sin 7A – sin 5A)] dk eku D;k gS\
3x (1) tan A (2) tan 4A
(3) tan 2 x (4) tan (3) cot 4A (4) cot A
2
(SSC CGL Tier-II (CBE) Exam. 19.02.2018)
(SSC CGL Tier-II (CBE) Exam. 18.02.2018)

BLAM–162
f=kdks.kfefr
36. What is the value of
(cos 40 – cos 140 )
[1 – sin (90q – 2A)] 43. What is the value of ?
(sin 80 + sin 20 )
?
[1 + sin (90q + 2A)]
(cos 40 – cos 140 )
[1 – sin (90q – 2A)] dk eku D;k gS\
[1 + sin (90q + 2A)]
dk eku D;k gS\ (sin 80 + sin 20 )

(1) sinA cosA (2) cot2A 2 1


(3) tan2A (4) sin2A cosA (1) 2 3 (2) (3) (4) 3
3 3
(SSC CGL Tier-II (CBE) Exam. 19.02.2018)
(SSC CGL Tier-II (CBE) Exam. 21.02.2018)
37. What is the value of sin 75° + sin 15°?
[1 – tan (90 – ) + sec (90 – )]
sin 75° + sin 15° dk eku D;k gS\ 44. What is the value of tan (90 – ) + sec (90 – ) + 1] ?

3 3 [1 – tan (90 – ) + sec (90 – )]


(1) 3 (2) 2 3 (3) (4) tan (90 – ) + sec (90 – ) + 1]
dk eku D;k gS\
2 2
(SSC CGL Tier-II (CBE) Exam. 19.02.2018)
38. What is the value of [(cos 3T + 2cos 5T + cos 7T) ÷ (1) cot (2) tan (3) sin (4) cos
2 2
(cos T + 2cos 3T + cos 5T)] + sin 2T tan 3T ?
(SSC CGL Tier-II (CBE) Exam. 21.02.2018)
[(cos 3T + 2cos 5T + cos 7T) ÷ (cosT + 2cos 3T+ cos
[sin (90 – A) + cos (180 – 2A)]
5T)] + sin 2T tan 3T dk eku D;k gS\ 45. What is the value of [cos (90 – 2A) + sin (180 – A)] ?
(1) cos 2T (2) sin 2T
(3) tan 2T (4) cotT sin 2T [sin (90 – A) + cos (180 – 2A)]
dk eku D;k gS\
(SSC CGL Tier-II (CBE) Exam. 19.02.2018) [cos (90 – 2A) + sin (180 – A)]
[2 sin (45q + ) sin (45q –
39. What is the value of
cos 2
)]
? FG A IJ cosA (2) cot A
(1) sin
H 2K 2
[2 sin (45q + ) sin (45q – )]
dk eku D;k gS\ A A
cos2
(3) tan (4) sinA cos
(1) 0 (2) tan 2T 2 2
(3) cot 2T (4) 1 (SSC CGL Tier-II (CBE) Exam. 21.02.2018)
(SSC CGL Tier-II (CBE) Exam. 19.02.2018)
40. What is the value of [cos (90° + A) ÷ sec (270 – A)] + 46. tan
FG IJ tan
FG IJ ?
[sin (270° + A) ÷ cosec (630° – A)]? H4 K H4 K
[cos (90° + A) ÷ sec (270° – A)] + [sin (270° + A) ÷ (1) 2 tan 2 (2) 2 cot 2
cosec (630° – A)] dk eku D;k gS\ (3) tan 2 (4) cot 2
(1) 3 sec A (2) tan A sec A
(3) 0 (4) 1 sin B b A g cos B b A g ?
(SSC CGL Tier-II (CBE) Exam. 19.02.2018)
47. sin B
b A g cos B b A g
41. What is the value of
cos B sin B cos A sin A
{bsin 4x + sin 4yg btan 2x – tan 2yg } ? (1) cos B – sin B (2) cos A – sin A
bsin4x – sin4yg
cos A sin A cos B sin B
(3) cos A + sinA (4) cos B + sin B
{bsin 4 x + sin 4yg btan 2x – tan 2y g } dk eku D;k gS\
bsin4 x – sin4y g FG 2 IJ FG 4 IJ then what
z cos
(1) tan2 (2x + 2y) (2) tan2x 48. If x cos = y cos
H 3 K H 3 K
(3) cot (x – y) (4) tan 2x + tan 2y
(SSC CGL Tier-II (CBE) Exam. 21.02.2018) 1 1 1
42. What is the value of is value of x y z ?
dk eku D;k gS\
;fn
FG 2 IJ z cos
FG 4 IJ gS] rc
e32 cos x – 48 c os
6 4
x 18 cos2 x – 1 j ?
x cos = y cos
H 3 K H 3 K
4 sin x cos x sin (60 – x )cos (60 – x )sin (60 x )cos (60 x)
1 1 1
(1) 4 tan 6x (2) 4 cot 6x
x y z bldk eku D;k gksxk\
(3) 8 cot 6x (4) 8 tan 6x
(SSC CGL Tier-II (CBE) Exam. 21.02.2018) (1) 1 (2) 2 (3) 0 (4) 3 cos Q

BLAM–163
f=kdks.kfefr
2 2 2
49. cos + cos ( + 120°) + cos ( – 120°) = ?
3 1
;fn r sin T = 1, r cos T = 3 gks] rks e 3 tan T  1 j dk
(1)
2
(2) 1 (3)
2
(4) O eku D;k gksxk\
50. If A + B + C = 180°, then what is the value of 1
(1) 3 (2) (3) 1 (4) 2
tan A tan B tan C 3
? (SSC Graduate Level Tier-II Exam. 29.09.2013)
tan A tan B tan C
6. If tan2 = 1 + 2 tan2 ( , are positive acute angles),
tan A tan B tan C
;fn A + B + C = 180° gS] rc tan A tan B tan C
dk then 2 cos – cos is equal to

eku D;k gksxk\ ;fn tan2 = 1 + 2 tan2 gks (tgk¡ , èkukRed U;wu dks.k
(1) 0 (2) 2 (3) 1 (4) –1 gSa)] rks 2 cos – cos fdlds cjkcj gksxk\
(1) 0 (2) 2 (3) 1 (4) –1
Miscellaneous
(SSC CHSL DEO & LDC Exam. 10.11.2013, Ist Sitting)

1. If x, y are acute angles, 0 < x + y < 90° and sin (2x – 7. If is a positive acute angle and 4 cos2 – 4 cos +1
20°) = cos (2y + 20°), then the value of tan (x + y) is : = 0, then the value of tan ( – 15° ) is equal to
;fn U;wu dks.k gSa] 0 < x + y < 90° vkSj sin (2x – 20°)
x, y ;fn /ukRed U;wudks.k gS vkSj 4 cos2 – 4 cos + 1 =
= cos (2y + 20°) rks tan (x + y) dk eku gS %
0, rks tan ( – 15° ) dk eku fuEufyf[kr esa ls fdlds cjkcj
1 3 gks xk \
(1) (2) (3) 3 (4) 1
3 2 1
(SSC CHSL DEO & LDC Exam. 11.12.2011 (1) 0 (2) 1 (3) 3 (4)
(IInd Sitting (Delhi Zone) 3
2. In a right-angled triangle XYZ, right-angled at Y, if (SSC CHSL DEO & LDC Exam. 9.11.2014)
8. If and are positive acute angles, sin (4 – ) = 1
XY = 2 6 and XZ – YZ = 2, then sec X + tan X is
1
,d ledks.k f=kHkqt XYZ esa] tks Y ij ledks.k gS] ;fn XY = and cos (2 + )= , then the value of sin ( + 2 ) is
2
2 6 rFkk XZ – YZ = 2 gks] rks sec X + tan X fdruk gksxk\
;fn vkSj /u U;wudks.k gS]a sin (4 – )=1 vkSj cos (2
1 6 + )
(1) (2) 6 (3) 2 6 (4)
6 2 1
(SSC Graduate Level Tier-II Exam. 16.09.2012) =
2
, rks sin ( +2 ) dk eku gSµ
1 1 (1) 0 (2) 1
3. If sin (A – B) = and cos (A + B) = where A > B >
2 2
3 1
0 and A + B is an acute angle, then the value of B is (3) (4)
2 2
1 1
;fn sin (A – B) = 2 vkSj cos (A + B) = 2 tgk¡ A > B > 0 rFkk (SSC CHSL DEO & LDC Exam. 02.11.2014 (IInd Sitting)

2
A+B U;wu dks.k gS rks B dk eku D;k gksxk\ 9. If r cose 3 j + (r sin –1)2 = 0 then the value of

(1) (2) (3) (4) r tan sec


6 12 4 2 is equal to
r sec tan
(SSC Graduate Level Tier-I Exam. 21.04.2013)
4. If T is positive acute angle and 3 (sec2 T + tan2 T) = 5, r tan sec
2
then which is right option ? ;fn (r cos – 3 ) + (r sin –1)2 = 0 rks
r sec tan
;fn T /ukRed U;wudks.k gS vkSj 3 (sec2 T + tan2 T) = 5, rks
fdlds cjkcj gksxk \
dkSu&lk fodYi lgh gS\
(1) cos 2T = sin 2T (2) cos 2T = sin T 4 5
(1) (2)
(3) cos 2T = tan T (4) cos 2T = cos T 5 4
(SSC Graduate Level Tier-I Exam. 19.05.2013)
3 5
5. If r sin T = 1, r cos T = 3 , then the value of (3) (4)
4 4
e 3 tan T  1 is j (SSC CHSL DEO & LDC Exam. 9.11.2014)

BLAM–164
f=kdks.kfefr
10. In an isosceles ABC, B is a right angle. There is a
(1) 2 sin A (2) 2 sin A
point D inside ABC. From point D, the foot of per-
pendicular on sides AB and AC are P and Q respec-
(3) 2 sin A (4) 2 sin A
tively. If AP = a cm, AQ = b cm and BAD = 15°, then
the value of sin 75° will be ? (SSC CHSL (10+2) LDC, DEO & PA/SA Exam, 06.12.2015
(Ist Sitting) TF No. 1375232)
,d lef}ckgq f=kHkqt ABC dk B ij cuk dks.k ledks.k gSA mlesa 16. The value of sin + sin ( + 120°) + sin ( + 240°) is
D, ABC ds varxZr ,d fcanq gSA ml fcanq D ls ABC dh equal + to
Hkqtkvksa AB rFkk AC ij cuk, yacksa ds ikn Øe'k% P rFkk Q sin + sin ( + 120°) + sin ( + 240°) dk eku cjkcj gksxk
gSaA rnuqlkj] ;fn AP = a lseh, AQ = b lseh rFkk BAD = 15°
(1) 0 (2) 1 (3) 3 (4) 2
gks] rks sin 75° fdruk gksxk\
(NDA)
2b a 3a 2a
(1) (2) (3) (4) sin 3 cos 3
3a 2b 2b 3b 17. If O < < 90° and x = and y =
sin cos
11. a, b, c are the lengths of three sides of a triangle
ABC. If a, b, c are related by the relation a2 + b2 + c2 cos 3 sin 3
, then which one of the following is
= ab + bc + ca, then the value of sin2A + sin2B + sin2C cos sin
is correct?
a, b ,oa c ,d f=kHkqt ABC dh rhu Hkqtkvksa dh yackb;k¡ gSaA ;fn sin 3 cos 3
a, b ,oa c dks a2 + b2 + c2 = ab + bc + ca ds laca/ ls tksM+ ;fn O < < 90° vkSj x =
sin cos
vkSj y =
fn;k tk, rks sin2A + sin2B + sin2C dk eku D;k gksxk \
cos 3 sin 3
3 3 3 3 9 cos sin
, rc fuEu esa ls dkSu&lk lgh gS\
(1) (2) (3) (4)
4 2 2 4 (1) x + y Implies = 15°
(SSC CGL Tier-II Exam. 12.04.2015 TF No. 567 TL 9) (2) x = –y Implies = 60°
12. In ABC, C = 90° and AB = c, BC = a, CA = b; then
the value of (cosec B – cos A) is (3) 2 x = y Implies = 30°

ABC esa] vkSj AB = c, BC = a, CA = b; rks


C = 90° 1
(cosec B – cos A) dk eku D;k gS\ (4) 3 x = y Implies = 22
2
(NDA)
c2 b2 a2 bc
18. The expression
(1) (2) (3) (4)
ab ca bc a2
(SSC CGL Tier-I Re-Exam, 30.08.2015) RS FG 3
3 sin 4
IJ b
sin 4 3 gUVW 2RSTsin FGH 2 IJK UVW +
6

13. If 0° < T < 90° and cosecT = cot2T, then the value of T H2 K
the expression cosec4T – 2cosec3T + cot2T is equal to: sin6 (5 – ) is equal to—
;fn 0° < T < 90° vkSj cosecT = cot2T gS]
rks O;atd cosec4T
– 2cosec3T + cot2T dk eku fdlds cjkcj gksxk\
RS FG 3
3 sin 4
IJ b
sin 4 3 gUVW 2RSTsin FGH 2 IJK UVW +
6

(1) 2 (2) 0 (3) 1 (4) 3 T H2 K


(SSC CHSL (10+2) LDC, DEO & PA/SA Exam, 15.11.2015 sin6 (5 – )
(Ist Sitting) TF No. 6636838)
14. If 4sin2T – 1 = 0 and angle T is less than 90°, the dk eku cjkcj gksxkA
value of cos2T + tan2T is : (Take 0° < T < 90°) (1) sin 2 + sin 3 (2) 3
;fn 4sin2T – 1 = 0 vkSj
dks.k T, 90° ls de gS] rks cos2T + (3) 1 (4) O
(NDA)
tan T dk eku D;k gksxk\ (ekuk] 0° < T < 90°)
2

19. The value of 3 cot 20 –4 cos 20 is—


17 13 11 12
(1) (2) (3) (4)
15 12 9 11 3 cot 20 –4 cos 20 dk eku D;k gksxkµ
(SSC CHSL (10+2) LDC, DEO & PA/SA Exam, 15.11.2015 (1) 1 (2) –1
(Ist Sitting) TF No. 6636838)
(3) 0 (4) None of these
15. If cos A + sin A = 2 cos A then cos A – sin A is equal 20. If x + y = z, then 1 + cos x + cos y + cos z is equal to
to : (where 0° < A < 90°) –
;fn cos A + sin A = 2 cos A rks cos A – sin A dk eku ;fn x + y = z, rc 1 + cos x + cos y + cos z eku cjkcj
fdlds cjkcj gksxk\ (;gk¡ 0° < A < 90°) gksxk µ

BLAM–165
f=kdks.kfefr
(3) 0
x y z x y z
(1) 4 cos cos sin (2) 4 cos cos cos (4) None of these
2 z 2 2 2 2
(NDA)
x y z x y z 27. If ABC are the angles of a triangle, then sin2A + sin2B
(3) 4 cos sin cos (4) 4 sin cos cos + sin2C – 2 cos A cos B cos C = ?
2 2 2 2 2 2
(NDA)
;fn ABC f=kHkqt osQ rhuksa dks.k gS] rks sin2A + sin2B + sin2C
21. The value of cos3 + cos3 (120° + ) + cos3 ( – 120°) – 2 cos A cos B cos C = ?
is– (1) 1 (2) 2 (3) 3 (4) 4
(NDA)
cos3 + cos3 (120°+ ) + cos3 ( – 120°) dk eku gksxkµ 28. What is tan4 A – sec4 A + tan2 A + sec2 A equal to ?
3 3 tan4 A – sec4 A + tan2 A + sec2 A dk eku cjkcj gSµ
(1) cos 3 (2) sec3 (1) 0 (2) 1 (3) 2 (4) –1
2 4
(NDA)
3 3 29. If x = sin + cos and y = sin . cos , then what is
(3) tan 3 (4) cos 3
2 4 the value of x4 – 4x2y –2x2 + 4y2 + 4y + 1?
(NDA) ;fn x = sin + cos vkSj y = sin . cos , rks x4 –4x2y –
22. cos 25° + cos 85° + cos 155° = ? 2x2 + 4y2 + 4y + 1 dk eku gksxk?
1 (1) 0 (2) 1
(1) 0 (2) (3) 2 (4) None of these
3
30. The angle A lies in the third quadrant and it satisfies
1 the equation 4 (sin2 A + cos A) = 1, what is the mea-
(3) (4) cos 275° sure of the angle A?
2
dks.k A r`rh;
ikB esa vofLFkr gS rFkk ;g 4 (sin2 A + cos A)
(NDA)
= 1 dks larq"V djrk gS rc dks.k A dk eki gSµ
23. If sin + ....to = sec4 , (1) 225° (2) 240°
sin sin sin
(3) 210° (4) None of these
then sin is equal to—
SHORT ANSWERS
;fn sin + sin sin sin ....to = sec4 ,

rkssin dk eku cjkcj gSµ Type-I


(1) sec2 (2) tan2 Degree, Radians, buosQ vkil esa
(3) sec2 tan2 (4) cos2
(NDA)
Conversions, clock osQ hands osQ chp esa cus
24. If tan2 = 2 tan2 + 1, then cos2 + sin2 equal is— angle ij based questions
;fn tan2 = 2 tan2 + 1, rks cos2 + sin2 eku cjkcj gSµ
(1) –1 (2) 0 1. (2) 2. (3) 3. (1) 4. (3) 5. (2) 6. (1)
(3) 1 (4) None of these 7. (1) 8. (2) 9. (3) 10. (3) 11. (2) 12. (3)
(NDA) 13. (3) 14. (3) 15. (4)

b1 tan 2 cot 62 g Type-II


25. If
btan 152 cot 88 g = k 3 , then the value of K

is– Angles ij based questions

b1 tan 2 cot 62
;fn tan 152 cot 88
g= ] rks dk eku gksxkµ
1. (2) 2. (4) 3. (3) 4. (2) 5. (4) 6. (3, 4)
b g k 3 K
7. (3) 8. (4) 9. (4) 10. (1) 11. (4) 12. (3)
13. (1) 14. (1) 15. (3) 16. (3) 17. (2) 18. (3)
1 1
(1) (2) –1 (3) 1 (4) 19. (1) 20. (1) 21. (1) 22. (2) 23. (2) 24. (2)
2 2
25. (1) 26. (3) 27. (1) 28. (4) 29. (3) 30. (1)
(NDA)
31. (1) 32. (4) 33. (2) 34. (1) 35. (1) 36. (3)
26. tan 5x tan 3x tan 2x = ?
37. (1) 38. (3) 39. (4) 40. (3) 41. (2) 42. (1)
(1) tan 5x – tan 3x – tan 2x
43. (1) 44. (1) 45. (1) 46. (4) 47. (1) 48. (4)
sin 5x sin 3x sin 2x 49. (2) 50. (2) 51. (2) 52. (1) 53. (1) 54. (3)
(2)
cos 5x cos 3x cos 2x 55. (1) 56. (2) 57. (1) 58. (2) 59. (3)

BLAM–166
f=kdks.kfefr
Type-III EXPLANATIONS
Trigonometric identities ij
based questions Type-I
Degree, Radians, buosQ vkil esa
1. (4) 2. (2) 3. (3) 4. (4) 5. (1) 6. (2)
7. (2) 8. (3) 9. (4) 10. (3) 11. (3) 12. (1) conversion, clock osQ hands osQ chp esa cus
13. (1) 14. (2) 15. (4) 16. (3) 17. (3) 18. (3) angle ij based questions
19. (4) 20. (4) 21. (1) 22. (4) 23. (2) 24. (3)
25. (2) 26. (1) 27. (3) 28. (3) 29. (1) 30. (2) 1. (2) radian = 180°
31. (4) 32. (1) 33. (1) 34. (3) 35. (1) 36. (3) 1 radian
37. (4) 38. (1) 39. (3) 40. (2) 41. (1) 42. (1)
43. (4) 44. (1) 45. (4) 46. (3) 47. (1) 48. (1)
FG 180 IJ o
180 7 630 FG 57 3 IJ o

49. (4) 50. (1) 51. (1) 52. (4) 53. (4) 54. (2)
=
H K =
22
=
11 H 11K
55. (3) 56. (1) 57. (4) 58. (2) 59. (2) 60. (1) 3 180
61. (2) 62. (3) = 57 60' 57
11 11

Type-IV 4
= 57 16' 60" 57 16 22
Trigonometric functions dh minimum, 11

maximum values, substitution,


2. (3)  1 radian =
FG 180 IJ o

rFkk series ij based questions H K


3 180 3
1. (3) 2. (4) 3. (2) 4. (4) 5. (4) 6. (3) radian = × = 108°
5 5
7. (3) 8. (1) 9. (4) 10. (3) 11. (1) 12. (1)
13. (2) 14. (1) 15. (4) 16. (1) 17. (2) 18. (2) 51 ' 17 '
19. (2) 20. (4) 21. (1) 22. (3) 23. (1) 24. (2) 3. (1) 63° 14' 51" = 63 14' 63 14'
60 20
25. (3) 26. (2) 27. (3) 28. (4) 29. (3) 30. (1)
31. (2) 32. (4) 33. (3) 34. (1) 35. (1) 36. (4)
= 63 14
FG 17 IJ '
63
FG 280 17 IJ '

37. (3) 38. (4) 39. (2) 40. (1) 41. (4) 42. (2) H 20 K H 20 K
43. (3) 44. (4) 45. (1)

= 63q
FG 297 IJ '

63q
FG 297 IJ q
99
Type-V H 20 K H 20 u 60 K = 63
400
Si n ( x + y), cos (x + y), tan (x + y) etc.
formulae ij based questions 25200 99 F 25299 IJ
= G
=
400 H 400 K
1. (1) 2. (2) 3. (2) 4. (3) 5. (1) 6. (3)
7. (4) 8. (2) 9. (1) 10. (2) 11. (4) 12. (3) 1 Radian
180
13. (4) 14. (2) 15. (4) 16. (2) 17. (2) 18. (2)
19. (3) 20. (2) 21. (2) 22. (1) 23. (3) 24. (2)
FG 25299 IJ 25299 FG 2811 IJ c
25. (2)
31. (4)
26. (4)
32. (2)
27. (2)
33. (4)
28. (2)
34. (4)
29.
35.
(2)
(4)
30. (3)
36. (3)
H 400 K 180 400
=
H 8000 K
37. (3) 38. (1) 39. (4) 40. (4) 41. (4) 42. (3) 4. (3) s = 16 cm.
43. (2) 44. (2) 45. (3) 46. (1) 47. (2) 48. (3)
O
49. (1) 50. (3)
T 50 cm.
Miscellaneous
A B
1. (4) 2. (2) 3. (2) 4. (2) 5. (4) 6. (1)
16 cm
7. (2) 8. (4) 9. (1) 10. (3) 11. (4) 12. (3)
13. (2) 14. (2) 15. (1) 16. (1) 17. (2) 18. (3) r = 50 cm.
19. (1) 20. (2) 21. (4) 22. (4) 23. (3) 24. (2) s 16 8 8 180
25. (2) 26. (1) 27. (2) 28. (1) 29. (1) 30. (3) T= Radian =
r 50 25 25

BLAM–167
f=kdks.kfefr
8 180 1008 18 i.e. 60° – 35° z 90° – 55°
= 7 = 18
25 22 55 55 50° – 40° z 90° – 50°
55° – 40° z 85° – 55°
FG 18 60'
IJ 18 20'
= 18°
H 55 K 8. (2) Octagon dk izR;sd angle =
1
( 2n 4) right angle
5. (2) ,d rotation esa angle = 2 8
1 12 90
55 55 = ( 2 8 – 4 ) 90 = 135
55 radians ds fy, total rotation = 8 8
2 22
2
7 180
 1 Radian
55 7 35
= rotation
2 22 4 3
135° = 135 = Radian
Required time 180 4

35 10
9. (3) cps gq, nksuksa angles dk sum
= 2.5 Second
4 3.5 4 5 4
= =
9 9
25qu 5
6. (1) T = 25° = Radian = Radian
180 36 1 4 2
Each angle =
2 9 9
B
12 10. (3) Hour hand }kjk 1 ?kaVs esa cuk;k x;k angle = 30°
40 m.
25°
A 12
O

s s 40 40 36
T= r=
r 5 5
36 1 9
2
4
;k 4 ?kaVs esa cuk angle
40 36 7
= m = 91.64 m 9 135
5 22
T 30
4 2
7. (1) Triangle osQ Angle Ÿ (a – d)°, a°, (a + d)°
? a d a a d 180 Minute hand }kjk 60 minute esa cuk angle = 360°
Ÿ 3a = 180° Ÿ a = 60° 15 minutes esa cuk angle

a d 60 60 1 360
? a 15 = 90°
d Radian 180 3 60
Required angle
60 d 1
Ÿ Ÿ 180° – 3d = 60° + d
60 d 3 135 45 1
– T 90 22
Ÿ 4d = 120° Ÿ d = 30° 2 2 2
? Angle of triangle Aliter :
a – d = 60° – 30° = 30° 60H 11M
a = 60° T=
2
a + d = 60 + 30 = 90°
Aliter : 60 2 11 15
=
option ls ge ns[k ldrs gSa 2
fliZQ 30°, 60°, 90° A.P. esa gS 120 165 45
D;ksafd buosQ chp dk varj leku gS = =
2 2
i.e. 60° – 30° = 90° – 60° = 30°
45 1
ckfd fn, gq, dks.kksa dk varj leku ugha gS = = 22
2 2

BLAM–168
f=kdks.kfefr
11. (2) ge tkurs gSa] 37.4 3
r=

O
37.4 3 7
r= = 1.7 × 21 = 35.7 cm
22

T 75 13. (3) Two angles = A and B tgk¡ A > B.


m

FG 135 IJ
c

cm
75

A + B = 135° =
H 180 K Radian

FG 3 IJ Radian
A 21 cm
B A+B=
H4K ...(i)

arc rFkk A – B = ...(ii)


Angle 12
radius Eq (i) rFkk (ii) dks add djus ij
21 3 9 10 5
T= 2A = =
75 4 12 12 12 6
5
F 7 IJ
T= G
R
A=
12
radian
H 25 K
5
12. (3) ;gk¡ 14. (3) Speed (l) = 90
18
25 m/s

distance moved in 11 sec = 25 × 11 = 275 m


A
Now,

l FG 275 IJ
60° =
r H 1500 K
r

= G
F 275 180 IJ
H 1500 K
B C
37.4 cm F I
T = 60°
GG 275 180 JJ F 21I
= G 1500 F 22 I J = G
l = 37.4 cm GH GH 7 JK JK H 2 JK = 10°301

r=?
ge tkurs gSa] 15. (4) C

1
FG IJ R

H 180 K
60
FG 60
IJ R
D
A
B
H 180 K AB = AC
AC = AD = AB
60 G J
F I R
ACB = ABC = x
H 3K DCA = ADC = x
We know that BCD esa]

l CDA + DCB + CBA = 180°


T 4x = 180°
r
2x = 90° = BCD
37.4
3 r 90° = 90 Radian
180 2

BLAM–169
f=kdks.kfefr
Type-II 2 4 4 2 2 2
cot T = = 2 1
Angles ij based questions 2 2

1. (2) sin 240° = sin(180° + 60°) or – e 2 j


1
= –sin 60°
7. (3) (1 + sec 20° + cot 70°) (1 – cosec 20° + tan 70°)
[  sin(180° + T) = – sinT]
= (1 + sec 20° + tan 20°) (1 – cosec 20° + cot 20°)
3 [  tan (90°– T) = cotT; cot (90° – T) = tan T]
= –
2 FG1 1 sin 20 IJ FG1 1 cos 20 IJ
7 FG IJ
= H cos 20 cos 20 K H sin 20 sin 20 K
= cos 2 –
2. (4) cos
4 4 H K 1 cos 20 sin 20 sin 20 1 cos 20
= ×
cos 20 sin 20
= cos [  cos (2 – T) = cos T]
4 (cos 20 sin 20 )2 1
=
1 sin 20 .cos 20
=
2 cos 2 20 sin 2 20 2 sin 20 .cos 20 1
= =2
sin 20 .cos 20
FG 5 IJ
FG 7 IJ
sin [  sin2 T + cos2 T = 1]
3. (3) tan
H6K H6K 8. (4) sin (45° + T) – cos (45° – T)
= sin (90° – (45° – T) – cos (45° – T)
F I F IJ
= tan GH – JK sin GH
6 6K [ cos(90qT) sin T]
[  tan (180° – T) = –tanT = cos (45° – T) – cos (45° – T) = 0
sin (180° + T) = –sinT] sin 39
9. (4) 2 tan 11 .tan 31 . tan 45°. tan 59° . tan 79°
FG – sin IJ cos51
6 H 6K
= – tan
– 3 (sin2 21° + sin2 69°)
sin 39
1 1 1 = 2 tan 11 .tan 31 .tan 45°.tan (90°–
= tan sin = = cos(90 39 )
6 6 3 2 2 3
31°).tan (90°–11°)– 3 [sin2 21° + sin2 (90° – 21°)]
4. (2) cosec (–1410°) = – cosec (1410°)
sin 39
[  cosec (– T) = – cosec T] = 2 tan 11 . tan 31 . tan 45°.cot 31°.cot 11°
sin 39
= –cosec (360° × 3 + 330°)
= –cosec (330°) – 3 (sin2 21° + cos2 21°)
= –cosec (360° – 30°) [  sin (90° – T)= cosT; cos (90° – T)
= sinT; tan (90° – T) = cotT)]
= cosec 30° = 2
5. (4) tan 330° = tan(360° – 30°) = –tan 30°  sin 2 cos2 1; tan .cot 1
=1+2–3=0
[  tan (360° – T) = –tan T]
10. (1) sin (2x – 20°) = cos (2y + 20°)
–1
= sin (2x – 20°) = sin (90°–2y – 20°) = sin (70° – 2y)
3
2x – 20° = 70° – 2y 2 (x + y) = 90° x + y = 45°
6. (3, 4) sin T + cos T = 2 cos (90° – T) sec (x + y) = sec 45° = 2
sinT + cos T = 2 sin T 11. (4) Given expression
= cot10°.cot 80°. cot20°. cot70°. cot 60°
[  cos (90° – T) = sin T]
= cot10°.tan10° . cot 20°. tan 20°. cot 60°
nksuksa rjiQ Square djus ij]
cos2T + sin2T + 2 cos T .sin T = 2 sin2T LM tan(90 ) cot ; tan .cot 1;cot 60
1 OP
cos T = sin T – 2 cos T. sin T
2 2
N 3 Q
sin2 T ls divide djus ij
1 1
cot2 T = 1 – 2 cot T cot2 T + 2 cot T – 1 = 0 = 1 .1.
3 3

BLAM–170
f=kdks.kfefr
12. (3) A = tan11° . tan 29° = sin2 5° + cos2 5° + sin2 25° + cos2 25° + sin2 45°
B = 2 cot 61° . cot 79° = 2 cot (90° – 29°) cot (90° –
1 1
11°) 1 1 2 2.5
2 2
= 2 tan 29° . tan 11° [  cot (90° – T) = tan T]
B=2A
19. (1) cot 18°
FG cot 72 .cos 2 22
1 IJ
13. (1) tan 15° . cot 75° + tan 75°. cot 15°
= tan 15°. cot (90° – 15°) + tan (90° – 15°). cot 15°
H tan 72 . sec 2 68 K
= tan2 15° + cot2 15° ....(i) cot 18
= cot 18°.cot 72°.cos222° +
tan 72 .sec 2 68
LM tan (90q ) cot OP
N cot (90q ) tan Q cot 18
= cot 18°.tan 18°.cos222° + cot 18 . cos268°
tan 15° = 2 3
[  cot 72° = tan (90°–18°) = tan 18°
1 2 3 blh izdkj tan 72° = cot 18°]
cot 15° = = 2 3
2 3 (2 3 )(2 3) = cos2 22°+ cos268° = cos222° + sin222° = 1

2 2 b
 tan 90q – T g b
cot T;sin 90q – T g cos T ;sin 2 T  cos2 T 1
tan2 15° + cot2 15° = 2 e 3 j e2 3 j
sin 25 cos 65 cos 25 sin 65
= 2 (4 + 3) = 14 20. (1)
tan 2 70 – cosec 2 20
[  (a – b)2 + (a + b)2 = 2(a2 +b2)]
14. (1) sec (7T + 28°) = cosec (30° – 3T) sin 25 cos(90 –25 ) cos 25 sin(90 –25 )
=
sec (7T + 28°) = sec (90°– (30°–3T)) tan 2 70 – cosec 2 (90 –70 )
7T + 28° = 90° – 30° + 3T LM sin(90° – T) = cos T; cos(90° – T) = sin T;OP
4T = 90° – 30° – 28° = 32° N cosec (90° – T) = sec T Q

32 sin 25 sin 25 cos 25 .cos 25


T= 8 =
4
tan 70 – sec 2 70
2

15. (3) tan


FG  T IJ 3 sin 2 25 cos2 25
H 2 2K = =
1
= –1
tan 70 – sec 2 70
2 –1
T
cot 3 cot 30q 30q Ÿ T 60q  sec 2 T – tan 2 T = 1;sin 2 T + cos2 T = 1
2 2
21. (1) Expression
1
cos T = cos 60° = 4
2
= cot2 + 3 cos2 150° – 4 cosec2 45° + 8 sin
3 6 2
3 tan 45 . tan 20 . tan 40

16. (3)
2 sin 68 2 cot 15

tan 50 tan 70 4 F 3 I 2

cos 22 5 tan 75 5 =
3
× ( 3 )2 + 3 × GH 2 JK – 4 ( 2 )2 + 8

cos 22 2 tan 75 3. tan 20 .cot 20 tan 40 .cot 40


= 2 – 4 3 3
cos 22 5 tan 75 5 = ×3+ –4×2+8
3 4
LMsin (90 ) cos ; OP 12 9 48 27
2 3 10 2 3 = + =
= 2 = 1 Mcot (90 ) tan ; PP 3 4 12
5 5 5 MNtan (90 ) cot Q 75 25
17. (2) tan10°.tan15°. tan75° . tan80° = =
12 4
= (tan10°. tan80°)(tan15°.tan75°)
= (tan10°. cot10°)(tan15°.cot15°) = 1 × 1 = 1 4 sinA (cos3 A) 4 cosA sin 3 A
[tan(90° – T) = cotT; tanT.cotT = 1] 22. (2) Expression =
cos (90 4A)
18. (3) (sin2 5° + sin2 85°) + (sin2 25° + sin2 65°) + sin2 45°
= sin2 5°+ sin2 (90°–5°) + sin2 25° + sin2 (90° – 25°) + 4sinA . cosA (cos2 A – sin 2 A)
=
sin2 45° – sin 4A

BLAM–171
f=kdks.kfefr
= – cos (x + y – x) = – cos y
2. sin 2A . cos 2A sin 4A
= = = –1
sin 4A sin 4A  cos(A B) cosA.cosB – sinA.sinB

LMcos 2A cos A  sin A OP 2 2 26. (3) cos 15°– cos 165°


= cos 15° – cos (180° – 15°)
Nsin A 2 sin A cos A Q = cos 15° + cos 15° (  cos (180° – T) = – cosT)
F
23. (2) Expression = cos G 90
I F 9I = 2 cos 15°
H 2 JK . cos GH 90 2 JK = 2 cos (45° – 30°)

= 2 cos 45q .cos 30q sin 45q sin 30q


F 3 IJ . sin FG 90 13 IJ
+ sin G 90
H 2K H 2 K [  cos(A – B] = cos A cosB + sin A sin B]

9 3 13
= 2
LM 1 u 3

1
u
1 OP
= sin . sin + cos . cos
2 2 2 2 MN 2 2 2 2 PQ
F 9 I e 3 1 j= 3 1
1 GG
2 sin .sin
2 2
2 cos
JJ =2
= 2 13 2 2 2
GH.cos
3 JK
2 2 27. (1) ;gk¡, P + Q + R = 60°
Let P = 0°, Q = 30° rFkk R = 30°
1
= (cos4T – cos5T + cos8T+ cos5T) cosQ cosR(cos P – sin P) + sinQ sinR (sinp – cosP)
2
cos 30° cos30° (cos0° – sin0°) + sin30° sin30° (sin0°–
[ 2sinA . sinB = cos (A – B) – cos (A + B); 2 cosA cos0°)
cosB = cos (A + B) + cos (A – B)]
3 3 1 1
=
1
(cos8T + cos4T)
=
2
u
2
b g
1  0  u 0 1
2 2
b g
2
3 1
1 8 4 8 –4 = 
= × 2 cos . cos 4 4
2 2 2
= 2 cos6T. cos2T 2 1
= =
4 2
LM cosC cosD 2cos
C D
.cos
C D OP
N 2 2 Q sin(180 – )sin(90 – ) – cot(90 – )
28. (4)
sin(90q –10 ) – cos( – 6 ) 1 tan 2

24. (2) cos FG – 10 IJ – sinb –6 g sin cos – tan


H2 K =
sec 2
cos10  cos 6
= sin
sin 10 – sin 6 sin cos 
= cos
 sin b – g sin;cos b – g – cos 2 sec 2

10  6
2 cos .cos
10 – 6
=
dsin cos 2  sin i u cos 2

2 2 cos
= 10  6 10 – 6
2 cos .sin = sin T (cos2T – 1) × cos T
2 2
= – sin T sin2T × cos2 T
2 cos 8 cos 2 = – sin3T cos2T
= cot 2T 29. (3) sin (630° + A) + cos A
2 cos 8 sin 2
= sin[360° + (270° + A)]+cos A
25. (1) Expression
= sin (270° + A) + cos A
= sin (90° – x) cos [ – (x – y)] + cos (90° – x) . sin [ –
(y – x)] = – cos A + cos A
= cosx (– cos (x – y)) + sinx . sin (y – x) =0
= cos x. cos (y – x) + sinx . sin (y – x) LM sin(360q ) sin OP
[ cos (– ) = cos ] Nsin(270q )  cos Q
BLAM–172
f=kdks.kfefr
30. (1) We know that
1 1
If 1 + 2 = 90° then sin 1. sin =1 x×
2 2 4
A/q
sin . sec (30° + ) = 1
x=
1 LMcos 2
A
1 tan 2 A OP
+ (30° + ) = 90°
2 = 60°
2 N 1 tan 2 A Q
37. (1) Given
= 30°
31. (1) cot . tan (90° – ) – sec (90° – ). cosec + (sin2 1 2 sin 60 .cos 60 1 2 sin 60 .cos 60
25° + sin2 65°) +
sin 60 cos 60 sin 60 cos 60
3 (tan 5°: tan15°. tan 30°. tan 75°.
tan 85°)
3 1 3 1
= (cot . cot – cosec . cosec ) + 1 + 1 2 1 2
2 2 2 2
FG1 1
1IJ = 3 1 3 1
3 H 3K 2 2 2 2

= –(cosec2 – cot2 )+1+1


2 3 2 3
= –1 + 1 + 1 = 1
2 2 2 3 2 3
F 1 IJ
–4 × G
2 = 3 1 3 1 = 3 1 3 1
32. (4) (1)2 + (1)2
H 2K =2–2=0 2 2

33. (2) Given 2 3 2 3 3 2 3 2 3 3


(sin2 1° + sin2 89°) + (sin2 11° + sin2 79°) 2

+.....+ (sin2 41° + sin2 49°) + sin2 45°


=
e 3 j b1g 2

1 1
=1+1+1+1+1+1+ =5 2 3
2 2 = 3.
2
34. (1) Given
38. (3) We know that If 1
+ 2
= 90° then sin 1
= cos 2
(1 – sin2 ) (1 – cos2 ) (1 + cos2 ) (1 + tan2 )
A/q
2 2 2 2
= cos . sin . cosec . sec
=1×1=1 sin (60° – ) = cos b 30 g
(60° – ) + b 30 g = 90°
[Note:– If 1 + 2 = 90° then sin 1
sec 2
= 1 and cos
1
.cosec 2 = 1 ]
30° + b g = 90°
5 tan
1 4 60
sin .cos cos .sec
35. (1) 2 6 4 3 6 12 sin
2 tan b g = tan 60° = 3.
39. (4) Given, tan 20° = K
FG 1 1 1IJ FG 1 2IJ 5 1 A/q
=
H 2 2 2K H 3 3K 12 1
tan 250 tan 340
tan 200 tan 110
1 2 5
= 0
4 3 12 b
tan 270 20 g b
tan 360 20 g
x x tan 30 2
=
b
tan 180 20 g b
tan 90 20 g
36. (3) Given
1 tan 2 30
1
= sin2 30° + 4 cot2 45° – sec2 60° K
cot 20 – tan 20 K 1 K2
= = 1 1 K2
x1 d tan 2 30 h tan 20 cot 20 K+
K
= sin2 30° + 4 cot2 45° – sec2 60°
d1 tan 30 2
h 40. (3) cos 24° + cos 25° + cos 155° + cos 204°
= cos 24° + cos 25° + cos (180° – 25°) + cos (180° + 24°)
1
x × cos (2 × 30°) = 4 4 = cos 24° + cos 25° – cos 25° – cos 24°
4 = 0

BLAM–173
f=kdks.kfefr
41. (2) For = 30°, sin + cos
4 3
3 3 4 2 8 1
1 3 1 3 3 4
=
2 2 2 9 25
4 8 8
For = 45°, sin + cos 4 4
1 1 1 1 2 48. (4) sin (B – C) cos (A – D) + sin (A – D) + sin (C – A). cos
= 2 (B – D)
2 2 2 2
Put A = B = C = D = 0
For = 60° sin + cos sin 0°. cos 0° + sin0° cos 0° + sin 0° cos 0°
= 0+0+0=0
3 1 3 1 49. (2)
=
2 2 2
For = 90° b
4 cos 90 g
A .sin 3 90 b A gb 4 sin 90 g
A .cos3 90 b A g
F
cosG
180 + 8A I
H 2 JK
sin + cos + 1 + 0 = 1
42. (1) sin 60° sin 120° + cos 240° cos 300°
= sin 60° sin 60° – cos 60°.cos 60°
4 sin A.cos3 A 4 cos A. sin 3 A
F 3 I F 3 I FG 1 IJ FG 1 IJ 3 1 2 1
= cos 90 + 4A b g
= GH 2 JK GH 2 JK H 2 K H 2 K =
4 4 4 2
4 sin A.cos3 A 4 cos A .sin 3 A
43. (1) Given cot 20° = P =
sin 4 A
tan 160 tan 110
1 + tan160 tan110
4 sin A.cos A cos 2 A sin 2 A
=
tan 180 20 b tan 90 20 g b g sin 4 A
= 1
b
tan 180 20 tan 90 20 g b g 4 sin A.cos A cos2 A
=
2 sin 2 A.cos2 A
1
tan 20 cot 20 P
P P2 1 4 sin A.cos A
= 1 b tan 20 cot 20 gb g =
1
1
P 2P = -2 2sin A.cos A
1
P
50. (2) Given,
44. (1) cos2 5° + cos2 10° + cos2 15° +......+ cos2 90°
= (cos2 5° + cos2 85°) + (cos2 10° + cos2 80°) + ....+(cos2 cos
LM180 OP.cosLM180 9 OP sin
LM180 3 OP sin LM180 13 OP
40° + cos2 50°) + cos2 45° + cos2 90° N 2 Q N 2 Q N 2 Q N 2 Q
1
0 = 8
1
= cos 90
FG IJ.cosFG 90 9 IJ FG
sin 90
3 IJ sin FGH 90 13 IJ
K
= (1 + 1 +........ 8 times) +
2 2 H 2 K H 2 K H 2 K 2

45. (1) sin (75° + 15°) sin (75° – 15°) 9 3 13


= sin 90° sin 60° = sin .sin cos .cos
2 2 2 2
Multiplying & dividing by 2
3 3
= 1
2 2
=
1 LM
2 sin .sin
9
2 cos .cos
13 OP
46. (4) tan 225° = tan (100 + 125) 2 N 2 2 2 2 Q
1
1
tan 100 tan 125
1 tan 100 tan 125
=
2
bcos 4 cos 5 g bcos 5 cos 8 g

tan 100° + tan 125° + tan 100°, tan 125° = 1 1


= cos 8 cos 4
47. (1) 2
4
cot 2
FG IJ 3 cos 2 150b g 4cosec 2 45 8 sin
FG IJ 1 LM 8 4 8 4 OP
3 H K
6 H 2K =
2
2.cos
N 2
.cos
2 Q
4 1
cot b30 g 2
b
3 cos2 90 60 g 4cosec 2 45 8 sin = 2 cos 6 .cos 2 cos 2 .cos 6 .
3 2 2

BLAM–174
f=kdks.kfefr
2 2 2 2 2
51. (2) 2 cosec 23° × cot 67°–sin 23°–sin 67°–cot 67° = cos 24° + cos 55° – cos 55° – cos 24° + cos 60°
= 2 . cosec2 (90° – 67°) × cot2 67° – sin2 23° – sin2 (90°
1
– 23°) – cot2 67° = cos 60° =
= 2 sec2 67° × cot2 67° – sin2 23° – cos2 23° – cot2 67°
2
= 2 cosec2 67° – 1 – cot2 67°
cos2 45 cos2 60 tan 2 30 sin 2 30
[sin2 23° + cos2 23° = 1] 56. (2) + 2 – 2 –
2
= cosec 67°+cosec 67°–1– cot2 67° = cosec267°+1–1
2 2 sin 60 sin 45 cot 45 cot 2 30

= cosec2 67° = sec2 23°


LM sec (90 ) cosec ; OP FG 1 IJ 2
FG 1 IJ 2
FG 1 IJ 2
FG 1 IJ
2

N sin (90 ) cos Q =


H 2K +
H 2K –
H 3K –
H 2K
3 F 3I 2
FG 1 IJ 2
b1g 2
e 3j
2

52. (1) sin2 30° cos2 45° + 5 tan230° +


2
sin2 90° – 3 cos2 GH 2 JK H 2K
90°
1 4 1 1 1
F I F 1 IJ
1
= G J × G
2 2
F 1 IJ
+5× G
2
3
=
2
×
3
+
4
×2–
3
×1–
4 3
H K H 2K
2 H 3K +
2
×1– 3×0
2 1 1 1 8 6 – 4 –1 9 3
= + – – = = =
3 2 3 12 12 12 4
1 1 1 3 1 5 3
= × +5× + = + + 57. (1) Given
4 2 3 2 8 3 2
cosec 39° = x
3 40 36 79 7 A/q
= = =3
24 24 24
53. (1) Expression FG 1 sin 2 39
IJ tan 2 51
1
H cosec 512
K sin 2 51 .sec 2 39
1 5 tan
S S 4
= sin . cos – cot . sec + 2 1
2 6 4 3 6 = (sin2 51° + sin2 39°) + tan 51
12 sin sin 2 51 .sec 2 39
2
= 1 + tan251° – 1
1 1 1 2 = tan2 51° = cot2 39°
1 5 1
= × × – × + = cosec2 39° – 1 = x2 – 1
2 2 2 3 3 12 1
[we know that If 1 + 2 = 90° then
1 2 5 3 8 5 sin2 1 + sin2 2 = sin 1 .sec 2 = 1]
= – + = =0 58. (2) Given the expression
4 3 12 12

3 4 1 tan A tan 3 . tan 15 . tan 30 . tan 75 . tan 87


54. (3) x sin260° – sec 60°. tan230° + sin245° . 1 tan A tan 27 . tan 39 . tan 51 . tan 60 . tan 63
2 5
tan2 60° = 0
1
1 1
1 tan A 1
F 3I 2
3 FG 1 IJ 2
4 FG 1 IJ 2 3
x .G
H 2 JK
2 1 tan A 1 1 3 3
– × 2.
2 H 3K + ×
5 H 2K ×( 3 ) = 0
[we know that + = 90° tan tan = 1]
1 2 1 2

3x 3 1 4 1 1 tan A 1
– ×2× + × ×3=0
4 2 3 5 2 1 tan A 3

3x 6 3x 6 3 – 3 tan A = 1 + tan A
1 + =0 =1– 4 tan A = 3
4 5 4 5
1
5 6 1 1 4 4 tan A = cot A 2
= = x=– × = 2
5 5 5 3 15
59. (3) sin 3x = cos (3x – 45°)
55. (1) Expression 3x + (3x – 45°) = 90°
= cos 24° + cos 55° + cos 125° + cos 204° + cos 300° 6x = 45°
= cos 24° + cos 55° + cos (180° – 55°) + cos (180° +
45
24°) + cos (360° – 60°) 3x = 22.5
2

BLAM–175
f=kdks.kfefr
Aliter :
Type-III Let x + y = 60°
Trigonometric identities ij x – y = 30° [  tan60° × tan30° = 1]
based questions on adding
2x = 90°
1. (4) p sin T = 3
x = 45°
p cos T = 1
tan
FG 2x IJ = tan
FG 2 45 IJ = tan30°=
1
Adding after squaring H 3K H 3 K 3
p2 sin2T + p2 cos2T = 3 + 1
6. (2) sin (A + B) = 1 = sin 90°
p2 (sin2T + cos2T) = 4 A + B = 90° ...(i)
p2 = 4 [  sin2 T + cos2 T = 1]
3
p=±2 cos (A – B) = cos 30° A – B = 30° ...(ii)
2
2. (2) sin (x + y) = cos [3 (x + y)]
sin (x + y) = sin [90° – (3x + 3y)] (i) + (ii) djus ij]
[  sin (90° – T) = cos T] A + B + A – B = 90°+ 30° = 120°
2 A = 120° A =60°
x + y = 90° – 3x – 3y
B = 30°
4x + 4y = 90° 2x + 2y = 45°
tan (2x + 2y) = tan 45° = 1 7. (2) sec T + tan T = 3
3. (3) 2 sin + 15 cos2 =7
1 sin
2 sin + 15 (1 – sin2 ) = 7[  cos2 =1 – sin2 ] 3 1 + sin T = 3 cos T
cos cos
2 sin + 15 – 15 sin2 )=7
15 sin2 – 2 sin –8=0 nksuksa rjiQ square djus ij]
15 sin 2
– 12 sin + 10 sin –8=0 1 + sin2T + 2 sin T = 3 cos2T = 3 (1 – sin2T)
3 sin (5 sin – 4) + 2 ( 5 sin – 4) = 0 1 + sin2T + 2 sin T = 3 – 3 sin2T
(3 sin + 2) (5 sin – 4) = 0 4 sin2T + 2 sin T – 2 = 0
2 sin2T + sin T – 1 = 0
4 2
sin = D;ksafd sin – 2 sin2T + 2 sin T – sin T – 1 = 0
5 3 2 sinT (sin T + 1) – 1 (sin T + 1) = 0
(sin T + 1) (2sin T – 1) = 0
1
16 3 LM cos 1 sin 2 OP
cos =
25 5 N Q sin T =
1
D;ksafd sin T –1
2
3 Aliter :
;k trigonometric triplets (3, 4, 5) ds }kjk cos =
5
secT + tanT = 3
3 1
cos 3 secT – tanT =
cot = 5 3
sin 4 4 on adding
5
1
4. (4) Given, 2 secT = 3
3
L2
sin 2 M
3 4
..... x
x 1 OP 1 secT
4 1
N1 2 3 x 2 Q =
3 2
sin 2 (x – 1) = sin 90°
2
2x – 2 = 90° secT =
3
x = 46°
5. (1) tan (x + y). tan (x – y) = 1 3
cosT =
tan (x + y) = cot (x – y) = tan (90° – x + y) 2
x + y = 90° – x + y 2x = 90°

2x sinT
F 3I 2
1
tan
3
= tan 30° =
1 = 1 GH 2 JK =
2
3

BLAM–176
f=kdks.kfefr
3 tan2 + 7 = 4 + 4 tan2
8. (3)
tan2 = 3
tan = 3
a
b
1
11. (3) tan 2T = cot 4
tan 4
tan 2T = tan (90° – 4T) [  tan (90°– ) = cot ]
a –b 2 2 2T = 90° – 4T
6T = 90° T = 15°
b tan 3T = tan 45° = 1
Given, sin =
a Aliter :
ekuk 2T = 30°
a b a b
T = 15° [  tan 30° × tan 60° = 1]
a b a b
tan 3T = tan 3 × 15°
a b a b = tan45°
= = 1
a2 b2
sin
2 12. (1) =1
=
2a
2 2 F
2
I =
cos
cos 30b g
a b a 2
b 2

GH a
JK sin sin
=1 =1
b
sin 90 30 g sin 60b g
9. (4) tan (T1 + T2) = 3 = tan 60°
sin = sin (60° – )
T1 + T2 = 60° and sec (T1 – T2)
= 60° – 2 = 60° = 30°
2
= = sec 30° 1 1
3 sin + cos 2 = sin 30° + cos 60° = =1
2 2
T1 – T2 = 30°
T1 = 45° and T2 = 15° 7
sin 2T1 + tan 3T2 13. (1) sin T – cos T = .....(i)
13
= sin 90° + tan 45°
sin T + cos T = x .....(ii)
=1+1=2
10. (3) 3 sin2 + 7 (1 – sin2 ) = 4
nksuksa equations dks square rFkk add djus ij]
3 sin2 + 7 – 7 sin2 = 4
7 – 4 sin2 = 4 49
2 (sin 2 cos2 ) = 169 x2
4 4 sin2 = 3

3 49 338 49
sin x2 2– [  sin2 T + cos2 T = 1]
2 169 169

3 1 289 17
cos 1 sin 2 1 = = x
4 2 169 13
14. (2) tan (2T + 45°) = cot 3T = tan (90° – 3T)
sin
tan 3 2T + 45° = 90° – 3T 5T = 90° – 45° = 45°
cos
T = 9°
Aliter :
15. (4) Given
3 sin2 + 7cos2 =4
nksuksa rjiQ cos2 ls divide djus ij 1 sin x 1 sin x
P=
2 2 1 sin x 1 sin x
3 sin 7 cos 4
cos2 cos2 cos2
3 tan2 + 7 = 4sec2 =
b1 sin x g
2
1 sin x
q
3 tan2 + 7 = 4( 1 + tan2 ) cos 2 x cos x

BLAM–177
f=kdks.kfefr
again, = cot 9°.cot 27°.cot 45°. cot 63°. cot 81° [  c = 180°]
1 sin x 1 sin x = cot 9°.cot 27°.cot 45°. cot (90° – 27°). cot (90° – 9°)
q=
cos 1 sin x = cot 9°.cot 27°.cot 45°. tan 27°. tan 9°
[cot (90° – T) = tan T]
cos2 cos = (cot 9°.tan9°).(cot 27°. tan 27°).cot 45° = 1
r
= cos 1 sin x
b g 1 sin x [  tan T. cot T = 1]
P = q = r. 20. (4) 3 cos 80°. cosec 10° + 2 cos 59° . cosec 31°
16. (3) Given, = 3cos(90°–10°). cosec 10° + 2 cos(90°– 31°) . cosec 31°
= 3 sin 10°. cosec 10° + 2 sin 31°. cosec 31°
b
2 1 sin g cos2 b
2 1 sin g cos2 =3+2=5
4
cos [  cos (90°– T) = sinT ; sinT . cosecT = 1]

h b1 g b1 g 2 sin T • cos
• T
=
d
41 sin 2
2cos 2
sin 2cos 2
sin 4
cos 21. (1)
cos T + sin T
=1
4
cos
Numerator rFkk Denominator dks sin T ls divide djus ij
=
4 cos2 2 cos2 1 sin b 1 sin g cos 4
2 cot
cos4 1
cot 1
4 cos 2 4 cos2 cos 4 2 – cot T = cot T + 1
= 1
cos 4 2 cot T = 1

A B C 1
cot
17. (3) A + B + C = 2
2 2 2
tan T cot T
sin
FG A B IJ sin
FG C IJ cos
C 22. (4) Expression = +
• T 1 • tan
1• cot • T
H 2 K H2 2 K 2
1
blh izdkj] tan tan 2 1
tan
= =
cosG
F A B IJ sin
C 1
1 1 tan tan 1 tan (1 tan )
H 2 K 2 tan

F A B IJ
cot G tan
C tan 2 1 tan 3 1
H 2 K 2
=
tan 1 tan (tan 1)
=
tan (tan 1)

tan G
F A B IJ cot
C (tan 1)(tan 2 tan 1)
H 2 K 2 =
tan (tan 1)
18. (3) tan T. tan2T = 1
tan 2 tan 1
1 =
tan T = = cot 2T tan T = tan (90° – 2T) tan
tan 2T
T = 90° – 2T 3T = 90° T = 30° = tan T + cot T + 1
sin2 2 T + tan2 2T = sin2 60° + tan260° 23. (2) (1 + tan T + sec T) (1 + cot T – cosec T)

F 3I + 3 2
= 1
FG sin 1 IJ FG1 cos 1 IJ
2 3 3 H KH K
H 2 JK d i
cos cos sin sin
= G = +3 = 3
4 4
cos sin 1 sin cos 1 (sin cos )2 1
Aliter : = . =
cos sin sin .cos
T = 30°
sin260° + tan260° sin 2 cos 2 2 sin .cos 1
=
sin .cos
F 3I 3 2
2 3 15 3
= GH 2 JK e j =
4
3 =
4
=3
4 1 2 sin .cos 1
=
sin .cos
3 5 7 9
19. (4) cot .cot .cot .cot .cot 2 sin .cos
20 20 20 20 20 = 2
sin .cos

BLAM–178
f=kdks.kfefr
24. (3) (sec A – cosA) + (cosec A – sinA) – (cot A – tan A)2
2 2
29. (1) tanT = 1 T = 45°
= sec2A + cos2A – 2secA cosA + cosec2A + sin2A–
2cosec A. sinA – cot2A – tan2A + 2 cotA. tanA 1 5
8
= (sec2A – tan2A) + (cos2A+ sin2A) + (cosec2A – cot2A) – 2 8 sin 5 cos 2 2
=1+1+1–2=3– 2=1 sin 3 2 cos3 7 cos 1 2 7
2 2 2 2 2
 secA.cosA 1;sinA.cosec A 1; tanA.cotA 1etc.
25. (2) cos T + cos T = 1
2 4
cos T = 1 – cos T = sin2T
4 2
13
2 2 13 2 2
sin =2
cos2 2
tan2T = cos2T 13 2 13
cos
2 2
tan T + tan4T = cos2T + cos4T = 1
2

1
4 30. (2) sinT + cosecT = 2 sinT + =2
26. (1) 5 tanT = 4 tanT = sin
5
sin2T – 2 sinT + 1 = 0 (sinT – 1)2 = 0
5 sin 3 cos sinT = 1 cosecT = 1
5 sin 3 cos cos sin100T + cosec100T = 1+ 1 = 2
=
5 sin 3 cos 5 sin 3 cos Aliter :
cos ;fn sin T + cosec T = 2 gks rks] sinp T + cosecp T = 2 gksxk
[Nr rFkk Dr dks cos T ls divide djus ij] sin100 T + cosec100 T = 2
31. (4) un = cosn + sinn
4 u6 = cos6 + sin6
5 3 4 3 1
5 tan 3 5
= = (cos2 )3 + (sin2 )3
5 tan 3 4 4 3 7
5 3 = (cos2 + sin2 )3 – 3 cos2 . sin2 (sin2 + cos2 )
5
[ a 3 b3 (a b )3 3ab (a b )]
sin cos 5
27. (3) 2 2
= 1 – 3 cos . sin
sin cos 4
u4 = cos4 + sin4

cos
FG sin 1
IJ (cos2 )2 + (sin2 )2 = (cos2 + sin2 )2 – 2 cos2 . sin2
H cos K 5 tan 1 5 = 1 – 2 cos2 . sin2

cos
FG sin I
1J
4 tan 1 4 2u6 – 3 u4 + 1 = 2 (1 – 3 sin2 cos2 ) – 3
H cos K (1 – 2 sin2 . cos2 )+1=2–3+1=0
32. (1) tanT + cotT = 5
2 tan 5 4 nksuksa rjiQ square djus ij]
2 5 4 (tanT + cotT)2 = 52
(By componendo and dividendo) tan2T + cot2T + 2 tanT.cotT = 25
tan T = 9
tan2T + cot2T = 25 – 2 = 23 [  tanT.cotT = 1]
2
tan 1 81 1 82 41 33. (1) tan + cot =2
=
tan 2 1 81 1 80 40
nksuksa rjiQ square djus ij (tan cot )2 = 4
sin cos tan2 + cot2 + 2tan . cot =4
28. (3) 3
sin cos
tan2 + cot2 =4–2=2 [tan .cot = 1]
sinT + cosT = 3sinT - 3 cosT
Aliter-1 :
4cosT = 2 sinT tanT = 2
;fn tan T + cot T = 2 gks rks] tanp T + cotp T = 2 gksxk
sin4T – cos4T = (sin2T + cos2T) (sin2T – cos2T)
tan2 T + cot2 T = 2
2 Aliter-2 :
tan 1
= sin2T – cos2T = cos2T (tan2 T – 1) = tan T + cot T = 2
sec 2
T = 45°
tan 2 1 4 1 3 tan2 T + cot2 T = tan245° + cot2 45°
1 tan 2
1 4 5 = 1+ 1
=2

BLAM–179
f=kdks.kfefr
34. (3) Given, = sec2x + cos2x – 2 = 7 – 2 = 5
[equation (i) ls ]
FG 1 cos IJ 39. (3) sin T + sin2T + sin3T = 1
2 tan A
2
H sin K sin T + sin3T = 1 – sin2T sin T (1 + sin2T) = cos2T
1 tan 2 A
1 G
F 1 cos IJ 2

square djus ij]


H sin K (sin2 T) (1 + sin2T)2 = cos4T
(1– cos2T) [1 + (1– cos2T)]2 = cos4T
b
2 1 cos B g sin 2 B
(1 – cos2T) (2 – cos2T))2 = cos4T
d
= sin B sin 2 B - 1 - cos2 B 2 cos B h (1 – cos2T) (4 – 4 cos2T cos4T) = cos4T
4 – 4 cos2T + cos4T – 4 cos2T + 4 cos4T – cos6T
2b1 cos B g sin B
= = cos4T
sin 2 B 1 cos2 B 2 cos B – cos6T + 4cos4T – 8 cos2T + 4 = 0
cos6T – 4cos4T + 8cos2T = 4
=
b
2 1 cos B sin B g 40. (2) cos2 + cos2 = 2
sin 2 B – sin 2B - cos2B – cos2B + 2cos B
1 – sin2 + 1 – sin2 = 2 sin2 + sin2 = 0

=
b
2 1 cos B sin B g sin = sin = 0
tan3 + sin5 = 0
= =0

–2cos2B 2 cos B
41. (1) sec2T – tan2T = 1
b
2 1 cos B sin B g sin B
sec2T + tan2T =
7
b
= 2cos B 1 – cos B
g =
cos B
= tan B.
12
35. (1) cos A = 1 – cos2 A = sin2 A sec4T – tan4T
sin2 A + sin4A = sin2A + cos2A = 1 7 7
36. (3) Given the expression = (sec2T – tan2T) (sec2T + tan2T) 1
12 12
LM 1 1 OP LM 1 1 OP 42. (1) sec2T + tan2T = 7
N cos cot Q N cos cot Q 1 + tan2T + tan2T = 7 2 tan2T = 7 – 1 = 6
= [sec + tan ] [sec – tan ] tan2T = 3 tanT = 3 T = 60°
= sec2 – tan2 = 1
43. (4) tan2 A + cot2A – sec2A . cosec2A

37. (4) 2 sin


LM x OP x 2 1 = tan2A + cot2A – (1 + tan2A) (1 + cot2A)
N2Q x2 = tan2A + cot2A – (1 + tan2A + cot2A + cot2A.tan2A)
= tan2A + cot2A – 1 – tan2A – cot2A – cot2A . tan2A
L 1OP n
2 sin M
2 1
x 1 let =–1–1=–2 [ tanA . cotA = 1]
N2 Q n2
44. (1) sec tan 3 ...(i)
1
2 × 1 = x2 + 2
x  sec2 tan 2 1
(sec tan )(sec tan ) 1
F
2 = Gx
1 IJ 2

2
H x K sec tan
1
...(ii)
3
1
x– 0 subtract djus ij / (i–ii) djus ij
x
38. (1) sec x + cos x = 3 1
sec tan sec tan = 3
nksuksa rjiQ square djus ij 3
sec2x + cos2x + 2 . sec x . cos x = 9
sec2x + cos2x = 9 – 2 = 7 ...(i) 3 1 1
2 tan tan tan 30
[ sec x .cos x 1] 3 3
Now, T = 30°
tan2x – sin2x = sec2x – 1 – (1 – cos2x)
tan 3 tan 90 = infinity/ f
[ sec 2 x tan 2 x 1]

BLAM–180
f=kdks.kfefr
47. (1) Here, Given
b g b g b
1  2 cot 2 90q x  2cosec 90q x cot 90q x g n
45. (4)
cosec b 90q x g  cot b90q x g = cos . cos
a

1  2 tan 2 x  2 sec x tan x y


= = cos . sin
bsec x  tan x g b

z
sec 2 x  tan 2 x  2 tan 2 x  2 sec x tan x = tan
c
=
(sec x  tan x ) A/q

[1 sec 2
 tan 2 ] x2 y2 z2
a2 b2 c2
sec 2 x tan 2 x 2 sec x tan x = sec2 . cos2 + sec2 .sin2 – tan2
=
bsec x tan x g = sec2 . (cos2 + sin2 ) – tan2
= sec2 – tan2 = 1.

=
bsec x  tan x g 2 48. (1) Given,
x cos – sin = 1
bsec x  tan x g Putting the value, x =1, = 0°
[  a2 + b2 –2ab = (a – b)2] Now, A/q
= secx – tanx x2 – (1 + x2) sin
= 12 – (1 + 12) sin 0°
46. (3) tanT + secT = ( x – 2) ...(1)
=1–2×0=1
(x 2)
49. (4) Given the expression
ge tkurs gSa] 6 sin4 + 3 cos4 = 2
sec2T – tan2T = 1 6 (1 – cos2 )2 + 3 cos4 = 2
(secT – tanT) (secT + tanT)= 1 6 (1 + cos4 – 2 cos2 ) + 3 cos4 = 2
6 + 6 cos4 – 12 cos2 + 3 cos4 – 2 = 0
1
(secT – tanT) = 9 cos4 – 12 cos2 + 4 = 0
sec tan
(3 cos2 – 2)2 = 0
1 3 cos2 = 2
= x –2
2 2
x 2 cos2 = cos =
3 3
x 2
secT – tanT = ...(2)
x –2
equation (1) rFkk (2) dks add djus ij
2 2 3
x –2 x 2 e 3 j –- e 2 j
2 secT = =1
x 2 x –2

( x – 2)2 (x 2)2
= 2 2
(x – 4)
Now,
2( x 2 4) (7 cosec6 + 8 sec6 )1/3
2 secT = 2
( x – 4) 1
LMF F 3 I IJ OP 2 3

MMNGGH 7 e 3 j
6

secT =
(x 2 4) = 8 GH 2 JK JK P =6
2
( x – 4) PQ
50. (1) Given the expression
x2 – 4
cosT = cos cos
x2 4 4
1 sin 1 sin

BLAM–181
f=kdks.kfefr
56. (1) Given the equation
cos 1 sin 1 sin
2
4 sec2 x – 2 sec x – sec x + 2 = 0
1 sin
sec x (sec x – 2) –1 (sec x – 2) = 0
2 cos (sec x – 2) (sec x – 1) = 0
4
cos 2 (sec x = 2 and 1)

1 If sec x = 2 If sec x = 1
cos cos 60
2 x = 60° x = 0° (not possible for O < x < 90°)
60°
51. (1) (sin + cosec )2 + (cos + sec )2 = K + tan2 + x = 60°
cot2 57. (4) If cosec + cot =P
putting = 45°
2P
FG 1 2J
I FG 1 2
I
2J
2

K 1 1
then sin =
P2 1
H 2 K H 2 K Here, P = 2

2 2 4
F 3 IJ 2
K+2= G
2
sin =
22 1
=
5
.
H 2K 58. (2) We know that
K+2 =9 K =7 If sin – cos =a
52. (4) Given the expression
then sin + cos = 2 a2
sec tan 209
Now,
sec tan 79
(By using Componendo & dividendo) 1
Here, a =
29
sin 209 79 288
tan 209 79 130
2
FG 1 IJ 2

1 144
sin
65 sin + cos =
H 29 K
sin 65 144
53. (4) we know that 1 1681 41
= 2 = .
P2 1 841 841 29
If sec + tan =P sec =
2P 59. (2) Given
Here P = 3 sin (60° + ) – cos (30° + )
Now, = sin (60° + ) – sin {90° – (30° + )}
= sin (60° + ) – sin (60° + )
sec =
b3g 2
1 5
= 0
2 3 3
54. (2) Given 60. (1)
3 sin x + 4 cos x = 2
17
3 cos x – 4 sin x = b3g b4g b2g
2 2 2
2

= 9 16 4 = 21
Note : We know that
If a sin x + b cos x = c 13

then a cos x – b sin x = a2 b2 c2 Given,


55. (3) Given the expression, 2
sin + sin2 = 1 tan =
13
sin = cos2
Now,
sin2 = cos4 = [squaring Both sides]
1 – cos2 = cos4 cosec 2 + 2 sec 2
cos4 = cos2 = 1 cosec 2 3 sec 2

BLAM–182
f=kdks.kfefr
4. (4) 2 sin2 T + 3 cos2 T = 2 sin2 T + 2cos2T + cos2T
F 17 I 2
F 17 I 2

= 2 (sin2T + cos2T) + cos2T = 2 + cos2T


GH 2 JK 2G
H J
13 K
17 34
Minimum value = 2 + 0 = 2 [  cos2 T > 0]
= = 4 13 21
F 17 I
2
F 17 I
2
17 51 5. (4) sin T = cos (90° – T);
GH 2 JK
3G
H J
13 K
4 3 sin (90° – T) = cos T
sin 85° = sin (90° – 5°) = cos 5°
61. (2) Given,
(sin2 5° + sin2 85°) + (sin2 10° + sin2 80°) + ..... to
sin x cos x 6 8 terms + sin2 45° + sin2 90°
sin x cos x 5
1 1
6 sin x – 6 cos x = 5 sin x + 5 cos x = 8 1 1 9
2 2
sin x = 11 cos x
6. (3) oqQy terms = 45
sin x
11 tan x 11  tn = a + (n – 1) d ]
cos x
Now, 88
89 = 1 + (n – 1) × 2 n 1 n = 45
2
tan x 2
1 b11g
2
1 122 61
. vc] (sin2 1° + sin2 89°) + (sin2 3° + sin2 87°) + ...... +
tan 2 x 1 b11g 2
1
=
120 60 sin2 45°
62. (3) 4 – 2 sin – 5 cos 0 = 02 = (sin2 1°+cos2 1°)+(sin2 3°+ cos2 3°) + .......... + 22
2 (1 – cos2 ) + 5 cos – 4 = 0
2 – 2cos2 + 5 cos – 4 = 0
FG 1 IJ 2
 sin (90 ) cos
2 cos2 – 5 cos + 2 = 0
terms +
H 2K
(2 cos –1)(cos –2) = 0
1 1
= 1 + 1 ..... 22 terms + = 22 +
1 2 2
cos = cos =2
2
1
(not possible for 0° < < 90°) = 22
2
3 7. (3) 152 (sin 30° + 2 cos2 45° + 3 sin 30° + 4 cos2 45°
tan = + .... + 17 sin 30° + 18 cos2 45°)
1
Now,
152
FG 1 2
1
3
1
4
1
... 17
1
18
1 IJ
cos + tan H2 2 2 2 2 2 K
=
1 3 1 2 3
152G
F1 1
3
2 .....
17 IJ
9
2 1 2 H2 2 2 K
Type-IV ;g ,d A.P. gS tgk¡
Trigonometric functions dh minimum, 1 1
maximum values, substitution, rFkk a=
2
, d = , n = 18
2
series ij based questions
152
LM18 FG 2 1
b18 1g 12 IJK OPQ
1. (3) 4 tan2T + 9 cot2T
= (2tanT–3cotT)2+2 × 3 × 2 tanT cotT
N2 H 2

Minimum value = 2 × 3 × 2 = 12 LMS =


n
[2a + (n • 1• )d
OP
[  (2tanT–3cotT)2 > 0 & tanT cotT=1] N n
2 Q
2. (4) (sin T + cos T) + sec2T + cosec2T + tan2T + cot2T
2 2

= 1 + sec2T – tan2T + cosec2T – cot2T + 2(tan2T + cot2T) LM FG


152 9 1
17 IJ OP 152 9 19
12996 ,oa
= 3 + 2 [(tanT–cotT) +2] >7 2
D;ksafd (tanT – cotT) 2
>0 NH 2 KQ 2
3. (2) cos4T + sin4T = (cos2T + sin2T)2 – 2 cos2T sin2T
12996 114
maximum value osQ fy,,
8. (1) cos 90° = 0
2 sin T.cos T = 0
2 2
cos 1°. cos 2° ... cos 179° = 0
(cos2 T + sin2T)2 = (1)2 = 1

BLAM–183
f=kdks.kfefr
9. (4) We Know that
1
=(sin2 1° + cos2 1°) + (sin2 2° + cos2 2°) +.... 44 terms+
dk
FG 1 IJ x
2
sin x . Max Value
H 2K [sin(90° – T) = cosT]

Now, 1
= 1 + 1 + ....44 terms +
2
dk
FG 1 IJ 113
[sin2T + cos2T = 1]
sin113 . cos113 Max. Value
H 2K 1 1
10. (3) tan 1° . tan 2° . tan 3° ... tan 89° = 44 + = 44
2 2
= (tan 1° . tan 89°) (tan 2°. tan88°) ... tan 45° 16. (1) Let P = 81 sinx. 27 cosx
= (tan 1°. tan(90° – 1°) ) (tan 2° tan (90° – 2°) .. tan 45° = 34sin x. 33cos x
= (tan 1° . cot 1°) (tan 2°. cot 2°) ... tan 45° = 1.1...1. = 34sin x + 3cos x ...(1)
=1
We know that
[tan (90° – T) = cot T]
11. (1) We know that a sin + b cos dk Max. Value a2 b2
2
a cosec2 + b sec2 dk Min-Value = e a b j ? 4 sin x + 3 cos x dk Max Value 42 32 = 5
Now, from eqn (1)
Max. Value of P = 35
16 cosec2 + 25 sec2 dk Min. Value
17. (2) sin 89° = sin (90°–1°) = cos 1°
2
= e 16 25 j = (4 + 5)2 sin 79° = sin (90°–11°) = cos 11°
sin 69° = sin (90°–21°) = cos 21°
= 92 = 81 sin 59° = sin (90°–31°) = cos 31°
sin 49° = sin (90°–41°) = cos 41°
12. (1) Y = sin x + 3 cos x Expression
= (sin21° + cos21°) + (sin211° + cos211°) + (sin221° +
F1 3 I cos221°) + (sin231° + cos231°) + (sin241° + cos241°) +
GH
= 2 2 sin x 2 JK
cos x = 2 sin (x + 60°) sin245°

For Max. Value,


=5+
FG 1 IJ 2
=5+
1
sin (x + 60°) = 1 = sin 90° H 2K 2
x + 60° = 90°
x + 30° 1
13. (2) let P = sin6 + cos6 =5 [  sin2T + cos2T = 1]
2
For Min Value of P
18. (2) A = sin2 + cos4
Q = 45°
Min Value of P = sin6 45° + cos6 45° for Max Value = 90°
for Min. Value = 45°
F 1 IJ FG 1 IJ
= G
6 6
1 1 1 Max. Value of A = sin2 90° + cos4 90° = 1
H 2K H 2K =
8 8 4 Min Value of A = sin2 45° + cos4 45°
14. (1) Expression = (cot 41°.cot 49°) . (cot 42°.cot 48°)(cot
43°.cot 47°) . (cot 44° . cot 46°) . cot 45° FG 1 IJ FG 1 IJ
2 4
1 1 3
= cot 41° . tan (90° – 49°) . cot 42° . tan (90° – 48°) . cot
=
H 2K H 2K =
2 4 4
43° . tan (90° – 47°) . cot 44° . tan (90° – 46°).
= (cot 41°. tan 41°) (cot 42°. tan 42°) . (cot 43° . tan So, we get
43°). (cot 44° . tan 44°) . 1 = 1 3
[  tan (90° – T) = cotT; tanT. cotT = 1] A .
4
15. (4) sin2 1° + sin2 2° + sin2 3° + ..... + sin2 89°
= (sin2 1° + sin2 89°) + (sin2 2° + sin2 88°) + .... 44
terms + sin2 45° 19. (4) tan2 – cos2 = x sin . cos . tan
4 3 4 4 3
= (sin2 1° + sin2 (90° – 1°) + (sin2 2° + sin2 (90° – 2°) +

FG 1 IJ 2
FG 1 IJ 2 1 1
3
44 terms +
H 2K 1–
H 2K =x.
2 2

BLAM–184
f=kdks.kfefr

1–
1
=x×
3 4 –1
=x×
3
=
FG 1 IJ FG 1 IJ FG FG 1 IJ FG 1 IJ
2 2 2 2
3
I
JJ
4 2 4 2
H 2 K H 2 K GH H 2 K H 2 K K
3 2 3
x= 4 = 1
3 2 = (1 + 3)
4

20. (4) x 
2 1
2 sin
FG x IJ =1
x2 H2K 22. (3) x sin3T + y cos3 T = sinT.cosT
(x sinT) . sin2 T + (y cos T) cos2 T = sin T.cos T
Ÿ
FG x 1 IJ 2
2 2 sin
FG x IJ Ÿ x
1
0 x sinT . sin2 T + x sin T. cos2T = sin T.cos T
H x K H 2K x
x sin T (sin2 T + cos2 T) = sin T.cos T
x = cos T
D;ksfa d sin T dh gksxh vkSj
FG x 1 IJ ve x sin T = y cos T
maximum value
H x K cos T . sin T = y cos T y = sin T
ugha gks ldrkA ] x2 + y2 = cos2 T + sin2 T = 1
21. (2) x2y2 (x2 + y2 + 3)
23. (1) x cos – y sin =2 ...(i)
= (cosecT – sinT)2 (secT – cosT)2 {(cosecT – sinT)2 +
x sin + y cos =4 ...(ii)

(secT – cosT) + 3} = G
F 1  sin TIJ FG 1  cos TIJ 2 2
equation (i) rFkk (ii) dks square rFkk add djus ij]
2
H sin T K H cos T K
x2cos2 + y2 sin2 – 2 xy sin .cos + x2sin2 + y2
R|F 1  sin TI  F 1  cos TI  3U| 2 2
cos2
+ 2xy sin .cos = 4 + 16
S|GH sin T JK GH cos T JK V|
T W 2
x (cos 2
+ sin 2
) + y2 (sin2 + cos2 ) = 20
x2 + y2 = 20
F 1  sin T IJ FG 1  cos T IJ
= G
2
2
2
2
24. (2) According to question
H sin T K H cos T K
x cos T – y sin T = x2 y2 ...(i)
R|F 1  sin T I F 1  cos T I U| 2
2
2
2

S|GH sin T JK  GH cos T JK  3V| cos 2 T sin 2 T



1
T W a2 b2 x 2  y2
...(ii)

F cos T IJ FG sin T IJ R|SFG cos T IJ  FG sin T IJ  3U|V


= G
2
2
2
2 2
2
2
2

Y
H sin T K H cos T K T|H sin T K H cos T K W|
= cos2T × sin2T
FG cos 6
T  sin 6 T  3 cos2 T.sin 2 T IJ
H cos2 T.sin 2 T K x B
= cos6T + sin6T + 3 cos2T sin2T X' O T X
x2 y
= RSecos j esin j UV
2
3
2
3
3 cos2 .sin 2
+
y2
T W A

= (cos2T + sin2T)3 – 3cos2T. sin2T (cos2T + sin2T) + 3


cos2T.sin2T
Y'
= 1 – 3cos2T.sin2T + 3 cos2T. sin2T = 1
Aliter :
Let, T = 45° y x
sin T cos T
x = cosec 45° – sin 45° x 2  y2
_ x 2  y2

2
1 1 equation (i) ls]
=
2 2
x y
cos T  .sin T 1
y = sec 45° – cos 45°
x 2  y2 x 2  y2
x2y2 (x2 + y2 + 3)

BLAM–185
f=kdks.kfefr
cos 2 T sin 2 T 1 sin cos
 = . (sin2T + cos2T + 2 sinT.cosT – 1)
a2 b2 x 2  y2 sin .cos

x2 y2 1 sin cos
 = . 2 sinT . cosT = 2p
dx 2
y a 2
i 2
dx 2
 y .b2
i 2 x 2  y2 sin .cos
30. (1) 4n = cosn + sin x
x 2 y2 46 = cos6 + sin6
 1
a 2 b2 = 1 – 3sin2 . cos2
44 = cos4 + sin4
x
25. (3) x = a sec . cos = sec . cos = 1 – 2 sin2 . cos2
a
A/q
z 2u6 – 3u4 + 2
blh izdkj, y = sec . sin , = tan = 2(1 – 3 sin2 . cos2 ) –3 (1 – 2 sin2 . cos2 ) + 2
b c
= 2 – 6 sin2 .cos2 – 3 + 6 sin2 . cos2 + 2
x 2 y2 z2 = 4 – 3 = 1.
 
a 2 b2 c 2 2nd Method
= sec2 . cos2 + sec2 . sin2 – tan2 let = S = 0°
= sec 2
(cos 2
+ sin 2
) – tan 2
= sec2 – tan2 =1 2u6 – 3u4 + 2 = 2 (cos6 + sin6 ) –3 (cos4 + sin4 ) +
2
26. (2) a cos T + b sin T = p ...(i)
= 2 (cos6 0° + sin6 0°) –3 (cos4 0° + sin4 0° + 2)
a sin T – b cos T = q ...(ii)
= 2–3+2=1
equation (i) rFkk (ii) dks square rFkk add djus ij]
1
a2 cos2 T + b2 sin2 T + 2 a b sinT . cos T + a2 sin2 T + b2
31. (2)
x
sec n FG n IJ n
sec
cos2 T – 2 a b sin T. cos T = p2 + q2 a HaK
a2 cos2 T + a2 sin2 T + b2 sin2T + b2 cos2 T = p2 + q2
a2 (cos2T + sin2T) + b2 (sin2T + cos2T ) = p2 + q2 1

a2 + b2 = p2 + q2
y
tan n
FG y IJ n
tan
27. (3) x = a (sinT + cosT) ,oa
b HbK
y = b (sinT – cosT) Now,
sec2 – tan2 =1
x y
sin cos ,oa sin cos 1 1
a b FG x IJ FG y IJ
x x
1
x2 y2 Ha K HbK
= (sinT + cosT) + (sinT – cosT) 2 2
a2 b2 n y
= sin2T + cos2T + 2 sinT.cosT + sin2T + cos2T – 2sinT . 32. (1) cos3 and sin 3
a b
cosT
A/q
= 2 (sin2T + cos2T) = 2
28. (4) x = a sinT – b cosT --- (i) 2 2

y = a cosT + b sinT --- (ii)


FG x IJ FG y IJ
3 3

equation (i) rFkk (ii) dks square rFkk add djus ij]
HaK HbK
x + y = (a sinT – b cosT)2 + (a cosT + b sinT)2
2 2 2 2

= a sin T + b cos T – 2ab sinT . cosT + a cos T + b


2 2 2 2 2 2 2
= dcos h dsin h
3 3 3 3

sin2T + 2ab sinT . cosT = cos2 + sin2 = 1.


= a2 (sin2T + cos2T) + b2 (cos2T + sin2T) = a2 + b2 33. (3) Given, y = b tan
[  sin2T + cos2T = 1] b
a cot
29. (3) sin T + cos T = p x = a sin cosec y
n
sec T + cosec T = q
1 1 sin cos a2 b2
2
– cot2
cos
+
sin
=q
sin .cos
=q n2 y 2 = cosec =1

34. (1) Given the expression


FG sin cos IJ ((sinT + cosT)2 –1) (sin + sec )2 + (cos + cosec )2
q (p2 – 1) =
H sin .cos K = (K + sec . cosec )2

BLAM–186
f=kdks.kfefr
Let = 45°
tan 2 .sin 2 = tan . sin
FG 1 I FG 1
2J
2
I
2J
2

K e 2 2 j
2 2 2
m – n = (m + n) (m – n)
H 2 K H 2 K = 2 tan . 2 sin
= 4 tan . sin
FG 3 IJ 2

2 bK 2g 2 = 4 mn
H 2K 39. (3) Sec Q. (38)
9 = (K + 2)2 40. (1) Given the expression
K = 1. cosec – sin = m and sec – cos =x
35. (1) Given the expression. putting = 45°
1 1
cos2 sin 3 m= 2 n= 2
K sin .cos 2 2
1 tan sin cos
1 1
cos2 sin 3 = =
K sin .cos 2 2
sin sin cos
1
cos A/q
2 2
cos3 sin 3
K sin .cos dm n h dmn h
2 3 2 3

cos sin sin cos


2 2
cos3 sin 3 LF 1 I 2
OP 3

MML 12 FGH 12 IJK OPP


2 3

= MG
1
g K MNH 2 JK
sin .cos
bcos sin 2 PQ N Q
bcos sin gdcos 2
sin 2 sin .cos h 2 2

cos sin F 1 IJ
= G
3
3 FG 1 IJ 3
3
=
1 1
1
= K + sin . cos
H 2K H 2K 2 2
1 + sin .cos = K + sin .cos K=1 41. (4) Given the expression
36. (4) Given, x = cot + tan and y = sec – cos
x = sec – tan and y = cosec + cot putting = 45°
Putting = 45°
1
x=1+1=2 y= 2
x= e 2 j
1 and y = 2 1 2

1
xy = e 2 1 je 2 j
1 2 1 1 =2 =
2
.

y– x= 2 1– 2 1 2 A/q
Thus., (x2 y)2/3 – (xy2)2/3
2
xy 1 y x
F 1 IJ
2
3 F2 F 1 I I 2 3

37. (3) Given = G2


H
2

2K
GH GH 2 JK JK 2 1 1
(tan 20°. tan40° tan 80°) tan 60°
= tan (3 × 20°). tan 60° 42. (2) Given the equation
a sec + b tan + c = 0 ...(1)
= 3 3 3 p sec + q tan + r = 0 ...(2)
[Note :– we know that tan . tan2 . tan 4 = tan 3 ] Multipling by q in eqn (1) and Multipling by b in equa-
tion (2), we get
38. (4) mn btan sin gb tan sin g
aq sec + bq tan + cq = 0
sin 2 bp sec + bq tan + br = 0
sin 2 – – –
= tan 2 sin 2 = sec (aq – bp) = br – cq
cos2

=
sin 2 1 cos2 d h sec = aq
br cq
bp
cos 2

BLAM–187
f=kdks.kfefr
n
Put the value of sec in eq (1) we, get
Type-V
pc ar
tan = aq sin (x + y), cos (x + y), tan (x + y) etc.
bp
formulae ij based questions
we know,
sec2 – tan2 =1 1. (1) ge turs gSa,
LM br qc OP LM pc
2
ar OP 2

1 cot (A + B) =
cotA ¹ cotB – 1
N aq bp Q N aq bp Q cotA  cotB
(br – qc)2 – (pc – ar)2 = (aq – bp)2
cotA ¹ cotB – 1
43. (3) x cos + y sin = 4 x=
cotA  cotB
x cos + x cos = 4 [Given x cos = y sin ]
2x.cos = 4 2. (2) tan 56° = tan(45° + 11°)
x cos = 2 = y sin
tan 45 tan 11
2 =
2 1 – tan 45 tan 11
cos = and sin = y
x
Now LM tan( A B)
tanA tan
 tan 45 1
OP
sin2 + cos 2
=1 N 1 – tanA tan Q
4 4
1 sin 11
y2 x2 1 tan 11 1
= = cos11
1 1 1 – tan 11 sin 11
1 1–
cos11
x2 y2
44. (4) we have cos11 sin 11
= cos11 – sin 11
3x sin + 2 y cos = 4] × 3
2x cos – 3y cos = 2] × 2
+
9 x sin + 6y cos + 4 x sin – 6y cos FG  x IJ = tan 4  tan x
= 12 + 4 3. (2) tan H 4 K 1 – tan ˜ tan x
13 x sin = 16 4
16
13 sin =
x
...(1) LM tan(A  B) OP
MM 1tanA  tanB P
Similary, 13 cos = y
2
...(2) N – tanA tanB PQ
squaring & Adding eqn (1) and (2) LM tan 1OP
169 sin2 + 169 cos2 =
256 4 N 4 Q
x2 y2
1 tan x
= 1 – tan x
256 4
169 .
x2 y2 4. (3) ;gk¡ ,
45. (1) We have
cosC – cosD = y
sin + cos =
a2 = 1 + 2 sin . cos y = cosC – cosD
a2 – 1 = 2 sin . cos
FG C D IJ sin FG C – D IJ
and y = –2 sin
H 2 K H 2 K
sin cos a
b = sin + cosec = = [;g cos C – cosD dk basic formula gS]
sin .cos sin .cos
Now
5. (1) ;gk¡,
A/q sinx cosy + cosx siny = 1
sin(x + y) = 1
a
b (a2 – 1) = 2 sin .cos = 2a. [  sin (A + B) = sinA cosB + cosA sinB]
sin .cos

BLAM–188
f=kdks.kfefr

sin(x + y) = sin
LM cos C cos D 2 cos
FG C D IJ cosFG C – D IJ OP
2
N H 2 K H 2 KQ
x+y=
2
F I
= 2cos G J cos x
6. (3) Here,
H 4K
3 3 2
cosx = – and <x< = cos x
5 2 2
x rhljs quadrant esa gS, rFkk ge tkurs gaS rhljs quadrant = 2 cos x
esa fliZQ tan rFkk cot (+ve) gksrs gSaA 9. (1) cos 105° = cos(60° + 45°)
ABC esa, = cos 60° cos 45° – sin 60° sin 45°
[  cos (A + B) = cosA cosB – sinA sinB]

1 1 3 1
= –
2 2 2 2

1– 3
=
2 2
10. (2) ;gk¡,
Pythagoras theorem osQ }kjk,
AC2 = AB2 + BC2 cos
FG – IJ cosFG – IJ
52 = (–3)2 + BC2
H4 K H4 K
16 = BC2
BC = 4
F I F I
– sin G – J sin G – J ?
We know that
H4 K H4 K
sin2A = 2sinA cosA ekuk
sin2x = 2 sinx cosx
– A
FG –4 IJ –3 24 4
=2×
H5K 5
=
25
– B
rhljs quadrant esa sine (–ve) gksrk gS]
[ 4
 ;gk¡, sinT (–ve) gSA
7. (4) ;gk¡, sinC + sinD = x cos
FG – IJ cosFG – IJ
H4 K H4 K
x = sinC + sinD

FG C D IJ FG C – D IJ F I F I
– sin G – J sin G – J
x = 2 sin
H 2 K cos
H 2 K H4 K H4 K
[ ;g sinC + sinD dk basic formula gS] = cosA cosB – sinA SinB
= cos(A + B)
FG x IJ cosFG – x IJ
8. (2) cos
H4 K H4 K = cos
LM – –
OP
N4 4 Q
F x – xI
= 2cos G
G4 4 JJ L
= cosM – –
OP
GH 2 JK N2 Q
L
= cosM – ( )
OP
F I N2 Q
G
cosG 4
x –
4
x
JJ FG IJ
GH 2 JK = sin (T + )  cos 2 –
H K sin

BLAM–189
f=kdks.kfefr
FG 75 15 IJ cos FG 75 – 15 IJ
11. (4) sin 75°+sin 15° = 2 sin H 2 K H 2 K
LMsin C sin D 2 sin
FG C D IJ cosFG C – D IJ OP
N H 2 K H 2 KQ
= 2 sin 45° cos 30°

1 3 3
= 2 = 15. (4) ge tkurs gSa]
2 2 2
12. (3) ge tkurs gSa] tan( + )=
2 cosA sinB = sin(A + B) – sin(A – B)
2 cos 45° sin 15° tan ( + )
= sin (45° + 15°) – sin (45° – 15°)
= sin 60° – sin 30°

3 1 =
= –
2 2

3 –1
=
2
13. (4) ge tkurs gSa]
cos2A = 1 – 2 sin2A
A
cosA = 1 – 2 sin2
2 =1

ekuk A = 45°
tan ( + )=1
45
cos 45° = 1 – 2 sin2
2
tan ( + ) = tan
1
2 sin2 22 = 1 – cos 45°
2 + = .

1
2 sin2 22 =1– 16. (2) ;gk¡, + =
2
(1 + tan )(1 + tan )
2 sin2 = = 1 + tan + tan + tan tan
= 1 + tan + tan + tan tan
Also, we know that,
sin2 =

tan ( + )=

tan =

14. (2)

(1 + tan )(1 + tan )


= 1 + 1 – tan tan + tan tan = 2
17. (2) cos ( – A) = a, cos ( – B) = b
ekuk = 90°
a = cos (90° – A) = sin A

BLAM–190
f=kdks.kfefr
b = cos (90° – B) = sin B Componendo rFkk dividendo dk bLrseky djosQ
cos A =

rFkk cos B =
A/q
sin2 (A – B) + 2 ab cos (A – B)
= (sin A. cos B – cos A. sin B)2 + 2ab (cos A. cos B + sin
A. sin B)

= a2 (1 – b2) + b2 (1 – a2) – 2ab + 2ab

= a2 – a2b2 + b2 – a2b2 + 2a2b2 = a2 + b2

18. (2)

tanx coty =

[ 2 cosA cosB = cos (A + B) + cos (A – B)]

+
21. (1) Given,

+ sin ( – ) =
[ cos (– T) = cos T]

=0

[ cos ( – T) = – cos T]
19. (2) Given the expression
3 tan . tan = 1
let = = 30° cos ( – ) =

3 × tan 30° tan 30° =

Now
A/q

20. (2) ;gk¡,


A/q
tan 2 = tan ( + + – )

BLAM–191
f=kdks.kfefr
25. (2) Expression
=
=

= =
=

= =1
22. (1)
[ sin2x + cos2x = 1]
26. (4) sin (B – C) cos (A – D)
= (sinB cosC – cosB sinC) (cosA cosD + sinA sinD)
= sinB cosC cosA cosD + sinB cosC sinA sinD
– cosB sinC cosA cosD – cosB sinC sinA sinD (1)
sin (A–B) cos(C–D) = (sinA cosB – cosA sinB) (cosC
[2sin A sin B = cos(A – B) – cos(A + B)] cosD) + sinC sinD)
= sinA cosB cosC cosD + sinA cosB sinC sinD – cosA
sinB cosC cosD – cosA sinB sinC cinD (2)
sin(C – A) cos(B – D)
= (sinC cosA – cosC sinA) (cosB cosD + sinB sinD)
= sinC cosA cosB cosD + sinC cosA sinB sinD – cosC
sinA cosB cosD – cosC sinA sinB sinD (3)
equations 1, 2 rFkk 3 add djus ij
sin(B – C) cos(A – D) + sin (C – A) cos(B – D) + sin (A
– B) cos (C – D) = 0

23. (3) 27. (2)

=
[cos( + T) = – cosT; cos(– T) = cosT;
sin( – T) = – sinT]

= cot x cotx = =
= cot2x.
24. (2) tan – tan =x
28. (2) ×

= ×

cot . cot = [Given cot – cot = y]


= ×

A/q cot ( – ) = = =
=

BLAM–192
f=kdks.kfefr

= × tanx =

= tanx . tanx = tan2x

29. (2) – =

= – =

=
=
= tan x + tanx
= 2 tanx

= =4

Aliter :
ekuk x = 30°

Aliter :
ekuk x = 30°
= =
-

- = = =4

31. (4) tan


+

= =

= 2 tan 30°
= 2 tan x (x = 30° ds fy,)
30. (3) (cos 2T + 3cos T) ÷ (cos T – sin T)
3 2 6 6

= = =

= –1
=

BLAM–193
f=kdks.kfefr
Aliter :
Let us take A = 0° 33. (4)

tan tan
=
= = 1 × tan

=–
= –1 = = =
32. (4) Given,

cos = 34. (4) cos (90° – B) sin (C – A) + sin(90° + A)


cos (B + C) – sin (90° – C) cos (A + B)
= sin B sin (C – A) + cos A cos ( B + C)
2 cos2 – cos C cos ( A + B)
Nr rFkk Dr dks 2 ls multiply djus ij]

cos

sin = =0

Aliter :
and, cos = cos (90°–B) sin(C – A) + sin (90° + A) cos (B + C) – sin
(90° – C) cos (A + B)
= sinB [sinC cosA – cosC sinA]
2 cos2 + cosA [cosB cosC – sinB sinC]
– cosC [cosA cosB – sinA sinB]
= cosA sinB sinC – sinA sinB cosC
cos + cosA cosB cosC – cosA cosB cosC
+ sinA sinB cosC = 0
35. (4) (cos7A + cos5A) ÷ (sin7A – sin5A)

sin

According to question.

= cos

= = = = cotA

BLAM–194
f=kdks.kfefr
Aliter :
36. (3)
ekuk T = 60°

+ sin120° tan180°
=

=
=

37. (3) sin75° + sin15°


= =–

=
= cos 120° = cos2T (T = 60° ds fy,)
= 2 sin 45°.cos30°
39. (4)

2 sinA sinB
= cos(A – B) – cos (A + B)
= =
=
38. (1) (cos3T + 2cos5T + cos7T) ÷ (cosT + 2cos3T + cos5T)
+ sin2T tan3T = =1 ( cos90° = 0)

= + sin2T tan3T
40. (4)

= + sin2T tan3T
=

=
= + sin2T tan3T

= sin2A + cos2A = 1
= + sin2T tan3T
41. (4)

= + sin2T tan3T

=
=
= cos2T

BLAM–195
f=kdks.kfefr
b g b g b
tan 2x  2y .cos 2 x  2y .sin 2 x  2y g =
4 cos 6 x
=
b g
sin 2x  2y .cos x . cos 2y Lcosb120q2x  120q2x g 
2 sin x cos x M
O
 sin b A  Bg sin A cos B  cos A sin B
N cosb120q2x  2x  120q2x gPQ
4 cos 6 x
=
tan b2x  2y g cos 2x .cos 2y  sin 2x sin 2y b g
sin 2 x cos 4 x  cos 240q
=
cos 2x .cos 2y
4 cos 6 x
= tan (2x + 2y) [1 – tan2x tan2y) =
sin 2 x cos 4 x  cos 180q60q b g
btan 2x  tan 2y g
= tan (2x + 2y) tan b2 x  2y g 4 cos 6 x
=
b
sin 2x cos 4 x  cos 60q g
tan A  tan B
 tan( A  B)
1  tan A tan B =
4 cos 6 x
sin 2 x
= tan2x + tan2y sin 2 x .cos 4 x u
Aliter : 2
Let, x = 15°, y = 0° 4 cos 6 x
=
(sin 4 15 sin 4 0 )[(tan 2 15 tan 2 0 ] 1 sin 2 x
(sin 4 15 sin 4 0 ) 2
b g
sin 2 x  4 x  sin 2 x  4 x  b 2
g
sin 60 0)(tan 30 – 0) 8 cos 6x
=
(sin 4 15 – sin 4 0 ) sin 6 x  sin 2x  sin 2x
= 8 cot 6x
3 1
cos 40 – cos140
2 3 1 43. (2)
sin 80 sin 20
3 3
2
–2 sin
FG 40
140
.sin
40 –140 IJ FG IJ
fodYi (Option), (4) ls
=
H 2 2 K H K
tan 2x + tan 2y = tan 2 × 15° + tan 0° (80 20 ) (80 –20 )
2 sin .
1 2 2
3  cosC – cosD
32 cos x  48 cos x  18 cos x  1
6 4 2 C D C–D
42. (3) ...(1) = – sin .sin
b g b
4 sin x cos x sin 60q x cos 60q x g 2 2
sinC + sinD
sin(60° + x) cos (60° + x)
We know that
= 2 sin .cos
cos6x = cos2.(3x)
= 2cos23x – 1
= 2[4cos3x – 3cosx]2 –1 =

=
= 2[16cos6x + 9cos2x – 24cos4x] –1
cos6x = 32cos6x + 18cos2x – 48cos4x – 1
equation (1) esa j[kus ij
=
=
44. (2)
[2sin(60° + x) cos (60° + x)]

=
= =

=
=

BLAM–196
f=kdks.kfefr

=
=

=
=
Aliter :
= ekuk] A = 90°

= = =1

fodYi] (Option) (3) ls

= tan = tan =1

46. (1)
= tan

Aliter :
Let, T = 60° =

= =

= =

fodYi] (Option) dh lgk;rk ls]


= = 2 tan 2 .
fodYi (2) ls]
47. (2) Given,

45. (3)
=

= =

BLAM–197
f=kdks.kfefr
48. (3) Given the expression, Pythagoras theorem osQ }kjk]

let x cos = y cos = Z cos Ÿ XY2 + YZ2 = XZ2 Ÿ

x cos = y cos (120 + ) = z cos (240 + ) = K Ÿ 24 = (XZ – YZ) (XZ + YZ) Ÿ XZ +YZ = 12 ...(ii)
nksuksa equations dks add djus ij]
= cos + (cos (120 + ) + cos (240 + )) 2 XZ = 14 Ÿ XZ = 7
? YZ = 7 – 2 = 5

= cos + 2 cos
? sec X = , tan X =

cos ? sec X + tan X =


= cos + 2 cos (180° + ). cos (–120°)
3. (2) sin (A – B) = = sin 30° A – B = 30°
= cos + 2. (– cos ) ×

= cos – cos =0 Again, cos (A + B) = = cos 60° A + B = 60°

on adding
A + B + A – B = 30° + 60° = 90°
49. (1) cos2 + cos2 ( + 120°) + cos2 ( – 120°) 2 A = 90° A = 45°
putting = 0° A – B = 30° B = A – 30° = 45° – 30° =15°
then expression
cos20° + cos2 (120°) + cos2 (–120°) radian

= 4. (2) 3 (sec2T + tan2T ) = 5


3 (1 + tan2T + tan2T ) = 5
50. (3) we know that 3 + 6 tan2T = 5 6 tan2T = 5 – 3 = 2
If A + B + C = 180° then
tan A + tan B + tan C = tan A. tan B tan C tan2T = =

tanT = = tan 30° T = 30°

Miscellaneous
cos2T = cos 60° =
1. (4) sin (2x – 20°) = cos (2y + 20°)
sin (2x – 20°) = sin (90° – 2y – 20°) sinT = sin 30° = cos2T = sin T
2x – 20° = 90° – 2y – 20°
2x + 2y = 70° + 20° = 90° 5. (4) r sin T = 1 ...(i)
x + y = 45° r cos T = ...(ii)
tan (x + y) = tan 45° = 1 Equation (i) dks (ii) ls divide djus ij
2. (2)
tan T = tan 30° T = 30°

6. (1) tan2 = 1 + 2 tan2


sec2 – 1 = 1 + 2(sec2 – 1)
[ sec2 = 1+ tan2 ]
sec2 = 2 sec2
XZ – YZ = 2 ... (i)

BLAM–198
f=kdks.kfefr

cos = cos 14. (3)

cos – cos = 0
7. (2) 4cos2T – 4cosT + 1 = 0 (2 cosT – 1)2 = 0
2 cosT – 1 = 0 2 cosT = 1

cosT = = cos 60° T = 60°

tan (T –15°) = tan (60° –15°) = tan 45° = 1


8. (4) sin (4 – ) = 1 = sin 90° 4 – = 90° ...(i)

cos (2 + )= = cos 60° 2 + = 60° ...(ii) AQD esa]


equations (i) rFkk (ii) dks add djus ij] sin 60° = AD =
4 – +2 + = 90° + 60°
APD esa]
6 = 150° = = 25°
sin 75° =
equation (ii) ls,
2 × 25° + = 60° = 60° – 50° = 10°
11. (4) a2 + b2 + c2
sin ( + 2 ) = sin (25° + 2 × 10°) = sin 45° = = ab + bc + ca
2a2 + 2b2 + 2c2
= 2ab + 2bc + 2ca
9. (1) (r cos T– )2 + (r sin T – 1)2 = 0
a2 + b2 + b2 + c2 + c2 + a2 – 2ab – 2bc – 2ca = 0
r cos T– =0 ,oa r sin T – 1 = 0 a2 + b2 – 2ab + b2 + c2 – 2bc + c2 + a2 – 2ca = 0
(a – b)2 + (b – c)2 + (c – a)2 = 0
r cos T = ,oa r sin T = 1
a–b=0 a=b
nksuksa dks square djosQ add djus ij b–c=0 b=c
r2 cos2T + r2 sin2T = 3 + 1 c–a=0 c=a
r2 (sin2T + cos2T) ABC ,d equilateral triangle gSA
=4 r2 = 4 r=2 A= B= C = 60°
sin2A + sin2B + sin2C
tan T = = = 3 sin2A = 3 × sin2 60°

,oa] r cosT = cosT


=3× = =

= secT =
12. (3)

= =
ABC esa]
AB2 = AC2 + BC2
c2 = a2 + b2 ...(i)
= = = ABC ls]

BLAM–199
f=kdks.kfefr

cosec B = = ...(ii) =

cos A = = = =

cosec B – cos A = – y =

= = =
13. (2) cosecT = cot T 2

cosecT = cosec2T – 1
=
cosec2T – cosecT = 1 ...(i)
Expression
cosec4T – 2cosec3T + cot2T =

= cosec4T – cosec3T – cosec3T + cosecT = 4 cos 2


[ cot2 T = cosec T] Putting = 60°, y = 4 cos (2 × 60°)
= cosec T (cosec T – cosecT) – cosecT (cosec2T–1)
2 2

= cosec2T × 1– cosec T × cosec T =4× = – 2 = –x


= cosec2T – cosec2T = 0
18. (3) According to the Question,
10. (2) 4 sin2T – 1 = 0 4 sin2T = 1

sin2T = sinT =

sinT = sin30° T = 30°


cos2T + tan2T = cos230° + tan230°

= + = + = = =

15. (1) cosA + sinA = cosA --- (i)


cosA – sinA = x (let) --- (ii) =
nksuksa equation dks square rFkk add djus ij
cos2A + sin2A + 2 sinA . cosA + cos2A + sin2A – 2 sinA =
cosA = 2 cos2A + x2
2 (cos2A + sin2A) = 2 cos2A + x2 = =1
x2 + 2 cos2A = 2
19. (1)
x2 = 2 – 2 cos2A = 2 (1 – cos2A) = 2 sin2A

x= sin A =
16. (1) sin + sin ( + 120°) + sin ( + 240°)
= sin + 2, sin
=

= sin + 2. sin (180 + ). cos 60° =

= sin + 2 × (– sin ).
=
= sin – sin = 0
17. (2) Given,
=

x= = 1.

BLAM–200
f=kdks.kfefr

20. (2) Given, x + y = z


=
A/q
(cos x + cos y) + (1 + cos z)

= 2.cos

฀–tan (62° – 2°) =

฀–tan 60° =
=

–฀
=
26. (1) tan (5x) = tan (3x + 2x)

= tan 5x =

tan 5x – tan 5x. tan3x. tan 2x = tan 3x + tan 2x


= . tan 5x – tan 3x – tan 2x = tan 5x. tan 3x. tan 2x
27. (2) Given,
21. (4) cos3 + cos3 (120° + ) + cos3 ( – 120°)
ABC is a triangle
let = 0°
A + B + C = 180°
we get
B + C = 180° – A
Now, A/q
= = cos 3 sin2 A + sin2 B + sin2 C
= 1 – cos2 A + 1 – cos2 B + sin2 C
22. (4) Given the expression,
= 2 – cos2 A – cos (B + C). cos (B – C)
cos 25° + cos 85° + cos 155°
= 2 – cos A [cos A – cos (B – C)]
= cos 25° + cos (360 – 275°) + cos (180 – 25°)
= 2 – cos A [– cos (B + C) – cos (B – C)]
= cos 25° + cos 275° – cos 25°
= 2 + cos A. 2 cos B. cos C
= cos 275°
sin2 A + sin2 B + sin2 C – 2cos A. cos B. cos C = 2
23. (3) Given the expression,
28. (1) Given,
tan4 A – sec4 A + tan2 A + sec2 A
= (tan2 A + sec2 A) (tan2 A – sec2 A) + (tan2 A + sec2 A)
= – tan2 A – sec2 A + tan2 A + sec2 A
sin + = 0
sin + sec2 = sec4 29. (1) Given,
sin = sec4 – sec2 x = sin + cos
= sec2 = (sec2 – 1) Squaring Both sides,
= sec2 . tan2 x2 = sin2 + cos2 + 2 sin . cos
24. (2) tan2 = 2 tan + 1 x2 = 1 + 2y x2 – 2y = 1
Adding Both sides by 1, we get again, Squaring Both sides, we get,
1 + tan2 = 2 (1 + tan2 ) x4 + 4y2 – 4x2 y = 1...(1)
sec2 = 2 sec2 x2 = 1 + 2y 2x2 = 2 + 4y
2
–2x + 4y + 1 = 1 ...(2)
on adding eqn (1) and (2), we get
x4 – 4x2y – 2x2 + 4y2 + 4y + 1 = 0
cos2 = 2 cos2 30. (3) 4 (sin2 A + cos A) = 1
cos2 = 1 + cos2 4 sin2 A + 4 cos A – 1 = 0
cos2 = cos2 –1 4 – 4 cos2 A + 4 cos A – 1 = 0
cos2 + sin2 = 0 4 cos2 A – 4 cos A – 3 = 0
25. (2) Given the expression, By solving it, we get

cos A = – A = 210° ‡‡‡

BLAM–201
f=kdks.kfefr

ADVANCED LEVEL QUESTIONS


(2)
1. equal to
(3)
ds cjkcj gSA
(4)
6. tan 20° tan 40° tan 80° equals to
(1) 2 sin T (2) 2 cos T
tan 20° tan 40° tan 80° cjkcj gS
(3) 2 sec T (4) 2 tan T

2. If tan x = , then what is value of (1) (2)

(3) (4)

7. If tan A = and tan B = , then the

value of A – B equals to
;fn tan x = gS] rc dk eku gksxk\
;fn tan A= rFkk tan B= rks A – B dk
(1) (2) eku gS

(1) (2)
(3) (4)

3. If cos T+ cos(120° + T) cos( T– 120°) = 0, then cos3


T+ cos3 (120° + T) + ฀cos3( T– 120°) equals to
(3) (4)

;fn cos T+ cos(120° + T) cos( T฀– 120°) = 0rks cos3 T 8. If sin x + sin y = a, cos x + cos y = b, then the value
T) + ฀cos ( T฀– 120°) ds cjkcj gSA
3 3 of cos (x – y) equals to
+ cos (120° +
;fn sin x + sin y = a, cos x + cos y = b rks cos (x – y)
(1) cos T (2) cos T 2 dk eku gS

(1) (2)
(3) cos 3 T (4) sin 3 T

4. If m tan(T– 30°) = n tan(T+ 120°), then the value of (3) (4)


cos 2Tequals to
9.
;fn m tan(T– 30°) = n tan( + 120°), rks cos 2Tdk eku
gS

(1) (2) equals to

(3) (4)

cjkcj gS
5. The value of cot 7 equals to
(1) (2)
cot 7 dk eku cjkcj gS
(3) (4)
(1)

BLAM–202
f=kdks.kfefr
10. If sin A = n sin B, then what is v alue of
16. equals to

(1) tan2A (2) tan3A


(3) tan 84 (4) tan 4A

;fn sin A = n sin B gS] rc dk eku


17. If cot x = , than the value of a cos2x + b sin 2x will
D;k gksxk\ be

(1) (2) ;fn cot x = , gks rks a cos2x + b sin 2x dk eku gksxk
(1) a (2) b
(3) (4)
(3) sin a (4)
11. The value of tan 75° is
tan 75° dk eku gksxk %
18. If and then the value of tan x

(1) (2) =?

;fn vkSj gS] rc tan x = ?


(3) (4)

12. sin2 (120° – A) + sin2A + sin2 (120° + A) equals to


sin2 (120° – A) + sin2A + sin2 (120° + A) dk eku gksxk (1) (2)

(1) (2)
(3) (4)

(3) (4) 19. If Y = a cos2 x + 2b sin x. cos x + C sin2 x and Z = a


sin2 x – 2b sin x. cos x + C cos2 x then
13. cos 20° + cos 100° + cos 140° equals to
;fn Y = a cos2 x + 2b sin x. cos x + C sin2 x vkSj Z =
cos 20° + cos 100° + cos 140° dk eku gksxk a sin2 x – 2b sin x. cos x + C cos2 x gS] rc
(1) 1 (2) 0 (1) Y + Z = a + c (2) Y + Z = a + b
(3) –1 (4) None of these (3) Y + a = x + b (4) None of these
14. cos 20° . cos 40° cos60° . cos 80° equals to
cos 20° . cos 40° cos60° . cos 80° dk eku gksxk
20. If sin 54° = then the val ue of

(1) (2)
is;

(3) (4)
;fn sin 54° – rks dk
15. If cos A = m cos B, then—
eku D;k gksxk\
;fn cos A = m cos B, rc
(1) (2)
(1)

(2) (3) (4)

(3) 21. If sin2x = n sin2y, then

(4) ;fn sin2x = n sin2y, rks ds cjkcj gksxk %

BLAM–203
f=kdks.kfefr

(1) (2)
(1) (2)

(3) (4)
(3) (4)
22. If a2 cosec2x – b2 cot2x = c2 then find the value of
cos2 x is equal to (assume b2 a2)
27. x, y, z are the lengths of three sides of a triangle
;fn a2 cosec2x – b2 cot2x = c2 rks cos2 x dk eku Kkr djsaA
ABC. if a, b, c are related by the relation a3 + b3 + c3
(b2 a2 eku)
= 3abc, then find the value of cos2A + cos2B + cos2C
is
(1) (2) x, y, z ABC dh rhu Hkqtkvksa dh yEckb;k¡ gSaA ;fn a, b, c,
a3 + b3 + c3 = 3abc, ls lacaf/r gSA rks cos2A + cos2B +
cos2C dk eku Kkr djsaA
(3) (4)
(1) (2)
23. If (sin T + sec T)2 + (cos T + cosec T)2 = (K + sec T
cosec T) + (sin T + cos T)2 – sin2 T –1, then find the
value of K2. (3) (4)
;fn (sin T + sec T)2 + (cos T + cosec T)2 = (K + sec T
cosec T) + (sin T + cos T)2 – sin2 T –1, rks K 2 dk eku Kkr
djsaA 28. If and then find the value
(1) 2 (2) 0
(3) 1 (4) 3 of 7 cot2A + cot2B is
24. If sin T + cos T = x, then find the value of sin6T +
sin4T + cos6T + cos4T. ;fn rFkk rks 7 cot2A + cot2B
;fn sin T + cos T = x, rks sin6T + sin4T + cos6T + cos4T
dk eku Kkr djsaA dk eku Kkr djsaA
(1) 1 (2) 4
(1) (2) (3) 0 (4) 2
29. If x = (1 – cos D) ˜ (1 – cos E) ˜ (1 – cos J) = (1 + cos D)
(1 + cos E) (1 + cos J), then find x D, E, J are acute
(3) (4) angles.
;fn x = (1 – cos D) ˜ (1 – cos E) ˜ (1 – cos J )
= (1 + cos D) (1 + cos E) (1 + cos J), rks x D, E, J
25. If A + B + C = then find the value of cos2A + (1) ± cos D cos E cos J (2) ±1
cos2B + cos2C. (3) ± tan D tan E tan J (4) ± sin D sin E sin J
30. If cot D = n cot E and cos D = m cos E then find the
;fn A +B+C= rks find cos2A + cos2B + cos2C value of sin2D.
;fn cot D = n cot E rFkk cos D = m cos E rks sin2D dk
dk eku Kkr djsaA eku Kkr djsaA
(1) 1 + 2 sinA sinB sinC
(2) 1 + 4 sinA sinB sinC
(1) (2)
(3) 1 – 4 sinA sinB sinC
(4) 1 – 2 sinA sinB sinC

(3) 1 (4)
26. Find the value of
31. If 10 cos4 D + 15 sin4 D = 6 then find the value of
81sec8D + 16 cosec8 D is
;fn 10 cos4 D + 15 sin4 D = 6 rks 81sec8D + 16 cosec8
D dk eku Kkr djsaA
dk eku Kkr djsaA (1) 250 (2) 1050
(3) 750 (4) 1250

BLAM–204
f=kdks.kfefr
–1 –1
32. If two angles of a triangle tan 2, tan 3 then who
will be third angles? (3) (4)
;fn fdlh f=kHkqt osQ nks dks.k tan–1 2, tan–1 3 gks rks rhljk
dks.k D;k gS\ 37. If sin A = M sin B, then
(1) tan–1 2 (2) tan–1 4 ;fn sin A = M sin B, rks

(3) (4) (1)

33. Find the value of cos ˜ cos ˜ cos ˜ cos (2)

˜ cos
(3)

cos ˜ cos ˜ cos ˜ cos ˜ cos dk


(4)
eku Kkr djsaA

(1) (2) 38. If then find the value of

(3) 1 (4)
is.
34. Find the maximum and minimum value of 643cosT˜
165sinT˜ 2562cosT˜ 10242sinT
643cosT˜ 165sinT˜ 2562cosT˜ 10242sinT dk vf/dre rFkk ;fn rks
U;wure eku Kkr djsaA
dk eku Kkr djsaA

(1) & (2) &


(1) (2)

(3) & (4) &


(3) (4)
35. If and then find the value of
39. If cot x ˜ cot y = 3 then find the value of 2 cos (x + y)
cot D. =?
;fn cot x ˜ cot y = 3 rks 2 cos (x + y) dk eku Kkr djsa =
?
;fn rFkk rks cot D dk eku Kkr
(1) 1 + sin (x – y) (2) cos (x – y)
djsaA (3) 1 (4) + tan (x + y)

40. If cot (1 + sin ) = 4m and cot = 4n


(1) (2)
then which of the following is a correct?

;fn cot (1 + sin ) = 4m vkSj cot = 4n gS]


(3) (4) rks fuEufyf[kr esa ls dkSu&lk ,d lgh gS\
(1) (m2 + n2)2 = mn (2) (m2 – n2)2 = mn
(3) (m2 – n2)2 = m2n2 (4) (m2 + n2)2 = m2n2
36. Find the value of
41. Find the value of × ?
dk eku Kkr djsaA

× dk eku Kkr djsa ?


(1) (2)

BLAM–205
f=kdks.kfefr
(1) 7 (2) –14 47. If and are complementary angles, then
(3) 14 (4) –7

what is equal
42. Find the value of + +
to.
;fn vkSj iwjd dks.k gS] rks
+ + dk eku Kkr djsaA

fdlosQ cjkcj gS\


(1) × tan 108° (1 – cot 18° ˜ tan 4°)
(1) 0 (2) 1
(3) 2 (4) None of these
(2) × tan 104° (1 + tan 72° ˜ tan 4°)
48. If x sin = y cos = then 4z2 (x2 + y2) is

(3) × tan 104° (1 + cot 18° ˜ tan 4°) equal ;

;fn x sin = y cos = gS] rks 4z2 (x2 + y2)


(4) × tan 104° (1 – cot 18° ˜ tan 4°)
fdlosQ cjkcj gS\
43. If 3(sin x – sin y) = (cos y – cos x) then find the (1) (x2 + y2)3 (2) (x2 – y2)2
2 2 3
(3) (x – y ) (4) (x2 + y2)2
49. If sin x + sin x = 1 then, the value of cos8x + 2 cos6
2
value of
x + cos4 x is;
;fn sin x + sin2 x = 1 gS] rks cos8x + 2 cos6 x + cos4 x
dk eku D;k gS\
;fn 3(sin x – sin y) = (cos y – cos x) rks rks (1) 0 (2) 1
dk eku Kkr djsa \ (3) 2 (4) 4
50. What is the value of
(1) 2 (2) 0
(3) –1 (4) 3 + sin2 tan3 ?
44. tan (270 + T) ˜ cos (90 – T) + tan (180 + T) ˜ sin (90
4 4

– T) = ?
+ sin2 tan3 dk eku
(1) (2)
D;k gS\
(1) cos2 (2) sin2
(3) tan2 (4) cot sin2
(3) (4)

45. If sin (a + b) ˜ cos (c – d) = sin (a – b) ˜ cos (c + d)


then,
1. (2) 2. (4) 3. (3) 4. (4) 5. (1)
;fn sin (a + b) ˜ cos (c – d) = sin (a – b) ˜ cos (c + d) rks,
6. (3) 7. (1) 8. (2) 9. (1) 10. (2)
(1) tana ˜ tanc ˜ tand = cotb
11. (1) 12. (2) 13. (2) 14. (2) 15. (1)
(2) tana ˜ tanb ˜ tand = tanc
16. (4) 17. (1) 18. (2) 19. (1) 20. (2)
(3) tana ˜ tanb ˜ tanc = tand
21. (1) 22. (1) 23. (3) 24. (4) 25. (2)
(4) tana ˜ tanc ˜ tand = –cotb
26. (4) 27. (1) 28. (4) 29. (4) 30. (1)
31. (4) 32. (3) 33. (1) 34. (4) 35. (2)
46. is equal; foQlosQ rqY; gS\
36. (3) 37. (3) 38. (3) 39. (2) 40. (2)
41. (2) 42. (4) 43. (1) 44. (4) 45. (4)
(1) 1 (2) 2
46. (1) 47. (2) 48. (2) 49. (2) 50. (1)
(3) tan (4) cot

BLAM–206
f=kdks.kfefr

1. (2)

=
=

=
= =

3. (3) Here,
= cos T cos(120° + T) + cos(T – 120°) = 0
Let cos T= a , cos(120° + T) = b , cos( T– 120°) = c
= a+b+c=0
a3 + b3 + c3 = 3abc
cos3 T cos3 (120° + T) + cos3(T – 120°)
=
= 3. cos T cos (120° + T) . cos (T– 120°)
=
= cos T 2
. cos (120° + T) cos (T– 120°)
=

= = cos T cos (120° + T + T – 120°) + cos (1 20°

= 2 cos T + T –T 120°)]


Aliter : 2 cos A cos B
Put T = 0 = cos (A + B) + cos (A – B)

= cos T cos 2 T + cos 240°]

= cos cos 2 + cos (180° + 60°)]

= cos cos 2 – cos 60°]

fodYiksa dh lgk;rk ls] fodYi (2) ls cos (180° + ) = – cos


2 cos 0°
= 2 cos 0°
= cos
=2

2. (4) Given, tan x =


= cos
According to question,

= cos [ 4cos2 – 3]

= [4cos3 – 3cos ]

=
= cos3

BLAM–207
f=kdks.kfefr
Aliter :
ekuk]T= 120° tan2 T =
cos 120° + cos (120° + 120°) cos (120° – 120°) = 0 we know that,

cos2T =

? cos3T + cos3 (120° + T) + cos3 (T – 120°)


=
= cos3 120° + cos3 (120° + 120°) + cos3 (120° –
120°)

=
=

cos2T =
fodYi] (Option) dh lgk;rk ls

cos3T = cos 3 × 120°


5. (1) cot 7 =

= cos 360°

( cos 360° = cos (270 + 90°) = sin 90° = 1)

= =

vr% fodYi (3) mÙkj gksxkA


4. (4) Here,
m tan ( – 30°) = n tan( + 120°) =

= ×
=

=
=

[ tan 120° = – ] =

m (3 tan2 – 1) = n (tan2 – 3)
2 2 =
3m tan – m = n tan – 3n
tan2 T (3m – n) = m – 3n

BLAM–208
f=kdks.kfefr
Aliter :
= ge tkurs gS,a
tanT˜tan (60 – T) tan (60 – T) = tan 3T
Here, T= 20°
= ? tan 3 × 20° = tan 60°
=
=
6. (3) tan 20°. tan 40°. tan 80° 7. (1) tan A = , tan B =

= we know that,

tan (A – B) =
=

=
=

=
=

=
=

tan (A – B) = 1

= A – B = 45° =

8. (2) sinx + siny = a ...(i)


= cosx + cosy = b ...(ii)
equation (i) rFkk (ii) dks square djos Q add djus ij]
a2 + b2 = (sinx + siny)2 + (cosx + cos y)2
a2 + b2 = sin2x + sin2y + 2sinx . siny + cos2x + cos2y
+ 2 . cosx . cosy
a2 + b2 = (sin2x + cos2x) + (sin2y + cos2y) +
2[sinx sin y + cosx . cosy]
=
a2 + b2 = 1 + 1 + 2 cos (x – y)
[ cos (A – B) = cos A cos B + sin A sin B]
a2 + b2 = 2(1 + cos (x – y)]

cos (x – y) = (a 2 + b2 – 2)
=
9. (1)

=
=

BLAM–209
f=kdks.kfefr
12. (2) sin2 (120° – A) + sin2A + sin2 (120° + A)

= + +

[ cos ( – T) = – cos T]
= (1 + cos 18°) (1 + cos 54°) (1– cos 54°) (1– cos 18°)
(1 – cos2 18°) (1 - cos2 54°)
sin2 18° sin2 54°
sin2 18° [sin (90 – 36°)]2 [cos(240° – 2A) + cos2A + cos(240° + 2A]
sin2 18° cos2 36°
[ sin (90° –T) = cos T]
[cos(240° – 2A) + cos(240° + 2A) + cos2A]

= =

10. (2) Given,


sin A = x sin B

By componaendo & dividend O

= [2cos 240° . cos(–2A) + cos2A]

= [2cos (180° + 60°) cos2A + cos2A]

= [–2cos 60° cos2A + cos2A]

=
tan

11. (1) tan 75° = tan (45° + 30°)


=

=
=

Aliter :
= = Let, A = 120°
= sin2 (120° – 120°) + sin2 120° + sin2 (120° + 120°)

=
= =

= =2+

BLAM–210
f=kdks.kfefr
13. (2) cos 20° + cos 100° + cos 140° Here, T = 20°

= cos 20° + 2 cos .cos cos 3 × 20° ˜ cos 60° = cos2 60°

= cos 20° + 2 cos cos (–20°)

= cos 20° + 2cos 120° cos 20° 15. (1) Given,


cos A = m cos B

= cos 20° + 2cos (180° – 60°) cos 20°


= cos 20° – 2cos60° cos20°

= cos 20° – cos 20° = 0


14. (2) cos 20° . cos 40° cos 60°. cos 80°

=
16. (4)

= =

Aliter :
We know that
ge tkurs gSa]

cos T˜cos (60 – T) cos (60 + T) = cos3T = tan 4A

BLAM–211
f=kdks.kfefr

17. (1) cot x =

tan x =

19. (1) y + z = a cos2 x + 2 b sin x.cosx + c sin2 x + a sin2


2
a cos2x + b sin2x = a(2cos x – 1) + b 2sinx cosx x – 2b sin x. cos x + C cos2 x
= a(sin2 x + cos2 x) + C (sin2 x + cos2 x)
= a+c

20. (2) sin 12°. sin48°. sin 54°


=
=

= =

=
= +

= =

=
=

=
= .

=a
18. (2) Given 21. (1) sin2x = n sin2y

componen do dividendo, ds }kjk


and
=

Now,
sin2 y + cos2 y = 1 =

dividing Both sides by cos2 x

BLAM–212
f=kdks.kfefr

= tan (x + y) cot (x – y)

= = (K + 2)2 + 2 – 2

22. (1) a2 cosec2x – b2 cot2 x = c2 (3)2 = (K + 2)2


a2 cosec2x – b2 (cosec2x – 1) = c2 K+2=3
a2 cosec2x – b2 cosec2x + b2 = c2
(a2 – b2) cosec2x = c2 – b2
24. (4) sinT + cosT = x
sin2T + cos2T + 2sinT˜ cosT = x2
cosec2x =

sinT˜ cosT =
sin2x =
nksuksa rjiQ square djus ij]
? cos x = 1 – sin x
2 2

sin2T˜ cos2T =

=1– Ÿsin6T + cos6T + sin4T + cos4T


= (sin2T + cos2T)3 – 3 sin2T cos2T + (sin2T + cos2T)2 –
2sin2T˜ cos2T
= = 1 – 5 sin2x ˜ cos2x + 1
= 2 – 5 sin2x ˜ cos2x

= =2– 5

23. (3) sin2T + sec2T + 2sinT˜ secT + cos2T + cosec2T + 2


cosT˜ cosecT
=2–
= K2 + sec2T ˜ cosec2T + 2K secT ˜ cosecT + sin2T +
cos2T + 2sinT˜ cosT – sin2T – 1

= K2 + sec2T ˜

cosec2T + 2K secT˜ cosecT + 1 + sin2T – sin2T – 1

= K2 + sec2T˜ 25. (2) cos2A + cos2B + cos2C


= 2 cos (A + B) ˜ cos (A – B) + 1 – 2 sin2 C
cosec2T + 2KsecT˜ cosecT + 1 – 1
1 + sec2T ˜ cosec2T + 2 secT ˜ cosecT = K2 + sec2T ˜
cosx + cosy = 2cos cos
cosec2T + 2K secT˜ cosecT
nksuksa rjiQ dh rqyuk djus ij
= 2cos ˜ cos (A – B) – 2sin2C + 1

= 2cosC ˜ cos (A – B) – 2sin2C + 1


Aliter :
= 1 + 2sinC [cos (A – B) – sinc]
T= 45°
(sin 45° + sec 45°)2 + (cos 45° + cosec45°)2 ± (K +
sec45 cosec)2 + (sin 45 + cos 45°)2 – sin2 × 45° – 1 = 1 + 2 sinC

= 1 + 2sinC [cos (A – B) – cos (A + B)]


Ÿ = 1 + 2sinC ˜ 2sinA sinB
= 1 + 4 sinA sinB sinC

BLAM–213
f=kdks.kfefr

sin B = = sin 45°


26. (4)
B = 45°

cot A ˜ tan B =
cos2x – sin2x = cos2x

= cos (45° – 30°)


= cos 45° ˜ cos 30° + sin 45° ˜ sin 30° cot A tan 45° =
cos (x – y) = cosx cosy + sinx sin y

cot A =

? 7 cot2A + cot2B =

27. (1) a3 + b3 + c3 – 3abc = 0


if a3 + b3 + c3 – 3abc = 0, then =
a2 + b2 + c2 – ab – bc – ca = 0
=2
(a – b)2 + (b – 1)2 + (c – a)2 = 0
2a2 + 2b2 + 2c2 – 2ab – 2bc – 2ca = 0 29. (4) x = (1 – cos D)(1 – cos E) (1 – cos J) ...(i)
a – b = 0, a = b x = (1 + cos D) (1 + cos E) (1 + cos J) ...(ii)
b – c = 0, b = c lehdj.k (i) rFkk (ii) dks xq.kk djus ij
c – a = 0, c = a x2 = [(1 – cos D) (1 – cos D)] [(1 – cos E) (1 + cos E)] [(1
? a = b = c, its means – cos J)
'ABC is a equilateral triangle. x2 = (1 – cos2 D) (1 – cos2 E) (1 – cos2 J)
? ‘A = ‘B = ‘C = 60° x2 = sin2D sin2E sin2J
cos2A + cos2B + cos2C = 3cos2A x = ± sinD sinE sinJ
= 3 × cos2 60° D, E, J is acute angle triangle, (D, E, J < 90°)
So, D, E, J will be in first quadrant.
=3× 30. (1) cot D = ncot E

( cos J = m cos E)
28. (4)
n sin D = m sin E ...(i)
2cos A = cos B ...(i) 2 sin A = sin B ...(ii) cos D = m cos E ...(ii)

lehdj.k (1) o (2) dks oxZ djus tksMu+ s ij lehdj.k (i) rFkk (ii) dk oxZ djus tksMu+ s ij
2 2 2
4 cos A + 4sin A = cos B + 7 sin B 2 n2 sin2D + cos2D= m2sin2E + m2 cos2E
4(sin2A + cos2A) = 1 – sin2B + 7sin2B n2 sin2D + 1 – sin2D = m2 (sin2E + cos2E)
4 – 1 = 6 sin2B sin2E (n2 – 1) = m2 – 1
3 = 6 sin2B
sin2E =
2
sin B =

BLAM–214
f=kdks.kfefr
31. (4) 10 cos D + 15 sin D = 6
4 4
= cos 120° ˜ cos 70° ˜ cos 50° ˜ cos 30° ˜ cos 10°
(cos4E ls Hkkx nsus ij)
10 + 15 tan4D = 6 sec4D = ˜sin 20° ˜ sin 40° ˜ cos 30° ˜ sin 80°
10 + 15 tan4D = 6 (1 + tan2D)2
10 + 15 tan4D = 6 + 6tan4D + 12 tan4D
9 tan4D – 12 tan2D + 4 = 0
(3tan4D – 2)2 = 0
3tan2D = 2
= × sin 3 × 20° ×

tanD =

34. (4) 643cos T˜ 165sin T˜ 2562cos T˜ 10242sin T


= 412cos T˜ 410sin T˜ 48cos T˜ 410sin T
= 412cos T+ 10sin T+ 8cos T+ 10sin T
= 420sin T+20cos T

Maximum =
sec D = & cosec D =

Minimum =
? 81 sec8D + 16 cosec8D =
? Maximum and Minimum values = and

= 81 × + 16 ×

= 1250 = and
32. (3) ABC esa]
A+ B+ C = 180° 35. (2) ,

tan–1 2 + tan–1 3 + = 180°


= sin E ...(i) = cos E ....(ii)
tan–1 = 180° –
nksuksa lehdj.k dk oxZ djds tksM+us ij

tan–1 = 180° – = sin2E + cos2E

tan–1 (–1) = 180° –


–1 = tan (180° – )
–1 = –tan

sin2D ls Hkkx nsus ij


tan = 1 = tan

= cosec2D
=

33. (1) cos ˜ cos ˜ cos ˜ = 1 + cot2D

cos ˜ cos

BLAM–215
f=kdks.kfefr
sec2 A – tan2A = 1
cot2D=
? sec2A = 1 & tan2A = 1

cot D = sec2A = & tan2A =

– = –
36. (3) =

=
39. (2)

= Componendo dividendo ds }kjk]

2cos (x + y) = cos (x – y).


= 40. (2) Given,
cot (1 + sin ) = 4 m
putting = 45° cot (1 – sin ) = 4n
=

1
37. (3)

4m = n=

Componendo dividendo ds }kjk]


m=

m2 – n2 = (m + n) (m – n)

=
38. (3)

mn =

?(m2 – n2)2 = mn

BLAM–216
f=kdks.kfefr
44. (4) tan (270 + T) ˜ cos4 (90 – T) + tan (180 + T) ˜ sin4
(90 – T)
41. (2) = – cot T ˜ sin4T + tan T ˜ cos4T
= tan T ˜ cos4T + cot T ˜ sin4T

= sin T ˜ cos3T – cos T ˜ sin3T

= × 2 sin T ˜ cos T + (cos2T ˜ sin4T)

= –14 45. (4) sin (a + b) cos (c – d) = sin (a – b) ˜ cos (c + d)

42. (4)

Componendo dividendo ds }kjk

tana ˜ cotb = –cotc ˜ cotd


tana ˜ tanc ˜ tand = –cotb.
46. (1) Putting = 45°
A/q

= [tan12° – tan 4° + tan 36° – tan 12° + tan 108° –

tan 36°] =

= [tan 108° – tan 4°]

× (1 + tan 108° ˜ tan 4°) =

= × tan 104° (1 – cot 18° ˜ tan 4°]


=
43. (1) 3 sinx – 3 siny = cosy – cosx
47. (2) Given
+ = 90°
Now A/q

=2

BLAM–217
f=kdks.kfefr
from eqn (4)

=
z2 = k2

=
=

4z2 (x2 + y2) = 4


= = 1.
=
48. (2) Given,
49. (2) Given. the expression,
x sin = y cos = sin x + sin2 x = 1
sin x = 1 – sin2 x
x sin = K y cos =K
sin x = cos2 x
n = cosec ...(1) y = K sec
sin2 x = cos4 x ...(1)
...(2)
A/q.
we taking
cos8 x + 2 cos6x + cos4x
x sin = y cos
= (cos4 x + cos2 x)2
= (sin2 x + cos2 x)2 [using (1)] = 1
...(3)
50. (1) Given,
and

z=K = + sin2 .tan3

= + sin2.tan3
= K

= + sin2 .tan3

= K
=

= K ...(4)
=
From eqn (1) & (2)
x2 + y2 = K2 (cosec2 + sec2 )
= K2 (tan2 + cot2 + 2) =
= K2 (tan + cot )2

=
= K2
[using the formula 2 sin A. sin B
= cos (A – B) – cos (A + B)]

=
K2 =

= = cos2 .

‡‡‡
=

BLAM–218
mQapkbZ ,oa nwjh

4 m¡QpkbZ ,oa nwjh


(HEIGHT AND DISTANCE)
Height and Distance trigonometry dk ,d cgqr gh pwafd angle 60°, 30° ls cM+k gS blfy, point S tower
important part gSA bl ij based questions exam esa t:j osQ T;knk utnhd gS point R osQ eqdkcysA
iwNs tkrs gSaA (3) tc dk eku increase gksxk rc Point, Vertical line
„ Important Definition : dh rjiQ c<+sxh rFkk tc dk eku decrease gksxk rc
Point, Vertical line ls nwj gksrh tk;sxhA
Angle of Elevation (mUu;u dks.k) % tc ge uhps
ls mQij dh rjiQ fdlh object, building, tower, tree etc.
dks ns[krs gSa rks gekjh vk¡[kksa dk ground osQ lkFk tks angle
curk gS mls angle of elevation dgrs gSaA
Note : Angle of e levation ges'kk ground ;k ground
osQ parallel line osQ lkFk curk gSA 3 2 1

1 < 2 < 3
;gk¡, Angle of elevation gksxk CAB
C

3 2 1

T „ Some Important Rules :


B (i) fdlh Hkh h m¡QpkbZ ds tower ds lkFk fdUgha nks points ds
A
Angle of Depression (voueu dks.k) : tc ge fdlh }kjk cuk;k x;k angles of elevation T1and T2 gks] rks]
m¡Qps LFkku ij [kM+s gks dj uhps fdlh object tSls car, bus, points ds chp dh nwjh
building etc. dks ns[krs gSa rks angle of depression curk gSA AB = d = h (cot T1 – cotT2)
Note : Angle of depression Hkh ground ;k ground
ls parallel line osQ lkFk fy;k tkrk gSA P
Angle of depression = ABC= ZCB
C
Z
T h

T2 T
T R A d B

„ Important points :
A
B
(ii) h m¡QpkbZ ds tower ds nksuksa rjiQ dksbZ nks points fLFkr gSA
(1) Height and Distance ij based questions djus osQ ;fn nksuksa points tower ds top ds lkFk angles of el-
fy, trigonometric table ftlesa angle (0° ls 90°) dh evation T1 and T2 cukrs gSa rks] nksuksa points ds chp dh
values nh xbZ gS ;kn gksuk cgqr t:jh gSA nwj h
(2) ftruk cM+k angle gksrk gS og mruk gh perpendicular d = h (cot T1 + cotT2)
osQ utnhd gksrk gSA
P
For ex. dksbZ nks points tower osQ lkFk 30° vkSj 60°
dk angle cukrs gSa rks budh position bl izdkj gksxhA
Q
h

T T
B R A
d
30° 60°
P
R S

BLAM–219
mQapkbZ ,oa nwjh
(iii) fdlh Hkh tower ds vk/kj ls a rFkk b nwjh ij fLFkr fdUgh Ratio Method :
nks points ds }kjk tower ds top ls cus angles of = 30°, 45° rFkk 60° Ratio Method esa fofHkUu Figure
elevation ,d nwljs ds complementary gks rks] tower esa iz;ksx
dh m¡QpkbZ
h = ab
1 1

P 45° 35° 30° 45°


1 K– e 3-1 j 3 1

K 3 K e 3 +1 j
h Fig (i) Fig (ii)

e 3 j
1
90–T T
R a A B 1
b
3 45°
(iv) nks towers (equal or unequal heights) ds tops ds 30°
}kjk ,d nwljs ds base ij cus angle of depression 1
,d nwljs ds complementary gks rks] Fig-(iii)

d = h1h 2
3
3
C
60° 45° 60° 45°
A 1 K e 3-1 j 1 3
K 3
K e 3 +1 j
Fig (iv) Fig (v)

h1 h2
e 3 –1 j

3
T 90°–T
C D 1
60°
d 45°
K 1

(v) nks towers (equal or unequal heights) ds tops ds Fig (vi)

}kjk muosQ chp esa fdlh fcUnq ij cus angles of depres-


sion ,d nwljs ds complementary gks rks] tower ds
3 3
chp dh nwjh]
d = 2 h1 h 2 60° 30° 60° 30°
1 2 1 3
D K 3 K 4

Fig (vii) Fig (viii)


A
2

h1 h2
1
60°
30°
T 90°–T
K 3
C P C

‰‰‰
d Fig (ix)

BLAM–220
mQapkbZ ,oa nwjh

PREVIOUS YEARS’ QUESTIONS


1. The angle of elevation of a tower from a distance 5. The angle of elevation of a ladder leaning against
50 m from its foot is 30°. The height of the tower a house is 60° and the foot of the ladder is 6.5
is metres from the house. The length of the ladder
,d ehukj dk mlds ewy vk/kj (ikn) ls 50 ehVj nwjh ls is
mUu;u dks.k 30° gSA rnuqlkj] ml ehukj dh mQ¡pkbZ fdruh ,d edku ds lkeus frjNh yxh lh<+h dk mUu;u dks.k 60° gS
gksxh\ vkSj lh<+h dk v/ksHkkx (ikn) edku ls 6.5 ehVj nwjh ij gSA
50 lh<+h dh yEckbZ fdruh gS \
(1) 50 3m (2) m
3 13
(1) metres (2) 13 metres
75 3
(2) 75 3 m (4) m
3 (3) 15 metres (4) 3.25 metres
(SSC Graduate Level Tier-II Exam. 29.09.2013 (SSC CAPFs SI, CISF ASI & DP SI Exam. 22.06.2014)
2. One flies a kite with a thread 150 metre long. If 6. A kite is flying at the height of 75m from the
the thread of the kite makes an angle of 60° with ground. The string makes an angle T (where cotT
the horizontal line, then the height of the kite
from the ground (assuming the thread to be in a 8
= ) with the level ground. Assuming that there
straight line) is 15
dksbZ 150 ehVj yacs /kxs ds lkFk irax mM+krk gSA ;fn irax dk is no slack in the string the length of the string is
/kxk {kSfrt js[kk ds lkFk 60° dk dks.k cuk, rks Hkwfe ls irax equal to :
dh mQ¡pkbZ (;g eku dj fd /kxk lh/h js[kk esa gS) gSµ ,d irax Hkwfe ls 75 ehVj dh Å¡pkbZ ij mM+ jgh gSA mldh Mksjh
(1) 50 metre (2) 75 3 metre 8
Hkwfe ds Lrj ls T dk dks.k cuk jgh gS (blesa cotT = 15 gS)
(3) 25 3 metre (4) 80 metre
FCI Assistant Grade-III Exam. 25.02.2012 (Paper-I)
eku ysa fd Mksjh esa dksbZ <hy ugha gS rks Mksjh dh yEckbZ fdruh
North Zone (Ist Sitting) gS \
3. A vertical post 15 ft high is broken at a certain (1) 85 metre (2) 65 metre
height and its upper part, not completely
(3) 75 metre (4) 40 metre
separated, meets the ground at an angle of 30°.
(SSC CGL Tier-I Exam, 16.08.2015
Find the height at which the post is broken. (Ist Sitting) TF No. 3196279)
15 iQhV mQ¡pk ,d [kM+k [akHkk dqN mQ¡pkbZ ij VwV x;k vkSj 7. A person of height 6ft. wants to pluck a fruit which
mldk mQijh Hkkx] tks iwjh rjg vyx ugha gqvk] i`Foh dks 30°
26
ds dks.k ij Li'kZ djrk gSA [kaHkk fdruh mQ¡pkbZ ij VwVk gS\ is on a
3
ft. high tree. If the person is standing
(1) 10 ft (2) 5 ft
8
e
(3) 15 3 2 – 3 ft j (4) 5 3 ft
3
ft. away from the base of the tree, then at
(SSC CHSL DEO & LDC Exam. 04.11.2012 (IInd Sitting)
4. The angle of elevation of a tower from a distance what angle should he throw a stone so that it
of 100 metre from its foot is 30°. Then the height hits the fruit ?
of the tower is 26
ehukj ds ikn ls 100 ehVj dh nwjh ij fLFkr ,d fcUnq ls ,d ,d O;fDr ftldk dn 6 iqQV gS 3 iqQV Å¡ps o`{k ls iQy
ehukj ds 'kh"kZ dk mUu;u dks.k 30° gSA ehukj dh Å¡pkbZ gS &
8
rksM+uk pkgrk gSA ;fn O;fDr o`{k ds rus ds vk/kj ls 3
iqQV
(1) 50 3 metre (2) 100 3 metre
nwjh ij [kM+k gS rks mls fdl dks.k ij iRFkj isaQduk pkfg, ftlls
50 100
(3) metre (4) metre og iQy ij tkdj yxs \
3 3
(1) 75° (2) 30° (3) 45° (4) 60°
(SSC CGL Tier-I Exam, 09.08.2015
(SSC CGL Tier-I Exam, 09.08.2015
(IInd Sitting) TF No. 4239378)
(IInd Sitting) TF No. 4239378)

BLAM–221
mQapkbZ ,oa nwjh
8. From the top of a light-house at a height 20 (1) 125 3 (2) 125( 3 – 1)
metres above sea-level, the angle of depression of
a ship is 30°. The distance of the ship from the (3) 125/( 3 – 1) (4) 125( 3 + 1)
foot of the light house is
(SSC CHSL DEO & LDC Exam. 10.11.2013, Ist Sitting)
leqæ ry ls 20 ehVj Å¡pkbZ ij ,d ykbV&gkml ds 'kh"kZ ls ,d 12. From the top of a hill 200 m high, the angle of
tgkt dk voueu dks.k 30° gSA ykbV&gkml ds v/ksHkkx ls depression of the top and the bottom of a tower
tgkt dh nwjh gS are observed to be 30° and 60°. The height of the
tower is (in m) :
(1) 20 m (2) 20 3 m 200 ehVj mQ¡pkbZ okyh ,d igkM+h dh pksVh ls] ,d ehukj ds
(3) 30 m (4) 30 3 m 'kh"kZ rFkk ry ds voueu dks.k 30° rFkk 60° ekis x,A
(SSC CAPFs SI, CISF ASI & Delhi Police SI
rnuqlkj] ml ehukj dh mQ¡pkbZ (ehVj esa) fdruh gS\
Exam. 22.06.2014 TF No. 999 KP0)
400 3 2 1
9. The length of the shadow of a vertical tower on (1) (2) 166 (3) 133 (4) 200 3
level ground increases by 10 metres when the 3 3 3
altitude of the sun changes from 45° to 30°. Then, (SSC CAPFs SI & CISF ASI Exam. 23.06.2013)
the height of the tower is 13. The angle of elevation of the top of a vertical tower
Hkwfe ds Lrj ij ,d ÅèoZ (ofVZdy)ehukj dh ijNkb± dh yEckbZ] situated perpendicularly on a plane is observed
as 60° from a point P on the same plane. From
ml le; 10 ehVj c<+ tkrh gS tc lw;Z dh rqaxrk (Å¡pkbZ) 45° another point Q, 10m vertically above the point
ls 30° ij ifjofrZr gksrh gSA ehukj dh Å¡pkbZ crk,¡A P, the angle of depression of the foot of the tower
is 30°. The height of the tower is
(1) 5 ( 3 + 1) metres (2) 5 ( 3 – 1) metres
fdlh eSnku ij yacor~ fLFkr mQèokZ/j ehukj ds 'kh"kZ dh mQ¡pkbZ
5 dk dks.k mlh eSnku ds P fcanq ls 60° fn[kkbZ nsrk gSA P,fcanq
(3) 5 3 metres (4) metres
3 ls mQèokZèkj 10 ehVj mQij Q fcanq ls ehukj ds ikn dh voufr
(SSC CHSL DEO & LDC Exam. 9.11.2014) dk dks.k 30° gSA ehukj dh mQ¡pkbZ fdruh gS \
10. An aeroplane when flying at a height of 5000m (1) 15 m (2) 30 m
from the ground passes vertically above another (3) 20 m (4) 25 m
aeroplane at an instant, when the angles of (SSC CGL Tier-I Exam. 19.10.2014)
elevation of the two aeroplanes from the same 14. There are two vertical posts, one on each side of a
point on the ground are 60° and 45° respectively. road, just opposite to each other. One
The vertical distance between the aeroplanes at post is108 metre high. From the top of this post,
that instant is the angle of depression of the top and foot of the
Hkwry ls 5000 eh- dh Å¡pkbZ ij mM+rk gqvk ,d foeku] ,d other post are 30° and 60° respectively. The height
of the other post (in metre) is
{k.k dks ,d vU; foeku osQ ÅèokZèkj Åij mM+rk gS] vkSj rc
,d lM+d ds nksuksa rjiQ ,d nwljs ds lkeus nks mQèokZ/j [kaHks
Hkwry osQ ,d fcUnq ls nksuksa foekuksa osQ mUu;u dks.k Øe'k% 60°
gSaA muesa ,d 108 ehVj mQ¡pk gSA bl [kaHks ds 'kh"kZ ls nwljs [kaHks
rFkk 45° gks tkrs gSaA rnuqlkj] ml {k.k] nksuksa foekuksa osQ chp
ds 'kh"kZ ,oa v/ksHkkx ds voueu dks.k Øe'k% 30° rFkk 60°
dh ÅèokZèkj nwjh fdruh gS\
gSA rnuqlkj nwljs [kaHks dh mQ¡pkbZ fdrus ehVj gksxh \
(1) 5000( 3 1) m (2) 5000(3 3) m (1) 36 (2) 72 (3) 108 (4) 110
(SSC CHSL DEO & LDC Exam. 11.12.2011
FG 1 IJ (Ist Sitting (East Zone)
(3) 5000 1 15. From the peak of a hill which is 300 m high, the
H 3 Km (4) 4500 m
angle of depression of two sides of a bridge lying
(SSC Graduate Level Tier-II Exam. 16.09.2012) on a ground are 45° and 30° (both ends of the
11. From a tower 125 metres high, the angle of bridge are on the same side of the hill). Then, the
depression of two objects, which are in horizontal length of the bridge is
line through the base of the tower, are 45° and ,d 300 eh- mQ¡ps ioZr f'k[kj ls Hkwfe ij cus lsrq ds nksuksa ik'oZ
30° and they are on the same side of the tower. ds voueu dks.k 45° vkSj 30° gSa vkSj lsrq ds nksuksa ik'oZ ioZr
The distance (in metres) between the objects is
ds ,d gh rjiQ gSaA rks lsrq dh yEckbZ gS
125 ehVj mQ¡ph ,d ehukj ls] nks oLrqvksa] tks ehukj ds vkèkkj
(1) 300( 3 – 1) m (2) 300( 3 + 1) m
ls {kSfrt js[kk esa gSa] ds voueu dks.k 45° rFkk 30° gSaA rnuqlkj]
;fn os oLrq,¡ ehukj ds ,d gh vksj gksa] rks muds chp dh nwjh 300
(3) 300 3 m (4) m
fdrus ehVj gksxh\ 3

BLAM–222
mQapkbZ ,oa nwjh
(SSC CAPFs SI, CISF ASI & Delhi Police SI Exam. 22.06.2014)
16. The angle of elevation of an aeroplane from a point 3 3 3
(1) (2) (3) (4) 3
on the ground is 60°. After 15 seconds flight, the 2 3 4
elevation changes to 30°. If the aeroplane is flying (SSC CGL Tier-II Exam, 2014 12.04.2015
(Kolkata Region) TF No. 789 TH 7)
at a height of 1500 3 m, find the speed of the
20. The two banks of a canal are straight and parallel.
plane
A, B, C are three persons of which A stands on
,d lery eSnku ds ,d fcUnq ls ,d foeku dk mUu;u dks.k one bank and B and C on the opposite banks. B
60° gSA 15 lsd.M dh mM+ku ds ckn] mUu;u dks.k 30° gks tkrk finds the angle ABC is 30°, while C finds that the
angle ACB 60°. If B and C are 100 metres apart,
gSA rnuqlkj] ;fn foeku 1500 3 eh- dh mQ¡pkbZ ij mM+ jgk
the breadth of the canal is
gks] rks mldh xfr Kkr dhft,A ,d ugj ds nksuksa rV lh/s vkSj lekarj gSaA A, B, C rhu O;fDr
(1) 300 m/sec (2) 200 m/sec
gSa ftuesa ls A ,d rV ij [kM+k gS vkSj B ,oa C lkeus ds rV
(3) 100 m/sec (4) 150 m/sec
(SSC Delhi Police S.I. (SI) Exam. 19.08.2012)
ij [kM+s gSaA B dks yxrk gS fd dks.k ABC, 30° dk gS tcfd
17. The distance between two vertical poles is 60 m. C dks yxrk gS fd dks.k ACB , 60° dk gSA ;fn B vkSj C,
The height of one of the poles is double the height 100 ehVj nwjh ij gSa rks ugj dh pkSM+kbZ fdruh gS \
of the other. The angle of elevation of the top of
the poles from the middle point of the line segment 25
joining their feet are complementary to each other. (1) metres (2) 20 3 metres
3
The height of the poles are :
nks mQèokZ/j [kaHkksa ds chp dh nwjh 60 ehVj gSA muesa ,d [kaHks 20
(3) 25 3 metres (4) metres
dh mQ¡pkbZ] nwljs ls nqxquh gSA mu [kaHkksa ds vk/kjksa dks tksMu+ s okyh 3
js[kk ds eè; fcanq ls mu [kaHkksa ds 'kh"kZ fcUnqvksa ds mUu;u dks.k (SSC CAPFs SI, CISF ASI & Delhi Police SI
Exam, 21.06.2015 IInd Sitting)
ijLij iwjd gSaA rnuqlkj mu [kaHkksa dh mQ¡pkbZ fdruh gS\
21. Two towers A and B have lengths 45m and 15m
(1) 10 m and 20 m (2) 20 m and 40 m respectively. The angle of elevation from the
(3) 20.9 m and 41.8 m (4) 15 2 m and 30 2 m bottom of the tower B to the top of the tower A is
(SSC CHSL DEO & LDC Exam. 21.10.2012 (IInd Sitting) 60°. If the angle of elevation from the bottom of
18. From two points on the ground and lying on a tower A to the top of the tower B is T then value
straight line through the foot of a pillar, the two of sinT is :
angles of elevation of the top of the pillar are A vkSj B nks ehukjksa dh yackbZ Øe'k% 45 ehVj vkSj 15 ehVj
complementary to each other. If the distances of
the two points from the foot of the pillar are 12 gSA ehukj B ds ry ls ehukj A ds 'kh"kZ dk mÂ;u dks.k 60°
metres and 27 metres and the two points lie on gSA ;fn ehukj A ds ry ls ehukj B ds 'kh"kZ dk mÂ;u dks.k T
the same side of the pillar, then the height (in gS rks sin T dk eku gS&
metres) of the pillar is
1 1 2
tehu ij vkSj [kaHks ds ikn ls gksdj ljy js[kk ij iM+s nks fcUnqvksa (1) (2) (3)
3
(4)
ls [kaHks ds 'kh"kZ ds mÂ;u dks.k ,d nwljs ds iwjd gSaA ;fn [kaHks 2 2 2 3
ds ikn ls nksuksa fcUnqvksa dh nwjh 12 ehVj vkSj 27 ehVj gS vkSj (SSC CGL Tier-I Exam, 16.08.2015
(IInd Sitting) TF No. 2176783)
nksuaksa fcanq [kaHks dh leku Hkqtk dh vksj iM+rs gSa rks [kaHks dh yackbZ 22. The top of two poles of height 24 m and 36 m are
(ehVj esa) fdruh gksxh \ connected by a wire. If the wire makes an angle of
(1) 12 (2) 18 (3) 15 (4) 16 60° with the horizontal, then the length of the
(SSC CAPFs SI, CISF ASI & Delhi Police SI wire is
Exam, 21.06.2015 (Ist Sitting) TF No. 8037731)
19. From an aeroplane just over a straight road, the
24 eh- vkSj 36 eh- mQ¡pkbZ okys nks [kaHkksa ds f'k[kj ,d rkj ls
angles of depression of two consecutive kilometre tksMs+ x, gSaA ;fn rkj lery ds lkFk 60° dk dks.k cukrh gS]
stones situated at opposite sides of the aeroplane rks rkj dh yackbZ gSµ
were found to be 60° and 30° respectively. The
height (in km) of the aeroplane from the road at (1) 6 m (2) 8 3 m (3) 8 m (4) 6 3 m
that instant, is (SSC GL Tier-I Exam. 19.05.2013 Ist Sitting)
23. The distance between two pillars of length 16
,d foeku ls Bhd lh/h lM+d ij] foeku dh foijhr fn'kkvksa
metres and 9 metres is x metres. If two angles of
esa fLFkr nks vuqØfed fdyksehVj iRFkjksa ds voueu dks.k Øe'k% elevation of their respective top from the bottom
60° vkSj 30° gSaA ml le; foeku dh lM+d ls Å¡pkbZ (fdeh of the other are complementary to each other, then
esa) fdruh gS \ the value of x (in metres) is

BLAM–223
mQapkbZ ,oa nwjh
16 ehVj vkSj 9 ehVj yacs nks [kaHkksa ds chp dh nwjh x ehVj gSA ,d ehukj ds vk/kj ls ,d fn'kk ds nks fcanq P rFkk Q Øe'k%
;fn mu [kaHkksa ds 'kh"kZ ds vius lkeus okys [kaHks ds vèkksHkkx ds ‘a’ rFkk ‘b’ nwjh ij gSaA mu fcanqvksa ls ehukj ds 'kh"kZ ds mUu;u
lkFk cus mUu;u dks.k ijLij iwjd dks.k gksa] rks x dk eku fdrus dks.k ijLij iwjd gSaA rnuqlkj ml ehukj dh mQ¡pkbZ fdruh gS\
ehVj gksxk\ a
(1) 15 (2) 16 (3) 12 (4) 9 (1) ab (2)
b
(SSC CHSL DEO & LDC Exam. 04.12.2011
(IInd Sitting (North Zone) (3) ab (4) a2b2
24. The angle of elevation of the top of a building (SSC FCI Assistant Grade-III Main Exam. 07.04.2013)
from the top and bottom of a tree are x and y 28. If the angle of elevation of a balloon from two
respectively. If the height of the tree is h metre, consecutive kilometre-stones along a road are 30°
then (in metre) the height of the building is and 60° respectively, then the height of the
balloon above the ground will be
,d Hkou ds 'kh"kZ ds lkFk ,d isM+ ds 'kh"kZ ,oa v/ksHkkx ls
mUu;u dks.k Øe'k% x vkSj y gSaA rnuqlkj ;fn ml isM+ dh ;fn ,d lM+d dh nks Øfed fdyksehVj f'kykvksa ls ,d xqCckjs
ds mUu;u dks.k Øe'k% 30° rFkk 60° gksa] rks i`Foh&ry ls ml
mQ¡pkbZ h ehVj gks] rks ml Hkou dh mQ¡pkbZ fdrus ehVj gS\
xqCckjs dh mQ¡pkbZ fdruh gksxh \
h cot x h cot y
(1) cot x cot y (2) cot x cot y 3 1 2
(1) km (2) km (3) km (4) 3 3 km
2 2 3
h cot x h cot y (SSC Graduate Level Tier-I Exam. 19.05.2013)
(3) cot x cot y (4) cot x cot y
29. A Navy captain going away from a lighthouse at
(SSC CHSL DEO & LDC Exam. 04.12.2011
(Ist Sitting (East Zone) the speed of 4 e 3j – 1 m/s. He observes that it
25. The angle of elevation of the top of a building and
takes him 1 minute to change the angle of
the top of the chimney on the roof of the building
elevation of the top of the lighthouse from 60° to
from a point on the ground are x and 45° 45°. What is the height (in metres) of the
respectively. The height of building is h metre. lighthouse ?
Then the height of the chimney, (in metre) is :
,d Hkou osQ 'kh"kZ vkSj Hkou osQ Nr ij fLFkr fpeuh osQ 'kh"kZ ,d ukSlsuk dIrku 4 e 3j – 1 eh-@ls- dh xfr ls ykbV
dk tehu osQ fdlh fcUnq ij mUu;u dks.k Øe'k% x rFkk 45° gkmQl ls nwj tk jgk gSA og fujh{k.k djrk gS fd ykbV gkmQl
gSA Hkou dh m¡QpkbZ h ehVj gS rks fpeuh dh m¡QpkbZ ehVj esa dh pksVh dk mUu;u dks.k 60° ls 45° cnyus ds fy, mls ,d
fdruh gS\ feuV yxrk gSA ykbV gkmQl dh mQ¡pkbZ (ehVj eas) D;k gS\
(1) h cot x + h (2) h cot x – h
(3) h tan x – h (4) h tan x + h (1) 240 3 (2) 480 e 3j – 1
(SSC CHSL DEO & LDC Exam. 11.12.2011
(IInd Sitting (East Zone) (3) 360 3 (4) 280 2
26. Two posts are x metres apart and the height of 30. A balloon leaves from a point P rises at a uniform
one is double that of the other. If from the mid- speed. After 6 minutes, an observer situated at a
point of the line joining their feet, an observer
finds the angular elevations of their tops to be distance of 450 3 metres from point P observes
complementary, then the height (in metres) of that angle of elevation of the balloon is 60°. Assume
the shorter post is that point of observation and point P are on the
same level. What is the speed (in m/s) of the
nks [kaHks x ehVj dh nwjh ij gSaA muesa ,d dh Å¡pkbZ] nwljs dh balloon?
nqxquh gSA rnuqlkj] ;fn muosQ ryksa dks tksM+us okyh js[kk osQ ,d xqCckjk ,d leku xfr ls fcUnq P ls NksM+k tkrk gSA 6 feuV
eè;fcUnq ls ,d i;Zo s{kd] muosQ 'kh"kks± osQ mUu;u dks.k ijLij
i'pkr~] fcUnq P ls 450 3 ehVj nwjh ij ,d leh{kd gS tks
iwjd ikrk gS] rks NksVs [kaHks dh Å¡pkbZ fdrus ehVj gS\
ns[krk gS fd xqCckjs dk mUu;u dks.k 60° gSA eku yhft, fd
x x x leh{kk fcUnq rFkk fcUnq P ,d gh Lrj ij gSaA xqCckjs dh xfr
(1) (2) (3) x 2 (4)
2 2 4 2 (ehVj/lsdMa esa) D;k gS\
(SSC Graduate Level Tier-II Exam. 16.09.2012) (1) 4.25 (2) 3.75 (3) 4.5 (4) 3.45
27. The angle of elevation of the top of a tower from 31. A ladder is placed against a wall such that it just
the point P and Q at distance of ‘a’ and ‘b’ reaches the top of the wall. The foot of the ladder
respectively from the base of the tower and in the is at a distance of 5 metres from the wall. The
same straight line with it are complementary. The angle of elevation of the top of the wall from the
height of the tower is base of the ladder is 15°. What is the length (in

BLAM–224
mQapkbZ ,oa nwjh
metres) of the ladder? h ehVj gSA fpeuh dh yEckbZ (ehñ) crkb,µ
,d lh<+h nhokj ds lgkjs bl izdkj [kM+h gS fd og nhokj dh (1) h cot x + h (2) h cot x – h
pksVh rd igq¡prh gSA lh<+h dk vk/kj nhokj ls 5 ehVj dh nwjh (3) h tan x – h (4) h tan x + h
ij gSA lh<+h ds vk/kj ls nhokj dh pksVh dk mUu;u dks.k 15° (SSC CHSL DEO & LDC 2011)

gSA lh<+h dh yEckbZ (ehVj eas)D;k gS\ 35. The distance between two vertical poles is 60 m.
The height of one of the poles is double the height
(1) 5 6 – 5 3 (2) 5 6 – 5 2 of the other. The angle of elevation of the top of
the poles from the midde point of the line segment
(3) 5 2 – 1 (4) 5 3 + 5 2 Joining their feet are complementary to each
(SSC CGL Tier-II (CBE) Exam. 09.03.2018) other. The height of the poles are :
32. From a point C, the angle of elevation of a tower nks ÅèokZ/j [kEHkksa osQ chp dh nwjh 60 ehVj gSA muesa ,d [kEHks
3 dh Å¡pkbZ nwljs [kEHks dh Å¡pkbZ ls nqxquh gSA mu [kEHkksa osQ
is such that its tangent is . On walking 560
4 vkèkkjksa dks tksMu+ s okyh js[kk osQ eè; fcUnq mu [kEHkksa osQ 'kh"kZ
metres towards the tower the tangent of the angle fcUnqvksa osQ mUu;u dks.k ijLij iwjd gSA rks mu [kEHkksa dh Å¡pkbZ
4 fdruh gSA
of elevation of the tower becomes . What is the
3 (1) 10 m and 20 m
height (in metres) of the tower ? (2) 20 m and 40 m
,d fcUnq C ls] ,d ehukj dk mÂ;u dks.k bl izdkj gS dh (3) 20.9 m and 41.8 m
3 (4) 15 2m and 30 2 m
mldh Li'kZT;k (VsutsaV) 4 gSA ehukj dh vksj 560 ehVj
(SSC CHSL DEO & LDC 2012)
4 36. From an Aeroplan just over a river, the angles of
pyus ij ehukj ds mÂ;u dks.k dh Li'kZT;k (VsutsaV) 3
gks depression to the bottom of trees on the opposite
bank of the river are found to be 60° and 30°
tkrh gSA ehukj dh mQ¡pkbZ (ehVj eas) D;k gS\ respectively. If the breadth of the river is 400
(1) 720 (2) 960 metres, then the height of the aeroplane above
(3) 840 (4) 1030
(SSC CGL Tier-II (CBE) Exam. 18.02.2018) the river at that instant is (Assume 3 1732
. )
33. At a point on a horizontal line through the base fdlh unh osQ Åij fLFkr ,d gokbZ tgkt ls unh osQ nksuksa vkSj
of a monument the angle of elevation of the top fLFkr isMk+ sa osQ ry osQ voueu dks.k 60° rFkk 30° gSA ;fn unh
of the monument is found to be such that its
dh pkSM+kbZ 400 ehñ gS] unh osQ Åij gokbZ tgkt dh Å¡pkbZ Kkr
1
tangent is . On walking 138 metres towards djsaA
5
(1) 173.2 metres (2) 346.4 metres
the monument the secant of the angle of elevation
(3) 519.6 metres (4) 692.8 metres
193 (SSC CHSL DEO & LDC 2014)
is found to be . The height of the monument 37. A pilot in an aeroplane at an altitude of 200 m
12
observes two points lying on either side of a river.
(in metre) is
If the angles of depression of the two points be
fdlh Lekjd ls oqQN nwjh ij fLFkr ,d fcanq ls Lekjd osQ mPp 45° and 60° then the width of the river is—
1 200 m dh Å¡pkbZ ij mM+us okys fdlh foeku dk ik;yV fdlh
fcUnq dk tangent 5 gSA Lekjd dh vksj 138 eh- pyus ij
unh osQ nksuks fdukjksa ij nks fcUnq ns[krk gS] ;fn nksuksa fcUnqvksa osQ
voueu dks.k 45° vkSj 60° gks rks unh dh pkSM+kbZ Kkr
193
Lekjd osQ mPp fcanq dk secant gSA Lekjd dh Å¡pkbZ dhft,µ
12
Kkr dhft,\ FG 200 200 IJm FG 200 200 IJm
(1) (2)
(1) 42 (2) 49 (3) 35 (4) 56 H 3 K H 3 K
(SSC CHSL DEO & LDC 2011 )
34. The angle of elevation of the top of a Chimney and
roof of the building from a point on the ground (3) 400 3m (4) G
F 400 IJ m

are x and 45° respectively. The height of building H 3K


is h metre. Then the height of the chimney (in (SSC CGL Tier–I 2016]
metre) is 38. A boat is moving away from an observation tower.
Hkwfe ij fLFkr ,d fcanq ls Nr ij fLFkr fdlh fpeuh osQ mPp It makes an angle of depression of 60° with an
fcUnq rFkk Nr dk mUu;u dks.k x rFkk 45° gSA bekjr dh Å¡pkbZ observer’s eye when at a distance of 50 m from

BLAM–225
mQapkbZ ,oa nwjh
the tower. After 8 sec, the angle of depression
becomes 30°. By assuming that it is running in SHORT ANSWERS
still water, the approximate speed of the boat is
,d uko fdlh izos{k.k Vkoj ls nwj tk jgk gS] tc og VkWoj ls 1. (2) 2. (2) 3. (2) 4. (4) 5. (2) 6. (1)
50 ehVj dh nwjh ij gS rks og iz{s kd dh n`f"V esa 60° dk 7. (2) 8. (2) 9. (1) 10. (3) 11. (2) 12. (3)
voue.k dks.k 30° dk gks tkrk gS] rks ;g ekurs gq, fd uko 13. (2) 14. (2) 15. (1) 16. (2) 17. (4) 18. (2)
Bgjs gq, ikuh esa py jgh gSA uko dh yxHkx pky (LihM) 19. (3) 20. (3) 21. (2) 22. (2) 23. (3) 24. (3)
crkb,\ 25. (2) 26. (1) 27. (1) 28. (1) 29. (1) 30. (2)
(1) 33 km/h (2) 42 km/h 31. (2) 32. (2) 33. (1) 34. (3) 35. (4) 36. (1)
(3) 45 km/h (4) 50 km/h 37. (1) 38. (3) 39. (1) 40. (4)
(SSC CGL Tier–I 2016)
39. An aeroplane is flying horizontally at a height of
1.8 km above the ground. The angle of elevation
EXPLANATIONS
of plane from point x is 60° and after 20 seconds.
Its angle of elevation from x becomes 30°, If point 1. (2) A
x is on ground, then what is the speed (in km/hr)
of aeroplane.
,d gokbZ tgkt Hkwfe ls 1-8 fdehñ dh Å¡pkbZ ij mM+ jgk gSA

Tower
fcUnq x ls tgkt dk mUu;u dks.k 60° rFkk 20° lsoQ.M i'pkr~
x ls mUu;u 30° gks tkrk gSA ;fn fcUnq x Hkwfe ij gS] rks gokbZ
tgkt dh xfr (fdehñ@?kaVk esa) D;k gS\ 30°

(1) 216 3 (2) 105 3 B 50m C

(3) 201 3 (4) 305 3 AB = Tower = h metre


(SSC CGL Tier-II 2018) BC = 50 metre
40. A ladder leaning against a wall makes an angle
ACB = 30°
12 ABC esa
with the horizontal ground such that sin .If
13
AB 1 AB
the foot of the ladder is 7.5 cm from the wall. tan 30° =
Then what is the height of the point where the BC 3 50
top of ladder touches the wall?
50
nhokj osQ lgkjs yxh gqbZ ,d lh<+h Hkwfe osQ {kSfrt dks.k cukrh AB =
3
metre

12 Aliter :
gSA rFkk sin 13
] ;fn lh<+h dk fupyk fljk nhokj ls 7.5
ACB esa]
lseh nwj gSA rks mls fcUnq dh Å¡pkbZ D;k gS\ tgk¡ lh<+h dk 'kh"kZ BCA = 30°, ABC = 90°
nhokj dks Nqrk gSA BAC = 60°
(1) 8 lseh- (2) 15 lseh- Using,
(3) 12 lseh- (4) 18 lseh-
(SSC CPO SI 2018)
30° : 60° : 90°
41. The length of the shadow of a vertical tower on 1 : 3 : 2
level ground increases by 10 m when the altitude
of the sun changes from 45° to 30°. The height AB : BC : AC
of the tower is :
tc lw;Z dk mUurka'k 45° ls 30° gks tkrk gS] rc mQèokZ/j BC = AB 3
VkWoj dh lery Hkwfe ij Nk;k dh yackbZ esa 10 m dh o`f¼ 50 = AB 3
gks tkrh gSA VkWoj dh mQ¡pkbZ Kkr dhft,A
50
(1) 10 3 m (2) 5 3 m AB metre
3

e j
(3) 5 3 1 m (4) 10 3 1 m e j 2. (2) AB = /kxs dh yackbZ = 150 metre
(SSC CGL Tier-II Exam. 18.11.2020) BAC = 60°

BLAM–226
mQapkbZ ,oa nwjh
Aliter :
B BCD esa]
BDC = 30°, CBD = 90°
m BCD = 60°
0
15 Using,

30° : 60° : 90°


60°
A C 1 : 3 : 2
BC : BD : CD
ABC esa
1 × CD = 2 × BC
BC 3 BC
sin 60° Ÿ 15 – x = 2x
AB 2 150
3x = 15
3
BC 150 75 3 metre x 5 feet
2
Aliter :
ABC esa] 4. (4) A
CAB = 60°, BCA = 90°
ABC = 30°
Using,
30° : 60° : 90° h
1 : 3 : 2
AC : BC : AB
30°
B C
2BC = 3 × AB 100 m

150 3
BC = AB = Tower = h metre
2
ABC esa]
BC 75 3 metre
AB 1 h
3. (2) AB = Post = 15 feet tan 30° = =
BC 3 100
Post ‘C’ point ij VwV x;k gS]
BC = x feet 100
AC = CD = (15 – x) feet h= metre
3

A Aliter :
ABC esa]
BCA = 30°, ABC = 90°
15 feet

C BAC = 60°
Using,
x feet
30° : 60° : 90°
30°
B D 1 : 3 : 2

CDB = 30° AB : BC : AC
rFkk AB = CD + BC = 15 feet
CD = 15 – x BC = AB 3
BCD esa] 100 = AB 3
BC 1 x
sin30° = = 15– x 100
CD 2 AB = h = metre.
3
2x = 15 – x 3x = 15 x = 5 feet

BLAM–227
mQapkbZ ,oa nwjh
ABC esa]
5. (2) A
AB
sinT =
AC

15 75
=
17 AC
B 60° C AC × 15 = 17 × 75
6.5 m 17 75
AC = = 85 metre
AB = house, AC = ladder 15
ABC esa]
BC 1 6.5 7. (2) A
cos 60° = =
AC 2 AC
AC = 2 × 6.5 = 13 metre
Aliter :
ABC esa]
BCA = 60°, ABC = 90°
BAC = 30° 26 T
ft E D
Using, 3

30° : 60° : 90° 6 feet


1 : 3 : 2
BC : AB : AC B C
8
feet
AC = BC × 2 3
AC = 6.5 × 2
AC = 13 metre.
26
A AB = Tree = feet
6. (1) 3
BE = CD = 6 feet
75m 26 26 18 8
AE = AB – BE = –6= = feet
T 3 3 3
C B 8
DE = BC = feet
A = kite dh Position 3
AC = /kxs dh yackbZ AED esa]
AB = 75 metre
8
8 AE 8 3 1
cotT = 3
15 tanT = = 8 = × =
ED 3 8 3
3
cosecT = 1 cot 2
tanT = tan 30° T = 30°

= 1
FG 8 IJ 2
= 1
64
A
H 15 K 225
8. (2)
30°
D

225 64 289 17
= = = 20
225 225 15 metre

15
sinT = 30°
17 B C

BLAM–228
mQapkbZ ,oa nwjh
AB = Height of light house = 20 metre
10 3 1 10( 3 1)
DAC = ACB = 30° = × =
3 –1 3 1 3 –1
ABC esa]
AB = 5 ( 3 +1) metre
1 20
tan 30° = Aliter -1 :
BC 3 BC

BC = 20 3 metre
Aliter :
ABC esa]
1
ACB = 30°, ABC = 90°
BAC = 60°
Using, 45° 30°

30° : 60° : 90° 3


1 : 3 : 2
AB : BC : AC
e 3 1j 10m

10 3 1
BC = AB 3 1
3 1 3 1
BC 20 3 metre
=5 e 3 j
1 m
9. (1) Aliter -2 :
A ge tkurs gSa]
d = h (cotT1 – cotT2) ; T1 < T2
10 = h (cot30° – cot45°)
hm
10 = h e 3 1 j
45° 30°
B C 10 3 1
D h=
xm 10 m 3 1 3 1

AB = Tower = h metre
BD = Shadow = x metre =
10 3 e 1 j
2
ADB = 45°
ACB = 30° h=5 e 3 j
1 metre
ABD esa]
AB AB h 10. (3) A
tan 45° = 1= = h=x ...(i)
BD BD x
ABC esa] xm

AB D
tan 30° = 5000 m
BC

1 h 1 h
= = 60°
3 x +10 3 h 10 45°
B C
3 h = h + 10 3 h – h = 10
‘ACB = 60°
10 ‘DCB = 45°
h ( 3 –1) = 10 h=
3 –1 AB = 5000 metre

BLAM–229
mQapkbZ ,oa nwjh
AD = ? = x metre (let) 125
? 'ABC esa] 1 x = 125 metre
x
AB 5000 ABD ls]
tan60 Ÿ 3
BC BC
AB
tan30
5000 BD
Ÿ BC = metre
3 1 125
= y = 125 3 metre
'DBC ls] 3 y

DB 5000 CD = y – x = 125 3 –125 = 125 ( 3 – 1) metre


tan45 DB BC
BC 3 Aliter -1 :

5000 A
? AD = AB – BD = 5000 –
3
1
F
= 5000 G1
1 IJ F
= 5000 G
3 1 I metre
H 3K H 3
JK 45° 30°
D
B 1 C
Aliter : 3
K

1 125

e 3 –1j e 3 j
1 125 3 e j
1m

Aliter -2 :
3
ge tkurs gSa]
1
60° d = h (cotT1 – cotT2) ; T1 < T2
45° CD = 125(cot30° – cot45°)
1
= 125 e 3 1 metre.j
3 5000m
12. (3) 30°
5000 A F
60°
e 3 1 j 3
e 3 1m j
200 m

30°
E D
11. (2) A
30°
45° 60°
B C
125 m

AB = Hill = 200 metre


ADE = 30°
ACB = 60°
DE = BC = x metre
45° 30° ABC ls]
B C D AB
tan 60° =
AB = Tower = 125 metre BC
BC = x metre, BD = y metre 200 200
3 x= metre
ABC esa] x 3
AB AED esa]
tan 45° =
BC AE
tan 30° =
DE

BLAM–230
mQapkbZ ,oa nwjh
1 AE 200 PB = 10 3 AB = PB 3
AE = metre
3 200 3 AB = 10 × 3 = 30 metre.
3
A 30° F
14. (2) 60°
200 400 1
CD = 200 – = = 133 metre
3 3 3 30

108 m
°
Aliter : E D

60
°
B C
1
30° AB = 108 metre
3
3 CD = x metre
2 60° ABC esa]
AB
tan 60° =
3 200 m BC

400 1 108 108


2 333 m 3 BC = 36 3 metre
3 3 BC 3
AED esas]
13. (2) A AE 1 108 x
tan 30° =
ED 3 36 3
108 – x = 36 x = 108 – 36 = 72 metre
Q C
30° Aliter :

10m ABC esa] AED esa]

60° 30° 30° : 60° : 90° 30° : 60° : 90°


P B
1 : 3 : 2 1 : 3 : 2
AB = Tower = h metre BC : 108 : AC AE : ED : AD
PQ = 10 metre
APB = 60°, 3 BC 108 ED = AE 3
CQB = QBP = 30° 108
108 AE 3
BC [  BC =
PBQ esa] 3 3
ED]
PQ 1 10
tan 30° = = 108
PB 3 PB AE 36
3
PB = 10 3 metre CD = 108 – 72 = 36 metre
APB esa]
15. (1) A E
AB 30°
tan 60° = 45°
PB

h 300 m
3 = 10 3 h = 3 × 10 3 = 30 metre

Aliter : 30°
45°
PQB esa] ABP esa] B
C
D

30° : 60° : 90° 30° : 60° : 90°


AB = hill = 300 metre
1 : 3 : 2 1 : 3 : 2 CD = bridge = x metre
10 : PB : QB PB : AB : AP

BLAM–231
mQapkbZ ,oa nwjh
ABC esa] BP
tan 60° =
AB AB
tan 45° =
BC
1500 3
300 3 AB = 1500 metre
1= AB
BC
ACQ esa]
BC = 300 metre
CQ
ABD esa] tan 30° =
AC
AB
tan 30° =
BD 1 1500 3
= AC = 1500 × 3
1 300 3 AC
= 300 + x = 300 3
3 300 + x = 4500 metre
x = 300 3 – 300 = 300 ( 3 – 1) metre PQ = BC = AC – AB = 4500 – 1500
Aliter-1 : = 3000 metre
3000
A plane dh speed = = 200 metre/second
15
Aliter :
APB esa]
1
30° : 60° : 90°
1 : 3 : 2
45° 30°
AB : 1500 3 : AP
B 1 C D
e 3 –1 j
K AB 3 = 1500 3
3
AB = 1500
1 300 M AQC esa]

e 3 –1 j e
300 3 – 1 m j 30° : 60° : 90°

Aliter -2 : 1 : 3 : 2
ge tkurs gSa] 1500 3 : AC : AQ
d = h (cotT1 – cotT2) ; T1 < T2
d = 300 (cot30° – cot45°) AC = 1500 3 3
AC = 4500 metre.
d = 300 e j
3 1 metre.
BC = 4500 – 1500 = 3000 metre.
3000
Direction of plane Speed of plane = = 200 m/sec.
16. (2) 15
P Q

17. (4) C

1500 3
metre
2h

60° A
A 30°
B C h
T 90°–T
P rFkk Q plane dh positions gS] B D
E
PAB = 60° ; QAC = 30°
BE = DE = 30 metre
PB = 1500 3 metre
AEB = T CED = 90° – T
ABP esa]

BLAM–232
mQapkbZ ,oa nwjh
ABE esa] Aliter :

AB
fn, x, angles ,d nwljs ds complementary gSaA
tan T =
BE h= ab

h h = 12 27
tan T =
30 h = 18 metre.
h = 30tan T ...(i)
19. (3) E A F
CDE ls] 60° 30°
2h
tan (90° – T) =
30

h
cot h 15 cot ...(ii) 60°
15 30°
By multiplying both equations B x D (1 – x ) C
h2 = 30 × 15 × tan T. cot T
A = Aeroplane dh position
h2 = 30 × 15 [  tan T . cot T = 1]
B rFkk C iRFkjksa dh position
h = 15 2 metre = AB ABD = 60°, ACD = 30°
2h = 30 2 metre = CD BD = x km.
CD = (1 – x) km.
18. (2) A ABD esa
AD
tan 60° =
BD
AD
3 = x
AD = 3 x km. ...(i)

ACD esa]
T 90°–T
AD
B 12m C D tan 30° =
CD
27m
1 AD
=
ekuk] ACB = T 3 1– x
ADB = 90° – T
1– x
BC = 12 metre, BD = 27 metre AD = km. ...(ii)
3
AB = Pillar = h metre
Equations (i) rFkk (ii) ls]
ABC esa]
1– x
AB h 3 x= 3x = 1 – x
tan T = = ....(i) 3
BC 12
ABD esa] 1
4x = 1 x= km.
4
AB h
tan(90° – T) = cotT = ....(ii) 3
BD 27
AD = 3 x = km.
4
(i) rFkk (ii) dks multiply djus ij]
Aliter :
h h
tanT. cotT = × ABD esa]
12 27
30° : 60° : 90°
h2 = 12 × 27 h = 12 27
1 : 3 : 2
= 2 2 3 3 3 3 = 2 × 3 × 3 = 18 metre x : AD : AB

BLAM–233
mQapkbZ ,oa nwjh
AD = x 3 y = 100 3 – 3 x y = 100 3 – 3 × 3y
ADC esa] y = 100 3 – 3y 4y = 100 3
30° : 60° : 90°
y = 25 3 metre
1 : 3 : 2 Aliter -1 :
AD : 1–x : AC ABD esa]

AD 3 = 1 – x 30° : 60° : 90°


1 : 3 : 2
x 3 × 3 =1–x
3x + x = 1 AD : x : AB
4x = 1
x = AD 3
1
x= x
4 AD = metre.
3

AD =
3
km.
ADC esa]
4
30° : 60° : 90°
Aliter-2 :
1 : 3 : 2
ge tkurs gSa]
d = h (cotT1 + cotT2) 100 – x : AD : AC
1 = h (cot30° + cot60°)
AD = (100 – x) 3
F 1I
1 = hG 3 J x
H 3K 3
= (100 – x) 3

F4I x = 300 – 3x
1 = hG
H 3 JK 4x = 300
300
x= = 75
3 4
h= km.
4 75
AD = = 25 3 metre.
3
20. (3) A Aliter-2 :
ge tkurs gSa]
d = h (cotT1 + cotT2)
100 = h (cot30° + cot60°)
F 1 I
B
30°
D
60°
C
100 = h GH 3
3
JK
100 m
x 100– x 100 3
h=
4
BD = x metre (let)
CD = (100 – x) metre h = 25 3 metre.
AD BC; AD = y metre
21. (2) P
ABD esa]
AD 1 y R
tan 30° = = x= 3y ...(i)
BD 3 x
45 m
ACD esa] 15 m
y y
tan 60° = 3 = T 60°
100 x 100 x Q S

BLAM–234
mQapkbZ ,oa nwjh
PQ = Tower A = 45 metre
RS = Tower B = 15 metre 23. (3) A
QS = x metre (let)
PSQ = 60° ; RQS = T D
PQS esa] 16 m
PQ 9m
tan 60° =
QS T 90°– T
B C
45 x
3 = x
ABC esa]
3 x = 45
16 16
tan (90° – T) = cotT = ...(i)
45 x x
x= = 15 3 metre
3 BCD ls]
RSQ ls] 9
tanT = ...(ii)
RS 15 x
1
tanT = = tanT =
QS 15 3 3 9 16
tanT . cotT
tan T = tan 30° T = 30° x x
x2 = 16 × 9 x = 4 × 3 = 12 metre
1
sin T = sin 30° = Aliter :
2
fn, x, angles ,d nwljs ds complementary gS]
22. (2) D x= ab
x= 9 16
A 60° E x = 12 metre.

36 m
24 m

24. (3) A

B C
x
E D
DE = 36 – 24 = 12 metre a
ADE esa,
h
DE 3 12
sin 60 .
AD 2 AD y
B C
12 2
= AD = 8 3 metre
3 CD = tree = h metre
Aliter : AB = building = a metre
ADE esa BC = ED = b metre
AED esa]
30° : 60° : 90°
AE a h
1 : 3 : 2 tanx = tan x = b = (a – h) cot x
ED b
AE : DE : AD ...(i)
ABC esa]
3 AD = 2 × 12
AB a
24 tany = tan y = b = a cot y ...(ii)
AD = = 8 3 metre BC b
3
Equations (i) rFkk (ii) ls,

BLAM–235
mQapkbZ ,oa nwjh
(a – h) cot x = a cot y AB
a cot x – h cot x = a cot y tan (90° – T) =
BO
h cot x = a (cot x – cot y)
h cot x 2h 4h
Ÿ cotT = .... (ii)
a = cot x – cot y metre x x
2

25. (2) D nksuksa equations dks multiply djus ij]


2h 4h
A tan .cot Ÿ x2 = 8h2
x x

h x2
Ÿ h2 =
8
45° x
B C
x
Ÿh= metre
AB = Buidling = h metre 2 2
AD = Chimney = y metre Aliter :
BCD esa] Point O ij cus nksuksa angles complementary gSa]
BD h y
tan45° = 1= BC = h + y ... (i) Ÿ d = 2 h1 h 2
BC BC

AB x= 2 h 2h
ABC esa] tan x =
BC
x = 2h 2
h
tan x = BC = h cot x ...(ii) x
BC h metre.
2 2
Equations (i) rFkk (ii) ls]
h + y = h cot x
y = (h cot x – h) metre
26. (1) CD = h metre, AB = 2h metre, 27. (1) A

C 90° – T
T
2h Q
P a B
h b
90°– T
T
B x/2 O x/2 D AB = Tower = h units
x
AQB = T APB = 90° – T
PB = a; BQ = b
x
OB = OD =
2
metre AQB esa]
'OCD esa] tan
AB
tan
h
...(i)
BQ b
h 2h ....(i)
tan
x x APB esa]
2
h h
'OAB esa] tan (90 ) cot T = ...(ii)
PB a

BLAM–236
mQapkbZ ,oa nwjh
Equation (i) rFkk (ii) dks multiply djus ij] AB = x 3

tan .cot
h h ABC esa]
b a h2 ab
30° : 60° : 90°
h ab unit 1 : 3 : 2
Aliter : AB : x+1 : AC
fn, x, nksuksa angles ,d nwljs ds complementary gSa]
(x + 1) = AB 3
h ab units
(x + 1) = 3x
1
x=
28. (1) A (Balloon) 2

3
AB = km.
2
Aliter : 2
ge tkurs gSa]
d = h (cotT1 – cotT2) ; T1 < T2
1 = h(cot30° – cot60°)
60° 30°
F 1 I
B
x km
D
1 km
C 1=h GH 3 J
3K

2
1= h
AB = Height of balloon = h km 3
BD = x km CD = 1 km
3
ABD esa] h= km
2
AB
tan 60° = 29. (1) A
BD

h h h
3 = x= km ...(i)
x 3
60° 45°
B
ABC esa] x C D

AB 1 h
tan 30° =
BC 3
= h
1
CD = 4 e j
3 – 1 × 60 = 240 e j
3 – 1 metre

3 ABC esa]

h h AB
3h= +1 3h– =1 tan 60° =
BC
3 3
h
3h – h 3 3 = h= 3 x metre
x
=1 2h = 3 h= km
3 2
ABD esa]
Aliter -1 : AB
tan 45° =
ABD esa] BD

30° : 60° : 90° h


1=
1 : 3 : 2 x + 240 e 3 –1 j
x : AB : AD

BLAM–237
mQapkbZ ,oa nwjh
h = x + 240 e 3 –1 j FG 1IJ
240 e j
3 –1 = h 1
H 3K
h
h= + 240 e 3 –1 j
3 F 3 –1 I
h
240 e j
3 –1 = h GH 3
JK
h–
3
= 240 e 3 –1 j
h 240 3 metre
e 3 –1 h j = 240 e 3 –1 j 30. (2) R
3

h = 240 3 metre
OR

B
60°
P Q
h
60° 45° PQR = 60°
A D C
d PQ = 450 3 metre
PQR esa,
Distance moved in 1 minute = 4 e j
3 – 1 × 60
PR
tan60°=
PQ
d = DC = 240 e 3 –1 j PR
Aliter – 1 : tan 60° =
450 3

PR
B = 3 =
450 3

PR = 450 3 × 3
3 = (450 × 3) metre
PR
balloon dhspeed =
6 minutes
60° 45°
A 1 D
e 3 –1 j
C
FG 450 3 IJ metre/second
K 3
= H 6 60 K
15
= = 3.75 metre/second
4
e 3 –1 j 240 3 – 1 e j Aliter :

3 240 3m C
Aliter – 2 :
Using relation
d = h (cotT1 – cotT2)
T1 < T2

240 e 3 –1 j 60°
= h (cot 45° – cot 60°) P 450 3 B

BLAM–238
mQapkbZ ,oa nwjh
PBC esa, 32. (2)
B
30° : 60° : 90°
1 : 3 : 2

PB : PC : BC

PC = PB 3 hm

PC = 450 3 × 3
PC = 1350 metre
T I
1350
balloon dh speed =
6 60
C
560 m
D
xm
A

1350
= = 3.75 m/s. 3 4
360 ;fn fn;k gqvk gS tanT = 4 rFkk tan I = 3
31. (2) ABC esa ,
ABC esa,

C AB
tan T =
AC
3 h
=
4 560 + x
= 3(560 + x) = 4h
= 1680 + 3x = 4h ... (i)
ABD esa,
AB
15° tan =
AD
A B 4 h
5 metre
3 x

AB 3h
x=
cos 15° = 4
AC
bl value dks equation (i) esa j[kus ij]
5
AC = 3h
cos15 1680 + 3 × = 4h
4
cos 15° = cos (45° – 30°)
= cos 45°. cos 30°+ sin 45°. sin 30° 9h
1680 = 4h 
4
1 3 1 1 3 1
= . . = 7h
2 2 2 2 2 2 1680 =
4
5 1680 u 4
AC = 2 2 h=
e 3 1 j 7
h = 240 × 4
10 2 3 –1 h = 960 cm
= Tower dh height = 960 metres.
e 3 1 j 3 –1
2nd Method
3 4
=
10 2 e 3 –1 j tan =
4
cot
3
2
4 3
=5 6 –5 2 tan = ,cot
3 4

BLAM–239
mQapkbZ ,oa nwjh
d 560 P h
h = cot = 960m tan x° =
1 cot 2 4 3 h
3 4 h tan x° = p + h
P = h tan x° – h
33. (1) A 35. (4)

h A
2h
h

90° –
B x C D B 30 C 30 D
K 138 m
In ABC,
h 30
A/q tan cot ...(1)
30 h
1 193 and
tan and sec
5 12 In EDC,
2h
cot =5 cot =
12
7
tan 90 –b g 30
Now, using the Trick, 2h
d = h (cot 1 – cot 2) cot ...(2)
30

138 = h 5
FG 12 IJ from eqn (1) & (2)
H 7 K 30 2h
h 30
193 2h2 = 30 × 30
7 h= 450 15 2
Thus
12
AB = 15 2 and CD = 30 2m .
138 7 138 7 36. (1)
h= 42m
35 – 12 23
34. (3) A
Ratio figure
A A

3
B
60° 30°
B D C 60° 30°
h K 400 m D 3 C
x° K 4

C 45° D
4 400
let AB = P 400
3 3 = 100 3
In BCD ( = 45°) 4
BC = CD = h = 100 × 1.732
In ACD = 173.2 m

BLAM–240
mQapkbZ ,oa nwjh
37. (1) d
Speed of the plane =
A t

Ratio figure
1.2 3
= km/h = 216 3 km/h
200 m FG 20 IJ
3 H 60 60K
40. (4)
45° 60° 45° 60°
B D C B C
A
K e 3 +1 j
3 200
13

e 3 1j
200 3 e 1 j 200
200
m
12
B
3 3 7.5 cm C 5
38. (3)
12
A 30° sin
60° 13

12
tan
3 15
In ABC
60° 30° 60° 30°
B 50 m C D 1 2 AB
tan
K 3 7.5
1 50
2 100 m (distance) 12 AB
5 7.5
d 100
the Speed of Boat = m / sc AB = 18 cm
t 8
41. (2)
100 18
= km / h = 45 km/h
8 5 A
Ratio fig.
39. (1)

A D

1
1.8 km

45° 30°
60°
45° 30°
30°
B C D
x K 3
B C K — 10 m

Ratio Fig.
e 3 1 j 10m

10 3 1
1
60° 30°
x 3 1 3 1
2 1
K 3
=
10 3 1e j = 5 3 e 1m j
2
3 1.8 km

3.6 3
the height of tower = 5 3 e j
1m

‡‡‡
2 1.2 3 km
3 3

BLAM–241
mQapkbZ ,oa nwjh

ADVANCED LEVEL QUESTIONS


1. A 1.5m tall boy is standing at some distance from high building and is flying his kite at an
a 30m tall building. The angle of elevation from elevation of 45°. Both the boys are on opposite
his eyes to the top of the building increases from sides of both the kites. Find the length of the
30° to 60° as he walks towards the building. The string, that the second boy must have so that
distance he walked towards the building will be the two kites meet
,d 1.5 ehVj yack yM+dk 30 ehVj mQ¡ps ,d edku ls oqQN tehu ij [kM+k ,d yM+dk 100 ehVj yacs /kxs ls 30° ds
nwjh ij [kM+k gS tSls&tSls og edku dh rjiQ c<+rk gS] edku mUu;u dks.k ij irax mM+k jgk gSA nwljk yM+dk 20 ehVj mQ¡ps
ds mQijh fljs dk mldh vk¡[kksa ls mUu;u dks.k 30° ls 60° edku ds Nr ij [kM+k gksdj 45° ds mUu;u dks.k ij irax
mM+k jgk gSA nksuksa yM+ds] nksuksa iraxksa ls foijhr fn'kk esa gSaA nwljs
gks tkrk gSA edku dh rjiQ mlds }kjk r; dh x;h nwjh gS
yM+ds ds irax ds /kxs dh yackbZ D;k gksxh rkfd nksuksa irax
18 fey lds?a
(1) 18 3 m (2) m
3 (1) 20 3 m (2) 30 2 m

19 (3) 20 2 m (4) 30 3 m
(3) 19 3 m (4) m
3 5. A round balloon of radius r subtends an angle
2. A man standing on the deck of a ship, which is 60° at the eye of the observer, while the angle of
10m above the water level, observes the angle of elevation of its centre is 45°. The height of the
elevation of the top of a hill as 60° and the angle centre of the balloon will be
of depression of the base of the hill 30°. The r f=kT;k okyk ,d xksy xqCckjk voyksdu dÙkkZ ds vk¡[k ij
distance of the hill from the ship and the height 60° dk dks.k cukrk gS] tcfd blds dsanz dk mUu;u dks.k
of the hill is. 45° gSA xqCckjs ds dsUnz dh mQ¡pkbZ gksxh
,d vkneh tgkt ij [kM+k gS tks ikuh ds Lrj ls 10 ehVj mQij r
2
gS] ,d igkM+h ds pksVh dk mUu;u dks.k 60° rFkk igkM+h ds (1)
r
(2)
2
vk/kj dk voueu dks.k 30° voyksfdr djrk gSA tgkt ls
(3)
2r (4) 2r
igkM+h dh nwjh rFkk igkM+h dh mQ¡pkbZ gS
6. ABCD is a rectangle with AD = 12 cm and DC =
(1) 17.32m, 40 m (2) 1.732 m, 40 m
20 cm. Line segment DE is drawn making an
(3) 40 m, 17.32 m (4) 40 m, 1.732m angle of 30° with AD, intersecting AB in E.
3. A straight highway leads to the foot of a tower. Length of DE and AE will be
A man standing at the top of the tower observes ABCD ,d vk;r gS] AD = 12 lseh rFkk DC = 20 lseh
a car at an angle of depression of 30°, which is js[kk[kaM DE [khapk x;k gS tks AD ds lkFk 30° dk dks.k
approaching the foot of the tower with a uniform
speed. Six seconds later, the angle of depression
cukrh gS rFkk AB dks E ij dkVrh gSA DE rFkk AE dh yackbZ
of the car is found to be 60°. The time taken by gksxhA
the car to reach the foot of the tower from this
A E B
point is
,d lh/h lM+d ,d ehukj ds ikn rd tkrh gSA ehukj ds 'kh"kZ
12 cm 30°
ij [kM+k ,d vkneh 30° ds voueu dks.k ij ,d dkj dks
ns[krk gS] tks ehukj ds ikn dh rjiQ ,d leku xfr ls vk jgh
D 20 cm C
gSA 6 lsadM ds ckn og dkj dk voueu dks.k 60° ikrk gSA
ml fcUnq ls ehukj ds ikn rd igq¡pus esa yxk le; gSA (1) 4 3 cm, 8 3 cm
(1) 2 sec. (2) 4 sec.
(2) 8 2 cm, 4 3 cm
(3) 5 sec. (4) 3 sec.
4. A boy is standing on the ground and flying a (3) 8 3 cm, 2 3 cm
kite with 100 m of string at an elevation of 30°.
Another boy is standing on the roof of a 20m (4) 8 3 cm, 4 3 cm

BLAM–242
mQapkbZ ,oa nwjh
7. A man on the top of a light house observes a 11. A contractor wants to build two sea-saw in a
boat coming directly towerds it. If it takes 10 park. For children below 5 years of age he wants
minutes for the angle of depression to change to make sea-saw in such a way that its top is
from 30° to 60°, how soon will it reach the light 1.5m above the ground and it makes an angle of
house 30° with ground. For children above 5 years of
,d vkneh ,d izdk'k ?kj ds 'kh"kZ ls vius rjiQ vkrs uko age the height of its top from ground is 3m and
its makes an angle of 60° with ground. The length
dks ns[krk gSA voueu dks.k 30° ls 60° gksus esa 10 feuV dk of sea saw will be .
le; yxrk gS] fdrus nsj esa og izdk'k ?kj ds ikl igq¡p
,d Bsdsnkj cPpksa dks [ksyus ds fy, ikdZ esa nks fiQlyuiV~Vh
tk,xk?
yxkuk pkgrk gSA 5 o"kZ ls de mez ds cPpksa ds fy, og ,d
(1) 4 min (2) 6 min
,slh fiQlyuiV~Vh yxkuk pkgrk gS ftldk f'k[kj 1.5 ehVj dh
(3) 5 min (4) 7 min mQ¡pkbZ ij gks vkSj {kSfrt /jkry ds lkFk 30° ds dks.k ij >qdh
8. The angle of elevation 'T' of the top of a light gksA 5 o"kZ ls vf/d mez ds cPpksa ds fy, og 3 ehVj dh
house at a point 'A' on the ground is such that
mQ¡pkbZ ij ,d vf/d <ky dh fiQlyuiV~Vh yxkuk pkgrk gS
5 tks Hkwfe ds lkFk 60° dk dks.k cukrh gS rks fiQlyuiV~Vh dh
tan T = . When the point is moved 240m
12 yackbZ Kkr djsa %
towards the light house, the angle of elevation
(1) 1 3 m (2) 3 3 m
3
becomes such that tan = . The height of
4
(3) 4 3 3 3 m (4) 2 3 m
light house will be
12. A man saw two objects in west direction. As he
tehu ij fLFkr fdlh fcUnq A ls izdk'k ?kj ds 'kh"kZ dk mUu;u moves C units towards north he found that both
5 the objects makes angle of his eyes and when
dks.k T bl izdkj gS rkfd tan T = tc fcUnq izdk'k ?kj be moves further C units in north direction. His
12
eyes makes an angle of with both the objects.
ds rjiQ 240 ehVj c<+rk gS rks mUu;u dks.k bl izdkj gS The distance between two objects.

rkfd tan =
3
izdk'k ?kj dh mQ¡pkbZ gksxh ,d O;fDr if'pe dh vksj ,d ljy js[kk esa nks oLrqvksa dks
4 ns[krk gSa tc og mÙkj dh vksj c nwjh r; djrk gS rks ns[krk
(1) 220 m (2) 225 m gS fd nksuksa oLrq,¡ mldh vk¡[k ij dks.k cukrh gSa vkSj tc
(3) 200 m (4) 215 m og mÙkj dh vksj c nwjh vkSj r; djrk gSa rks nksuksa oLrq,¡ dks.k
9. The angle of elevation of a cliff from a fixed point cukrh gSaA oLrqvksa ds chp dh nwjh gS %
A is 45°. After going up a distance of 600m
towards the top of the cliff at an inclination of 3a 3c
(1) cot cot (2) cot  cot
30°, it is found that the angle of elevation is
60°. The height of cliff will be
3c 3b
,d fLFkj fcUnq ls ,d pksVh dk mUu;u dks.k 45° gSA pksVh (3) 2 cot cot (4) cot cot
ds rjiQ 30° ds ij 600 ehVj tkus ij mUu;u dks.k 60° gks
13. The angle of elevation of h m high pole from a
tkrk gSA pksVh dh mQ¡pkbZ gksxhA point at a distance of x m from its base is 30°.
(1) 819.6 m (2) 810 m On moving d m towards the pole the angle of
elevation changes to 60° then which one of the
(3) 850 m (4) 825 m
given options is true.
10. The height of a building at a distance of 100m
from a tower in 40m. The angle by the top and m¡QpkbZ h okys ,d LrEHk ls x eh- nwjh ij /jkry ij ,d fcUnq
bottom of tower to the top of building is right ij mlosQ 'kh"kZ dk mUu;u dks.k 30° gSA LrEHk dh vksj ‘d’
angle. Height of tower will be. eh- pyus ij mU;;u dks.k 60° gks tkrk gS] rc fuEufyf[kr esa
,d ehukj ls 100 eh- dh nwjh ij fLFkr ,d Hkou dh Å¡pkbZ dkSu &lk ,d lgh gSA
40 eh- gSA ehukj ds 'kh"kZ ,oa vk/kj }kjk Hkou ds 'kh"kZ ij 3d
ledks.k cuk;h tkrh gSA rks ehukj dh Å¡pkbZ Kkr dhft,\ (1) x = d + h (2) x =
2
(1) 250 m (2) 290 m
5d
(3) 300 m (4) 240 m (3) x = (4) x = 2d
2

BLAM–243
mQapkbZ ,oa nwjh
14. There are two parts of a tower of more than 100m eksgu ,d unh ds fdukjs [kM+k gksdj ns[krk gS fd unh ds nwljs
1 fdukjs ij ,d o`{k }kjk cuk dks.k 60° gSA ;fn og fdukjs ls
high. Lower part is of total height of tower, at
3 50 ehVj ihNs tkrk gS rks dks.k 30° gks tkrk gSA unh dh pkSM+kbZ
a distance of 40 m from the base of lower part an D;k gS\
angle Q is formed with upper part whose tangent
(1) 15 m (2) 30 m
1
is , the height of tower is (3) 55 m (4) 25 m
2
100 ehVj ls vf/d m¡QpkbZ okys ,d mQèokZ/j lrEHk ds nks 18. From a point M on a level ground, the angle of
Hkkx gSaA uhps okyk iwjs dk ,d frgkbZ gS] blosQ fupys Hkkx ds elevation of the top of tower is 30°. If the tower is
400 m height, the distance of point M from the
iSj ls 40 ehVj nwjh ij {kSfrt ry esa fLFkr fcUnq ij mQijh fgLlk foot of the tower is :
1
,d dks.k vUrfjr gS] ftldk VSatVsa gS] rks LrEHk dh m¡QpkbZ {kSfrt ry ij fLFkr fdlh fcUnq M ls ,d VkWoj ds 'kh"kZ dk
2
mUu;u dks.k 30° gSA ;fn VkWoj dh m¡QpkbZ 400 ehVj gSA VkWoj
D;k gS \
ds ikl ls fcUnq M dh nwjh D;k gS \
(1) 100 m
(1) 640 m (2) 692 m
(2) 200 m
(3) 550 m (4) 720 m
(3) 120 m
(4) 150 m 19. An observer 5 m tall is 20 3 m away from a
15. On moving c metres towards the top of a cliff a tower. The angle of elevation from his eye to the
top of the tower is 60°. The height of the tower
man found that the angle of depression of a point
on the line joining the base of cliff on horizontal is?
ground is . Moving further c metres towards the
top of cliff the angle of depression with the same 5 ehVj yack ,d iz{s kd ,d VkWoj ls 20 3 ehVj dh nwjh ij
point becomes Angle of inclination of path with gS] mldh vk¡[k ls VkWoj ds 'kh"kZ dk mUu;u dks.k 60° gSA VkWoj
ground is. dh m¡QpkbZ gS ?
fdlh vfHkur ry ij c ehVj nwjh pyus ds ckn ,d vkneh (1) 30 m (2) 65 m
ns[krk gS fd <ky ds ikn ls xqtjus okys {kSfrt ry ij dh vksj (3) 60 m (4) 45 m
,d oLrq dk voueu dks.k gS rFkk fiQj c ehVj nwjh pyus 20. The angle of elevation of the top of a light house
ds ckn voueu dks.k gks tkrk gS ikn {kSfrt ds lkFk ry dk a point M on the Ground is such that = tan–1
>qdko Kkr djsa %
(1) sin–1 (2sin – cos )
FG 9 IJ .
–1
H 40 K When the point is moved 180 metre
(2) cot (2cot – cot )
towards the light house, the angle of elevation
(3) tan–1 (2cot – cot )
(4) cot–1 (2cot – cot ) = tan–1
FG 3 IJ . The height of
becomes such that H 4K
16. The angle of elevation of the top of the tower
light house is?
observed from each of the three points P, Q, R on
the Ground, forming a triangle is the same angle. tehu ij fLFkr fdlh fcUnq M ls ,d izdk'k ?kj ds 'kh"kZ dk
If M is the circum radius of the triangle PQR,
then find the height of the tower?
mUu;u dks.k
FG 9 IJ
bl izdkj gS fd = tan–1 40 . tc fcUnq
tehu ij fLFkr rhu fcUnqvksa P, Q, rFkk R ls ,d VkWoj ds 'kh"kZ
H K
dk mUu;u dks.k ,d f=kHkqt cukrs gSa ftlosQ dks.k leku gSA izdkj ?kj dh rjiQ 180 ehVj cgrk gS rks mUu;u dks.k bl
;fn M PQR dh ifj f=kT;k gS rks VkWoj dh m¡QpkbZ D;k gS\ FG 3 IJ
izdkj gS fd = tan–1 4 . rks izdk'k ?kj dh m¡Qpkb gS ?
H K
(1) M sin (2) M tan
(3) M sec (4) M cot
350 300
17. Mohan standing on the bank of a river observes (1) m (2) m
8 11
that the angle substended by a tree on the
opposite bank 60°. When he retires 50 metre from
405 400
the bank, he finds the angle to be 30°. The breadth (3) m (4) m
of the river is ? 7 11

BLAM–244
mQapkbZ ,oa nwjh
21. From the top of a light house 110 metres height ,d O;fDr VkWoj ds 'kh"kZ ls VkWoj ls nwj tkrh gqbZ uko dk
with its base at the sea-level, the angle of voyksdu djrk gSA uko O;fDr ds vk¡[k ls 60° dk voueu
depression of boat is 15°. The distance of the boat
from the foot of the light house is : dks.k cukrh gS] tc VkWoj ls 100 3 ehVj dh nwjh ij gSA 30
110 ehVj m¡Qps izdk'k ?kj ds 'kh"kZ ls] leqnz ry ij ,d uko lsd.M ds ckn voueu dks.k 30° gks tkrk gSA uko dh xfr
dk voueu dks.k 15° gSA izdkj ?kj ds ikn ls uko dh nwjh gS % yxHkx D;k gS] ;g ekurs gq, fd uko 'kkar ty esa py jgh gS?
(1) 35.2 kmph (2) 11.54 kmph
(1) 55 ( 4 2 3) m (2) 55 ( 4 2 3) m
(3) 41.6 kmph (4) 26.08 kmph
(3) 50 ( 4 2 3) m (4) 50 ( 4 2 3) m 25. The top of a 12 metre high tower-makes an angle
22. A boy standing in the middle of a field, observes of elevation of 75° with the bottom of an electric
a flying balloon in the north at an angle of pole and angle of elevation of 30° with the top of
elevation 30° and after 15 second, he observes the pole. What is the height of the electric pole?
the same balloon in the south at an angle of ,d 12 ehVj m¡Qps VkWoj dk 'kh"kZ ,d fctyh ds iksy ds vkèkkj
elevation of 45°. If the balloon flies all along in a
ls 75° dk mUu;u dks.k cukrk gS rFkk iksy ds 'kh"kZ ls 30°
10 dk mUu;u dks.k cukrk gSA fctyh ds iksy dh m¡QpkbZ D;k gS?
straight line at a height of m, then its speed
3
in kmph is? (1) (12 3 3) m (2) (24 8 3) m
,d yM+dk ,d eSnku ds eè; esa [kM+k gS] og mÙkj esa ,d
(3) (24 8 3) m (4) (12 3 3 ) m
mM+rs gq, cSywu dks mÙkj esa 30° ds mUu;u dks.k ij ns[krk gS
rFkk 15 lsoQ.M ds ckn nf{k.k esa 45° dk mUu;u dks.k ij mlh 26. Two vertical poles 10 m and 40 m high stand a
part on the horizontal plane. The height of the
10
cSywu dks ns[krk gSA ;fn cSywu 3
ehVj dh m¡QpkbZ ij ,d point of intersection of the line joining the topof
Each pole to the foot of the other is?
lhèkh js[kk esa mM+ jgk gks rks mldh fdeh@?kaVk esa xfr D;k gS ?
nks mèokZ/j iksy ftudh m¡QpkbZ 10 ehVj rFkk 40 ehVj gSA ,d
(1) 2 kmph (2) 3.5 kmph
{kSfrt ry ij [kM+s gSaA ,d iksy ds 'kh"kZ ls nwljs iksy ds ikn ls
(3) 3.8 kmph (4) 4 kmph 'kh"kZ dks feykus okyh js[kk ds izfrPNsn fcUnq dks m¡QpkbZ D;k gS ?
23. From a horizontal point observes the angle of (1) 15 m (2) 10 m
12 (3) 6 m (4) 8 m
elevation of the tower is cos . When moved
13
224 metre towards the tower the angle of SHORT ANSWERS
5
elevation becomes sec , the height of the 1. (3) 2. (1) 3. (4) 4. (2) 5. (3) 6. (4)
4
tower is? 7. (3) 8. (2) 9. (1) 10. (2) 11. (4) 12. (3)

,d {kSfrt fcUnq ls ,d VkWoj dk mUu;u dks.k bl izdkj gS 13. (2) 14. (3) 15. (4) 16. (2) 17. (4) 18. (2)
19. (2) 20. (3) 21. (2) 22. (3) 23. (2) 24. (3)
12
fd cos 13
VkWoj dks vksj 224 ehVj c<+us ij mUu;u 25. (2) 26. (4)

5
dks.k] sec , gks tkrk gSA VkWoj dh m¡QpkbZ gS ? EXPLANATIONS
4
(1) 120 m (2) 210 m 1. (3)
(3) 250 m (4) 180 m E

24. A man is watching from the top of a tower a boat


speeding away from the tower, the boat makes
an angle of depression of 60° with the man’s eye 30 m 28.5
when at a distance of 100 3 metres from the 30° 60°
tower. After 30 seconds, the angle of depression A D
1.5 m F
becomes 30°. What is the approximate speed of B C
the boat, assuming that it is running in still
ADE esa,
water?

BLAM–245
mQapkbZ ,oa nwjh
30° : 60° : 90°
1 : 3 : 2
AB : BC : AC
3
BC AB 3

30° 60° BC 10 3 = 10 × 1.732 = 17.32 m


2 1
3 h 10 3 3 30 m
igkM+h dh mQ¡pkbZ = (10 + 30) m = 40 m
3 28.5
3. (4) C
28.5
2 2
3

57 3
= 19 3m
3 3
Aliter : 30°
60°
d = h (cotT1 – cotT2) A B
D
= 28.5 (cot30° – cot60°)
ACD esa,
F 1 I 30° : 60° : 90°
= 28.5 G 3 J
H 3K
1 : 3 : 2
AD : AC : CD
28.5 2 3
=
3 3 AC AD 3
ABC esa,
d 19 3 m
30° : 60° : 90°
1 : 3 : 2
2. (1) D
AC : AB : BC

AB AC 3
hm
AD + DB AC 3
60°
A E AD + DB AD 3 u 3 = 3AD
30°
2AD = DB
10 m 10 m
dkj ,d leku xfr ls py jgh gSa
30° B ls D rd tkus esa yxk le; D ls A rd tkus esa yxk
B C
le; ls nqxquk gSA
ADE esa, fy;k x;k le; = 3 seconds
4. (2)
30° : 60° : 90°
D
1 : 3 : 2
AE : DE : AD

DE 3 AE 0m
45°
10 E C
h 3 AE
20 m
ABC esa, 30°
A B
F

BLAM–246
mQapkbZ ,oa nwjh
ADF esa,
A E B
6. (4)
30° : 60° : 90°
1 : 3 : 2
DF : AF : AD 12 cm
30°
2 × DF = AD
D C
AD 100 20 cm
DF 50 m
2 2
ADE esa,
DE = 50 – 20 = 30 m
30° : 60° : 90°
DEC esa,
1 : 3 : 2
45° : 45° : 90°
AE : AD : DE
1 : 1 : 2
AE 3 AD
DE : EC : DC
AD
DC DE 2 AE
3
DC 30 2 m
12 3
AE
5. (3) 3 3
B
AE 4 3 cm
O Also, DE = 2AE

30° DE 2 4 3
C
60° DE 8 3 cm
A 45°
D
7. (3) A

AOB esa,

30° : 60° : 90°


1 : 3 : 2 h

BO : AB : AO
AO = 2BO 60° 30°
C
AO 2r B y x
D
AOD esa,
Using,
45° : 45° : 90° d = h (cot T1 – cot T2)
1 : 1 : 2 x = h (cot 30° – cot 60°)

OD : AD : AO
h
FG 3–
1 IJ
AO OD 2
H 3 K
AO 2h
OD x
2 3
Distance = speed × time
2r
2 2h
= Speed × 10
3
OD 2r

BLAM–247
mQapkbZ ,oa nwjh
2h h 4h
Speed m BD
10 3 5 3 3

ABD esa, BD dh value equation (1) esa j[kusa ij_


30° : 60° : 90° 4h 12h
240 +
3 5
1 : 3 : 2
y : h : AD 12h 4h
240
5 3
3y h
36h 20h
240
h 15
y
3
240 15
h
rd fy;k x;k le; light house 16
Distance h = 225 m
speed 2nd Method
5 12
h tan cot
12 5
h
3
5 3 3 4
tan cot
4 3
= 5 minutes
d 240
225m
8. (2) C h = cot 1 cot 2
12 4
5 3

9. (1) C

h
E 60°
m F
B 600
A 240 m D
30° 45°
A B
ABC esa, D
BC AED esa,
tan
AB 30° : 60° : 90°
5 h 1 : 3 : 2
12 AB ED : AD : AE

12h AE = 2ED
AB
5 AE
ED
2
12h
240 + BD ...(1)
5 600
ED
BDC esa, 2

BC ED 300 m
tan =
BD
Also, AD 3 ED
3 h
4 BD AD 300 3 m

BLAM–248
mQapkbZ ,oa nwjh
ABC esa, Aliter :

45° : 45° : 90° d2


H= h
1 : 1 : 2 h
BC : AB : AC 100 100
H= 40 290 m
AB = BC 40
CEF esa, 11. (4)
30° : 60° : 90°
A
1 : 3 : 2
EF : CF : CE
D
CF EF 3

h – FB = (AB – AD) 3 1.5 m

30° 60°
h – 300 = (h – 300 3) 3
E B C
h – 300 = h 3 – 900
BDE esa]
h( 3 – 1) = 600
DE DE
cosec 30 2
600 BD 1.5
h=
(1.732 – 1) DE = 3 m

600 ABC esa]


h= 819.6 m
0.732 AC AC 2
cosec 60
AB 3 3
10. (2) A
2
AC = 3 2 3 m
x 3
T E
12. (3) ekuk fd A vkSj B nks objects gSaA
B 90° PQ = c, QR = c,
(90°–T
40 )
AQB = , ARB =
D
C 100 m R

BCE esa]
c
BC E
tan (90– T) =
BE
Q
40 2
cot T = D c
100 5 T
5
tan T =
2
B x A y P
ABE esa]
AB Let AB = x, AP = y rFkk PQA = T
tan T =
BE y
APQ esa] tan c
5 x
x 250
2 100 x y
Tower dh height 250 + 40 = 290 m
BQP esa tan b g c

BLAM–249
mQapkbZ ,oa nwjh
vc] tan D = tan [(T + ) – T]] 4
2h  3d xu
tan (  )  tan 3
= 1  tan (  ) tan
2 3h  3d
x
4
x y y
c c cx equation (ii) ls h dh value j[kus ij
=
1
FGx y y IJ c2 xy y2
H c c K 3 x 3
d x
2 3 4
c2 xy y2
cot = x 3 x 3 3
cx x d, d, x d
2 4 2 4 2
cx cot = c2 + xy + y2 .......(i)
blh izdkj 2 cx cot = (2c)2 + xy + y2 ....(ii)
14. (3) B
equation (ii) rFkk (i) djus ij
3c2 = cx (2 cot – cot )
2x
3c 3
x
2cot cot
C
13. (2)
x
3
A

A 40 m D

h
ekuk AB, x m
x 2x
60° 30° tgk¡ AC 3
, BC
3
B D d C
x 1
rFkk tanT = 2
ABD esa] ekuk ADC=
h x
tan 60
BD 3 x 1
tan , tan
40 120 2
h
3
x d ABD esa]

AB x
h 3x 3d .....(i) tan b g AD 40
ABC eas]
 tanT = tan [(T + )– ]
h
tan 30 tan ( ) tan
x
= 1 tan ( ). tan
h 1 x
, h
x 3 3 x x
1 40 120
equation (i) rFkk (ii) dks add djus ij x x
2 1 .
2h
F
xG 3
1 IJ 3d
40 120
H 3K

BLAM–250
mQapkbZ ,oa nwjh
x
1 60 P N
2 4800 x 2
4800
4800 + x2 = 160 x
x2 – 160x + 4800 = 0
2 O
(x – 40) (x – 120) = 0 M
Q R
x = 40 ;k 120
Desired result = 120
 Tower dh height 100 m ls vf/d gSA
15. (4) ekuk fd ABD ,d inclined path gS rFkk AB = BD
From OPM,
= c vc PAB rFkk PAD esa sin formulae ls]
ON
PA AB c tan =
OP
sin ( ) sin sin ....(i)
ON = OP tan T
PA AD 2c ON = M tan T
vkSj sin ( ) sin sin ....(ii) 17. (4)

D P
E

30° 60°
Q x R
50 m
B M

T–D
PMR ls]
P A E
h
tan 60° =
x
equation (ii) esa ls (i) Hkkx nsus ij]
h 3x ...(i)
PA sin ( ) 2c sin
sin ( ) PA sin c PQR,

h
sin cos cos sin tan30° =
50 x
sin
1 h
2(sin cos cos sin )
= 3 50 x
sin
sin T cot – cos T = 2 (sin T cot – cos T) 50 + x = 3h
2 cos T – cos T = sin T (2 cot – cot )
cot T = 2 cot – cot 50 + x = 3 3x
T= cot–1 (2cot – cot ) 50 + x = 3x
16. (2) We know that the tower makes equal angles 2x = 50
at the vertices of the triangle, therefore foot of x = 25 m
the tower is at the circum centre.

BLAM–251
mQapkbZ ,oa nwjh
2nd method 20. (3)

30° 60°
2 1
K 3

M N
180 m
2 50
1 25m MNO esa]
18. (2)
N ON FG tan =
9 IJ
tan =
MN H 40 K
9 ON
= ...(i)
400 m 40 180 + BN
BNO esa]
30°
M
O
ON FG tan =
3 IJ
tan =
BN H 4 K
NO
tan 30° = 3 ON
MO
4 BN
1 400
4
3 MO BN = ON ...(ii)
3
MO = 400 3 = 400 × 1.732 From equation (i) & equation (ii),
= 692 m 9 ON
40 4
19. (2) AD = BC = 20 3 180 ON
3
AB = DC = 5 m
180 × 9 = 120N = 40 ON
E 180 × 9 = 28 ON
180 9
ON =
28

405
m
7
21. (2) Let OP be the light house and the boat is at
point P.
A 60° OQP = QPM = 15°
D

5m
M P
15°
B 20 3 C 110m
Q 15° O
ED
tan 60° = In QOP,
AD
OP
ED tan 15° =
3 OQ
20 3
ED = 60 110
tan (45° – 30°) =
CE = CD + ED = 5 + 60 = 65 m OQ

BLAM–252
mQapkbZ ,oa nwjh
1 distance
1– Now, speed =
3 110 time
1 OQ
1 1 15.77 18
3 =
15 5
3 1 110 = 3.8 kmph
3 1 OQ 23. (2)
A
F 3 1 I
OQ = G J 110
H 3 1K

15
( 3 1) ( 3 1) 3
OQ = 110 × 12
( 3 1) ( 3 1) 13 5×5 3 × 5  cos
39 13
5
OQ = 55 ( 4 2 3 ) m. sec
4
22. (3) ekuk yM+dk point P ij gSA
10 m 20
16
3
Q R B 224 m C 4×5 D
12 × 3
10 m  16 units 224
30° 1 units 14
15 units 14 × 15
P = 210 m
45° 24. (3)

60°
T P
T S 30°
10 m
3
PQR esa,
QR
tan 30° =
QP 30° 60°
Q
R 100 3 S
10
1 3
RPS esa
3 QP
PS
QP = 10 m tan 60° =
RS
PTS esa,
TS PS
tan 45° = 3
PT 100 3

10 PS = 300 m
3 PQS esa
1
PT PS
tan 30° =
10 QS
PT =
3
1 300
Total distance travelled by balloon
3 100 3 QR
10
= QP + PT = 10 + = 15.77 m QR + 100 3 300 3
3

BLAM–253
mQapkbZ ,oa nwjh
QR = 200 3 = 200 × 1.732
24 8 3 (2 3)
QR = 346.4 m h
(2 3) (2 3)
346.4 18
vc, uko dh Speed =
30 5 (24 8 3 )(2 3)
= 41.6 km/hr
48 24 3 16 3 24

25. (2) A (24 8 3 )m


26. (4)
S
12-h

30° 12
B E
x
40
h P
h
T
75°
D 10
x h
x
Q R
ekuk, U
h cotx h cot
BE = CD = x
BC = ED = h (40 – h) cot x
In ABC,
40 cot x
12 h RTU esa,
tan 30° =
x
UR
...(i) cot =
x 3 (12 h ) h
ACD esa, UR h cot
12
tan 75° = blh izdkj,
x
QU h cot x
(  tan 75° = tan (45° + 30°)

(2 3) QRS esa
40
12 cot x
2 3= QR
3 (12 h )

equation (i) ls QR 40cot x

PQR rFkk QRS esa


4 3
2 3 = (12 h ) 10 cot = 40 cot x

cot 4 cot x
24 2h 12 3 3h 4 3
 h cot = (40 – h) cot x
24 8 3 h( 3 2) h × 4 cotx = (40 – h) cot x
4h = 40 – h
24 8 3 5h = 40
h
2 3 h = 8 metre
‡‡‡
BLAM–254
T;kfefr

5 T;kfefr (GEOMETRY)
Geometry mathematics dh og branch gS ftlesa points, (2) Alternate Angles : / ,dkarj dks.k :
lines, triangles, quadrilateral, circles etc. geometrical
figures osQ ckjs esa vè;;u fd;k tkrk gSA b
js[kk : The angle of straight line is 180°. /
Line / a
Alternate
ljy js[kk dk dks.k 180° gksrk gSA a Angles
180° b
i.e. b
a

a b 180 a= a (,dkarj dks.k)


z Angle / dks.k : Angle rc curk gS tc nks line segments b= b
fdlh ,d point ij intersect djrh gSA nksuksa line segment Note : Alternate angles equal gksrs gSaA
dk origin point same gksrk gSA
(3) Vertically opposite Angles / lEeq[k dks.k :
c a
i.e T= Angle (dks.k) a a
c
z Parallel lines / lekarj js[kk : Parallel lines dHkh Hkh ,d b b
nwljs ls ugha feyrh gS rFkk buosQ chp dk distance ges'kk same b
b a
jgrk gSA
i.e Vertically opposite Angles
lkeka
leka rjrjjs[jskk[kk
a= a, b= b, c, = c
z Transverse Line / fr;Zd js[kk : Transversal line nks ;k a= a, b= b
nks ls vf/d parallel lines dks dkVrh gSA
c= c
Transverse line
a b 180
(fr;Zd js[kk)
(4) ,d gh rjiQ osQ nks interior angles dk sum 180° gksrk gSA
z Basic of Angles
(1) Corresponding Angles / laxr dks.k :
a c

c
a b d

d b

a b 180 c d 180
c a
(5) ,d gh rjiQ osQ nks exterior angles dk sum 180° gksrk gSA
d b a x

(corresponding Angles)

a= a
b y
b= b
c= c
d= d x y 180
Note : Corresponding Angles equal gksrs gSaA a b 180

BLAM–255
T;kfefr
A
Triangle / f=kHkqt : 3 sides ls f?kjs gq, region dks
triangle dgrs gSaA
;gk¡] ABC ,d triangle gSA
Note : fdlh Hkh triangle dh sides dks mlosQ opposite
angles osQ small letter osQ form esa fy[kk tkrk gSA
B C
A ;gk¡ ABC esa] A < 90°, C < 90° gSA
B < 90°,

BC a gSA
ABC acute angled triangle

AC b
c b (b) Right Angled Triangle / ledks.k f=kHkqt : Triangle
AB c ftldk dksbZ ,d angle 90° dk gks] Right angled Triangle
B a C dgykrk gSA
A
Types of Triangle :
(i) On the basis of sides / Hkqtk ds vk/kj ij
(a) Equilateral Triangle / leckgq f=kHkqt : Triangle
ftldh rhuksa sides equal gksA
AB = BC = CA ;k a = b = c
B C
A
;gk¡ ABC esa] B = 90
ABC right angled triangle gSA
c b (c) Obtuse Angled Triangle / vf/ddks.k f=kHkqt :
Triangle ftldk dksbZ ,d angle 90° ls cM+k rFkk 180° ls NksVk
gksrk gS] Obtuse angled triangle dgykrk gSA
B a C A

(b) Isosceles Triangle / lef}ckgq f=kHkqt : Triangle


ftldh dksbZ nks sides equal gksA
;gk¡] AB = BC
;k] c = a
B B C
;gk¡] ABC esa]
90° < B < 180° gSA
c a
ABC, obtuse angled triangle gSA
Nature of Triangle / f=kHkqt dh izÑfr %
A C (i) ;fn fdlh triangle esa fdlh ,d side dk square ckdh
b
(c) Scalene Triangle / fo"keckgq f=kHkqt : Triangle ftldh nksuksa sides osQ squares osQ sum osQ cjkcj gks rks] triangle
right angled triangle gksxkA
rhuksa sides unequal gksA
[a2 + b2 = c2 ]
;gk¡ a b c ;k AB BC CA
Example : 4 cm, 3 cm rFkk 5 cm okyk triangle fdl
A izdkj dk gksxk \
c b Sol. ;gk¡] 52 = 42 + 32 (25 = 25)
Triangle ,d right angled triangle gSA
B a C (ii) ;fn fdlh triangle osQ nksuksa sides osQ square dk sum

(ii) On the basis of angle / dks.k ds vk/kj ij rhljh side osQ square ls NksVk gks] rks triangle obtuse angled
triangle dgykrk gSA
(a) Acute Angled Triangle / U;wudks.k f=kHkqt : Triangle
[a2 + b2 < c2]
ftlosQ rhuksa angles 0° ls cM+s rFkk 90° ls NksVs gks] Acute angle
Example : 3 cm, 5 cm, rFkk 9 cm okyk triangle fdl
Triangle dgykrk gSA
izdkj dk gksxk\
BLAM–256
T;kfefr
Sol. : ;gk¡] 32 + 52 < 92 = (34 < 81)
A D
Triangle obtuse angled triangle gSA
(iii) ;fn fdlh triangle ds nksuksa sides osQ square dk sum
rhljh side osQ square ls cM+k gks rks triangle acute angled
triangle dgykrk gSA
Example : 5 cm, 6 cm, rFkk 7 cm okyk triangle fdl B C E F
izdkj dk gksxk \
Sol. (5)2 + (6)2 > (7)2 (61 > 49) AB BC AC

Triangle acute angled triangle gSA DE EF DF
Similar Triangles / le:i f=kHkqt : ;fn fdlh ,d ABC ~ DEF
triangle ds rhuksa angles nwljs triangle osQ corresponding
(iii) S – A – S (Side – Angle – Side) : ;fn nks triangles
angles osQ equal gks rks] nksuksa triangles similar gksaxAs
osQ corresponding sides dk ,d Pair Proportional gks rFkk
;gk¡] A = A', B = B', C = C' gSA Included Angles (varfjr dks.k) cjkcj gks rks os triangles
ABC vkSj A'B'C' similar gksaxs
Similar gksaxAs
A A'
A D

B C B' C'
B C E F
;fn nks triangles similar gksrs gSa rks mudh corresponding
sides dk ratio equal gksrk gSA AB AC
 and LA = LD
DE DF
AB BC CA
i.e. ABC ~ DEF.
A' B' B' C' CA
le:irk izes; (Symmetry Theorem) : – *Corollary (A – A Similarity) : ;fn ,d triangle osQ
nks Angles nwljs triangle osQ nks Angles osQ Øe'k% (respectively)
(i) A – A – A (dks.k&dks.k&dks.k)
cjkcj gks rks ;s nks triangles similar gksaxAs
(ii) S – S – S (Hkqtk&Hkqtk&Hkqtk)
(iii) S – A – S (Hkqtk&dks.k&Hkqtk) A D
(i) A – A – A (Angle – Angle – Angle) : ;fn nks
f=kHkqtksa osQ Corresponding Angles cjkcj gks rks triangles
Similar gksaxaAs

A D B C E F

 A = D and B= E
ABC ~ DEF.
* nks Similar Triangles esaµ
B C E F
a1 P1 h1 M1 A1
 A= D a2 P2 h2 M2 A2
B= E ABC ~ DEF
where,
C= F
a = sides
(ii) S – S – S (Side – Side – Side) : ;fn nks triangles
p = perimeter
osQ Corresponding Sides Proportional gksa rks os triangles h = height
Similar gksaxAs

BLAM–257
T;kfefr
M = Median
A D
A = Area.
lok±xle f=kHkqt : ;fn nks
Congruent Triangles /
dk shape vkSj size same gks rks nksuksa triangle
triangles
congruent gksrs gSaA
B C E F

A D ;gk¡] AB = DE, BC = EF vkSj AC = DF gSA


ABC DEF gksxkA
# (2) S-A-S (Side-Angle-Side) : ;fn fdlh triangle dh nks
sides vkS j muos Q chp dk angle fdlh nwl js triangle dh
corresponding sides vkSj angle osQ cjkcj gks rks (S-A-S)
B C E F
congruency gksrh gSA
;gk¡] A= D A D
B= E
C= F
AB = DE
BC = EF B C E F
CA = FD
;gk¡] AB = DE, AC = DF vkSj A= D gSA
nksuksa triangle congruent gSA
ABC DEF gksxkA
Note : nks congruent triangles ges'kk similar gksrs gSaA (3). A-S-A (Angle-Side-Angle) : ;fn fdlh triangle osQ
ysfdu nks similar triangles t:jh ugha dh congruent gks] oks dksbZ nks angles vkSj muosQ chp dh side fdlh nwljs triangle osQ
congruent gks Hkh ldrs gSa ugha Hkh gks ldrsA corresponding angles vkSj side os Q equal gks rks (A-S-A)
For example : congruency gksrh gSA
A E
D

B C D F
;gk¡] A= E, C= F vkSj AC = EF gSA
B C ABC DEF gksxkA
E F (4) R.H.S. (Right-Hand-Side) : ;fn fdlh right angled
ABC ~ DEF (~ Similar dk sign) triangle dh dksbZ nks sides fdlh nwljs right angled triangle osQ
D;ksafd] A= D corresponding sides osQ cjkcj gks rks (R.H.S) congruency
B= E gksrh gSA
C= F A F
ysfdu ABC DEF ( Congruent ugha dk sign)
D;ksafd] AB DE
BC EF
CA FD
Congruency related theorems / lok±xlerk laca/h
izes; B C D E

(1) S-S-S (side-side-side) % ;fn fdlh triangle dh rhuksa ;gk¡] B = D = 90° vkSj AB = DF
sides fdlh nwljs triangle dh rhuksa corresponding sides osQ rFkk AC = EF gSA
cjkcj gks rks (S - S - S) congruency gksrh gSA ABC DEF gksxkA

BLAM–258
T;kfefr
Right Angled Triangle (ledks.k ) ls Releated a
theorem :– (i) Area of Isosceles triangle = 4b 2 a2
4

A
B 4b 2 a 2
(ii) the height of Isosceles triangle =
2
or (iii) ;fn fdlh triangle dh nks Medians cjkcj gks rks og
triangle Isosceles triangle gksxkA

A
B C C D A

F E
(i) AB × BC = BD × AC

AB AC
BD BC
B C
2
AB BE = BF
(ii) AB2 = AD × AC or AD =
AC ABC, Isosceles triangle gksxkA
z equilateral Triangles ls Related theorems :–
BC2
(iii)CD = or BC2 = AC × CD
AC A
1 1 1 a a
(iv)
BD 2 AB 2 BC 2
h
(v) BD2 = AD × DC
(vi) ;fn P and Q Øe'k% ABC osQ sides CB rFkk BA dk
B C
Mid–Point gS rFkk B = 90° rks K a

A 3 2 4A
(i) A = a a=
4 3

3 2h
(ii) h = a a
2 3
(iii)3a2 = 4h2 or a2 + a2 + a2 = 4h2

B C (iv) A
P

(a) 4 CQ2 = 4BC2 + AB2


h2
(b) 4 AP2 = 4AB2 + BC2
(c) 4 (CQ2 + AP2) = 5AC2 h3

Isosceles Triangles ls Related theorems–


h1
B C
A
H = h1 + h2 + h3
1
C
b b=c A=
3
bh 1 h2 h3 g 2

2
a=
3
bh
1 h2 h3 g
B C
a
(v) If AD BC then AD 2 3BD 2

BLAM–259
T;kfefr
A C

Z
X D
E O
B D C
Where ABC is a equilateral FY
A B
(vi) A ;gk¡] CF bisects C
BE bisects B vkSj
F E AD bisects A
O rhuksa angle bisectorsdk meeting point gSA
Note : Incentre ls triangle ij cuk, x, perpendiculars
B D C
dh length equal gksrh gSA
Inequilateral Incentre dks centre ys dj vkSj perpendicular length
3 (AB2 + BC2 + AC2) = 4 (AD2 + BE2 + CF2) dks radius ysdj triangle osQ vanj cuk;k x;k circle, In circle
Median / ekfè;dk : fdlh Hkh triangle dk median dgykrk gSA rFkk incircle triangle dh rhuksa sides dks touch
opposite side dks equal parts esa divide djrk gSA djrk gSA
C C
incircle
r r
O
r
A B
A D B (i)

;gk¡] CD, ABC dk median gSA D;ksafd ;g AB dks nks A


equal parts esa divide djrk gSA
i.e. AD = DB o
Centroid / dsUæd : ftl point ij triangle osQ rhuksa
medians ,d nwljs dks dkVrs gSa] centroid dgykrk gSA
B C

C A
BOC = 90° +
2

F E B
O AOC = 90° +
2
C
A D B AOB = 90° +
2
;gk¡] ABC dk centroid 0 gSA ABC ABC] (ii) fdlh Hkh triangle dk Area mldh var% f=kT;k (In-
Note : Centroid fdlh Hkh triangle osQ medians dks Redius) r rFkk Semi-perimeter (v¼Z ifjfefr) dk xq.kuiQy
(Product) gksrk gSA
2 : 1 osQ ratio esa divide djrk gSA
AO OC OB 2 r s
;gk¡ ] OE OD OF 1
Incentre / vUr% dsUæ : fdlh triangle osQ interior angle r
S
osQbisector ftl point ij feyrs gSa mls triangle dk incentre
= area of triangle
dgrs gSaA r = In-radius

BLAM–260
T;kfefr
S = Semi-perimeter
A
a b c
where S =
2
Circumcentre / ifjdsUæ : fdlh Hkh triangle osQ F E
perpendicular bisectors tgk¡ feyrs gSa mls circumcentre
dgrs gSaA

B B D C

BOC = 180° – A
AOC = 180° – B
F E AOB = 180° – C
fofHkUu triangle es a os Q Unz (Centre) ls Related
O Important Results :–
A D C (i) In Right-angled Triangle:—

;gk¡] O, ABC dk circumcentre gSA


A
Note % (i) Circumcentre ls triangle osQ vertex dh distance
equal gksrh gSA
i.e. AO = BO = CO F C
a
(ii) Circum Circle / ifjo`Ù k : Circumcentre dks
triangle dk centre ysdj vkSj circumcentre ls vertex dh D
length dks radius ysdj cuk;k x;k circle, circum circle
dgykrk gSA circum circle, triangle osQ rhuksa vertex ls gksdj
xqtjrk gSA B E c
b
(i)

OE = OF = OD = r (In-radius)
A
a b c
In-radius (r) =
2
O C
Circum-radius (R) =
2
(ii) In Isosceles triangle :–
B C
A

BOC = 2 A b h b
AOC = 2 B
AOB = 2 C
abc
(ii) R = where B C
4 D
R = circum-redius
= area of a
a, b, c, are the sides of ABC
yEcosQUnz (Ortho-Centre) : fdlh triangle osQ verticals FGb – a IJ a
ls lkeus okyh Sides ij Mkyk x;k perpendicular ftl point H 2K 2
r=
ij feyrk gSA mls Ortho-centre dgrs gSa h

BLAM–261
T;kfefr
a2 ABC esa BC dks D rd extend fd;k x;k gSA
R= Exterior angle = Sum of two opposite interior
2h
angles.
(iii)In equilateral triangle :–
i.e. ACD = CAB + ABC
A (4) fdlh Hkh triangle esa] equal sides osQ opposite okys
angles ges'kk equal gksrs gSaA
a a ;fn] AB = AC gks rks] B= C gksxkA
A
o

B a C

a B C
r=
2 3 (5) fdlh Hkh triangle esa equal angles osQ opposite okyh
sides ges'kk equal gksrh gSA
a B
R=
3

r 1
R 2r always
R 2
Some important theorems related to triangle /
A C
f=kHkqt ls lacafèkr dqN egRoiw.kZ çes; %
;fn] B = C gks rks] side AC = AB gksxkA
(1) fdlh Hkh triangle osQ rhuksa interior angles dk sum
(6) fdlh Hkh triangle osQ fdUgha nks sides dk sum rhljs side
180° gksrk gSA
ls cM+k gksrk gSA
A C

B C
A B
A + B + C = 180°
AB + BC > CA ;k AB + AC > BC ;k BC + CA > AB
(2) fdlh Hkh triangle osQ rhuksa exterior angles dk sum
(7) fdlh Hkh triangle osQ fdUgha nks sides dk varj rhljh side
360° gksrk gSA
i.e., A + B + C = 360°
ls NksVk gksrk gSA
C
C

A B
AB – BC < CA ;k
A
B AB – AC < BC ;k
BC – CA < AB
(3) fdlh Hkh triangle dk exterior angle, opposite (8) fdlh Hkh triangle esa] cM+s angle dh opposite okyh
interior angles osQ sum osQ cjkcj gksrk gSA side smaller angle osQ opposite okyh side ls cM+h gksrh gSA
A C

B D A
C
B

BLAM–262
T;kfefr
;fn A> B rks BC > AC
C
(9) fdlh Hkh triangle esa cM+s side osQ opposite okyk angle
NksVs side osQ opposite okys angle ls cM+k gksrk gSA
C
Y X

O
;fn AC > BC rks B> A A Z B
A
B ;fn fdlh Hkh triangle osQ internal bisector fdlh point 0
fdlh Hkh right angled triangle esa , hypotenuse
(10) ij feys rks
lcls cM+h side gksrh gSA ;gk¡ longest side = BC
C A

O C
/2 /2
B
A B B/ 2 C/ 2
(11) fdlh Hkh triangle osQ rhuksa medians dk sum triangle B C
1
osQ 2 perimeter ls T;knk gksrk gSA A
BOC = T 90
2
C
Note : (fn;k x;k relation cgqr important gS] bl ij based
questions cgqr ckj iwNs tk pqosQ gSaA)
F D 180° T ("Reflexive Angle") ฀ 360° : dksbZ angle T ,
O 180° ls cM+k rFkk 360° ls NksVk gks rks mls reflexive angle dgrs
gSaA
A E B
Complementary Angles : dksbZ nks angles dk sum 90° gks
;fn AD, CE vkSj BF, ABC osQ median gks] rks
rks og complementary angles dgykrs gSaA
AB BC CA
AD + CE + BF >
2 a b 90
(12) ;fn fdlh triangle dh nks sides dks extend fd;k tk,
* Supplementary Angles : dksbZ nks angles dk sum 180°
rFkk exterior angles osQ bisectors fdlh point 0 ij feys rks
gks rks oks supplementary angles dgykrs gSaA
‘ACB
‘AOB = 90q – a b 180
2
C PROPERTIES :
Property 1. : ;fn fdlh ABC esa, AD, A dk interior
bisector gks rksA
A B
A

O
X Y
Note : (fn;k x;k relation cgqr important gS] bl ij
based questions cgqr ckj iwNs tk pqosQ gSaA)
(13) fdlh Hkh triangle dk perimeter mlosQ vertex ls cuk,
x, perpendiculars osQ sum ls T;knk gksrk gSA
B D C
i.e. (AB + BC + CA) > (AX + BY + CZ)

BLAM–263
T;kfefr
AB BD Mid-point theorem (eè; fcanq izes;) :
(i)
AC DC A
Property 2. : ;fn fdlh ABC esa, AD, A dk exterior
bisector gks rksA
A
D E

B C
ABC esa] ;fn (D,E), AB rFkk AC osQ mid-points gks rks
B C D
(i) DE||BC
AB BD
(ii) 1
AC DC DE BC
(ii)
Example based on property I 2
Ex. In ABC, AD is interior bisector of Angle A if AB
= 10cm and AC=14cm and DC = 6cm. Find the length Area of ADE
of BD ? (iii) 1
= Area of ABC
ABC esa, AD, dk lef}Hkktd gSA ;fn AB = 10 lseh rFkk
A 4
AC=24 lseh rFkk DC = 6 lseh gks rks] BD dh yEckbZ Kkr djsa ? Quadrilateral / prqHkqZt : 4 sides ls enclosed region dks
Sol. A quadrilateral dgrs gSAa
A D A D

C
B B C

D
B D C A D
w.k.t
AD, BAC dk angle bisector gS A C

AB BD
B C
AC DC B

10 BD prqHkZqt osQ izdkj (Types of Quadrilateral)


=
24 DC
1. Rectangle / vk;r : Quadrilateral ftldh opposite
5 5
u6 =
5 sides equal gks rFkk each angle 90° dk gks rectangle dgykrk gSA
BD = × DC =
12 12 2
A D
BD 2.5 cm

Median (ekfè;dk) : O O
(i) Opposite side dks equal parts esa divide djrh gSA B C
AD median gS] BD = DC
;gk¡ ABCD esa]
A
AB = CD vkSj AD = BC rFkk
A= B= C= D = 90°
a b Median (i) Rectangle osQ diagonals equal gksrs gSaA AC = BD
(ii) Rectangle osQ diagonals ,d nwljs dks 90° ij bisect
djrs gSaA
B D C

BLAM–264
T;kfefr
AOB = AOD = BOC = COD = 90° (i) Cyclic quadrilateral osQ opposite angles dk sum 180°
rFkk AO = OC = OB = OD gksrk gSA
(iii) Rectangle dh opposite sides parallel rFkk equal D
gksrh gSA A
2. Square / oxZ : Quadrilateral ftldh pkjksa sides equal
vkSj pkjksa angles 90° osQ gksrs gSa square dgykrk gSA
C
B
A D
;gk¡ ABCD ,d cyclic quadrilateral gSA
Z : Quadrilateral ftlds
5. Parallelogram / lekukUrj prqHkqt
O
opposite sides equal vkSj parallel gks parallelogram dgykrk gSA
B C
(i) Parallelogram osQ opposite angles equal gksrs gSaA
;gk¡ ABCD esa] A= C vkSj B= D
AB = BC = CD = DA rFkk A = B = C = D = 90° (ii) Parallelogram dk izR;sd diagonal parallelogram dks nks
ABCD ,d square gSA equal triangles esa divide djrk gSA
(i) Square osQ nksuksa diagonals cjkcj gksrs gSaA BCD ABD vkSj ACB ACD
(ii) Square osQ nksuksa diagonals ,d nwljs dks 90° ij dkVrs gSaA (iii) Parallelogram osQ diagonals ,d nwljs dks bisect djrs gSaA
AOD = DOC = COB = BOA = 90° AO = OC rFkk BO = OD
rFkk AO = OB = OC = OD (iv) Parallelogram osQ mid-points dks join djus ls cuk
(iii) Square dh opposite sides parallel gksrh gSaA quadrilateral Hkh parallelogram gksrk gSA
3. Trapezium / q : Quadrilateral ftldh
leyEc prqHkZt A Q
D
one pair of opposite sides parallel gksrh gS rFkk ckdh nks sides
parallel ugha gksrh gS trapezium dgykrk gSA O
P R
A D
B C
S
P, Q, R, S sides A B, BC, CD, DA osQ mid points gSA
B C rFkk ABCD Parallelogram gSA
;gk¡ ABCD esa AD rFkk AB D C
BC PQRS ,d parallelogram gSA
ABCD Trapezium gSA (v) Parallelogram osQ sides os Q square dk sum blosQ diagonals
4. Rhombus : / fo"kedks.k leprqHkqt Z : Quadrilateral osQ square osQ sum dk vk/k gksrk gSA
ftlosQ pkjksa sides equal rFkk opposite sides parallel gksrs gSa 1 2
Rhombus dgykrk gSA Rhombus dk angle 90° dk ugha gksrk gS
l 2  b2
2
d
d1  d 22 i
tcfd blosQ diagonals 90° ij bisect djrs gSaA
AOD = DOC = COB = AOB = 90° i.e, 2 l  b
2
d
2
i dd 2
1  d 22 i
AO = OC = OB = OD Question based on above relation :
Q.1 Length of two adjacent sides are 16cm, 12.cm
A D the length of one diagonal is 14cm. Find the length of
other diagonal. / nks vklUu Hkqtkvksa dh yEckb;k¡ 16 lseh- rFkk
12 lseh gSa rFkk ,d fooQ.kZ dh yEckbZ 14 lseh gSA nwljs fod.kZ dh
O yEckbZ Kkr djsa\
Sol. w.k.t
B C
e
2 l2 j
b 2 = d12 d 22
;gk¡ AB = BC = CD = DA rFkk AB CD vkSj BC AD gSA
ABCD rhombus gSA 2e16 2
j
122 = 142 d 22
6. fdlh quadrilateral osQ pkjksa vertex dks touch djrs gq,
dksbZ circle pass gks rks og quadrilateral, cyclic quadrilateral 2(256 144 ) = 196 d 22
gksrk gSA

BLAM–265
T;kfefr
z Property 4 : “Trapezium” (leyac prqHkqZt)
d22 = 800 – 196
A B
d22 = 604 X

diagonal d2 604 cm D C

SOME NOTABLE POINTS / oqQN mi;ksxh fcUnq ar ( ABD) = ar ( ACB)


ar ( DBC) = ar ( CAD)
(i) fdlh Hkh quadrilateral dk perimeter mlosQ diagonals
(Same base rFkk same parallels ds chp esa cus gSa)
osQ sum ls T;knk gksrk gSA z Property 5 :-
(ii) fdlh Hkh quadrilateral osQ diagonals dk sum qua- (Base BC dks rhu cjkcj Hkkxksa esa Hkkx djus ij)
drilateral osQ opposite sides osQ sum ls T;knk gksrk gSA
D
z
A
Property 1 : ;fn nks paralleogram dk base same gks rks
mudk area cjkcj gksxkA
b
rectangle

1
2 h(same)
C P Q B

Area of AQB : ABCD dk Area = ?


1
× ×1:3×
(i) = (2) equal. 2
Area 1 = Area 2 [D;kafd Area = Base × Height] 1:6
z rFkk same parallels ij cus z
Property 2 : “Same base
Property 6:–
f=kHkqt (Triangle) dk Area cjkcj gksxk**A fdlh Parallelogram esa]
A D

A2
A3
A1

B C

AC 2 + BD2 = 2 (AB2 + BC2)


z Property 7:–
fdlh Rhombus esa]
a
A D
A 1 = A2 = A3 (Area equal)
z Property 3 :- ;fn ,d leku vk/kj (base) rFkk mQ¡pkbZ
a
(Height) leku gkssus ij a

B C
a
rectangle triangle AC 2 + BD2 = 4AB2 = 4BC2 = 4CD2 = 4AD2
let AC = d1
BD = d2
and AB = BC = CD = AD = a
1
Area of triangle = Area of rectangle then d12 d 22 4a 2
2

BLAM–266
T;kfefr
z Property 8:– Proof :
fdlh Trapezium esa] A
(i)

A D

E F D E

B C
B C
ABCD is a Trapezium
ADE ~ ABC
E and F are mid Point of AB and CD
AD AE
1
EF =
2
ADb BC g BD EC
* IMP Q (Asked Several times) *
(ii) A D Q.1 In a ABC , C = 90° CD is perpendicular on
the side AB. If BC = a, CA =b, AB=c, CD = p, find
E F relation among it (a, b, p).
M N
(ledks.k ABC esa C ledks.k gSa CD, AB ij yac gSA BC
= a, CA =b, AB=c, CD=p gS rks a, b, p esa D;k laca/ gSA)
B C
Sol. A
1
MN
2
BCb AD g
(iii) A D
c
b D
o

p
B C

OB OC C a B
OD OA
ge tkurs gSa
a2 + b2 = c2 ...(i)
(By Pythagorous theorem)
z Thale’s Theorem (ize s;) : 1 1
i.e a×b= ×c×p
2 2
A
(  Area ACB = Area ACB.)
ab
c= ...(ii)
p
D E C dh value equation 1 esa j[kus ij

a2 + b 2 =
FG ab IJ 2
a 2b 2

B
HpK p2
C
a2 b2 1
fdlh ABC esa] ;fn BC DE gks rks, DE ckfd nks sides dks + =
p2
equal ratio esa divide djsxhA a 2b2 a 2b2
AD AE 1 1 1
BD EC a2 b2 p2

BLAM–267
T;kfefr
z Theorem (ize s;) : Centre /dsUæ : oSlk point tgk¡ ls circle ds pkjksa vksj ?ksjs dh
(i) Obtuse triangle (vf/ddks.k): nwjh leku gks dsUnz dgykrk gSA ;gk¡ “O” circle dk dsUnz gSA
Radius / f=kT;k : Circle ds dsUnz ls circumference dks
A feykusokyhline radius dgykrh gS] bls r ls fu#fir fd;k tkrk gSA
Some important theorems based on circle/ o`Ùk
ij vkèkkfjr dqN izeq[k izes;
Obtuse angle (1) The angles made by equal chords on the circum
"draw a" Perpendicular
(vf/ddks.k) ference of circle are equal./ leku thok }kjk o`Ùk dh ifjfèk ij
( ,d yac [khapk ) cus dks.k vkil esa leku gksrs gSaSA
D C
D B C
ADC esa
AC 2 = AD2 + DC2
= AD2 + (BD + BC2) A B
= AD2 + BD2 + BC2 + 2BD × BC
(2) The angle made by largest chord (diameter) on the
AC 2 AB 2 BC2 2BD BC
circumference of circle is 90° / o`Ùk dh egÙke thok (O;kl)
Note : a2 + b2 = c2 right angle }kjk o`Ùk dh ifjfèk ij cuk dks.k ledks.k (90°) gksrk gSA
a2 + b2 < c2 obtuse angle
C
a2 +b2 > c2 Acute angle
(ii) Acute Angle triangle (U;wudks.k f=kHkqt) :
A A B
O

draw a AAD ;gk¡ AB — o`Ùk dh egÙke thok (O;kl) gSA


ks.k) )
wud gle

(,d yack AD [khpk) C = 90°


(U; t e An

(3) The angle made by same chord on circumference of


the circle is half of the angle made on centre of the circle./
cu

leku thok }kjk o`Ùk dh ifjfèk ij cuk dks.k] o`Ùk ds dsUnz ij cus dks.k
(A

dk vkèkk gksrk gSA


B D C
C
In ADC
AC 2 = AD2 + DC2
= AD2 + (BC – BD)2
= AD2 + BC2 + BD2 - 2BC BD
O
AC 2 AB 2 BC2 2BD BC A B
z Centroid (osQaæd) :
Divide it into 6 equal parts : i.e./ vFkkZr~ AOB = 2 ACB
f=kHkqt dks 6 cjkcj Hkkxksa esa foHkkftr
djrk gSA (4) The perpendicular drawn from centre of the circle
Circle / o`Ùk : fdlh ,d point dks centre esa ysdj cuk;h x;h to the chord of circle divides the chord in two equal parts./
geometrical figure circle dgykrh gSA
o`Ùk ds dsUnz ls thok ij Mkyk x;k yEc thok dks nks cjkcj Hkkxksa esa
ck¡Vrk gSA
E

O
A B
O
vFkkZr~ AC = CB

C D A C B

BLAM–268
T;kfefr
(5) The line joining the mid-point of chord to the centre (8) If two chords of a circle meet at a point outside the
of circle is perpendicular to the chord. / o`Ùk ds thok ds eè; circle then / ;fn fdlh o`Ùk dh nks thok,¡ o`Ùk ds ckgj fdlh fcUnq
fcUnq ls dsUnz dks feykus okyh js[kk thok ij yEc gksrh gSA ij feys]a

A
B
O O
D
C

A C B
;gk¡ o`Ùk dh AB vkSj CD thok o`Ùk ds ckgj c<+ dj O fcUnq
Here, C is the mid-point of AB and O is the centre of
ij fey jgh gS] rks
the circle, then OC AB / ;gk¡ AB dk eè; fcUnq C vkSj O o`Ùk AO BO CO DO
dk dsUnz gS rks
OC AB (9) The perpendicular bisector of chord of any circle
(6) If two chords of a circle intersect each other inside passes through centre of the circle./ fdlh Hkh o`Ùk dh thok
the circle then two triangles thus formed are similar to dk yEc lef}Hkktd o`Ùk ds dsUnz ls gksdj tkrk gSA
each other. / ;fn o`Ùk dh nks thok,¡ o`Ùk ds vUnj ,d nwljs dks C
izfrNsfnr djs rks cus gq, nks f=kHkqt (fp=k esa) vkil esa le:i gksrs gSaA

D
O
B
O
A D B
A Here, CD bisects chord AB at right angle. So, CD passes
through centre O. / ;gk¡ thok AB dks CD yEc lef}Hkkftr dj
C jgk gSA
CD o`Ùk dsUnz O ls gksdj tk,xhA
(10) The equal chords of circle are at equal distance
Here, the chord AB and CD bisect each at O. /;gk¡ o`Ùk from centre/ fdlh Hkh o`Ùk ds leku thok dsUnz ls leku nwjh ij
dh AB vkSj CD, thok O fcUnq ij izfrNsfnr dj jgk gSA gksrs gSaA
A= D and B= C P
C D
,oa DOB = COA
Then, from theorem (1)
So, both triangles are of equal angles and similar. / O
vr% nksuksa f=kHkqt leku dksf.kd ,oa le:i gq,A
(7) Any tangent CD of a circle and chord AB meets at
point C, outside of circle, / o`Ùk dh dksbZ Li'kZ js[kk CD vkSj ,d A Q B

thok AB o`Ùk ds ckgj fdlh fcUnq C ij feysa rks Here, chord AB = chord CD then their distance from
centre is equal. / ;gk¡ thok AB = thok CD gSA rks budh dsUnz ls
CB CA CD 2 gksxkA nwjh leku gksxhA vFkkZr~
OP = OQ
(11) The chord made from equal distance from centre
are equal. / dsUnz ls leku nwjh ij cus thok vkil esa leku gksrs gSaA
A In the above figure, If OP = OQ then AB = CD / mQij
B ds fp=k esa ;fnOP = OQ rks AB = CD
C (12) If two circles cut each other at two points then
from any point from diameter of two circles the line joining
the end-points passes through other cutting point./ ;fn nks

D
o`Ùk ,d nwljs dks fdUgha nks fcUnqvksa ij dkVsa vkSj dVku ds fdlh ,d
fcUnq ls nksuksa o`Ùk dk O;kl cuk;k tk, rks nksuksa o`Ùkksa ds O;kl ds vafre
Nksjksa dks feykus okyh js[kk o`Ùkksa ds nwljs dVku fcUnq ls gksdj xqtjrh gSA

BLAM–269
T;kfefr
(^^;fn thok centre dh vksj gksrh gS rks yackbZ T;knk gksrh gS ,oa
;fn thok centre ls nwj gksrh gS rks thok dh yackbZ de gksrh gSA**)
A

C D
B

Here, two circles A and B cut each other. Then AC and


AD are diameters respectively. / nksuksa o`Ùk ;gk¡ A vkSj B ij ,d (ii) If perpendicular drawn from the centre of circle
nwljs dks dkV jgs gSaA AC rFkk AD Øe'k% budk O;kl gSA on the chord, it bisects the chord.
So, Point B is on line CD./ (;fn thok dks nks cjkcj Hkkxksa esa ckVrs gS rks og os QUnz ls yac gksxk)
z Other properties of circle:-
? B fcUnq CD js[kk ij gksxkA
(13) If perpendicular is drawn from centre of two
concentric circles then, AC = BD. / nks ladsUnzh o`Ùk ds dsUnz ls O
thok ij yEc Mkyk tk, rks] AC = BD gksxkA
2

O
Ch or d (thok) a rc(pki)
C D (i) Angle subtended by an arc at the centre of
A B
circle is the twice of the angle subtended on the
remaining part (circumference).
Circle (o`Ù k) : (^^,d gh thok ls osQUnz in cuk dks.k ifjf/ ij cus dks.k dk nqxuk
gksrk gSA)
E C

F D Ch or d(thok)
A B
A B
" equal length of chords substend equal angle at Ch or d (thok)
centre of circle. (ii) All the angles subtended on the same chord
(" cjkcj yEckbZ dh thok o`Ùk osQ osQ Unz ij ges'kk cjkcj dks.k on circumference are equal.
cukrh gSA) (^^,d gh thok ls ifjf/ ij cus LkHkh dks.k cjkcj gksrs gSaA)
OA = OB [(f=kT;k) (radius)]
AB = CD = EF = [chord (thok)] C D

decreases
A B
O
* diameter (O;kl)
increases

(i) If chord is towards the centre, then length of


chord is more & if we go away from centre, then length (iii) Angles subtended on diameter are right angle
of chord is small. or 90°.

BLAM–270
T;kfefr
(^^O;kl
ls ifjf/ ij cus lHkh dks.k 90° ;k ledks.k gksrs gSA) a+b=b+c
Cyclic Quadrilateral (pozQh; prqHkqt) Z : a c

In cyclic quadrilateral, Angles of Interior & opposite


side exterior angles are equal.(pozQ h; prqHkqZt essa vkeus&lkeus
x
osQ vakrfjd ,ao ckg~; dks.k cjkcj gksrs gSa)
a b i.e A C [Angle A = Angle C]

Properties of commond chord :


y
(1) Two circles whose centre A, B and radii r1 & r2
c.m intersect to each at the two points C and D
such that AC, BC are the tangents of the circle find
(i) Su m o f op pos ite ang les of a c ycl ic common chord.
quadrilaterial is 180°.
(¶nks o`Ùk ftudk osQUnz A o B gS ,oa f=kT;k r1, r2 lseh- gS tks
(pozQ h; prqHkqtZ osQ vkeus&lkeus osQ dks.kksa dk ;ksx 180° gksrk gSA)
,d nwljs dks C o D fcUnw ij bl izdkj ls dkVrs gS fd AC o BC
x y 180 ,d nwljs osQ Li'kZ js[kk gks rks budh mH;fu"V thok CD dh yackbZ
i.e a b 180 D;k gksxhA)

x y a b 360q C
r1 r2
90°
x
(ii) A B
x
2
r 1 + r 22
D
Common chord
(mH;fu"B thok)

ge tkurs gSa
1 1
r12 r22 CD = × r1 × r 2
Whose all vertices touch the circumference. 2 2
(pozQh; prqHkqZt osQ lHkh 'kh"kZ ifjf/ dks Li'kZ djrs gSA)
r1 r2
2 + 2 = 360° CD =
r12 r22
180
r1 r2
Common Chord( mHk; fu"B thok)
r12  r22
A D
(2) Two circles having same radius and passing
a through their centres.
(nks cjkcj o`Ùk tks ,d nwljs osQ osQUnz ls gksdj tkrs gSA izR;sd
b c exterior angle dh f=kT;k r lseh gS rks budh mH;fu"B thok dh yackbZ D;k gksxhA)
B C
A

r r
Opposite angles h
a + b = 180° B r C

b + c = 180° b c (Straight line) Angle

BLAM–271
T;kfefr
AB = AC = BC
ABC is an equilateral triangle (leckgq gksxk) Direct C.T = d2 br
1 r2 g 2

^^leckgq f=kHkqt dh mQpkbZ =h where d = distance between two centers (nks


[Height of a equilateral triangle = h] osQUnzksa osQ chp dh nwjh )
w.k.t
r1, r2 = (f=kT;k,¡) radii
3 Transverse Common Tangent (fr;Zd mHk;fu"B Li'kZ
h= r
2 js[kk)
3
Common Chord ( mHk; fu"B thok ) 2h 2u r
2 r1 r2

2h 3r d

(3) Two circles intersect to each other, length of the


common chord is 2x & their radius are r1 & r2 cm.
Find the distance between their centre. Transverse C.T (fr;Zd js[kk) =
(nks o`Ùk tks ,d nwljs dks dkVrs gS] mH;fUk"B thok 2x gS rFkk
f=kT;k,¡ r1, r2 gS rks nksuks o`Ùkksa osQ osQUnz osQ chp dh nwjh D;k gksxh\)
T.C.T = d2 br1 r2 g 2

z fdlh triangle osQ dks.kksa (Angles) A, B and C osQ


O
Bisector ml triangle osQ ifjo`Ùk (Circum-circle) dks
r1 r2
x Øe'k% fcUnqvksa D, E, F ij cut djrs gS rks DEF osQ dks.k
A B
Dx FG 90 – A IJ, FG 90 B IJ rFkk FG 90 C IJ gksaxsA
H 2K H 2 K H 2 K
w.k.t
In OAD, A
OA 2 = OD2 + AD2

r12 = x2 + AD2
F E
AD 2 = r12 - x2 (i)
In OBD,
OB2 = OD2 + BD2
BD2 = OB2 – OD2
B C
= r22 – x2 (ii)
Add (i) & (ii)

AD 2 BD 2 r12 r22 2x 2
D

Direct Common Tangent(mHk;fu"B Li'kZ js[kk)


C.T (mH;fu"B) A
FDE 90
2
(r1–r 2) 90°
A
B
B
FED 90
d 2
r2 r1
d
C2 C1 C
DFE 90
2

‡‡‡

BLAM–272
T;kfefr

PREVIOUS YEARS’ QUESTIONS


(1) 19 (2) 20 (3) 28 (4) 21
Type-1 (SSC FCI Assistant Grade-III Main Exam. 07.04.2013)
Questions Based on Angles, 7. The sides of a triangle are 16 cm, 12 cm and 20
(Complementary, supplementry, angle cm. Find the area.
bisector etc. ,d f=kHkqt dh Hkqtk,¡ 16 lseh, 12 lseh vkSj 20 lseh gSaA {ks=kiQy
Kkr dhft,A
1. If two supplementary angles differ by 44°, then (1) 64 cm2 (2) 112 cm2
one of the angles is (3) 96 cm2 (4) 81 cm2
;fn nks laiwjd dks.kksa esa 44° dk varj gS rks muesa ls ,d dks.k gS (SSC CHSL DEO and LDC Exam, 20.10.2013)

(1) 68° (2) 65° (3) 102° (4) 72° 8. In the given figure below, If AB || CD and CE
(SSC CGL Tier-I Exam, 09.08.2015 ED, then the value of x —
(IInd Sitting) TF No. 4239378)
uhps fn, x, fp=k esa] ;fn AB || CD vkSj CE ED, rks x
2. Two supplementary angles are in the ratio 2 : 3.
The angles are dk eku gSµ
nks laijw d dks.k 2 : 3 ds vuqikr esa gSaA dks.k fdruh fMxzh ds gSa \
E
(1) 33°, 57° (2) 66°, 114°
A 37° B
(3) 72°, 108° (4) 36°, 54°
(SSC CGL Tier-I Re-Exam. (2013) 20.07.2014 (IInd Sitting) 90°
3. If the complement of an angle is one–fourth of its
supplementary angle, then the angle is x°
;fn fdlh dks.k dk iwjd mlds laijw d dks.k dk ,d&pkSFkkbZ gks C D
rks dks.k D;k gksxk\
(1) 53° (2) 63° (3) 37° (4) 45°
(1) 60° (2) 30° (3) 90° (4) 120° (SSC CPO SI & Assistant Intelligence Officer Exam, 2012]
(SSC CHSL (10+2) LDC, DEO & PA/SA Exam, 20.12.2015 9. In the figure given below, line k and l are parallel
(Ist Sitting) TF No. 9692918)
to each other. The value of a° + b° is —
4. The measure of an angle whose supplement is three
uhps fn, x, fp=k esa] js[kk,¡ k vkSj l lekarj gSA a°+ b° dk
times as large as its complement, is
eku gSµ
ml dks.k dk eki D;k gS ftldk laiwjd blds iwjd ds rhu xquk
ftruk cM+k gS \
k
(1) 75° (2) 30° (3) 45° (4) 60° a°
(SSC CGL Tier-I Exam, 09.08.2015
(Ist Sitting) TF No. 1443088)
5. The angles of a triangle are (x + 5)°, (2x – 3)° and 45°
(3x + 4)°, What will be the value of x ?
,d f=kHkqt ds dks.k (x + 5)°, (2x – 3)° vkSj (3x + 4)° gSaA b°
x dk eku gSµ l

(1) 30° (2) 31° (3) 29° (4) 28°


(1) 45° (2) 100° (3) 180° (4) 360°
(SSC FCI Assistant Grade Exam. 2012)
(SSC CGL Tier-I Exam, 2012 and SSC CPO SI &
1 1 1 Assistant Intelligence Officer Exam, 2012)
6. The sides of a triangle are in the ratio : :
4 6 8 10. Taking any three of the line segments out of seg-
and its perimeter is 91 cm. Then what will be the ments of length 2 cm, 3 cm, 5 cm and 6 cm, the
difference of longest and smallest sides ? number of triangles that can be formed is :

1 1 1 2 lseh-] 3 lseh-] 5 lseh- vkSj 6 lseh-yackbZ okys [kaMksa esa ls fdUgha


,d f=kHkqt dh Hkqtk,¡ : :
4 6 8
ds vuqikr esa gSa vkSj mldk rhu js[kk&[kaMksa dks ysdj fdrus f=kHkqt cuk, tk ldrs gS\
ifjeki 91 lseh gSA rnuqlkj mldh lcls yach vkSj lcls NksVh (1) 3 (2) 2 (3) 1 (4) 4
(SSC Graduate Level Tier-I Exam. 21.04.2013)
Hkqtkvksa dh yackbZ dk varj fdruk gS\

BLAM–273
T;kfefr
11. In ABC, A+ B = 65°, B+ C = 140°, then
1
find B. 17. In ABC, AD is median and AD BC. If BAD
2
,d ABC esa] A + B = 65°, B + C = 140° gSa] = 30°, then ACB will be—
rks mlesa B Kkr dhft,A
1
(1) 40° (2) 25° (3) 35° (4) 20° ABC esa] AD ekfè;dk gS] vkSj AD 2
BC. rnuqlkj ;fn
(SSC Graduate Level Tier-I Exam. 19.05.2013)

12. In triangle ABC, BAC = 75°, ABC = 45°. BC


BAD = 30° gks] rks ACB dk eki fdruk gksxk \
(1) 90° (2) 45° (3) 30° (4) 60°
x (SSC CGL Tier-I Exam, 2012)
is produced to D. If ACD = x°, then % of
3 18. ABC is a obtuse angle triangle in which A is
60° is obtuse and O is its orthocentre, If, ‘BOC = 54°
f=kHkqt ABC esa] BAC = 75°, ABC = 45°. BC dks D the what will be ‘BAC ?
,d vf/d dks.kh; f=kHkqt ABC esa A vf/d dks.k gS vkSj O
x
rd c<+k;k x;k gSA ;fn ACD = x°, rks 60° dk 3
% mldk yacdsanz gSA rnuqlkj] ;fn ‘BOC = 54° gks] rks ‘BAC
fdruk gksxk\
gSµ
(1) 108° (2) 126° (3) 136° (4) 116°
(1) 30° (2) 48° (3) 15° (4) 24°
(SSC CGL Tier-I Exam, 2012)
(SSC Graduate Level Tier-I Exam. 19.05.2013 Ist Sitting)
19. The bisectors of ‘B and ‘C of a '$BC meets at
13. In a ABC, if 2 A=3 B = 6 C, value of B is
point P. If ‘BAC= 80°, then ‘BPC will be ?
ABC
'ABC ds ‘B rFkk ‘C ds ckã f}Hkktd P fcanq ij feyrs gSaA
;fn 2 A = 3 B = 6 C gS] rks B dk eku gSµ rnuqlkj] ;fn ‘BAC= 80° gks] rks ‘BPC fdruk gksxk\
(1) 60° (2) 30° (3) 45° (4) 90°
(1) 50° (2) 40° (3) 80° (4) 100°
(SSC CHSL DEO & LDC Exam. 02.11.2014 (IInd Sitting)
(SSC CGL Tier-I Exam, 2012)
1 20. In a triangle ABC, BC is produced to D so that
14. In a triangle ABC, A+ B+ C = 140°, then CD = AC. If BAD=111° and ACB = 80°, then
2
B is the measure of ABC is :
,d f=kHkqt ABC esa] BC dks D rd bl izdkj c<+k;k x;k
1
,d f=kHkqt ABC esa A
2
B C = 140° rks B gS fd CD = AC gks tk,A rnuqlkj] ;fn BAD = 111°
vkSj ACB = 80° gks] rks ABC dk eki D;k gksxk\
fdruk gksxk \
(1) 31° (2) 33° (3) 35° (4) 29°
(SSC CAPFs SI & CISF ASI Exam. 23.06.2013)
(1) 50° (2) 80° (3) 40° (4) 60° 21. The side BC of ABC has been extended to D. If
(SSC CGL Tier-II Exam. 21.09.2014)
15. If in ABC, ABC = 5 ACB and BAC = 3 1
ACD = 108° and B= , then what will be
2
ACB, then ABC = ?
;fn ABC es]a ABC = 5 ACB vkSj BAC = 3 ACB ?

gS] rks ABC = ? ABC dh BC Hkqtk, D rd c<+kbZ xbZ gSA rnuqlkj] ;fn
(1) 130° (2) 80° (3) 100° (4) 120° 1
(SSC CHSL DEO & LDC Exam. 20.10.2013) ACD = 108° vkSj B = 2 gks] rks fdruk gksxk\
16. There are 3 points A, O, B on a line-segment and
(1) 36° (2) 72° (3) 108° (4) 59°
C is the point not on line AOB. If AOC = 40o
(SSC CGL Tier-I Exam, 2012)
and OX, OY are interior and exterior bisector of
22. If in a ABC, base BC is extended in both sides
AOC, then what will be BOY ?
then sum of exterior angles of B and C will be —
,d js[kk&[kaM esa rhu fcUnq A,O,B gSa vkSj C ,d ,slk
;fn fdlh f=kHkqt ABC esa] vk/kj BC dks nksuksa vksj c<+k fn;k
fcUnq gS] tks AOB ij ugha gSA rnuqlkj] ;fn AOC=40o gks
tk,] rks B rFkk C ij cká dks.kksa dk ;ksxiQy gksxkµ
vkSj OX, OY, AOC ds Øe'k% vkarfjd ,oa cká f}Hkktd
gksa] rks BOY fdruk gksxk \ A
(1) A (2) A (3) A (4)
2 2
(1) 70o (2) 80o (3) 72o (4) 68o
[SSC CGL Tier-I Exam, 2012 and SSC CPO SI & Assistant
(SSC CGL Tier-I Exam, 2012) Intelligence Officer Exam, 2012]

BLAM–274
T;kfefr
23. The exterior angles obtained on producing the base ABC esa] ABC rFkk ACB ds vkarfjd f}Hkktd I ij
BC of a triangle ABC in both ways are 120° and feyrs gSa] vkSj mlesa BAC = 50° gSA rnuqlkj] BIC dk eki
105°, then the vertical A of the triangle is of
measure
fdruk gksxk\
(1) 105° (2) 115° (3) 125° (4) 130°
,d f=kHkqt ABC ds vk/kj BC dks nksuksa vksj c<+kus ij ml ij
(SSC CHSL DEO & LDC Exam. 10.11.2013, Ist Sitting)
cus ckg~; dks.k Øe'k% 120° rFkk 105° gSA rnuqlkj] f=kHkqt ds 30. The angle between the external bisectors of two
'kh"kZ dks.k A dk eki fdruk gksxk\ angles of a triangle is 60°. Then, the third angle
(1) 36° (2) 40° (3) 45° (4) 55° of the triangle is
(SSC CHSL DEO & LDC Exam. 27.10.2013 IInd Sitting)
,d f=kHkqt ds nks dks.kksa ds cká f}Hkktdksa ds chp cuk dks.k
24. In ABC, B = 60° and C = 40°. If AD and AE
60° gSA rnuqlkj] ml f=kHkqt dk rhljk dks.k fdruk gksxk\
be respectively the internal bisector of A and
perpendicular on BC, then the measure of DAE (1) 40° (2) 50° (3) 60° (4) 80°
is (SSC Graduate Level Tier-I Exam. 11.11.2012 (Ist Sitting)
31. In ABC, AB = AC, O is a point on BC such that
ABC esa, rFkk C = 40° gSA ;fn AD rFkk AE
B = 60°
BO = CO and OD is perpendicular to AB and OE
Øe'k% A dk vkarfjd f}Hkktd rFkk BC ij yac gksa] rks is perpendicular to AC. If BOD = 30° then mea-
DAE dk eki fdruk gksxk \ sure of AOE is
(1) 5° (2) 10° (3) 40° (4) 60°
ABC esa] AB = AC, BC ij O ,d ,slk fcanq gS fd BO =
(SSC CHSL DEO & LDC Exam. 28.10.2012 (Ist Sitting)
25. Internal bisectors of B and C of ABC inter- CO gS vkSj OD, AB ds yacor~ gS vkSj OE, AC ds yacor~ gSA
sect at O. If BOC = 102°, then the value of BAC ;fn BOD = 30°, rks AOE dk eki D;k gS \
is (1) 45° (2) 60° (3) 75° (4) 30°
ABC esa B rFkk C ds vkarfjd f}Hkktd O ij ,d&nwljs dks (SSC CGL Tier-II Exam, 2014 12.04.2015
(Kolkata Region) TF No. 789 TH 7)
dkVrs gSAa rnuqlkj] ;fn BOC = 102° gks, rks BAC fdruk
1
gksxk\ 32. In ABC, BAC = 90° and AB BC . Then the
2
(1) 12° (2) 24° (3) 48° (4) 60°
(SSC CHSL DEO & LDC Exam. 28.10.2012 (Ist Sitting)
measure of ‘ACB is :
1
26. In a triangle ABC, A = 90°, C = 55°, AD BC . ABC esa BAC = 90° vkSj AB BC gSA rnuqlkj
2
What is the value of BAD ?
‘ACB dk eki fdruk gS \
f=kHkqt ABC es]a A = 90°, C = 55°, AD BC . BAD (1) 60° (2) 30° (3) 45° (4) 15°
dk eku fdruk gksxk \ FCI Assistant Grade-III Exam. 05.02.2012 (Paper-I)

(1) 35° (2) 60° (3) 45° (4) 55° East Zone (IInd Sitting)

(SSC GL Tier-I Exam. 19.05.2013 Ist Sitting) 33. The side BC of a triangle ABC is extended to D. If
27. If I is the incentre of a ABC and ‘A = 60°, then 1
‘BIC will be— ACD = 120° and ABC =
2
CAB, then the val-
;fn I, ,d f=kHkqt ABC dk var% dsanz gks vkSj ‘A = 60° gks] ue of ABC is
rks ‘BIC fdruk gksxk\ f=kHkqt ABC dh Hkqtk BC dks D rd c<+k fn;k tkrk gSA ;fn
(1) 100° (2) 120° (3) 150° (4) 110°
1
(SSC CGL Tier-I Exam, 2012)
ACD = 120° vkSj ABC = CAB, rks ABC dk
28. Internal bisectors of angles B and C of a trian- 2
gle ABC meet at O. If BAC = 80°, then the value eku D;k gksxk \
of BOC is (1) 80° (2) 40° (3) 60° (4) 20°
,d f=kHkqt ABC ds B rFkk C ds vkarfjd f}Hkktd O ij (SSC CHSL DEO Exam. 16.11.2014 (Ist Sitting)
feyrs gSaA rnuqlkj] ;fn BAC = 80° gks] rks BOC dk eku 34. If I be the incentre of ABC and B = 70° and C
D;k gksxk\ = 50°, then the magnitude of BIC is
(1) 120° (2) 140° (3) 110° (4) 130°
;fn fcUnq I ABC dk var%dsanz gks vkSj B = 70° vkSj C
(SSC Delhi Police S.I. (SI) Exam. 19.08.2012) & (SSC
FCI Assistant Grade-III Main Exam. 07.04.2013) = 50°, rks BIC dk ifjek.k gS %
29. In ABC, the internal bisectors of ABC and (1) 130° (2) 60° (3) 120° (4) 105°
ACB meet at I and BAC = 50°. The measure of (SSC CGL Tier-I Re-Exam. (2013) 27.04.2014)
BIC is

BLAM–275
T;kfefr
35. ABC is a triangle, in which AC = BC and ‘ABC 40. In ABC, D and E are two mid points of sides AB
and AC respectively. If BAC = 40° and ABC = 65°
= 50°. It’s side BC is extended up to D, so that
then CED is :
BC = CD , ‘BAD = ? ABC esa D vkSj E Øe'k% Hkqtk AB vkSj AC ds nks eè; fcanq
ABC ,d f=kHkqt gS] ftlesa AC = BC vkSj ‘ABC = 50° gSaA ;fn BAC = 40° vkSj ABC = 65° gS rks CED gS %
gSA mldh Hkqtk BC dks D rd c<+k;k x;k gS] rkfd BC = (1) 130° (2) 75° (3) 125° (4) 105°
(SSC CGL Tier-I Exam, 16.08.2015
CD gks tk,A rnuqlkj ‘BAD fdruk gks tk,xk\ (IInd Sitting) TF No. 2176783)
(1) 50° (2) 45° (3) 75° (4) 90° 41. ABC is a triangle and the sides AB, BC and CA
(SSC CGL Tier-I Exam, 2012) are produced to E, F and G respectively. If CBE
36. ABC is an isosceles triangle such that AB = AC = ACF = 130° then the value of GAB is
and B = 35°. AD is the median to the base BC. ABC ,d f=kHkqt gS vkSj AB, BC vkSj CA Hkqtk,¡ Øe'k% E,
Then BAD is:
F ,oa G rd foLrkfjr dh tkrh gSAa ;fn CBE = ACF =
ABC ,d lef}ckgq f=kHkqt gS] ftlesa AB = AC rFkk B = 130° gS rks GAB dk eku gS
35° gSA mlesa AD, vkèkkj BC dh ekfè;dk gSA rnuqlkj]
(1) 100° (2) 130° (3) 80° (4) 90°
BAD fdruk gksxk\ (SSC CGL Tier-I Exam, 09.08.2015
(1) 70° (2) 35° (3) 110° (4) 55° (IInd Sitting) TF No. 4239378)
(SSC GL Tier-I Exam. 21.04.2013, Ist Sitting)
AB BD
37. ABC is an isosceles triangle with AB = AC. The 42. In a ABC, , B = 70 and C = 50°,
AC DC
side BA is produced to D such that AB = AD. If
ABC = 30°, then BCD is equal to then BAD = ?
ABC ,d lef}ckgq f=kHkqt gS] ftlesa AB = AC gSA mlesa Hkqtk AB BD
ABC eas] AC , B = 70 vkSj C = 50° rks
BA dks D rd
c<+k;k x;k gS rkfd AB = AD gks tk,A rnuqlkj] DC
;fn ABC = 30° gks] rks BCD fdlds cjkcj gksxk\ BAD = ?
(1) 45° (2) 90° (3) 30° (4) 60° (1) 60° (2) 20° (3) 30° (4) 50°
(SSC CHSL DEO & LDC Exam. 10.11.2013, IInd Sitting) (SSC CAPFs SI, CISF ASI & Delhi Police SI
Exam. 22.06.2014 TF No. 999 KP0)
38. In ABC, B = 90°, C = 45° and D is the mid-
43. Let in ABD, ADB = 20° and C is the point on
point of AC. If AC = 4 2 unit, then BD is— BD such that AB = AC and CD = CA, then what
ABC esa] B = 90°, C = 45° vkSj D, AC dk eè; will be BAC?

fcUnq gSA ;fn AC = 4 2 unit] rks BD gS eku yhft, ABD esa ADB = 20° vkSj C,BD ij ,d ,slk
fcUnq gS] ftlesa AB = AC rFkk CD = CA gS] rks ABC dk
(1) 2 2 unit (2) 4 2 unit eki fdruk gksxk \
5 (1) 40° (2) 45° (3) 60° (4) 30°
(3) unit (4) 2 unit (SSC CGL Tier-I Exam, 2012)
2
(SSC CGL Tier-I Exam, 2012, and SSC CPO SI & Assistant 44. In any ABC, If B and C are base angles which
Intelligence Officer Exam, 2012) are bisected by BO and CO, then what will be
39. If in a triangle ABC as drawn in the figure, AB = BOC?
AC and ACD = 120°, then A is equal to fdlh f=kHkqt ABC esa] ;fn B rFkk C ds vkèkkj dks.k] Øe'k%
;fn fuEufyf[kr vkd`fr esas n'kkZ, x, f=kHkqt ABC esa AB = AC BO rFkk CO }kjk f}Hkkftr dj fn, tk,¡] rks ‘ BOC fdruk
vkSj ACD = 120°, rks A fdlds cjkcj gS \ gksxk\
A
(1)
A
(2)
A
(3)
b A g (4) A
2 2 2 2 2
(SSC CGL Tier-I Exam, 2012)
45. G is the centrold of ABC. If AG = BG then
120°
BGC is—
B C D
ABC dk dsanzd G gSA ;fn AG = BG, rks BGC gSµ
(1) 50° (2) 60° (3) 70° (4) 80° (1) 90° (2) 30° (3) 60° (4) 120°
(SSC CGL Tier-I Exam. 19.10.2014 (Ist Sitting) [SSC CPO SI & Assistant Intelligence Officer Exam, 2012]

BLAM–276
T;kfefr
46. Let O be the in-centre of a triangle ABC and D be 52. The internal bisectors of the angles B and C of a
a point on the side BC of 'ABC, such that OD A
A
BC. If ‘BOD = 15°, then ‘ABC = triangle ABC meet at I. If BIC = + x, then x
2
ekuk fd O f=kHkqt ABC dk var% osQUnz gS vkSj D, ABC dh is equal to ?
Hkqtk BC ij ,d fcanq gS ftlls OD BCA ;fn BOD =
,d f=kHkqt ABC ds dks.k B vkSj C ds vkUrfjd f}Hkktd I ij
15° rks] ABC =
(1) 75° (2) 45° (3) 150° (4) 90° A
feyrs gSaA ;fn BIC = 2 + x gS] rks x cjkcj gS \
(SSC CHSL DEO & LDC Exam. 11.12.2011
(Ist Sitting (Delhi Zone)
(1) 60° (2) 30° (3) 90° (4) 45°
47. In triangle PQR, points A, B and C are taken on (SSC CHSL DEO Exam. 02.11.2014 (Ist Sitting)
PQ, PR and QR respectively such that QC=AC and 53. O is the orthocentre of ABC. Then BOC + BAC
CR = CB. If QPR = 40°, then ACB is equal to :
is equal to
fdlh f=kHkqt PQR esa] PQ, PR rFkk QR ij fcanq A, B rFkk C
fcUnq O, ABC dk yacdsUnz gS] rks BOC + BAC dk eku
bl izdkj gSa fd QC = AC rFkk CR = CB gSA rnuqlkj] ;fn
fdruk gksxk\
QPR = 40° gks] rks ACB fdruk gksxk\
(1) 120° (2) 135° (3) 180° (4) 90°
(1) 140° (2) 40° (3) 70° (4) 100° (SSC CGL Tier-II Exam, 2014 12.04.2015
(SSC CHSL DEO & LDC Exam. 21.10.2012, IInd Sitting) (Kolkata Region) TF No. 789 TH 7)
48. D is a point on the side BC of a triangle ABC 54. In given figure. AB||CD||EF. If EA AB BEF =
such that AD BC. E is a point on AD for 61° find the value of a, b, and c?
which AE : ED = 5 : 1. If BAD = 30° and tan (
;fn nh xbZ vkÑfr esa] AB||CD||EF ;fn EA AB vkSj
ACD) = 6 tan ( DBE), then ACB =
BEF = 61° a, b, vkSj c dk eku Kkr djks\
,d f=kHkqt ABC, dh Hkqtk BC ij D ,d ,slk fcanq gS]
(1) a = 119, b = 119, c = 92
ftlls AD BC gSA mlesa AD ij E ,d ,slk fcanq gS] (2) a = 92, b = 119, c = 112
ftlls AE : ED = 5 : 1 gSA rnuqlkj] ;fn BAD = 30° (3) a = 119, b = 119, c = 29
vkSj tan ( ACD) = 6 tan ( DBE) gks] rks ACB fdruk (4) a = 29, b = 119, c = 119
gksxk\
(1) 30° (2) 45° (3) 60° (4) 15°
(SSC Graduate Level Tier-II Exam. 29.09.2013) C
49. In a ABC, A : B : C = 2 : 3 : 4. A line CD
E
drawn to AB, then the ACD is : A
ABC esa A: B: C = 2 : 3 :4 gSA mlesa ,d js[kk 61°
CD AB ds gSA rnuqlkj] ACD fdruk gksxk \ D
(1) 40° (2) 60° (3) 80° (4) 20° b
(SSC Graduate Level Tier-I Exam. 21.04.2013)
50. In a triangle ABC, the side BC is extended up to
D. Such that CD = AC, if BAD = 109° and ACB
B F
= 72° then the value of ABC is a
f=kHkqt ABC esa Hkqtk BC fcUnq D rd bl izdkj c<+k;h tkrh
gS fd CD = AC, ;fn BAD = 109° vkSj ACB = 72° rks
55. In an isosceles triangle, if the vertex angle is twice
ABC dk eku D;k gksxk\ the sum of the base angles, then the measure of
(1) 35° (2) 60° (3) 40° (4) 45° the half of the vertex angle of the triangle is
(SSC Graduate Level Tier-I Exam. 21.04.2013)
,d lef}Hkqt f=kHkqt esa] ;fn 'kh"kZ dks.k vkèkkj dks.kksa ds ;ksx
2 2 2
51. In a ABC, AB + AC = BC and BC = 2 AB , ls nqxquk gS] rks f=kHkqt ds vkèks 'kh"kZ dks.k dk eki D;k gksxk \
then ABC is : (1) 60° (2) 70° (3) 80° (4) 50°
2 2 2 (SSC CGL Tier-II Exam, 2014 12.04.2015
,d ABC esa] AB + AC = BC vkSj BC = 2 AB gS] (Kolkata Region) TF No. 789 TH 7)
rks ABC fdruk gksxk\ 56. In a right-angled triangle, the product of two sides
(1) 30° (2) 45° (3) 60° (4) 90° is equal to half of the square of the third side i.e.,
(SSC CHSL DEO & LDC Exam. 21.10.2012 (IInd Sitting) hypotenuse. One of the acute angle must be

BLAM–277
T;kfefr
,d ledks.k f=kHkqt esa] nks Hkqtkvksa dk xq.kuiQy rhljh Hkqtk ABCD ,d pØh; leyac gS ftlesa AD || BC. ;fn ABC
(vFkkZr~ d.kZ) ds oxZ dk vk/k gSA rnuqlkj] mldk ,d U;wu = 70°, rks BCD dk eki gS %
dks.k fdruk gksuk pkfg,\ (1) 60° (2) 70° (3) 40° (4) 80°
(1) 60° (2) 30° (3) 45° (4) 15° (SSC CHSL DEO & LDC Exam. 11.12.2011
(IInd Sitting (Delhi Zone)
(SSC Graduate Level Tier-II Exam. 29.09.2013
64. ABCD is a cyclic trapezium whose sides AD and
57. In an isosceles triangle, if the unequal angle is
BC are parallel to each other. If ABC = 72°, then
twice the sum of the equal angles, then each equal
the measure of the BCD is
angle is
fdlh lef}ckgq f=kHkqt esa ;fn vleku dks.k leku dks.kksa ds ABCD ,d pØh; leyac prqHkqZt gS] ftldh AD rFkk BC
;ksxiQy dk nqxquk gks] rks izR;sd leku dks.k gSµ Hkqtk,a ijLij lekukarj gSaA ;fn ABC = 72° gks] rks BCD
(1) 120° (2) 60° (3) 30° (4) 90° dk ekUk D;k gksxk \
(SSC GL Tier-I Exam. 19.05.2013 Ist Sitting) (1) 162° (2) 18° (3) 108° (4) 72°
58. ABC is an equilateral triangle and CD is the in- (SSC CHSL DEO & LDC Exam. 11.12.2011
(Ist Sitting (East Zone)
ternal bisector of C. If DC is produced to E
such that AC = CE, then CAE is equal to 65. If an exterior angle of a cyclic quadrilateral be
50°, then the interior opposite angle is :
ABC ,d leckgq f=kHkqt gS vkSj CD, C dk vkarfjd f}Hkktd
gSA ;fn DC, E rd bl izdkj c<+k;k tk, fd AC = CE, rks ;fn ,d pØh; prqHkqZt dk cká dks.k 50° gks] rks mldk
CAE fdlds cjkcj gksxk \
vkarfjd lEeq[k dks.k fdruk gksxk\
(1) 130° (2) 40° (3) 50° (4) 90°
(1) 45° (2) 75° (3) 30° (4) 15°
(SSC CHSL DEO & LDC Exam. 11.12.2011
(SSC CGL Tier-I Exam. 26.10.2014) (IInd Sitting (East Zone)
59. The measure of each interior angle of a regular 66. A cyclic quadrilateral ABCD is such that AB =
polygon with 8 sides is BC, AD = DC, AC BD, CAD = T. Then the angle
,d le cgqHkqt ftldh 8 Hkqtk,¡ gks]a ds izR;sd vkarfjd dks.k ABC =
dk eki crkb,A ,d pØh; prqHkZt
q ABCD esa AB = BC, AD = DC, AC
(1) 135° (2) 120° (3) 100° (4) 45° BD, CAD = gSA rnuqlkj] mlesa ABC fdlds cjkcj
(SSC CHSL (10+2) LDC, DEO & PA/SA Exam, 20.12.2015
(Ist Sitting) TF No. 9692918)
gks xk \
60. Measure of each interior angle of a regular poly-
gon can never be : (1) T (2) (3) 2T (4) 3T
2
og eki dkSu lk gS] tks fdlh lecgqHkqt osQ izR;sd vkarfjd dks.k (SSC Graduate Level Tier-I Exam. 19.05.2013)
dk eki dHkh ugha gks ldrk\ 67. In the following figure, AB be diameter of a circle
(1) 150° (2) 105° (3) 108° (4) 144° whose centre is O. If AOE = 150°, DAO = 51°
(SSC CHSL DEO & LDC Exam. 11.12.2011
then the measure of CBE is :
(IInd Sitting (East Zone) fuEu vkd`fr esa AB, ,d O dsanz okys o`Ùk dk O;kl gSA
61. The sum of all internal angles of a regular poly- rnuqlkj] ;fn AOE = 150°, DAO = 51° gks] rks CBE
gon whose one external angle is 20° is
dk eki fdruk gksxk \
,sls lecgqHkqt ds lHkh var% dks.kksa dk ;ksxiQy D;k gksxk ftldk
,d cfg"dks.k dks.k 20° gks \ D
(1) 6400° (2) 3200° (3) 2880° (4) 1440° E
(SSC CHSL (10+2) DEO & LDC Exam. 16.11.2014,
Ist Sitting TF No. 333 LO 2) 150°
51°
62. In a quadrilateral ABCD, the bisectors of A and A
O B C
B meet at O. If C = 70° and D = 130°, then
measure of AOB is
,d prqHkZt
q ABCD esa] A vkSj B ds f}Hkktd O ij feyrs
gSaA ;fn C = 70° vkSj D = 130°, AOB dk eki D;k gS\ (1) 115° (2) 110° (3) 105° (4) 120°
(1) 40° (2) 60° (3) 80° (4) 100° (SSC CAPFs SI & CISF ASI Exam. 23.06.2013)
(SSC CGL Tier-I Exam. 19.10.2014 TF No. 022 MH 3) 68. ABCD is a cyclic quadrilateral. AB and DC are
63. ABCD is a cyclic trapezium such that AD||BC, if produced to meet at P. If ADC = 70° and DAB =
ABC = 70°, then the value of BCD is: 60°, then the PBC + PCB is

BLAM–278
T;kfefr
ABCD ,d pØh; prqHkqt
Z gSA AB vkSj DC dks P ij feykus ds
A
fy, c<+k;k tkrk gSA ;fn ADC = 70° vkSj DAB = 60°
74.
rks PBC + PCB fdruk gksxk\
(1) 130° (2) 150° (3) 155° (4) 180° O
(SSC Graduate Level Tier-I Exam. 21.04.2013)
69. In a cyclic quadrilateral A+ C= B+ D=? B C
,d pØh; prqHkqZt esa A+ C= B+ D=?
BC is a chord of the circle having centre O. A is
D C the point on big arc BC in it, as shown above.
.
O What will be equal to ‘BAC + ‘OBC ?
A B O dsanz okys ,d o`Ùk dh ,d thok BC gSA mlesa cM+h pki BC
ij A ,d ,slk fcanq gS] tks mQij fp=k esa fn[kk;k x;k gSA rnuqlkj
(1) 270° (2) 360° (3) 90° (4) 180° ‘BAC + ‘OBC fdlds cjkcj gksxk\
(SSC CGL Tier-I Exam. 26.10.2014)
(1) 120° (2) 60° (3) 90° (4) 180°
70. If ABCD be a cyclic quadrilateral in which (SSC CGL Tier-I Exam, 2012)
A=4x°, B = 7x°, C = 5y°, D = y°, then x : y is 75. Two circles cut each other at P and Q. PA and PB
;fn ABCD ,d pØh; prqHkqZt gks ftlesa A=4x°, B = are two diameters. Then ‘AQB ?
7x°, C = 5y°, D = y° rks x : y fdruk gksxk \ nks o`Ùk ,d&nwljs dks P vkSj Q ij izfrPNsn djrs gSaA PA rFkk
(1) 3 : 4 (2) 4 : 3 (3) 5 : 4 (4) 4 : 5 PB nks O;kl gSaA rks ‘AQB gSµ
(SSC CGL Tier-II Exam. 21.09.2014) (1) 120° (2) 135° (3) 160° (4) 180°
(SSC CGL Tier-I Exam, 2012)
71. The point of intersection of the diagonals AC and
BD of the cyclic quadrilateral ABCD is P. If APB 76. A circle whose centre is O passes through points
= 64° and CBD = 28°, the measure of ADB is A, B and C and ‘BAO = 30°, ‘BCO = 40° and
‘AOC = x°, then what is the value of x ?
pØh; prqHkqt
Z ABCD ds fod.kZ AC vkSj BD dk izfrPNsn fcanq A, B rFkk C ls xqtjus okys ,d o`Ùk dk dsanz O gS vkSj ‘BAO
P gSA ;fn APB = 64° vkSj CBD = 28° gS rks ADB dk
= 30°, ‘BCO = 40° vkSj ‘AOC = x° gS rks x dk eku D;k
eki D;k gS \
gS\
(1) 32° (2) 36° (3) 56° (4) 28°
(1) 220° (2) 140° (3) 210° (4) 280°
(SSC CGL Tier-II Exam, 2014 12.04.2015
(SSC CGL Tier-I Exam, 2012)
(Kolkata Region) TF No. 789 TH 7)
77. The length of a chord of a circle is equal to the
72. ABC is a cyclic triangle and the bisectors of BAC,
radius of the circle. The angle which this chord
ABC and BCA meet the circle at P, Q, and R
subtends in the major segment of the circle is
respectively. Then the angle RQP is
equal to
ABC ,d pØh; f=kHkqt gS vkSj BAC, ABC vkSj BCA ,d o`Ùk dh thok dh yackbZ o`Ùk dh f=kT;k ds cjkcj gSA bl thok
ds f}Hkktd o`Ùk ij Øe'k% P, Q, vkSj R fcUnq ij feyrs gSaA rks }kjk o`Ùk ds nh?kZ [kaM esa cuk;k tkus okyk dks.k gS%
RQP cjkcj gS & (1) 30° (2) 45° (3) 60° (4) 90°
(SSC CHSL DEO & LDC Exam. 04.12.2011 (Ist Sitting
B B (North Zone) & (SSC GL Exam. 11.11.2012 (Ist Sitting)
(1) 90 – (2) 90
2 2 78. O is the centre and arc ABC subtends an angle of
130° at O. AB is extended to P. Then PBC is
C A
(3) 90 (4) 90 – O ,d o`Ùk dk dsanz gS vkSj pki ABC, O ij 130° dk dks.k
2 2
cukrh gSA rnuqlkj ;fn AB dk P rd foLrkj dj fn;k tk,] rks
(SSC CGL Tier-I Exam, 09.08.2015
(IInd Sitting) TF No. 4239378) PBC fdruk gksxk\
73. ABCD is a cyclic quadrilateral. Diagonals AC and (1) 75° (2) 70° (3) 65° (4) 80°
BD meets at P. If APB = 110° and CBD = 30°, (SSC Delhi Police Sub-Inspector (SI) Exam. 19.08.2012)

then ADB measures 79. Two chords AB and CD of a circle with centre O
intersect each other at the point P. If AOD = 20°
ABCD ,d pØh; prqHkqt
Z gSA fod.kZ AC vkSj BD fcUnq P ij and BOC = 30°, then BPC is equal to:
feyrs gSaA ;fn APB = 110° vkSj CBD = 30° gS rks O dsanz okys ,d o`Ùk dh nks f=kT;k,¡ AB rFkk CD ,d&nwljs dks
ADB dk eki gS P fcanq ij dkVrh gSaA rnuqlkj] ;fn AOD = 20° vkSj BOC
(1) 55° (2) 30° (3) 70° (4) 80°
= 30° gks] rks BPC fdruk gksxk\
(SSC CGL Tier-I Exam, 16.08.2015
(IInd Sitting) TF No. 2176783) (1) 50° (2) 20° (3) 25° (4) 30°
(SSC CHSL DEO & LDC Exam. 21.10.2012 (IInd Sitting)

BLAM–279
T;kfefr
80. AB is the chord of a circle with centre O and DOC 86. AB is a chord to a circle and PAT is the tangent to
is a line segment originating from a point D on the circle at A. If ‘BAT = 75° and ‘BAC = 45°, C
the circle and intersecting AB produced at C such being a point on the circle, then ‘ABC is equal
that BC = OD. If BCD = 20°, then AOD = ? to
AB, O dsUnz okys o`Ùk dh thok gS vkSj DOC o`Ùk ij D fcanq AB ,d o`Ùk dh thok gS vkSj PAT o`Ùk dh A ij Li'kZ js[kk gSA
ls 'kq# gksus okyk vkSj AB dks mlds c<+k, x, Hkkx ij C fcUnq ;fn BAT = 75° vkSj BAC = 45° ,oa C o`Ùk ij ,d fcanq
ij dkVus okyk ,d js[kk [kaM gS fd BC = ODA ;fn BCD gS] rks ABC gSµ
= 20°, rks AOD = fdruk gksxk\
(1) 40° (2) 45° (3) 60° (4) 70°
(1) 20° (2) 30° (3) 40° (4) 60°
(SSC CHSL DEO & LDC Exam. 11.12.2011
(SSC Graduate Level Tier-I Exam. 21.04.2013) (Ist Sitting (Delhi Zone)
81. A, B and C are the three points on a circle such 87. The tangents are drawn at the extremities of
that the angles subtended by the chords AB and diameter AB of a circle with centre P. If a tangent
AC at the centre O are 90° and 70° respectively.
to the circle at the point C intersects the other
BAC is equal to
two tangents at Q and R, then the measure of the
A, B rFkk C ,d o`Ùk ij rhu ,sls fcanq gSa fd AB rFkk AC ‘QPR is
thokvksa }kjk dsanz O ij cus dks.k Øe'k% 90° rFkk 70° gSaA P osQUnz okys ,d o`Ùk osQ O;kl AB osQ Nksjksa ij Li'kZ js[kk,¡
rnuqlkj] BAC fdlds cjkcj gS\ [khaph xbZ gSaA ;fn fcanq C ij o`Ùk dh ,d Li'kZ js[kk vU; nks
(1) 70° (2) 80° (3) 90° (4) 100°
(SSC Graduate Level Tier-I Exam. 19.05.2013)
Li'kZ js[kkvksa dks Q rFkk R ij izfrPNsn djsa rks QPR dk eki
82. A, B, C, D are four points on a circle. AC and BD gSµ
intersect at a point E such that BEC = 130o and (1) 45° (2) 60° (3) 90° (4) 180°
ECD = 20°. BAC is
(SSC CHSL DEO & LDC Exam. 11.12.2011
,d o`Ùk ij pkj fcUnq A, B, C, D gSaA AC vkSj BD ,d fcUnq (Ist Sitting (Delhi Zone)

E ij izfrPNsn djrs gSa vkSj BEC = 130o rFkk ECD = 88. The tangents at two points A and B on the circle
20° BAC gSµ with centre O intersects at P ; If in quadrilateral
(1) 120° (2) 90° (3) 100° (4) 110° PAOB, ‘AOB: ‘APB = 5 : 1, then measure of ‘APB
(SSC GL Tier-I Exam. 19.05.2013 Ist Sitting) is:
83. Each of the circles of equal radii with centres A O dsanoz kys o`Ùk ij nks fcUnq, A rFkk B Li'kZ js[kk,¡ P ij
and B pass through the centre of one another izfrPNsn djrh gSaA ;fn prqHkqt
Z PAOB esa AOB : APB =
circle they cut at C and D then DBC is equal to 5 : 1 rks APB dk eki gS %
A rFkk B dsankz sa okys leku f=kT;kvksa ds nks o`Ùkksa esa ls izR;sd (1) 30° (2) 60° (3) 45° (4) 15°
,d&nwljs ds dsanz ls xqtjrk gSA ;fn os C vkSj D ij dkVsa rks (SSC CHSL DEO & LDC Exam. 11.12.2011
(IInd Sitting (Delhi Zone)
DBC dk eku gS %
89. A, B, C are three points on a circle. Tangent on A
(1) 60° (2) 100° (3) 120° (4) 140°
meets with extended BC at T. ‘BTA = 40° and
(SSC CGL Tier-I Re-Exam. (2013) 27.04.2014)
‘CAT = 44°. What is the angle made by BC on
84. The chord of a circle is equal to its radius. The
angle subtended by this chord at the minor arc of centre ?
the circle is A, B, C ,d o`Ùk ij rhu fcUnq gSaA A ij Li'kZ js[kk c<+h gqbZ
,d o`Ùk dh thok mldh f=kT;k ds cjkcj gSA o`Ùk ds y?kq pki BC ls T ij feyrh gSA ‘BTA = 40° ,oa ‘CAT = 44°
ij bl thok }kjk varfjr dks.k fdruk gksxk \ gSA BC }kjk o`Ùk ds dsanz ij cuk;k x;k dks.k gSµ
(1) 75° (2) 60° (3) 150° (4) 120° (1) 84° (2) 92° (3) 96° (4) 104°
(SSC CAPFs SI, CISF ASI & DP SI (SSC Graduate Level Tier-II Exam.04.09.2011)
Exam, 21.06.2015 IInd Sitting)
90. From a point P, two tangents PA and PB are drawn
85. Two tangents are drawn from a point P to a circle
to a circle with centre O. If OP is equal to diameter
at the points A and B. If AOP = 600 O is centre,
of the circle, then APB is
then APB = ?
,d o`Ùk ds fcUnq A rFkk B ij ,d fcUnq P ls nks Li'kZ js[kk,¡ ,d fcUnq P ls] O dsaæ okys ,d o`Ùk ij nks Li'kZ js[kk,¡ PA vkSj
[khaph xbZ gSaA O ml o`Ùk dk dsUnz gSA rnuqlkj] ;fn AOP = PB [khaph xbZ gSaA ;fn OP o`Ùk ds O;kl ds cjkcj gS] rks APB
600 gks] rks APB fdruk gksxk \ gSµ
(1) 1200 (2) 900 (3) 600 (4) 300 (1) 45° (2) 90° (3) 30° (4) 60°
(SSC CGL Tier-I Exam, 2012) (SSC CHSL DEO & LDC Exam. 20.10.2013)

BLAM–280
T;kfefr
91. If PA and PB are two tangents to a circle with
centre O such that AOB = 110°, then APB is P
;fn PA vkSj PB ,d o`Ùk ftldk dsUnz O gS dh nks Li'kZ js[kk,¡
bl izdkj gSa ftlesa dsanz O gS] ftlls AOB = 110° gS] rks
APB gSµ Y
(1) 90° (2) 70° (3) 60° (4) 55° 105°
(SSC CHSL DEO Exam. 02.11.2014 (Ist Sitting) X
92. In triangle PQR, the internal bisector of Q and
R meets at O. If QPR = 70°, then what is the 75° 60°
value (in degrees) of QOR? Q R
f=kHkqt PQR esa] Q rFkk R dk vkarfjd f}Hkktd O ij
(1) 28 (2) 18 (3) 21 (4) 24
feyrs gSaA ;fn QPR = 70° gS] rks QOR dk eku (fMxzh esa)
(SSC CGL Tier-II (CBE) Exam. 18.02.2018)
D;k gS\ 96. In the given figure, PQ = PS = SR and QPS = 40°,
(1) 45 (2) 125 (3) 115 (4) 110 then what is the value of QPR (in degrees)?
(SSC CGL Tier-II (CBE) Exam. 09.03.2018)
nh xbZ vkÑfr eas, PQ = PS = SR rFkk QPS = 40° gks] rks
93. PQRS is a cyclic quadrilateral. PR and QS inter-
QPR dk eku (fMxzh eas) D;k gS\
sect at T. If SPR = 40° and PQS = 80°, then
what is the value (in degrees) of PSR?
P
PQRS ,d pØh; prqHkqt
Z gSA PR rFkk QS, T ij izfrPNsn
djrs gSaA ;fn SPR = 40° rFkk PQS = 80° gSa] PSR dk
eku (fMxzh esa) D;k gS\
(1) 60 (2) 40 (3) 80 (4) 100
(SSC CGL Tier-II (CBE) Exam. 09.03.2018)
94. In the given figure, PSR = 105° and PQ is the
diameter of the circle. What is the value (in de- Q S R
grees) of QPR?
(1) 45 (2) 60
nh xbZ vkÑfr esa, PSR = 105° rFkk PQ o`Ùk dk O;kl gSA
(3) 75 (4) 50
QPR dk eku (fMxzh eas) D;k gS\
(SSC SGL Tier-II (CBE) Exam. 19.02.2018)
97. In the given figure, if AD = 12 cm., AE = 8 cm and
Q EC = 14 cm, then what is the value (in cm.) of
BD?

R nh xbZ vkÑfr esa] ;fn AD = 12 lseh-] AE = 8 lseh- rFkk EC


= 14 lseh- gS] rks BD dk eku (lseh- esa) D;k gS\

A
S
70° E
P
D
(1) 75 (2) 15 (3) 30 (4) 45
70°
(SSC CGL Tier-II (CBE) Exam. 09.03.2018)
B C
QR 14
95. In the given figure, if = and PY = 18 cm,
XY 9
50
then what is the value (in cm.) of PQ? (1) (2) 15
3
QR 14
nh xbZ vkÑfr esa] ;fn XY = 9 rFkk PY = 18 lseh gks] rks 8 44
(3) (4)
3 3
PQ dk eku (lseh- esa) D;k gS\ (SSC SGL Tier-II (CBE) Exam. 19.02.2018)

BLAM–281
T;kfefr
98. In the given figure, O is the centre of the circle 101. Three lines x, y, z are Parallel and represent an-
and QOR = 50°, then what is the value of RPQ gles in the figure AFB = ?
(in degrees)? rhu js[kk,¡ x, y, z lekukUrj gS vkSj vkÑfr esa dks.k n'kkZ;s x,
nh xbZ vkÑfr eas O ,d o`Ùk dk dsUnz gS rFkk QOR = 50° gSA AFB = ?
gS] rks RPQ dk eku (fMxzh eas ) D;k gS\
D E
x
R 125° 80°
C

30°
y
Q A B

O
P z
F

(1) 20° (2) 15° (3) 30° (4) 10°


(1) 15 (2) 25 102. In given figure AB||GH||DE and GF||BD||HI,
FGC = 80° then the value of CHI = ?
(3) 20 (4) 30
(SSC SGL Tier-II (CBE) Exam. 19.02.2018) nh xbZ vkÑfr esa AB||GH||DE vkSj GF||BD||HI,
99. Two straight line in the given figure PQ and RS FGC = 80° rc CHI dk eku D;k gksxk\
meet each other at O. If SOT = 75° then find the
value of a, b and c ?
A G F
nh xbZ vkÑfr esa nks lh/h js[kk PQ vkSj RS ,d nwljs ij ‘O’
ij feyrh gSA ;fn SOT = 75° rc a, b, c dk eku Kkr djsA

R
P B C D
2c
4b a
b
I H E
75°

(1) 80° (2) 120° (3) 100° (4) 160°


103. In given figure AB||CD, PEB = 80°, QHB =
S
Q 120° and PQR = x° find the value of x is:
T
nh xbZ vkÑfr esa AB||CD, PEB = 80°, QHB = 120°
(1) a = 84°, b = 21°, C = 48° vkSj PQR = x°, x dk eku Kkr djsaA
(2) a = 48°, b = 20°, C = 50°
A C
(3) a = 72°, b = 24°, C = 54°
(4) a = 64°, b = 28°, C = 45°
100. In the given figure. PQ and RS are Parallel of
lines PQ and M. Find the middle Angle? G Q
E x°
nh xbZ vkÑfr esa PQ vkSj RS, lekukUrj gSA js[kkvksa PQ vkSj P
80° H 120°
M osQ chp dks.k Kkr djksA
P F
55°
R
L
M B D
155°
30°
(1) 40° (2) 20°
R 25°
S (3) 100° (4) 30°
(1) 175° (2) 177° (3) 179° (4) 180°

BLAM–282
T;kfefr
104. In the given figure AD||BC If ABC = 72° then (1) 149° (2) 74.5°
BCD = ? (3) 62° (4) None of these
;fn nh xbZ vkÑfr esa AD||BC ;fn ABC = 72°, rc 108. In the given figure, quadrilateral ABCD has the
BCD = ? common divisor of AM and DM respectively bi-
D sect A and D. AMD = ?
uhps fn, x, prqHkqt
Z ABCD esa AM rFkk DM Øe'k% A rFkk
D. lef}Hkktd gSA vkÑfr esa AMD = dk eku gksxkA

A D
A C
72° M

B 50° 100°
B C
(1) 108° (2) 36°
(3) 90° (4) 72° (1) 75° (2) 70°
105. QSR = ? (3) 50° (4) 105°

109. In a triangle ABC, AB = 6 3 cm. AC = 12 cm


and BC = 6 cm. Then measure of B is equal to:
Q
;fn f=kHkqt ABC, AB = 6 3 cm. AC = 12 cm vkSj BC
S o = 6 cm BC = 6 gS] rks B dk eku fdruk gksxk\
(1) 90° (2) 45°
60° (3) 70° (4) 60°
P
R (SSC CGL Tier-II Exam. 18.11.2020)

(1) 55° (2) 90° 110. The exterior angle obtained on producing the base
(3) 70° (4) 60° of a triangle both the ways are 121° and 104°.
106. Find the value of x = ? What is the measure of the largest angle of the
x dk eku Kkr djsaA triangle?
A D fdlh f=kHkqt ds vk/kj dks vkxs c<+kus ij nksuksa vkSj cuus okys
30°
cfg"dks.k 121° vkSj 104° gSaA f=kHkqt ds lcls cM+s dks.k dk
eku D;k gS\
(1) 75° (2) 76°
O (3) 74° (4) 66°
(SSC CGL Tier-II Exam. 18.11.2020)
100° x°
C 111. ABC is an equilateral triangle with side 12 cm
B
and AD is the median. Find the length of GD if
G is the centroid of ABC.
(1) 40° (2) 45° ABC ,d leckgq f=kHkqt gS ftldh izR;sd Hkqtk 12 cm vkSj
(3) 50° (4) 60° AD ekfè;dk gS A GD dh yackbZ Kkr djsa ;fn G, ABC dk
107. In given figure, the diameter AB and Centre O of
dsna zd gksA
the circle is APB = ?
nh xbZ vkÑfr esa AB o`Ùk dk O;kl o O osQUnz gS] APB = ? (1) 6 3 cm (2) 3 3 cm

A (3) 4 3 cm (4) 2 3 cm
C (SSC CGL Tier-II Exam. 18.11.2020)
112. In a triangle ABC, D is a point on BC such that
o 31° P AB BD
. If B = 68° and C = 52°, then mea-
AC DC
D sure of BAD is equal to :
B

BLAM–283
T;kfefr
5. In ABC, DE || AC. D and E are two points on AB
AB BD
f=kHkqt ABC esa] D, js[kk BC ij ,slk fcanq gS fd A and CB respectively. If AB = 10 cm and AD = 4
AC DC
cm, then BE : CE is
;fn B = 68° vkSj C = 52° rks BAD dk eku fdruk ABC esa DE AC gSA mlesa D rFkk E Øe'k% AB rFkk CB
gksxk\ ij nks fcanq gSaA rnuqlkj] ;fn AB = 10 lseh rFkk AD = 4 lseh
(1) 50° (2) 40°
gks] rks BE : CE fdruk gksxk\
(3) 60° (4) 30°
(1) 2 : 3 (2) 2 : 5
(SSC CGL Tier-II Exam. 18.11.2020)
(3) 5 : 2 (4) 3 : 2
(SSC Graduate Level Tier-I Exam. 19.05.2013)
Type-II 6. The points D and E are taken on the sides AB
Questions Based on Length of sides 1 1
of a Triangle and AC of ABC such that AD = AB, AE =
3 3
AC. If the length of BC is 15 cm, then the length
1. In a ABC, AB = 3 cm, AC = 4cm and the bisector of DE is :
of A, AD meets BC at D. then BD : DC equals ABC dh Hkqtkvksa AB rFkk AC ij nks fcUnq D rFkk E bl
to—,d f=kHkqt ABC esa] AB = 3 lseh-] AC = 4 lseh- rFkk 1 1
Adk lef}Hkktd AD Hkqtk BC dks fcUnq D ij feyrk gSA rc] izdkj pqus x, gSa fd AD = AB rFkk AE = AC gSaA ;fn
3 3
BD : DC cjkcj gksxkµ BC dh yEckbZ 15 lseh gks] rnuqlkj DE dh yEckbZ fdruh gS\
(1) 9 : 16 (2) 16 : 9 (1) 10 cm (2) 8 cm (3) 6 cm (4) 5 cm
(3) 3 : 4 (4) 4 : 3 (SSC CHSL DEO & LDC Exam. 04.12.2011
(Ist Sitting (East Zone)
(SSC DEO & LDC Exam. 28.11.2010, IInd Sitting)
7. A straight line parallel to side BC in ABC, in-
2. The base of a triangle is 12 3 and the angles on tersects AB and AC at P and Q respectively AP =
base are 30° and 60° respectively, the height of QC PB =4 unit and AQ = 9 unit, then Length of
the triangle will be— AP will be ?
ABC esa BC osQ lekarj ,d js[kk AB vkSj AC dks Øe'k% P
,d f=kHkqt dk vk/kj 12 3 gS vkSj vk/kj ij nks dks.k Øe'k%
vkSj Q ij dkVrh gSA ;fn AP = QC, PB = 4 bdkbZ rFkk AQ
30° vkSj 60° gSa] rks f=kHkqt dk 'kh"kZ yac fdruk gS \
= 9 bdkbZ rks AP dh yEckbZ D;k gS\
(1) 12 cm. (2) 6 cm. (1) 2.5 unit (2) 3 unit
(3) 10 3 cm. (4) 9 cm. (3) 6unit (4) 6.5 unit
(SSC CHSL DEO & LDC Exam. 04.12.2011
(SSC CGL Tier-II Exam, 12-04-2015 (Kolkata Zone)
(Ist Sitting (East Zone)
3. Inside a triangle ABC, a straight line parallel to 8. In ABC, PQ is parallel to BC. If AP : PB = 1 : 2,
BC intersects AB and AC at the point P and Q and AQ = 3cm, then AC will be—
respectively. If AB = 3 PB, then PQ : BC is
ABC esa PQ, BC ds lekukarj gSA rnuqlkj] ;fn AP : PB =
,d f=kHkqt ABC esa BC ds lekarj ,d ljy js[kk AB rFkk AC 1 : 2 vkSj AQ = 3 lseh- gks]rks AC fdlds cjkcj gksxk\
dks Øe'k% P rFkk Q fcanqvksa ij dkVrh gSA rnuqlkj ;fn AB = (1) 6 cm (2) 9 cm (3) 12 cm (4) 8 cm
3 PB gks] rks PQ : BC fdruk gksxk\ (SSC CHSL DEO & LDC Exam. 11.12.2011
(Ist Sitting (East Zone)
(1) 1 : 3 (2) 3 : 4
9. In 'ABC, D and E are points on AB and AC re-
(3) 1 : 2 (4) 2 : 3 spectively such that DE || BC and DE divides
(SSC FCI Assistant Grade-III Main Exam. 07.04.2013) the 'ABC into two parts of equal areas. Then
4. The sum of three altitudes of a triangle is ratio of AD and BD is
fdlh f=kHkqt dh rhuksa mQ¡pkb;ksa dk ;ksx fdruk gksrk gS \ ABC esa] D rFkk E Øe'k% AB rFkk AC ij nks ,sls fcUnq
(1) equal to the sum of three sides gSa] fd DE||BC gS] vkSj DE, f=kHkqt ABC dks nks cjkcj
rhuksa Hkqtkvksa dk ;ksx {ks=kiQy okys Hkkxksa esa foHkkftr dj nsrh gSA rnuqlkj AD rFkk
(2) less than the sum of sides / Hkqtkvksa ds ;ksx ls de BD dk vuqikr fdruk gS\
(3) greater than the sum of sides (1) 1 : 1 (2) 1 : 2 1
Hkqtkvksa ds ;ksx ls vf/d
(3) 1 : 2 (4) 1 : 2 1
(4) twice the sum of sides / Hkqtkvksa ds ;ksx dk nksxquk
(SSC Graduate Level Tier-I Exam. 19.05.2013) (SSC Graduate Level Tier-II Exam.16.09.2012)

BLAM–284
T;kfefr
10. AD is the median of a triangle ABC and O is the 15. If the length of every median of an equilateral
centroid such that AO = 10 cm. The length of OD triangle is 6 3 , then what will the perimeter ?
(in cm) is
AD ,d f=kHkqt ABC dh ekfè;dk gS] O mldk dsUnzd gS vkSj ;fn ,d leckgq f=kHkqt dh izR;sd ekfè;dk dh yackbZ 6 3 lseh
AO = 10 lseh, rks OD dh yackbZ lseh esa gSµ gks] rks ml f=kHkqt dk ifjeki fdruk gksxk \
(1) 24 cm (2) 32 cm (3) 36 cm (4) 42 cm
(1) 4 (2) 5 (3) 6 (4) 8
(SSC FCI Assistant Grade-III Exam. 11.11.2012 First Sitting)
FCI Assistant Grade-III Exam. 25.02.2012 (Paper-I)
16. If the length of the three sides of a triangle are 6
North Zone (Ist Sitting)
cm, 8 cm and 10 cm, then the length of the me-
11. D is any point on side AC of 'ABC. If P, Q, X, Y dian to its greatest side is ?
are the mid-points of AB, BC, AD and DC respec-
;fn fdlh f=kHkqt dh rhu Hkqtkvksa dh yEckb;k¡ 6 lseh-] 8 lseh-
tively, then the ratio of PX and QY is
rFkk 10 lseh- gks]a rks f=kHkqt dh lcls cM+h Hkqtk ij [khaph xbZ
ABC dh Hkqtk AC ij dksbZ fcUnq D gSA P, Q, X, Y Øe'k% ekfè;dk dh yEckbZ gksxh \
AB, BC, AD rFkk DC osQ eè; fcanq gSa rks PX vkSj QY dk (1) 8 cm (2) 6 cm (3) 5 cm (4) 4.8 cm
vuqikr gSµ (SSC Data Entry Operator Exam. 31.08.2008)
17. In a triangle ABC, BAC = 90° and AD is
(1) 1 : 2 (2) 1 : 1 (3) 2 : 1 (4) 2 : 3
perpendicular to BC. If AD = 6 cm and BD = 4 cm,
(SSC CHSL DEO & LDC Exam. 11.12.2011
then the length of BC is
(Ist Sitting (Delhi Zone)
12. In ' ABC, P and Q are mid-point of sides AB and ,d f=kHkqt ABC esa] BAC = 90° vkSj AD BC ds yacor gSA
AC respectively. R is a point on PQ and PR : RQ = ;fn AD = 6 lseh- vkSj BD = 4 lseh] rks BC dh yackbZ gSµ
1 : 2. If PR = 2 cm, then BC = ? (1) 8 cm (2) 10 cm (3) 9 cm (4) 13 cm
(SSC CHSL DEO & LDC Exam. 04.11.2012 (IInd Sitting)
' ABC esa P vkSj Q Øe'k% Hkqtkvksa AB rFkk AC ds eè;
18. In a right angled ABC, ABC = 90°; BN is
fcUnq gSaA R ,d fcUnq gS [k.M PQ ij vkSj PR : RQ = 1 : 2 perpendicular to AC, AB = 6 cm, AC = 10 cm.
;fn PR = 2 lseh, rks BC = ? Then AN : NC is
(1) 4 cm (2) 2 cm ,d ledks.k f=kHkqt ABC esa ABC = 90° gS vkSj BN,
(3) 12 cm (4) 6 cm AC ij yEc gSA mlesa AB = 6 lseh-] AC = 10 lseh gS] rks
(SSC Graduate Level Tier-II Exam.04.09.2011) AN : NC fdruk gksxk \
13. D and E are two points on the sides AC and BC (1) 3 : 4 (2) 9 :16 (3) 3 : 16 (4) 1 : 4
respectively of ABC such that DE =18 cm, CE = 5 (SSC Graduate Level Tier-I Exam. 11.11.2012 (Ist Sitting)

cm and DEC = 90°. If tan ABC = 3.6, then AC 19. BL and CM are medians of ABC right-angled at
: CD =
3 5
ABC dh Hkqtkvksa AC rFkk BC ij D rFkk E nks ,sls fcanq gSa] A and BC = 5 cm. If BL = cm, then the
2
ftuesa DE = 18 lseh] CE = 5 lseh vkSj DEC = 90° gSA length of CM is
rnuqlkj] ;fn tan ABC = 3.6 gks] rks AC : CD fdruk gksxk\ ABC esa] tks A ij ledks.kh; gS vkSj ftlesa BC = 5 lseh
(1) BC : 2 CE (2) 2 CE : BC
3 5
(3) 2 BC : CE (4) CE : 2 BC gS] nks ekfè;dk,¡ BL rFkk CM gSaA rnuqlkj] ;fn BL =
2
(SSC Graduate Level Tier-II Exam. 29.09.2013
14. If every side of an equilateral triangle is extended
lseh gS] rks CM dh yackbZ fdruh gksxhA
(1) 2 5 cm (2) 5 2 cm
by 2 units, area will be increased (3 + 3 ) unit2.
What is the length of the side ? (3) 10 2 cm (4) 4 5 cm
;fn ,d leckgq f=kHkqt dh izR;sd Hkqtk esa 2 ek=kd o`f¼ dj nh (SSC CHSL DEO & LDC Exam. 10.11.2013, Ist Sitting)
20. ‘ABC = 90° and on being BC parallel to DE, if
tk,] rks mlds {ks=kiQy esa 3 + 3 oxZ ek=kd dh o`f¼ gks tk,xhA A B
rnuqlkj] ml f=kHkqt dh izR;sd Hkqtk dh yackbZ fdruh gS\ = 12, BD = 6 and BC = 10 then, what will be the
length of DE ?
(1) 3 unit (2) 3 unit ‘ABC = 90° vkSj BC,DE ds lekukarj gksus ij] ;fn AB =
12, BD = 6 rFkk BC = 10 gks] rks DE dh yackbZ fdruh
(3) 3 3 unit (4) 1 3 unit
gksxh\
(SSC CHSL DEO and LDC Exam, 28.10.2012, First Sitting)

BLAM–285
T;kfefr
25. In triangle ABC a straight line parallel to BC in-
A
tersects AB and AC at D and E respectively. If AB
= 2AD then DE : BC is

C
f=kHkqt ABC esas BC osQ lekukarj ,d ljy js[kk AB vkSj AC
B
dks Øe'k% D vkSj E ij dkVrh gSA ;fn AB = 2AD rks DE
E : BC fdruk gksxk \
D
(1) 2 : 3 (2) 2 : 1 (3) 1 : 2 (4) 1 : 3
(1) 16 (2) 15 (3) 12 (4) 14 (SSC CGL Tier-II Exam. 21.09.2014)
(SSC CGL Tier-I Exam, 2012) 26. ABC is a triangle in which DE || BC and AD : DB
21. In ABC, AD is the internal bisector of A, meet- = 5 : 4. Then DE : BC is
ing the side BC at D. If BD = 5 cm, BC = 7.5 cm, ABC ,d ,slk f=kHkqt gS ftlesa DE || BC vkSj AD : DB =
then AB : AC is
5:4 gSa] rks DE : BC D;k gS\
'ABC esa] AD, ‘A dk vkarfjd f}Hkktd gS vkSj Hkqtk BC dks
(1) 4 : 5 (2) 4 : 9 (3) 9 : 5 (4) 5 : 9
D esa feyrk gSA ;fn BD = 5 cm, BC = 7.5 cm, rks AB : AC gSµ (SSC CGL Tier-II Exam. 12.04.2015 TF No. 567 TL 9)
(1) 2 : 1 (2) 1 : 2 (3) 4 : 5 (4) 3 : 5 27. In ABC, BAC = 90° and AD BC. If BD = 3 cm
(SSC CHSL DEO & LDC Exam. 04.12.2011 and CD = 4 cm, then the length of AD is
(Ist Sitting (North Zone)
ABC esa] BAC = 90° vkSj AD BC gSA ;fn BD = 3
22. In ABC A = B = 60°, AC = 13 cm. The lines AD
lseh- vkSj CD = 4 lseh- gS] rks AD dh yackbZ gS
and BD intersect at D with D = 90°. If DB = 2 cm, (1) 3.5 cm (2) 5 cm
then the length of AD is
(3) 2 3 cm (4) 6 cm
ABC es]a A = B = 60° ,oa AC = 13 lsehA AD vkSj BD
(SSC CGL Tier-II Exam, 25.10.2015, TF No. 1099685)
js[kk,¡ fcUnq D ij bl izdkj dkVrh gSa fd D = 90° ;fn DB 28. AD is perpendicular to the internal bisector of
= 2 lseh] rks AD dh yEckbZ fdruh gksxh \ ABC of ABC. DE is drawn through D and par-
allel to BC to meet AC at E (mid point). If the
(1) 3 cm (2) 3.5 cm
length of AC is 12 cm, then the length of AE (in
(3) 4 cm (4) 4.7 cm cm.) is
(SSC CHSL DEO Exam. 16.11.2014 (Ist Sitting)
AD, ABC ds ABC ds vkarfjd f}Hkktd dk yac gSA DE
23. In ABC, D is the mid-point of BC. Length AD is
dks D ls gksdj vkSj BC ds lekukarj cuk;k tkrk gS ftlls AC,
27 cm. N is a point in AD such that the length of
DN is 12 cm. The distance of N from the centroid E ij fey lds (eè; fcanq)A ;fn AC dh yackbZ 12 lseh- gS]
of ABC is equal to rks AE dh yackbZ (lseh- esa) fdruh gksxh\
ABC esa] D, BC dk eè;&fcUnq gSA AD dh yEckbZ 27 lseh (1) 3 (2) 8 (3) 4 (4) 6
(SSC CGL Tier-II Exam, 25.10.2015, TF No. 1099685)
gSA N, AD ij ,slk fcUnq gS fd DN dh yEckbZ 12 lseh gSA
29. In ABC, E and D are points on sides AB and
ABC ds dsUnzd ls N dh nwjh fdruh gksxh \
AC respectively such that ABC = ADE. If AE =
(1) 3 cm (2) 6 cm 3 cm, AD = 2 cm and EB = 2 cm, then length of
(3) 9 cm (4) 15 cm DC is
(SSC CHSL DEO Exam. 16.11.2014 (Ist Sitting)
ABC esa E vkSj D Øe'k% AB vkSj AC Hkqtkvksa ij bl izdkj
24. For a triangle ABC,D and E are two points on AB
fcanq gSa fd ABC = ADE gSA ;fn AE = 3 lseh, AD =
1 1 2 lseh vkSj EB = 2 lseh, rks DC dh yackbZ fdruh gS\
and AC such that AD = AB, AE = AC. If BC
4 4 (1) 4 cm (2) 4.5 cm (3) 5.0 cm (4) 5.5 cm
= 12 cm, then DE is (SSC CGL Tier-I Exam. 19.10.2014 TF No. 022 MH 3)
30. If in a triangle ABC, BE and CF are two medians
f=kHkqt ABC ds fy, AB vkSj AC ij D vkSj E fcUnq bl izdkj perpendicular to each other and if AB = 19cm and
1 1 AC = 22 cm then the length of BC is
gSa fd AD = 4 AB, AE = 4 AC gSA ;fn BC = 12 lseh rks
;fn f=kHkqt ABC esa ] BE vkSj CF ,d nwljs ds yacor~ nks
DE fdruk gksxk\ ekfè;dk js[kk,a gSa vkSj ;fn AB = 19 lseh- rFkk AC = 22 lseh-
(1) 5 cm (2) 4 cm rks BC dh yackbZ fdruh gS\
(3) 3 cm (4) 6 cm (1) 19.5 cm (2) 26 cm (3) 20.5 cm (4) 13 cm
(SSC CAPFs SI, CISF ASI & Delhi Police SI
(SSC CGL Tier-I Re-Exam. (2013) 27.04.2014)
Exam, 21.06.2015 (Ist Sitting) TF No. 8037731)

BLAM–286
T;kfefr
31. O is the centroid in ' ABC and AD, BE, CF are ,d lef}ckgq ledks.k f=kHkqt dk ifjeki 2p lseh gSA rnuqlkj
three medians and area of ' AOE = 15 cm2 then mldk {ks=kiQy fdruk gS\
area of BDOF is—
' ABC esa O dsand
z gS vkSj AD, BE, CF rhu ekfè;dk,¡ gSa vkSj e
(1) 3  2 2 p cm2 j e
(2) 3 – 2 2 p2 cm2 j
' AOE dk {ks=kiQy =15 lsehñ2 rks prqHkqt
Z BDOF dk {ks=kiQy gS
(1) 20 cm2 (2) 30 cm2 e
(3) 2 – 2 p cm2j e
(4) 2  2 p2 cm2 j
(3) 40 cm2 (4) 25 cm2 (SSC FCI Assistant Grade-III Main Exam. 07.04.2013)
(SSC CHSL DEO and LDC Exam, 2011)
37. Suppose 'ABC be a right-angled triangle where
32. A straight line parallel to side BC in ABC inter- ‘A = 90° and AD ABC. If ' ABC = 40 cm2, 'ACD
sects the sides AB and AC at D and E respective-
= 10 cm2 and AC = 9 cm, then the length of BC is
ly. If area of ABE is 36 cm2 then area of ACD
will be ? eku fyft, ABC ,d ledks.k f=kHkqt gS] ftldk A = 90°
f=kHkqt ABC ds vk/kj BC ds lekukarj ,d ljy js[kk ml vkSj AD gSA rnuqlkj] ;fn ABC = 40 lseh2]
BC
2
f=kHkqt dh AB rFkk AC Hkqtkvksa dks Øe'k% D rFkk E fcUnqvksa ACD = 10 lseh rFkk AC = 9 lseh gks] rks BC dh yackbZ
ij dkVrh gSA rnuqlkj ;fn ADE dk {ks=kiQy 36 oxZ lsaeh gks fdruh gksxh \
rks ACD dk {ks=kiQy fdruk gksxk \ (1) 12 cm (2) 18 cm
2 2 (3) 4 cm (4) 6 cm
(1) 18 cm (2) 36 cm
(SSC Graduate Level Tier-II Exam.16.09.2012)
(3) 18 cm (4) 36 cm 38. ABC is an equilateral triangle. P and Q are the
(SSC FCI Assistant Grade Exam, 2012 and SSC (10+2) DEO
& LDC Exam, 04-12-2011 North Zone : IInd Sitting) points on AB and AC respectively such that

33. The area of an equilateral triangle is 9 3 m2. Then PQ || BC . If PQ = 5 cm then what is the area of
what will be the length of its median ? APQ ?
ABC ,d leckgq f=kHkqt gSA mlesa AB rFkk AC ij P rFkk
,d leckgq f=kHkqt dk {ks=kiQy 9 3 oxZ ehVj gSA rnuqlkj
mldh ekfè;dk dh yackbZ fdrus ehVj gksxh\ Q nks ,sls fcanq gSa] ftuesa PQ || BC dk lacaèk gSA rnuqlkj ;fn
(1) 2 3 m (2) 3 3 m (3) 3 2 m (4) 2 2 m PQ = 5 lseh gks] rks APQ dk {ks=kiQy fdruk gksxk\
(SSC Graduate Level Tier-II Exam. 29.09.2013, IInd Sitting) 25 25
34. If the area of an equilateral triangle is a and height (1) cm2 (2) cm2
4 3
b2
is b,then what will be the value of ? 25 3
a (3) cm2 (4) 25 3 cm2
4
b2 (SSC CHSL DEO & LDC Exam. 11.12.2011
;fn ,d leckgq f=kHkqt dk {ks=kiQy a rFkk mQ¡pkbZ b gks] rks (IInd Sitting (East Zone)
a
39. Two triangles A BC and DEF are similar, in which
dk eku D;k gksxk \ AB = 10cm, DE = 8cm. The ratio of areas of ABC
and DEF is—
1 1
(1) 3 (2) (3) 3 (4) nks f=kHkqt ABC rFkk DEF le:i gSa ftuesa AB = 10 lseh-]
3 3
DE = 8 lseh-A f=kHkqt ABC vkSj DEF ds {ks=kiQy dk vuqikr
(SSC CAPF SI & CISF ASI Exam, 23.06.2013)
35. The area of an isosceles triangle is 4 unit . If third 2 gSµ
side is of length 2 unit, what is the length of equal (1) 4 : 5 (2) 25 : 16
sides ? (3) 64 : 125 (4) 4 : 7
(SSC CHSL DEO & LDC Exam. 21.10.2012 (IInd Sitting)
,d lef}ckgq f=kHkqt dk {ks=kiQy 4 oxZ ek=kd gSA ;fn mldh 40. If ABC is similar to DEF and BC = 3 cm, EF =
rhljh Hkqtk 2 ek=kd dh gks] rks izR;sd leku Hkqtk dh yackbZ fdruh 4 cm and area of ABC = 54 cm2, then what will
gksxh \ be area of DEF ?
;fn ABC, DEF ds le:i gks vkSj BC = 3 lseh] EF =
(1) 4 unit (2) 2 3 unit
4 lseh rFkk ABC dk {ks=kiQy = 54 lseh gks] rks DEF dk
(3) 17 unit (4) 3 2 unit {ks=kiQy fdruk gksxk\
(SSC CHSL DEO & LDC Exam. 28.10.2012, Ist Sitting) (1) 66 cm2 (2) 78 cm2
36. The perimeter of a right angled isosceles triangle (3) 96 cm2 (4) 54 cm2
is 2p cm. What is its area ? (SSC Graduate Level Tier-I Exam. 21.04.2013)

BLAM–287
T;kfefr
41. The area of two similar triangles ABC and DEF
are 20 cm2 and 45cm2 respectively. If AB = 5 cm, (23 21) (15 21)
(1) (2)
then what will equal to DE ? 4 4
nks le:i f=kHkqtksa ABC rFkk DEF ds {ks=kiQy Øe'k% 20 lseh2
(17 21) (23 21)
rFkk 45 lseh2 gSAa rnuqlkj] ;fn AB = 5 lseh gks] rks DE fdlds (3) (4)
5 5
cjkcj gksxh\
(SSC CHSL DEO & LDC Exam. 17.02.2018)
(1) 6.5 cm (2) 7.5 cm
45. In the given figure, in triangle STU, ST = 8 cm, TU
(3) 8.5 cm (4) 5.5 cm = 9 cm and SU = 12 cm, QU = 24 cm, SR = 32 cm
(SSC CAPF SI & CISF ASI Exam, 23-06-2013) and PT = 27 cm. What is the ratio of the area of
42. The area of two similar triangles are 360 cm2 and triangle PQU and area of triangle PTR ?
250 cm2. If the side of Ist triangle is 8cm then nh xbZ vkÑfr esa] f=kHkqt STU esa, ST = 8 lseh, TU = 9 lseh
what will be the length of corresponding side of rFkk SU = 12 lseh gSA QU = 24 lseh SR = 32 lseh rFkk PT
2nd triangle ?
= 27 lseh gSaA f=kHkqt PQU ds {ks=kiQy rFkk f=kHkqt PTR ds
nks le:i f=kHkqtksa ds {ks=kiQy 360 oxZ lseh vkSj 250 oxZ lseh {ks=kiQy esa D;k vuqikr gS\
gSaA ;fn igys f=kHkqt dh ,d Hkqtk dh yackbZ 8 lseh gS] rks nwljs
f=kHkqt dh laxr Hkqtk dh yackbZ gSµ
1 1
(1) 6 cm (2) 6 cm
5 3

2
(3) 6 cm (4) 6 cm
3
(SSC CHSL DEO & LDC Exam. 20.10.2010)
43. For a triangle ABC, D, E, F are the mid-points of
its sides. If ar ABC = 24 sq. units then ar DEF (1) 3 : 7 (2) 4 : 9
is (3) 2 : 3 (4) 5 : 2
,d f=kHkqt ABC ds fy,] D, E, F mldh Hkqtkvksa ds eè; fcanq (SSC CHSL DEO & LDC Exam. 20.02.2018)
46. In triangle XYZ, G is the centroid. If XY = 11 cm,
gSaA ;fn ABC = 24 oxZ ;wfuV rks DEF gSµ
YZ = 14 cm. and XZ = 7 cm, then what is the
(1) 4 sq. units (2) 6 sq. units value (in cm.) of GM?
(3) 8 sq. units (4) 12 sq. units f=kHkqt XYZ esa, G dsUnzd gSA ;fn XY = 11 lseh, YZ = 14
(SSC CGL Tier-I Re-Exam. (2013) 27.04.2014) lseh rFkk XZ = 7 lseh gSa] rks GM dk eku (lseh esa) D;k gS\
44. In the given figure, PQR is a triangle and quadri-
lateral ABCD is inscribed in it. QD = 2 cm., QC = X
5 cm., CR = 3 cm., BR = 4 cm., PB = 6 cm., PA = 5
cm and AD = 3 cm. What is the area (in cm2.) of
the quadrilateral ABCD ?
G
nh xbZ vkÑfr esa, PQR ,d f=kHkqt gS rFkk prqHkqt
Z ABCD mlesa
vafdr fd;k x;k gSA QD = 2 lseh-, QC = 5 lseh-, CR = 3
lseh-, BR = 4 lseh-, PB = 6 lseh-, PA = 5 lseh- rFkk AD = Y M Z
Z ABCD dk {ks=kiQy (lseh2. esa) D;k gS\
3 lseh- gSaA prqHkqt
(1) 6 (2) 4
(3) 2 (4) 3
P (SSC CHSL DEO & LDC Exam. 20.02.2018)
47. PQR is a triangle. S and T are the midpoints of
A the sides PQ and PR respectively. Which of the
B following is TRUE?
D I. Triangle PST is similar to triangle PQR.
1
II. ST = (QR) III. ST is parallel to QR.
2
Q C R

BLAM–288
T;kfefr
51. ABCD is a quardilateral P and R the midpoints of
PQR ,d f=kHkqt gSA S rFkk T Øe'k% Hkqtkvksa PQ rFkk PR ds DC and BC respectively line PR intersect the dig-
eè; fcUnq gSaA fuEufyf[kr esa ls dkSu&lk lR; gS\ onal AC at Q. Distance CQ be equal to what?
I. f=kHkqt PST, f=kHkqt PQR ds leku gSA ABCD ,d prqHkqt Z gSA P vkSj R Øe'k% DC vkSj BC osQ
1 eè;&fcUnq gSaA js[kk PR, fod.kZ AC, dks Q ij izfrPNsfnr djrh
II. ST =
2
(QR) III. ST, QR ds lekarj gSA gSA nwjh CQ fdlosQ cjkcj gksxh\
(1) Only I and II (2) Only II and III AC BD
(3) Only I and III (4) All I, II and III (1) (2)
4 3
(SSC CGL Tier-II (CBE) Exam. 09.03.2018)
48. ABC is a triangle in which ABC = 90°. BD is BD AC
perpendicular to AC. Which of the following is (3) (4)
4 3
TRUE?
I. Triangle BAD is similar to triangle CBD. 52. In the given figure, xy || AC and xy divides trigular
II. Triangle BAD is similar to triangle CAB. region ABC into two Parts equal in area. Deter-
III. Triangle CBD is similar to triangle CAB. Ax
ABC ,d f=kHkqt gS ftlesa ABC = 90° gSA BD, AC ij mine .
AB
yEc gSA fuEufyf[kr esa ls dkSu&lk lR; gS\ fn, x, vk¡dM+s esa xy || AC vkSj xy foHkktu gSA f=kdks.kh;
I. f=kHkqt BAD, f=kHkqt CBD ds leku gSA
Ax
II. f=kHkqt BAD, f=kHkqt CAB ds leku gSA {ks=k ABC {ks=k esa nks Hkkxksa osQ cjkcj gSA fuèkkZfjr djs AB ?
III. f=kHkqt CBD, f=kHkqt CAB ds leku gSA
(1) Only I (2) Only II and III A
(3) Only I and III (4) All I, II and III
(SSC CGL Tier-II (CBE) Exam. 09.03.2018) x

49. In the given figure, AQ = 4 2 cm., QC = 6 2


cm., and AB = 20 cm. If PQ is parallel to BC,
then what is the value (in cm.) of PB?

nh xbZ vkÑfr esa, AQ = 4 2 lseh-, QC = 6 2 lseh- rFkk


AB = 20 lseh. gSA ;fn PQ, BC ds lekukarj gS] rks PB dk B Y C
eku (lseh- eas) D;k gS\
2 1 2 1
(1) (2)
A 2 2

2 1 2 1
P Q (3) (4)
2 2
53. In the figure given above ABD = PQD = CDQ

B C
= If AB = x, PQ = z, and CD = y then which
2
(1) 8 (2) 12
one of the following is correct.
(3) 6 (4) 15
(SSC SGL Tier-II (CBE) Exam. 19.02.2018)
50. ABC consists of a median passing through A and Åij fn, vk¡dMs esa ABD = PQD = CDQ = 2 . vxj
E. The midpoint of AD and BE Increase then AC
meets on F AF = ? AB = x, PQ = z, vkSj CD = y gS] rks fuEu esa ls dkSu&lk lgh
ABC esa AD, A ls tkrh gqbZ ekfè;dk gS] vkSj E, AD dk gS\
eè;&fcUnq gS rFkk BE c<+kus ij AC dks F ij feyrh gSA AF A
foQlosQ cjkcj gSA
AC AC C
(1) (2) x P
5 4
Y
AC AC Z
(3) (4) B D
3 2 Q

BLAM–289
T;kfefr
58. In ABC, BD AC at D, E is a point on BC such
1 1 1 1 1
(1) x (2) x that BEA = xo. If EAC = 46° and EBD = 60°,
y z z y
then the value of x is:
1 1 1 1 1 2 ABC, esa D ij BD gSA BC ij ,d fcanq E bl izdkj
AC
(3) z y x (4) x y z gS fd BEA = gSA ;fn EAC = 46° vkSj EBD =
xo .
60° gS rks x dk eku Kkr djsaA
54. AB, EF and CD are Parallel lines. Given that EG
= 5 cm, GC = 10 cm, AB = 15 cm and DC = 18 cm (1) 76° (2) 68° (3) 78° (4) 72°
What is the value of AC? (SSC Delhi Police SI, CAPFs SI & CISF ASI
(CPO) Exam 23.11.2020)
AB, EF vkSj CD vkSj lekukarj js[kk,¡ gSA ;g eku ysus ij EG
ar( ABC) 4
= 5 lseh, GC = 10 lseh, AB = 15 lseh vkSj DC = 18 lseh 59. Let ABC ~ RPQ and . If AB = 3
ar( PQR) 9
AC = ?
cm, BC = 4 cm and AC = 5 cm, then RP (in cm) is
D equal to :

ar( ABC) 4
A eku ysa ABC ~ RPQ vkSj ar( PQR) 9 gSA ;fn AB =
E 3 lseh-, BC = 4 lseh- vkSj AC = 5 lseh-, gS rks RP (lseh- esa)
G dk eku Kkr djsaA
B C (1) 12 (2) 6 (3) 5 (4) 4.5
F
(SSC Delhi Police SI, CAPFs SI & CISF ASI
(1) 20 cm (2) 24 cm (3) 25 cm (4) 28 cm (CPO) Exam 23.11.2020)
55. PQR is right-angled at , PR = 5 cm and QR = 4
cm. If the lengths of sides of another triangle ABC Type-III
are 3 cm , 4 cm and 5 cm, then which one of the Questions Based on Polygons
following is correct?
PQR , PR = 5 cm vkSj QR = 4 cm ij ;g ledks.k gSA 1. The sum of the internal angles of a regular poly-
gon is 1440° The number of sides is
vxj ,d vU; f=kHkqt ABC osQ inksa dh yEckbZ 3 cm , 4 cm
,d le cgqHkqt ds vkarfjd dks.kksa dk ;ksx 1440° gSA Hkqtkvksa
rFkk 5 cm gS] rks buesa ls dkSu&lk fuEufyf[kr lgh gS\
dh la[;k fdruh gS \
(1) Area of PQR is double that of ABC
(1) 8 (2) 10 (3) 12 (4) 6
(2) Area of ABC is double that of PQR
(SSC CGL Tier-II Exam, 2014 12.04.2015
Q (Kolkata Region) TF No. 789 TH 7)
(3) B= 2. If each interior angle of a polygon is 144°, then
2
what will be the no. of sides of the polygon ?
(4) Both triangles are congruent
56. Two sides of a triangle are of length 3 cm and 5
;fn ,d lecgqHkqt dk izR;sd vkarfjd dks.k 144° gks] rks ml
cm. If the length of the third side is ‘x’ cm, then: cgqHkqt dh dqy Hkqtk,a fdruh gksaxh \
,d f=kHkqt dh nks Hkqtkvksa dh yackbZ 3 lseh- vkSj 8 lseh- gSA (1) 10 (2) 20 (3) 24 (4) 36
(SSC CHSL DEO and LDC Exam, 04.02.2011
;fn rhljh Hkqtk dh yackbZ ‘x’ lseh- gks] rks fuEu esa ls dkSu&lk IInd Sitting & (SSC (10+2) DEO and LDC
fodYi lgh gS \ Exam, 04.12.2011 East Zone : First Sitting)

(1) 5 < x (2) 5 < x < 11 3. Each interior angle of a regular polygon is three
times its exterior angle, then the number of sides
(3) 0 < x < 11 (4) x > 11
of the regular polygon is :
(SSC CHSL (10+2) Tier-I Exam. 17.03.2020)
57. In a ABC, DE is parallel to BC, AD = 3 cm, AE = ,d le cgqHkqt dk izR;sd vkarfjd dks.k mlds ckã dks.k dk
4 cm and AC = 10 cm, then the value of BD in frxquk gSA rnuqlkj ml le cgqHkqt dh Hkqtk,¡ fdruh gSa \
centimetres is : (1) 9 (2) 8 (3) 10 (4) 7
ABC esa, DE, BC osQ lekukarj gS] AD = 3 lseh- AE = 4 FCI Assistant Grade-III Exam. 05.02.2012 (Paper-I)

lseh- vkSj AC = 10 lseh- gS] rks lsaVhehVj esa BD dk eku Kkr East Zone (IInd Sitting)
4. The number of sides in two regular polygons are
djsaA
in the ratio 5 : 4 and the difference between each
(1) 3.5 (2) 7.5 (3) 4.5 (4) 5.5 interior angle of the polygons is 6°. Then the num-
(SSC CHSL (10+2) Tier-I Exam. 19.03.2020) ber of sides are

BLAM–290
T;kfefr
nks le cgqHkqtksa esa Hkqtkvksa dh la[;k 5 : 4 osQ vuqikr esa gS vkSj 10. If the ratio of an external angle and an internal
cgqHkqtksa osQ gj var% dks.k esa varj 6° gSA rks Hkqtkvksa dh angle of a regular polygon is 1 : 17, then the
number of sides of the regular polygon is
la[;k gSµ
;fn ,d le cgqHkqt ds cfg"dks.k vkSj var% dks.k dk vuqikr
(1) 15, 12 (2) 5, 4 (3) 10, 8 (4) 20, 16
1 : 17 gS] rks le cgqHkqt dh Hkqtkvksa dh la[;k D;k gksxh \
(SSC CHSL DEO & LDC Exam. 11.12.2011
(Ist Sitting (Delhi Zone) (1) 20 (2) 18 (3) 36 (4) 12
5. Ratio of the number of sides of two regular poly- (SSC CHSL DEO Exam. 16.11.2014 (Ist Sitting)

gons is 5 : 6 and the ratio of their each interior 11. The ratio of each interior angle to each exterior
angle is 24 : 25. Then the number of sides of these angle of a regular polygon is 3 : 1. The number of
two polygons are sides of the polygon is
nks le cgqHkqtksa esa Hkqtkvksa dh la[;k dk vuqikr 5 : 6 gS vkSj ,d le cgqHkqt ds izR;sd vkarfjd dks.k dk izR;sd cfg"dks.k ls
izR;sd vkarfjd dks.kksa dk vuqikr 24 : 25 gSA rnuqlkj mu vuqikr 3 % 1 gSA cgqHkqt dh fdruh Hkqtk,¡ gSa \
cgqHkqtksa dh Hkqtkvksa dh la[;k okLro esa fdruh gS\ (1) 9 (2) 7 (3) 6 (4) 8
(SSC CGL Tier-I Exam, 09.08.2015
(1) 20, 24 (2) 15, 18 (3) 10, 12 (4) 5, 6 (IInd Sitting) TF No. 4239378)
(SSC CHSL DEO & LDC Exam. 11.12.2011
(Ist Sitting (East Zone)
12. The interior angle of a regular polygon exceeds its
exterior angle by 108°. The number of the sides of
6. If the each interior angle of a regular polygon is
the polygon is
3
equal to times of two right angle, then num- ,d le cgqHkqt dk var%dks.k mlds cká dks.k ls 108° vf/
5
ber of sides is —
d gSA cgqHkqt dh Hkqtkvksa dh la[;k fdruh gS\
(1) 12 (2) 16 (3) 14 (4) 10
3 (SSC CGL Tier-II Exam, 25.10.2015, TF No. 1099685)
;fn fdlh le cgqHkqt dk izR;sd dks.k nks ledks.kksa ds 5
xq.kk
13. If the sum of all interior angles of a regular poly-
ds cjkcj gS] rks Hkqtkvksa dh la[;k gSµ gon is 14 right angles, then its number of sides is
(1) 3 (2) 5 (3) 6 (4) 8
(SSC Graduate Level Tier-II Exam.04.09.2012) ;fn fdlh le&cgqHkqt ds lHkh var%dks.kksa dk ;ksxiQy 14
7. The sum of all interior angles of a regular polygon ledks.k gks rks mldh Hkqtkvksa dh la[;k crkb,A
is twice the sum of all its exterior angles. The (1) 8 (2) 9 (3) 7 (4) 6
number of sides of the polygon is (SSC CHSL (10+2) LDC, DEO & PA/SA Exam, 20.12.2015
,d le cgqHkqt osQ lHkh vkarfjd dks.kksa dk ;ksx] mlosQ lHkh (Ist Sitting) TF No. 9692918)

cká dks.kksa osQ ;ksx dk nqxquk gSA rnuqlkj] ml cgqHkqt dh Hkqtk,¡ 14. In the given figure, ABCDEF is a regular hexagon
whose side is 6 cm. APF, QAB, DCR and DES are
fdruh gSa\ equilateral triangles. What is the area (in cm2.)
(1) 10 (2) 8 (3) 12 (4) 6 of the shaded region?
(SSC Graduate Level Tier-II Exam.16.09.2012)
nh xbZ vkÑfr esa] ABCDEF "kV~Hkqt gS ftldh Hkqtk 6 lseh-
8. The ratio between the number of sides of two
regular polygons is 1 : 2 and the ratio between gSA APF, QAB, DCR rFkk DES leckgq f=kHkqt gSaA Nk;kafdr
their interior angles is 2 : 3. The number of sides Hkkx dk {ks=kiQy (lseh-2 esa) D;k gS\
of these polygons is respectively
nks le cgqHkqtksa dh Hkqtkvksa dh la[;kvksa osQ chp dk vuqikr
P Q
1 % 2 gS vkSj muosQ vkarfjd dks.kksa osQ chp dk vuqikr 2 % 3
A
gSA rnuqlkj] mu nksuksa cgqHkqtksa dh Hkqtkvksa dh la[;k] Øe'k%
fdruh gS\
F B
(1) 6, 12 (2) 5, 10 (3) 4, 8 (4) 7, 14
(SSC Graduate Level Tier-II Exam.16.09.2012)
9. In a regular polygon if one of its internal angle is
greater than the external angle by 132°, then the
number of sides of the polygon is
E C
;fn ,d le cgqHkqt esa bldk ,d vkarfjd dks.k mlds cká
dks.k ls 132° cM+k gS] rks ml cgqHkqt dh Hkqtkvksa dh la[;k gSµ
D
(1) 14 (2) 12 (3) 15 (4) 16 S R
(SSC CHSL DEO Exam. 02.11.2014 (Ist Sitting)

BLAM–291
T;kfefr
19. Measure of each interior angle of a regular poly-
(1) 24 3 (2) 18 3
gon can never be:
(3) 72 3 (4) 36 3 fdjxZ cgqHkqt osQ vkarfjd dks.k dk eku dHkh Hkh ugh gks ldrk
(SSC CHSL DEO & LDC Exam. 17.02.2018) gSA
15. In the given figure, ABCD is a square of side 14 (1) 150° (2) 105° (3) 108° (4) 144°
cm. E and F are mid-points of sides AB and DC 20. Each internal angle of regular polygon is two times
respectively. EPF is a semicircle whose diameter its external angle.
is EF. LMNO is square. What is the area (in cm.2)
Then the number of sides of the polygon is:
of the shaded region?
fdlh cgqHkqt dk vkarfjd dks.k mlosQ ckÞ; dks.k ls nks xq.kk
nh xbZ vkÑfr esa] ABCD 14 lseh. Hkqtk okyk ,d oxZ gSA E
gSA cgqHkqt dh Hkqtkvksa dh la[;k Kkr djsa\
rFkk F Øe'k% AB rFkk DC Hkqtk ds eè; fcUnq gSAa EPF, ,d
(1) 8 (2) 6 (3) 5 (4) 7
v¼Zo`Ùk gS ftldk O;kl EF gSA LMNO ,d oxZ gSA Nk;kafdr 21. Ratio of the number of sides of two regular poly-
Hkkx dk {ks=kiQy (lseh-2 esa) D;k gS\ gons is 5 : 6 and the ratio of their each interior
angle is 24 : 25. Then the number of sides of these
A E B two polygon are.
E nks cgqHkqt dh Hkqtkvksa dh la[;k dk vuqikr 5 : 6 gS] rFkk muosQ
vkarfjd dks.kksa dk vuqikr 24 : 25 gS] rks bu nksuks cgqHkqtks dh
P L N Hkqtkvksa dh la[;k Kkr djsa\
(1) 10, 12 (2) 20, 24 (3) 15, 18 (4) 35, 42
O 22. In a regular Polygon, the exterior and interior
D F C angles are in the ratio 1 : 4. The number of sides
of the polygon is
(1) 108.5 (2) 94.5 (3) 70 (4) 120 fdlh cgqHkqt esa ckÞ; rFkk vUr% dks.k dk vuqikr 1 : 4 gSA
(SSC CHSL DEO & LDC Exam. 17.02.2018)
cgqHkqt esa Hkqtk,¡ Kkr djs\
16. ABCD is a trapezium in which AB is parallel to
(1) 5 (2) 10 (3) 3 (4) 8
CD and AB = 4 (CD). The diagonals of the trape-
23. The interior angles of a Polygon are in A.P. The
zium intersects at O. What is the ratio of area of
triangle DCO to the area of the triangle ABO? F2 I c

ABCD ,d leyEc gS ftlesa AB, CD ds lekukarj gS rFkk smallest angle is GH 3 JK and the common differ--
AB = 4(CD) gSA leyEc ds fod.kZ O ij izfrPNsn djrs gaSA
ence is 5°. Find the number of sides of the poly-
f=kHkqt DCO ds {ks=kiQy dk f=kHkqt ABO ds {ks=kiQy ls D;k gon.
vuqikr gS\ fdlh cgqHkqt osQ vUr% dks.k lekUrj Js<+h esa gSA lcls NksVk dks.k
(1) 1 : 4 (2) 1 : 2
(3) 1 : 8 (4) 1 : 16 F2 Ic

(SSC SGL Tier-II (CBE) Exam. 21.02.2018) GH 3 JK rFkk] lkoZ vUrj 5° gS cgqHkqt dh Hkqtkvksa dh la[;k
17. The difference between the interior and exterior
angles at a vertex of a regular polygon is 150°. Kkr djsaA
The number of sides of the polygon is (1) 2 (2) 6 (3) 7 (4) 9
fdlh cgqHkqt osQ vkUrfjd rFkk ckÞ; dks.kksa dk vUrj 150° gSA 24. Each of the five angles of a Polygon is 172° and
each of the other angles is 160° sides of a poly-
cgqHkqt dh Hkqtkvksa dh la[;k Kkr djsa\
gon. Find the number of sides of the polygon.
(1) 10 (2) 15 (3) 24 (4) 30
18. The number of sides in two regular Polygons are
,d cgqHkqt osQ ik¡p dks.kksa esa ls izR;sd 172° gS] vkSj vU; dks.k
in the ratio of 5 : 4. the difference between their esa ls izR;sd 160° gSA cgqHkqt dh Hkqtkvksa fd la[;k gSA
Interior angles of the Polygon is 6. Then the num- (1) 20 (2) 21 (3) 22 (4) 23
ber of sides are 25. If a Polygon has 104 diagona, then the polygon
nks cgqHkqtksa esa Hkqtkvksa fd la[;kvksa dk vuqikr 5 : 4 gSA ;fn will have a number of sides
cgqHkqtksa osQ vkarfjd dks.kksa dk vUrj 6° gks rks Hkqtkvksa dh la[;k fdlh cgqHkqt esa 104 fod.kZ gS] rks cgqHkqt esa Hkqtkvksa fd la[;k
Kkr djsa\ gksxh\
(1) 15, 12 (2) 5, 4 (3) 10.8 (4) 20, 16 (1) 12 (2) 14 (3) 16 (4) 18

BLAM–292
T;kfefr
6. A quadrilateral ABCD circumscribes a circle and
Type-IV
AB = 6 cm, CD = 5 cm and AD = 7 cm. The length
Questions Based on Quadrilaterals of side BC is
(Rhombus, Trapezium etc.) ,d prqHkqZt ABCD, ,d o`Ùk dks ifjxr djrk gS vkSj mlesa
AB = 6 lseh] CD = 5 lseh rFkk AD = 7 lseh gSA rnuqlkj]
1. ABCD is a trapezium where AD || BC. The diago-
Hkqtk BC dh yackbZ fdruh gksxh\
nal AC and BD intersect each other at the point
O. If AO = 3, CO = x – 3, BO = 3x – 19 and DO = x – (1) 4 cm (2) 5 cm (3) 3 cm (4) 6 cm
(SSC CHSL DEO & LDC Exam. 10.11.2013, Ist Sitting)
5, the value of x is
7. ABCD is a cyclic quadrilateral. AB and DC when
ABCD ,d leyEc gS] ;gk¡ AD || BC gSA fod.kZ AC vkSj produced meet at P, if PA = 8 cm, PB = 6 cm,
BD ,d&nwljs dks fcanq O ij ijLij dkVrs gSaA ;fn AO = 3, PC = 4 cm, then the length (in cm) of PD is
CO = x – 3, BO = 3x – 19 vkSj DO = x – 5, rks x dk eku ABCD ,d pØh; prqHkZtq gSA AB vkSj DC tc c<+kbZ tkrh gS]
D;k gS \ rks os P ij feyrh gSa] ;fn PA = 8 lseh- gS] PB = 6 lseh- gS]
(1) – 8, 9 (2) 8, – 9 (3) – 8, – 9 (4) 8, 9 PC = 4 lseh- gS] rks PD dh yackbZ fdruh gS\
(SSC CGL Tier-II Exam, 2014 12.04.2015 (1) 8 cm (2) 6 cm (3) 10 cm (4) 12 cm
(Kolkata Region) TF No. 789 TH 7) (SSC CGL Tier-II Exam, 25.10.2015, TF No. 1099685)
2. If the side PQ of a rhombus is 6cm and PQR = 8. ABCD is a trapezium in which AB = CD and
1200, then what will be length of QS ? AD||BC and AD = 5 cm and BC = 9 cm. If the
;fn ,d leprqHkqZt PQRS dh Hkqtk PQ dh yackbZ 6 lseh gks area of ABCD is 35 cm2, then CD = ?
vkSj PQR = 1200 gks] rks QS dh yackbZ fdrus lseh gksxh \ ABCD ,d ,slk leyac prqHkqtZ gS] ftlesa AB = CD rFkk
(1) 4 (2) 6 (3) 3 (4) 5 AD||BC vkSj AD = 5 lseh rFkk BC = 9 lseh gSaA rnuqlkj]
(SSC CGL Tier-I Exam, 2012) ;fn ABCD dk {ks=kiQy 35 oxZ lseh gks] rks CD dh yEckbZ
3. The diagonals of a rhombus are 12 cm and 16 cm fdruh gksxh \
respectively. Then what will be the length of the (1) 29 cm (2) 5 cm
side of rhombus ?
(3) 6 cm (4) 21 cm
,d leprqHkqZt ds fod.kZ Øe'k% 12 lseh- rFkk 16 lseh- gS rks
(SSC CGL Tier-I Exam, 2012)
le prqHkqZt ds Hkqtk yackbZ D;k gksxh\ 9. The ratio of parallel sides of a trapezium is 2 : 3
(1) 8 cm (2) 6 cm and their minimum distance is 12 cm. If the area
of that trapezium is 480 cm2, then length of long-
(3) 10 cm (4) 12 cm
est side in parallel sides is —
(SSC CGL Tier-II Exam, 29.09.2013)
4. The one angle from four angles of a rhombus is
,d leyac dh lekukarj Hkqtk,¡ 2 : 3 ds vuqikr esa gSa vkSj mudh
60°. If the every side of rhombus is 8 cm, then U;wure nwjh 12 lseh- gSA ;fn ml leyac dk {ks=kiQy 480 oxZ
what will be the length of longest diagonal ? lseh gks] rks lekarj Hkqtkvksa esa T;knk yach Hkqtk dh yackbZ fdruh
,d leprqHkqZt ds pkj dks.kksa esa ls ,d dks.k 60° dk gSA ;fn gksxh\
leprqHkqZt dh izR;sd Hkqtk dh yackbZ 8 lseh gS rks cM+s fod.kZ dh (1) 56 cm (2) 36 cm (3) 42 cm (4) 48 cm
(SSC CHSL DEO & LDC Exam. 21.10.2012, IInd Sitting)
yackbZ D;k gksxh\ 10. Length and breadth of a rectangle are 8 cm. and 6
(1) 8 3 cm (2) 8 cm cm. respectively. The rectangle is cut on its four
vertices such that the resulting figure is a regular
8 octagon. What is the side (in cm.) of the octa-
(3) 4 3 cm (4) cm gon?
3
(SSC Graduate Level Tier-I Exam. 21.04.2013, IInd Sitting)
,d vk;kr dh yEckbZ rFkk pkSM+kbZ Øe'k% 8 lseh- rFkk 6 lseh-
5. In a rhombus ABCD, A = 60° and AB = 12 cm, gSaA vk;r dks mlds pkj 'kh"kks± ij bl izdkj dkVk tkrk gS
then diagonal BD will be— fd feyus okyh vkÑfr ,d le v"VHkqt dh Hkq tk (lseh-
,d leprqHkZqt ABCD esa A = 60° vkSj AB = 12 lseh- gS esa) D;k gS\
rks fod.kZ BD dh yackbZ gS (1) 3 e 11j – 7 (2) 5 e 13 j – 8
(1) 10 cm (2) 2 3 cm
(3) 6 cm (4) 12 cm (3) 5 e 7 j – 11 (4) 6 e 11 j – 9
(SSC CGL Tier-I Exam, 09.08.2015, IInd Sitting) (SSC CHSL DEO & LDC Exam. 17.02.2018)

BLAM–293
T;kfefr
11. ABCD is a Parallelogram and E is the mid point (1) 3 : 2 (2) 2 : 1
of DC, F is the mid-point of AB. If BE and DF (3) 3 : 1 (4) 5 : 2
meet AC in M and L respectively, them LM is equal 15. The angles of a parallelogram make the equcliv-
to ior of.
ABCD lekukarj prqHkZqt gS] vkSj E, DC osQ eè;&fcUnq gSA F, lekarj prqHkqt
Z osQ dks.kksa osQ lef¼Hkktd cukrs gSa
AB osQ eè; fcUnq gSA ;fn BE vkSj DF Øe'k% M vkSj L esa (1) vk;r (2) jXeprq HkqZt
AC ij feyrh gSA rc LM = ?
(3) oxZ (4) leayc prqHkqtZ
AC AC AC 2AC 16. In the quardrilateral ABCD, now, AB = 10 cm the
(1) (2) (3) (4)
4 3 2 3 length of the sides and the Perpendicular are now
12. In the figure given below, ABCD is a Parallelo- 7 cm and 8 cm. Find the value of AD = ?
gram. P is a point in BC such the PB : PC = 1 : 2, lekarj prqHkZt
q ABCD esa AB = 10 Hkqtkvksa AB rFkk AD Mkys
DP Produced meets AB Produced at Q. If the area x, 'kh"kZ yacksa dh yackbZ 7 cm l 8 cm gSaA AD dk eku Kkr
of the BPQ is 20 sq units, what is the area of djsaA
the DCP?
(1) 8.50 cm (2) 8.25 cm
uhps fn, x, vkÑfr esa ABCD ,d lekUrj prqHkqZt gS] ,d (3) 8.75 cm (2) 9.00 cm
fcUnq P, BC ij gS] bl izdkj gS fd] PB : PC = : 2, fd 17. In the given picture ABCD is a parallelogram and
c<+k;k x;k DP c<+k;k x, AB ls Q ij feyrs gSA ;fn BPQ E, F, G, H respectively AO, DO, CO and BO. The
dk {ks=kiQy 20 oxZ bdkbZ gSA rks DCP = dk {ks=kiQy D;k gSA EF FG GH HE
?
midpoint of are so
AD DC CB BA
A B
Q
fn, x, fp=k esa ABCD ,d lekUrj prqHkqt
Z gS] rFkk E, F, G,
H Øe'k% AO, DO, CO o BO os Q eè;fca n q gS a rks
EF FG GH HE
P ?
AD DC CB BA
D C
D C
(1) 20 Sq units (2) 30 Sq units F G
(3) 40 Sq units (4) None of these
13. Two similar parallelograms have corresponding E H
sides in the ratio 1 : K what is the ratio of their
areas?
A B
nks leku lekUrj prqHkqt Z dk vuqikr 1 : K esa blh i{k osQ gksrs (1) 1 : 1 (2) 1 : 2
gS] muosQ {ks=kksa dk vuqikr D;k gS\ (3) 1 : 3 (2) 1 : 4
(1) 1 : 3K2 (2) 1 : 4K2 18. ABCD is a trapezium and AB||CD||MN, M and
(3) 1 : K2 (4) 1 : 2K2 N are the mid-point of AD and BC respectively. If
14. In the figure given above M is the mid-point of AB = 14 cm and MN = 15 cm then what is the
the side CD of the Parallelogram ABCD what is value CD ?
ON : OB ? ABCD ,d leyc prqHkqt
Z gS] rFkk AB||CD||MN M rFkk
Åij fn, x, vkÑfr esa M lekraj prqHkqt
Z ABCD osQ lkbM N Øe'k% Hkqtkvksa AD rFkk BC osQ eè;fcanq gS ;fn AB = 14
CD dk eè;&lgdehZ gSA ON : OB = ? cm rFkk MN = 15 lsehñ rks CD dk eku Kkr djks\
A B A B

o
C M N
D
M

D C
(1) 16 cm (2) 18 cm
N (3) 8 cm (2) 10 cm

BLAM–294
T;kfefr
19. ABCD is Square, F is the midpoint of the side AB
Type-V
1 Questions Based on Circles
and E Find on the BC is such that BE = BC. If
3
the area of FBE is 147 m2 then find the length of 1. The in-radius of an equilateral triangle is of length
AC = ? 3 cm. Then the length of each of its medians is
ABCD ,d oxZ gS] F Hkqtk AB dk eè;fcUnq gS] E Hkqtk BC ,d lef=kckgq f=kHkqt dh vkarfjd f=kT;k 3 lseh gSA rnuqlkj ml
1 f=kHkqt dh izR;sd ekfè;dk dh yEckbZ fdruh gksxh\
ij bl izdkj gS] fd BE =
3
BC ;fn FBE dk {ks=kiQy
9
(1) 12 cm (2) cm
147 m gS] rks AC dh yEckbZ Kkr djsa\
2 2

(1) 21 2m (3) 4 cm (4) 9 cm


(SSC CHSL DEO & LDC Exam. 11.12.2011
(2) 63 m (Ist Sitting (East Zone)
2. What will be the area of in-circle which is in-
(3) 63 2m
scribed in an equilateral triangle having side 6
(4) 42 2m cm ?
,d leckgq f=kHkqt] ftldh Hkqtk 6 lseh- gS] ds var%o`Ùk dk
A B
{ks=kiQy fdruk gkssxk\

(1) cm2 (2) 3 cm2


2
(3) 6 cm2 (4) 3 cm2
(SSC CHSL DEO and LDC Exam. 2011)
3. If the circumradius of an equilateral triangle be
D C 10 cm, then the measure of its in-radius is ?
;fn ,d leckgq f=kHkqt dh ifjf=kT;k 10 lseh- gks] rks mldh
20. Rectangle/prqHkqt
Z ABCD esa AB + BC + CD + DA gS% var%f=kT;k dh eki fdruh gksxh\
(1) 2 BD ls vf/d (1) 5 cm. (2) 10 cm.
(3) 20 cm. (4) 15 cm.
(2) 2 BD ls de
(SSC CHSL DEO & LDC Exam. 04.12.2011
(3) 2 BD osQ cjkcj (IInd Sitting (East Zone)

(4) buesa ls dksbZ ugha 4. What will be the ratio of area of circum-circle
and incircle of an equilateral triangle ?
21. In the given figure, ABCD is a rectangle and P is
a point on DC such that BC = 24 cm, DP = 10 cm
,d leckgq f=kHkqt ds ifjo`Ùk rFkk var%o`Ùk ds {ks=kiQyksa dk
and CD = 15cm. If AP produced intersects BC vuqikr fdruk gksxk \
produced at Q, then find the length of AQ. (1) 2 : 1 (2) 4 : 1 (3) 8 : 1 (4) 3 : 2
(SSC FCI Assistant Grade Exam, 2012)
fn, x, fp=k esa] ABCD ,d vk;r gS vkSj DC ij fcanq P ,sls
5. The length of the two sides forming the right an-
fLFkr gS fd BC = 24 lseh, DP = 10 lseh vkSj CD = 15 gle of a right-angled triangle are 6 cm and 8 cm.
lseh gSA ;fn c<+kbZ xbZ js[kk AP c<+kbZ xbZ js[kk BC dks fcanq The length of its circum-radius is :
Q ij izfrPNsfnr djrh gS] rks AQ dh yackbZ Kkr dhft,A ,d ledks.k f=kHkqt dh ledks.k cukus okyh nks Hkqtkvksa dh
yackbZ 6 lseh rFkk 8 lseh gSA rnuqlkj] mldh ifj&f=kT;k dh
A D
yackbZ fdruh gS\
P (1) 5 cm (2) 7 cm (3) 6 cm (4) 10 cm
(SSC CHSL DEO & LDC Exam. 04.11.2012, Ist Sitting)

Q 6. In the given figure, O is the centre of circle and


B C OX is perpendicular to OY. If the area of XOY is
32, then find the area of circle.
(1) 24 cm (2) 26 cm
fuEufyf[kr fp=k esa O, o`Ùk dk dsUnz gS vkSj XO, OY ij yac
(3) 39 cm (4) 35 cm
(SSC CGL Tier-II Exam. 18.11.2020) gSA ;fn mlesa f=kHkqt XOY dk {ks=kiQy 32 gks] rks o`Ùk dk {ks=kiQy
fdruk gksxk \
BLAM–295
T;kfefr
11. The radius of two concentric circles are 9 cm and
15 cm. If the chord of the greater circle be a
tangent to the smaller circle, then the length of
that chord is
O
nks ladsanzh o`Ùkksa dh f=kT;k,¡ 9 cm vkSj 15 cm gSaA ;fn cM+s o`Ùk
dh thok NksVs o`Ùk dh Li'kZ js[kk gks rks ml thok dh yackbZ gSµ
(1) 24 cm (2) 12 cm
X Y
(3) 30 cm (4) 18 cm
(1) 64 (2) 256 (3) 16 (4) 32 (SSC CHSL DEO & LDC Exam. 04.12.2011
(Ist Sitting (North Zone)
(SSC CGL Tier-I Exam, 2012)
7. Two equal circles pass through their centres. If 12. Two equal circles of radius 4 cm intersect each
radius of every circle is 5 cm. Then, what will be other such that each passes through the centre
the length of common chord : of the other. The length of the common chord is :
nks cjkcj ds o`Ùk ,d&nwljs ds dsUnz ls xqtjrs gSaA ;fn muesa 4 lseh f=kT;k okys nks ,d leku o`Ùk ,d&nwljs dks bl rjg
izR;sd o`Ùk dh f=kT;k 5 lseh- gks] rks mudh mHk;fu"B thok dh dkVrs gSa fd izR;sd o`Ùk nwljs ds dsanz ij ls Hkh xqtjrs gSaA rnuqlkj
yEckbZ fdruh gksxh \ muds mHk;fu"B thok dh yackbZ fdruh gS \
5 3 (1) 2 3 cm (2) 4 3 cm
(1) 5 (2) 5 3 (3) 10 3 (4)
2 (3) 2 2 cm (4) 8 cm
(SSC CGL Tier-I Exam, 2012) FCI Assistant Grade-III Exam. 05.02.2012 (Paper-I)
8. AB and CD are two parallel chords which are op- East Zone (IInd Sitting)
posite to their parallel diameter. AB = 6 cm and 13. The length of two chords AB and AC of a circle
CD = 8cm. If radius of circle is 5cm then what is are 8 cm and 6 cm and BAC = 90°, then the
the distance between both chords ? radius of circle is
AB rFkk CD nks lekukarj thok,¡ gSa] tks vius lekarj O;kl dh ,d o`Ùk dh nks thok,¡ AB vkSj AC Øe'k% 8 lseh- rFkk 6 lseh-
nks foijhr fn'kkvksa esa [khaph xbZ gSaA buesa AB = 6 lseh rFkk CD yEch gSa vkSj BAC = 90° gSA rnuqlkj ml o`Ùk dh f=kT;k
= 8 lseh gSA rnuqlkj] ;fn o`Ùk dh f=kT;k 5 lseh gks] rks mu
fdruh gS\
thokvksa ds chp dh nwjh] lseh esa] fdruh gksxh\ (1) 25 cm (2) 20 cm
(1) 7 (2) 8 (3) 5 (4) 3 (3) 4 cm (4) 5 cm
(SSC CGL Tier-I Exam, 2012)
(SSC CHSL DEO & LDC Exam. 04.12.2011
9. There are two chords AB and BC in a circle whose (Ist Sitting (East Zone)
centre is O. If P and Q are the mid-points on AB 14. Two equal arc makes 60° and 75° angle on the
and BC respectively. What type of OQBP will be centre in two circles. What is the ratio of radii of
? the two circles ?
AB rFkk BC, O dsanz okys ,d o`Ùk dh nks thok,¡ gSaA rnuqlkj] nks o`Ùkksa ds dsankz sa ij nks cjkcj yackbZ dh pkisa Øe'k% 60° rFkk
;fn P rFkk Q Øe'k% AB rFkk BC ds eè;&fcUnq gksa] rks prqHkqt
Z 75° ds dks.k cukrh gSaA rnuqlkj mu nksuksa o`Ùkksa dh f=kT;kvksa dk
OQBP fdl izdkj dk gksxk\
vuqikr fdruk gksxk\
(1) Rhombus (2) Cyclic
(1) 5 : 2 (2) 5: 4 (3) 3 : 2 (4) 2: 1
(3) Rectangle (4) Square (SSC CHSL DEO & LDC Exam. 04.12.2011
(SSC CGL Tier-I Exam, 2012) East Zone : IInd Sitting and SSC Graduate Level
10. Two circles with the same radius r cut each other Tier-I Exam, 21.02.2013, IInd Sitting)
and pass through the centre. The length of com- 15. If two concentric circles are of radii 5 cm and 3
mon chord is — cm, then the length of the chord of the larger
leku f=kT;k r okys nks o`Ùk ,d nwljs dk izfrPNsn djrs gSa vkSj circle which touches the smaller circle is
,d nwljs ds dsanz ls xtjrs gSaA rks mHk;fu"B thok dh yackbZ gSµ ;fn nks ladsUnzh o`Ùkksa dh f=kT;k,¡ 5 lseh rFkk 3 lseh dh gksa]
rks muesa cM+s o`Ùk dh ml thok dh yackbZ fdruh gksxh] tks NksVs
3
(1) r (2) 3r (3) r (4) 5r o`Ùk dks Li'kZ djrh gS\
2
(1) 6 cm (2) 7 cm (3) 10 cm (4) 8 cm
(SSC CGL Tier-I Exam, 2012)
(SSC Graduate Level Tier-II Exam. 29.09.2013)

BLAM–296
T;kfefr
16. Chords AB and CD of a circle intersect externally (1) 36 cm (2) 17 cm
at P. If AB = 6 cm, CD = 3 cm and PD = 5 cm, then
(3) 34 cm (4) 26 cm
the length of PB is
(SSC CHSL DEO & LDC Exam. 10.11.2013 (IInd Sitting)
,d o`Ùk dh thok,¡ AB rFkk CD ,d&nwljs dks o`Ùk ds ckgj 22. The three equal circles touch each other externally.
P ij dkVrh gSaA rnuqlkj] ;fn AB=6 lseh] CD=3 lseh rFkk PD If the centres of these circles be A, B, C then ABC
=5 lseh gks] rks PB dh yackbZ fdruh gksxh \ is
(1) 5 cm (2) 7.35 cm rhu leku o`Ùk ,d nwljs dks ckgj ls Li'kZ djrs gSaA ;fn bu o`Ùkksa
(3) 6 cm (4) 4 cm
ds dsanz A, B, C gksa rks f=kHkqt ABC gS %
(SSC Delhi Police S.I. (SI) Exam. 19.08.2012)
17. Two circles of same radius 5 cm, intersect each (1) a right angled triangle
other at A and B. If AB = 8 cm, then the distance (2)an equilateral triangle
between the centre is : (3) an isosceles triangle
5 lseh dh ,dleku f=kT;k okys nks o`Ùk ,d&nwljs dks A rFkk (4) a scalene triangle
B ij dkVrs gSA ;fn AB = 8 lseh gks] rks mu nksuksa ds dsaæksa dh (SSC CGL Tier-I Re-Exam. (2013) 27.04.2014)

nwjh fdruh gksxh\ 23. Three circles of radii 3.5 cm, 4.5 cm, 5.5 cm touch
one-another exteernally. What will be the
(1) 6 cm (2) 8 cm
perimeter of triangle made by joining centres (in
(3) 10 cm (4) 4 cm
cm) ?
(SSC Graduate Level Tier-I Exam. 21.04.2013)
18. The length of the common chord of two circles of 3.5 lseh] 4.5 lseh] 5.5 lseh f=kT;k ds rhu o`Ùk ckgj ls
radii 30 cm and 40 cm whose centres are 50 cm ,d&nwljs dks Li'kZ djrs gSaA o`Ùkksa ds dsnzksa dks feykdj cus f=kHkqt
apart, is (in cm)
dk ifjeki (lseh esa) fdruk gksxk\
f=kT;k 30 lseh vkSj 40 lseh okys nks o`Ùkksa dh mHk;fu"B thok (1) 27 (2) [(3.5)2 + (4.5)2 + (5.5)2]
dh yackbZ] ftuds dsanz 50 lseh nwjh ij gSa] (lseh esa) gS (3) 27 (4) 13.5
(1) 12 (2) 24 (SSC CGL Tier-I Re-Exam. (2013) 20.07.2014 IInd Sitting)
(3) 36 (4) 48 24. A, B, C are three points on the circumference of a
(SSC GL Tier-I Exam. 21.04.2013 IInd Sitting)
19. Two circles touch each other externally. The dis- circle and if AB AC 5 2 cm and BAC = 90°,
tance between their centre is 7 cm. If the radius find the radius.
of one circle is 4 cm, then the radius of the other
A,B,C ,d o`Ùk dh ifjf/ ds rhu fcanq gSa vkSj ;fn AB =
circle is
nks o`Ùk ckgj ls ,d&nwljs ls feyrs@dks Li'kZ djrs gSaA muds dsankz sa AC = 5 2 lseh vkSj BAC = 90° gS] rks f=kT;k Kkr
ds chp nwjh 7 lseh gSA ;fn ,d o`Ùk dh f=kT;k 4 lseh gks] rks dhft,A
nwljs o`Ùk dh f=kT;k fdruh gksxh\ (1) 10 cm (2) 5 cm
(1) 3.5 cm (2) 3 cm (3) 20 cm (4) 15 cm
(3) 4 cm (4) 2 cm (SSC CGL Tier-I Exam. 19.10.2014 (Ist Sitting)
(SSC GL Tier-I Exam. 19.05.2013 Ist Sitting) 25. The angle subtended by a chord at its centre is
20. If the chord of a circle is equal to the radius of 60°, then the ratio between chord and radius is
the circle, then the angle subtended by the chord
at a point on the minor arc is
dsaæ ij thok }kjk varfjr dks.k 60° gS rks thok vkSj f=kT;k ds
;fn ,d o`Ùk dh thok mldh f=kT;k ds cjkcj gks] rks NksVh pki chp vuqikr D;k gksxk\
ij ml thok }kjk cuk;k x;k dks.k fdruk gksxk\ (1) 1 : 2 (2) 1 : 1

(1) 150° (2) 60° (3) 2 :1 (4) 2 : 1


(3) 120° (4) 30° (SSC CGL Tier-I Re-Exam. (2013) 27.04.2014)
(SSC CHSL DEO & LDC Exam. 10.11.2013 (IInd Sitting)
26. In a given circle, the chord PQ is of length 18 cm.
21. The radii of two circles are 10 cm and 24 cm. What AB is the perpendicular bisector of PQ at M. If
is the radius of that circle, whose area will be
MB = 3 cm, then the length of AB is
equal to sum of the area of these circle ?
fn;s x, o`Ùk esa thok PQ, 18 lseh yach gSA AB, PQ dk M
nks o`Ùkksa dh f=kT;k,¡ 10 lseh rFkk 24 lseh gSaA rnuqlkj] ml o`Ùk
dh f=kT;k fdruh gksxh] ftldk {ks=kiQy bu nksuksa o`Ùkksa ds {ks=kiQy ij yEc n~foHkktd gSA ;fn MB = 3 lseh] rks AB dh yackbZ
ds ;ksxiQy ds cjkcj gS\ D;k gksxh \

BLAM–297
T;kfefr
P feyrh gSaA rks P foHkkftr djrk gS AB dksµ
A
(1) Externally in ratio 5 : 2
cká :i ls 5 : 2 ds vuqikr esa
(2) Internally in ratio 2 : 5
O
vkUrfjd :i ls 2 : 5 ds vuqikr esa
M
P 3 9 Q (3) Internally in ratio 5 : 2
B vkUrfjd :i ls 5 : 2 ds vuqikr esa
(1) 27 cm (2) 30 cm (3) 28 cm (4) 25 cm (4) Externally in ratio 7 : 2
(SSC CAPFs SI, CISF ASI & Delhi Police SI cká :i ls 7 : 2 ds vuqikr esa
Exam, 21.06.2015 IInd Sitting)
31. PA and PB are two tangents drawn from point P
27. AB and CD are two parallel chords of a circle ly-
to the circles at point A and B whose centre is O.
ing on the opposite side of the centre and the
distance between them is 17 cm. The length of OAPB must be —
AB and CD are 10 cm and 24 cm respectively. The PA vkSj PB dk cká fcUnq P ls O dsanz okys ,d o`Ùk ij [khaph
radius (in cm) of the circle is : xbZ nks Li'kZ js[kk,¡ gSa tgk¡ fcUnq A rFkk B Li'kZ ds fcUnq gSaA
AB vkSj CD ,d o`Ùk dh nks lekukarj thok gSa tks dsUnz ds prqHkqt
Z OAPB vo'; gksxkµ
foijhr vksj fLFkr gSa vkSj muds chp dh nwjh 17 lseh- gSA AB (1) A rectangle (2) A rhombus
vkSj CD dh yackbZ Øe'k% 10 lseh- vkSj 24 lseh- gSA o`Ùk dh
(3) A square (4) A cyclic quadrilateral
f=kT;k (lseh- esa) gS % (SSC CPO SI & Assistant Intelligence Officer Exam, 2012)
(1) 13 (2) 9 32. SR is the common tangent of the circles whose
(3) 18 (4) 15 radii are 8 cm and 3 cm, the circles are at the
(SSC CGL Tier-I Exam, 16.08.2015
(IInd Sitting) TF No. 2176783)
distance of 13 cm. If S and R are the point of
28. Two circles having radii 4cm and 9cm touch each contact then length of SR is —
other externally at a point and a common tan- SR ozQe'k% 8 lseh rFkk 3 lseh f=kT;kvksa okys o`Ùkksa dh] ftuds
gent touches both the circles at point P and Q. dsankz sa esa 13 lseh dh nwjh gS] mHk;fu"V vuqLi'kZ js[kk gSA ;fn fcUnq
Then what is the area of square having side PQ ?
S rFkk R ozQe'k% laidZ ds fcUnq gSa] rks SR dh yackbZ gSµ
Øe'k% 4 lseh rFkk 9 lseh f=kT;k okys nks o`Ùk ,d&nwljs dks ,d
(1) 12 cm (2) 11 cm (3) 17 cm (4) 10 cm
fcUnq ij ckgj ls Li'kZ djrs gSa vkSj mudh ,d mHk;fu"B Li'kZ (SSC CPO SI & Assistant Intelligence Officer Exam, 2012
js[kk Øe'k% P rFkk Q fcUnqvksa ij mUgsa Li'kZ djrh gSA rnuqlkj and SSC CGL Tier-I Exam, 2012)

PQ Hkqtk okys oxZ dk {ks=kiQy fdruk gksxk \ 33. If the radii of two circles be 6 cm and 3 cm and
the length of the transverse common tangent be
(1) 97 cm2 (2) 194 cm2
8 cm, then the distance between the two centres
(3) 72 cm2 (4) 144 cm2
(SSC CGL Tier-I Exam, 2012)
is
29. If a chord makes 45° with the tangent drawn from ;fn nks o`Ùkksa dh f=kT;k,¡ 6 lseh rFkk 3 lseh gksa vkSj mHk;fu"B
one end itself is of 6 cm long. then what will be vuqizLFk Li'kZ js[kk dh yackbZ 8 lseh gks] rks nksuksa o`Ùkksa osQ osaQæksa
the radius of circle ?
osQ chp dh nwjh fdruh gksxh\
;fn ,d thok] rks vius ,d fljs ij [khaph xbZ Li'kZ js[kk ds
lkFk 45° dk dks.k cukrh gS] dh yEckbZ 6 lseh gks] rks ml o`Ùk (1) 145 cm (2) 140 cm
dh f=kT;k fdruh yEch gksxh\ (3) 150 cm (4) 135 cm
(1) 6 2 cm (2) 5 cm (SSC Graduate Level Tier-II Exam.16.09.2012)
(3) 3 2 cm (4) 6 cm 34. P and Q are two points on a circle with centre at
(SSC CGL Tier-I Exam, 2012) O. R is a point on the minor arc of the circle,
30. A and B are the centres of the circles whose radii between the points P and Q. The tangents to the
are 5 cm and 2 cm respectively. The common tan- circle at the points P and Q meet each other at
gent of circles AB extended to P. The P divides the point S. If PSQ = 20°, then PRQ = ?
AB — P vkSj Q ,do`Ùk] ftldk dsanz O gSa] ij fLFkr nks fcanq gSaA P vkSj
A vkSj B nks o`Ùkksa ds dasanz gSa ftudh f=kT;k,¡ Øe'k% 5 lseh vkSj Q ds chp ml o`Ùk dh NksVh pki ij ,d fcanq R gSA ml o`Ùk
2 lseh gSA o`Ùkksa dh mHk;fu"B vuqL i'kZ js[kk,¡ c<+kbZ gqbZ AB dks dh P vkSj Q fcanqvksa dh Li'kZ js[kk,¡ ,d&nwljs ls fcanq S ij

BLAM–298
T;kfefr
feyrh gSaA rnuqlkj] ;fn PSQ = 20° gks] rks PRQ fdlds
9 15
cjkcj gks xk\ (1)
2
cm2 (2) 12 15 cm2
(1) 80° (2) 200°
(3) 160° (4) 100° (3) 9 15 cm2 (4) 6 15 cm2
(SSC GL Tier-I Exam. 21.04.2013, Ist Sitting) (SSC GL Tier-I Exam. 21.04.2013, IInd Sitting)
35. Three circles of radii a, b, c touch one-another 39. In the given figure, ABC is a right angled triangle.
externally. The area of triangle formed by joining ABC = 90° and ACB = 60°. If the radius of the
centres is — smaller circle is 2 cm, then what is the radius (in
a, b, c f=kT;kvksa okys rhu o`Ùk ,d nwljs dks ckgj ls Li'kZ djrs cm.) of the larger circle?

gSaA muds dsnzksa dks tksM+us ls cus f=kHkqt dk {ks=kiQy gSµ nh xbZ vkÑfr esa] ABC ,d ledks.k f=kHkqt gSA ABC = 90°
rFkk ACB = 60° gSA ;fn NksVs o`Ùk dh f=kT;k 2 lseh- gS] rks
(1) (a b c )abc cM+s o`Ùk dh f=kT;k (lseh- esa) D;k gS\
(2) (a b c ) ab bc ca
(3) ab + bc + ca
(4) None of these
(SSC GL Tier-I Exam. 21.04.2013, IInd Sitting)
36. The radii of two concentric circles are 13 cm and
8 cm. AB is a diameter of the bigger circle and BD
is a tangent to the smaller circle touching it at D
and the bigger circle at E. Point A is joined to D.
The length of AD is (1) 5 cm (2) 6 cm
(3) 4 cm (4) 3 cm
nks ladsUnzh o`Ùkksa dh f=kT;k,¡ 13 lseh rFkk 8 lseh gSaA AB cM+s
40. In the given figure, B and C are the centres of the
o`Ùk dk O;kl gS vkSj BD ,d Li'kZ js[kk gS vkSj NksVs o`Ùk dks two circles. ADE is the common tangent to the
D ij Li'kZ djrh gS vkSj cM+s o`Ùk dks E ijA fcanq A dks D ls two circles. If the ratio of the radius of both the
tksM+ fn;k x;k gSA rnuqlkj] AD dh yackbZ fdruh gksxh\ circles is 3 : 5 and AC = 40, then what is the
value of DE?
(1) 20 cm (2) 19 cm (3) 18 cm (4) 17 cm
(SSC CHSL DEO & LDC Exam. 27.10.2013 IInd Sitting)
nh xbZ vkÑfr esa] B rFkk C nks o`Ùkksa ds dsUnz gSaA ADE nksuksa o`Ùkksa
37. A circle (with centre at O) is touching two inter- dh ,d mHk;fu"B Li'kZ js[kk gSA ;fn nksuksa o`Ùkksas dh f=kT;kvksa dk
secting lines AX and BY. The two points of con- vuqikr 3 : 5 gS rFkk AC = 40 gS] rks DE dk eku D;k gS\
tact A and B subtend an angle of 65° at any point
E
C on the circumference of the circle. If P is the
D
point of intersection of the two lines, then the
measure of APO is A
B C
,d o`Ùk (ftldk dsanz O gS) ijLij dkVus okyh js[kkvksa AX rFkk
BY dks Li'kZ dj jgk gSA ml o`Ùk dh ifjf/ ds fdlh fcanq C ij]
A rFkk B] 65° dk ,d dks.k cukrs gSAa rnuqlkj ;fn P mDr nksuksa (1) 3 15 (2) 5 15
Li'kZ js[kkvksa dk izfrNsnu fcanq gks] rks APO dk eki fdruk (3) 6 15 (4) 4 15
gksxk \ (SSC CHSL DEO & LDC Exam. 17.02.2018)
(1) 25° (2) 65° 41. In the given figure, O is centre of the circle. Circle
(3) 90° (4) 40° has 3 tangents. If QPR = 45°, then what is the
(SSC CHSL DEO & LDC Exam. 28.10.2012 (Ist Sitting) value (in degrees) of QOR ?
38. C1 and C2 are two concentric circles on centre O. nh xbZ vkÑfr esa] O o`Ùk dk dsUnz gSA o`Ùk ij 3 Li'kZ js[kk,¡ gSaA
Their radii are 12 cm and 3 cm respectively. From
;fn QPR = 45° gS] rks QOR dk eku (fMxzh esa) D;k gS\
point A on C1, two tangents are drawn on C2 at
point B and C. Then area of ABOC is — Q
C1 vkSj C2 O ij dsankz sa ds lkFk nks ladsanzh o`Ùk gSaA mudh f=kT;k,¡
O P
Øe'k% 12 lseh vkSj 3 lseh gSaA o`Ùk C1 ij ,d fcanq A ls o`Ùk
C2 ij [khaph xbZ nks Li'kZ&js[kkvksa ds Li'kZ fcanq B rFkk C gSaA rks R
prqHkqt
Z ABOC dk {ks=kiQy gSµ (1) 67.5 (2) 72 (3) 78 .5 (4) 65
(SSC CHSL DEO & LDC Exam. 17.02.2018)

BLAM–299
T;kfefr
42. In the given figure, two identical circles of radius
4 cm. touch each other. A and B are the centres CQ
of the two circles. If RQ is a tangent to the circle, N M
R S
then what is the length (in cm.) of RQ? A B
O P
nh xbZ vkÑfr esa] nks leku o`Ùk ftudh f=kT;k 4 lseh- gSa
,d&nwljs dks Li'kZ dj jgs gSaA nksuksa o`Ùkksa ds dsUnz A rFkk B gSaA
;fn RQ o`Ùk ij ,d Li'kZ js[kk gS] rks RQ dh yEckbZ (lseh- DL
esa) D;k gS\

R
(1) e5 3 j – 3 (2) e4 3 j – 2
S
(3) e2 5 j – 1 (4) e2 6 j – 1
Q P
A B (SSC CHSL DEO & LDC Exam. 20.02.2018)
45. In the given figure, PQRS is a square of side 20 cm
and SR is extended to point T. If the length of QT
is 25 cm, then what is the distance (in cm.) be-
(1) 3 3 (2) 2 6
tween the centres O1 and O2 of the two circles?
(3) 4 2 (4) 6 2 nh xbZ vkÑfr esa] PQRS, 20 lseh Hkqtk okyk ,d oxZ gS rFkk
(SSC CHSL DEO & LDC Exam. 17.02.2018) SR dks fcUnq T rd c<+k;k x;k gSA ;fn QT dh yEckbZ 25 lseh
43. In the given figure, PQRS is a square inscribed in gS] rks nksuksa o`Ùkksa ds dsUnz O1 rFkk O2 ds eè; dh nwjh (lseh- esa)
a circle of radius 4 cm. PQ is produced till point
D;k gS\
Y. From Y a tangent is drawn to the circle at point
R. What is the length (in cm.) of SY? P Q

nh xbZ vkÑfr esa] PQRS, 4 lseh f=kT;k okys ,d o`Ùk esa vafdr
,d oxZ gSA PQ dk fcUnq Y rd c<+k;k x;k gSA o`Ùk ij Y ls O1

fcUnq R ij ,d Li'kZ js[kk [khaph x;h gSA SY dh yEckbZ (lseh- O2

esa) D;k gS\ S R T

(1) 5 10 (2) 4 10
S R
(3) 8 5 (4) 16 2
(SSC CHSL DEO & LDC Exam. 20.02.2018)
O
46. Two parallel chords are one the one side of the
Y
centre of a circle. The length of the two chords is
P Q
24 cm. and 32 cm. If the distance between the
two chords is 8 cm., then what is the area (in
(1) 4 10 (2) 2 10 (3) 6 10 (4) 3 5 cm2.) of the circle?
(SSC CHSL DEO & LDC Exam. 20.02.2018) nks lekarj thok,¡ ,d o`Ùk ds dsUnz dh ,d vksj gSaA nksuksa
44. In the given figure, CD and AB are diameters of thokvksa dh yEckbZ 24 lseh rFkk 32 lseh gSA ;fn nksuksa thokvksa
circle and AB and CD are perpendicular to each
ds eè; 8 lseh dh nwjh gS] rks o`Ùk dk {ks=kiQy (lseh2- esa) D;k
other. LQ and SR are perpendiculars to AB and
CD respectively. Radius of circle is 5 cm, PB : PA
gS\
= 2 : 3 and CN : ND = 2 : 3. What is the length (in (1) 724.14 (2) 832.86
cm.) of SM? (3) 924.12 (4) 988.32
(SSC CGL Tier-II (CBE) Exam. 09.03.2018)
nh xbZ vkÑfr esa] CD rFkk AB o`Ùk ds O;kl gSa rFkk AB rFkk
47. A line cuts two concentric circles. The lengths of
CD ,d&nwljs ij yEc gSaA LQ rFkk SR Øe'k% AB rFkk CD
chords formed by that line on the two circles are
ij yEc gSaA o`Ùk dh f=kT;k 5 lseh gS] PB : PA = 2 : 3 rFkk 4 cm and 16 cm. What is the difference (in cm.2)
CN : ND = 2 : 3 gSaA SM dh yEckbZ (lseh- eas) D;k gS\ in squares of radii of two circles?

BLAM–300
T;kfefr
,d js[kk nks ladfs Unzr o`Ùkksa dks dkVrh gSA ml js[kk }kjk o`Ùkksa 51. Two circles touch each other at point X. A com-
ij cukbZ xbZ thokvksa dh yEckbZ 4 lseh- rFkk 16 lseh- gSaA nksuksa mon tangent touch them at two distinct points Y
and Z. If another tangent passing through X cut
o`Ùkksa dh f=kT;kvksa ds oxks± dk varj (lseh-2 esa) D;k gS\ YZ at A and XA = 16 cm., then what is the value
(1) 240 (2) 120 (in cm.) of YZ?
(3) 60 (4) 90 nks o`Ùk ,d&nwljs dks fcUnq X ij Li'kZ djrs gSaA ,d leku Li'kZ
(SSC CGL Tier-II (CBE) Exam. 18.02.2018)
js[kk mUgsa nks vyx fcUnqvksa Y rFkk Z ij Li'kZ djrh gSA ;fn X
48. In the given figure, AB is a diameter of the circle
with centre O and XY is the tangent at a point C. ls xqtjus okyh ,d vU; Li'kZ js[kk YZ dks A ij dkVrh gS rFkk
If ACX = 35°, then what is the value (in degrees) XA = 16 lseh- gS] rks YZ dk eku (lseh- esa) D;k gS\
of CAB? (1) 18 (2) 24
nh xbZ vkÑfr eas] AB ,d o`Ùk ftldk dsUnz O gS] dk O;kl (3) 16 (4) 32
gS rFkk XY, fcUnq C ij ,d Li'kZ js[kk gSA ;fn ACX = 35° (SSC SGL Tier-II (CBE) Exam. 19.02.2018)
52. There are 8 equidistant points A, B, C, D, E, F, G
gS] rks CAB dk eku (fMxzh esa) D;k gS\
and H (in same order) on a circle. What is the
A X value of FDH (in degrees)?
,d o`Ùk ij 8 lekukarj fcUnq A, B, C, D, E, F, G rFkk H
(blh Øe eas) gSaA FDH dk eku (fMxzh eas) D;k gS\
O C (1) 22.5 (2) 45
(3) 30 (4) 42.5
(SSC SGL Tier-II (CBE) Exam. 19.02.2018)
53. In the given figure, triangle PQR is a right angled
triangle of Q. If PQ = 35 cm and QS = 28 cm, then
B Y
what is the value (in cm) of SR?
(1) 45 (2) 35
nh xbZ vkÑfr esa] f=kHkqt PQR, Q ij ,d ledks.k f=kHkqt gSA
(3) 55 (4) 65
(SSC CGL Tier-II (CBE) Exam. 18.02.2018)
;fn PQ = 35 lseh rFkk QS = 28 lseh gS rks SR dk eku
49. Two circles are having radii 9 cm. and 12 cm. The (lseh- esa) D;k gS\
distance between their centres is 15 cm. What is
the length (in cm.) of their common chord? P
nks o`Ùkksa dh f=kT;k,¡ 9 lseh- rFkk 12 lseh- gSaA nksuksa ds dsUnzkas ds
eè; dh nwjh 15 lseh- gSA mudh lkekU; thok dh yEckbZ (lseh-
esa) D;k gS\ S
(1) 6.8 (2) 13. 6
(3) 7.2 (4) 14. 4
Q
(SSC SGL Tier-II (CBE) Exam. 19.02.2018) R
50. Two circles touch each other at point X. Two com-
mon tangents of the circles meet at point P and (1) 35.33 (2) 37.33
none of the tangents passes through X. These
(3) 41.33 (4) 43.33
tangents touch the smaller circle and the larger
54. Two circles with centres A and B of radii 5 cm
circle at points B and C. If the radius of the larg-
and 3 cm respectively touch each other internal-
er circle is 15 cm. and C.P. = 20 cm., then what is
ly. If the perpendicular bisector of AB meets the
the radius (in cm.) of the smaller circle?
bigger circle at P and Q, then the value of PQ is
nks o`Ùk fcanq X ij ,d&nwljs dks Li'kZ djrs gSaA o`Ùkksa dh nks leku
5 cm, 3 cm f=kT;k okys rFkk A, B osQUnz okys nks o`Ùk ,d
Li'kZ js[kk,¡ fcanq P ij feyrh gSa rFkk dksbZ Hkh Li'kZ js[kk X ls
nwljs dks vkarfjd :i ls Li'kZ djrs gSaA AB dk yEc lef¼Hkktd
ugha xqtjrh gSA ;g Li'kZ js[kk,¡ NksVs o cM+s o`Ùk dks fcanq B rFkk
cM+s&o`Ùk dks fcUnq P rFkk Q ij feyrk gS] rc PQ gS]
C ij Li'kZ djrh gSaA ;fn cM+s o`Ùk dh f=kT;k 15 lseh- rFkk CP
= 20 lseh- gS] rks NksVs o`Ùk dh f=kT;k (lseh- esa) D;k gS\ (1) 6 cm (2) 2 6 cm
(1) 3.5 (2) 3.75
(3) 4.25 (4) 4.45 (3) 3 6 cm (4) 4 6 cm
(SSC SGL Tier-II (CBE) Exam. 19.02.2018) [SSC CPO SI 2014]

BLAM–301
T;kfefr
55. PQ and RS are common tangents to two circles 58. A line cuts two concentric circles. The lengths of
intersecting at A and B. AB when produced both chords formed by that line on the two circles are
sides, meet the tangents PQ and RS at X and Y, 4 cm and 16 cm. What is the difference (in cm2)
respectively. IF AB = 3 cm, xy = 5 cm, then PQ (in in squares of radii of two circles?
cm) will be ,d js[kk nks laosQfUnzr o`Ùkksa dks dkVrh gSA ml js[kk }kjk o`Ùkkas
PQ vkSj RS nks o`Ùkksa dh loZfu"B Li'kZ js[kk,¡ gS] tks A vkSj B ij cukbZ xbZ thokvksa dh yEckbZ 4 lsehñ rFkk 16 lseh gSA nks
ij ,d nwljs dks dkVrs gSaA nksuksa rjiQ (Hkqtk) AB js[kk dks o`Ùkksa dh f=kT;kvksa osQ oxks± dk varj (lsehñ2 esa) D;k gS\
c<+kus ij Øe'k% X vkSj Y fcUnq ij Li'kZ js[kk,¡ PQ vkSj RS (1) 240 (2) 120
ij feyrh gSaA ;fn AB = 3 lseh] XY = 5 lseh- gks rks crkb, (3) 60 (4) 90
fd PQ fdrus lseh gksxk\ (SSC CGL Tier-II 2017)

(1) 3 cm (2) 4 cm 59. In figure triangle ABC is drawn such that AB is


tangent to a circle at A whose radius is 10 cm
(3) 5 cm (4) 2 cm
and BC passes through centre of the circle. Point
56. In the given figure, PR and ST are perpendicular
C lies on the circle. If BC = 36 cm and AB = 24
to tangent QR. PQ paisess through centre O of
cm, then what is the area (in cm2) of triangle ABC?
the circle whase diameter is 10 cm. If PR = 9 cm
then what is the length (in cm) of ST. nh xbZ vkÑfr esa] f=kHkqt ABC dks bl izdkj crk;k x;k gS fd
nh xbZ vkÑfr esa Li'kZ js[kk QR ij PR rFkk ST yEc gSA PQ AB ,d 10 lseh- f=kT;k okys o`Ùk dh A ij Li'kZ js[kk gSa rFkk
o`Ùk osQ osQUnz fcUnq O ls xqtjrh gS ftldk O;kl 10 lseh gSA ;fn BC o`Ùk osQ osQUnz ls gksdj xqtjrh gSA fcUnq C o`Ùk ij fLFkr gSA

PR dh yEckbZ 9 lseh gS rks ST dh yEckbZ (lseh esa) D;k gSa\ ;fn BC = 36 lsehñ rFkk BC = 24 lsehñ gks] rks f=kHkqt ABC
dk {ks=kiQy (lsehñ2 esa) D;k gS\
P

O C 10
O
10
9 D
10
S
B
A 24
R H T Q
(1) 134.5 (2) 148
(3) 166.15 (4) 180
(1) 1 (2) 1.25
60. In the given figure QRU = 72°, TRS = 15° and-
(3) 1.5 (4) 2
PSR = 95°, then what is the value (in degrees) of
57. I and O are respectively the incentre and circum-
PQR = ?
centre of a triangle ABC. The line AI produced
intersects the circumcircle of ABC at the point nh xbZ vkÑfr esa] QRU = 72°, TRS = 15° rFkk PSR
D. If ABC = x°, BID = y° and BOD = z°, then = 95° rks PQR dk eku (fMxzh esa) D;k gS\
z x
?
y P Q
IrFkk O ABC osQ var% rFkk ifjosQUnz gSaA js[kk AI c<+k;h x;h
rFkk ABC osQ ifjo`r dks fcUnq D ij izfrPNsfnr djrh gSA ;fn
ABC = x°, BID = y° rFkk BOD = z° gS] rc
S
z x
?
y T R U

(1) 3 (2) 1
(1) 85 (2) 95
(3) 2 (4) 4
(3) 75 (4) 90
(SSC CGL Tier-I 2013)
(SSC CGL Tier-II 2017)

BLAM–302
T;kfefr
61. In the figure, PQ is the diameter of the circle, (1) 1 : 3 (2) 1 : 9
what is the measure (indegrees of ) QRS = ? (3) 1 : 8 (4) 1 : 4
nh xbZ vkÑfr esa PQ o`Ùk dk O;kl gSA QSR dk eki (fMxzh 65. A square is inscribed in a quarter circle in such a
esa) D;k gS\ way the two of its adjacent vertices on the radius
are equidistant from the centre and other two
R vertices lie of the circumference. In the side on
P
F 5I
square is GH 2 JK cm, then what is radius (in cm)
43 of the circle?
,d pkSFkkbZ o`Ùk esa ,d oxZ bl izdkj cuk;k x;k gS fdlh mldh
S
Q f=kT;k ij fLFkr nks layXu 'kh"kZ osQUnz ls leku nwjh ij gS rFkk

(1) 23° (2) 37° nks vU; 'kh"kZ ifjf/ ij fLFkr gSA ;fn oxZ dh Hkqtk
FG 5 IJ
(3) 47° (4) 57° H 2K
62. ABC is a cyclic triangle and the bisectors of BAC, lsehñ gS rks o`Ùk dh f=kT;k (lsehñ esa) D;k gS\
ABC and BCA meet the circle at P, Q and R
respectively. Then the angle RQP is (1) 2 (2) 2.5
ABC ,d pØh; f=kHkqt gS vkSj BAC, ABC vkSj BCA (3) 5 (4) 10
osQ lef¼Hkktd Øe'k% P, Q rFkk R fcUnq o`Ùk ij feyrs gS] 66. AB and AC are the two tangents to a circles whase
RQP = ? radius is 6 cm. If BAC = 60; then what is the

(1) 90 –
B
(2) 90 +
C value (in cm) of d AB 2
AC 2 =? h
2 2
AB rFkk AC ,d o`Ùk ij nks Li'kZ js[kk,¡ gS ftldh f=kT;k 6
A B
(3) 90 (4) 90
2 2
(SSC CGL Tier-I 2015)
lseh- gSA ;fn BAC = 60° gS rks d AB 2
AC 2 h dk eku
63. XY and XZ are tangents to a circle. ST is anothe (lseh esa) D;k gS\
tangent to the circle at the point R on the circle
which intersects XY and XZ at S and T respec- (1) 6 6 (2) 4 6
tively. If XY = 9 cm and TX = 15 cm, then RT is
(3) 9 3 (4) 8 3
XY rFkk XZ ,d o`Ùk dh Li'kZ&js[kk,¡ gSA o`Ùk ij R fcanq ls
,d vksj Li'kZ js[kk ST [khph tkrh gS] tks XY rFkk XZ dks (SSC CGL Tier-II 2017)

Øe'k% S rFkk T fcUnq ij dkVrh gSA ;fn XY = 15 lseh vkSj 67. The radius of the circumcircle of a right angled
triangle is 15 cm and they radius of its incircle is
TX = 9 lseh- rks RT = ?
6 cm. Find the sides of the triangle.
(1) 4.5 cm (2) 3 cm
(3) 7.5 cm (4) 6 cm fdlh ledks.k f=kHkqt osQ ifjo`Ùk rFkk var% o`Ùk dh f=kT;k
(SSC CGL Tier-I 2015) Øe'k% 15 cm rFkk 6 cm gSA f=kHkqt dh Hkqtk,¡ Kkr djks\
64. If the given figure, E and F are the centres of two
(1) 30, 40, 41 (2) 18, 24, 30
identical circles. What is the ratio of area of tri-
angle AOB to the area of triangle DOC? (3) 30, 24, 25 (4) 24, 36, 20
(SSC CGL Tier-I 2013)
nh xbZ vkÑfr esa E rFkk F nks le:i o`Ùkksa osQ osQUnz gSaA f=kHkqt
68. A square is inscribed in a quarter-circle in such a
AOB osQ {ks=kiQy dk f=kHkqt DOC osQ {ks=kiQy ls D;k vuqikr
manner that two its adjacent vertices lie on the
gS\ two radii at an equal distance from the centre,
while the other two vertices lie on the circular
A B
are. If the square has sides of length x, them the
radius of the circle is:
O prqFkkZ'k o`Ùk ij ,d oxZ bl rjg [khpk tkrk gS fd mlosQ vklu
E F 'kh"kZ osQUnz ls leku nwjh ij f=kT;k ij fLFkr gSaA tcfd nks vkSj
'kh"kZ o`Ùkh; pki ij fLFkr gSA ;fn oxZ osQ Hkqtk dh yEckbZ x gS]
D C rks o`Ùk dh f=kT;k Kkr djsa\
BLAM–303
T;kfefr
71. AB is a chord of circle with O as centre. C is a
16 x 2x
(1) (2) poin on the circle such that. OC AB and OC in-
x 4 x tersects AB at P. If PC = 2 cm and AB = 6 cm then
the diameter of the circle is—
5x AB, O osQUnz okys o`Ùk dh thok gSA o`Ùk ij fcUnq C bl izdkj
(3) (4) 2x
2 gS fd OC AB vkSj f=kT;k OC thok AB dks P ij dkVrh gSA
(SSC CGL Tier-I 2015) ;fn PC = 2 lsehñ vkSj AB = 6 lseh- gks rks o`Ùk dk O;kl
69. In the given figure O is the centre of the circle OQ fdruk gS\
is perpendicular to RS and SRT = 30°. If RS = (1) 6 lseh (2) 6.5 lseh
2
10 2 then what is the value of PR = ? (3) 13 lseh (4) 12 lseh
(SSC CGL Tier-I 2016)
nh xbZ vkÑfr esa O o`Ùk dk osQUnz gS OQ, RS dk yEc rFkk 72. Two chords of length 20 cm and 24 cm are drawn
perpendicular to each other in a circle of radius
SRT = 30° gSA ;fn RS = 10 2 gS] rks PR2 dk eku D;k is 15 cm. What is the distance between the points
gksxk\ of intersection of these chords (in cm) from the
center of the circle?
15 lseh- f=kT;k okys o`Ùk esa 20 lseh- rFkk 24 lseh- dh nks

P
thokvksa dks ,d nwljs ij yEcor~ [khpk tkrk gSA o`Ùk osQ osQUnz
S
rFkk bu thokvksa dks dkVus okys fcUnq osQ chp dh nwjh (lsehñ)
o fdruh gS\
(1) 206 (2) 182
Q
(3) 250 (4) 218
R T 73. In tangents drawn at point A and B of a circle
with centre O, meet at P. If AOB = 120° and AP
= 6 cm then what is the area of triangle (in cm)
APB?
(1) 200 1 e 3 j (2) 300, 2 e 3 j osQUnz O okys o`Ùk ij fLFkr fcUnq A rFkk B ls Li'kZ js[kk,¡ [khph
tkrh gS] tks fcUnq P ij feyrh gSA ;fn AOB = 120° rFkk AP
(3) 200 2 e 3 j (4) 100 3 e 2 3 j =6 lseh gSa rks f=kHkqt APB dk {ks=kiQy (lseh2 esa) D;k gSa\
70. In the given figure OQ = QR = RT and O is the (1) 6 3 (2) 8 3
centre of the circle. What is the PTQ?
(3) 9 (4) 9 3
nh xbZ vkÑfr esa OQ = QR = RT rFkk O o`Ùk dk osQUnz gSaA
74. In the given figure, TB passes through centre O.
PTQ D;k gS\ What is the radius of the circle?
nh xbZ vkÑfr esa] TB osQUnz O ls xqtjrh gSA o`Ùk dh f=kT;k
D;k gS\
T
T
6

S R P O
4
A
P O Q x
B

(1)
e2 5 j (2)
e3 2 j
3 5

(1) 30° (2) 60°


(3)
e3 5 j (4)
e2 3 j
(3) 45° (4) 90° 2 5

BLAM–304
T;kfefr
75. In the given figure, PQR is a triangle in which, 78. Two circles touch each other externally. The dis-
PQ = 24 cm, PR = 12 cm, and altitude PS = 8 cm. tance between their centres is 8 cm. If the radius
If PT is the diameter of the circum-circle, then of one circle is 3 cm, then the radius of the other
what is the length (in cm) of circumradius? circle is
nh xbZ vkÑfr esa ,d f=kHkqt gS] ftlesa PQ = 24 lseh PR = nks o`Ùk ,d nwljs dks ckár% ls Li'kZ djrs gSAa muosQ osaQnzksa osQ chp
12 lseh- rFkk 'kh"kZyEc PS = 8 lseh gSA PT o`Ùk dk O;kl gS dh nwjh 8 lseh- gSA ;fn ,d o`Ùk dh f=kT;k 3 lseh- gS] rks nwljs
rks ckÞ; f=kT;k dh yEckbZ (lseh- esa) D;k gS\ o`Ùk dh f=kT;k Kkr djsaA
(1) 6 cm (2) 4 cm
P
(3) 3 cm (4) 5 cm
(SSC CHSL (10+2) Tier-I Exam. 18.03.2020)

R B
Q 79.
S
E F

T
A C
(1) 15 (2) 18 D
(3) 20 (4) 21
A circle is inscribed in the triangle ABC whose
76. In the figure, chords AB and CD of a circle inter-
sides are given as AB = 10, BC = 8, CA = 12 units
sect externally at P. If A B = 4 cm, CD = 11 cm
as shown in the figure. The value of AD × BF is :
and PD = 15 cm, then the length of PB is :
fuEukafdr fp=k esa] o`Ùk dh thok,a AB vkSj CD ,d&nwljs dks f=kHkqt ABC esa ,d var%o`Ùk [khapk x;k gS ftldh Hkqtk,¡ AB
= 10, BC = 8, CA = 12 bdkbZ osQ :i esa nh xbZ gS tSlk fd
ckg~; :i ls fcanq P ij izfrPNsfnr djrh gSA ;fn AB = 4 lseh,
CD = 11 = lseh vkSj PD = 15 lseh gS] rks PB dh yackbZ Kkr
fp=k esa n'kkZ;k x;k gSA AD × BF dk eku Kkr dhft,A
(1) 18 units (2) 21 units
dhft,A
(3) 16 units (4) 15 units
B (SSC CHSL (10+2) Tier-I Exam. 17.03.2020)
A 80. A pair of tangents AB and AC are drawn from a
point which is at a distance of 10 cm from the
P
centre O of a circle of radius 6 cm, then the area
C of quadrilateral ABOC in cm2, is :
D
fdlh ,sls fcanq ls] Li'kZjs[kk&;qXe AB vkSj AC dks [khapk tkrk
(1) 10 cm (2) 8 cm
gS] tks 6 lseh- f=kT;k okys o`Ùk osQ osaQnz O ls 10 lseh- dh nwjh
(3) 14 cm (4) 12 cm
(SSC CGL Tier-II Exam. 18.11.2020) ij gS] rks prqHkqZt ABOC dk {ks=kiQy lseh-2 esa Kkr djsaA
77. In the given figure, chords A B and CD are inter- (1) 52 (2) 72
secting each other at point L. Find the length of (3) 60 (4) 48
AB. (SSC CHSL (10+2) Tier-I Exam. 17.03.2020)

nh xbZ vkÑfr esa] thok,a AB vkSj CD ,d&nwljs dks fcanq L 81. A circle is inscribed in a triangle ABC. It touches
sides AB, BC and AC at points R, P and Q respec-
ij izfrPNsfnr djrh gSaA AB dh yackbZ Kkr djsaA
tively. If AQ = 3.5 cm, PC = 4.5 cm and BR = 7 cm,
then the perimeter (in cm) of the triangle ABC is:
D 11 cm f=kHkqt ABC osQ vanj ,d o`Ùk [khapk tkrk gSA ;g Hkqtk AB,
C
6 cm BC vkSj AC dh ozQe'k% fcanq R, P vkSj Q ij LIk'kZ djrk gSA
A x cm
L ;fn AQ = 3.5 cm, PC = 4.5 cm vkSj BR = 7 cm gS rks
B 4 cm
ABC dk ifjeki (lseh esa) Kkr djsaA

(1) 22.5 cm (2) 21.5 cm (1) 45 (2) 28


(3) 15 (4) 30
(3) 24.5 cm (4) 23.5 cm
(SSC Delhi Police SI, CAPFs SI & CISF ASI
(SSC CHSL (10+2) Tier-I Exam. 17.03.2020)
(CPO) Exam 23.11.2020)

BLAM–305
T;kfefr
82. PA and PB are two tangents from a point P out- 37. (2) 38. (3) 39. (2) 40. (3) 41. (2) 42. (3)
side the circle with centre O at the points A and
B on it. If APB = 130°, then OAB is equal to: 43. (2) 44. (3) 45. (1) 46. (1) 47. (4) 48. (2)
49. (2) 50. (3) 51. (1) 52. (2) 53. (1) 54. (3)
O osaQnz okys fdlh o`Ùk osQ ckgjh fcanq P ls] ml o`Ùk ij fLFkr
fcanq A vkSj B rd [khaph xbZ nks Li'kZ js[kk,¡ PA vkSj PB gSaA 55. (4) 56. (2) 57. (3) 58. (1) 59. (4)

;fn APB = 130° gS rks OAB dk eku Kkr djsaA


(1) 50° (2) 35° Test-3
(3) 65° (4) 45°
(SSC Delhi Police SI, CAPFs SI & CISF ASI
Questions Based on Polygons
(CPO) Exam 24.11.2020)

1. (2) 2. (1) 3. (2) 4. (1) 5. (3) 6. (2)


SHORT ANSWERS
7. (4) 8. (3) 9. (3) 10. (3) 11. (4) 12. (4)
Test-1 13. (2) 14. (3) 15. (2) 16. (4) 17. (3) 18. (1)
Questions Based on Angles 19. (2) 20. (2) 21. (1) 22. (2) 23. (4) 24. (2)
25. (3)
1. (1) 2. (3) 3. (1) 4. (3) 5. (3) 6. (4)
7. (3) 8. (1) 9. (1) 10. (2) 11. (2) 12. (4)
Test-4
13. (1) 14. (2) 15. (3) 16. (1) 17. (4) 18. (2)
Questions Based on Quadrilaterals
19. (1) 20. (4) 21. (2) 22. (2) 23. (3) 24. (2)
25. (2) 26. (4) 27. (2) 28. (4) 29. (2) 30. (3)
(Rhombus, trapezium etc.)
31. (2) 32. (2) 33. (2) 34. (3) 35. (4) 36. (4)
37. (2) 38. (1) 39. (2) 40. (4) 41. (1) 42. (3) 1. (4) 2. (2) 3. (3) 4. (1) 5. (4) 6. (1)
43. (1) 44. (1) 45. (4) 46. (3) 47. (4) 48. (3) 7. (4) 8. (1) 9. (4) 10. (1) 11. (2) 12. (4)
49. (1) 50. (1) 51. (2) 52. (3) 53. (3) 54. (2) 13. (3) 14. (2) 15. (1) 16. (3) 17. (2) 18. (1)
55. (1) 56. (3) 57. (3) 58. (4) 59. (1) 60. (2) 19. (4) 20. (1) 21. (3)
61. (3) 62. (4) 63. (2) 64. (4) 65. (3) 66. (3)
67. (3) 68. (1) 69. (4) 70. (2) 71. (2) 72. (1)
Test-5
73. (4) 74. (3) 75. (4) 76. (1) 77. (1) 78. (3)
Questions Based on Circles
79. (3) 80. (4) 81. (2) 82. (4) 83. (3) 84. (3)
85. (3) 86. (3) 87. (3) 88. (1) 89. (4) 90. (4)
91. (2) 92. (2) 93. (1) 94. (2) 95. (1) 96. (2) 1. (4) 2. (4) 3. (1) 4. (2) 5. (1) 6. (1)
97. (3) 98. (2) 99. (1) 100. (4) 101. (2) 102. (1) 7. (2) 8. (1) 9. (2) 10. (2) 11. (1) 12. (2)
103. (2) 104. (4) 105. (4) 106. (3) 107. (2) 108. (1) 13. (4) 14. (2) 15. (4) 16. (2) 17. (1) 18. (4)
109. (1) 110. (2) 111. (4) 112. (4) 19. (2) 20. (2) 21. (4) 22. (2) 23. (1) 24. (2)
25. (2) 26. (2) 27. (1) 28. (4) 29. (3) 30. (1)
Test-2 31. (4) 32. (1) 33. (1) 34. (4) 35. (1) 36. (2)
Questions Based on Length of 37. (1) 38. (3) 39. (2) 40. (4) 41. (1) 42. (3)
sides of a Triangle 43. (1) 44. (4) 45. (1) 46. (2) 47. (3) 48. (3)
49. (4) 50. (2) 51. (4) 52. (2) 53. (2) 54. (4)
1. (3) 2. (4) 3. (4) 4. (2) 5. (4) 6. (4)
55. (2) 56. (1) 57. (3) 58. (3) 59. (4) 60. (1)
7. (3) 8. (2) 9. (2) 10. (2) 11. (2) 12. (3)
13. (3) 14. (1) 15. (3) 16. (3) 17. (4) 18. (2) 61. (3) 62. (1) 63. (4) 64. (2) 65. (2) 66. (1)

19. (1) 20. (2) 21. (1) 22. (1) 23. (1) 24. (3) 67. (2) 68. (3) 69. (3) 70. (2) 71. (2) 72. (1)

25. (3) 26. (4) 27. (3) 28. (4) 29. (4) 30. (3) 73. (4) 74. (3) 75. (2) 76. (2) 77. (1) 78. (4)
31. (2) 32. (2) 33. (2) 34. (3) 35. (3) 36. (2) 79. (2) 80. (4) 81. (4) 82. (3)

BLAM–306
T;kfefr
=6:4:3
EXPLANATIONS 6x + 4x + 3x = 91
91
Type–1 13x = 91 x= 7
13
Questions Based on Angles,
Required varj
(Complementary, Supplementry,
= 6x – 3x = 3x = 3 × 7 = 21 cm
Angle Bisector etc.
7. (3) Semi-perimeter
1. (1) Supplementary angles = x rFkk 180° – x (let) 16 12 20
=
According to the question 2
180° – x – x = 44° 48
2x = 180° – 44° = 24 cm
2
= 136°
Triangle dk area
136
x= = 68° = s (s a )(s b )(s c)
2
2. (3) Supplementary angles dk sum 180° gksrk gSA = 24(24 16 )( 24 – 12)( 24 – 20 )
2x + 3x = 180° = 24 8 12 4
5x = 180° 2
= 96 cm
180
x= = 36° Aliter :
5
2x = 2 × 36° = 72° ,oa 3x = 3 × 36 = 108°
3. (1) ekuk angle = x°
20
According to the question, 16

1
90° – x = (180° – x)
4 12
360° – 4x = 180° – x
4x – x = 360° – 180° 1
Area of right angle triangle = × Base × Height
3x = 180° 2
x = 60°
4. (3) ekuk angle = x° 1
= × 12 × 16
2
According to the question,
180° – x = 3(90° – x) = 96 cm2
180° – x = 270° – 3x
3x – x = 270° – 180° 8. (1) A E B
2x = 90°
90 37°
x= = 45° 90°
2
5. (3) fdlh Hkh triangle osQ angles dk sum 180° gksrk gSA
x + 5° + 2x – 3° + 3x + 4° = 180°
6x + 6° = 180°
6x = 180° – 6° = 174°
174
x 29 C D
6
1 1 1 AEC + CED + DEB = 180°
6. (4) Triangle dh sides dk ratio = 4 : 6 : 8
37° + 90° + DEB = 180
DEB = 180° – 127° = 53°
1 1 1
= 24 : 24 : 24 EB || CD
4 6 8
BED = EDC = 53°
[4, 6, 8 L.C.M. = 24]

BLAM–307
T;kfefr
9. (1) A 4
k ABC + ABC = 180°
a° 5
m
O D 9 ABC = 180° × 5
b° l
B ABC = 100°
k || l || m
16. (1)
BOA = 45°
AOD = a° ,oa DOB = b°
a° + b° = AOB = 45°
10. (2) fdlh Hkh triangle osQ fdUgha nks sides dk sum rhljh
side ls cM+k gksrk gSA
(3, 5, 6) ,oa (2, 5, 6) OX, Ă AOC dk bisector gSA
11. (2) A + B = 65° AOC = 2 COX
C = 180° – 65° = 115° OY, Ă BOC dk bisector gSA
B + C = 140°
BOC = 2 COY
B = 140° – 115° = 25°
AOC + BOC = 2 COY + 2 COX = 180°
12. (4) A
2 ( COX + YOC) = 180°
75° XOY = 90°
AOX + XOY + BOY = 180°
45° x°
BOY = 180° – 90° – 20° = 70°
B C D 2nd Method
ACB = 180° – 75° – 45° = 60° linear pair osQ angles dk Sum 180° gksrk gSA
ACD = 180° – 60° = 120° = x BOC = 180° – 40° = 140°
x 120
60° dk % = 60° × = 24° 140
3 300 BOY =
2
70 (OY, BOC dk bisector gSA)
13. (1) 2 A=3 B=6 C
2 A 3 B 6 C
= = 17. (4)
6 6 6

A B C
= =
3 2 1
A: B: C=3:2:1

FG 2 IJ × 180° = 2
B= H1 2 3 K 6
× 180° = 60°

14. (2) A+ B+ C = 180° ...(i) BD = DC = AD


B BAD = 30°
A+ + C = 140° ...(ii)
2 ABD ls]
equation (i) – (ii), BAD = 30°
ABD = BAD = 30°
B
= 180° – 140° ADB = 180° – 2 × 30° = 120°
2
ADC = 180° – 120° = 60°
B AD = DC
= 40°
2 DAC = ACD = 60°
B = 80° ACB = 60°
15. (3) ABC + ACB + BAC = 180° 18. (2) BAC = 180° – BOC = 180° – 54° = 126°

1 3 A
ABC + ABC + ABC = 180° 19. (1) BPC = 90° – = 90° – 40° = 50°
5 5 2

BLAM–308
T;kfefr
20. (4) A A = 180° – 60 – 40° = 80°
80q
BAD = = 40°
2
B C D BAE = 180° – 60° – 90° = 30°
ACB = 80° DAE = 40° – 30° = 10°
ACD = 180° – 80° = 100° 2nd Method
We know that
80q
CAD = CDA =
= 40° B– C 60 – 40
2 10
DAE = =
BAC = 111° – 40° = 71° 2 2
ABC = 180° – 71° – 80° = 29° 25. (2)
21. (2) A

108°
B C D
A+ B+ C = 180°
ACD = ABC + BAC
‘B ‘C ‘A
A  90q –
108°= A 2 2 2
2
BOC esa
3 A ‘B ‘C
108  180q
2 BOC +
2 2
108 2 ‘A
A= 72 102° + 90° – = 180°
3 2
22. (2) A ‘A
= 102° + 90° – 180° = 12°
2
A = 24°
Aliter :
D E BAC
B C
BOC = 90° +
ABD = – B, ACE = – C 2
ABD + ACE = 2 – (B + C) = 2 – ( – A) = +A BAC
23. (3) 102° = 90° +
A 2
BAC
= 12°
2
BAC = 24°
D E
B C 26. (4) C
ABD = 120°
ABC = 180° – 120° = 60° 55°
ACE = 105°
D
ACB = 180° – 105° = 75°
BAC = 180° – 60° – 75° = 45°

24. (2)

A B

A = 90°, C = 55°,
B = 90° – 55° = 35°
ADB = 90°
BAD = 90° – 35° = 55°

BLAM–309
T;kfefr
A 30. (3) A
27. (2) BIC = 90° + = 90° + 30° = 120°
2

28. (4) A
B C
1 2
O
P O Q
B C
ABC + CBP = 180°
BAC = 80°
B + 2 1 = 180°
B + C = 180° – 80° = 100°
2 1 = 180° – B
B C
50 1
2 2 1 = 90° – B
2
OBC + OCB = 50°
iqu%] ACB + QCB = 180°
BOC = 180° – 50°= 130°
Aliter : 1
2 = 90° – C
2
BAC
BOC = 90° + BOC esa]
2
80 1 2 BOC = 180
= 90° +
2 1 1
90 B+ 90
= 90° + 40° = 130° 2 2
29. (2) C+ BOC = 180°
A
1 1
BOC = ( B C) = (180 A)
2 2
I 1
BOC = 90° – A
2

1
B C 60° = 90° – A
2
B+ C = 180° – 50° = 130° A = 60°
BIC esa] Aliter :
IBC + ICB + BIC = 180°
A
B C
+ BIC = 180°
2 2

1
BIC = 180° – ( B+ C)
2 B C
130q
= 180° –
2
60°
= 180° – 65° = 115°
Aliter : P O Q
BAC
BIC = 90° + 1
2
BOC = 90 – A
2
50
= 90° +
2 1
60° = 90° – A
= 90° + 25° 2
BIC = 115° A = 60°

BLAM–310
T;kfefr
31. (2) A 34. (3) A

D E
I

B O C B C
BDO = 90°
BOD = 30° 70
IBC = 35 ;
DBO = 60° 2

B= C = 60°  AB AC 50
ICB = 25 ;
2
EOC = 30°
BIC = 180° – 35° – 25° = 180° – 60° = 120°
AO is bisector of BC
Aliter :
DOE = 120°
BAC
AOE = AOD = 60° BIC = 90
2
60
= 90  BAC = 180° – (70° + 50°)
32. (2) 2
= 90° + 30° = 60°
= 120°
BIC = 120°
35. (4)

1
AB = BC
2

AB 1 AC = BC
BC 2 ABC isosceles triangle gSA
ABC = CAB = 50°
1
sin C = ACB = 180° – 100° = 80°
2
BC = CD = AC
C = 30°
ACD isosceles triangle gSA
33. (2) A CAD = CDA
ACB = 2 CAD
80q
CAD = = 40°
2
BAD = 50° + 40° = 90°
120°
36. (4) A
B C D
CAB = 2 ABC
ACB + ACD = 180°
ACB + 120° = 180°
ACB = 180° – 120° = 60° 35°
A + B = 180° – 60° = 120° B D C
2 B + B = 120°
3 B = 120° AB = AC
ABC = ACB = 35°
120
B= = 40° ADB = 90°
3 BAD = 55°,

BLAM–311
T;kfefr

D
40. (4) A
37. (2)

D E

B C

BAC = 40°,
30° ABC = 65°
B C ACB = 180° – 40° – 65° = 75°
DE BC
AB = AC = AD
AED = ACB = 75°
ABC = ACB = 30°
BAC = 180° – 60° = 120° CED = 180° – 75° = 105°
DAC = 180° – 120° = 60° 41. (1) G
ADC + ACD = 120°
120 A
ACD = 60
2
BCD = ACB + ACD
= 30° + 60° = 90°

38. (1) A

D B C F
x
E
45° CBE = 130°
B x C
ABC = 180° – 130° = 50°
2x2 = AC2 ACF = 130°
2 ACB = 180° – 130° = 50°
2x2 = 4 2 e j 32 BAC = 180° – 50° – 50° = 80°
2
x = 16 GAB = 180° – 80° = 100°
x = 4 unit 42. (3)

BD = AB2 AD 2

= 16 8 = 2 2 unit

39. (2) A
AB BD
AC DC
120° AD, A dk bisector gSA
B C D 1 180 70 50 60
BAD = ( BAC) 30
2 2 2
ACB = 180° – 120°
43. (1)
= 60°
AB = AC
ABC = ACB
= 60°
BAC = 60°

BLAM–312
T;kfefr
ADB = 20°
AB = AC ; CD = CA 47. (4) P

ABC = ?
40°
CAD = 20°
ACD = 180° – 40° = 140° A B
ACB = 180° – 140° = 40°
ABC = ACB = 40°
44. (1) Q
C R

AC= QC
QAC = CQA = x
CR = CB
CBR = CRB = y
PQR esa]
A+ B C
x+ y + 40° = 180
1
2
b A+ B C g 2
x+ y = 140° ...(i)
iqu%
B C A ACQ + ACB + BCR = 180°
2 2 2 2 180° – 2x+ ACB+180° – 2y = 180°
OBC esa] ACB = 2 (x + y) – 180°
OBC + OCB + BOC = = 2 × 140° – 180° = 100°
B C
BOC = 48. (3)
2 2

BOC =
FG AIJ
=
A
H2 2 K 2 2

A
45. (4)

G
BAD = 30°
ABD = 60°
B C
AD
AG = BG = GC tan ACD DC
tan D BE DE
AGB = AGC = BGC = 120°
BD
A
AD BD BD
= 6
DC DE DC
BD
6 6
O DC
46. (3)
BD = DC
15° ACB = 60°
B D C
blfy,, ABC ,d equilateral triangle gSA
BO, B dk internal bisector gSA 49. (1) 2x + 3x + 4x = 180°
ODB = 90°; BOD = 15° 9x = 180°
OBD = 180° – 90° – 15° = 75° x = 20°
ABC = 2 × 75° = 150° Triangle osQ angles gksaxs 40°, 60° ,oa 80°

BLAM–313
T;kfefr
A B C
D + + BIC = 180°
2 2
40°
A
90° – + BIC = 180°
2
60° 80°
B C
E A
BIC = 180° – 90° +
2
AB CD
DCE = ABC = 60° A
= 90° +
ACB + ACD + DCE = 180° 2
ACD = 180° – 140° = 40° x = 90°
Aliter :
50. (1)
ge tkurs gSa
BAC
BIC = 90° +
2

BAC
BIC = x +
ACD = 180° – ACB 2
= 180° – 72° = 108° x = 90°
72q
CAD = ADC = = 36° 53. (3) A
2
ABC = 180° – 109° – 36° = 35°
D F
51. (2) A O

B C B E C

AB2 + =AC2 BC2


BAC = 90°
BOC = 180° – A BOC + BAC = 180°
AB2 + AC2 = 2AB2
54. (3) EAB = ECD = AEF = 90°
AB2 = AC2
AB = AC CED + DEF = 90°
ABC = ACB = 45° C + 61° = 90°
C = 29°
52. (3) A Similarly, b = 180° – 61° = 119° = a
Thus
a = b = 119° and c = 29°

55. (1) A

B C

ABC esa]
A + B + C = 180°
B + C = 180° – A
1 A B D C
( B+ C) = 90° –
2 2
ABC esa]
BIC esa]
AB = AC

BLAM–314
T;kfefr
ABC = ACB = x° 59. (1) Interior angle gksxk
BAC = 4x
(2n – 4) 90
4x ° + x ° + x ° = 180° =
n
6x ° = 180°
(2 8 – 4) 12 90
x = 30° = × 90° = = 135°
8 8
BAC = 4 × 30° = 120°
1 FG 2n 4 IJ 90
BAD =
2
BAC = 60° 60. (2) Each interior angle = H n K
56. (3) A b2n 4g 90
105
n
(2n – 4) × 6 = 7n
12n – 24 = 7n
5n = 24
24
B C n=
5
possible ugha gSA

AC 2
61. (3) Polygon dh number of sides = n
AB × BC =
2 360
(n) = = 18
AC2 = 2 AB × BC 20
AB2 + BC2 = 2 AB × BC lHkh interior angles dk sum = (2n – 4) × 90°
(AB – BC)2 = 0
= (2 × 18 – 4) × 90° = 32 × 90° = 2880°
AB = BC
62. (4) D C
BAC = ACB = 45°
57. (3)
O

A B
A + B + C + D = 360°
A + B + 70° + 130° = 360°
A + B = 360° – 70° – 130° = 160°
AOB esa]
B= C
OAB + OBA + AOB = 180°
A = 2( B + C)
A=4 C A B
AOB = 180°
4 C+ C+ C = 180° 2 2
6 C = 180° 1
C = 30° 2
a A f
B + AOB = 180°
58. (4) A
1
160 + AOB = 180°
2
D AOB = 180° – 80° = 100°
2nd Method
B C We know that
E C D 130 70
BCD = DCA = 30° AOB 100
`2 2
DCE = 180°
63. (2) A D
ACE = 180° – 30° = 150°
AC = CE
30°
CAE = CEA = = 15° B C
2

BLAM–315
T;kfefr
ABC + CDA = 180°
CDA = 180° – 70° = 110° 67. (3)
BCD = 180° – 110° = 70°

64. (4) E

A D

AOE = 150°, DAO = 51°


B C EOB = 180° – 150° = 30°
OE = OB
150q
OEB = OBE = = 75°
ABC + CDA = 180° 2
CDA = 180° – 72° = 108° CBE = 180° – 75° = 105°
AD || BC
68. (1)
BCD = ADE
D
= ABC = 72° C
2nd Method
We know that fdlh Cyclic Quadrilateral esa ,d Hkqtk A B
P
c<+kus ls cuk ckâkdks.k vfHkHkq[k var% dks.k osQ cjkcj gksrk gSA
ADE = B = 72° ADC = 70°
ABC = 180° – 70° = 110°
65. (3) PBC = 70°
D C BCD = 180° – 60° = 120°
PCB = 60°
PBC + PCB = 70° + 60° = 130°
A E
B 69. (4)
D C
ABC + ADC = 180°
CBE = 50°
ABC = 180° – 50° = 130° O
ADC = 180° – 130° = 50°
A B

66. (3) A D
90° Concyclic quadrilateral osQ opposite angles dk sum
= 180°
E A+ C= B+ D = 180°
B C 70. (2) A

B+ D = 180° vkSj A+ C = 180°


BAC = BCA
B D
DAC = DCA O

DAB = DCB = 90°


DAC = T
ADE = 90° – T = CDE C

ABC = 180° – 2(90° – T)


Concyclic quadrilateral osQ opposite angles dk sum
= 180° – 180° + 2T = 2T = 180°

BLAM–316
T;kfefr
A + C = 180° 4x + 5y = 180° ...(i) APD = 180° – 64° = 116°
B + D = 180° 7x + y = 180° ...(ii) ADB =180° – 116° – 28°
By equation (ii) × 5 – (i), = 180° – 144° = 36°
35x + 5y = 900°
4x + 5y = 180° 72. (1) A
– – –
R Q
31x = 720
720
x=
31
Equation (ii) ls,
720
7x + y = 180° 7 y = 180° B C
31
5040q 5580q5040q 540q P
y = 180q – =
31 31 31

720 540 BQP = BAP


x:y= : =4:3
31 31 A
BQP =
2
Aliter :
BQR = BCR
D C 1
y 5y BQR = C
2

1
BQP + BQR = ( A+ C)
2
4x 7x
A B 1
PQR = (180°– B) [ A+ B+ C=
2
180°]

Cyclic quadrilaterals ds opposite angles dk sum = 90° –


B
180° gksrk gSA 2
A+ C= B+ D = 180°
4x + 5y = 7x + y 73. (4)
7x – 4x = 5y – y A D
3x = 4y

x 4
y 3
P

B C
71. (2) A D

P APB = 110° = CPD


APD = 180° – 110° = 70° = BPC
B C PCB = 180° – 70° – 30° = 80°
same arc osQ }kjk circle osQ circumference ij cuk, x,
APB = 64° angles equal gksrs gSaA
CBD = 28°
ACB ;k PCB = ADB = 80°
CBD = CAD = 28°

BLAM–317
T;kfefr
74. (3) A 78. (3)
D

O
O C
A

B C B
P

BOC = 2 BAC AOC = 130°


OB = OC
1
OBC = OCB ADC = 130 65
2
BOC
OBC = 90 = 90° – BAC PBC = ADC = 65°
2
BAC + OBC = 90° 79. (3)
A D
75. (4)
P
P

A Q B O

PQA = PQB = 90° C B


AQB = PQA + PQB = 90° + 90° = 180°
BOC = 2 BAC
76. (1) AOD = 2 DCA
C BOC + AOD = 2 ( BAC + DCA) = 2 BPC
O 2 BPC = 20° + 30° = 50°
BPC = 25°
B

A 80. (4)

OA = OB = OC
D
BAO = ABO = 30° O
AOB = 180° – 2 × 30° = 120°
iqu% OB = OC A 20°
BCO = CBO = 40° B
C
COB = 180° – 2 × 40° = 100°
AOC = AOB + COB = 120° + 100° = 220°
BC = OB = OA = OD = Radius of circle
C OB = OC
77. (1) BOC = OCB = 20°
OBA = BOC + OCB
O = 20° + 20° = 40°
(external angle of OBC)
OA = OB OAB = OBA = 40°
A B AOB = 180° – 40° – 40° = 100°
DOA = 180° – ( AOB + BOC)
AO = OB = AB = 180° – (100+20°)
AOB = 60° = 180° –120°
ACB = 30° = 60°

BLAM–318
T;kfefr
81. (2) B OA = AB = OB
AOB = 60°
ADB = 30°
fdlh Hkh arc osQ }kjk circle osQ centre ij cuk;k x;k angle
90°
A O circumference ij cus angle dk nksxu q k (twice) gksrk gSA
70° ADBC cyclic quadrilateral gSA
ADB + ACB = 180°
30° + ACB = 180°
C ACB = 180° – 30° = 150°
AOB = 90° ; OA = OB = r
85. (3) A
BAO = ABO = 45°
AOC = 70° ; OA = OC = r
P O
70
OAC = OCA = = 35°
2
B
BAC = 45° + 35° = 80°
Right angled triangle OAP and OPB esa,
82. (4)
AP = PB , OA = OB
OP = OP
OAP OPB
AOP = POB ,oa APO = OPB
AOP ls]
APO = 180° – 90° – 60° = 30°
APB = 2 × 30 = 60°

BEC = 130° 86. (3) T


DEC = 180° – 130° = 50°
O
EDC = 180° – 50° – 20° = 110° 75°
A B
BAC = EDC = 110° 45°

(Angles same arc ij cus gSaA)


P C
83. (3) C
If a line touches a circle and from the point of
A B contact a chord is drawn, the angles which this
chord makes with the given line are equal re-
spectively to the angles formed in the correspond-
D ing alternate segments.
;fn dksbZ js[kk fdlh o`Ùk dks Li'kZ djs vkSj ;fn Li'kZ fcUnq ls
ABD esa]
,d thok [khaph tk, rks og thok nh gqbZ js[kk ds lkFk tks dks.k
AD = BD = AB = radius
cukrh gS os Øe'k% rnuq:ih ,dkarj o`Ùk [k.Mksa esa cus dks.kksa ds
ACB esa] cjkcj gksrs gSaA
AC = CB = AB = radius ACB = BAT = 75°
DBC = 60° + 60° = 120° ABC = 180° – 45° – 75° = 60°
84. (3) D
87. (3)
Q C R

O
A B
P

A B
C

BLAM–319
T;kfefr
PCR ,oa RBP esa] 91. (2) A
PC = PB (radii)
RC = RB O P
PR common gS
PCR RBP B
CPR = RPB AOB = 110°
blh izdkj] CPQ = QPA OAP = 90°
QPR = 90° D;ksafd APB = 180° OBP = 90°
APB + AOB + OAP + OBP = 360°
88. (1) APB + 110° + 90° + 90° = 360°
APB = 180° – 110° = 70°
O 92. (2) P

A B O

P Q R
OAP = OBP = 90°
QPR = 70°
AOB + APB = 180°
ekuk PQR = 2x rFkk PRQ = 2y
5 APB + APB = 180°
6 APB = 180° APB = 30° PQR esa,
QPR + PQR + PRQ = 180°
89. (4) B = 70° + 2x + 2y = 180°
C 110q
x+y= = 55°
40° 2
T
A 44° rFkk QOR esa]
CAT = 44° OQR + ORQ + QOR= 180°
BTA = 40° x + y + QOR = 180°
ACT = 180° – 44° – 40° = 96° 55° + QOR = 180°
CAT = CBA = 44° QOR = 180° – 55°
BCA = 180° – 96° = 84° QOR = 125°
BAC = 180° – 84° – 44° = 52° Aliter :
BC osQ }kjk centre ij cuk;k x;k angle
= 2 × 52° = 104° P
90. (4) A
70°

O P
D

B
Q R
OA = OB = r
OP = 2r
1
AP = PB = 4r 2 r2 3r QOR = 90° +
2
P

OA r 1
sin APO = 1
OP 2r 2 = 90 + × 70°
2
APO = 30°
APB = 60° = 125°

BLAM–320
T;kfefr
93. (1) PQRS ,d cyclic quadrilateral gSA 95. (1) PQR esa,
SPR = SQR
P
(same arc ij cus angles equal gksrs gSa)
Y
P X 105°
Q
80°
40° T 75° 60°
Q R

R PQR + PRQ + QPR = 180°


= QPR = 180° – (75° + 60°)
S
QPR = 45°

SQR = 40°
rFkk PYX + XYR = 180°
PQR = 80° + 40° PYX = 180° – 105°
= 75°
= 120°
PQR + PSR = 180° PXY esa]
PXY = 180° – (75° + 45°)
LMCyclic quadrilateralis ds opposite OP = 60°
Nangles dk sum 180q gksrk gS Q PXY rFkk PQR esa
120° + PSR = 180° PYX = PQR (75° each)
‘PSR 60q PXY = QRP = 60°
PXY ~ PRQ
94. (2) PQ circle dk diameter gSA
PY PX YX
PRQ = 90°
PQ PR QR

Q P
P

75° 60° 75° 60°


R
Y X Q R

18 9
PQ 14
S
PQ = 28 cm
P 96. (3) ;gk¡, PQ = PS = SR
P
rFkk
PQR + RSP = 180°
[Cyclic quadrilateral ds
opposite angles dk sum 180° gksrk gS]
PQR + 105° = 180°
PQR = 75° Q S R
PQR esa]
QPS = 40°
PQR + RQP + QPR = 180º
PQ = PS
90° + 75° + QPR = 180°
PQS = PSQ
QPR
fdlh Hkh triangle esa equal angles ds opposite okyh
= 180° – 165°
sides equal gksrh gSA
QPR = 15°

BLAM–321
T;kfefr
PQS esa 98. (2) R
QPS + PQS + PSR = 180°
40° + 2 PSQ = 180°
2 PSQ = 140° Q
PSQ = 70°
PSR = 180° – 70° = 110° O
P
rFkk PS = SR
PRS = SPR
PSR esa] QOR = 50°
PSR + SRP + SPR = 180° ORQ esa]
110° + 2 SPR = 180° OQ = OR = radius
2 SPR = 70°
ORQ = OQR
SPR = 35°
QPR = QPS + SPR fdlh Hkh triangle ds equal angles dh opposite sides Hkh
= 40° + 35° equal gksrh gSA
QPR = 75°
97. (3) ;gk¡ AD = 12 cm ORQ esa]
AE = 8cm ORQ + OQR + QOR = 180°
EC = 14 cm 2 ORQ + 50° = 180°
BD = ? 2 ORQ = 130°

A ORQ = 65°
rFkk, PRQ = 90°
70° E PRO = 90° – 65° = 25°
rFkk OR = OP
D
RPO = PRO
70° RPO = 25°
B C 99. (1) Given the figure,
ADE rFkk ABC esa 75° + b + a = 180° (linear pair)
DAE = BAC (common) 75° + b + 4b = 180° [a = 4b (vertically opp. Angles)]
AED = ABC (70° each) 5b = 105°
ADE ~ ACB (AA ds }kjk)
105
b=
A A 5
b = 21
a = 4 × 21° = 84°
again,
70° 70° 75° + b = 2c
B C E D
96
75° + 21° = 2c C= 48
AB AC 2
=
AE AD
Thus,
AB 8 +14 a = 84°, b = 21°, C = 48°
= [  AC = AE + EC]
8 12
100. (4)  PQ||RS ...(1)
22 u 8 PQR = QRS (Alt. Angles)
AB =
12 and
44 SRL + RLM = 180°
AB =
3 RS||LM ...(2)
44 8 from eqn (1) & (2)
vc, BD =  12 =
3 3 the angles between PQ and LM = 180°

BLAM–322
T;kfefr
101. (2) COD
CBP = =15.5°
D E 2
X P
In CBP,
125° 80°

C APB = 180° – 90° – 15.5° = 74.5


80°
Y 30°
30°
Q B C 100 50
A B 108. (1) AMD = = = 75°
2 2
x
Z R 109. (1) A
F

let AFB = x°

b
MBQ = ABF = 30° (vertically app. Angles)

=
6
DCE = ACB = 80° (vertically app. Angles)

=
c
Now,
125° + DBA = 180° (linear pair)
DBA = 55° B a=6 C
In FBC,
80° + (30° + 55°) + x = 180° c2 a 2 b2
x = 180° – 165° = 15° cos B
2ca
102. (1) GF||HI
2
CHI = FGC = 80°
103. (2) PEF = EGH = 80° (corresponding angles) =
e6 3 j b6g b12g 2 2

QGH = 180° – 80° = 100° 2 6 3 6


In QGH,
108 36 144
x° + 100° = 120° =
72 3
x = 20°
104. (4) In cyclic Quadrilateral 0
B + D = 180° = 0 cos 90
72 3
D = 180° – 72° = 108°
AD||BC = B 90
BCD = 180° – 108° = 72°
110. (2) A
QPR
105. (4) QSR = 90° –
2

60
= 90 60
121°
2 104°
50° 76°
106. (3) BAC = BDC = 30°
P B C Q
In BPC,
x = 180° – (100° + 30°) = 50° ABC = 180° – 121° = 59°
107. (2) ACB = 180° – 104° = 76°
BAC = 180° – (59° + 76°)
A = 180° – 135° = 45°
C
111. (4)
A

o 31°
P
12 12
G
D
B

BC AP B D C

BCP = 90° K 12

BLAM–323
T;kfefr
ABD ls]
3
AD = 12 = 6 3
2 AD
tan 30° =
BD
1
Now, GD = 6 3 2 3 cm
3 1 AD
=
3 x
112. (4) A
x
AD = ...(i)
3
ACD ls]
AD
tan 60° =
CD
68° 52°
AD
B D C 3 =
12 3 – x
Given
AD = 3 (12 3 – x)
AB BD
AC DC = 36 – 3x ...(ii)

AD, A dk Bisector gksxk x


= 36 – 3x
A = 180° – (68° + 52°) = 60° 3

60 x = 36 3 – 3x
BAD = 30
2
4x = 36 3
Type-II 36 3
x= =9 3
Questions Based on Length of sides 4
of a Triangle
x 9 3
AD = = = 9 cm
1. (3) AD, A dk bisector gS 3 3
Aliter :

AB BD 3 30° 60°
=3:4
AC DC 4
3 1
4
2. (4) A
4 12 3

12 3
3 3 9.
4
Aliter - 2 :
d = h (cotT1 + cotT2)
30° D 60°
B C BC = AD (cot30° + cot 60°)
12 3 cm
F 3
1I
BD = x cm (let)
12 3 AD GH J
3K

e
CD = 12 3 – x j cm 12 3 AD u
4
3
ADB = ADC = 90°
AD = 9 cm.

BLAM–324
T;kfefr
3. (4) A AD CE
=
BD BE
BD BE
P Q =
AD CE

10 – 4 BE
B C 4 CE

APQ ~ ABC BE 3
=
CE 2
AP AQ PQ
AB AC BC 6. (4) A
AB 3
vc AB 3
AB – AP 1
PB 1
D E
AB – AP 1
AB 3

AP 1
1 B C
AB 3

AP 1 2 PQ AD AE 1
1 = =
AB 3 3 BC AB AC 3
PQ : BC = 2 : 3
DE 1
4. (2) A BC 3

15
DE = = 5 cm
R Q 3

7. (3) A

B C
P
P Q
AP < AB
BQ < BC
CR < AC B C
(AP + BQ + CR) < (AB + BC + AC)
PQ || BC
5. (4) A
PB QC AP
= =
D AP AQ AQ
AP2 = PB. AQ
= 4 × 9 = 36
AP = 6 unit
B C 8. (2) A
E

DE || AC
P Q
ABC ~ DBE
AB BC B C
=
BD BE  PQ BC
AB BC AP AQ 1
–1= –1
BD BE PB QC 2

BLAM–325
T;kfefr
QC 2 A
11. (2)
AQ 1

QC AQ 3 X
AQ 1 P D

AC = 3AQ = 9 cm Y
9. (2) DE BC
B C
Q
A
1
PX BD ,oa PX = BD
2
D E
1
QY BD ,oa QY = BD
2
PX : QY = 1 : 1
B C
12. (3) A
BDEC 1
?
'ADE 1 P Q
R
BDEC
Ÿ +1=1+1
'ADE
B C
2
ABC AB PR 1 2 1
Ÿ 2
ADE AD 2 RQ 2 RQ 2

AB RQ = 4 cm
Ÿ 2
AD PQ = PR + RQ
= 2 + 4 = 6 cm
AB
Ÿ 1 2 1 fdlh Hkh triangle ds nks side ds mid point dks join djus
AD
okyh line third side ds parallel rFkk vk/h gksrh gSA
BD
Ÿ 2 1 BC = 2PQ
AD
=2×6
AD 1 = 12 cm
Ÿ BD = 2 1
2 1
13. (3) C
5

10. (2) D, side BC dk mid-point gSA


cm

Point O centroid gS rFkk ;g AD dks 2 : 1 ratio esa


divide djrk gSA 90° E
D m
A 18c

O A B
B C
D DEC = 90°
DE = 18 cm
AO 2 CE = 5 cm
OD 1
DE 18
tan C = 3.6
10 2 CE 5
OD 1 tan ABC = 3.6
OD = 5 cm C= B

BLAM–326
T;kfefr
AC = AB
16. (3) A
C + D = 90°
2 C + 2 D = 180°
C + A + B = 180° D
2 C + A = 180°
A=2 D
B C
AC 2CD
BC CE ;gk¡] 62 + 82 = 102
AC × CE = 2CD × BC ABC right angled triangle gS
AC 2BC AC = 10 cm
CD CE D, BC dk mid-point gS
AC : CD = 2BC : CE D triangle dh vertices A, B, C ls equal distance ij
14. (1) Equilateral triangle dh side = x unit gSA
AD = BD = CD = 5 cm
3
4
e
(x 2)2 x2 = 3 +j 3
17. (4) C
3
(4x 4) 3 3
4
D
3x 3 3 3 = 3x 3
x= 3 4
6
15. (3) A
A B

BAC = 90°

AB = AD 2 BD 2 = 36 16 = 52 cm
B C
D ABD rFkk ABC similar gSA
AB BD
;fn] AB = x cm
BC AB
x AB2 = BC × BD
BD = cm
2 52 = BC × 4
ABD ls] 52
BC = = 13 cm
AB = BD2 + AD2
2
4

x2 2 18. (2) A
x2 = e6 3 j
10

4
cm

N
6 cm

x2
x2 – 36 3
4
C
B
3x 2
36 3
4 BC = 102 62 100 36
x2 = 36 × 4
x = 6 × 2 = 12 cm = 64 8 cm
Equilateral triangle dk perimeter 1
ar ( ABC) = BC AB
= 3 × 12 = 36 cm 2
2nd Methods
1
= 8 6 24 sq. cm
P= 2 3 h 2 3 6 3 36m 2

BLAM–327
T;kfefr
iqu%]
55 1 20
1 =
AC BN 24 3 4 3
2
60
1 = 20 2 5 cm
10 BN 24 3
2
2nd Methods
24 we know that
BN =
5 In right angled triangle Let CM = x
4 (CM2 + BC2) = 5BC 2
NC = BC2 BN 2

576 32
F F 3 5 I IJ 2

= 64 cm 4 x2GG GH 2 JK JK 5 52
25 5 H
32 50 32 18
AN = 10 x2
45 125
5 5 5
4 4
18 32 x2 = 20
AN : NC = : = 9 : 16
5 5
x 2 5
2nd Method
BC = 102 6 2 8 cm 20. (2) A
we know that in right-angled triangle

AN FG AB IJ FG 6 IJ
2 2
9 B C
CN H BC K H 8 K 16

19. (1) A E
D
M 90°
L BC || DE
ABC = ADE = 90°, ACB = AED
ABC ~ ADE
B C
AB BC
ABC ls]
AD DE
AB2 + AC2 = 25 ...(i)
ABL ls] 12 10
18 DE
F3 5I 2
18 10
= G
AB2 + AL2
H 2 JK DE =
12
15

AC 2 45 21. (1) A
AB2 +
4 4
4AB2 + AC2 = 45
3AB2 + 25 = 45
20
AB2 =
3
B D C
4 20 135 80 55
AC2 = 45 – = AD, A dk internal bisector gSA
3 3 3
AB BD 5 5
2 AB 2 AC DC
=
7.5 – 2
=
2.5
=2:1
CM = AC2 AM 2 = AC
4

BLAM–328
T;kfefr
25. (3) A
22. (1) A

D E
90° D
B C

B C
AB 2
A= B= C = 60° AD 1
ADE ~ ABC
BC = AB = AC = 13 cm
AB BC 2
BD = 2 cm
AD DE 1
ABD esa]
DE 1
AB2 = BD2 + AD2 =1:2
BC 2
2
26. (4)
e 13 j = (2)2 + AD2 (By Pythagoras theorem)

AD2 = 13 – 4 = 9 A
AD = 9 = 3 cm
4
23. (1) A a
D E
5
N
B C
O

B D C DE || BC
ADE = ABC
AD = 27 cm
AED = ACB
Centroid = O
AA – similarly osQ }kjk
1 1 ADE ~ ABC
OD = AD = 27 = 9 cm
3 3 AB BC 4
DB
= also =
ND = 12 cm AD DE AD 5
ON = DN – OD = 12 – 9 = 3 cm DB 4
+1= +1
24. (3) AD 5
A
DB + AD 4+5
=
AD 5
D E
AB 9 BC
= =
AD 5 DE
B C
DE 5
= =5:9
AD AB BC 9
AE AC 2nd Method.
ADE ~ ABC ADE ~ ABC
AD AE DE 5
1 1
DE = BC = 12 3 cm AB AC BC 9
4 4

BLAM–329
T;kfefr
27. (3) B ADE ,oa ABC esa]
ABC = ADE
D
A= A
ABC ADE
A C AB BC AC
BD = 3 cm AD DE AE
CD = 4 cm AE EB AC
ABC ls] AD AE
AB2 + AC2 = 72
3 2 AC
AB2 + AC2 = 49 ...(i)
2 3
ABD ls]
5 AC
AB = AD2 + 32 = AD2 + 9
2
...(ii)
2 3
ADC ls]
2AC = 5 × 3
AC = AD2 + 16
2
...(iii)
15
equations (ii) rFkk (iii) dks add djus ij AC =
2
AB2 + AC2 = AD2 + 9 + AD2 + 16
49 = 2AD2 + 25 15
DC = AC – AD = 2
2AD2 = 49 – 25 = 24 2
AD2 = 12
15 4 11
= = 5.5 cm
AD = 12 2 2

= 2 3 cm
30. (3) A
2nd Method
In BAC
AD2 = BD × DC
AD = 3 4 2 3 cm. F E
28. (4)

A
B C
F
E
D FE||BC
AFE ~ ABC
4(AB2 + AC2) = 5BC2
B C
4(192 + 222) = 5BC2
DE BC BC2 = 4 × 169
rFkk E, AC dk mid-point gSA BC = 2 × 13 = 26 cm
1 1
AE = AC = × 12 = 6 cm 31. (2) A
2 2

29. (4) A F E

E D B D C

ar BDOF
= 2 × 15 = 30 cm2
B C

BLAM–330
T;kfefr
32. (2) A 2b
BC = 2x =
3

1
D E ar ( ) = BC AD
2

1 2b
a= b
2 3
B C

DBC rFkk EBC same base rFkk same parallels ds b2


3
chp cus gSA a
ar DBC = ar BEC Aliter :
ar ABC– DBC Let, leckgq f=kHkqt dh Hkqtk x
= ar ABC – ar BEC
According to question,
ar ADC
= ar ABE 3 2
= 36 cm2 leckgq dk Area = x a
4
33. (2) A
3
leckgq dk Height = x b
4

F 3 xI 2

b2
GH 2 JK 3 4
a 4 = 3
B D C 3 2 3
x
4
3
× (side)2 = 9 3
4 A
35. (3)
(side)2 = 9 × 4 = 36
side = 36 6m
BD = 3 m
AD = AB 2 BD 2 62 32
B D C
= 36 9 27 3 3 m
AB = AC = x unit
34. (3) A
BD = DC = 1 unit
AD = AB2 – BD 2

= x2 –1

C 1
B × BC × AD = 4
D
2
AD = b
1
BD = DC = x × 2 × x2 –1
2
AD =4
tan60° =
BD
x2 –1 = 4
b 2
3 x – 1 = 16
x
x2 = 17
b x = 17 unit
x
3

BLAM–331
T;kfefr
36. (2) A 38. (3) A

x
P Q
D

B x C
B C
AB = BC = x cm
PQ || BC
AC = 2x cm APQ = ABC = 60°
AQP = ACB = 60°
2x 2x 2p
3
x 2 e 2 j 2p ar APQ =
4
bPQg 2

2p 3 25 3
x=
2 2
=
4
b5g 2
4
sq. cm

A D
2p 2 2
=
2 2 2 2

=
2p 2 e 2 j = p 2 e 2 j B C E F
4 2
ar ABC AB 2 100 25
1 39. (2) = 25 : 16
ABC dk area = AB BC ar DEF DE 2 64 16
2
40. (3) A D
1 2 1 2 2
= x
2 2
p 2 e 2 j
1 2
=
2
.p 4 e 2 4 2 j
B C E F
ABC ~ DEF
= 3 e j
2 2 p sq. cm 2

ar ABC 32 54 9
37. (2) ' ACD ,oa 'ABC esa] Ÿ
ar DEF 42 ar DEF 16
C
16 54
ar DEF = = 96 sq. cm
9
D
A D

A B
‘CDA = ‘CAB = 90° B C E F
‘C, common gSA ar ABC AB 2

? 'ACD ~ 'ABC 41. (2)


ar DEF DE 2
ar ACD AC 2
? 20 25
ar ABC BC 2
45 DE 2
10 92 45 25 225
Ÿ DE2 = =
40 BC2 20 4

Ÿ BC2 4 92 15
DE = = 7.5 cm
? BC = 2 × 9 = 18 cm 2

BLAM–332
T;kfefr
42. (3) A D 4 21
= 1 = = sinR
25 5

PQ 2 PR 2 QR 2
cos P =
B C E F 2 PQ PR

ar ABC AB 2 102 102 82


=
ar DEF DE 2 2 10 10

360 8u8 200 64


=
250 DE 2 200

8 8 250 82 5 2 =
136
=
68
=
17
DE2 = = 200 25
360 62 100

8 5 20 sin P = 1  cos 2P
DE =
6 3
2 289 336
= 1 =
=6 cm 625 625
3

43. (2) A 4 21
=
25
PQR dk area
E F
1
=
4
× QR × 4PQ 2 QR 2

B D C 1
= × 8 × 4 100 64
4
1
ar DEF = ar ABC
4 =2 400 64 = 2 336
1 = 8 21 sq. cm.
= 24 6 sq. unit
4

44. (3)

PQ = PR = 10 cm; QR = 8 cm.
DQC dk area
PQR = PRQ 1
= DQ × QC sinQ
2
PQ 2 QR 2 PR 2
cosQ =
2 PQ QR 1 21
= ×2×5×
2 5
102 8 2 102
= =
2 10 8 21 sq. cm.

8 8 2 1
= = BCR dk area = 2 × BR × CR sinR
2 10 8 5

sin Q = 1 – cos2Q 1 21
= ×4×3×
2 5

BLAM–333
T;kfefr
6 21 Area of STU 36 1
= sq. cm. ...(ii)
5 Area of PTR 27 12 9
Area of APB Multiply equation (i) × equation (ii),

1 Area of PQU
= × AP × PB × sin P =4:9
2 Area of PTR

1 4 21 X
= ×5×6× 46. (1)
2 25

12 21
= sq. cm G
5
quadrilateral ABCD dk area
Y M Z
F
= G8
6 21 12 21 I
H 21 21
5 5 JK sq. cm. YZ = x; XY = z; XZ = y
1
XM = e
2 z2 j
y2 – x 2
F 35 21 6 21 12 21 I
2
= G JK sq. cm.
H 5
=
1
2 (11)2 (7)2 – (14)2
2
17 21 1
= sq. cm.
5 = 2(121 49) – 196
2

45. (2) 1
= 340 – 196
2

1 1
= 144 = 2 × 12 = 6 cm.
2

1
GM = XM
3

FG 1 6IJ
1
= H3 K cm. = 2 cm.
Area of PUQ = × 18 × 24 × sin PUQ
2 47. (4) S rFkk T, PQ rFkk PR ds eè; fcanq gSaA
= 216 sin PUQ P
= 216 sin (180 – SUT)
= 216 sin SUT S T
1
Area of SUT = × 9 × 12 × sin SUT Q R
2
= 54 sin SUT 1
ST QR rFkk ST = 2 QR
Area of PUQ 216 4
...(i) As ST QR
Area of SUT 54 1
PQR ~ PST
1
Area of STU = × 8 × 9 × sin STU rhuksa statements correct gS
2
48. (2) ;gk¡,
= 36 sin STU
A
1
Area of PTR = × 27 × 24 × sin PTR D
2
= 27 × 12 × sin (180 – STU)
B C
= 27 × 12 × sin STU

BLAM–334
T;kfefr
ABC = 90° rFkk BD AC, AC = AF FG GC
BAD rFkk CAB esa = AF + AF + AF
BAD = CAB (common) = 3AF
BDA = CBA (90° each)
1
BAD ~ CAB AF = AC
3
CBD rFkk CAB esa
51. (1) given
CDB = CBA (90° each)
ABCD is a parallelogram and
DCB = ACB (common)
P and R respectively
CBD ~ CAB
DC and BC midpoint gSA
fliZQ statement II rFkk III correct gSA
So AC and BD intersect to each other at point O
49. (2) ;gk¡,
1
A OC = AC
2

P Q A B

B C R
O

AQ = 4 2 cm

QC = 6 2 cm Q
AB = 20 cm D C
P
PQ, BC osQ parallel gS
APQ ~ ABC CBD esa P rFkk R respectively DC and BC mid-
vc, point gS]
AP AQ PR||BD, PQ||DQ rFkk RQ||BO
AB AC Again,
OCD esa PQ||OD
AP 4 2
20
=
10 2
So, Q, OC dk eè;&fcUnq gSA
[midpoint]
20 u 4
AP = 1 1 1 1
10 then CQ = OC AC AC
2 2 2 4
AP = 8 cm PB = 20 – 9 = 12 cm
52. (2) We have, xy||AC
50. (3) given ABC esa AD median and AD dk Midpoint
and, Area ( Bxy) = Area ( xYCA)
E gSA fcUnq D rd js[kk\ l ||BF [khph
Area ( ABC = 2 Area ( Bxy) ...(i)
tks AC dks fcUnq G ij dkVrh gSA Now
ADG esa BF||l ;k EF||DG rFkk E AD dk midpoint xy||AC and BA is a transversal
gSA Bxy = BAC ...(ii)
 F, AG dk midpoint gSa Thus, in ฀’s BAS and Bxy, we have
AF = GC ...(1) xBy = ABC
Bxy = BAC
iqu% FBC esa] BF||DG rFkk D, BC dk midpoint gSa]
Therefore, AA–Criterion of similarity,
vr% G, CF dk midpoint gksxkA we have
CG = GF ...(2) BAC ~ Bxy
lHkh (1) vkSj lHkh (2) ls
AF = GF = GC ...(3)
b
Area BAC BA 2 g
Area BxY = Bx 2
b g
vc]

BLAM–335
T;kfefr
55. (4)
BA 2
2= (using (i)) P A
Bx 2

BA = 2Bx

5
5

3 cm

cm
cm
BA = 2 (BA – Ax)
90° 90°
Ax 2 1
e 2 j
1 BA 2 Ax
AB 2
Q
4 cm
R B C

53. (1) From similar ’s ABD and PQD From PQR, PQ = 52 42 9 3


x BD From ABC, BC = 5 3 16 4 and ABC
2 2

z QD
= 90°
Both triangles PQR and ABC are congruent
QD BD
1 1 ...(i) 56. (2)
x z A
From Similar ’s BDC and BQP,

Y BD C b
z BQ

BQ BD
1 1 ...(ii) B a C
Y Z We know that
From (i) and (ii), we have Third side
(a – b) < c < (a + b)
QD BQ BD BQ QD BD (8 – 3) < x < (8 + 3)
1 1 1 1 1 1 1 5 < x < 11
x y z x y x y
57. (3)
A
From third and fifth rations, we get
3 4
1 1 1
10
x y z D E

54. (3) In EFG and GCD x 6


EFG = GDC and EGF = CGD
EFG ~ GCD B C
Given
EG EF 5 EF
EF 9 cm DE||BC
CG DC 10 18
AD = 3 cm
Now, in ACB and ECF AE = 4 cm
ACB = ECF AC = 10 cm EC = 10 – 4 = 6 cm
ACB ~ ECF BD = x (let)
AC AB AC 15 ADE ADE
CE CF CG GE 9 AD AE
DB EC
AC 15
10 5 9 3 4 2
x 6 3
15 15 225
AC = 25 9
9 9 x 4.5 cm
2

BLAM–336
T;kfefr
58. (1) 2n–4=4×3
A
2n = 12 + 4 = 16
46°
n=8
D
4. (1) ekuk number of sides = 5x rFkk 4x.
M
(2 5 x 4 )90 (2 4x 4) 90
6
x° 5x 4x
60°
B E C LMEach interior angle FG 2n 4 IJ 90
OP
In AMD, N H n K Q
AMD = 90° – 46° = 44° = BME
(10x – 4) × 360° – (8x – 4) × 450° = 20x × 6°
In BME,
(10x – 4) × 12 – (8x – 4)15 = 4x
60° + 44° + x° = 180°
120x – 48 – 120x + 60 = 4x
x = 180° – (104°) = 76°
4x = 12
59. (4)
x=3
A R Number of sides= 15 and 12
5. (3) ekuk number of sides = 5x rFkk 6x.

3 5 10x 4
5x 24
12x 4 25
6x
B 4 C P Q

ABC ~ RPQ
LMEach interior angle = (2n 4) 90 OP
N n Q
ar b ABCg FG AB IJ 2
5x 2 6 24
ar b PQR g H RP K 5 6x 2 25

4 3 5x 2 4
x = 4.5 cm 6x 2 5
9 x
25x – 10 = 24x – 8
Type–III x = 10 – 8 = 2
Questions Based on Polygons Number of sides = 10 vkSj 12
6. (2) ekuk polygon dh number of sides = n.
1. (2) ekuk polygon dh number of sides = n.
Interior angles dk sum = (2n – 4) × 90°
FG 2n  4 IJ u 90q = 3 u 180q
(2n – 4) × 90° = 1440°
H n K 5

(2n – 4) = 1440° ÷ 90° FG 2n 4 IJ 90 108


2n – 4 = 16 H n K
2n = 16 + 4 = 20
n = 20 2 = 10 FG 2n 4 IJ 10 12
2. (1) ekuk polygon dh number of sides = n.
H n K
20n – 40 = 12n
FG 2n 4 IJ 90 = 144
20n – 12n = 40
H n K 8n = 40 n = 5
(2n – 4) × 10 = 16n 7. (4) Interior angles dk sum
20n – 40 = 16n = (2n – 4) × 90°
20n – 16n = 40 Exterior angles dk sum = 360°
4n = 40 (2n – 4) × 90° = 360° × 2
n = 10 Ÿ 2n – 4 = 2 × 360° ÷ 90° = 8
3. (2) ekuk polygon dh number of sides = n. Ÿ 2n – 4 = 8
b2n  4g90q 360q Ÿ 2n = 12
u3 Ÿn=6
n n

BLAM–337
T;kfefr
8. (3) Each interior angle 360
Each external angle = = 10°
(2n 4) 90 n
n 360
n= = 36
10
4) 90
(2n
? n 2 11. (4) ekuk polygon dh number of sides = n
( 4n 4) 90 3 According to the question,
2n
(2n 4) 90 360 3
(2n 4) 2 2 : =
n n 1
Ÿ
4n 4 3
(2n 4) 90 3
=
2n 4 1 360 1
Ÿ
4n 4 3
2n – 4 3
Ÿ 6n – 12 = 4n – 4 4
=
1
Ÿ 6n – 4n = 12 – 4 = 8
2n – 4 = 4 × 3 = 12
Ÿ 2n = 8 Ÿ n = 4
2n = 12 + 4 = 16
9. (3) ekuk polygon dh number of sides = n
16
(2n – 4) 90 360 n= =8
– = 132° 2
n n Aliter :
180°n – 360° – 360° = 132°n
External angle 3
180°n – 132°n = 720° =
Internal angle 1
48°n = 720°
720 1
External angle = 180 = 45°
n= = 15 3 1
48
For any polygon sum of all external angle = 360°
10. (3) ekuk polygon dh number of sides = n
According to question, 360
Each external angle = = 45
n
Exterior angle 1
= n=8
Interior angle 17 12. (4) ekuk regular polygon dh number of sides = n.
360 According to the question,
n 1
= (2n – 4) 90 360
(2n 4) 90 17 – = 108°
n n
n
(2n – 4) 5 20
360 1 – = 6°
= n n
(2n 4) 90 17
10n – 20 – 20 = 6n
4 1 10n – 6n = 40
=
2n 4 17 4n = 40
2n – 4 = 4 × 17 n = 40 ÷ 4 = 10
2n – 4 = 68 Aliter :
2n = 68 + 4 = 72 Let internal angle = x
external angle = y
72 Given,
n= = 36
2
x – y = 108
Aliter : x + y = 180
External angle 1
= 2x = 288
Internal angle 17 = x = 144°
Ext. angle + Int. angle = 180° gksrk gSA y = 180° – 144° = 36°

1 360q
n = 10.
External angle =
17 1
× 180° = 10° 36q

BLAM–338
T;kfefr
13. (2) n sides okys regular polygon osQ interior angles 22
dk sum = (2n – 4) right angles = × 7 × 7 = 77 sq. cm.
14
2n – 4 = 14 Diagonal of square LENO
2n = 14 + 4 = 18 = 7 cm.
18
n= =9 Its side = cm.
2 2
P Q
14. (3) 7 7 49
A Its area = =
2 2 2
= 24.5 sq. cm.
F B
shaded region dk area
= 94.5 sq. cm.
16. (4) ;gk¡ AB = 4CD
E C AB 4
CD 1

D A B
S R
O
3
izR;sd equilateral triangle dk area = × side2
4
D C
3 AOB rFkk COD esa
= × 6 × 6 = 9 3 sq. cm.
4
OAB = OCD rFkk
regular polygon ABCDEFA dk area OBA = ODC
= 6 × 9 3 sq. cm. AOB ~ DOC (AA similarity ds }kjk)
= 54 3 sq. cm. ar COD CD 1 2
1
ar AOB = AB 2 = 4 2 = 16
four equilateral triangles APF, ABQ, DES rFkk CDR
ar COD : ar AOB = 1 : 16
dk area = 4 × 9 3 sq. cm. 17. (3) According to question
= 36 3 sq. cm Given : Interior angle
– Exterior angle = 150° ...(i)
shaded region dk area
We know Interior angle + Exterior angle
= 180°...(ii)
e
= 54 3 36 3 – 2 9 3 j sq. cm. Solve equation (i) and (ii)
= 72 3 sq. cm. Interior angle = 165°
Exterior angle = 15°
E B
15. (2) A 360 360
E no. of sides = Exterior angle = 24
15
P L N 18. (1) Let the no. of sides is 5x and 4x
According to questions
O
D F C FG180 360 IJ FG180 360 IJ 6
square dk area ABCD
H 5x K H 4x K
= 14 × 14 = 196 sq. cm. 360 360
180 180 6
semi-circle dk radius = 7 cm. 5x 4x

360 360
r2 6
Its area = 4x 5x
2

BLAM–339
T;kfefr
21. (1) let x1 = 5x
360
FG 1 1 IJ 6 and x2 = 6x
H 4x 5x K using the formula.
1 1
x 3 5x 2
20x 60 180
5x 24
No. of sides are 5x and 4x 6x 2 25
180
= 5 × 3, 4 × 3 6x
= 15, 12
19. (2) According to question b5x 2g 6 24
n No. of sides b6x 2g 5 25
Interior angle
25x – 10 = 24x – 8

=
b2 x 4 g 90
180
360 x=2
x x no of sides x1 = 5 × 2 = 10 and x2 = 6 × 2 = 12
22. (2) Ex : In.
360 1:4 5 180°
(1) 150° = 180° –
x 1 36°

360 360
30 no. of sides = 10
x 36

x = 12 23. (4) Here,


the interior angles of polygon are in A.P.
360 where,
(2) 105° = 180° –
x
2 2 180
a= 120
360 3 3
75
x and d = 5°
sum =120° + 125° + .......... + to x terms
24
x=
5 x
(x – 2) × 180° =
2
2 120 b x 1g 5
360
(3) 108° = 180° =
x x
(x – 2) × 180° = 5x 235
2
360
72 x2 + 47x = 72x – 144
x
x2 – 25x + 144 = 0
x=5 (x – 9) (x – 16) = 0
360 x = 9 (16 is not possible)
(4) 144° = 188° – 24. (2) let the sides of polygon = x
x
A/q
360 (5 × 172°) + (x – 5) × 160° = (x – 2) × 180°
36
x 172° + 32 x – 160 = 36x – 72°
x = 10 4x = 84
only 105° angle which can never be interior an- x = 21
gle of regular polygon. 25. (3) Using the formula, we get
20. (2) Inte. Ex.
2:1
b
x x g
3
104
2
3 180°
x (x – 3) = 208
1 60°
16 × 13 = 208
360 x = 16
x= 6 (no. of sides)
60 So, no. of sides = 16.

BLAM–340
T;kfefr
ABO = 60°
Type-IV
OA
Questions Based on Quadrilaterals sin 60°
AB
(Rhombus, trapezium etc.)
3
1. (4) D 8 = OA
A 2

OA = 4 3
O
AC = 8 3 metre
B C
5. (4) A
AO × BO = OD × OC
3 (3x – 19) = (x – 5) (x – 3)
9x – 57 = x2 – 8x + 15 D B
x2 – 17x + 72 = 0
x2 – 9x – 8x + 72 = 0
x (x – 9) – 8 (x – 9) = 0 C
(x – 8) (x – 9) = 0
Rhombus ABCD esa,
x=8 ;k 9
AB = AD = 12 cm
2. (2) S
R BAD = 60°
90° ABD = ADB = 60°
BD = 12 cm D;ksafd ABD ,d equilateral triangle
O
P
gSA
Q 6. (1) A D
1
PQO = PQR = 60°
2
POQ ls]
OPQ = 180° – 90° – 60° = 30°

OQ
sin OPQ = B C
PQ
AB + CD = AD + BC
1 6 + 5 = 7 + BC
OQ = PQ sin 30°= 6 3
2 BC = 11 – 7 = 4 cm
QS = 2 × 3 = 6 cm 7. (4) A
B
3. (3) Rhombus dh side = 6 2 2
8 10
P
4. (1) A C
D
Clearly,
AP × BP = PD × PC
B
O
D 8 × 6 = PD × 4
8 6
PD = = 12 cm
4

C 8. (1) A D

BAD = 60°
B C
BAO = 30° E F

BLAM–341
T;kfefr
ar trap (ABCD) equation (i) ls]
x2 + y2 = (8 – 2x)2
1
= (AD + BC) DF x2 + (x – 1)2 = 64 – 32x + 4x2
2
x2 + x2 – 2x + 1 = 64 – 32x + 4x2
1 2x2 – 30x + 63 = 0
35 = (5 9) DF
2
30 900 4 2 63
35 = 7 × DF x=
4
DF = 5

FC =
9 5
2 cm
LM ax 2
bx c 0 OP
2 MM b b2 4ac P
PQ
CD = DF 2 FC2 52 22 N x 2a

= 25 4 = 29 cm
30 900 504
9. (4) Trapezium dh sides = 2x and 3x cm =
4
1
(2x 3x ) 12 480 30 396 30 6 11 15 3 11
2 = = =
4 4 4
480
5x 80
6 15 – 3 11
x= <8
2
80
x 16
5 15 3 11 13 11
y= –1 =
Longest side = 3x = 16 × 3 = 48 cm 2 2

10. (1) x2 y2

F 15 3 11 I F 13
2
3 11 I 2

= GH 2 JK GH 2 JK
225 90 11 99 169 99 78 11
=
AE = x cm. 4
AL = y cm.
592 – 168 11
EL = x2 y 2 cm. =
4
= 148 – 42 11

AB = 2x + x2 y 2 cm. = 99 49 – 2 7 99

AD = 2y + x2 y 2 cm. 2
e j
e
= 3 11 – 7 j = 3 11 – 7 cm.

2x + 2 2 =8 11. (2)  DE and FB are equal and Parallel


x y
DEBF is a parallelogram
x2 y 2 = 8 – 2x ...(i)
D E C
2y + x2 y2 = 6

x2 y 2 = 6 – 2y ...(ii) M
8 – 2x = 6 – 2y
2x – 2y = 8 – 6 L
x–y=1
y=x–1
A F B

BLAM–342
T;kfefr
EM||DL
ECM and DCL (similar) A B
EC CM 10

DE ML
O 6
EC = DE 5
CM = ML 9
D M 2 4 C
Similarly AL = ML
1
1 3
ML = AC
3
12. (4) We know that ratio of the areas of two similar
triangles is equal to the ratio of squares of their
8
corresponding sides
N
b
Area DPQ g PB2 20 1
b
Area DPC g PC2 b
Area DPC g 4 1= 2 (vertically opposite angle)
Area ( DPC) = 80 sq units 3= 4 + 9 (Alterna (ASA) interion angle)
13. (3) Let the sides of a parallelogram be x, y and xk, DMN BMC
yk DN = BC = AD
AN 2
So, AN = 2BC ...(1)
C E BC 1
In OAN and OBC,
5 = 6 (Vertically Opposite Angle)
Y
7 = 8 (Alternate Interior Angle)
(Alternate Angle)
9 = 10
A B D
x OAN ~ OBC
So, the side will be in same ratio
T S
AN ON 2 ON
BC OB 1 OB
(From equation i ) bg
yk 15. (1)
16. (3)

P Q R D C
xk
7 cm

Since sides of two parallelogram are in the ratio N


of 1 : k. 8 cm
ABC ~ PQT
A M 10 cm B
AC BC
PT QT area of parallelogram = Base × height
area ( ABCD) = AD × BN
BC y 1
= (10 × 7) cm2 ...(1)
QT yk k
also area ( ABCD) = AD × BN
Let BC = z and QT = zk = (AD × 8) cm2 ...(ii)
Ratio of areas of two similar parallelogram
(i) rFkk (ii) ls]
x z 1 10 × 7 = AD × 8
=
xk zk k2
35
14. (2) In DMN and BMC, AD = = 8.75 cm
4
DM = MC (Given)

BLAM–343
T;kfefr
17. (2) From the mid point theorem. let CQ = G cm
EF FG GH HE 1 PCQ ~ ABQ
AD DC CB BA 2 CQ PC
BQ AB
EH FG GH HE
AD DC CB BA a 5
a 24 15
1
2
b
AD DC CB BA g 1 3a = a + 34
bAD DC CB BA g 2 a = 12
In ABQ [By pythagorean Triplet 15, 36, 39]
1 AQ = 39 cm
18. (1) MN = (AB + CD) 2 × 15 = 14 + CD
2
Type-V
CD = 16 cm.
Questions Based on Circles
19. (4) D C
1. (4) A

2x B C

A H 3x B esa centroid, incentre, ortho-


Equilateral triangle
centre ,d common point ij coincide djrs gSaA
Let, side of square = 6x
1 Height
Inradius
then × 3x × 2x = 147 3
2
x2 = 49 x=7 Height = Median = 3 × 3 = 9 cm.
Side of square = 42 m
AC2 = (42)2 + (42)2 = 2 (42)2 2. (4) A

AC = 42 2 m
20. (1) We know, fdlh triangle osQ nks side (Hkqtk) dk sum
rhljh side lsa cM+k gksrk gSA O
ABD esa AB + DA > BD ...(i)
and BDC ; BC + CD > BD ...(ii) B C
D
(i) and (ii) ls AB + BC + CD + DA > 2BD
DB = DC = 3 cm
21. (3) A D
AD = AB 2 BD 2 6 2 – 32
10
= 36 9 27 3 3 cm
15 P OD = in radius
1
= 3 3 3 cm
5 3
in-circle dk area
B 24 C a Q
= r2 = 3 3 = 3 cm2

BLAM–344
T;kfefr
2nd Method 5. (1) A
a 6
t 3
2 3 2 3
2
Area = e 3j 3 cm 2 90°
B C
3. (1) A ABC = 90°
AC = Circle dk diameter
F E AC = 62 82 36 64 = 100 = 10 cm
O Circum-radius = 5 cm

B D C 6. (1)
Let AB = x cm
O
x
BD = cm
2

X Y
x2 3
AD = x2 x cm
4 2
XOY = 90°;OX = OY = radii
XOY right angled triangle gSA
1 3 x
OD = x cm 1
3 2 2 3 × (OX) × (OY) = 32
2
OB = BD 2 OD2 r2 = 2 × 32 = 64
r= 64 8
x2 x2 4x 2 x Area = r2 = 64 sq. unit
= cm
4 12 12 3
7. (2) C
x
10 x 10 3 cm A B
3 O

x 10 3 D
OD = 5 cm
2 3 2 3
5
Aliter : AO = OB =
2
circum radius (R)
leckgq f=kHkqt ds in radius r = FG 5 IJ 2
2 OC = 2 2 = 52
10
AC OA H 2K
r= = 5cm
2
25 100 25 75 5 3
4. (2) Equilateral triangle = a unit = 25 = =
4 4 4 2
a
In-radius = 5 3
2 3 CD = 2 OC = 2 = 5 3 cm
2
a
Circum radius =
3 F
8. (1) C D
Required ratio

=
FG a IJ : FG a IJ
2 2
1 1
: 4 :1
O
H 3K H2 3K 3 12 A E B

BLAM–345
T;kfefr
OE AB
12. (2) A
BE = AE = 3 cm
,oa OF CD
FD = CF = 4 cm
O O'
OBE ls] C

OE OB2 BE 2
B
= 2 2
5 3 4 cm OC = 2 cm
OFD ls] OA = 4 cm

OF 52 42 3 cm AC= 42 22
EF = OE + OF = 4 + 3 = 7 cm
16 4 12 = 2 3 cm
9. (2) A AB = 4 3 cm
O
P 13. (4)
C C

Q
B 6
OPB = OQB = 90°
OPB + OQB = 180° A 8 B
,oa PBQ + POQ = 180°
blfy,, ;g ,d cyclic quadrilateral gSA BAC = 90°
BC circle dk diameter gS]
10. (2) A
BC = AB2 AC2
C D
O = 82 62 = 64 36
B = 100 = 10 cm
r Circle dk radius = 5 cm
CO = OD =
2 60
CA = r 14. (2) = 60° = = radian
1 180 3

r2 3r 75 5
AO = AC2 OC2 = r2 = T2 = 75° = = radian
4 2 180 12

3r l 1
AB = 2 3r T=
2 r r

r1 2 5 3
11. (1) 5: 4
r2 1 12
15
O
15. (4) A
5
cm
B D C

BO = OC = 15 cm C
O3c
OD = 9 cm m

BD = 152 92 = 24 6 = 12 cm
B
BC = 2 × 12 = 24 cm

BLAM–346
T;kfefr
OC = 3 cm OO = 7
OA = 5 cm r1 + r2 = 7
4 + r2 = 7
AC = 52 32 4
r2 = 7 – 4 = 3 cm
AB = 2 AC = 8 cm

20. (2)
B
16. (2) 6 cm
A O
P
C 3 cm D

AB = 6 cm _ CD = 3 cm A B
PD = 5 cm _ PB = ?
OA = OB = AB
PA × PB = PC × PD
OAB equilateral triangle gSA
(PB – 6) PB = 2 × 5
PB2 – 6PB – 10 = 0 AOB = 60°
21. (4) ekuk radius = r cm
6 36 40 r2 = r12 + r22
PB =
2 r2 = r12 + r22 = 102 + 242
= 100 + 576 = 676
= 6 76 = 6 8.7
7.35 cm
2 2 r= 676 = 26 cm
17. (1) A
22. (2)

C B C
O Oc

B
A
AC = 4 cm
OA = 5 cm
Circle dk radius = r units
OC = 52 42 3 cm
AB = BC = CA = 2r unit
OO = 2 × 3 = 6 cm
blfy,, ABC ,d equilateral triangle gSA
18. (4)
A 23. (1)
30 40
E
A B
B C D

D F
E
BC = x CD = 50 – x C
AC 2 = 302 – x2 = 402 – (50 – x)2
900 – x2 = 1600 – 2500 + 100x – x2 AE = AD = 3.5 cm
100x = 1800 x = 18
BE = BF = 4.5 cm
AC = 302 182 48 12 = 24 CD = CF = 5.5 cm
AE = 2 × 24 = 48 cm Perimeter of triangle = AB + BC + CA
= AE + EB + BF + FC + CD+ DA
19. (2) = 2(AE + BE + CD)
= 2(3.5 + 4.5 + 5.5)
o r
1 r2 oc
= 2 × 13.5 = 27 cm

BLAM–347
T;kfefr
24. (2) 2nd Method

C Pythagoreon triplet
In, AFO, 5, 12, 13
O In, DEO, 12, 5, 13
FE = 5 + 12 = 7
A
OD = 13 cm
B
26. (2) A
OAB ,oa OCA esa]
OC = OA = OB = radii
2 OAB + AOB = 180°
2 OAC + AOC = 180°
O
AOB + AOC = 360° – 2 ( OAB + OAC)
= 360° – 2 × 90° = 180° M
AB = AC P Q
AOB = 90°
OAB = 45° B
OB OB
sin OAB = sin 45° = PQ = 18 cm
AB 5 2
PM = MQ = 9 cm
1 OP = OB = r cm
OB = 5 2 . sin 45° = 5 2 × = 5 cm
2 MB = 3 cm
OPM ls,
25. (2)
OP2 = PM2 + OM2
r2 = 92 + (r – 3)2
O r2 = 81 + r2 – 6r + 9
81 – 6r + 9 = 0
A C B 6r = 90 r = 15
AB = 30 cm

OA = OB = r unit
A D
AOC = 30°; AC = CB
AOC esa] 5 12
27. (1) F E
AC O
sin AOC = 12
OA 5

AC 1 AC B C
sin30 =
r 2 r

r AB = 10 cm
AC = AF = FB = 5 cm
2
CD = 24 cm
r CE = DE = 12 cm
AB = 2 r unit
2
ekuk] OE = x cm
Required ratio = 1 : 1 OF = (17 – x) cm
Aliter : ODE ls]
OA = OB
OD = OE 2 DE 2 = x 2 122 ...(i)
OAB = OBA = 60°
OAB equilateral triangle gSA OAF ls]
OA = OB = AB
OA = OF 2 AF 2 = (17 – x )2 52 ...(ii)

BLAM–348
T;kfefr
 OA = OD 30. (1)
C
x 2 122 = (17 – x )2 52
D
x2 + 144 = 289 – 34x + x2 + 25 5
2
34x = 289 + 25 – 144 = 170 P
A B
170
x= =5
34 BDP ~ PAC
Equation (i) ls, AC AP 5
=5:2
BD BP 2
OD = x 2 122
31. (4)
= 52 144

= 169 = 13 cm B
O
2nd method
Pnthayorean triplet
In, AFO : 5, 12, 13
A
In DEO : 12, 5, 13
FE = 5 + 12 = 17 P
OD = 13 cm
OB BP
Q
28. (4) P ,oa OBP = 90°
OAP+ OBP=90° + 90°= 180°
OAPB esa]
r1 r2 OAP + APB + AOB + OBP = 360°
APB + AOB = 180°
Cyclic quadrilateral gksxk
r1 + r2 = 13 cm,
S
r2 – r1 = 9 – 4 = 5 cm
R
2
PQ dcentres osQ chp dk distance i br 2 r1 g
2
r1 13 cm r2

e13 2
52 j = 12 cm

Square dk area = 12 × 12 = 144 sq, cm


29. (3) C 2
32. (1) SR dcentres osQ chp dk distancei br 1 r2 g 2

A D B
b13g b5g
2 2
18 8 =12 cm
O
33. (1)
P S
ABC = 45°
ABO = 45°
X Y
BD = 3 cm
R Q
BD
cos OBD =
OB Transverse tangent dh length
3 1 3
cos 45
OB OB XY 2 (r1 r2 )2
2

OB = 3 2 cm Ÿ 8 XY 2 92

BLAM–349
T;kfefr
Ÿ 64 = XY – 81 2
BD = 132 82
Ÿ XY2 = 64 + 81 = 145
Ÿ XY = 145 = 169 64 = 105 cm
ODB ~ AEB
34. (4) P
OD OB
AE AB

T R S 8 13
O
AE 26
AE = 16 cm
Q AE = 16 cm _ AED = 90°

OPS = OQS = 90° AD = AE 2 DE 2


PSQ = 20°;
POQ = 160° AD = 256 105 361 = 19 cm
PTQ = 80°
PRQT concyclic quadrilateral gSA 37. (1)
PRQ = 180° – 80° = 100°

35. (1)

A a b B
a b

c c
C

ACB = 65°
x = AB = a + b AOB = 2 × 65° = 130°
y = BC = b + c OAP = 90°; AOP = 65°
z = CA = a + c APO = 180° – 90° – 65° = 25°

AB BC CA 38. (3) A
s= a b c
2
B C C1
ar ( ABC)
O
= s (s x )(s y )(s z)
C2
= (a b c )abc
AB = AC = fdlh ,d point ls draw fd, x, tan-
E gents.
36. (2)
OB = OC = 3 cm
D OA = 12 cm
ABO = 90°
A B
O AB = 122 32 = 15 9 = 3 15
1
OAB = = OB × AB
2

1 9 15
ODB = 90° = 3 3 15
2 2
OD = 8 cm
OB = 13 cm ar (OBAC) = 9 15 sq. cm

BLAM–350
T;kfefr
3 40 8 x
39. (2) =
5 40

3 40 8 x
=
1 8
24 = 40 – 8x
8x = 40 – 24 = 16
x=2
DG = 2 cm.
DE = 4 15 Units
DCG = 30°
41. (1) Q
DG
sin 30° =
CD
O P
2
= R
2 CD
QP = PR PQR = PRQ
CD = 4 cm.
 QPR = 45°
CGD = CFE
DCG = ECF 180 45
PQR = = 67.5°
CGD ~ CFE 2
DG CD 180 67.5
EF CE OQR =
2
2 4 = 56.25° = ORQ
= QOR = 180°– 2 × 56.25°
x 4 2 x
= 180° – 112.5° = 67.5°
tgk¡ EF = x cm.
1 2 42. (3) R S
=
x 6 x
2x = 6 + x Q P
x = 6 cm. A B
E
D
40. (4) QP = 16 cm.
A SA RP
B C
SP = AP 2 SA 2
larger circle dk radius
= 122 42
= 5x units
smaller circle dk radius = 16 8 8 2 cm.
= 3x units
RQ = RS = x cm.
DE =
RP = (x + 8 2 ) cm.
b D istan ce between cen tre g  br 2
1  r2 g 2

PQR esa ,
= (8 x ) 2
(5x 3x )2 RP2 = RQ2 + QP2
(x + 8 2 )2 = x2 + 162
= 64 x 2
4x 2 = 60x 2
x2 + 16 2 x + 128
= 2 15x units ...(i) 2
= x + 256
ADB = AEC = 90°
ABD = ACE 16 2 x = 256 – 128 = 128
ABD ~ ACE 128
BD AB x= = 4 2 cm.
16 2
CE AC

BLAM–351
T;kfefr
PB : PA = 2 : 3
43. (1) 2
S R PB = ×10 = 4 cm.
5

3
O PA = ×10 = 6 cm.
5
Y
P Q OP = 6 – 5 = 1 cm. = MN
Again,
Let, QY = x cm and YR = y cm. CN : ND = 2 : 3
square dk diagonal = PR = 8 cm. 2
CN = ×10 = 4 cm.
8 5
PQ = = 4 2 cm.
2
3
PR RY ND = ×10 = 6 cm.
5
PRY esa, ON = 1 cm.
PR2 + RY2 = PY2 ONS esa,
2 SM = x cm (let)
(8)2 + y2 = 4 2 e x j ...(i)
SN2 + ON2 = OS2
QRY esa, (x + 1)2 + 12 = 52
RQ2 + QY2 = RY2 (x + 1)2 = 25 – 1 = 24
2
e4 2 j
2
+ x2 = y2 ...(ii) e
= 2 6 j
equations (i) rFkk (ii) ls x+1= 2 6
2 2
(8)2 + 4 2 e j + x2 = 4 2 e x j e
x = 2 6 – 1 cm. j
64 + 32 + x2 = 32 + 8 2 x + x2 Q
45. (1) P
8 2 x = 64
O1
64
x= = 4 2 cm. O2
8 2 D
A
SY = PS2 PY 2 S C R B T

QR = SR = 20 cm.
2 2
= e 4 2 j e2 4 2 j QT = 25 cm.

RT = QT 2 – QR 2
= 32 128 = 160
= 4 10 cm. = (25)2 – (20)2

= 625 – 400
C Q
44. (4)
= 225 = 15 cm.
R N M S
A B QT FG 25 IJ cm.
O P O2B = r; Ex-radius =
2
= H2K
QR = a; RT = b
(a – r) + (b – r) = 2R
(20 – r) + (15 – r) = 25
D L
35 – 2r = 2s
AB = CD = 10 cm. 2r = 35 – 25 = 10

BLAM–352
T;kfefr
10 47. (3) ;gk¡, AB = 4 cm
r= = 5 cm.
2
O1C = 10 cm.
O1D = 5 cm.
O1DO2 esa
CB = O2D = 15 cm. O
A
C
O1O2 = O1D 2 DO 22 P B D

= 52 152 = 25 225

= 250 = 5 10 cm.
CD = 16 cm
46. (2) AB = 24 cm,
CD = 32 cm AB 4
PB = = = 2 cm
PQ = 8 cm 2 2
PB = 12 cm rFkk QD = 16 cm CD 16
PD = = 8 cm
2 2
A P 12 B OPB esa
C 8 Q 16 D
OB2 = OP2 + PB2
x OP2 = r12 – 22
O (ekuk OB = r1)
OP2 = r12 – 4 .....(1)
OPD esa]
OD2 = OP2 + PD2
PBO esa, r22 = OP2 + 82 (ekuk OD = r2)
OB2 = PB2 + PO2 OP2 = r22 – 64 .... (2)
OB2 = 122 + (8 + x)2 ...(i) equation (1) rFkk (2) ls
In QOD,
r12 – 4 = r22 – 64
OD2 = QD2 + OQ2
r22 – r12 = 60
OD2 = 162 + x2 ... (ii)
48. (3) ACB = 90°
As
ACX = 35°
OB = OD = circle ds radii
OB2 = OD2 A x
equation (i) rFkk (ii) ls
122 + (8 + x)2 = 162 + x2
144 + 64 + x2 + 16x = 256 + x2 O C
16x = 256 – 208
16x = 48
x 3 Y
B

Cirlce dk radius = 16 2  32 ACX + ACB + BCY = 180°


= 256  9 90° + 35° + BCY = 180°
BCY = 180° – 125°
= 265 cm BCY = 55°
Circle dk area = Sr2 CAB = 55°
2 Tangent ds lkFk cuk gqvk exterior angle, triangle dks
= 3.14 × e 265 j opposite interior angle ds cjkcj gksrk gSA
= 3.14 × 265 49. (4) C1 C2 = 15 cm
= 832.86 cm2 ekuk PO = x rFkk C1 O = y

BLAM–353
T;kfefr
C2O = 15 – y
OP = 225  400
P
OP = 625
OP = 25 cm
C1 O C2 Let radius of small circle = r
O' X = r
Q
PX = 25 – 15 = 10
C1OP esa] PO' = 10 – r
C1P2 = PO2 + C1O2 PO'B is similar to POC
PO2 = C1P2 – C1O2 OC OP
=
PO 2 92  y 2 ...(i) O' B O' P

In C2OP 15 25
=
C2P2 = PO2 + C2O2 r 10 - r
PO2 = C2P2 – C2O2 150 – 15r = 25r
40r = 150
PO 2
12  (15  y )
2 2
...(2)
r = 3.75 cm
equation (i) rFkk (2) ls 51. (4) Here, XA = 16 cm
92 – y2 = 122 – (15 – y) 2 YZ = ?
81 –y2 = 144 – 225 – y2 + 30y
30y = 81 + 81
162
y= x
30
y = 5.4
Y
PO2 = 81 – (5.4)2 A
= 81 – 29.16 Z Com
t an m on
PO = 5184. gen
t
PO = 7.2
PQ dh length = 2PO tangents dh properties ds }kjk
= 2 × 7.2 YA = XA rFkk AZ = XA
= 14.4 cm XA = 16 cm
50. (2) ;gk¡, CP = 20 cm YA = AZ = 16cm
OC = 15 cm YZ = YA + A Z
O'B = ? = 16 + 16 = 32 cm
52. (2) lHkh points equal distance ij gSa]
FH = FD
FHD = FDH

x
A
P O' O
H B
B

C
G C

PCO esa]
OP2 = OC2 + PC2 F D
[Pythagoras theorem ds }kjk]
E
OP2 = 152 + 202

BLAM–354
T;kfefr
FDH esa] Given,
FDH + FHD + DFH = 180° AR = AP = 5 cm, MB = 3 cm
2 FDH + DFH = 180° AB = 5 – 3 = 2 cm
DH circle dk diameter gS 2
AN = 1 cm
2 FDH + 90° = 180° 2
2 FDH = 90° In PNA
FDH = 45°
PN = 52 12
53. (2) P
= 2 6 cm
PQ = 2 × PN

S = 2 2 6
35
= 4 6 cm

28 P X Q
55. (2)
A
Q R
let QR = x
and SR = y

PR b35g 2
x2 = 1225 x2 B
R Y S
we know that
PQ × QR = QS × PR Given,
35 × x = 28 × 2 xy = 5 cm and AB = 3 cm
1225 x
 AX = BY = a (let)
LM 35 x OP 2

1225 x2
XY = XA + AB + BY
N 28 Q 5 = 2a + 3
a = 1 cm
25 2 Now
x x2 1225
16 PX2 = AX.XB
140 PX2 = 1 × 4 = 4
x= =1×4=4
3
PX = 2
Again,
Similarly, QX = 2
SR2 = QR2 – QS2
So, PQ = 2 + 2 = 4 cm

y2 =
FG 140 IJ b28g
2
2

H3K 56. (1) P


5
o
y = 37.33 cm 5
9 S
54. (4) P

R M T Q

Given,
N PR = 9 cm
M R
A B PS = 10 cm
OP = OS = 5 cm
let 5 = a cm
Now,
Q PRQ ~ OMQ

BLAM–355
T;kfefr
Given,
PR PQ
AB = 16 cm AP = 8 cm
OM OQ
CD = 4 cm PC = 2 cm
9 10 a 5 from AOP and COP
a
5 5 a 4 OA2 – AP2 = OC2 – PC2
again, OA2 – OC2 = AP2 – PC2
PRQ ~ STQ = 82 – 22
= 64 – 4 = 60 cm
5 59. (4) Given,
10
PR PQ 9 4 BC = 36 cm
ST SQ ST 5
OA = 10 cm
4
1
GR ( ABC) = 36 10 180 cm2
9 45 2
ST = 1 cm
ST 5
57. (3) A 60. (1)
z z
P
2 2 Q

I

O

S 95°

B C

15° 72°
D T R U
Given the Figure,
BZD = Z° In cyclic Quadrilateral,
Z PSR + PQR = 180°
BAD = PQR = 180° – 95° = 85°
2
Angle subtended on the circum circle is half the
angle substended on the centre of circle and I is 61. (3) P R
In–centre.
x
ABI = IBC =
2
43°
In ABI, S
BAI + IBA = BID (Enterion angle) Q
z x
y RPQ + 90° – 43° = 47°
2 2
RPQ = RSQ = 47° (v¼Zo`Ùk osQ dks.k)
z + x = 2y
z x A
2
y

62. (1) R Q

O
58. (3) A B B C
C P D

BLAM–356
T;kfefr
RBPQ is a cyclic Quadrilateral.
RQP + RBP = 180° 65. (2) A
RQP = 180° – RBP ...(1)
Now
E
RBP = RBA + ABC + CBP

C A
RBP = B F
2 2 B
2x
from eqn (1) y x

FG C
B
A IJ C
y
D E
RQP = 180° – H 2 2 K
let the sides of square = x
= 180° –
b A B g b B C g and BC = CD = y (let)
2 Now,
2y2 = x2
FG 180 B IJ 90
B
= 180° – H 2 K 2
y2 =
x2
2
63. (4) In BCF,
15 c
Y m
S 2

R X
CF2 = y2 + e 2x j
9 cm R2 = y2 + 2x2
Z T
x2 5x 2
= 2x 2
We know that 2 2
length of tangents drawn from a point to the cir-
5x
cle are equal the Radius of circle (R) =
xy = xz = 15 cm 2
ZT = XZ – XT = 15 – 9 = 6 cm
5 5
64. (2) = (from )
2 2

A x B 5
= = 2.5 cm
1 2
o
B
2
E F 6
60°
66. (1) O
30°
30° A

D x x x 6
C
C
AOB ~ COD
In ABO
AB 1 AB
CD tan 60° =
2 OB

x 3 AB
1 AB = 6 3 cm
3x 1 6
2
and AB = AC
1 1 Now
9
2
AB 2 AC 2 2AB 2 = 2 6 3 = 6 6 cm

BLAM–357
T;kfefr
A
70. (2) T

67. (2) M
o
B C
R
AM = 15 cm = R (circum-radius) S
OM = 6 cm = r (In-radius) 60°

AC = 15 × 2 = 30
In Right-angled triangle (ABC)
60° 60°
We using pythagorean Triplet
P O Q
18, 24, 30
Now, the sides of ABC are 18, 24 and 30 cm.
68. (3) See Q.No. 65
Given,
OQ = QR = RT
69. (3) P OQ = OR (Radius)
ORS is a equilateral triangle
O 60°
S R= O= Q = 60°
30°
30° PQRS is a cyclic Quadrilateral
RSP = 120° and SPQ = 60°

60°
Now, In TPQ
PTQ + TQP + QPT = 180°
R
PTQ + 60° + 60° = 180°
PTQ = 60°
Given
D
RS = 10 2 cm

RQ = QS = 5 2 cm
ORS is a equilateral triangle 71. (2) O
In 3 3
OQR A 2 P B
C
OQ
tan 60° = let
RQ
OD = OC = r
3 OQ CD = 2r
OQ = 5 6 cm
1 5 2 PC = 2 cm
PD = 2r – 2
 OP = OS = OR = RS = 10 2 cm Now
In PQR AP × PB = PD × PC
PR2 = PQ2 + PQ2 3 × 3 = (2r – 2) × 2
2 2
13
e
= 10 2 5 6 j e5 2 j r=
4

= 400 200 3 = 200 2 e 3 j CD =


13
×2 =
13
6.5 cm
4 2

BLAM–358
T;kfefr
PB = 4 + 5 = 9
72. (1) P In PTB,
15 BT = 92 62 45 3 5
M O
3 5
OB =
2
R A N S
Q 75. (2) P
12
8
let PQ = 2a
24 R
PM = MQ = a = 10
S
RS = 2b O
RN = NS = b = 12 and OA = C Q
we know that
2r2 = a2 + b2 + c2 T
2 2 2
(2 × 15 × 15) = 10 + 12 + C
Given,
C2 = 450 – 100 – 144 = 206
PR = 12 cm
= 206
PS = 8 cm
C= 206. PQ = 24 cm
Area of triangles are equal
73. (4) A QP × PR = QR × PS
6 cm 24 × 12 = QR × 8
O
120°
60° P QR = 36 cm

6 cm OQ =
36
= 18 cm.
2
B

 AP = PB B
A
180 60
PAB = PBA = = 60°
2 76. (1) P
APB is a equilateral triangle
C 11
15
3 D
ar ( APB) = 6 6 = 9 3 cm2
4
Given
T AB = 4 cm
6 CD = 11 cm
BD = 15 cm
74. (3) P PC = 15 – 11 = 4 cm
o
PA = x (let)
4 We know that,
x
A PA × PB = PC × PD
B x × (x + 4) = 4 × 15
x2 + 4x – 60 = 0
we know that,
(x + 10) (x – 6) = 0
PT2 = PA × PB
x=6
62 = 4 × (4 + x)
Therefore,
x=5
PB = x + 4 = 6 + 4 = 10 cm

BLAM–359
T;kfefr

77. (1) D B
11 80. (4)
C
6 6
10 cm
L O A
4 6
x B
A C

we know that In ABO,


LD × CL = AL × BL
17 × 6 = (4 = x) × 4
AB = b10g b6g
2 2
8 cm

102 = 16 + 4x ar ( ABO) = ar ( ACO)


4x = 86
FG 1 6 8
IJ
x = 21.5 cm area of quadrilateral ABOC = 2 × H2 K
= 48 cm2

78. (4)
A B 81. (4)
P A

3.
5
Given AB = 8 cm R Q
PB = 3 cm
AP = 8 – 3 = 5 cm 7

79. (2) B P 4.5 C


B
10
–a

AQ = AR = 3.5 cm
–a
10

E F
10

BR = BP = 7 cm
8
PC = QC = 4.5 cm
12

a
–a

Perimeter of DABC = 2(3.5 + 4.5 + 7) = 30 cm

K
A a D 12–a C 83. (3) A
K 12

O 130° P
Given AB = 10
BC = 8
CA = 12 B
let AE = a = AD
DC = 12 – a = FC AOB + APB = 180°
BE = BF = 10 – a AOB = 180° – 130° = 50°
BC = 10 – a + 12 – a In AOB
8 = 22 – 2a  OA OB
2a = 14
180 – 50
a=7 OAB = OBA = = 65°
2
AD × BE = 7 × (10 – 7) = 7 × 3 = 21
‡‡‡

BLAM–360
T;kfefr

ADVANCED LEVEL QUESTIONS


1. In the given figure
2 = 1 and 3 = 4. The value of T equals to C
A E
fn, x, fp=k es]a 2 = 1 rFkk 3 = 4 rks T dk eku cjkcj gS z
90° 55°

P
30° 3 y
2 55
1
T 4 B x D F

(1) 125°, 35°, 125° (2) 35°, 125°, 125°


(1) 30° (2) 60°
(3) 125°, 125°, 35° (4) 35°, 125°, 35°
(3) 45° (4) 40° 5. In the given figure, AB || CD, then the value of x
2. A square is given in which 3 = 4. Then which will be
of them is correct
fn, x, fp=k esa, AB || CD rks] x dk eku gksxk
,d oxZ fn;k x;k gS ftleas 3 = 4 rks bueas ls dkSu lgh
gS A B

A B 132°
48°
E E
148°
3 4
C x D
D C

(1) ED = EC (2) ED EB (1) 100° (2) 110°


(3) 130° (4) 120°
(3) ED > EB (4) ED < EB
6. In the given figure, X =62°, XYZ = 54°. If YO
3. In the given figure, AB || CD then the value of y and ZO are the bisectors of XYZ and XZY,
will be then the values of OZY and YOZ will be
fn, x, fp=k esa, AB || CD rks y dk eku gksxk fn, x, fp=k esa, X =62°, XYZ = 54°. ;fn ZO rFkk
YO, XYZ rFkk XZY ds lef}Hkktd gSa rks OZY rFkk
YOZ dk eku gksxk
C
X
A
2y 62°
y
y 5y
B O
D
54° 64°
(1) 30° (2) 20°
(3) 40° (4) 60° Y Z
4. In the given figure, A B || CD and CD || EF.
Also, EA AB. If BEF = 55°, then the values of (1) 32°, 120° (2) 32°, 121°
x, y and z will be (3) 30°, 120° (4) 121°, 32°
7. Suppose that ABC is an equilateral triangle. If
fn, x, fp=k esa, AB || CD rFkk CD || EF. EA AB
the arm BC is extended to D point. So that BC =
;fn BEF = 55° rks x, y rFkk z dk eku gksxk 2 CD, then AD2 is equal to?

BLAM–361
T;kfefr
eku yhft, fd ABC ,d leckgq f=kHkqt gSA ;fn Hkqtk BC dks (1) PQ + QR = QS (2) PQ + QR < QS
D fcUnq rd c<+k;k tkrk gS ftlls fd BC = 2 CD, rks AD2 (3) PQ + PR > QS (4) PQ – QR > QS
fdlosQ rqY; gS\ 11. In the given figure, AB = 8.5 cm, BC = 6 cm and
(1) 3 CD2 (2) 4 CD2 CA = 7.2 cm. Then x°, y° and z° can be written as
(3) 5 CD2 (4) 7 CD2 fn, x, fp=k esa, AB = 8.5 lseh, BC = 6 lseh rFkk CA =
8. ABC is an isosceles right triangle. If r is in-radius 7.2 lseh rks x°, y° rFkk z° fy[ks tk ldrs gS
R
and R is circum-radius then =?
r A x°
ABC ,d lef¼ckgq ledks.kh; f=kHkqt gSA ;fn r mldh var%

cm

7.
2
5
R

cm
8.
f=kT;k rFkk R ifjf=kT;k gS] rks r fdlosQ cjkcj gS\

C
(1) 1 2 (2) 3 2 B 6 cm z°
5
(3) (4) 3 2 (1) x° < y° < z° (2) y° < x° < z°
2
(3) z° < x° < y° (4) z° < y° < x°
9. In the given figure, If TU || SR and TR || AS, then
12. S is any point on side QR of a PQR then which
a and b equals to
of them is correct
fn, x, fp=k eas, ;fn TU || SR vkSj TR || AS rks a rFkk
S, PQR ds Hkqtk QR ij dksbZ fcUnq gS rks buesa ls dkSu lgh
b cjkcj gS
gS?
(1) PQ + QR + RP > PS
V
(2) PQ + QR + RP > 2PS
25°
A (3) PQ + QR + RP > SP
(4) PQ + QR + RP < 2PS
a 13. ABCD is a square. M is the mid-point of AB and
U
PQ CM meets AD at P and CB produced at Q.
T 90° S Then which of them is correct
ABCD ,d oxZ gSA M, AB dk eè; fcUnq gS rFkk PQ CM
tks AD dks P ij feyrk gS rFkk c<+k, x, CB dks Q ij feyrk
R
gS rks buesa ls dkSu lgh gS?

50° b D C
P Q

(1) 40°, 115° (2) 115°, 40°


(3) 60°, 105° (4) 55°, 110°
P
10. In the given figure, T is a mid-point of side QR of
PQR and S is a point such that RT = ST. Then 1
which of them is correct 3
A B
M 4
fn, x, fp=k esa, T, PQR ds Hkqtk QR dk eè; fcUnq gS rFkk 2
S ,d ,slk fcUnq gS rkfd RT = ST rks buesa ls dkSu lk lgh Q
gS ?
S Q (1) PA = QB (2) PA < QB
(3) PA > QB (4) PA = 2BQ
14. In the given figure, AC is the median as well as
altitude to BD. In ACE, AD is the median to
T CE. Then which of the them is correct.
fn, x, fp=k eas] AC ekfè;dk gS] lkFk gh BD ij yEc gSA
ACE, esa AD, CE dk ekfè;dk gS rks buesa ls dkSu lgh gSA
R P

BLAM–362
T;kfefr
(1) 20.5 cm (2) 13 cm
A (3) 26 cm (4) 19.5 cm
18. AB is a diameter of the circle, CD is a chord equal
to the radii of the circle. AC and BD when
extended intersect at point E. AEB equals to
AB o`r dk O;kl gSA CD ,d thok gS tks o`r ds f=kT;k ds
cjkcj gSA tc AC vkSj BD dks c<+k;k tkrk gS rks fcUnq ij
B
C D
E izfrPNsn djrs gSaA AEB cjkcj gS
(1) 30° (2) 45°
(1) AB + CD > AE (2) AB + BC = AE (3) 60° (4) 65°
(3) AB + DE < AE (4) None of these 19. The radii of the two concentric spheres A and B
15. In the given figure, PQ > PR, QS and RS are the are respectively r and 2r. A cone drawn under B
bisectors of Q and R respectively, the find the is projected into A. What is the curved surface
correct relation between the sides SQ and SR will area of the cone?
be
nks laoZQUnzh xksyks A rFkk B dh f=kT;k,¡ Øe'k% r rFkk 2r gSA B
fn, x, fp=k, PQ > PR, QS vkSj RS Øe'k% Q rFkk R osQ varxZr ,d 'kaoqQ [khpk tkrk gS tks A esa ifjxr gSA 'kaoqQ dk
ds lef}Hkktd gSa rks Hkqtk SQ vkSj SR ds chp lgh laca/ oØ i`"Bh; {ks=kiQy fdruk gS\
gksxk
(1) 2 r 2 (2) 4 r 2
P (3) 6 r 2 (4) 8 r 2
20. The bisector of B of an isosceles triangle ABC
with AB = AC meets the circum circle of ABC at
P. If AP and BC produced meet at Q, then CQ
S equals to
54° ,d lef}ckgq f=kHkq t ABC ftleas AB = AC, B dk
lef}Hkktd ABC ds ifjo`r ls P ij feyrk gSa ;fn AP rFkk
Q R
BC dks c<+k;k tkrk gS rks Q ij feyrs gS rks CQ cjkcj gS
(1) SQ < SR (2) SQ > SR CA
(3) SQ = SR (4) None of these (1) CA (2)
2
16. Two circles whose centres are O and O' intersect
at P. Through P, a line l parallel to OO' intersecting 3
(3) 2CA (4) CA
the circles at C and D is drawn. The value of CD 2
equals to 21. ABCD is a rectangle of sides 36 cm and 18 cm
nks o`r ftuds dsanz O rFkk O' gSa] ,d nwljs dks P ij izfrPNsn respectively. E is the point on side CD such that
djrs gSA P ls gksdj ,d js[kk OO' ds lekukarj [khaph x;h gS DE 9 cm. a circle is drawn as show in figure.
tks o`rksa dks C rFkk D ij dkVrk gSA CD dk eku gS Find the radius of circle.
ABCD ,d vk;r gS ftldh Hkqtk,¡ Øe'k% 36 lseh- rFkk 18
C A P D lseh- gSaA E Hkqtk CD ij ,d fcUnq bl izdkj gS rkfd DE =
B 9 cm ,d o`Ùk [khapk x;k gS tSlk fd fp=k esa fn[kk;k x;k gS
O O'
o`Ùk dh f=kT;k Kkr djsa \

D E 9 cm C
(1) 2OO' (2) OO'
OO' 3
(3) (4) OO' 18 cm
2 2
17. If in a triangle ABC, BE and CH are perpendicular
to each other by two median lines and if the AB A B
36 cm
is now = 19 cm and AC = 22 cm then what’s the
length? (1) 4(15 (2) 60
2 15 ) 6 13
;fn f=kHkqt ABC esa] BE vkSj CH ,d nwljs osQ yEcor nks
ekfè;dk js[kk,¡ gS vkSj ;fn AB = 19 cm vkSj AC = 22 cm (3) 6(5 13 ) (4) 15 8 17
rks BC dh yEckbZ fdruh gS\
BLAM–363
T;kfefr
22. Points L, M and N divide the sides of triangle (1) 12.5 cm (2) 15 cm
PQR in ratio 2 : 7, 1 : 1 and 3 : 1 as shown in the
(3) 7.2 cm (4) 5 cm
figure. What fraction of the area of triangle PQR
is the area of triangle LMN and RMN? 26. A circle have three tangents, which given in the
figure and angle QPR is 50°, find the value of y?
fcUnq L, M rFkk N f=kHkqt PQR dh Hkqtkvksa dks 2 : 7, 1 : 1
rFkk 3 : 1 osQ vuqikr esa ck¡Vrh gS tSlk fd fp=k esa n'kkZ;k x;k ,d o`Ùk dh rhu Li'kZ js[kk,¡ gSa tks fp=k esa nh x;h gSa rFkk
gSA LMN rFkk RMN dk {ks=kiQy PQR ds {ks=kiQy dk QPR is 50° gS y dk eku Kkr djsa \
dkSu&lk Hkkx gS \
P
1 M 40°
2 N Q
L
3

O K 50° P
Q R
1 1 M y
(1) 8 : 3 (2) 9 : 5
(3) 72 : 13 (4) 72 : 31
23. If the length of two altitudes of a triangle is 8 R
and 12 cm, then which of the following cannot N
be the length of the third altitude?
;fn ,d f=kHkqt dh nks Å¡pkbZ;ksa dh yackbZ 8 lseh- rFkk 12 lseh-
(1) 40° (2) 50°
gSa rks fuEu esa ls dkSu rhljh Å¡pkbZ dh yackbZ ugha gks ldrh
(1) 11 (2) 15 (3) 30° (4) 15°
2
(3) 23 (4) 27 27. Sides of a triangle are (K + 2K + 2), (4K + 2) and
24. In the given figure PQR is a triangle, then finds (K3 – 2), then find the greatest value of angle in
its perimeter? triangle?
fn, x, fp=k esa PQR ,d f=kHkqt gS rks bldh ifjeki Kkr f=kHkqt dh Hkqtk,¡ (K2 + 2K + 2), (4K + 2) rFkk (K3 – 2) gSa
djs a \
rls f=kHkqt osQ lcls cM+s dks.k dk eku Kkr djsa \
P
(1) 90° (2) 110°
(3) 60° (4) 120°
14

112 28. In given figure PQR is right Angled.Right angled


6

at Q. A semicircle is drawn inside the triangle.


Find out the radius of semicircle?
nh x;h vkÑfr esa PQR ,d ledks.k f=kHkqt gS tks Q ij
ledks.k gSA f=kHkqt osQ vanj ,d v¼Zo`r [khapk x;k gSA v¼Zo`r
Q R
dh f=kT;k Kkr djsa \

2 18 Q
(1) 100 cm (2) 60 cm
(3) 50 cm (4) 80
25. Find the radius of semicircle?
v¼Zo`Ùk dh f=kT;k Kkr djsa \

P P R
4 9

41 cm 39 39
40 cm (1) (2) cm
97 97

36 35
Q R (3) (4) cm
9 cm 65 95

BLAM–364
T;kfefr
29. In the given figure find the radius of smaller circle? 31. Which of the following cannot be the area of
nh x;h vkÑfr esa NksVs o`Ùk dh f=kT;k Kkr djsa \ triangle which perimeter is 90 cm.
fuEufyf[kr esa ls dkSu f=kHkqt dk {ks=kiQy ugha gks ldrk ftldh
M ifjeki 90 cm gS
(1) 389.7 cm (2) 360 cm2
2
(3) 330 cm (4) 388 cm2
13 32. Find the area (in approx.) of triangle with altitudes
5
of lengths 18 cm, 21 cm and 35 cm.
f=kHkqt dk {ks=kiQy (yxHkx) Kkr djsa ftldh Å¡pkb;k¡ 18
N O cm, 21 cm rFkk 35 cm gSA
12
(1) 367.5 cm2 (2) 249.7 cm2
2
(3) 350 cm (4) 450 cm2
(54 4 26 ) 33. C and D diameter, are now points on circle
(1) cm
25 formed. ABD = 75° and DAC = 35° BDC = ?
In figure
(27 2 26 )
(2) cm C vkSj D O;kl AB ij fu£er v¼Zo`r ij fcUnq gSA ABD =
25
75° vkSj DAC = 35° gSA BDC fdruk gS\ fp=k esa
(27 2 26 )
(3) cm C
25

(54 4 26 ) D
(4) cm
25
30. In the given figure PQRS is a parallelogram. Side 35° 75°
QP extended to A. B is a point side PS. RB is A
extended to meet QP at A. If BO = 15 cm, OA = 35
cm, then find AP (in cm) ? (1) 130° (2) 110°
(3) 90° (4) 100°
nh x;h vkÑfr esa] PQRS ,d lekarj prqHkqtZ gSA Hkqtk QP dks
34. A circle with radius 4 cm is placed in a right angle.
A rd c<+k;k x;k gS] B Hkqtk PS ij ,d fcUnq gSa RB dks c<+k;k
Another smaller circle is also placed as shown in
x;k gS tks QP ls A ij feyrk gSA ;fn BO = 15 lseh-] OA = figure. What is the radius of smaller circle?
35 lseh- rks AP (lseh- es)
a Kkr djas \ 4 cm f=kT;k okyk ,d o`Ùk ledks.k osQ vanj cuk gqvk gSa ,d
nwljk NksVk o`Ùk tSlkfd fp=k esa fn[kk;k x;k gS] Hkh cuk gqvk
A gSA NksVs o`Ùk dh f=kT;k D;k gS \

B 4 cm
P S

O
B O

Q R
(1) (12 8 2 ) cm

50 200 (2) (8 4 2 ) cm
(1) cm (2) cm
3 3
(3) (9 6 2 ) cm
190 100
(3) cm (4) cm (4) (7 2 2 ) cm
3 3

BLAM–365
T;kfefr
35. There are three circles with centers P, Q and R. PA vkSj PB fdlh o`Ùk dh Li'kZ js[kk,¡ gSa vkSj O ml o`Ùk dk
Each having a radius 16 cm. The three circles osaQnz gSA f=kT;k 5 lseh- vkSj PO dh yackbZ 13 lseh- gSA ;fn
intersect each other as shown in the figure below.
If AB = 3 cm, FE= 7 cm and CD = 10 cm. Find the M
perimeter of triangle formed by joining the centers f=kHkqt PAB dk {ks=kiQy M gS] rks dk eku Kkr dhft,A
15
of the three circles?
MN rFkk O osaQnz okys rhu o`Ùk gSaA izR;sd dh f=kT;k 16 cm 24 24
gSA rhuksa o`Ùk ,d nwljs dks izfrPNsn djrs gSa tSlk fd fp=k esa (1) (2)
13 13
fn[kk;k x;k gSA ;fn AB = 3 cm FE = 7 cm rFkk CD = 10
cm rks rhuksa o`Ùkksa osQ osQUnzksa dks tksM+us ls cus f=kHkqt dk ifjeki 12 12
(3) (4)
Kkr djsa 13 13
(SSC CHSL (10+2) Tier-I Exam. 17.03.2020)

M SHORT ANSWERS
C
B 1. (1) 2. (1) 3. (2) 4. (3) 5. (1) 6. (2)
A D 7. (4) 8. (1) 9. (2) 10. (3) 11. (4) 12. (2)
13. (1) 14. (1) 15. (2) 16. (1) 17. (2) 18. (3)
F O 19. (3) 20. (1) 21. (3) 22. (2) 23. (4) 24. (1)
N E
25. (3) 26. (4) 27. (4) 28. (2) 29. (4) 30. (2)
31. (1) 32. (1) 33. (1) 34. (1) 35. (3) 36. (2)
(1) 72 cm (2) 65 cm
37. (2)
(3) 76 cm (4) 82 cm
36. Calculate the area of shaded region?
Nk;kafdr Hkkx osQ {ks=kiQy dh x.kuk djsa \ EXPLANATIONS
D C 1. (1) ;gk¡] 2= 1 rFkk
3= 4
nksuksa dks add djus ij
2+ 3= 1+ 4
12
nksuksa rjiQ 30° add djus ij
2 + 3 + 30° = 1+ 4 + 30°
T = 30°
A B 2. (1) ge tkurs gSa fd Square dk izR;sd adjacent angle
18
90° gksrk gSA
(1) 30 (2 ) cm2 (2) 36 (3 ) cm2 ADC = BCD ...(i)
(3) 40 (5 ) cm2 (4) 30 ( 4 ) cm2 Also 3= 4 ...(ii)
eq. (ii) dks eq. (i) ls ?kVkus ij
37. ADC – 3 = BCD – 4
ADC – 3 = BCD – 3
A
EDC = ECD
O ED = EC (Equal anglesds lkeus okyh sides
cjkcj gksrh gSaA)
P
B 3. (2) ;gk¡, AB || CD rFkk BD transversal gSA
PA and PB are tangents to the circle and O is the ABD + CDB = 180° (Co-interior angles)
centre of the circle. The radius is 5 cm and PO is y + 2 y + y + 5 y = 180°
13 cm. If the area of the triangle PAB is M, then 9 y = 180°
y = 20°
M
the value of
15
is y dh value gksxh 20°.

BLAM–366
T;kfefr
4. (3) ;gk¡, CD || EF rFkk EP transversal gSA XZY = 64°
EPD + FEP = 180° OY rFkk OZ, XYZ rFkk XZY ds angle bisector gSaA
y + 55° = 180° 1
y = 125° OZY = × 64° = 32°
2
Now, AB || CD rFkk CD || EF ge tkurs gSa
AB || EF
EAB + FEA =180° 1
YOZ = 90° + YXZ
(  Transversal ds ,d gh vksj ds interior angle dk 2
sum 180° gksrk gSA) 1
= 90° +
× 62°
90° + z + 55° = 180° (  EA AB EAB 2
= 90°)
= 90° + 31°
z + 145° = 180° YOZ = 121°
z = 35° OZY = 32° & YOZ = 121°
Also, AB || CD rFkk BP transversal gSA 7. (4)
x=y
x = 125°
A
Hence, x = 125°, y = 125° and z = 35°.
5. (1) AB || CD ds parallel ,d line cukb, EF

A B

132°
148° F
E

C x B M C D
D

AB||EF rFkk AE transversal gSA Let AB = BC = CA = x


BAE + AEF = 180° x
132° + AEF = 180° then, BM = MC =
2
AEF = 48° where, AM is median
Now, FEC = AEC – AEF
= 148° – 48° x
and CD =
= 100° 2
EF || CD rFkk EC transversal gSA Now AMC, AM2 = AC2 – MC2 [by Pythagorous]
FEC = x = 100° x2 3x 2
6. (2) ;gk¡, X = 62° and XYZ = 54° = x2 –
4 4
AMD, AD2 = AM2 + MD2
X
3x 2
x2
7x 2
7
FG x IJ
2
FG
7CD 2  CD
x IJ
62° =
4 4 H 4K H 2 K
8. (1)
O
54° A
Y Z

XYZ esa ,
XYZ + YXZ + XZY r
54° + 62° + XZY = 180°
B C
XZY = 180° – 116°

BLAM–367
T;kfefr
Let AB = BC = x SRQ = RPQ + RQP
 ABC is Isscdeles right angle 90° = 50° + b
b = 40°
AC AB 2 BC 2 Also TR || AS
= UTR = VAS
x2 x2 x 2
VAU = 90°
S = semi-peremetre of ABC
In VAU,
VUT = UVA + VAU
=
x x 2x
=
2x 2x 2x 1 e 2 j a = 25° + 90°
2 2 2
a = 115°
1 x2 a = 115°, b = 40°
and area of ABC = x x 10. (3) In PQR,
2 2
PQ + PR > QR
x2
S Q
2 x
r =
5
2x
e 1 2 j e1 2 j 2
2 T

abc
and R R P
4D
( fdlh triangle ds nks sides dk sum rhljh side ls
x x
=x×x× 2 cM+k gksrk gSA)
2
PQ + PR > QT + RT
PQ + PR > QT + ST (As RT = ST given)
R x 2
Ratio =
r x
1 2 QST, esa QT + ST > QS ...(ii)

e1 2 j 2 equation (i) rFkk (ii) ls


PQ + PR > QS

9. (2) ;g fn;k gqvk gS, 11. (4) ;gk¡,


A x°
cm

7.
2

V
5

cm
8.

25°
A

a z° C
B 6 cm
U

T 90° S AB = 8.5 cm,


BC = 6 cm rFkk
CA = 7.2 cm
R AB > CA > BC
C> B> A
(cM+s sideokys ds opposite
50° b
P Q okyk angle cM+k gksrk gSA)
izR;sd term dks (–1) ls multiply djus ij
TU || RS – C<– B<– A
UTR = SRQ izR;sd term esa 180° tksMu+ s ij
SRQ = 90° (180° – C) < (180° – B) <180° – A)
In RPQ, z° < y° < x°

BLAM–368
T;kfefr
12. (2) PQR, esa] QR. ij S dksbZ point gS 1 1
PRQ > PQR
2 2
P
SRQ > SQR
(QS rFkk RS Q rFkk R ds bisector gSa)
SQ > SR

Q 16. (1) As, OA "


R
S
C A P D "
PQ + QS > PS ...(i) B
O
(Triangle ds nks sides dk sum rhljs side ls cM+k gksrk gSA) O'
Similarly, in PRS,
SR + RP > PS ...(ii) OA CP
equations (i) rFkk (ii) dks add djus ij CA = AP
PQ + QS + SR + RP > 2PS CP = 2AP also ...(i)
PQ + (QS + SR) + RP > 2PS O'B "
PQ + QR + RP > 2PS
O'B DP
13. (1) M is mid point of AB. D C BP = BD ...(ii)
In PAM & QBM
PD = 2PB
AM = MB
CD = CP + PD
PAM = QBM
P CD = 2AP + 2PB
(90° each)
1 CD = 2(AP + PB) = 2AB = 2OO'
rFkk 3 = 4 A 3 B 17. (2)
M 42
(Vertically opposite angles)
Q
PAM QBM (ASA cong) A
PM = QM and PA = QB
14. (1) fdlh Triangle esa Median vkSj altitude leku gSa rks
ckdh nks Hkqtk,¡ cjkcj gksrh gSA cM+s angle ds lkeus okyk side F E
cM+k gksrk gS

B C

 AB2 + AC2 = 5BC2


B E (19)2 + (22)2 = 5BC2
C D

then, 845
BC = = 13 cm
AB = AD 5
In ADE, 18. (3) In OCD, we have
AD + DE > AE OC = OD = CD
As, AD is median, then CD = DE
AB + CD > AE P E
15. (2) ;fn fn;k gqvk gS
PQ > PR C D
PRQ > PQR S
(cM+s side okys ds
A B
opposite okyk angle Q R O
cM+k gksrk gS)

BLAM–369
T;kfefr
OCD ,d equilateral triangle gSA ACB = AQC + QAC
COD =60°, ( fdlh triangle esa exterior angle lkeus ds interor
1 angle ds sum ds cjkcj gksrk gSA)
We have, CBD = COD
D ABC = AQC + QAC [  AB = AC
2
ACB = ABC]
CBD 30 2 PBC = AQC + PBC
PBC = AQC
ACB semi-circle dk dks.k gSA
[ PBC = PAC (angles in the same
ACB = 90°
segment)]
BCE = 180° – ACB = 180° – 90° = 90°
PAC = AQC
BCE esa] QAC = AQC
BCE = 90° rFkk CBE = CBD = 30° CQ = CA
BCE + CBE + CEB = 180°
90° + 30° + CEB = 180° 21. (3) M
CEB = 60°

AEB 60 D E 9 cm C
19. (3) r and 2r = radii
18 cm
B

2r A B
36 cm
o r
M common gS
C B= C
A
MCE ~ MBA

EC MC EM
height of cone = OA + OB AB MB MA
= r + 2r = 3r
9 MC
Radious of cone = OC 2
OA 2 36 MC + 18
MC + 18 = 4MC
= 4r 2 r2 = 3r
MC = 6 cm.
l = 9r 2 3r 2 AM2 = AB2 + BM2
= (36)2 + (24)2
= 12r 2 = 2 3r = 1872
area of cone = 3r 2 3r 6 r2 AM = 12 13
20. (1) In AQC, we have
36 24 12 13
A radius =
2

= 30 6 13

= 6(5 13 ) cm
P
1
B 2 1 sin P
Area of PLN 2 1
22. (2) Area of PQR 1 18
9 4 sin P
C 2
Q

BLAM–370
T;kfefr
By tangent,
1
1 7 sin Q
Area of Q L 2 7 2 ab 6 14
Area of PQR 1 18
2 9 sin Q
2 2 ac 2 18

2 bc 112
1
1 3 sin R ab = 126 9 × 14
Area of R N 2 3
Area of PQR 1 18 ac = 18 9×2
2 4 sin R
2 bc = 28 14 × 2
a=9
Let, Area of PQR = 72 (LCM of 18, 18, 8)
b = 14
1 c=2
Area of PLN = × 72 = 4
18 PQ = 46 cm.
QR = 22 cm.
7
Area of QML = × 72 = 28 RP = 32 cm.
18
Triangle dk Perimeter = 46 + 22 + 32
3 = 100 cm
Area of RMN = × 72 = 27
8
Area of LMN = 72 – (4 + 28 + 27) 25. (3) P
= 13
Required ratio = 72 : (13 + 27)
= 72 : 40
41 41
=9:5
23. (4) Let h1, h2, h3 be the altitudes of the triangle 40
for, any triangle, the length of altitude is
R' R
h1h 2 h1h 2 Q
h3 , 9 9
h1 h 2 h1 h 2

12 8 12 18
h3 r
12 8 12 – 8 s

12 8 12 18 1
h3 40 18
120 4 2
4.8 < h3 < 24 41 41 18
2
the length of the third altitude will always be
greater then 4.8 and loss than 24.
20 18 2
27 cannot be the length of the third altitude. r=
100
24. (1)
= 7.2 cm
P
26. (4)
b
14

2b

b 112 M 40°
6

c °
50 S
a

50°
c O K 50° P
y
a

y S
Q 2a 2c R
R
N P
2 18

BLAM–371
T;kfefr
OMQ & OKQ are congruent triangle.
r
Similar, sin = ...(i)
4
ONR & OKR are congruent triangle
RNO esa]
quadrilateral MONP esa,
M + O + N + P = 360° r
sin (90 – ) =
90° + 50° + 50° + 2y = 90° + 50° = 360° 9
2y = 360° – 330°
r
cos = ...(ii)
30 9
y
2
equation (i) & (ii) ls]
y = 15°
27. (4) Let, K = 2 r2 r2
sin2 + cos2 =
Sides K3 + 2K + 2 = 14 16 81
4K + 2 = 10
K3 – 2 = 6 8r 2 16r 2
1=
16 81
M
16 81
r2 =
10 6 97

36
N O r= cm
97
14

By cos Rule, 29. (4) M

102 6 2 14 2
cos M =
2 10 6
13
136 196 5 R P
=
2 10 6
2
r
60 N O
= R 10T
120 12

1
2 5 12 13
R=
2
cos M = cos 120°
R = 2 cm
M 120 PO2 = PR2 + RO2 = 4 + 100

PO = 104 = 2 26 cm.
28. (2) Q
PRO & QTO are similar triangles.

QT QO OT
M N PR PO OR

r r r OQ
90– 2 2 26
P R
4 O 9 OQ = r 26

 P+Q=2+r
PMO esa]
2 + r + r 26 = 2 26

BLAM–372
T;kfefr
 3 units (15 + 35)
r( 26 1) 2( 26 1)
50
1 unit
2( 26 1) ( 26 1) 3
r=
( 26 1) ( 26 1)
50
4 unit ×4
2(26 1 2 26 ) 3
25
200
= cm
3
2(27 2 26 )
=
25
31. (1) P
(54 4 26 )
= cm
25 r q

Q R
30. (2) A p

 p + q + r = 90 cm.
We know that equilateral triangle has maximum
area.
3p = 90
p = 30 cm
B
P S
15 3
Area = (side)2
O 4

35 3
= × 30 × 30
Q R 4

= 225 3
BOS ~ ROQ
= 222 × (1.732)
BS OB = 389.7 cm2
QR OR 32. (1) Let length of altitudes h1, h2 & h3.
BS 15 3 h = 18 cm, h2 = 21 cm, h3 = 35 cm
QR 35 7 1 F1 1 1 IF 1 1 1 IF1 1 1 I F1 1 1 I
PB = PS – BS
GH h
1 h2 h3
JK GH h1 h2 h3
JK GH h
1 h2 h3
JK GH h
1

h 2 h3
JK
=7–3
= 4 units 1 FG 1 1 1 IJ FG 1 1 1 IJ FG 1 1 1 IJ FG 1 1

1 IJ
H 18 21 35 KH 18 21 35 K H 18 21 35 K H 18 21 35 K
A
1 FG 35 30 18 IJ FG 35 30 18 IJ FG 35 – 30 18 IJ FG 35 30 – 18 IJ
H 630 KH 630 K H 630 K H 630 K
4
1 83 13 23 47 630 630
4 =
P B 7 630 630 630 630 1080
= 367.5 cm2
3
33. (1) External angle AOC = 360° – 100° = 260°

1 1
Q R ABC = AOC = 260 130
7 2 2

BLAM–373
T;kfefr
34. (1) ekuk smaller circle dk radius = r. We know that Area (1) equals to Area (2).
A 1
Area of ABC = × 18 × 12
2
= 108 cm2
Shaded region dk area = 108 – (Area of circle)
O' 4 cm
= 108 – (6)2
4 cm
4 cm D = 108 – 36
r
P
r 2 = 36(3 – ) cm2.
B r
r O
OO’ = O’D + DP + PO A
4 2 4 r r 2 12
5
4( 2 1) r( 2 1) 37. (2) O P
M 13
5
4( 2 1) ( 2 1) 12
r B
( 2 1) ( 2 1)

Given,
4(2 1 2 2 )
r= OA = 5 cm and OP = 13 cm
2 1
AP = 12 cm [By Phythagorean Triplet 5, 12, 13]
r = (12 8 2 ) cm In OA P,
35. (3) ge tkurs gSa fd 1 1
MA = 16 cm AP OA OP AM
2 2
MB = MA – AB
= 16 – 3 12 × 5 = 13 × AM
= 13 cm
60
blh izdkj] AM =
13
cm
AN = 13 cm.
NE = 16 cm 60 120
NF = NE – FE AB = 2 cm
13 13
= 16 – 7 = 9 cm
In ABP (Isosceles triangle)
blh izdkj]
EO = 9 cm 1
ar ( ABP) = a 4b 2 a2
OC = 16 cm 4
OD = OC – CD
= 16 – 10 = 6 cm
blh izdkj] M=
1 120
4 12 12
FG 120 IJ 2

MC = 6 cm
4 13 H 13 K
Perimeter of triangle = MB + AB + AN + NF + FE
+ EO + OD + DC + MC 1 120 14400
= 576
= 13 + 3 + 13 + 9 + 7 + 9 + 6 + 10 + 6 = 76 cm 4 13 169

36. (2) D C 1 120 288


=
2
4 13 13

12 1 120 288
M 4 13 13 24
.
1 15 15 13
A
18
B ‡‡‡

BLAM–374
{ks=kfefr

6 {ks=kfefr (MENSURATION)
Mensuration ({ks=kfefr) : Chapter esa ge fofHkUu 1
izdkj ds geometrical figures TkSls fd triangle, circle, Area base height
2
quadrilateral (Rectangle, Square, Parallelogram,
Rhombus, Trapezium etc.), Cube, Cuboid, Cylinder, vxj triangle right angled triangle ugha gS rks bldk
(ii)
Spherc, Cone, Frustum etc. ds ckjs esa study djrs gSA area ge Heron’s formulae ls fudkyrs gSa] tSls
Competitive exams esa iwN s tkus okys questions es a
Mensuration dk viuk ,d vyx egÙo gSA bl Chapter ls A= b gb
S S –a S –b S –c gb g
iwNs tkus okys fofHkUu izdkj ds questions dks è;ku esa j[krs gq,
a b c
Åij crk, x, lHkh geometrical figures dh properties tgk¡] S = gksxk rFkk a, b rFkk c sides dh lengths
2
rFkk formulae dks cgqr gh vklku rjhds ls explain fd;k x;k
gksaxhA
gSA Mensuration ds ckjs esa lqurs gh gekjs fnekx esa ;g terms
Classification of Triangles
tSls fd Perimeter, Circumference, Area rFkk Volume vk
tkrs gSaA lp iwNs rks Mensuration ds yxHkx lHkh iz'uks esa buds (a) Right angled Triangle(ledks .k f=kHkqt ) :
ckjs esa gh iwNk tkrk gSA Mensuration esa vxj vki full marks Triangle ftldk dksbZ ,d angle 90° dk gksA

score djuk pkgrs gSa rks vki dks mij fn, x, geometrical
figures ij based formulae ;kn gksus pkfg,A geus ;gk¡ ij lHkh
Hy
formulae rFkk Concepts dks cgqr gh ljy rjhds ls le>kus dk
Perpendicular
a po
ten
iw.kZ iz;kl fd;k gSA us
e
A. TRIANGLE (f=kHkqt) % ml geometrical figure dks
dgrs gS ftldh 3 sides gksrh gSA
Base
B

1
c h Area = b h
2

A C (b) Isosceles Triangle (lef}ckgq f=kHkqt) : Triangle


b
ftldh dksbZ nks sides equal gksaA
1. Perimeter (ifjfefr) : fdlh Hkh geometrical figure
dh lHkh sides dh dqy yEckbZ dks perimeter dgrs gSaA A
;gk¡ ij fn, x, 'ABC dk perimeter gksxk&
a a
b AB BC CA g
2. Sum of three angles of a triangle : fdlh Hkh
triangle ds rhuksa angles dk Sum 180° gksrk gS] A b A
A B C 180
Perimeter = 2a + b
3. Area ({ks=kiQy) : fdlh Hkh geometrical figure ds
1
}kjk ?ksjk x;k Hkkx mldk Area dgykrk gSA Area =
4
b e4a 2
– b2 j
Methods of Finding Area of a Triangle.
(i) vxj triangle ,d right angled triangle gks rks bldk 4a 2 b 2
area gksxk& height =
2

BLAM–375
{ks=kfefr
(c) Equilateral Triangle (leckgq f=kHkqt) % Triangle (c)

ftldh rhuks sides equal gksa rFkk izR;sd angle 60° dk gksA
A
A

a a a a
h

A a A
B C
3 2 4A K a
(a) A = a a
4 3

3a 2 4h 2
3
h= a
2 or a2 + a2 + a2 = 4h2
AB 2 + BC2 + AC2 = 4AD2
2h
a= a 2
3 or h .
3
where a = side
A = Area (d) Right isosceles Triangle (lef}ckgq ledks.k
h = height f=kHkqt) : ,slk isosceles triangle ftldh equal sides ds
P = perimeter chp dk angle 90° gksA
1
h2
B
A=
3 c=b

P
P = 3a a=
3
(b) A b C

Perimeter = (2 + 2 )b
A
1 2 1
Area = b (hypotenuse)2
2 4
h3 h2 = 2a2
h2
EXAMPLES

1 1 1
h1 Ex. 1. The sides of a triangle are in the ratio : : . If
2 3 4
B C the perimeter of the triangle is 52 cm, the length
of the smallest side is :
H = h1 + h2 + h3 1 1 1
,d f=kHkqt dh Hkqtk,a 2 : 3 : 4 ds vuqikr esa gSA ;fn f=kHkqt
1
A=
3
bh 1 h2 h3 g 2
dk ifjeki 52 lseh- gks rks lcls NksVh Hkqtk fdruh yEch
gksxh\
2
a= (h1 + h2 + h3) (1) 24 cm (2) 18 cm
3
(3) 12 cm (4) 9 cm

BLAM–376
{ks=kfefr

Sol. (3) Triangle dh sides dk ratio 1 : 1 : 1 = 6 : 4 : 3 Sol. (1) A


2 3 4
Ratio sum = 13
Length of smallest side F E

3 52 O
12 cm.
13 B C
D
Ex. 2. The sides of a triangle are 3 cm, 4 cm and 5
cm. The area (in cm2) of the triangle formed by
Let the side of the equilateral triangle be x cm.
joining the mid points of this triangle is :
and OD = 5 OE = 4 OF = 3
fdlh f=kHkqt dh Hkqtk,¡ 3 lseh-] 4 lseh- vkSj 5 lseh- gSa A
bl f=kHkqt dh Hkqtkvksa osQ eè;fcUnqvksa dks feykus ls cus AOB + BOC + COA = ABC
f=kHkqt dk {ks=kiQy (lseh-2 esa) gSµ
1 1 1 3 2
x u3 u x u 4  u x u 5 = x
3 3 2 2 2 4
(1) 6 (2) 3 (3) (4)
2 4
3 24
6= x Ÿx 8 3
Sol. (3) 4 3

3
Area of ABC = u side2
4

3
= u 8 3 u 8 3 = 48 3 sq. cm
3 4 5 4
S= 6 cm
2
2nd Method
Area of greater Here, h1 = 3, h2 = 4, h3 = 5

b gb gb g
6 6 3 6  4 6 5 1
b3 4 5g 2
area of =
3
6 u 3 u 2 u1 36 6 sq. cm
Area of smaller 1 3
= 144 = 48 3 cm 2
6 3 3 3
sq. cm
4 2 Ex. 4.If the numerical value of the perimeter of an
( No. of smaller = 4)
equilateral triangle is 3 times the area of it,
Ex. 3. Length of the perpendiculars from a point in
the interior of an equilateral triangle on its sides then the length of each side of the triangle is
are 3 cm, 4 cm and 5 cm. Area of the triangle is ;fn fdlh leckgq f=kHkqt ds ifjeki dk la[;kRed eku mlds
fdlh leckgq f=kHkqt ds var% {ks=k esa fLFkr ,d fcUnq ls mldh {ks=kiQy dk xquk gS] rks f=kHkqt dh gj Hkqtk dh yackbZ gSµ
3
Hkqtkvksa ij Mkys x, yEcksa dh yEckb;k¡ 3] lseh] 4 lseh rFkk 5
(1) 2 units (2) 3 units
lseh gSaA f=kHkqt dk {ks=kiQy gksxkµ
(3) 4 units (4) 6 units
(1) 48 3 cm2
Sol. (3) If the side of the equilateral triangle be x units,
(2) 54 3 cm2 then,
(3) 72 3 cm2
F 3x I 2 3x 2
(4) 80 3 cm2
3x 3 GH 4 JK 3x
4
x = 4 unit

BLAM–377
{ks=kfefr
Ex. 5. The altitude drawn to the base of an isosceles
triangle is 8 cm and its perimeter is 64 cm. The
area (in cm2) of the triangle is
,d lef}ckgq f=kHkqt ij 'kh"kZ ls vkèkkj rd [khapk x;k yac 8
lseh gS vkSj mldk ifjeki 64 lseh gSA rnkuqlkj] ml f=kHkqt dk
{ks=kiQy fdrus lseh2 gS\
(1) 240 (2) 180 (3) 360 (4) 120
(i) Perimeter = a + b + c + d

Sol. (4) A P a +b + c + d
(ii) S = =
2 2
8
x cm x (iii) Area = b gb gb
S S –a S –b S –c S –d gb g
(iv) Area = b
4 d1d 2 g – eb
2 2
d2 – a2 – c2 j
B y D y C
Classification of Quadrilaterals.
(a) Rectangle (vk;r) : ,d ,slk Quadrilateral ftlosQ
AB = AC = x
opposite sides parallel rFkk equal gksrs gSA Rectangle dk
BD = DC = y
izR;sd angle 90° dk gksrk gSA blds diagonals ,d nwljs dks
AD2 = x2 – y2 x2 – y2 = 82 = 64 ...(i)
bisect djrs gSaA
Now x + x + 2y = 64 (Perimeter)
Note : Rectangle ds diagonals 90 ° ij intersect ugha
2x + 2y = 64 x + y = 32 ...(ii)
djrs gSaA
(i) ÷ (ii) ls]

x2 y2 64
x y 32

x–y=2
x + y = 32
x–y=2
P
(i) perimeter = 2 ( l + b) 1+b=
2x = 34 2
x = 17 cm (ii) Area = l × b
x + y = 32 (iii) diagonal = l2 b2
y = 32 – 17 = 15 cm
(iv) ;fn fdlh rectangle dh length dks x% Increase

ABC =
1
× BC × AD
rFkk breadth dks y% Increase fd;k tk;s rks mlosQ Area esa gqvk
2
FG
ifjorZu ¾ x y 100 %
xy IJ
1 H K
= 30 8 = 120 sq. cm
2 For Increase +
For Decrease –
B. Quadrilateral (prqHkZt q ) : dksbZ Hkh geometrical
(v) ;fn fdlh Rectangle osQ vanj ;k ckgj ;k length and
figure ftlosQ 4 sides gksrs gSaA
breadth osQ parallel a pkSM+kbZ dk jkLrk gks rksµ

BLAM–378
{ks=kfefr
(a) ckgj ls vanj dh vksjµ (b) Square (oxZ): Quadrilateral ftlds pkjks sides
jkLrk dk Area = 2d (l + b – 2d) equal gksa rFkk izR;sd angle 90° dk gks square dgykrk gSA
A D
E d H

b d d

(i) Perimeter = 4a
(ii) Area = a2

(iii) diagonal = 2a
F d G
B C P2 d2
l (iv) Area (A) = a2 =
16 2
(b) vanj ls ckgj dh vksjµ
where
jkLrk dk Area = 2d (1 + b + 2d)
A = Area
a = side

E H P = perimeter
d = diagonal
d
A D (v) tc fdlh Square osQ Side dks x% Increase fd;k
FG
tkrk gS rc mlosQ Area esa gqvk ifjorZu = 2x 100 %
x2 IJ
d d H K
(vi) nks oxZ (Square) esaµ
B d C a1 P1 d1 A1
a2 P2 d2 A2
F G
where
a1
(c) ;fn jkLrk length & breadth osQ Parallel gksµ a 2 = the ratio of sides

P1
d P2 = the ratio of Perimeters

d1
d 2 = the ratio of diagonals

A1
d d A 2 = the ratio of areas

(c) Parallelogram (lekarj prqHkZtq ) : Quadrilateral


ftlosQ opposite sides equal rFkk parallel gksrs gSA

jkLrk dk Area = d (l + b – d)
BLAM–379
{ks=kfefr
(i) Perimeter = 2 ( l + b)
1
(ii) Area = Base × €Height = l × h Area =
2
ba b g h
(d) Rhombus: Parallelogram ftlosQ pkjksa sides cjkcj
Circle (o`Ùk) : ,d ,slk geometrical figure ftlds
gksrs gS rFkk diagonals ,d nwljs dks 90° ij dkgrs gSaA
centre ls circumference (ifjf/) dh nwjh leku jgrh gSA
B a C

a
a

A a D

1 (i) Circumference = 2Sr


(i) Area = d .d
2 1 2 (ii) Area = Sr2
(iii) ;fn fdlh Circle osQ Radius dks x% Increase fd;k
P 1
(ii) a =
4 2
bd g bd g
1
2
2
2

FG x2 IJ
tk;s rks mldk Area esa gqvk ifjorZu ¾ 2x 100 % H K
(iii) P = 4a = 2 d1 b g bd g 2
2
2

(iv) nks Circle esaµ


d1 d2
A1 Fr I Fd I Fc I
2 2 2

(iv) H = bd g bd g
1
2
2
2
A2
GH r JK GH d JK GH c JK
1

2
1

2
1

(v)
A1
A a D A 2 = the ratio of areas,

r1
a a r2 = the ratio of radius,

60° 120°
d1
B a C d 2 = the ratio of diameters,

where a = 60° or 120° c1


small diagonal (AC) = a c 2 = the ratio of circumferences,

large diagonal (BD) = 3a V. v¼Zo`Ùk (Semi-Circle) esa


2
Area of Rhombus = a sin
(e) Trapezium (leyac prqHkqt Z ) : Quadrilateral
ftlosQ pair of opposite sides parallel rFkk unequal gksrs gSa
trapezium dgykrk gSA

B b C
r r
c d
h 36
Circumference (ifjf/) = r + 2r = 7 r
A a D
r2
Area =
Perimeter = a + b + c + d 2

BLAM–380
{ks=kfefr
(a) Arc (pki) : Circumference ds ,d fgLls dks arc
;k (pki) dgrs gSA

For equilateral triangle

a
Radius, r =
2 r 3
length of arc = = × circumference.
360 360q

(b) Sector ([kaM) : Circle ds arc vkSj nksuks radii ds Area = a2


3
chp ds Hkkx dks sector dgrs gSaA
Polygon (cgqHkqt) : Polygon ,d ,slk geometrical
(Radius Singular, Radii Plural
figure gksrk gS ftldh n – sides gksrh gSaA polygons generally
gksrs gS Pentagon, hexagon, etc.

r2 For regular polygon


Area of sector =
360
(i) T =
FG n – 2 IJ 180
Classification of circle (var% o`Ùk) H n K
(i) Incircle or Inscribed circle : (var% o`r) fdlh 1
Hkh triangle ds rhuks sides dks touch djrs gq, triangle ds (ii) Area = × n × r (r = inscribed circle dk radius)
2
vanj cuk;k x;k circle incircle dgykrk gSA
Rooms, Walls, Roof, Floor

1
(iii) n side okys Polygon dk Area = n × 2 side h

n
=
4
bsideg 2
tan
2

n 180
=
4
bsideg 2
cot
n
Equilateral Triangle ds fy,
[a length of the side of equilateral ] 3 3
(iv) Area of hexagon (le"kV~Hkqt) =
2
bsideg 2

a
Radius, r =
2 3
4 nhokj okys Room ij based questions vDlj exams
a2 esa iwNs tkrs gSA ;gk¡ ij bl ij based formulae fn, x, gSaA
Area =
12 (i) Area of floor = lb
(ii) Circum circle (ifjo`Ùk) : fdlh Hkh triangle ds (ii) Area of 4 walls = 2h (l + b)
rhukas vertex dks touch djrs gq, ckgj ls cuk;k x;k circle (iii) Area of 4 walls and floor = 2h (l + b) + lb
circum circle dgykrk gSA (iv) Area of 4 walls, floor and roof = [2h(l + b) + 2lb]

BLAM–381
{ks=kfefr
EXAMPLES rhljs square dk area = 64 cm2
Side of square = 64 8 cm
Ex. 1. The sides of a rectangular plot are in the ratio
5:4 and its area is equal to 500 sq.m. The perim- Perimeter of square
eter of the plot is : = (4 × 8) cm = 32 cm
,d vk;rkdkj IykWV dh Hkqtkvksa esa 5 : 4 dk vuqikr gS vkSj Ex. 3. The amount of rice produced in a square field of
bldk {ks=kiQy 500 oxZ ehú gS A IykWV dh ifjeki gSµ side 50 m is 750 kg. The amount of rice produced
in a square field of side 100 m will be
(1) 80m. (2) 100m.
50 ehVj Hkqtk okys ,d oxkZdkj [ksr esa 750 fdxzk pkoy dk
(3) 90m. (4) 95m.
mRiknu gksrk gSA 100 ehVj Hkqtk okys oxkZdkj [ksr esa mRiUu
Sol. (3) Let length = 5 x, width = 4 x
pkoy dh ek=kk fdruh gksxh \
area = (5x × 4 x) (1) 2000 kg
= 500 sq. m (2) 3000 kg
(3) 3500 kg
20x2 = 500
(4) 1500 kg
= x2 = 25
Sol. (2) 50 m side okys field dk area
x =5
= 50 × 50 = 2500 sq. metre
i.e. length = 5x = 25 m
100 m side okys field dk area = 100 × 100
rFkk width = 4x = 20 m
= 10000 sq. metre
Plot dk perimeter
 2500 sq. metre 750 kg.
= 2(l + b)
10000 sq. metre
= 2 × (20 + 25)
= 90 metre 750
× 10000 kg. = 3000 kg.
2500
Ex. 2. The perimeter of two squares are 40 cm and 24
cm. The perimeter of a third square , whose area Ex. 4. The number of paving stones each measuring
is equal to the difference of the areas of these 2.5m × 2m required to pave a rectangular court-
squares, is yard 30m long and 17.5 m wide, is
nks oxks± ds ifjeki 40 lseh rFkk 24 lseh gSaA ml rhljs oxZ] 30 eh-yacs vkSj 17.5 eh- pkSM+s vk;rkdkj izkax.k esa iRFkj fcNkus
ftldk {ks=kiQy bu nks oxks± ds {ks=kiQyksa ds vUrj ds cjkcj gS] ds fy, 2.5 eh- × 2 eh- eki ds fdrus iQ'khZ iRFkjksa dh
dk ifjeki gksxkµ vko';drk iM+xs h \
(1) 80 (2) 33 (3) 99 (4) 105
(1) 34 cm (2) 32 cm
(3) 38 cm (4) 30 cm Area of courtyard
Sol. (4) Number of paving stones =
Area of a stone
Sol. (2) igys square dh side

40 30 17.5
= 10 cm = = 105
4 2.5 2

nwljs square dh side Ex. 5.A street of width 10 metres surrounds from out-
side a rectangular garden whose measurement is
24 200 m × 180 m. The area of the path (in square
= 6 cm
4 metres) is

nksuksa ds areas dk varj 10 ehVj pkSM+kbZ dk ,d iryk jkLrk@iFk 200 eh- × 180 eh.

= (10 × 10 – 6 × 6) sq. cm ds eki okys ,d vk;rkdkj m|ku dks ckgj ls ?ksjrk gSA iFk dk
= (100 – 36) sq. cm {ks=kiQy (oxZ ehVj esa) gSµ
= 64 sq. cm (1) 8000 (2) 7000 (3) 7500 (4) 8200

BLAM–382
{ks=kfefr
Sol. (1) Ex. 7. The circumference of a circle is 100 cm. The side
200 m of an in square in it is —
,d o`Ùk dh ifjfèk 100 lseh gSA bl o`Ùk ds var% oxZ dh ,d
180 m
10 m Hkqtk dh eki gksxhµ

50 2
Area of garden without street (1) 25 2 cm (2) cm
= 200 × 180 = 36000 sq. metre
Area of garden with street 25 2
(3) 50 2 cm (4) cm
= (200 + 20) (180 + 20) = 220 × 200
= 44000 sq. m
Sol. (2)
path dk area
= 44000 – 36000
= 8000 sq. m
2nd Method
Area of the Path = 2d (t + b + 2d)
= (2 × 10) (200 + 180 + 2 × 10) Circumference of circle
= 20 × 400 = 8000 m2 × d = 100
Ex. 6 The perimeter of a triangle is 24 cm and the
circumference of its in-circle is 44 cm. Then the 100
d= cm
22
area of the triangle is (taking = ) diagonal of in square = diameter of circle
7
;fn ,d f=kHkqt dh ifjfefr 24 lseh gS vkSj var% o`Ùk dh ifjf/ 100
= cm
22
44 lseh gS] rks f=kHkqt dk {ks=kiQy D;k gS \ (eku ysa] = 7 ) One side of square

(1) 56 sq cm (2) 84 sq cm 1
= u diagonal
(3) 48 sq cm (4) 68 sq cm 2
Sol. (2)
A 1 100 0 2
= cm
2
F E Ex.8. The outer circumference of a circular race-track
O
is 528 metre. The track is everywhere 14 metre
wide. Cost of levelling the track at the rate of Rs.
10 per sq. metre is :
B D C
incircle dk centre O gS ,d o`Ùkkdkj nkSM+ iFk dh ckgjh ifjf/ 528 ehVj gSA iFk lCk
OD = OE = OF = r txg ls 14 ehVj pkSM+k gSA 10 #i, izfr oxZehVj dh nj ls iFk
2 r = 44 dks lery djus dh ykxr D;k gksxh\
(1) 77660 (2) 66760
22 44 7
2× × r = 44 r= = 7 cm (3) 76760 (4) 67760
7 2 22
Sol. (4) External radius of circular path = R metre
ABC dk area
2 R = 528
1
= (AB + BC + CA) × r 22 528 7
2 2 R = 528 R= = 84 m
7 2 22
1 In-radius (r) = 84 – 14 = 70 m
= × 24 × 7 = 84 cm2
2
path dk area

BLAM–383
{ks=kfefr
22 Sol. (2) Here, 32 + 42 = 52,
2 2
= (R – r ) = (842 – 702) right angled triangle
7
22
= (84 + 70) (84 – 70) A
7

22
= × 154 × 14 = 6776 cm2 D
7
0
Required cost = 6776 × 10 = 67760
Ex.9. A circle has been formed under an equilateral , B C
of side 8 m. What will be the area in place trian-
gle but circle is not included (approx.) ?
8 ehVj dh Hkqtk okys leHkqth; f=kHkqt eaas ,d o`Ùk cuk;k x;k 5
Radius of circum circle C2 = cm. = R
gSA f=kHkqt ds vanj o`Ùk osQ fcuk f?kjk gqvk vuqekfur {ks=kiQy 2
fdruk gksxk \ In–radis (r)
(1) 11 m2 (2) 20 m2
a b –c 3 4–5
(3) 22 m2 (4) 21 m2 = = = 1 cm
2 2
Sol. (1) inradius of circle
Area of C1
a 8 4
= = = m Area of C2
2 3 2 3 3

12 1 4
= = =
FG 5 IJ 2 25 25
H 2K 4

SURFACE AREAS AND VOLUMES


(3 – DIMENSIONAL FIGURES)

shaded portion dk area vc rd vkius mu lHkh geometrical figures ds ckjs esa tku
= Area of – area of circle fy;k gS ftUgsa ge 2 – Dimensional figure Hkh dgrs gSAa Generally
2 – D figures ls eryc og lHkh geometrical figures tks txg

=
3
×8×8–
F 4 IJ
× G
2

= 16 3 –
22
×
16 rks ?ksjrs gS ysfdu Volume ugha gksrk gSA ;gk¡ ij ge mu lHkh
4 H 3K 7 3
geometrical figures tSls dh (cube, cuboid, cone, cylinder,

22 sphere, hemisphere, frustum) ds ckjs esa i<sa+xs tks txg rks


= 16 × 1.732 – × 16 = 28 – 17 = 11 m2 ?ksjrs gSa rFkk lkFk gh lkFk budk Volume Hkh gksrk gSA Area dks
21
Ex.10. Let C1 and C2 be the inscribed and circumscribed measure djus dh unit, cm2, m2, (inch)2, (mm)2 etc. gksrh
circles of a triangle with sides 3cm, 4cm and 5cm gS rFkk Volume dks measure djus dh unit, cm3, m3 (inch)3,
area of C1
then area of C is (mm)3 etc. gksrh gSA
2
(1) Cuboid (?kukHk) : Cuboid 6 rectangular faces
eku ysa fd C1 vkSj C2 ,d f=kHkqt ds var% o`Ùk vkSj ifjo`Ùk gSa
ls feydj cuk gksrk gSA Opposite faces dh yackbZ cjkcj gksrh gSA
ftldh Hkq tk,¡ 3 ls eh-] 4 lse h-] vkS j 5 lse h- gS aA rks
C1 dk {ks=kiQy
C2 dk {ks=kiQy gS
9 4 9 16
(1) (2) (3) (4)
25 25 16 25

BLAM–384
{ks=kfefr
(i) Total Surface Area , A = 2 (lb + bh + hl) sq. Sol. (1) The internal dimensions of the box are :
units. (20–2) × (12–2) (10 – 2)
(ii) Volume V = l.b.h
= 18 × 10×8
(iii) Diagonal D = l 2
b 2
h 2 Volume of wood

(2) Cube (?ku) : Cuboid ftldh rhuks sides equal gks = 20 × 12 × 10 – 18 × 10 × 8


cube dgykrk gSA ;gk¡ ij l = b = h = a fy;k x;k gSAigure : = 2400 – 1440
= 960 cm3
D C Ex. 3. If the volume of two cubes are in the ratio 27 :
64, then the ratio of their total surface area is :
;fn nks ?kuksa ds vk;ruksa dk vuqikr 27 : 64 gS] rks muds lEiw.kZ
a i`"Bh; {ks=kiQyksa dk vuqikr gS&
H G (1) 27 : 64 (2) 3 : 4
(3) 9 : 16 (4) 3 : 8
(SSC CGL Prelim Exam. 24.02.2002 (First Sitting)
A B l 3
27
Sol. (3) l 3 64
[l and l1 are the sides of cubes
1
a
l 3
E a F l1 4

(i) Total surface area, A = 6a2 sq. units buosQ surface areas dk ratio
(ii) Volume, V = a3 cubic. units
6l 2 9
= 9 : 16
(iii) Body diagonal, D = a 3 units 6l12 16

(iv) Face diagonal = 2a units. Ex.4. Diagonal of a cube is 6 3 cm. Ratio of its total
surface area and volume (numerically) is
Example
,d ?ku dk fod.kZ 6 3 lseh gSA rnuqlkj] mlds dqy i`"Bh;
Ex.1. If the total surface area of a cube is 96 cm , its 2
{ks=kiQy rFkk vk;ru dk (la[;kRed) vuqikr fdruk gS\
volume is (1) 2 : 1 (2) 1 : 6
;fn ,d ?ku dk dqy i`"Bh; {ks=kiQy 96 oxZ lsaVhehVj gS] rks (3) 1 : 1 (4) 1 : 2
mldk vk;ru gSµ Sol. (3) cube dk diagonal = 6 3 cm
(1) 56 cm3 (2) 16 cm3
(3) 64 cm3 (4) 36 cm3 3 × edge
Sol. (3) ekuk cube dk fdukjk = x cm = 6 3 cm
Surface area, 6x2 = 96 Edge
96 = 6 cm = 6
x2 = 16 x = 16 4 cm
6 Total surface area : Volume
Volume of cube = (edge)3 = 6 × 62 : 63 = 1 : 1
= (4)3 = 64 cm3 Ex.5. Find the length of the longest rod that can be
Ex.2. A wooden box 20 cm × 12 cm × 10 cm. Thickness placed in a hall of 10 m length, 6 m breadth and
of wood is 1 cm. Volume of wood to make the box 4 m height.
(in cubic cm) is ml lcls yEch NM+ dh yEckbZ Kkr dhft, tks 10 ehñ
ydM+h osQ ,d cDls dh eki 20 lseh- × 12 lseh- × 10 lseh yEckbZ] 6 ehñ pkSM+kbZ vkSj 4 ehñ Å¡pkbZ okys ,d gkWy esa
gS A ydM+h dh eksVkbZ 1 lseh- gS A bl cDls dks cukus esa yxh j[kh tk ldrh gS\
ydM+h dk vk;ru (?ku lseh- esa) gSµ (1) 2 38 m (2) 4 38 m
(1) 960 (2) 519
(3) 2 19 m (4) 19 m
(3) 2400 (4) 1120

BLAM–385
{ks=kfefr
Ex. Cricket/ Tenis Ball, Foot Ball
Sol. (1) Required length = 102 62 42

152 2 38 m
(3) Cylinder (csyu) : fdlh Hkh rectangle dks mldh
,d side ds pkjks rjiQ ?kqekus ij izkIr 3–D figure cylinder
dgykrk gSA

B r C
r = sphere dk radius.
4 3
1. Volume = Sr
h 3
2. Surface Area = 4Sr2
3. nks xksys (Sphere) esa
A D
A1 F r I and V F r I
2 3

r = cylinderdh radius A2
GH r JK V GH r JK
1

2
1

2
1

2
h = cylinder dh height
1. Volume = Sr2h cubic units
V1 F A1 I 3

2. Curved surface Area = 2Srh sq. units.


V2 = GH A2 JK
3. Area of base = Area of top = Sr2
4. Total surface Area = 2Sr2 + 2Srh 4. tc rhu NksVs&NksVs xksys osQ ftudh f=kT;k (radius) r1, r2
= 2Sr(h + r) rFkk r3 gks] rks fi?kykdj cM+k xksyk cuk;k tk;s ftldh f=kT;k R gks
Hollow cylinder ([kks[kyk csyu) rks
v = h (R2 – r2) r1 r2 r3 R
A = 2 h (R – r) 3 4 5 6
S = 2 rh + 2 Rh + 2 (R2 – r2) 6 8 10 12

(5.) Hemisphere (v¼Zxksyk) : Sphere dks chp ls


B
R dkVus ij Hemisphere curk gSA

h r
2 3
(i) Volume = Sr
3
(ii) Curved surface area = 2Sr2 sq. units
(iii) Total surface area = 3Sr2 sq. units
r (6) Cone ('kao qQ) : fdlh Hkh right angled triangle dks
mlds height ds pkjks rjiQ ?kqekus ls tks 3 – D figure curk gS
mls Cone dgrs gSaA
where C
v = Volume
A = curved surfaces Area l
S = Total surface Area h
(4) Sphere (xksyk) : Circle ds 3-D Lo:i dks sphere
B r
dgrs gSaA A

BLAM–386
{ks=kfefr
r = radius of cone (1) 7 : 6 (2) 6 : 7 (3) 3 : 7 (4) 7 : 3
h = height of cone Sol. (4) Curved surface of cylindrical pillar
l = slant height = 2 rh = 264 m2
l= r2 h2

1 2
(i) Volume = Sr h cubic units
3

(ii) Curved Surface Area = Srl = Sr r 2 h 2 cubic


units
(iii) Area of base = Sr2 sq. unit
(iv) Total surface area = Sr (l + r) sq. units.
Volume = r2h = 924 m3
(7) Frustum of a cone ('kaoqQ dk fNUud) : ;fn
fdlh 'kadq dks blds base ds parallel dkVk tk, rks lower r2h 924
part dks frustum dgrs gSaA 2 rh 264
Ex. - ckYVh
924 2
r =7m
264

d 7u2 14 m 7
? =7:3
r h 264 u 7 6m 3
2 u 7 u 22
s
Frustum h
Ex.2. Two right circular cylinders of equal volume have
of the Cone their heights in the ratio 1 : 2. The ratio of their
R radii is :
leku vk;ru ds nks yEc o`Ùkh; csyuksa dh mQ¡pkb;ksa dk vuqikr
S = Slant height 1 : 2 gS A muds v¼ZO;klksa dk vuqikr gSµ
R = Radius of bigger part
(1) 2 :1 (2) 2 : 1
r = Radius of smaller part
h = Height of frustum (3) 1 : 2 (4) 1 : 4
Sol. (1) ekuk radi gksxh r1 rFkk r2
(i) Slant height = h 2
bR – r g 2

h rc r12h1 r2 h2 tgk¡ h1 rFkk h2 are heights


2

(ii) Volume = (R2 + r2 + Rr) CU. Units


3 According to question h1 : h 2 = 1 : 2
(iii) Slant surface area = S (R + r) s
(iv) Total surface area = S [(R2 + r2) + s (R+r)] sq. So] r1 : r2
units h 2 : h1 2: 1 2 :1

Example Ex.3. The height of a conical tank is 60 cm and the


diameter of its base is 64cm. The cost of painting
Ex.1. The curved surface area of a cylindrical pillar it from outside at the rate of 35 per sq. m. is :
is 264 m 2 and its volu me is 924 m 3 .
,d 'kaDokdkj Vadh dh mQWpkbZ 60 lsehú gS vkSj mlds vkèkkj
FG Taking 22IJ dk O;kl 64 lsehú gS A 35 #ú izfr oxZ ehú dh nj ls mls ckgj
H 7 K
. Find the ratio of its diameter
ls isUV djkus dk O;; D;k gksxk \
to its height.
(1) 52.00 approx.
,d csyukdkj LrEHk dh oØ i`"B {ks=kiQy 264 ehñ2 gS vkSj
(2) 39.20 approx.
22
mldk vk;ru 924 ehñ3 gSA 7
ekurs gq,] blds O;kl (3) 35.20 approx.
(4) 23.94 approx.
dk bldh Å¡pkbZ ls vuqikr Kkr dhft,\
BLAM–387
{ks=kfefr
(1) 2 : 3 (2) 4 :9
Sol. (4) Slant height l 602  322
(3) 8 : 27 (4) 64 : 729
3600  1024 Sol. (3) ekuk radii = r1, r2

4624 68 cm According to the question,

Area of the curved surface = rl 4 r12 4


4 r22 9
22
= u 32 u 68 cm3
7
r12 4
Required cost r22 9
22 u 32 u 68 u 35
r1 2
7 u 10000
r2 3
= 23.94
Ex.4. The slant height of a conical mountain is 2.5 km 4 3
r1
and the area of its base is 1.54 km2. Taking V1 3 r13
V2 4 3 r23
22 r2
, the height of the mountain is : 3
7

,d 'kaDokdkj ioZr dh fr;Zd Å¡pkbZ 2-5 fdeh gS vkSj mlds FG 2 IJ 3


8
= 8 : 27
22 H 3K 27
vkèkkj dk {ks=kiQy 1-54 fdeh-2 gSA ysrs gq, ioZr dh
7 2nd Method
Å¡pkbZ gS&
v1 F A1 I 3

(1) 2.2 km (2) 2.4 km


v2 GH A2 JK
(3) 3 km (4) 3.11 km

F 4I 3

FG 2 IJ 3
8
Sol. (2) = G 9J = .
H K H 3K 27

Ex.6. Three solid metallic balls of radii 3 cm, 4 cm


2.5 km
h and 5 cm are melted and moulded into a single
solid ball. The radius of the new ball is :
èkkrq dh rhu Bksl xasnks]a ftudh f=kT;k,¡ 3 lsñehñ] 4 lsñehñ
r vkSj 5 lsñehñ ga]S dks fi?kykdj ,d vU; Bksl xsna cukbZ tkrh
gSA bl ubZ xsna dh f=kT;k gS&
(1) 2 cm (2) 3 cm
r2 = 1.54 km2
(3) 4 cm (4) 6 cm
22 2
r = 1.54 4 4
7 Sol. (4) r13  r23  r33 R3
3 3
r = 0.7 km
(3)3 + (4)3 + (5)3 = R3
height of mountain 3
R = 27 + 64 + 125
=h= 2
(2.5) – (0.7 ) 2 = 216 = (6)3
km = 2.4 km R = 6 cm
2nd Method

Ex.5. The surface areas of two spheres are in the ra- R3 = r13 r23 r33
tio 4 : 9. Their volumes will be in the ratio = 33 + 43 + 53 = 63
nks xksyksa ds i`"Bh; {ks=kiQy 4 % 9 ds vuqikr esa gSA muds vk;ruksa R=6
dk vuqikr gksxk µ ‡‡‡

BLAM–388
{ks=kfefr

PREVIOUS YEARS’ QUESTIONS

Type-1 6. One acute angle of a right angled triangle is


Questions Based on Area and perimeter of double the other. If the length of its hypotenuse
is 10 cm, then its area is
Triangle, Quadrilaterals and Circle.
fdlh ledks.k f=kHkqt dk ,d U;wudks.k nwljs dk nqxquk gSA
1. The ratio of area of two triangles is 4 : 3 and ratio ;fn blds d.kZ dh yEckbZ 10 lseh gks] rks bldk {ks=kiQy
of their heights is 3 : 4. Find the ratio of their gks xkµ
base ?
25
nks f=kHkqtksa esa] {ks=kiQyksa dk vuqikr 4%3 gS vkSj Å¡pkbZ;ksa dk (1) 3 cm2 (2) 25 cm2
2
vuqikr 3% 4 gSA buds vkèkkjksa dk vuqikr Kkr dhft,\
75
(1) 16 % 9 (2) 9 % 16 (3) 9 % 12 (4) 16 % 12 (3) 25 3 cm2 (4) cm2
2
(SSC CGL Prelim Exam. 24.02.2002 (Second Sitting)
2. The base of a triangle is 15 cm and height is 12 (SSC CPO S.I. Exam. 09.11.2008)
cm. The height of another triangle of double the 7. The median of an equilateral triangle is 6 3 cm.
area having the base 20 cm is : The area (in cm2) of the triangle is
fdlh f=kHkqt dk vkèkkj 15 lseh- gSa vkSj Å¡pkbZ 12 lseh gSaA ,d
,d leckgq f=kHkqt dk ekfè;d 6 3 lseh- gSA ml f=kHkqt dk
vU; f=kHkqt dh Å¡pkbZ ftldk {ks=kiQy bl f=kHkqt ds {ks=kiQy dk
{ks=kiQy (lseh2 esa) gS
nqxquk gS vkSj vkèkkj 20 lseh- gS] fuEu gS&
(1) 72 (2) 108
(1) 9 cm (2) 18 cm
(3) 72 3 (4) 36 3
(3) 8 cm (4) 12.5 cm
(SSC CHSL DEO & LDC Exam. 02.11.2014 (IInd Sitting)
3. The area of a right-angled isosceles triangle having
8. If the diagonal and area of a rectangle are 25 cm
hypotenuse 16 2 cm is and 168 cm2 respectively, then what will be the
length ?
d.kZ 16 2 lseh okys ledks.k lef}ckgq f=kHkqt dk {ks=kiQy
;fn fdlh vk;r ds ,d fod.kZ rFkk {ks=kiQy dh eki Øe'k% 25
gks xkµ
lseh rFkk 168 lseh2 gks] rks vk;r dh yackbZ fdruh gksxh\
(1) 144 cm2 (2) 128 cm2
(1) 31 cm (2) 24 cm
(3) 112 cm2 (4) 110 cm2
(SSC (South Zone) Investigator Exam. 12.09.2010) (3) 17 cm (4) 7 cm
4. The ratio of sides which make right angle in right (SSC Graduate Level Tier-II Exam, 01.08.2010)
angled triangle is 5 : 12. If the area of triangle is 9. A hall 25 metres long and 15 metres broad is
270 cm2 what is the hypotenuse ? surrounded by a verandah of uniform width of
3.5 metres. The cost of flooring the verandah, at
fdlh ledks.k f=kHkqt dh ledks.k cukus okyh Hkqtk,¡ 5 : 12 ds
27.50 per square metre is
vuqikr esa gSaA ;fn f=kHkqt dk {ks=kiQy 270 lseh2 gS] rks d.kZ dh
25 ehVj yEch vkSj 15 ehVj pkSM+s ,d lHkkxkj ds pkjksa vksj
yEckbZ gksxh %
3-5 ehVj dh ,d leku pkSM+kbZ dk ,d cjkenk gSA ml cjkens
(1) 39 cm (2) 42 cm dk iQ'kZ 27.50 izfr oxZ ehVj dh nj ij rS;kj djus dh dqy
(3) 45 cm (4) 51 cm ykxr fdruh gksxh \
Lukrd Lrjh; Tier-II ijh{kk] 01-08-2010)
(SSC (1) 9149.50 (2) 8146.50
5. The length of three medians of a triangle are 9 cm, (3) 9047.50 (4) 4186.50
12 cm and 15 cm. The area (in sq. cm) of the (SSC GL Tier-I Exam. 11.11.2012 (Ist Sitting)
triangle is 10. The length and breadth of a rectangle are doubled.
,d f=kHkqt dh rhu ekfè;dkvksa dh yackb;k¡ 9 lseh] 12 lseh Percentage increase in area is
rFkk 15 lseh gSaA rnuqlkj ml f=kHkqt dk {ks=kiQy fdrus oxZ ,d vk;r dh yackbZ vkSj pkSM+kbZ nqxquh dj nh xbZ gSA {ks=kiQy
lseh gS\ esa izfr'kr o`f¼ gS
(1) 24 (2) 72 (3) 48 (4) 144 (1) 150% (2) 200% (3) 300% (4) 400%
(SSC Graduate Level Tier-II Exam.16.09.2012) (SSC CHSL DEO & LDC Exam. 20.10.2013)

BLAM–389
{ks=kfefr
11. The ratio between the length and the breadth of a 15. The diagonals of a rhombus are 32 cm and 24
rectangular park is 3 : 2. If a man cycling along cm respectively. The perimeter of the rhombus is:
the boundary of the park at the speed of 12 kmph ,d leprqHkZqt ds fod.kZ Øe'k% 32 lseh- vkSj 24 lseh-
completes one round in 8 minutes, then the area gSaA bl leprqHkqZt dh ifjeki gS&
of the park is equal to
(1) 80 cm (2) 72 cm
,d vk;rkdkj ikdZ dh yackbZ vkSj pkSM+kbZ ds chp vuqikr 3 :
(3) 68 cm (4) 64 cm
2 gSA ;fn dksbZ O;fDr 12 fdeh izfr ?kaVs dh xfr ls ikdZ dh
(SSC CGL Prelim Exam. 24.02.2002 (First Sitting)
ckmaMhª ds pkjksa vkSj lkbfdy pykdj 8 feuV esa ,d pDdj iwjk 16. The perimeter of a rhombus is 40 cm. If one of the
dj ysrk gS] rks ikdZ dk {ks=kiQy fdruk gS\ diagonals be 12cm long, what is the length of the
(1) 152600 m2 (2) 153500 m2 other diagonal ?
(3) 153600 m2 (4) 153800 m2 ,d leprqHkqZt dk ifjeki 40 lsehñ gSA ;fn nksuksa fod.kks± esa ls
(SSC CGL Tier-I Re-Exam, 30.08.2015) ,d dh yEckbZ 12 lsehñ gS] rks nwljs fod.kZ dh yEckbZ D;k gS\
12. A playground is in the shape of a rectangle. A
(1) 12 cm (2) 136 cm
sum of 1,000 was spent to make the ground
usable at the rate of 25 paise per sq. m. The (3) 16 cm (4) 44 cm
breadth of the ground is 50 m. If the length of
(SSC CGL Prelim Exam. 24.02.2002 (Middle Zone)
the ground is increased by 20 m, what will be
17. The sides of a quadrilateral are in the ratio 3 : 4 :
the expenditure (in rupees) at the same rate per
sq. m. ? 5 : 6 and its perimeter is 72 cm. The length of its
greatest side (in cm) is
,d [ksy dk eSnku] vk;rkdkj gSA ml eSnku dh Hkwfe dks
fdlh ,d prqHkqt Z dh Hkqtk,¡ 3 : 4 : 5 : 6 ds vuqikr esa gSa rFkk
bLrseky yk;d cukus osQ fy, 25 iSls izfr oxZ eh- dh nj ij
bldk ifjeki 72 lseh gSA bldh lcls cM+h Hkqtk dh yEckbZ
1,000 [kpZ fd, x,A ml eSnku dh pkSM+kbZ 50 eh- gSA ;fn
(lseh esa) gksxhµ
ml eSnku dh yackbZ 20 eh- c<+k nh tk,] rks izfr oxZ eh- dh
(1) 24 (2) 27 (3) 30 (4) 36
mlh nj ij] oqQy fdrus #i, [kpZ gksaxs\ (SSC (South Zone) Investigator Exam. 12.09.2010)
(1) 1,250 (2) 1,000 (3) 1,500 (4) 2,250 18. Sides of a parallelogram are in the ratio 5 : 4. Its
(SSC Graduate Level Tier-II Exam.16.09.2012) area is 1000 sq. units. Altitude on the greater
13. A took 15 sec. to cross a rectangular field side is 20 units. Altitude on the smaller side is
diagonally walking at the rate of 52 m/min. and
,d lekarj prqHkqt Z dh Hkqtkvksa dk vuqikr 5 : 4 gSA mldk
B took the same time to cross the same field along
its sides walking at the rate of 68 m/min. The {ks=kiQy 1000 oxZ bdkbZ gSA cM+h Hkqtk ij rqaxrk 20 bdkbZ gSA
area of the field is : NksVh Hkqtk ij rqaxrk gSµ
A fdlh vk;rkdkj eSnku dks mlosQ fod.kZ osQ vuqfn'k 52 (1) 30 units (2) 25 units
eh@feuV dh pky ls pydj 15 lsd.M esa ikj djrk gS rFkk (3) 10 units (4) 15 units
(SSC CHSL DEO & LDC Exam. 11.12.2011
B bl [ksr dks mldh Hkqtkvksa osQ vuqfn'k 68 eh@feuV dh (Ist Sitting (Delhi Zone)
pky ls pydj mrus gh le; esa ikj djrk gS A bl [ksr dk 19. The adjacent sides of a parallelogram are 36 cm
{ks=kiQy gSµ and 27 cm in length. If the distance between the
(1) 30 m2 (2) 40 m2 shorter sides is 12 cm, then the distance between
the longer sides is
(3) 50 m2 (4) 60 m2
(SSC CGL Prelim Exam. 11.05.2003 (First Sitting) ,d lekUrj prqHkqt Z dh fudVorhZ Hkqtk,¡ 36 lseh- rFkk 27 lseh
14. A path of uniform width runs round the inside of gSaA rnuqlkj ;fn ml prqHkqt Z dh NksVh Hkqtkvksa dh nwjh 12 lseh-
a rectangular field 38 m long and 32 m wide. If gks] rks cM+h Hkqtkvksa dh nwjh fdruh gksxh \
the path occupies 600m2, then the width of the
(1) 10 cm (2) 12 cm
path is
,d 38 eh- yEcs rFkk 32 eh- pkSM+s vk;rkdkj {ks=k ds vUnj (3) 16 cm (4) 9 cm
(SSC CHSL DEO & LDC Exam. 11.12.2011
dh vksj pkjksa rjiQ ,dleku pkSM+kbZ okyk ,d iFk cuk;k x;k (Ist Sitting (East Zone)
gSA ;fn iFk dk {ks=kiQy 600 eh-2 gks] rks iFk dh pkSM+kbZ gksxhµ 20. The ratio of the length of the parallel sides of a
(1) 30 m (2) 5 m trapezium is 3:2. The shortest distance between
(3) 18.75 (4) 10 m them is 15 cm. If the area of the trapezium is 450
(SSC CGL Prelim Exam. 04.02.2007 (First Sitting) cm2, the sum of the length of the parallel sides is

BLAM–390
{ks=kfefr
,d leyac prqHkqZt dh lekukarj Hkqtkvksa dh yackbZ dk vuqikr 25. The diagonal of a quadrilateral shaped field is 24m
3 : 2 gSA muds chp dh U;wure nwjh 15 lseh- gS rFkk leyac and the perpendiculars dropped on it from the
remaining opposite vertices are 8m and 13m. The
prqHkqt
Z dk {ks=kiQy 450 lseh2 gSA rnuqlkj lekukarj Hkqtkvksa dh area of the field is
yackbZ dk ;ksx fdruk gksxk\
prqHkqt Z vkdkj ds {ks=k dk fod.kZ 24 ehVj gS vkSj 'ks"k lEeq[k
(1) 15 cm (2) 36 cm
'kh"kks± ls cuk, x, yac 8 ehVj vkSj 13 ehVj gSaA {ks=k dk {ks=kiQy
(3) 42 cm (4) 60 cm fdruk gS \
(SSC MTS (Non-Technical) Exam. 27.02.2011) (1) 252 m2 (2) 156 m2
21. The area of a field in the shape of a trapezium (3) 96 m2 (4) 1152 m2
measures 1440 m2. The perpendicular distance (SSC CAPFs SI, CISF ASI & Delhi Police SI
between its parallel sides is 24 m. If the ratio of the Exam, 21.06.2015 (Ist Sitting) TF No. 8037731)
parallel sides is 5 : 3, the length of the longer parallel 26. The radius of a wheel is 21 cm. How many
side is : revolutions will it make in travelling 924 metres ?

leyEc vkdkj ds ,d {ks=k dk {ks=kiQy 1440 ehú2 gS A FG use 22 IJ


lekUrj Hkqtkvksa ds chp dh yEcr~ nwjh 24 ehú gS A ;fn H 7 K
lekUrj Hkqtkvksa dk vuqikr 5 : 3 gks] rks cM+h lekUrj Hkqtk fdlh ifg, dh f=kT;k 21 lsúehú gS A 924 ehVj dh nwjh r;
dh yEckbZ gksxh A djus esa bldks fdrus pDdj yxkus iM+xs sa \
(1) 75 m (2) 45 m F 22
dk I
izÕkksx dhft,A
(3) 120 m (4) 60 m H 7 K
(SSC CGL Prelim Exam. 08.02.2004 (First Sitting) (1) 7 (2) 11
22. The length of each side of a rhombus is equal to (3) 200 (4) 700
the length of the side of a square whose diagonal (SSC CGL Prelim Exam. 04.07.1999 (Second Sitting)

is 40 2 cm. If the length of the diagonals of the 27. The circle whose radius is 5 cm and arc is 3.5 cm,
then area of sector will be
rhombus are in the ratio 3 : 4, then its area (in
cm2) is fdlh o`Ùk ds ,d [kaM dk {ks=kiQy] ftldh f=kT;k 5 lsehú gS
,d leprqHkqZt dh izR;sd Hkqtk dh yackbZ ml oxZ dh Hkqtk dh rFkk tks ,d 3.5 lsehú yEch pki }kjk cuk gS] gksxk—
(1) 8.5 cm2 (2) 8.75 cm2
yackbZ ds cjkcj gS] ftldk fod.kZ 40 2 lseh gSA rn~uqlkj] 2
(3) 7.75 cm (4) 7.50 cm2
;fn ml leprqHkqZt ds fod.kks± dh yackb;k¡ 3 : 4 ds vuqikr esa (SSC CGL Prelim Exam. 04.07.1999 (First Sitting)
gksa] rks mldk {ks=kiQy fdrus lseh2 gksxk\ 28. Two circles are concentric whose circumference
are 88 cm and 132 cm respectively, Find the area
(1) 1550 (2) 1600 (3) 1535 (4) 1536
of ring ?
(SSC CHSL DEO & LDC Exam. 10.11.2013 (IInd Sitting)
23. The perimeter of a rhombus is 40 cm and the nks ladsUæ o`Ùkksa] ftudh ifjfèk;ka Øe'k% 88 lseh- vkSj 132
measure of an angle is 60°, then the area of it is : lseh- gSa] ds chp ds oy; dk {ks=kiQy Kkr dhft,A
,d leprqHkqZt dk ifjeki 40 lseh- gS vkSj ,d dks.k dk eki (1) 780 cm (2) 770 cm
60° gS] rks mldk {ks=kiQy gS % (3) 715 cm (4) 660 cm
(SSC CGL Prelim Exam. 27.02.2000 (Second Sitting)
2 2
(1) 100 3 cm (2) 50 3 cm 29. A path of uniform width surrounds a circular
park. The difference of internal and external
2
(3) 160 3 cm (4) 100 cm2 circumference of this circular path is 132 metres.
(SSC CHSL DEO & LDC Exam. 11.12.2011
(IInd Sitting (Delhi Zone) 22
Its width is : (Take S )
24. The area of a rhombus is 256 square cm. and one 7
of its diagonals is twice the other in length. Then fdlh o`Ùkkdkj ikoZQ osQ pkjksa vksj ,d leku pkSM+kbZ dk ,d
length of its larger diagonal is iFk cuk gqvk gS A bl o`Ùkkdkj iFk dh vkarfjd vkSj ckgjh
,d leprqHkZqt dk {ks=kiQy 256 oxZ lseh gS vkSj mldk ,d ifjfèk;ks a dk va r j 132 ehVj gS A mldh pkSM + k bZ gS µ
fod.kZ nwljs ls yackbZ esa nqxquk gS] rks mlds o`gÙkj fod.kZ dh yackbZ
fdruh gS \
FG 22
yhft,
IJ
H 7 K
(1) 32 cm (2) 16 cm
(1) 22m (2) 20 m
(3) 48 cm (4) 24 cm (3) 21m (4) 24m
(SSC CGL Tier-II Exam. 12.04.2015 TF No. 567 TL 9) (SSC CGL Prelim Exam. 11.05.2003 (First Sitting)

BLAM–391
{ks=kfefr
30. The ratio of the outer and the inner perimeter 34. Three circles of equal radius ‘a’ cm touch each
of a circular path is 23 : 22. If the path is 5 other. The area of the shaded region is :
metres wide, the diameter of the inner circle is : ‘a’ lseh ,dleku f=kT;k okys rhu o`Ùk ,d&nwljs dks Li'kZ dj
fdlh o`Ùkkdkj ekxZ dh cká rFkk vkUrfjd ifjekiksa dk vuqikr jgs gSaA rnuqlkj] Nk;kafdr {ks=k dk {ks=kiQy fdruk gSaA
23 : 22 gS A ;fn ekxZ dh pkSM+kbZ 5 ehVj gS] rks vkUrfjd
o`Ùk dk O;kl gksxk \
(1) 110 m (2) 55 m
(3) 220 m (4) 230 m
(SSC CGL Prelim Exam. 08.02.2004 (First Sitting)
31. The area of the shaded region in the figure given
below is
uhps nh xbZ vkÑfr esa Nk;kafdr {ks=k dk {ks=kiQy gSµ
B

F 3 I a sq.cm 2

A D
(1) GH 2 JK
m C

(2) G
F 6 3 – I a sq.cm 2
H 2 JK
(1)
a2 FG IJ
1 sq. units (3) e 3– ja sq.cm
2
2 2 H K
(2) a2 (S – 1) sq. units
F2 3
(4) G
I a sq.cm
2
(3) a2
FG 1IJ sq. units H 2 JK
H2 K
(SSC CAPFs SI & CISF ASI Exam. 23.06.2013)
2 35. Two equal maximum sized circular plates are cut
a
(4) (S – 1) sq. units off from a circular paper sheet of circumference
2
352 cm. Then the circumference of each circular
(SSC CGL Prelim Exam. 04.02.2007 (First Sitting)
plate is
32. If the radius of circle extended by 1 cm, the area
will increased 22 cm2 more. What is initial radius
nks leku vf/dre vkdkj dh o`Ùkkdkj IysVksa dks ,d o`Ùkkdkj
of circle ? dkxt dh 'khV ftldh ifjf/ 352 lseh gS] esa ls dkVk tkrk gSA
;fn fdlh o`Ùk dh f=kT;k esa 1 lseh dh o`f¼ dj nh tk, rks izR;sd o`Ùkh; IysV dh ifjf/ crk,¡A
mldk {ks=kiQy 22 lseh2 vfèkd gks tkrk gSA o`Ùk dh vkjafHkd (1) 176 cm (2) 150 cm
f=kT;k gSµ (3) 165 cm (4) 180 cm
(SSC CHSL DEO & LDC Exam. 16.11.2014)
(1) 6 cm (2) 3.2 cm 36. In the given figure, OED and OBA are the sector
(3) 3 cm (4) 3.5 cm of circle whose centre is O. Find the area of shaded
(SSC CGL Prelim Exam. 04.02.2007 (First Sitting) portion.
& (SSC CGL Tier-II Exam. 16.09.2012)
fuEufyf[kr vkÑfr esa] OED vkSj OBA dsanz O okys o`Ùk ds
33. Find the area of a sector in a circle which is made
{ks=k gSaA Nk;kafdr Hkkx dk {ks=kiQy crkb,A
FG 22 IJ
in 6 minutes by 7 cm long minute hand. H 7 K D

fdlh nhokj&?kM+h dh 7 lseh- yEch feuV dh lqbZ }kjk 6 feuV 4m A


22 O 45°
ds vUrjky esa laLif'kZr {ks=k dk {ks=kiQy gksxk [S = 7
yhft,]
3m B
(1) 154 cm2 (2) 92.4 cm2
(3) 15.4 cm2 (4) 4.4 cm2 E
(SSC SAS Exam, 27.06.2010)

BLAM–392
{ks=kfefr
11 2 11 2 W X
(1) m (2) m
2 16
P Q
Z Y
11 2 11 2
(3) m (4) m A B E F
8 4
(SSC CAPFs SI, CISF ASI & Delhi Police SI D C G
H
Exam, 21.06.2015 (IInd Sitting) TF No. 310524)
L M
37. A circular swimming pool is surrounded by a S R
concrete wall 4m wide. If the area of the concrete
11 O N
wall surrounding the pool is that of the pool, (1) 31.36 (2) 125.44
25
(3) 62.72 (4) 156.8
then the radius (in m) of the pool is : (SSC CGL Tier-II Exam. 17.02.2018)
,d orqZy fLofeax iqy 4 ehVj pkSM+h daØhV dh nhokj ls f?kjk 41. In the given figure, AB AE, EF, FG and GB are
gqvk gSA ;fn iqy dks ?ksjus okyh daØhV dh nhokj dk {ks=kiQy iqy semicircles. AB = 56 cm. and AE = EF = FG = GB.
What is the area (in cm2.)
11
dk 25 gS rks iqy dh f=kT;k (ehVj esa) fdruh gS\ of the shaded region?
nh xbZ vkÑfr esa] AB AE, EF, FG rFkk GB v¼Zo`Ùk gSaA AB
(1) 8 (2) 16 = 56 lseh- rFkk AE = EF = FG = GB gSaA Nk;kafdr Hkkx dk
(3) 30 (4) 20 {ks=kiQy (lseh-2 esa) D;k gS\
(SSC CGL Tier-I Exam, 16.08.2015
(IInd Sitting) TF No. 2176783)
38. A is the centre of circle whose radius is 8 and B is
the centre of a circle whose diameter is 8. If these
two circles touch externally, then the area of the
circle with diameter AB is
A, 8 bdkbZ f=kT;k okys o`Ùk dk osQUnz gS vkSj B, 8 bdkbZ O;kl
A E F G B
okys o`Ùk dk osQUnz gSA ;fn ;s nksuksa o`Ùk ckgj ls Li'kZ djrs gSa]
(1) 414.46 (2) 382.82
rks AB O;kl okys o`Ùk dk {ks=kiQy (oxZ bdkbZ eas) fdruk gS\ (3) 406.48 (4) 394.24
(1) 36 S (2) 64 S (SSC CGL Tier-II Exam. 17.02.2018)
(3) 144 S (4) 256 S 42. In the given figure. ABCD is a square. EFGH is a
(SSC CGL Tier-II Exam. 21.09.2014) square formed by joining mid points of sides of
39. The difference between the radii of the bigger circle ABCD. LMNO is a square formed by joining mid
points of sides of EFGH. A circle is inscribed inside
and smaller circle is 14 cm and the difference
EFGH. If area of circle is 38.5 cm2., then what is
between their areas is 1056 cm2. Radius of the
the area (in cm2.) of square ABCD?
smaller circle is
nh xbZ vkÑfr esa] ABCD ,d oxZ gSA ABCD dh Hkqtkvksa ds
cM+s o`Ùk vkSj NksVs o`Ùk dh f=kT;kvksa ds chp varj 14 lseh gS vkSj
dsUnz fcUnqvksa dks tksM+dj ,d oxZ EFGH cuk;k x;k gSA
muds {ks=kiQy ds chp varj 1056 lseh2 gSA NksVs o`Ùk dh
EFGH dh Hkqtkvksa ds dsUnz fcUnqvksa dks tksM+dj ,d oxZ
f=kT;k gSµ
LMNO cuk;k x;k gSA ,d o`Ùk dks oxZ EFGH esa vafdr
(1) 7 cm (2) 5 cm
fd;k x;k gSA ;fn o`Ùk dk {ks=kiQy 38.5 lseh2 gS] rks oxZ
(3) 9 cm (4) 3 cm
ABCD dk {ks=kiQy (lseh2- eas) D;k gS\
(SSC CGL Tier-I Re-Exam. (2013) 20.07.2014 (Ist Sitting)

40. In the given figure, radius of a circle is 14 2 cm. A E B


PQRS is a square. EFGH, ABCD, WXYZ and
LMNO are four identical squares. What is the total L M
area (in cm2.) of all the small squares ?
F G
nh xbZ vkÑfr es]a ,d o`Ùk dh f=kT;k 14 2 lseh- gSA PQRS ,d
oxZ gSA EFGH, ABCD, WXYZ rFkk LMNO pkj leku oxZ O N
gSAa lHkh NksVs oxks± dk dqy {ks=kiQy (lseh-2 es)a D;k gS\ D H C

BLAM–393
{ks=kfefr
(1) 98 (2) 196 tSlk fd vkÑfr esa n'kkZ;k x;k gS] ABCD rhuksa o`Ùkksa ds dsUnzkas
(3) 122.5 (4) 171.5 ls xqtjrh gSA AB = 2 lseh rFkk CD = 1 lseh gSaA ;fn eè; o`Ùk
(SSC CGL Tier-II Exam. 20.02.2018)
dk {ks=kiQy] 'ks"k nksuksa o`Ùkksa ds {ks=kiQyksa dk vkSlr gS] rks BC
43. In the given figure, PQRS is a rectangle and a
semicircle with SR as diameter is drawn. A circle dh yEckbZ (lseh- esa) D;k gS\
is drawn as shown in the figure. If QR = 7 cm,
then what is the radius (in cm.) of the small circle? A
nh xbZ vkÑfr esa] PQRS ,d vk;r gS rFkk SR O;kl okyk ,d
v/Zxksyk cuk;k x;k gSA tSlk fd vkÑfr esa n'kkZ;k x;k gS fd
,d o`Ùk cuk;k x;k gSA ;fn QR = 7 lseh gS] rks NksVs o`Ùk dh
f=kT;k (lseh- esa) D;k gS\ B

P Q
C

D
S R

(1) 21 + 14 2
(1) e 6j –1 (2) e 6j + 1
(2) 21 – 14 2 (3) e 6j –3 (4) e 6j + 3
(3) Both 21 + 14 2 and 21 – 14 2 46. ABCD is a rectangle. P is a point on the side AB
as shown in the given figure. If DP = 13, CP = 10
(4) None of these
and BP = 6, then what is the value of AP?
(SSC CGL Tier-II Exam. 20.02.2018)
44. In the given figure, ABCD and BEFG are squares ABCD ,d vk;r gSA P, Hkqtk AB ij ,d fcUnq gS tSlk fd
of sides 8 cm. and 6 cm respectively. What is the nh xbZ vkÑfr esa n'kkZ;k x;k gSA ;fn DP = 13, CP = 10 rFkk
area (in cm2.) of the shaded region? BP = 6 gks] rks AP dk eku D;k gS\
nh xbZ vkÑfr eas] ABCD rFkk BEFG Øe'k% 8 lseh rFkk 6
lseh Hkqtk okys oxZ gSaA Nk;kafdr Hkkx dk {ks=kiQy (lseh2- esa) D C
D;k gS\

D C

A P B
G
F
(1) 105 (2) 133
(3) 12 (4) 10
(SSC CGL Tier-II Exam. 19.02.2018)
47. The sum of radii of the two circles is 91 cm. and
A the difference between their area is 2002 cm2. What
E B
is the radius (in cm.) of the larger circle?
nks o`Ùkksa dh f=kT;kvksa dk ;ksx 91 ls-eh gS rFkk muds {ks=kiQy
(1) 14 (2) 12 ds eè; dk varj 2002 lseh2 gSA cM+s o`Ùk dh f=kT;k (lseh- eas)
(3) 8 (4) 16
D;k gS\
(SSC CGL Tier-II Exam. 09.03.2018)
45. ABCD passes through the centres of the three (1) 56 (2) 42
circles as shown in the figure. AB = 2 cm. and CD (3) 63 (4) 49
= 1 cm. If the area of middle circle is the average (SSC CGL Tier-II Exam. 19.02.2018)
of the areas of the other two circles, then what is 48. PQR is a right angled triangle in which PQ = QR.
the length (in cm.) of BC? If the hypotenuse of the triangle is 20 cm, then
what is the area (in cm(2) of the triangle PQR?

BLAM–394
{ks=kfefr
PQR dk ledks.k f=kHkqt gS ftleas PQ = QR gSA ;fn f=kHkqt the floor. Black tiles are laid in the first row on
dk d.kZ 20 lseh gS] rks f=kHkqt PQR dk D;k {ks=kiQy (lseh2 all sides. If white tiles are laid in the on third of
the remaining and blue tiles in the rest how many
esa) esa D;k gS \ blue tiles will be there?
(1) 100 2 (2) 100 ,d dejs osQ iQ'kZ dh yEckbZ vkSj pkSM+kbZ Øe'k% 20 ft rFkk
10 ft gSA iQ'kZ ij rhu fofHkUu jaxksa osQ 2ft vk;keh oxZ VkbYl
(3) 50 2 (4) 50
(SSC CGL Tier-II Exam. 21.02.2018)
yxkus gSA lHkh vksj igyh drkj esa VkbYl dkys jax osQ yxk,
49. PQRS is a square whose side is 20 cm. By joining x,] 'ks"k esa ls ,d frgkbZ esa lisQn jax vkSj ckdh cps gq, uhys
opposite vertices of PQRS are get four triangles. jax osQ VkbYl yxk, tk,¡ rks uhys jax osQ VkbYl dh la[;k D;k
What is the sum of the perimeters of the four gksxh\
triangles? (1) 16 (2) 32 (3) 48 (4) 24
PQRS ,d oxZ gS ftldh Hkqtk 20 lseh gSA PQRS ds foijhr 53. A square of are 40 Sq. cm is inscribed in a circle
'kh"kks± dks feykus ij pkj f=kHkqt izkIr gksrs gSaA pkjksa f=kHkqtksa ds as shown in the figure. The area (in sq. cm) of the
ifjekiksa dk ;ksx D;k gS \ semi-circle is
,d oxZ dk {ks=kiQy 40 oxZ lseh gS tks ,d o`Ùk osQ vanj gS tSlk
(1) 40 2 (2) 80 2 + 80
fd fp=k esa fn[kyk;k x;k gSA v¼Zo`Ùk dk {ks=kiQy (lsehñ2 esa)
(3) 40 2 + 40 (4) 40 2 + 80 fdruk gksxk\
(SSC CGL Tier-II Exam. 21.02.2018)
50. The area of a circle whose radius is 6 cm is trisected
by two concentric circles. Find the radius of the
smallest circle.
,d o`Ùk dk {ks=kiQy ftldk f=kT;k 6 lseh gS] tks nks losQUnzh;
o`Ùkksa }kjk lef=kHkkftr gS] lcls NksVs o`Ùk osQ f=kT;k Kkr djsA
(1) 3 3 cm (2) 2 3 cm (1) 20 (2) 25 (3) 30 (4) 40
54. In the adjoining figure. It the radius of each of
(3) 4 3 cm (4) None of these
the four outer circles is r. What is the radius of
(C.P.O. — 2008) the inner circle?
51. A rectangle of certain dimensions is Chopped off
from one corner of a larger rectangle as shown.
fn[kk;s x, fp=k esa] pkjksa ckâk o`Ùkksa esa ls izR;sd dh f=kT;k r
AB = 8 cm and BC = 4 cm. The Perimeter of the gS rks Hkhrjh o`Ùk dh f=kT;k D;k gS\
figure ABCPQRA (in cm) is;
fn, x, fp=k esa ,d cM+s vk;r osQ dksus ls oqQN vk;keksa osQ
vk;kr dkVk tkrk gS] AB = 8 lseh vkSj BC = 4 lsehñ fn, x,
fLFkr esa ABCPQRA dk ifjf/ Kkr djs%

P C

R Q
4 cm

8 cm
A B 2 1
(1)
2 1
r (2)
2
r (3) e 2 j
1r (4) 2r
(1) 24 lseh (2) 28 lseh
55. Three circles of diameter 10 cm each are bound
(3) 36 lseh (4) 48 lseh together by rubber band as shown in the figure.
52. The length and breadth of the floor of room are The length of the rubber band (in cm) if it is
20 feet and to feet respectively square tiles of 2 stretched is
feet length of different colours are to be laid on

BLAM–395
{ks=kfefr
10 lseh- O;kl okys 3 o`Ùk ,d&nwljs dks Li'kZ djrs gS rFkk mUgsa eku ys fd C1 vkSj C2 ,d f=kHkqt osQ varo`Ùk rFkk ifjo`r gS] vkSj
,d jcj }kjk ck¡/k tkrk gSA jcj dh yEckbZ Kkr djsa\ f=kHkqt dh Hkqtk,¡ 3 lsehñ 4 lsehñ rFkk 5 lsehñ gSA rks
C1 dk {ks =kiQy
=?
C2 dk {ks =kiQy
9 16 9 4
(1) (2) (3) (4)
25 25 16 25
59. A circular swimming pool is surrounded by a
concrete wall 4 m wide. If the area of the concrete
11
wall surrounding the pool is that of the pool
25
then the radius (in cm) of the pool.
(1) 30 (2) 30 + 10 ,d o`rkdkj Lfofeax iwy 4 ehñ pkSM+s ,d nhokj ls f?kjk gSA ;fn
(3) 10 (4) 60 + 20 11
(SSC CPO (SI, ASI & Intelligence Officer) 2011) nhokj dk {ks=kiQy Lfofeax iwy osQ {ks=kiQy dk 25 gS] rks Lfofeax
56. C1 and C2 are two concentrice circles with centre
at O. Their radii are 12 cm and 3 cm respectively. iqy dh f=kT;k (ehñ esa) Kkr djsaA
B and C are the point of contact of two tangents (1) 8 (2) 16 (3) 30 (4) 20
drawn to C2 from a point A lying on the circle C1. (SSC CGL Tier-I 2015)
Then the area of the quadrilateral ABOC is. 60. The area of an isosceles trapezium is 176 cm2 and
C1 rFkk C2 nks laosQUnzh; o`Ùk gS] ftudk osQUnz O gSA ftudh 2
the height is of the sum of its parallel sides.
f=kT;k,¡ 12 lseh rFkk 3 lseh gSaA o`Ùk C1 ij fcUnq A bl izdkj 11
gS] fd fcUnq A bl izdkj gS] fd fcUnq A ls o`Ùk C2 ij [khph If the ratio of the length of the parallel sides is 4
xbZ Li'kZ js[kk,¡ o`Ùk C2 dks fcUnq B rFkk C ij Li'kZ djrh gSaA : 7 then the length of a diagonal (in cm) is
rc prqHkqtZ ABOC dk {ks=kiQy Kkr djsa\ ,d lef}ckgq leyac dk {ks=kiQy 176 lseh2 gSA vkSj Å¡pkbZ
2
9 15 bldh lekarj Hkqtkvksa osQ ;ksx dk 11 gSA ;fn lekarj Hkqtkvksa
(1) sq. cm (2) 12 15 sq. cm
2
dh yackbZ dk vuqikr 4 : 7 gS] rks fod.kZ dh yackbZ D;k gS\
(3) 9 15 sq. cm (4) 6 15 sq. cm (lseh- esa)
(SSC CGL Tier-I 2013)
(1) 2 137 (2) 24 (3) 137 (4) 28
57. The area of a circle is proportional to the square
of its radius. A small circle of radius 3 cm is drawn (SSC CGL Tier-II 2015)

within a larger circle of radius 5 cm. Find the 61. The perimeter of a certain isoscetes right triangle
ratio of the area of the annular zone to the area is 10 + 10 2 cm. What is the length of the
of the larger circle (Area of the annular zone is
hypotenuse of the triangle?
the difference between the area of the larger circle
and that of the smaller circle) fdlh lef¼ckgq ledks.k f=kHkqt dh ifjefr 10 + 10 2 cm
fdlh o`Ùk dk {ks=kiQy mldh f=kT;k osQ oxZ osQ lekuqikrh gSA gSA f=kHkqt osQ d.kZ dh yackbZ fdruh gksxh\
,d 3 lseh f=kT;k okyk o`Ùk 5 lseh f=kT;k okys o`Ùk osQ vUnj (1) 5 cm (2) 10 cm
[khpk tkrk gSA Annular zone rFkk cM+s o`Ùk osQ {ks=kiQy dk
(3) 5 2 cm (4) 10 2 cm
vuqikr Kkr djsa\ (SSC CPO SI Tier-I 2016)
cM+s o`Ùk rFkk NksVs o`Ùk osQ chp dk Hkkx Annular Zone 62. There is rectangular garden of 480 metre 160
dgykrk gSA metre. A path of width 3 metre is build outside
(1) 9 : 16 (2) 9 : 25 (3) 16 : 25 (4) 16 : 27 the garden. What is the are (in m2) of the path ?
(SSC CGL Tier-I 2013) ,d vk;rkdkj cxhpk 480 ehVj 160 ehVj dk gSA ,d 3
58. Let C1 and C2 be the inscribed and circumscribed ehVj pkSM+k jkLrk cxhps osQ ckgj tkrk gSA jkLrs dk {ks=kiQy (eh2
circles of a triangle with sides 3 cm. 4 cm and 5
esa) D;k gS\
area of C1 (1) 3984 (2) 1892
cm then area of C is ?
2 (3) 2796 (4) 3876

BLAM–396
{ks=kfefr
63. In the given figure, ABCDEF is a regular hexagon (1) (3 + ) a2 (2) (4 – ) a2
of side 12 cm P.Q and R are the mid points of the (3) (51 – 2 ) a2
(4) (2 – 3) a2
sides AB, CD and EF respectively. What is the 66. In a triangle ABC, AB = AC and the perimeter of
area (in cm2) of triangle PQR ?
ABC is 8(2 + 2 ) cm. If the length of BC is 2
nh xbZ vkÑfr esa ABCDEF ,d le "kV~Hkqt gS] ftldh Hkqtk
times the length of AB, then find the area of
12 lseh gSA P, Q rFkk R Øe'k% Hkqtkvksa AB, CD rFkk EF osQ
ABC.
eè; fcUnq gSA f=kHkqt PQR dk {ks=kiQy (lseh2 esa) D;k gS\
f=kHkq t ABC es a] AB = AC vkSj ABC dk ifjeki
E D 8(2 2 )cm gSA ;fn BC dh yackbZ Hkqtk AB dh yackbZ dh
R Q
2 xquh gS rks ABC dk {ks=kiQy Kkr dhft,A
F C (1) 32 cm2 (2) 28 cm2
(3) 16 cm2 (4) 36 cm2
(SSC CGL Tier-II Exam. 18.11.2020)
67. The perimeter of a rectangle is 50 cm. Its area
A P B and length are in the ratio of 5 : 1. Find the
length of the rectangle.
(1) 27 6 (2) 81 3 ,d vk;r dh ifjfefr 50 lseh- gSA blosQ {ks=kiQy ,oa yEckbZ
(3) 54 3 (4) 54 6
dk vuqikr 5 : 1 gSA vk;r dh yEckbZ Kkr dhft,A
(SSC CGL Tier-II 2018) (1) 15 cm (2) 20 cm
64. In the given figure, PQR is a quadrant whose (3) 18 cm (4) 22 cm
radius is 7 cm. A circle is inscribed in the (SSC CHSL (10+2) Tier-I Exam. 17.03.2020)
quadrant as shown in the figure. What is the area 68. If the perimeter and length of a rectangle are in
(in cm2) of the circle? the ratio 6 : 1 and the area of the rectangle is 288
cm2, Find the length of the rectangle.
nh xbZ vkÑfr esa PQR ,d o`Ùk[k.M gS] ftldh f=kT;k 7 cm
gSA tSlk fd vkÑfr esa n'kkZ;k x;k fd o`rk[k.M esa ,d o`Ùk dks ;fn vk;r osQ ifjeki vkSj yackbZ dk vuqikr 6 : 1 gS vkSj vk;r
vafdr fd;k x;k gSA o`r dk {ks=k (lseh esa) D;k gS\ dk {ks=kiQy 288 lseh-2 gS] rks vk;r dh yackbZ Kkr djsaA
(1) 10 cm (2) 12 cm
(3) 8 cm (4) 9 cm
P (SSC CHSL (10+2) Tier-I Exam. 18.03.2020)
69. The perimeter of an isosceles triangle is 125 cm.
If the base is 33cm, find the length of the equal
sides.
,d lef}ckgq f=kHkqt dk ifjeki 125 lseh- gSA ;fn vk/kj 33
lseh- gS] rks leku Hkqtkvksa dh yackbZ Kkr djsaA
(1) 32cm (2) 46cm
(3) 34cm (4) 42cm
(SSC CHSL (10+2) Tier-I Exam. 18.03.2020)
70. The area of a field in the shapes of a hexagon is
Q R 1944 3m 2 . What will be the cost (in Rs.) of
fencing it at the rate of Rs. 11.50 per meter ?
(1) 385 – 221 2 (2) 308 – 154 2 ,d "kV~Hkqtkdkj [ksr dk {ks=kiQy 1944 3m gSA mlds pkjksa
(3) 154 – 77 2 (4) 462 – 308 2 vksj 11-50 izfr ehVj dh nj ls ckM+ yxkus esa fdruh ykxr
(SSC CGL Tier-II 2018) (#i;s esa) vk,xh \
65. If a square metal sheet of side 2a is cut into 4 (1) 2,256 (2) 3,200
equal circular disks. Then the area of the metal (3 2,785 (4) 2,484
which will necessarily be wasted will be— (SSC Delhi Police SI, CAPFs SI & CISF ASI
(CPO) Exam 24.11.2020)
2a Hkqtk okys ,d dkfvkj 'khV dks 4 cjkcj o`Ùkkdkj fMLd
71. A rectangular lawn whose length is twice of its
esa dkVk x;k gS rks bl izfØ;k esa u"V gqbZ /krq dk {ks=kiQy fdruk breadth is extended by having four semi-circular
gksxk\ portions on its sides. What is the total area (in

BLAM–397
{ks=kfefr
2
m ) of the lawn if the smaller side of the rectangle (1) 120 (2) 150
is 12 m ? (Take = 3.14) (3) 200 (4) 330
,d vk;rkdkj ykWu] ftldh yackbZ mldh pkSM+kbZ dh rqyuk esa (SSC CHSL DEO & LDC Exam. 04.12.2011
(IInd Sitting (North Zone)
nksxquh gS] Hkqtkvksa ij pkj v/Zo`Ùkkdkj Hkkxksa osQ lkFk c<+k;k tkrk
4. The length of a room floor exceeds its breadth by
gSA ;fn vk;r dh NksVh Hkqtk 12 ehVj gS] rks ykWu dk oqQy 20 m. The area of the floor remains unaltered
{ks=kiQy (ehVj2 esa) Kkr djsaA ( = 3.14 ys)a when the length is decreased by 10 m but the
(1) 548.32 (2) 444 breadth is increased by 5 m. The area of the floor
(3) 853.2 (4) 308.64 (in square metres) is :
(SSC Delhi Police SI, CAPFs SI & CISF ASI ,d dejs osQ iQ'kZ dh yackbZ] mldh pkSM+kbZ ls 20 ehVj vfèkd
(CPO) Exam 24.11.2020)
gSA mlosQ iQ'kZ dk {ks=kiQy rc Hkh vifjofrZr jgrk gS] tc yackbZ
72. The perimeter of a square is the same as the
10 ehVj de dj nh tkrh gS vkSj pkSM+kbZ 5 ehVj c<+k nh tkrh
perimeter of a rectangle. The perimeter of the
square is 40 m. If its breadth is tow-thirds of its gSA rnuqlkj iQ'kZ dk {ks=kiQy fdruk (oxZ ehVj esa) gS\
length, then the area (in m2) of the rectangle is: (1) 280 (2) 325 (3) 300 (4) 420
(SSC CHSL DEO & LDC Exam. 11.12.2011
fdlh oxZ dk ifjeki ] fdlh vk;r osQ ifjeki osQ cjkcj gSA (IInd Sitting (East Zone)
oxZ dk ifjeki 40 eh gSA ;fn bldh pkSM+kbZ bldh yackbZ dh 5. There is a hall of length 15 m and width 12 m. If
nks&frgkbZ gS] rks vk;r dk {ks=kiQy eh2 esa Kkr djsaA the sum of area of surface and ceiling is equal to
(1) 96 (2) 84 the sum of area of all four wall. Then what is the
(3) 100 (4) 121 volume of that hall ? (in m3)
(SSC Delhi Police SI, CAPFs SI & CISF ASI ,d gkWy 15 ehVj yEck vkSj 12 ehVj pkSM+k gSA ;fn ml gkWy
(CPO) Exam 23.11.2020) ds iQ'kZ rFkk Nr ds {ks=kiQyksa dk ;ksx mldh pkjksa nhokjksa ds
Type-II {ks=kiQyksa ds cjkcj gks] rks ml gkWy dk vk;ru fdrus ?ku ehVj
Questions Based on Area and Perimeter of gksxk \
Four Walls of a Room (1) 720 (2) 900 (3) 1200 (4) 1800
(SSC CHSL DEO & LDC Exam. 04.12.2011
East Zone : First Sitting and SSC CAPF SI
1. The breadth of a rectangular hall is three-fourth & CISF ASI Exam. 23-06-2013)
of its length. If the area of the floor is 768 sq. m.,
then the difference between the length and breadth Type-III
of the hall is: Questions Based on Surface Area &
,d vk;rkdkj gkWy dh pkSM+kbZ mldh yEckbZ dk rhu&pkSFkkbZ gS A Volume of Cube and Cuboid
;fn iQ'kZ dk {ks=kiQy 768 oxZ ehú gks] rks gkWy dh yEckbZ vkSj 1. If the volume of two cubes are in the ratio 27:1,
pkSM+kbZ dk vUrj gS the ratio of their edge is :
(1) 8 metres (2) 12 metres ;fn nks ?kuksa ds vk;ruksa esa 27 : 1 dk vuqikr gS rks budh
(3) 24 metres (4) 32 metres
(SSC CGL Prelim Exam. 04.07.1999 (First Sitting)
Hkqtkvksa esa vuqikr gS—
2. What is the number of square tiles which can be (1) 3 : 1 (2) 27:1 (3) 1:3 (4) 1:27
surfaced in a room which is 15 m 17 cm long and (SSC CGL Prelim Exam. 04.07.1999 (IInd Sitting) &
(SSC S.O. Commercial Audit Exam. 16.11.2003)
9 m 2 cm width ?
2. Find the length of the largest rod that can be
fdlh dejs osQ 15 eh- 17 lseh- yEcs vkSj 9 eh- 2 lseh- pkSM+s placed in a room 16m long, 12m broad and
iQ'kZ ij yxkbZ tk ldus okyh oxZ Vkbyksa dh U;wure la[;k
2
fdruh gS \ 10 m. high.
3
(1) 840 (2) 841
(3) 820 (4) 814 ml yEch ls yEch NM+ dh yEckbZ Kkr dhft, tks ,d 16
(SSC CGL Prelim Exam. 11.05.2003 (First Sitting) 2
3. The area of the four walls of a room is 660 m2 and ehVj yEcs] 12 ehVj pkSM+s rFkk 10 ehVj mQaps dejs esa
3
its length is twice its breadth. If the height of the
room is 11 m, then area of its floor (in m2) is j[kh tk ldrh gS A
,d dejs dh pkjksa nhokjksa dk {ks=kiQy 660 oxZ ehVj gS vkSj (1) 23 m. (2) 68 m.
mldh yEckbZ] mldh pkSM+kbZ dh nqxquh gSA ;fn dejs dh mQapkbZ 2 1
(3) 22 m. (4) 22 m.
11 ehVj gks] rks mlds iQ'kZ dk {ks=kiQy (oxZ ehVj esa) 3 3
fdruk gksxk\ (SSC CGL Prelim Exam. 04.07.1999 (Second Sitting)

BLAM–398
{ks=kfefr
3. The edges of a cuboid are in the ratio 1 : 2 : 3 and 48 ehVj yEch vkSj 31.5 ehVj pkSM+h ,d fuEu Hkwfe dh mQ¡pkbZ
its surface area is 88cm2 . The volume of the 6.5 Mseh-c<+kus ds fy, ik'oZ ds Hkw[kaM esa ,d 27 ehVj yEch
cuboid is:
vkSj 18.2 ehVj pkSM+h [kkbZ [kksnh xbZA [kkbZ dh xgjkbZ fdruh
fdlh ?kukHk dh Hkqtkvksa dk vuqikr 1 : 2 : 3 gS A bldk gksxh\
i`"Bh; {ks=kiQy 88 lseh 2 gS A ?kukHk dk vk;ru gSµ (1) 1 metre (2) 2 metre
(1) 120 cm3 (2) 64 cm3 (3) 5 metre (4) 7 metre
(SSC Multi Tasking Staff Exam. 16.02.2014)
(3) 48 cm3 (4) 24 cm3
(SSC CGL Prelim Exam. 04.07.1999 (IInd Sitting) & 9. If the every side of a cube is increased by 50%,
(SSC CHSL DEO & LDC Exam. 28.10.2012) what will the surface area be increased in
4. How many cubes, each of edge 3 cm, can be cut percentage ?
from a cube of edge 15 cm? ;fn ,d ?ku dk izR;sd fljk 50% c<+k fn;k tk,] rks mlds
15 lseh- Hkqtkokys fdlh ?ku esa ls 3 lseh- Hkqtkokys dqy i`"Bh; {ks=kiQy esa fdrus izfr'kr o`f¼ gks tk,xh\
fdrus ?ku dkVs tk ldrs gSa\ (1) 125% (2) 50% (3) 100% (4) 75%
(1) 25 (2) 27 (3) 125 (4) 144 (SSC Graduate Level Tier-II Exam.29.09.2013, IInd Sitting)

(SSC CGL Prelim Exam. 27.02.2000 (Second Sitting) 10. Some bricks are arranged in an area measuring
5. Three solid iron cubes of edges 4 cm, 5 cm and 20 cu. m. If the length, breadth and height of
6 cm are melted together to make a new cube. each brick is 25 cm, 12.5 cm and 8 cm respectively,
62 cm3 of the melted material is lost due to then in that pile the number of bricks are (suppose
improper handling. The area (in cm2) of the whole there is no gap in between two bricks)
surface of the newly formed cube is dqN b±Vsa 20 ?ku ehVj eki ds {ks=kiQy esa j[kh tkrh gSA ;fn izR;sd
yksgs ds rhu Bksl ?ku] ftudh Hkqtk,¡ 4 lseh] 5 lseh rFkk 6 b±V dh yackbZ] pkSM+kbZ vkSj Å¡pkbZ Øe'k% 25 lseh] 12.5 lseh vkSj
lseh dh gSa] ,d&lkFk fi?kykdj ,d u, ?ku esa <kys x, gSaA 8 lseh gS] rks ml <sj esa b±Vksa dh la[;k fdruh gksxh (ekuk fd nks
bl izfØ;k esa [kjkc izc/a u ds dkj.k 62 lseh3 fi?kyh gqbZ b±Vksa ds chp dksbZ varjky ugha gS)\
èkkrq xqe gks xbZ gSA rnuqlkj] u, cus ?ku dk dqy i`"Bh; (1) 6,000 (2) 8,000
{ks=kiQy fdrus lseh2 gksxk \ (3) 4,000 (4) 10,000
(1) 294 (2) 343 (SSC CGL Tier-I Exam. 26.10.2014)
(3) 125 (4) 216 11. The length, breadth and height of a wooden box
(SSC CHSL DEO & LDC Exam. 10.11.2013, IInd Sitting) with a lid are 10 cm, 9 cm and 7 cm, respectively.
6. A cistern of capacity 8000 litres measures The total inner surface of the closed box is 262
externally 3.3 m by 2.6 m by 1.1 m and its walls cm2. The thickness of the wood (in cm.) is
are 5 cm thick. The thickness of the bottom is : ,d <Ddunkj ydM+h ds cDls dh yackbZ] pkSM+kbZ vkSj Å¡pkbZ
,d Vadh] ftldh èkkfjrk 8000 yhVj gS] osQ ckgjh eki 3·3 Øe'k% 10 lseh-] 9 lseh vkSj 7 lseh- gSaA can cDls dh dqy
eh- × 2·6 eh- × 1·1 eh- gSa rFkk bldh nhokjksa dh eksVkbZ 5 Hkhrjh lrg 262 lseh2 gSA ydM+h dh eksVkbZ (lseh- esa) fdruh gS\
lseh- gS A blosQ uhps dh ryh dh eksVkbZ gSµ (1) 2 (2) 3
(1) 1 m (2) 1.1 m 23
(3) 1 dm (4) 90 cm (3) (4) 1
3
(SSC CGL Prelim Exam. 11.05.2003 (First Sitting)
(SSC CGL Tier-II Exam, 2014 12.04.2015
7. A soap cake is of size 8 cm × 5 cm × 4 cm. The (Kolkata Region) TF No. 789 TH 7)
number of such soap cakes that can be packed in 12. A cuboidal shaped water tank, 2.1 m long and 1.5
a box measuring 56 cm × 35 cm × 28 cm is : m broad is half filled with water. If 630 litres more
,d lkcqu dh fVDdh dh eki 8 lseh- × 5 lseh- × 4 lseh- gS A water is poured into that tank, the water level
,d ckWDl ftldk eki 56 lseh- × 35 lseh- × 28 lseh- gSa] esa will rise
,slh ftruh fVfDd;k¡ iSd dh tk ldrh gSa] mudh la[;k gksxh A ,d ?kukHkkdkj ikuh dh Vadh 2.1 ehVj yach vkSj 1.5 ehVj
(1) 49 (2) 196 (3) 243 (4) 343 pkSM+h gS vkSj vkèkh Hkjh gSA ;fn mlesa 630 yhVj ikuh vkSj Mkyk
(SSC CGL Prelim Exam. 08.02.2004 (Second Sitting) tk, rks ty Lrj c<+dj fdruk gks tk,xk\
8. There is a plot of length 48 m and width 31.5. to (1) 0.15 cm (2) 0.20 metre
increase its height 6.5 cm, it is dug a tank of
(3) 0.18 cm (4) 2 cm
length 27 m and 18 m. What will be the depress
(SSC CHSL (10+2) LDC, DEO & PA/SA Exam, 20.12.2015
of tank ? (Ist Sitting) TF No. 9692918)

BLAM–399
{ks=kfefr
13. The perimeter of one face of a cube is 20 cm. Its
volume will be Y
x B
,d ?ku ds ,d iQyd dk ifjeki 20 lseh gSA bldk vk;ru
fdruk gksxk \
Z
(1) 625 cm3 (2) 100 cm3
(3) 125 cm3 (4) 400 cm3
(SSC CGL Tier-I Exam, 09.08.2015
(Ist Sitting) TF No. 1443088)
14. A cube of edge 6 cm is painted on all sides and
(1) 48 ( 3 ) 1 (2) 24 4 e 3j 1
then cut into unit cubes. The number of unit cubes
with no sides painted is (3) 28 6 e 3j (4) 32 3 e 3j
6 lseh Hkqtk okys ,d ?ku dks] mlds lHkh iQydksa dks jaxus ds 18. A right prism has a square base with side of base
ckn] bdkbZ ds ?kuksa esa dkV fn;k x;k gSA rnuqlkj] mu bdkbZ ds 4 cm. and the height of prism is 9 cm. The prism
?kuksa dh la[;k fdruh gS] ftuds fdlh Hkh iQyd dks jaxk ugha is cut in three parts of equal heights by two planes
x;k gS\ parallel to its base. What is the ratio of the volume
of the top, middle and the bottom part
(1) 0 (2) 64
respectively?
(3) 186 (4) 108
(SSC Delhi Police S.I. (SI) Exam. 19.08.2012)
,d le fizTe dk vk/kj 4 lseh- Hkqtk okyk oxZ gS rFkk fizTe
15. 2 cm of rain has fallen on a square km of land. dh mQ¡pkbZ 9 lseh- gSA fizTe dks mlds vk/kj ds lekarj nks ryksa
Assuming that 50% of the raindrops could have }kjk leku mQ¡pkbZ ds rhu Hkkxksa esa dkVk x;k gSA Øe'k% mQijh
been collected and contained in a pool having a eè; rFkk fupys Hkkxksa ds vk;ru dk vuqikr D;k gS\
100 m × 10 m base, by what level would the water (1) 1 : 8 : 27 (2) 1 : 7 : 19
level in the pool have increased ? (3) 1 : 1 : 1 (4) 1 : 7 : 20
,d oxZ fdeh {ks=kiQy dh Hkwfe ij 2 lseh o"kkZ gqbZ gSA ;g (CGL Tier-II, Exam. 17.02.2018)

ekudj fd 50% o"kkZ dh cw¡nsa ,df=kr djosQ] ;fn 100 eh- × 19. There is a box of cuboid shape. The smallest side
of the box is 20 cm and largest side is 40 cm.
10 eh- vkèkkj okys ,d rky esa tek dh tkrha] rks ml rky esa
Which of the following can be volume (in cm3.) of
ikuh dk Lrj fdruk c<+ tkrk\ the box?
(1) 1 km (2) 10 m ?kukHk vkdkj dk ,d cDlk gSA cDls dh lcls NksVh Hkqtk 20
(3) 10 cm (4) 1 m lseh rFkk lcls cM+h Hkqtk 40 lseh gSA fuEufyf[kr esa ls cDls
(SSC Graduate Level Tier-II Exam.16.09.2012)
dk vk;ru (lseh3- esa) D;k gks ldrk gS\
16. A solid cube has side 8 cm. It is cut along diagonals
(1) 18000 (2) 12000
of top face to get 4 equal parts. What is the total
(3) 36000 (4) 42000
surface area (in cm2.) of each part?
(SSC CGL Tier-II Exam. 09.03.2018)
,d Bksl ?ku dh Hkqtk 8 lseh gSA og mQij lrg ds fod.kks± ij 20. A cuboid has dimensions 8 cm. × 10 cm × 12 cm.
4 leku Hkkxksa esa dkVk tkrk gSA izR;sd Hkkx dk dqy i`"Bh; It is cut into small cubes of side 2 cm. What is
{ks=kiQy (lseh2- eas) D;k gS\ the percentage increase in the total surface area?
,d ?ku dk vk;ke 8 lseh × 10 lseh × 12 lseh gSA bls 2 lsseh
(1) 96 + 64 2 (2) 80 + 64 2
Hkqtk okys NksVs ?kuksa esa dkVk tkrk gSA i`"Bh; {ks=kiQy eas fdrus
(3) 96 + 48 2 (4) 80 + 48 2 izfr'kr dh o`f¼ gqbZ gS\
(SSC CHSL DEO & LDC Exam. 20.02.2018) (1) 286.2 (2) 314.32
17. A right triangular pyramid XYZB is cut from cube (3) 250.64 (4) 386.5
as shown in figure. The side of cube is 16 cm. X, Y (SSC CGL Tier-II Exam. 21.02.2018)
and Z are mid-points of the edges of the cube. What 21. If the surface area of a sphere is 1386 cm2, then
is the total surface area (in cm.2) of the pyramid?
its volume is :
FG Take 22 IJ
tSlk fd vkÑfr esa n'kkZ;k x;k gS fd ,d ?ku ls ,d ledks.kh; H 7 K
f=kHkqtkdkj fijkfeM XYZB dkVk x;k gSA ?ku dh Hkqtk 16 lseh-
;fn fdlh xksys dk i`"Bh; {ks=kiQy 1386 cm2 gS] rks bldk
gSA X, Y rFkk Z ?ku ds 'kh"kks± ij eè; fcUnq gSaA fijkfeM dk dqy
i`"Bh; {ks=kiQy (lseh-2 eas) D;k gS\ vk;ru fdruk gksxk\
FG 22
ysa IJ
H 7 K
BLAM–400
{ks=kfefr
3 3
(1) 8451 cm (2) 4851 cm ;fn ,d csyu dh Å¡pkbZ mldh ifjf/ ls 4 xquk gS] rks csyu
(3) 5418 cm3 (4) 4581 cm3
dk vk;ru ifjf/ c, dh n`f"V ls D;k gksxk \
(SSC CGL Tier-II Exam. 18.11.2020)
22. The ratio of the height and the diameter of a
2c 3
right circular cone is 6: 5 and its volume is (1) (2) 4Sc3
2200
cm 2. W hat is its sl ant hei ght ?
7
c3
FG Take 22 IJ (3) (4) 2Sc3
H 7 K (SSC CGL Tier-II Exam. 12.04.2015 TF No. 567 TL 9)

,d yEc o`Ùkh; 'kadq dh mQ¡pkbZ vkSj O;kl dk vuqikr 6 % 5 4. A rectangular piece of paper of dimensions 22 cm
by 12 cm is rolled along its length to form a
200
gS vkSj mldk vk;ru cm3 gSA bldh fr;Zd mQ¡pkbZ D;k cylinder. The volume (in cu.cm.) of the cylinder
7
22
so formed is ( use )
F
gS\ GH
22
ysa IJ 7
7 K ,d 22 lseh- × 12 lseh- foek ds dkxt ds vk;rkdkj VqdM+s
(1) 26 cm (2) 13 cm dks csyukdkj :i esa yEckbZ ds vuqfn'k yisVk tkrk gSA bl izdkj
(3) 25 cm (4) 5 cm
22
(SSC CGL Tier-II Exam. 18.11.2020)
cus csyu dk vk;ru (lseh-3 esa) gS ( 7
dk iz;ksx djsa)
Type-IV
(1) 562 (2) 412
Questions Based on Cylinder, Cone, Sphere,
(3) 462 (4) 362
Hemisphere etc. (SSC CAPFs SI, CISF ASI & Delhi Police SI
Exam. 22.06.2014 TF No. 999 KP0)
1. Volume of a right circular cylinder of height 21
5. A cylindrical tank of diameter 35 cm is full of
cm and base radius 5 cm is :
water. If 11 litres of water is drawn off, the water
21 lseh mQaps vkSj 5 lseh vk/kj dh f=kT;k okys yac o`Ùkh; FG 22 IJ
csyu dk vk;ru fdruk gksxk\ level in the tank will drop by : use H K
7
(1) 1255 cm3 (2) 1050 cm3
35 lsehú O;kl okyh ,d csyukdkj Vadh iw.kZr;k ikuh ls
(3) 1175 cm3 (4) 1650 cm3
Hkjh gqbZ gS A ;fn blesa ls 11 yhVj ikuh fudky fy;k tk,
(SSC CHSL (10+2) LDC, DEO & PA/SA
Exam, 06.12.2015 (Ist Sitting) TF No. 1375232)
rks Vadh esa ikuh ds Lrj esa tks deh vk;sxh] og gS—
2. Two circular cylinders of equal volume have their F 22
dk izÕkksx dhft, I
heights in the ratio 1 : 2. Ratio of their radii is H 7 K
22 1 6
(Take ) (1) 10 cm. (2) 12 cm.
7 2 7
leku vk;ru ds nks orqZy csyuksa dh Å¡pkbZ 1:2 vuqikr esa gSA 3
(3) 14 cm. (4) 11 cm.
22 7
mudh f=kT;k dk vuqikr gS (eku ysa fd 7
) gSA (SSC CGL Prelim Exam. 04.07.1999 (Second Sitting)
6. The base radii of two cylinders are in the ratio 2 :
(1) 1 : 4 (2) 1 : 2 3 and their heights are in the ratio 5 : 3. The
ratio of their volumes is :
(3) 2 :1 (4) 1 : 2
nks csyuksa dh vk/kj f=kT;k,¡ 2 : 3 ds vuqikr esa gSa vkSj mudh
(SSC CAPFs SI, CISF ASI & Delhi Police SI
Exam. 22.06.2014 TF No. 999 KP0)
Å¡pkbZ;ksa dk vuqikr 5 : 3 gSA muds vk;ruksa dk vuqikr gS&
(1) 27 : 20 (2) 20 : 27
3. If the height of a cylinder is 4 times its
circumference, the volume of the cylinder in terms (3) 9 : 4 (4) 4 : 9
of its circumference c, is (SSC CGL Prelim Exam. 24.02.2002
& 13.11.2005 (Ist Sitting)

BLAM–401
{ks=kfefr
7. The diameter of a cylinder is 7 cm and its height 11. A well 20 m in diameter is dug 14 m deep and
the earth taken out is spread all around it to a
22
is 16 cm. Using the value of S = , the lateral width of 5 m to form an embankment. The height
7
of the embankment is :
surface area of the cylinder is
20 eh- O;kl okyk ,d dqvk¡ 14 eh- dh xgjkbZ rd [kksnk
,d csyu dk O;kl 7 lseh vkSj mldh mQ¡pkbZ 16 lseh gSA
tkrk gS vkSj blls fudkyh x;h feV~Vh dks mlds pkjksa vksj 5
22 eh- dh pkSM+kbZ rd iQSykdj ,d pcwrjk cuk fn;k tkrk gS A
S=
7
dk eku mi;ksx djrs gq,] ml csyu ds ik'ohZ; iQyd
bl pcwrjs dh mQ¡pkbZ gksxhµ
dk {ks=kiQy fdruk gksxk\ (1) 10 m (2) 11 m
(1) 352 cm.2 (2) 350 cm.2
(3) 11.2 m (4) 11.5 m
(3) 355 cm.2 (4) 348 cm.2 (SSC CGL Prelim Exam. 08.02.2004 (Second Sitting)
(SSC CHSL DEO & LDC Exam. 04.12.2011 12. A hollow cylindrical tube 20 cm long, is made of
(IInd Sitting (East Zone)
iron and its external and internal diameters are
8. The radius of a cylinder is 10 cm and height is 4 8 cm and 6 cm respectively. The volume of iron
cm. The number of centimetres that may be added
either to the radius or to the height to get the FG 22 IJ
same increase in the volume of the cylinder is
used in making the tube is H 7 K
,d csyu dh f=kT;k 10 lseh rFkk Å¡pkbZ 4 lseh gSA rnuqlkj] ,d [kks[kyh csyukdkj uyh] tks yksgs dh cuh gS] 20 lseh- yEch
ml csyu dh f=kT;k ;k Å¡pkbZ esa] fdrus lseh tksM+s tk,¡] ftlls gS rFkk blosQ ckgjh vkSj vkarfjd O;kl Øe'k% 8 lseh- vkSj 6
ml csyu osQ vk;ru esa Hkh mruh gh o`f¼ gks tk,\ lseh- gSa A bl uyh osQ cukus esa iz;qDr yksgs dk vk;ru gS
(1) 5 cm (2) 4 cm
FG 22 IJ µ
(3) 25 cm (4) 16 cm H 7 K
(SSC Graduate Level Tier-II Exam.16.09.2012)
9. A right circular cylinder of height 16 cm is covered (1) 1760 cu.cm. (2) 880 cu.cm.
by a rectangular tin foil of size 16 cm × 22 cm. (3) 440 cu.cm. (4) 220 cu.cm.
The volume of the cylinder is (SSC CGL Prelim Exam. 11.05.2003 (Second Sitting)

16 lseh mQ¡pkbZ okys ,d yEc&o`Ùkh; csyu dks ,d 16 lseh 13. Water flows through a cylindrical pipe, whose
radius is 7 cm, at 5 metre per second. The time, it
× 22 lseh lkbt dh vk;rkdkj fVu 'khV }kjk <dk x;k gSA
takes to fill an empty water tank, with height 1.54
csyu dk vk;ru gksxkµ metres and area of the base (3 × 5) square metres,
(1) 352 cm3 (2) 308 cm3
(3) 176 cm 3
(4) 616 cm3
FG
is take
22 IJ
(SSC CGL Prelim Exam. 04.02.2007 (First Sitting)
H 7 K
10. A solid cylinder has total surface area of 462 7 lseh- v¼ZO;kl okys ,d csyukdkj ikbZi ds }kjk 5 ehVj izfr
1 lsd.M dh jÝrkj ls ikuh cg jgk gSA vk/kj dk {ks=kiQy (3×5)
sq.cm. Its curved surface area is rd of the total
3 oxZehVj rFkk mQ¡pkbZ 1.54 ehVj okys [kkyh ikuh ds VSad dks Hkjus
surface area. Then the radius of the cylinder is
22
,d Bksl csyu dh dqy lrg dk {ks=kiQy 462 lseh2 gSA mldh esa yxus okyk le; gksxk ( 7
yhft,)µ
1 (1) 6 minutes (2) 5 minutes
oØh; lrg dk {ks=kiQy] dqy lrg ds {ks=kiQy dk 3 gSA
(3) 10 minutes (4) 9 minutes
rnuqlkj ml csyu dh f=kT;k fdruh gS\ (SSC CGL Prelim Exam. 27.07.2008 (Second Sitting)

(1) 7 cm (2) 3.5 cm 14. The height of a circular cylinder is increased six
times and the base area is decreased to one-ninth
(3) 9 cm (4) 11 cm
of its value. The factor by which the lateral
(SSC (CHSL DEO & LDC Exam. 04.12.2011
(Ist Sitting (East Zone)
surface of the cylinder increases is

BLAM–402
{ks=kfefr
,d o`Ùkkdkj csyu dh Å¡pkbZ 6 xquh c<+k nh tkrh gS vkSj mlosQ 18. The perimeter of the base of a right circular cone
is 8 cm. If the height of the cone is 21 cm, then
1
vkèkkj esa] mlosQ eku dk 9 Hkkx de dj fn;k tkrk gSA its volume is:
,d yac o`Ùkh; 'kadq ds vk/kj dk ifjeki 8 lseh gSA ;fn ml
rnuqlkj] ml csyu osQ ik'oZ i`"B esa fdrus Hkkx dh o`f¼ gks
'kadq dh mQ¡pkbZ 21 lseh gks] rks mldk vk;ru fdruk gksxk\
tk,xh\
112
1 2 3 (1) 108 S cm3 (2) cm3
(1) 2 (2) (3) (4)
2 3 2
(SSC Graduate Level Tier-II Exam.16.09.2012) 108
15. The area of the curved surface and the area of the (3) 112 S cm3 (4) cm3
base of a right circular cylinder are a square cm
(SSC GL Tier-I Exam. 21.04.2013, Ist Sitting)
and b square cm respectively. The height of the
cylinder is 19. If the ratio of the diameters of two right circular
cones of equal height be 3 : 4, then the ratio of
,d yac o`Ùkh; csyu ds oØh; i`"B vkSj mlds vk/kj dk their volume will be
{ks=kiQy Øe'k% a oxZ lseh rFkk b oxZ lseh gSA rnuqlkj mldh
;fn nks ,dleku mQ¡pkbZ ds yac&o`Ùkh; 'kadqvksa ds O;klksa dk
mQ¡pkbZ fdruh gksxh \
vuqikr 3 : 4 gks] rks muds vk;ruksa dk vuqikr fdruk gksxk\
2a a b (1) 3 : 4 (2) 9 : 16
(1) cm (2) cm
b 2 (3) 16 : 9 (4) 27 : 64
(SSC CHSL DEO & LDC Exam. 10.11.2013, Ist Sitting)

a a 20. The volume of a right circular cone is 1232 cm3


(3) cm (4) cm and its vertical height is 24 cm. Its curved surface
2 b 2 b
area is
(SSC CHSL DEO & LDC Exam. 28.10.2012 (Ist Sitting)
16. The perimeter of the base of a right circular cylinder
,d yac o`Ùkh; 'kaoqQ dk vk;ru 1232 lseh3 gS] vkSj mldh
is ‘a’ unit. If the volume of the cylinder is V cubic ÅèokZèkj Å¡pkbZ 24 lse h gSA rnuqlkj] mlosQ oØkdkj i`"B dk
unit, then the height of the cylinder is {ks=kiQy fdruk gS\
,d yac o`Ùkh; csyu ds vk/kj dk ifjeki a ,dd gSA rnuqlkj] (1) 154 cm2 (2) 550 cm2
;fn ml csyu dk vk;ru V ?ku ,dd gks] rks csyu dh mQ¡pkbZ (3) 604 cm 2
(4) 704 cm2
fdruh gksxh\ (SSC CGL Prelim Exam. 11.05.2003 (Ist Sitting) &
(SSC GL Tier-II Exam.16.09.2012 & 29.09.2013)

4a 2 V 4 a2 21. If the ratio of volumes of two cones is 2 : 3 and


(1) unit (2) unit
V the ratio of the radii of their bases is 1 : 2, then
the ratio of their heights will be
a 2V 4 V
nks 'kadqvksa ds vk;ruksa dk vuqikr 2 % 3 gS vkSj muds vkèkkjksa
(3) unit (4) unit
4 a2
dh f=kT;kvksa dk vuqikr 1 % 2 gSA mudh Å¡pkbZ;ksa dk vuqikr gSµ
(SSC Graduate Level Tier-I Exam. 19.05.2013)
(1) 8 : 3 (2) 3 : 8
17. A sphere is placed inside a right circular cylinder
so as to touch the top, base and the lateral surface (3) 4 : 3 (4) 3 : 4
of the cylinder. If the radius of the sphere is R, (SSC CGL Tier-I Re-Exam. (2013) 27.04.2014)

the volume of the cylinder is 22. The radius of base and slant height of a cone are
in the ratio 4 : 7. If its curved surface area is 792
,d yac orqZy csyu ds vanj ,d xksyd dks bl izdkj j[kk tkrk
cm2, then the radius (in cm) of its base is [Use S
gS fd og csyu ds 'kh"kZ] ry vkSj i'p i`"B dks Li'kZ djsA = 22/7]
xksyd dh f=kT;k R gSA csyu dk vk;ru gS
fdlh 'kadq ds vkèkkj dh f=kT;k rFkk mldh fr;Zd mQ¡pkbZ 4%7
8 ds vuqikr esa gSaA ;fn bldk oØ i`"Bh; {ks=kiQy 792 lseh2 gS]
(1) 2SR3 (2) 4SR3 (3) 8SR3 (4) SR 3
3 rks blds vkèkkj dh f=kT;k (lseh esa) gksxhA [ S = 22/7
(SSC CAPFs SI, CISF ASI & Delhi Police SI yhft,]µ
Exam. 22.06.2014 TF No. 999 KP0)

BLAM–403
{ks=kfefr
(1) 8 (2) 12 26. The radius of the base of a Conical tent is 12 m.
(3) 14 (4) 16 The tent is 9 m high. Find the cost of canvas
(SSC (South Zone) Investigator Exam. 12.09.2010) required to make the tent, if one square metre of
23. The radius of the base of a conical tent is 16 metre. canvas costs 120 (Take S = 3.14 )

3
,d 'kadquqek racw ds vk/kj dh f=kT;k 12 ehVj gSA racw 9 ehVj
If 427 sq. metre canvas is required to construct mQ¡pk gSA racw cukus ds fy, visf{kr dSuokl dh ykxr Kkr
7
the tent, then the slant height of the tent is : dhft,] ;fn dSuokl ds ,d oxZ ehVj dh dher 120 gSA
(S = 3.14 ysa)
FG Take 22 IJ
H 7 K (1) 67, 830 (2) 67, 800
(3) 67, 820 (4) 67, 824
,d 'kaDokdkj racw osQ vkèkkj dh f=kT;k 16 ehVj gSA rnuqlkj (SSC CGL Tier-I Re-Exam. (2013) 20.07.2014 (Ist Sitting)
27. The volume of a right circular cone is equal to
3
;fn ml racw dks cukus esa 427 7 oxZ ehVj dSUol dh the volume of a right circular cylinder. The height
and the radius of the cylinder are 9 cm and 20 cm
vko';drk iM+ h gks ] rks ml ra c w dh frjNh Å¡ p kbZ respectively. If the height of the cone is 108 cm,
then its radius, (in cm) is
FG 22
ekudj IJ fdruh jgh gksxh\
H 7 K yEc o`Ùkkdkj 'kadq dk vk;ru yEc o`Ùkkdkj csyu ds vk;ru
ds cjkcj gSA csyu dh Å¡pkbZ vkSj f=kT;k Øe'k% 9 lseh- vkSj
(1) 17 metre (2) 15 metre
20 lseh- gSaA ;fn 'kadq dh Å¡pkbZ 108 lseh- gS] rks mldh f=kT;k
(3) 19 metre (4) 8.5 metre
fdruh (lseh- esa) gksxh \
(SSC CHSL DEO & LDC Exam. 11.12.2011
(IInd Sitting (East Zone) (1) 12 (2) 14 (3) 20 (4) 10
24. If the radii of the circular ends of a truncated (SSC CGL Tier-II Exam, 2014 12.04.2015
(Kolkata Region) TF No. 789 TH 7)
conical bucket which is 45 cm high be 28 cm and
7 cm, then the capacity of the bucket in cubic 28. If the area of the base of a cone is increased by
100%, then the volume increases by
F
centimetre is GH use S
22 IJ ;fn fdlh 'kadq ds vk/kj ds {ks=kiQy dks 100% c<+k fn;k
7 K
tk,] rks vk;ru fdruk c<+ tk,xk \
;fn 45 lseh Å¡ph fdlh [kafMr 'kaoqQ ckYVh osQ o`Ùkkdkj Nksjksa (1) 200% (2)182% (3) 141% (4) 100%
dh f=kT;k,¡ 28 lseh rFkk 7 lseh gSa rks ?ku lsaVhehVj esa ckYVh (SSC CGL Tier-II Exam, 2014 12.04.2015
dh èkkfjrk gS (Kolkata Region) TF No. 789 TH 7)
29. A right circular cone of height 20 cm and base
FG 22
iz ; ksx djsa
IJ radius 15 cm is melted and cast into smaller cones
H 7 K of equal sizes of height 5 cm and base radius 1.5
cm. The number of cones cast are
(1) 48510 (2) 45810 (3) 48150 (4) 48051
20 lseh Å¡ps vkSj 15 lseh vk/kj dh f=kT;k okys ,d
(SSC CHSL DEO & LDC Exam. 11.12.2011
(Ist Sitting (Delhi Zone)
leo`Ùkkdkj 'kaoqQ dks xyk;k tkrk gS vkSj mls 5 lseh Å¡ps rFkk
25. A semi-circular sheet of metal of diameter 28 cm 1-5 lseh vk/kj dh f=kT;k okys leku vkdkj ds NksVs&NksVs
is bent into an open conical cup. The depth of 'kaoqQvksa esa <kyk tkrk gSA <ys gq, 'kaoqQvksa dh la[;k fdruh gksxh\
the cup is approximately (1) 300 (2) 150 (3) 400 (4) 100
,d /krq dh 28 lseh- O;kl okyh vèkZo`Ùkkdkj pknj dks eksM+dj (SSC Constable (GD) Exam, 04.10.2015, Ist Sitting)

[kqys 'kaDokdkj I;kys esa cny fn;k x;k gSA rnuqlkj ml I;kys 30. A plane divides a right circular cone into two parts
of equal volume. If the plane is parallel to the
dh xgjkbZ yxHkx fdruh gksxh\ base, then the ratio, in which the height of the
(1) 11 cm (2) 12 cm cone is divided, is
(3) 13 cm (4) 14 cm ,d ry yac o`Ùkh; 'kaoqQ dks leku vk;ru okys nks Hkkxksa esa
(SSC CHSL DEO & LDC Exam. 04.12.2011 foHkkftr djrk gSA ;fn ry vk/kj ds lekarj gks] rks 'kaoqQ dh
(IInd Sitting (East Zone)
Å¡pkbZ dks fdl vuqikr esa foHkkftr fd;k tk,xk\
BLAM–404
{ks=kfefr
(1) 1 : 3 2 (2) 1 : 2 f=kT;k,a ijLij tqM+ tk,¡A rnuqlkj ml 'kadq dh oØ lrg dk
{ks=kiQy fdruk gks tk,xk \
(3) 1 : 3 2 + 1 (4) 1 : 3 2 – 1
(1) Sr2 cm2 (2) 4Sr2 cm2
(SSC CGL Tier-II Exam, 25.10.2015, TF No. 1099685)
31. A right triangle with sides 9 cm, 12 cm and 15 cm r2
(3) cm2 (4) 2Sr2 cm2
is rotated about the side of 9 cm to form a cone. 4
The volume of the cone so formed is : (SSC CHSL DEO & LDC Exam. 11.12.2011
(Ist Sitting (East Zone)
9 lseh, 12 lseh vkSj 15 lseh yach Hkqtkvksa okys ledks.k f=kHkqt
35. If a right circular cone is separated into solids of
dks ;fn 9 lseh Hkqtk ds ifjr% ?kqek;k tkrk gS rkfd ,d 'kadq cu
volumes V1, V2 , V3 by two planes parallel to the
ldsA bl izdkj cus 'kadq dk vk;ru fdruk gksxk\ base, which also trisect the altitude, then V1 : V2
(1) 327 S cm3 (2) 330 S cm3 : V3 is
(3) 334 S cm3 (4) 324 S cm3 ;fn ,d leyach o`Ùkkdkj 'kadq dks V1, V2 , V3 vk;ru okys
(SSC CHSL (10+2) LDC, DEO & PA/SA Exam, 06.12.2015 3 ,sls Bksl [kaMksa esa ckaV fn;k tk,] ftuds nks vk/kj] ewy vkèkkj
(Ist Sitting) TF No. 1375232)
ds lekarj gksa vkSj mQ¡pkbZ;k¡ Hkh lef=kHkkftr gksa] rks V1 : V2 : V3
32. The volume and area of right circular cone are
equal in numerical volume. If the height of cone dk vuqikr fdruk gksxk\
(1) 1: 2 : 3 (2) 1: 4 : 6
1 1
h and radius is r, then the value of 2 + 2 equal (3) 1: 6 : 9 (4) 1: 7 : 19
h r
(SSC CHSL DEO & LDC Exam. 04.12.2011
to— (Ist Sitting (East Zone)
,d yac o`Ùkh; 'kaoqQ ds ik'ohZ; i`"B ds vk;ru vkSj {ks=kiQy dk 36. A sphere and a hemisphere have the same volume.
la[;kRed eku leku gSA ;fn 'kaoqQ dh Å¡pkbZ h vkSj f=kT;k r gS] The ratio of their radii is

1
,d xksys rFkk v/Zxksys dk vk;ru cjkcj gSA mudh f=kT;kvksa dk
1
rks + dk eku gS vuqikr gSµ
h2 r2
(1) 1 : 2 (2) 1 : 8
1 1 3
(1) (2) (3) 1 : 2 (4) 1 : 2
3 9
(SSC CHSL DEO & LDC Exam. 04.11.2012 (IInd Sitting

9 3 37. A solid hemisphere is of radius 11 cm. The curved


(3) (4) surface area in sq. cm is
1 1
(SSC CGL Tier-II Exam, 25.10.2015) ,d Bksl xksyk/Z dh f=kT;k 11 lseh- gSA rnuqlkj mlds oØ
33. If h, c, v are respectively the height, curved surface i`"B dk {ks=kiQy fdrus oxZ lseh gksxk\
area and volume of a right circular cone, then (1) 1140.85 (2) 1386.00
the value of 3Svh3 – c2h2 + 9v2 is (3) 760.57 (4) 860.57
;fn ,d yac&o`Ùkh; 'kadq dh mQ¡pkbZ] oØh; i`"B dk {ks=kiQy rFkk (SSC GL Tier-I Exam. 11.11.2012 (Ist Sitting)

vk;ru Øe'k% h, c, v gksa] rks 3Svh3 – c2h2 + 9v2 dk eku 38. The total surface area of a sphere is 8S square
unit. The volume of the sphere is
fdruk gksxk\
(1) 2 (2) –1 ,d xksyd dk dqy i`"Bh; {ks=kiQy 8S oxZ ,dd gSA xksyd dk
(3) 1 (4) 0 vk;ru gSµ
(SSC Graduate Level Tier-II Exam. 29.09.2013)
8 2 8
34. A right angled sector of radius r cm is rolled up (1) cubic unit (2) cubic unit
3 3
into a cone in such a way that the two binding
radii are joined together. Then the curved surface
area of the cone is 8 3
(3) 8 3 cubic unit (4) cubic unit
5
r lseh f=kT;k okyk ,d ledks.kh; lsDVj eksM+ dj 'kadq ds :i
(SSC GL Tier-I Exam. 19.05.2013 Ist Sitting)
esa bl izdkj ifjofrZr dj fn;k x;k gS] rkfd mls cka/us okyh

BLAM–405
{ks=kfefr
39. A sphere and a hemisphere have the same radius. (1) 3 cm (2) 4 cm
Then the ratio of their respective total surface areas
(3) 5 cm (4) 6 cm
is
(SSC CGL Prelim Exam. 04.07.1999 (Ist Sitting) &
,d xksyd vkSj ,d xksyk/Z dh f=kT;k ogh gSA rks muds (SSC CPO S.I. Exam.12.01.2003)

vius&vius dqy i`"B {ks=k dk vuqikr gS % 44. By melting a solid lead sphere of diameter 12 cm,
three small spheres are made whose diameters
(1) 2 : 1 (2) 1 : 2 (3) 4 : 3 (4) 3 : 4
are in the ratio 3 : 4 : 5. The radius (in cm) of the
(SSC CGL Tier-I Re-Exam. (2013) 27.04.2014)
smallest sphere is
40. The volume of hemi-spherical pot are 6.4 l and
12 ls-eh- O;kl okys lhls ds ,d Bksl xksys dks fi?kykdj rhu
21.6 l. What is the ratio of their internal radii ?
NksVs vkdkj ds xksys cuk, x, gSa] ftuds O;klksa dk vuqikr
nks v/Zxksykdkj crZuksa dh /kfjrk 6.4 yhVj rFkk 21.6 yhVj gSA
3 : 4 : 5 gSA lcls NksVs xksys dh f=kT;k (ls-eh- esa ) gksxhµ
rnuqlkj mudh vkarfjd f=kT;kvksa dk vuqikr fdruk gS \ (1) 3 (2) 6 (3) 1.5 (4) 4
(1) 4 : 9 (2) 16 : 81 (3) (SSC CGL Prelim Exam. 13.11.2005 (Second Sitting)
2 : 3 (4) 2 : 3
45. A hollow spherical metallic ball has an external
(SSC CGL Tier-I Exam. 19.10.2014, IInd Sitting)
1
88 diameter 6 cm and is cm thick. The volume of
41. The volume of a sphere is
21
a14f 3
cm3. The 2

the ball (in cm3) is


FG Take =
22 IJ
curved surface area of the sphere is (Take S
22
)
H 7 K
7
èkkrq dh ,d [kks[kyh xsan dk ckgjh O;kl 6 lseh gS rFkk mldh
88
fdlh xksys dk vk;ru
21
b14g 3
lseh-3 gS A xksys dk oØ i`"Bh; 1
eksVkbZ 2 lseh- gS A bl xsan dk vk;ru (lseh-3 esa)µ

{ks=kiQy gS
FG 22
yhft,
IJ µ FG 22 IJ gS
H 7 K H 7
yhft,
K
(1) 2424 cm2 (2) 2446 cm2
2 2 2 2
(3) 2484 cm2 (4) 2464 cm2 (1) 41
3
(2) 37
3
(3) 47
3
(4) 40
3
(SSC CGL Prelim Exam. 11.05.2003 (Second Sitting)
(SSC CGL Prelim Exam. 08.02.2004 (Second Sitting)
42. Three solid metallic spheres of diameter 6 cm, 8
cm and 10 cm are melted and recast into a new 1
46. The diameter of the moon is assumed to be of
solid sphere. The diameter of the new sphere is : 4

èkkrq ds rhu Bksl xksyksa] ftuds O;kl 6 lseh-] 8 lseh- vkSj 10 the diameter of the earth. Then the ratio of the
volume of the earth to that of the moon is
lseh- gSa] dks fi?kykdj ,d u, Bksl xksys ds :i esa <kyk x;k
gSA u, xksys dk O;kl gS& 1
paæek dk O;kl] i`Foh ds O;kl dk 4 ekuk x;k gSA rnuqlkj]
(1) 4 cm (2) 6 cm
i`Foh rFkk paæek ds vk;ruksa dk vuqikr fdruk gksxk\
(3) 8 cm (4) 12 cm
(1) 64 : 1 (2) 1 : 64 (3) 60 : 7 (4) 7 : 60
(SSC CGL Prelim Exam. 24.02.2002 (First Sitting)
(SSC CHSL DEO & LDC Exam. 28.10.2012, Ist Sitting)
43. If the radius of a sphere is increased by 2 cm. its
47. A solid metallic sphere of radius 8 cm is melted
surface area increased by 352 cm2. The radius of to form 64 equal small solid spheres. The ratio of

sphere before change is : use


FG 22 IJ the surface area of this sphere to that of a small
H 7 K sphere is
8 lseh- v¼Z O;kl okys ,d Bksl /krq ds xksys dks fi?kyk dj
;fn fdlh xksys dh f=kT;k esa 2 lsehú dh o`f¼ dh tk, rks mlosQ
64 cjkcj NksVs Bksl xksys cuk, x, gSaA bl xksys ds i`"Bh;
i`"Bh; {ks=kiQy esa 352 lsehú2 dh o`f¼ gks tkrh gS A ifjorZu
{ks=kiQy dk ,d NksVs xksys ds i`"Bh; {ks=kiQy ls vuqikr gksxkµ
ls igys xksys dh f=kT;k Fkhµ
F 22
dk iz; ksx dhft,A I (1) 4 : 1 (2) 1 : 16 (3) 16 : 1 (4) 1 : 4
H 7 K (SSC CGL Prelim Exam. 27.07.2008 (First Sitting)

BLAM–406
{ks=kfefr
48. The radii of two solid iron spheres are 1 cm and 6
1
cm respectively. A hollow sphere is made by (1) th of the radius of the hemisphere
4
melting the two spheres. If the external radius of
the hollow sphere is 9 cm, then its thickness (in (2) radius of the hemisphere
cm) is
1
Bksl yksgs s ds nks xksydksa dh f=kT;k,a Øe'k% 1 lseh- vkSj 6 lseh- (3)
2
of the radius of the hemisphere
gSaA nksuksa xksydksa dks xykdj ,d [kks[kyk xksyd cuk;k tkrk gSA
;fn [kks[kys xksyd dh cká f=kT;k 9 lseh- gS] rks bldh eksVkbZ 1
(4) th of the radius of the hemisphere
6
(lseh- esa) fdruh gS\
(SSC CGL Tier-II Exam, 25.10.2015, TF No. 1099685)
(1) 2 (2) 1.5 (3) 0.5 (4) 1
53. The ratio of weights of two spheres of different
(SSC CGL Tier-II Exam, 25.10.2015, TF No. 1099685)
materials is 8 : 17 and the ratio of weights per 1
49. A hemispherical bowl has 3.5 cm radius. It is to
cc of materials of each is 289 : 64. The ratio of
be painted inside as well as outside. The cost of
radii of the two spheres is
painting it at the rate of Rs. 5 per 10 sq. cm will
be:
fHkUu&fHkUu lkefxz;ksa ds nks xksyksa ds otuksa dk vuqikr 8 : 17
gS vkSj izR;sd dh lkexzh ds izfr ?ku lseh otu dk vuqikr
,d v¼Z&xksykdkj dVksjs dh f=kT;k 3.5 lseh- gSA bls Hkhrj vkSj
289 : 64 gSA nksuksa xksyksa dh f=kT;kvksa dk vuqikr D;k gksxk \
ckgj ls jaxk tkuk gSA 5 #i, izfr 10 oxZ lseh- dh nj ij bldh
(1) 8 : 17 (2) 4 : 17 (3) 17 : 4 (4) 17 : 8
jaxkbZ dh ykxr D;k vk,xh\ (SSC CGL Tier-I Re-Exam. (2013) 20.07.2014 (IInd Sitting)
(1) 77 (2) 100 (3) 175 (4) 50 54. A hemisphere and a cone have equal base. If their
(SSC CHSL (10+2) LDC, DEO & PA/SA heights are also equal, the ratio of their curved
Exam, 15.11.2015 (Ist Sitting) TF No. 6636838) surface will be :
50. A hemispherical bowl has internal radius of 6 cm. ,d v¼Zxksys vkSj ,d 'kadq ds vkèkkj cjkcj gSaA ;fn mudh
The internal surface area would be : (Take S ¾ 3-
Å¡pkbZ Hkh cjkcj gks] rks muds oØ i`"Bksa dk vuqikr gksxk&
14)
,d v/Z xksykdkj dVksjs dh vkarfjd f=kT;k 6 lseh gSA vakrfjd (1) 1 : 2 (2) 2 :1
i`"Bh; {ks=kiQy fdruk gksxk \ (ekuk S ¾ 3-14) (3) 1 : 2 (4) 2 : 1
(1) 225 cm2 (2) 400 cm2 (SSC CGL Prelim Exam. 24.02.2002 & 13.11.2005
(Ist Sitting) & (IInd Sitting) & (SSC CHSL DEO &
(3) 289-75 cm2 (4) 226-08 cm2 LDC Exam. 11.12.2011 (Ist Sitting) (Delhi Zone)
(SSC CHSL (10+2) LDC, DEO & PA/SA 55. Two iron sheets each of diameter 6 cm are
Exam, 06.12.2015 (IInd Sitting) TF No. 3441135)
immersed in the water contained in a cylindrical
51. A ball of lead 4 cm in diameter is covered with vessel of radius 6 cm. The level of the water in the
gold. If the volume of the gold and lead are equal, vessel will be raised by
then the thickness of gold [given 3
2 1.259] is O;kl 6 lseh- okyh nks yksgs dh xksfy;k¡ ,d 6 lseh- v¼Z O;kl
approximately okys ,d csyukdkj cjru esa Mkys x, ikuh esa MqcksbZ tkrh gSaA
4 lsehO;kl okys ysM osQ xksys dks Lo.kZ ls doj fd;k tkrk gSA cjru esa ikuh dk ry fdruk mQij mBsxk \
;fn Lo.kZ vkSj ysM dk vk;ru cjkcj gS rks Lo.kZ dh eksVkbZ (1) 1 cm (2) 2 cm
(3) 3 cm (4) 6 cm
yxHkx fdruh gksxh \ [fn;k gS fd 3
2 1.259]
(SSC CGL Prelim Exam. 27.07.2008 (First Sitting)
(1) 5.038 cm (2) 5.190 cm 56. A cylindrical rod of iron whose height is eight
(3) 1.038 cm (4) 0.518 cm times its radius is melted and cast into spherical
(SSC CGL Tier-II Exam. 21.09.2014) balls each of half the radius of the cylinder. The
52. If a hemisphere is melted and four spheres of equal number of such spherical balls is
volume are made, the radius of each sphere will yksgs dh ,d csyukdkj NM+] ftldh mQapkbZ] mldh f=kT;k dh
be equal to vkB xquh gS] fi?kykdj ml csyu dh vk/h f=kT;k okyh
;fn ,d xksyk/Z dks xykdj leku vk;ru ds pkj xksyd cuk, xksykdkj xsanksas esa cny nh tkrh gSA rnuqlkj bl izdkj cukbZ
tkrs gSa] rks izR;sd xksyd dh f=kT;k fdlds cjkcj gksxh\ xbZ xksykdkj xsanksa dh la[;k fdruh gksxh \

BLAM–407
{ks=kfefr
(1) 12 (2) 16 60. A right cylindrical vessel is full with water. How
(3) 24 (4) 48 many right cones having the same diameter and
(SSC CHSL DEO & LDC Exam. 04.12.2011 height as that of the right cylinder will be needed
(IInd Sitting (North Zone)
22
57. The ratio of the volume of a cube and of a solid to store that water ? (Take S = ).
7
sphere is 363 : 49. The ratio of an edge of the
cube and the radius of the sphere is (taking ,d leyac csyukdkj crZu] ikuh ls Hkjk gqvk gSA mlh ikuh dks
22 j[kus ds fy,] ml crZu ds cjkcj O;kl rFkk mQ¡pkbZ okys fdrus
)
7
yac 'kadqvksa dh vko';drk gksxh \
FG 22
ekudj IJ
,d ?ku vkSj ,d Bksl xksys ds vk;ruksa dk vuqikr 363 : 49
H 7 K
gSA rnuqlkj ml ?ku ds ,d fljs dh yEckbZ vkSj xksys dh f=kT;k (1) 4 (2) 2 (3) 3 (4) 5
(SSC Delhi Police S.I. (SI) Exam. 19.08.2012)
22
dk vuqikr fdruk gksxk\ ( ekudj) 61. The radii of the base of a cylinder and a cone
7
are in the ratio 3 : 2 and their heights are in
(1) 7 : 11 (2) 22 : 7
(3) 11 : 7 (4) 7: 22 the ratio 2 : 3 . Their volumes are in the ratio
(SSC CHSL DEO & LDC Exam. 04.12.2011 of
(Ist Sitting (East Zone)
,d csyu vkSj 'kad q ds vk/kjksa dh f=kT;kvksa dk vuqikr
58. From a right circular cylinder of radius 10 cm
and height 21 cm, a right circular cone of same 3 : 2 gS vkSj mudh mQ¡pkb;ksa dk vuqikr 2: 3 gSA
base-radius is removed. If the volume of the rnuqlkj] muds vk;ruksa dk vuqikr D;k gksxk\
remaining portion is 4400 cm3, then the height of
(1) 3: 2 (2) 3 3 : 2
22
the removed cone (taking S = ) is :
7 (3) 3:2 2 (4) 2 : 6

10 lseh- f=kT;k vkSj 21 lseh- mQ¡pkbZ okys ,d yEc&o`Ùkh; csyu (SSC GL Tier-I Exam. 11.11.2012 (Ist Sitting)

ls mlh vkèkkj f=kT;kokyk ,d yEco`Ùkh; 'kadq fudkyk x;k gSA 62. A solid spherical copper ball, whose diameter is
14 cm, is melted and converted into a wire having
;fn 'ks"k Hkkx dk vk;ru 4400 lseh3 gS] rks fudkys x, 'kadq dh diameter equal to 14 cm. The length of the wire is
22 rkacs dh ,d Bksl xksykdkj ckWy dks ftldk O;kl 14 ls-eh- gS]
mQ¡pkbZ gS% ( 7
ysa )
xykdj 14 lseh- ds cjkcj O;kl okys rkj esa cnyk tkrk gSA rkj
(1) 15 cm (2) 18 cm dh yackbZ fdruh gksxh\
(3) 21 cm (4) 24 cm 16
(1) 27 cm (2) cm
(SSC CHSL DEO & LDC Exam. 11.12.2011 3
(IInd Sitting (Delhi Zone)

59. If a solid cone of volume 27S cm3 is kept inside a 28


(3) 15 cm (4) cm
hollow cylinder whose radius and height are that 3
of the cone, then the volume of water needed to (SSC Constable (GD) Exam. 12.05.2013 Ist Sitting)
fill the empty space is 63. The volume of a cylinder and a cone are in the
;fn ,d Bksl 'kaoqQ dks] ftldk vk;ru 27S lseh3 gS] mlh osQ ratio 3 : 1. Find their diameters and then compare
them when their heights are equal.
cjkcj f=kT;k rFkk Å¡pkbZ okys ,d [kks[kys csyu esa j[kk tkrk
,d csyu vkSj ,d 'kadq ds vk;ru 3 : 1 ds vuqikr esa gaSA muds
gS] rks nksuksa osQ chp osQ fjDr LFkku dks Hkjus osQ fy, vko';d
O;kl Kkr djsa vkSj mudh rqyuk djsa tc mudh Å¡pkb;k¡
ikuh dk vk;ru fdruk gksxk\
cjkcj gSaA
(1) 3S cm3 (2) 18S cm3
(1) Diameter of cylinder = 2 times of diameter of cone
(3) 54S cm3 (4) 8lS cm3
(SSC Graduate Level Tier-II Exam.16.09.2012)
csyu dk O;kl 'kadq ds O;kl dk nqxquk

BLAM–408
{ks=kfefr
(2) Diameter of cylinder = Diameter of cone ,d [kks[kys xksykdkj [kksy dh vkarfjd ,oa ckgjh lrgksa ds
csyu dk O;kl 'kadq dk O;kl 8
O;kl Øe'k% 6 lseh- vkSj 10 lseh- gSaA ;fn bls xykdj
(3) Diameter of cylinder > Diameter of cone 3
csyu dk O;kl 'kadq dk O;kl lseh- yackbZ dk Bksl csyu cuk;k tkrk gS] rks csyu dk O;kl
(4) Diameter of cylinder < Diameter of cone (lseh- esa) fdruk gksxk \
csyu dk O;kl 'kadq dk O;kl (1) 10 (2) 14
(SSC CHSL DEO & LDC Exam. 20.10.2013) (3) 16 (4) 7
64. The radius of cross-section of a solid cylindrical (SSC CGL Tier-II Exam, 2014 12.04.2015
(Kolkata Region) TF No. 789 TH 7)
rod of iron is 50 cm. The cylinder is melted down
and formed into 6 solid spherical balls of the same 68. A cone, a cylinder and a hemisphere stand on
radius as that of the cylinder. The length of the equal bases and have equal heights. The ratio of
rod (in metres) is their volumes is
yksgs dh ,d Bksl csyukdkj NM+ ds vuqizLFk&[k.M dh f=kT;k ,d 'kadq] ,d csyu vkSj ,d xksyk¼Z leku vk/kj ij j[ks x,
50 lseh gSA ml csyu dks fi?kykdj] 6 ,slh xksykdkj xsanksa esa gSa vkSj mudh Å¡pkbZ cjkcj gSA muds vk;ru dk vuqikr D;k
<kyk x;k gS] ftudh f=kT;k csyu dh f=kT;k ds cjkcj gSA gksxk \
rnuqlkj] ml NM+ dh yackbZ fdrus ehVj gksxh\ (1) 2 : 3 : 1 (2) 2 : 1 : 3
(1) 0.8 (2) 2 (3) 1 : 3 : 2 (4) 1 : 2 : 3
(3) 3 (4) 4 (SSC CGL Tier-II Exam, 2014 12.04.2015
(Kolkata Region) TF No. 789 TH 7)
(SSC CHSL DEO & LDC Exam. 27.10.2013 IInd Sitting)
69. A cylindrical vessel of diameter 24 cm contains
65. The rain water from a roof 22 m × 20 m drains
some water. If two spheres of radii 6 cm each are
into a cylindrical vessel having a diameter of 2 lowered into the water until they are completely
m and height 3.5 m. If the vessel is just full, immersed, then the water level (in cm) in the vessel
then the rainfall (in cm) is :
will rise by
fdlh 22 ehVj × 20 ehVj lkbt okyh Nr ls o"kkZ dk ikuh 2
24 lseh- O;kl okys ,d csyukdkj crZu esa dqN ikuh gSA ;fn 6
ehVj O;kl rFkk 3.5 ehVj Å¡pkbZ okys ,d csyukdkj crZu esa lseh- f=kT;k okys nks xksys ikuh esa rc rd Mqcks, tk,¡ tc rd
fxjrk gSA ;fn crZu Bhd iwjk Hkj x;k gS] rks lseh- esa o`f"V gS% os iwjh rjg Mwc u tk,¡ rks crZu esa ikuh dk Lrj fdruk
(1) 2 (2) 2.5 (3) 3 (4) 4.5 (lseh- esa) c<+ tk,xk \
(SSC CHSL DEO & LDC Exam. 27.11.2010)
(1) 12 (2) 6
66. If S1 and S2 be the surface area of a sphere and
(3) 4 (4) 9
the curved surface area of the circumscribed
(SSC CGL Tier-II Exam, 2014 12.04.2015
cylinder respectively, then S1 is equal to
(Kolkata Region) TF No. 789 TH 7)
;fn Sl rFkk S2 Øe'k% ,d xksys dk i`"B rFkk mlds ifjxr 70. From a solid right circular cylinder of length 4
csyu dk oØi`"B iznf'kZr djrsa gksa] rks S1 cjkcj gksxk cm and diameter 6 cm, a conical cavity of the
same height and base is hollowed out. The whole
3 1 surface of the remaining solid (in square cm.) is
(1) S (2) S
4 2 2 2
,d 4 lseh yackbZ vkSj 6 lseh O;kl okys Bksl leo`Ùkkdkj csyu
2 ls mruh gh Å¡pkbZ vkSj ry okyk 'kaDokdkj fooj dkVk tk, rks
(3) S (4) S2
3 2 'ks"k Bksl csyu dh lrg (oxZ lseh- esa) fdruh gksxh \
(SSC CGL Prelim Exam. 27.07.2008 (Second Sitting) (1) 48 S (2) 15 S
67. The diameters of the internal and external surfaces
(3) 63 S (4) 24 S
of a hollow spherical shell are 6 cm and 10 cm
(SSC CGL Tier-II Exam. 12.04.2015 TF No. 567 TL 9)
respectively. If it is melted and a solid cylinder of
71. A spherical ball of radius 1 cm is dropped into a
8 conical vessel of radius 3 cm and slant height 6
length cm is made, then the diameter (in cm)
3 cm. The volume of water (in cm3), that can just
of the cylinder is immerse the ball, is

BLAM–409
{ks=kfefr
;fn 3 lseh f=kT;k okys vkSj 6 lseh frjNh Å¡pkbZ okys fdlh 76. If each edge of a cube is increased by 40%, the
percentage increase in its surface area is
'kaDokdkj crZu esa 1 lseh dh f=kT;k okyh xksykdkj oLrq fxjkbZ
tk, rks ikuh dk fdruk vk;ru (lseh3 esa) ml oLrq dks Mqcks ;fn ,d ?ku ds izR;sd fljs esa 40% dh o`f¼ dj nh tk,] rks
ldrk gS\ mlds i`"Bry ds {ks=kiQy esa fdrus izfr'kr o`f¼ gks tk,xh\
(1) 40% (2) 60%
5 (3) 80% (4) 96%
(1) (2)
3 3 (SSC Multi-Tasking Staff Exam. 10.03.2013, Ist Sitting : Patna)
77. If the radius of a right circular cylinder is decreased
4
(3) 3 S (4) by 50% and its height is increased by 60%, its
3
volume will be decreased by
(SSC CGL Tier-II Exam. 12.04.2015 TF No. 567 TL 9)
;fn fdlh yEc o`Ùkkdkj csyu ds v¼ZO;kl esa 50% dh deh
72. The percentage increase in the surface area of a
rFkk bldh mQ¡pkbZ esa 60% dh o`f¼ dh tk,] rks blds vk;ru
cube when each side is doubled is
esa deh gks tk;sxh
,d ?ku ds izR;sd fdukjs dks nqxquk dj nsus ij ik'oZ ds
(1) 10% (2) 60%
lrgh {ks=kiQy esa gksus okyh o`f¼ izfr'krrk gS
(3) 40% (4) 20%
(1) 50% (2) 200% (SSC CGL Prelim Exam. 04.02.2007 (Second Sitting)
(3) 150% (4) 300% 78. The length, breadth and height of a cuboid are in
(SSC CGL Tier-I Exam, 09.08.2015 the ratio 1 : 2 : 3. If they are increased by 100%,
(IInd Sitting) TF No. 4239378)
200% and 200% respectively, then compared to
73. If water is freezed to become ice, its volume is the original volume the increase in the volume of
increased by 10%, then if the ice is melted to the cuboid will be
water again, its volume will be decreased by :
,d ?kukHk dh yEckbZ] pkSM+kbZ rFkk mQ¡pkbZ 1 : 2 : 3 ds vuqikr
;fn ciQZ cukus ds fy, ikuh dks tek;k tkrk gS rks mldk
esa gSaA ;fn buesa Øe'k% 100%, 200% rFkk 200% dh o`f¼
vk;ru 10» c<+ tkrk gS] fiQj ls ;fn ciQZ dks ikuh cukus dh tk;] rks vkjafHkd vk;ru dh rqyuk esa] ?kukHk ds vk;ru esa
ds fy, xyk;k tk, rks mldk vk;ru fdruk ?kV tk,xk \ fdruh o`f¼ gksxh \
1 1 (1) 5 times (2) 18 times
(1) 9% (2) 9 % (3) 8% (4) 9 %
11 2 (3) 12 times (4) 17 times
(SSC CHSL (10+2) LDC, DEO & PA/SA Exam, 06.12.2015 (SSC CGL Prelim Exam. 04.02.2007 (Second Sitting)
(IInd Sitting) TF No. 3441135) 79. 5 persons will live in a tent. If each person requires
74. If the radius of a cylinder is decreased by 50 % 16m2 of floor area and 100m3 space for air then
and the height is increased by 50 %, then the the height of the cone of smallest size to
change in volume is accommodate these persons would be
;fn ,d csyu dh f=kT;k 50% de dj nh tkrh gS vkSj 5 O;fDr ,d VsUV esa jgsaxsA ;fn izR;sd O;fDr dks 16 oxZ ehVj
Å¡pkbZ 50% c<+k nh tkrh gS] rks vk;ru esa fdruk ifjorZu iQ'khZ; {ks=k vkSj ok;q ds fy, 100 ?ku ehVj ds vUrjky dh
gksxk \ vko';drk gS rks mu O;fDr;ksa dks mleas txg nsus ds fy, y?kqre
(1) 52.5 % (2) 67.5 % vkdkj ds 'kadq dh Å¡pkbZ fdruh gksxh \
(3) 57.5 % (4) 62.5 % (1) 16 metre (2) 10.25 metre
(SSC CHSL DEO & LDC Exam. 02.11.2014 (IInd Sitting) (3) 20 metre (4) 18.75 metre
75. If the diameter of a circle is increased by 8%, then (SSC CGL Tier-I Exam, 16.08.2015 (Ist Sitting) TF No. 3196279)
its area is increased by : 80. A conical cup is filled with ice-cream. The ice-
;fn fdlh o`Ùk dk O;kl 8% c<+k fn;k tk,] rks {ks=kiQy esa cream forms a hemispherical shape on its open
top. The height of the hemispherical part is 7 cm.
fdruh o`f¼ gksxh \ The radius of the hemispherical part equals the
(1) 16.64% (2) 6.64% height of the cone. Then the volume of the ice-
(3) 16% (4) 16.46%
(SSC Multi-Tasking Staff Exam. 10.03.2013) cream is
LM 22 OP
N 7 Q
BLAM–410
{ks=kfefr
,d 'kaDokdkj di vkblØhe ls Hkjk gSA vkblØhe vius [kqys dVksjs dks [kkyh djus ds fy, fdruh cksryksa dh vko';drk
'kh"kZ ij vèkZxksykdkj vkÑfr cukrh gSA vèkZxksykdkj Hkkx dh gksxh \
Å¡pkbZ 7 leh gSA vèkZxksykdkj Hkkx dh f=kT;k 'kaoqQ dh Å¡pkbZ (1) 30 (2) 40
(3) 50 (4) 60
osQ cjkcj gS rks vkblØhe dk vk;ru fdruk gksxk\
LM 22 OP (SSC CHSL DEO & LDC Exam. 16.11.2014)
N 7 Q 84. A flask in the shape of a right circular cone of
(1) 1078 cubic cm (2) 1708 cubic cm height 24 cm is filled with water. The water is
(3) 7108 cubic cm (4) 7180 cubic cm poured in right circular cylindrical flask whose
(SSC CGL Tier-II Exam. 21.09.2014)
1
81. A large solid sphere is melted and moulded to form radius is rd of radius of the base of the circular
3
identical right circular cones with base radius and
cone. Then the height of the water in the
height same as the radius of the sphere. One of
cylindrical flask is
these cones is melted and moulded to form a
smaller solid sphere. Then the ratio of the surface ,d ÝykLd (FkeZl) tks yEc o`Ùkh; 'kadq vkdkj dk gS vkSj
area of the smaller to the surface area of the larger mldh Å¡pkbZ 24 ls- eh- gS] ikuh ls Hkjk gSA bl ikuh dks ,d
sphere is yEc o`Ùkh; csyukdkj ÝykLd esa mM+syk tkrk gS] ftldh f=kT;k
,d cM+s Bksl xksys dks xykdj le:i yac o`Ùkkdkj 'kaoqQ cuk, 1
tkrs gSa ftudh vkèkkj dh f=kT;k vkSj Å¡pkbZ xksys dh f=kT;k osQ o`Ùkh; 'kadq ds rys dh f=kT;k dk 3 gSA rks csyukdkj ÝykLd esa
cjkcj gSA buesa ls ,d 'kaoqQ dks xykdj NksVk Bksl xksyk cuk;k ikuh dh Å¡pkbZ D;k gksxh \
tkrk gSA NksVs xksys osQ i`"B {ks=kiQy vkSj cM+s xksys osQ i`"B {ks=kiQy
(1) 32 cm (2) 24 cm
dk vuqikr D;k gksxk \
(3) 48 cm (4) 72 cm
4 3 (SSC CHSL DEO & LDC Exam. 9.11.2014)
(1) 1 : 33 (2) 1 : 22 85. Two cubes of sides 6 cm each are kept side by
side to form a rectangular parallelopiped. The
2 4
area (in sq. cm) of the whole surface of the
(3) 1 : 3 3 (4) 1 : 2 3 rectangular parallelopiped is
(SSC CGL Tier-II Exam. 21.09.2014) 6 lseh Hkq tkvksa okys nks ?kuksa dks ikl&ikl j[kdj ,d
82. A sphere is cut into two hemispheres. One of them vk;rkdkj lekarj "kV~iQyd cuk;k x;k gSA rnuqlkj] ml
is used as bowl. It takes 8 full bowls to fill a
vk;rkdkj lekarj "kV~iQyd ds dqy i`"B dk {ks=kiQy (oxZ
conical vessel of height 12 cm and radius 6 cm.
The radius of the sphere (in centimetre) will be lseh esa) fdruk gksxk \
(1) 432 (2) 360
,d xksyk nks xksyk¼ks± esa dVk gqvk gSSA muesa ls ,d dk mi;ksx
(3) 396 (4) 340
dVksjs ds :i esa fd;k tkrk gSA 12 lseh mQ¡pkbZ vkSj 6 lseh f=kT;k
(SSC GL Tier-I Exam. 11.11.2012, Ist Sitting)
okys ,d 'kaDokdkj crZu dks Hkjus ds fy, blds 8 dVksjs yxrs 86. An elephant of length 4 m is at one corner of a
gSaA xksys dh f=kT;k (lsVa hehVj) essa fdruh gksxh \ rectangular cage of size (16 m × 30 m) and faces
(1) 3 (2) 2 towards the diagonally opposite corner. If the
(3) 4 (4) 6 elephant starts moving towards the diagonally
opposite corner it takes 15 seconds to reach this
(SSC CGL Tier-I Exam. 19.10.2014 (Ist Sitting)
corner. Find the speed of the elephant.
83. A hemispherical bowl of internal radius 15 cm
contains a liquid. The liquid is to be filled into ,d 4 ehVj yEck gkFkh (16 ehVj × 30 ehVj) vkdkj ds
cylindrical shaped bottles of diameter 5 cm and vk;rkdkj fiatjs ds ,d dksus esa gSA ;fn gkFkh fod.kZr%
height 6 cm. The number of bottles required to lEeq[k dksus dh vksj tkuk vkjaHk djrk gS] rks mls ml dksus
empty the bowl is
esa igq¡pus esa 15 lsd aM yxrs gSaA gkFkh dh xfr Kkr djsa A
,d v/Zxksykdkj dVksjk gS ftlesa rjy inkFkZ Hkjk gS vkSj mldh (1) 1 m/sec (2) 2 m/sec
vkUrfjd f=kT;k 15 lseh gSA ml rjy inkFkZ dks 5 lseh O;kl
(3) 1.87 m/sec (4) 1.5 m/sec
vkSj 6 lseh mQ¡pkbZ okyh csyukdkj cksryksa esa Hkjk tkrk gSA ml (SSC CHSL DEO & LDC Exam. 02.11.2014 (IInd Sitting)

BLAM–411
{ks=kfefr
87. The radius of base of solid cone is 9 cm and its 90. A right circular cylinder has height as 18 cm and
height is 21 cm. It cut into 3 parts by two cuts, radius as 7 cm. The cylinder is cut in three equal
which are parallel to its base. The cuts are at parts (by 2 cuts parallel to base). What is the
height of 7 cm. and 14 cm. from the base
percentage increase in total surface area?
respectively. What is the ratio of curved surface
areas of top, middle and bottom parts respectively? ,d yEcor xksykdkj csyu dh yEckbZ 18 lseh rFkk f=kT;k 7
,d Bksl 'kadq ds vk/kj dh f=kT;k 9 lseh gS rFkk mldh mQ¡pkbZ lseh gSA csyu dks rhu cjkcj Hkkxksa eas dkVk tkrk gS (vkèkkj ds
21 lseh gSA bls nks dVko] tks vk/kj ds lekarj gS ls 3 Hkkxksa lekarj 2 dVko }kjk)A dqy i`"Bh; {ks=kiQy eas fdrus izfr'kr dh
eas dkVk x;kA dVko vk/kj ls Øe'k% 7 lseh rFkk 14 lseh dh o`f¼ gqbZ gS\
mQ¡pkbZ ij gSaA Øe'k% mQijh] eè; rFkk fupys Hkkxksa ds oØ i`"Bh; (1) 62 (2) 56
{ks=kiQy dk vuqikr D;k gS\ (3) 48 (4) 52
(1) 1 : 4 : 8 (2) 1 : 3 : 5 (SSC CHSL DEO & LDC Exam. 20.02.2018)
(3) 1 : 3 : 9 (4) 1 : 6 : 12 91. The curved surface area of a cylinder is 594 cm2
(SSC CGL Tier-II Exam. 20.02.2018)
and its volume is 1336.5 cm3. What is the height
88. Radius of base of a hollow cone is 8 cm. and its (in cm.) of the cylinder?
height is 15 cm. A sphere of largest radius is put
inside the cone. What is the ratio of radius of ,d csyu dk oØ i`"Bh; {ks=kiQy 594 lseh gS rFkk mldk
base of cone to the radius of sphere? vk;ru 1336.5 lseh2 gSA csyu dh mQ¡pkbZ (lseh esa) D;k gS\
,d [kks[kys 'kadq ds vk/kj dh f=kT;k 8 lseh- rFkk mldh (1) 14 (2) 21
mQ¡pkbZ 15 lseh- gSaA lcls cM+h f=kT;k okyk ,d xksyk ml 'kadq (3) 24 .5 (4) 10.5
esa Mkyk tkrk gSA 'kadq ds vkèkkj dh f=kT;k dk xksys dh f=kT;k (SSC CGL Tier-II Exam. 09.03.2018)
ls D;k vuqikr gS\ 92. A solid metal sphere has radius 14 cm. It is melted
(1) 5 : 3 (2) 4 : 1 to form small cones of radius 1.75 cm. and height
(3) 2 : 1 (4) 7 : 3 3.5 cm. How many small cones will be obtained
(SSC CHSL DEO & LDC Exam. 17.02.2018) from the sphere?
89. A solid cylinder has radius of base 14 cm. and ,d /krq ds Bksl xksys dh f=kT;k 14 lseh gSA bls fi?kykdj
height 15 cm. 4 identical cylinders are cut from
each base as shown in the given figure. Height of 1.75 lseh f=T;k rFkk 3.5 lseh mQ¡pkbZ okys 'kadqvksa esa cuk;k
small cylinder is 5 cm. What is the total surface x;kA xksys ls fdrus NksVs 'kadq cuk, tk ldrs gSa\
area (in cm2.) of the remaining part?
(1) 512 (2) 256
,d Bksl csyu dss vk/kj dh f=kT;k 14 lseh- rFkk mQ¡pkbZ 15 (3) 1024 (4) 2048
lseh- gSA tSlk fd vkÑfr esa n'kkZ;k x;k gS fd blds izR;sd (SSC CGL Tier-II Exam. 09.03.2018)
vkèkkj ls 4 leku csyu dkVs gSaA NksVs csyu dh mQ¡pkbZ 5 lseh- 93. A metallic hemispherical bowl is made up of steel.
gSA 'ks"k Hkkx dk dqy i`"Bh; {ks=kiQy (lseh-2 esa) D;k gS\ The total steel used in making the bowl is 342
cm3. The bowl can hold 144 cm3. water. What is
the thickness (in cm.) of bowl and the curved
surface area (in cm2.) of outer side?
bLikr ls /krq dk ,d v/Zxksykdkj dVksjk cuk;k x;k gSA dVksjs
dks cukus esa dqy 342 lseh3 bLrikr dk iz;ksx fd;k x;k gSA
dVksjk esa 144 lseh3 ty vk ldrk gSA dVksjs dh eksVkbZ (lseh
eas) rFkk ckgjh lrg dk oØ i`"Bh; {ks=kiQy (lseh2 eas) D;k gSa\
(1) 6,162 (2) 3,162
(1) 3740 (2) 3432
(3) 6,81 (4) 3,81
(3) 3124 (4) 2816
(SSC CGL Tier-II Exam. 09.03.2018)
(SSC CHSL DEO & LDC Exam. 17.02.2018)

BLAM–412
{ks=kfefr
94. The height of a cone is 45 cm. It is cut at a height
,d 'kadq dh mQ¡pkbZ 24 lseh gS rFkk vk/kj dk {ks=kiQy 154
of 15 cm. from its base by a plane parallel to its
base. If the volume of the smaller cone is 18480 lseh2 gSA 'kadq ds oØ i`"Bh; {ks=kiQy (lseh2 esa) D;k gS\
cm3.,then what is the volume (in cm3.) of the (1) 484 (2) 550 (3) 525 (4) 515
original cone? (SSC CGL Tier-II Exam. 21.02.2018)

,d 'kadq dh mQ¡pkbZ 45 lseh gSA bls vk/kj ls 15 lseh mQij 99. A right circular cylinder is formed. A = sum of
total surface area and the area of the two bases.
,d ry }kjk mlds vk/kj ds lekukarj dkVk tkrk gSA ;fn NksVs
B = the curved surface area of this cylinder. If
'kadq dk vk;ru 18480 lseh3 gS] rks ewy 'kadq dk vk;ru A : B = 3 : 2 and the volume of cylinder is 4312
(lseh3 esa) D;k gS\ cm3 , then what is the sum of area (in cm2 .) of
(1) 34650 (2) 61600 the two bases of this cylinder?
(3) 36960 (4) 62370 ,d le o`Ùkdkj csyu cuk;k tkrk gSA A = dqy i`"Bh; {ks=kiQy
(SSC CGL Tier-II Exam. 19.02.2018) dk ;ksx rFkk nks vk/kjksa dk {ks=kiQyA B = bl csyu dk oØ
95. The ratio of the curved surface area and total surface i`"Bh; {ks=kiQyA ;fn A : B = 3 : 2 rFkk csyu dk vk;ru
area of a right circular cylinder is 2 : 5. If the total
4312 lseh 3 gS] rks bl csyu ds nksuksa vk/kjksa ds {ks=kiQy (lseh2
surface area is 3080 cm2., then what is the volume
(in cm3.) of the cylinder ? esa) dk D;k ;ksx gS \
,d yEcor~ o`Ùkkdkj csyu ds oØ i`"Bh; {ks=kiQy rFkk dqy (1) 154 (2) 308
(3) 462 (4) 616
i`"Bh; {ks=kiQy dk vuqikr 2 : 5 gSA ;fn dqy i`"Bh; {ks=kiQy
(SSC CGL Tier-II Exam. 21.02.2018)
3080 lseh2 gS] rks csyu dk vk;ru (lseh3- eas) D;k gS\
100. A solid sphere has a radius 21 cm. It is melted
(1) 4312 6 (2) 3822 6 to form a cube. 20% material is wasted in this
process. The cube is melted to form hemisphere.
(3) 4522 6 (4) 4642 6 In this process 20% material is wasted. The
(SSC CGL Tier-II Exam. 19.02.2018) hemisphere is melted to form two spheres of equal
radius. 20% material was also wasted in this
96. The radius and height of a solid cylinder are
process. What is the radius (in cm.) of each new
increased by 2% each. What will be the
sphere?
approximate percentage increase in volume?
,d Bksl csyu dh f=kT;k rFkk mQ¡pkbZ izR;sd dks 2% ls c<+k;k ,d Bksl xksys dh f=kT;k 21 lseh gSA bls fi?kykdj ,d ?ku
tkrk gSA vk;ru esa yxHkx fdrus izfr'kr dh o`f¼ gksxh\ cuk;k tkrk gSA bl izfØ;k esa 20% lkexzh O;FkZ gks tkrh gSA ?ku
(1) 6.76 (2) 5.88
dks fi?kykdj ,d v¼Zxksyk cuk;k tkrk gSA bl izfØ;k eas 20%
(3) 6.12 (4) 3.34
lkexzh O;FkZ gks tkrh gSA v¼Zxksys dks fi?kykdj nks leku f=kT;k
(SSC CGL Tier-II Exam. 19.02.2018) okys nks xksys cuk, tkrs gSaA bl izfØ;k esa Hkh 20% lkexzh Hkh
97. Two identical hemispheres of maximum possible O;FkZ gks xbZ FkhA izR;sd u, xksys dh f=kT;k (lseh eas) D;k gS\
size are cut from a solid cube of side 14 cm. The
bases of the hemispheres are part of the two (1) 4.2 e 4j
3
(2) 2.1 e 2j
3

opposite faces of cube. What is the total volume


(in cm.3) of the remaining part of the cube?
(3) 2.1 e 4j
3
(4) 8.4 e 4j
3

nks le:i vf/dre laHko eki okys v¼Zxksyksa dks ,d 14 lseh


(SSC CGL Tier-II Exam. 21.02.2018)
Hkqtk okys Bksl ?ku ls dkVk tkrk gSA v¼Zxksyksa ds vk/kj ?ku
101. How many Brick (number near to next hundred)
ds nks foijhr iQyd ds Hkkx gSaA ?ku ds 'ks"k Hkkx dk dqy will be required to build a wall 30 m long, 30 cm
vk;ru (lseh3- eas) D;k gS\ thick and 5 m high with a provision of 2 doors,
(1) 1556.33 (2) 898.5 each 4 m × 2.5 m and each brick being 20 cm ×
(3) 1467.33 (4) 1306.67 16 cm × 8 cm when one ninth of the wall is filled
(SSC CGL Tier-II Exam. 19.02.2018) with time?
98. The height of a cone is 24 cm and the area of the ;fn nhokj 30 m yach] 30 cm eksVh rFkk 5 m Å¡ph gS rFkk
base is 154 cm2. What is the curved surface area blesa nks njokts gS ftldh yEckbZ 4 m pkSM+kbZ 2.5 m gS ;fn
(in cm2.) of the cone? izR;sd bZV 20 cm yEch 16 cm pkSM+h rFkk 8 cm Å¡ph gS rks
BLAM–413
{ks=kfefr
nhokj ls yxus okyh bZVksa dh la[;k (la[;k osQ vkf[kjh vad 31. (3) 32. (3) 33. (3) 34. (4) 35. (1) 36. (4)
37. (4) 38. (1) 39. (2) 40. (1) 41. (4) 42. (2)
1
lSdM+k osQ ikl gks) crkb,A ;fn nhokj dk 9 ok¡ Hkkx pwuk ls 43. (2) 44. (2) 45. (1) 46. (1) 47. (4) 48. (2)
49. (2) 50. (2) 51. (1) 52. (1) 53. (2) 54. (3)
Hkjk gS\ 55. (3) 56. (3) 57. (3) 58. (4) 59. (4) 60. (1)
(1) 13400 (2) 13600 61. (2) 62. (4) 63. (2) 64. (4) 65. (2) 66. (1)
(3) 13500 (4) 13900 67. (2) 68. (2) 69. (2) 70. (4) 71. (3) 72. (1)
102. A rectangular swimming pool of length 20 m and
breadth 10 m has a uniformly sloping floor. It is
Test-2
1 m in depth on one end of its length and 3 m at Questions Based on Area and perimeter of
the other. How much water in ne eded to four walls of a room
completely fill the pool ?
20 m dh yEckbZ vkSj 10 m dh pkSM+kbZ okys ,d vk;rkdkj 1. (1) 2. (4) 3. (3) 4. (3) 5. (3)
fLofeax iwy esa ,d leku :i ls >qdk gqvk iQ'kZ gSA bldh
yackbZ osQ ,d Nksj dh xgjkbZ 1 m vkSj nwljs Nksj dh xgjkbZ Test-3
3 m gSA bl iwy dh iwjh rjg ls Hkjus osQ fy, fdrus ikuh dh
Questions Based on Surface Area &
Volume of Cube and Cuboid
t:jr gksxh\
(1) 800 cm (2) 300 cm 1. (1) 2. (3) 3. (3) 4 .(3) 5. (1) 6. (3)
(3) 480 cm (4) 400 cm 7. (4) 8. (2) 9. (1) 10. (2) 11. (4) 12. (2)
103. The Shape of an object is a right circular cylinder 13. (3) 14. (2) 15. (2) 16. (1) 17. (4) 18. (3)
with a hemi-sphere on bottom and a right circular
19. (1) 20. (4) 21. (2) 22. (2)
cone on the top. The radius of the cyclindrical
part is 5 cm and the height of cylinder part is 2.6
times the radius. What is the total height of the
Test-4
object, if the surface area of the object is 770 Questions Based on Cylinder, Cone, Sphere,
cm2? Hemisphere etc.
,d oLrq dk vkdkj bl izdkj gS ,d yacs o`Ùkh; csyu osQ
1. (4) 2. (3) 3. (3) 4. (3) 5. (4) 6. (2)
fupys fgLls ij v¼Zxksyk gSa ,oa Åij okys fgLls ij ,d yac
7. (1) 8. (1) 9. (4) 10. (1) 11. (3) 12. (3)
o`Ùkh; 'kaoqQ gSA csyukdkj Hkkx dh f=kT;k 5 cm gS vkSj
13. (2) 14. (1) 15. (3) 16. (4) 17. (1) 18. (2)
csyukdkj Hkkx dh Å¡pkbZ f=kT;k ls 2-6 xquk gSA ;fn oLrq dk
19. (2) 20. (2) 21. (1) 22. (2) 23. (4) 24. (1)
i`"B {ks=kiQy 770 lseh2 gS rks oLrq dh oqQy Å¡pkbZ D;k gksxh\
25. (2) 26. (4) 27. (4) 28. (4) 29. (3) 30. (4)
(1) 18 cm (2) 35 cm 31. (4) 32. (2) 33. (4) 34. (3) 35. (4) 36. (4)
(3) 12 cm (4) 30 cm 37. (3) 38. (1) 39. (3) 40. (4) 41. (4) 42. (4)
43. (4) 44. (1) 45. (3) 46. (1) 47. (3) 48. (4)
SHORT ANSWERS
49. (1) 50. (4) 51. (4) 52. (3) 53. (1) 54. (2)

Test-1 55. (2) 56. (4) 57. (2) 58. (3) 59. (3) 60. (3)
Questions Based on Area and perimeter of 61. (2) 62. (4) 63. (2) 64. (4) 65. (2) 66. (2)
Triangle, Quadrilaterals and Circle. 67. (2) 68. (3) 69. (3) 70. (1) 71. (1) 72. (4)
73. (2) 74. (4) 75. (1) 76. (4) 77. (2) 78. (4)
1. (1) 2. (2) 3. (2) 4. (1) 5. (2) 6. (1) 79. (4) 80. (1) 81. (4) 82. (1) 83. (4) 84. (4)
7. (4) 8. (2) 9. (3) 10. (3) 11. (3) 12. (1) 85. (2) 86. (2) 87. (2) 88. (1) 89. (2) 90. (2)
13. (4) 14. (2) 15. (1) 16. (3) 17. (1) 18. (2) 91. (2) 92. (3) 93. (1) 94. (4) 95. (1) 96. (3)
19. (4) 20. (4) 21. (1) 22. (4) 23. (2) 24. (1) 97. (4) 98. (2) 99. (2) 100. (4) 101. (2) 102. (4)
103. (4)
25. (1) 26. (4) 27. (2) 28. (2) 29. (3) 30. (3)

BLAM–414
{ks=kfefr
1
EXPLANATIONS 4. (1)
2
base × height = area

Test-1 1
5x 12x 270
2
Questions Based on Area and perimeter of
30x2 = 270
Triangle, Quadrilaterals and Circle.
x2 = 9
x=3
1
b1 h1 Two sides = 15 and 36
area of 1 2 4
1. (1) area of 2 1 3
b2 h2 hypotenuse = l 2  b2
2

b1 4 h2 4 4 152 36 2 225 1296


= 16 : 9
b2 3 h1 3 3 1521 39 cm
2. (2) igys triangle dk area 2
5. (2) AG = 9 u = 6 cm
1 3
= × 15 ×12 cm2
2 [  AG : GD = 2 : 1]
nwljs triangle dk area 15 × 12 cm A
(  nwljs triangle dk area igys ds nksxquk gS)
According to the question,
F E
1
× base × height
2 G
=15 × 12
B D C
1
× 20× h
2 2
BG = 12 = 8 cm
= 15 × 12 3
Required height 2
GC = 15 = 10 cm
15 u12 u 2 3
= = 18 cm
20 1
ar 'ABG = 6 8 = 24 sq. cm
3. (2) A 2
? Area of 'ABC = 3 × 24
16 2 cm
= 72 sq. cm.

B C 6. (1) A
AB = BC = x; (iz'u ls)
60° 10
AC = 16 2

2 30°
e
x2 + x2 = 16 2 j B
90°
C
2x2 = 16 × 16 × 2
x2 = 16 × 16 ekuk C = T rFkk A =2 T
Ÿ T + 2T + 90° = 180°
x = 16
3T= 90°
1 Ÿ T= 30°
Area of triangle = × base × height
2 ‘C= 30° and ‘A = 60°
1 'ABC ls
= 16 16
2 AB 1 AB
= 128 cm2 sin 30° =
AC 2 AC

BLAM–415
{ks=kfefr
1 1 ABC dk area
AB = AC = u10 5 cm
2 2
3 3
= × side2 = × 12 × 12
BC = 4 4
AC  AB
2 2
10  5
2 2

= 100  25 = 36 3 sq. cm
75 5 3 cm
1 2nd Method
Area of ABC = AB u BC
2 h = 6 3 cm

1 25 3 1
= u5u5 3 cm2
2 2 Area = h2
3
2nd Method
1
ratio fig. = 6 3 6 3
3

A = 36 3 cm

60° 8. (2) diagonal l 2  b2 25


2 2
l + b = 625 ...(i)
Area = lb = 168 ...(ii)
2 (l + b)2 = l2 + b2 + 2lb = 625 + 2 × 168 = 961
3
l + b = 961 = 31 ...(iii)
(l – b)2 = l2 + b2 – 2lb
= 625 – 168 × 2 = 289
l–b= 289 = 17 ....(iv)
30°
B C
(iii) rFkk (iv) ls]
1
2l = 48 l = 48 24 cm
2 10 2
1 5 (BC) 2nd Method
3 5 3 AB b g
1 25 3
= 5 5 3 =
2 2 25 7

7. (4) A

24

By phythagorean Triplet
B D C 7, 24, 25
AB = BC = AC = a cm l = 24 cm

AD = median = 6 3 cm 9. (3) Verandah dk area


= (25 + 2 × 3.5) (15 + 2 × 3.5) – 25 × 15
3a = 32 × 22 – 25 × 15
=6 3
2 = 704 – 375
= 329 sq. m
6 3 2
a= = 12 cm flooring dk [kpZ = 329 × 27.5
3
= 9047.5

BLAM–416
{ks=kfefr
10. (3) Area esa izfr'kr o`f¼ (i) Sum of sides in the same direction

FG
= 100 100
100 100 IJ
% = 300% AD + DC
15
17 metre
H 100 K 68
60
11. (3) Length of park = 3x metre (let) l + b = 17 ... (ii)
Breadth = 2x metre
Perimeter of park = Distance covered by cyclist
bl  b g 2
l 2  b 2  2lb

Park dk ifjeki = cyclist }kjk r; dh xbZ nwjh 289 = 169 + 2lb [(i) ls ] 2lb = 120 lb = 60
12 8 8
=
60
=
5
km bl bg bl bg 2 2
4 lb = 289 – 240 = 49

l–b=7 ...(iii)
F8
= GH 1000 m
IJ eq.(ii) rFkk (iii) ls]
5 K l = 12, b = 5
= 1600 m
area = 12 × 5 = 60 sq. m
According to the question,
2nd Method
2 (3x + 2x) = 1600
Phythagorean Triplet
10x = 1600
5, 12, 13 ls
1600
x= = 160 Area = 12 × 15 = 60 m2
10
Park dk area 14. (2) 38
= 3x × 2x = 6x2 = 6 × (160)2
= 153600 sq. m
12. (1) vk;rkdkj eSnku dk area 32

expenditure 1000
expenditure per sq. metre 1
4 ekuk fd path dh pkSM+kbZ = x m
= 4000 sq. metre Rectangular field dk area
area 4000 = 38 × 32 = 1216 m2
? length = 80 metre
side 50 Rectangular field dk area without path
field dh ubZ length = 100 metre = (38 – 2x ) (32 – 2x )
area = 100 × 50 = 5000 sq. m = 1216 – 64x – 76x +4x2
? Required [kpZ = 1216 – 140x + 4x2
FG
= 5000
1IJ Path dk area
H 4 K
= 1250
= 1216 – 1216 + 140x – 4x2
13. (4) D C = 140x – 4x2
140x – 4x2 = 600
13 m 4x2 – 140x + 600 = 0
x2 – 35x + 150 = 0
A B x2 – 30x –5x +150 = 0
x (x– 30) –5 (x –30) = 0
diagonal dh yackbZ (x– 5) (x –30) = 0
15 x = 5 because x 30
=AC 52 =13 metre
60 Aliter : path dk area = 2w( l + b – 2w)
= 600 = 2w(70 – 2w)
13 Ÿ l  b
2 2
l 2  b2 169 ... (i)
w=5

BLAM–417
{ks=kfefr
15. (1) ge tkurs gSa rhombus ds –diagonals ,d nwljs dks Sides = 50 rFkk 40 units
90° ij dkVrs ij gSaA 40 × h = 1000
1000
h 25 unit
40
19. (4) Area of parallelogram = base × height
= 27 × 12 = 324 sq. cm
Again 324 = 36 × h

rhombus dh side 324


h= 9 cm
36

=
Fd I Fd I
1
2
2
2
20. (4) Trapezium dk area
H 2K H 2K
1
tgk d1, d2 diagonals gS] =
2
(sum of parallel sides) × altitude

= 162 122 400 20 cm 1


450 = (3x + 2x) × 15
Perimeter of the rhombus = 20 × 4 = 80 cm 2
16. (3) 4 × length of a side = 40 cm 450 2
5x = 60 cm
length of a side 15

40
= = 10 cm 21. (1) D 5x C
4

D 10 cm C 24

m
6c
O A 3x B
cm

cm

Trapezium dk area
10

cm
10

6
1
(sum of parallel sides) × height
A 10 cm/ B 2

1
ge tkurs gSa rhombus ds diagonals ,d nwljs dks 90° ij 1440 (5x  3x ) u 24
2
dkVrs ij gSaA
1
AOB esa] 1440 u 8x u 24
2
OB = (10) 2 – (6) 2 1440 1440
8x x 15
= 12 12 8
100 – 36 64
The longer parallel side
= 8 cm
Diagonal BD = 8 × 2 = 16 cm = 15 × 5 = 75 m
17. (1) 3x + 4x + 5x + 6x = 72
22. (4) D
18x = 72
x=4
Largest side = 6x = 6 × 4 = 24 cm.
18. (2) ekuk parallelogram dh side gksxh 5x rFkk 4x C
Base × Height = Area of parallelogram
A O
5x × 20 = 1000
1000
x 10
5 20
B

BLAM–418
{ks=kfefr
Side of rhombus = side of square 2nd Method
1 P = 40
= 40 2 = 40 cm a = 10 cm
2
Area = a2 sin
AC BD; AOD = 90°
= (10)2 × sin 60°
AC = 3x rFkk BD = 4x cm
3
3x = 100 × = 50 3 cm2
AO = ; OD = 2x cm 2
2
AOD ls] 24. (1) rhombus dk ,d diagonal = d1 = x cm.
OA2 + OD2 = AD2 nwljk diagonal = d2 = 2x cm.
1
FG 3x IJ 2
4x 2 402 Rhombus dk area = 2 d1. d2
H 2K
9x2 + 16x2 = 1600 × 4 1
d .d = 256
25x2 = 6400 2 1 2
x2 = 6400 ÷ 25 = 256
1
x= x × 2x = 256
256 = 16 2
AC = 3 × 16 = 48 cm x2 = 256
BD = 4 × 16 = 64 cm x= 256 = 16 cm
rhombus dk area
cM+k diagonal
1 = 2x = 2 × 16 = 32 cm
= AC BD
2
25. (1) D
C
1
= 48 64 = 1536 sq. cm.
2 E

F
23. (2) A
B
60° A

AC = 24 metre
B D
BE = 8 metre
DF = 13 metre
Quadrilateral ABCD dk area
C
= ABC dk area + ACD dk area
A = 60° C = 60° 1 1
= × AC × BE + × AC × DF
360q120q 2 2
B= D= = 120°
2
1
ABD = ADB = 60° = (24 × 8 + 24 × 13)
2
ABD ,d equilateral triangle gSA
1
40 = × 24 (8 +13)
Now, side = = 10 cm 2
4
1
AB = AD = 10 cm = × 24 × 21 = 252 sq.metre
2
Rhombus dk area
26. (4) ekuk pDdjksa dh la[;k = n
3 wheel }kjk ,d pDdj esa r; dh xbZ nwjh
2 × Area of ABD = 2 × × (AB)2
4 22
2 r 2u u 21
7
3
=2× × 10 × 10 = 50 3 cm2 = 2 × 22 × 3 cm
4

BLAM–419
{ks=kfefr
So, 924 m = 924 × 100 cm, No. of revolution to 30. (3) nksuksa circle ds circumferences dk radii = 23 : 22.
travel the distance in n. nksuksa circle dk radii gksxk r rFkk R
924 m = 924 × 100 cm 2 R 23 R 23
924 100 rks 2 r 22 r 22
pDdjksa dh la[;k] n = = 700
2 22 3 Difference = 23 – 22 = 1
arc radius vr% 1 unit = 5 tgk¡ path dh pkSM+kbZ = 5 cm
27. (2) Trick : Sector dk area = 2 tgk¡ jkLrs dh pkSM+kbZ 5 ehVj gS A
5 u 3.5 So, radius of incircle
= 8.75 cm2
2 = 22 unit = 22 × 5 = 110 m
28. (2) A1 circle dk circumference = 2SR1 diameter = 2 × 110 = 220 m
88 7
88 = 2SR1 R1 14 cm 31. (3) B
2 22
A2 circle dk circumference = 2SR2
132 u 7 A D
Ÿ R2 21 cm C
2 u 22
Ring dk area R 22 – R12

22 ekuk circle dk radius = a units


eR 2
2 – R12 j 7
b gb
21 – 14 21  14 g
a2
22 u 7 u 35 Semi circle dk area = sq.units
= 35 × 22 = 770 cm 2
7
nksuksa triangles ABC rFkk BCD isosceles rFkk cjkcj
29. (3) ekuk internal radius = r
gS
external radius = R
ekuk fd ikoZQ dh vkarfjd f=kT;k r ,oa cká f=kT;k R gS A 1 2
izR;sd dk area a
2
(lM+d lfgr)
Internal rFkk external circumferences dk varj132 1 2
Area of both triangles 2 a a 2 sq.units
m. 2
2 R 2 r 132 Shaded region dk area
2 bR r g 132 a2
a2 a2
FG 1IJ sq. unit
2 H2 K
32. (3) ekuk original radius = r cm.
According to the question,
S(r+1)2 – Sr2 = 22
22
S (r2+2r+1–r2) = 22 (2r+1) 22
132 u 7 7
R–r 21 m
2 u 22 22 7
2r +1 7
Path dh pkSM+kbZ = 21 m 22
2nd Method 6
2r = 7–1= 6 r 3 cm
the breadth of path 2
7 33. (3) area in 60 minutes = Sr 2
bR r g 44
C1 b C2 g area in 6 minutes
7 r2 22 7 7
= 132 = 21 m = = = 15.4 cm2
44 10 7 10

BLAM–420
{ks=kfefr
34. (4) 37. (4)

A C
B
O

B A

AB = BC = CA = 2a cm ekuk pool dk radius = r m


ABC ,d equilateral gSA Shaded part dh pkSM+kbZ = 4 m
BAC = ACB = ABC = 60° OB = (r + 4) metre

3 According to the question,


ABC dk area =
4
b g
u side
2

11
S × OB2 – S × OA2 = S × OA2
25
3
= u 4a 2 = 3 a2 sq.cm.
4 11 2
(r + 4)2 – r2 = r
rhuksa sectors dk area 25

11 2
60 a2 r2 + 8r + 16 – r2 = r
= 3 a2 = sq.cm. 25
360 2
shaded region dk area 11 2
8r + 16 = r
25
F2 3– Ia 2 200r + 400 = 11r2
= 3a 2 –
2
a2 = GH 2 JK sq.cm.
11r2 – 200r – 400 = 0
11 r2 – 220r + 20r – 400 = 0
35. (1) 11r (r – 20) + 20 (r – 20) = 0
(r – 20) (11r + 20) = 0
r = 20 metre
20
 r – metre
11

38. (1)
Circular paper sheet dk radius
A B
352 352 352 7 8 cm 4 cm
= = = = 56 cm
2 22 2 22
2
7
56 Diameter = AB = 8 + 4 = 12 units
izR;sd plate dk radius = = 28 cm
2 12
Radius = = 6 units
izR;sd plate dk circumference = 2Sr 2
22 Area of circle
=2 28 = 176 cm
7 = Sr2 = S × 62 = 36S sq. unit
36. (4) Shaded portion dk area 39. (2) Larger circle dk radius = R cm
45 cM+s o`Ùk dh f=kT;k = R
= (R2 – r2)
360 Smaller circle dk radius = r cm
R – r = 14 cm rFkk S (R2 – r2) = 1056
1
= (42 – 32) = S × ×7×1
8 8 1056 1056 7
R2 – r2 = =
22
22 1 11 2
= × ×7= m R2 – r2 = 336
7 8 4

BLAM–421
{ks=kfefr
(R + r) (R – r) = 336 22
shaded circle dk area = × 11.2 × 11.2
336 7
R+r= = 24 cm
14 = 394.24 sq. cm.
(R + r) – (R – r) = 24 – 14
A E B
2r = 10 r = 5 cm 42. (2)
L M
40. (1) W X
F G
P Q
Z Y
O N
A B E F
D C G D H C
H

S
L M
R circle dk area = 38.5 sq. cm.
r2 = 38.5
O N 22
× r2 = 38.5
7
SQ = square dk diagonal =
circle dk diameter = 28 2 cm. 38.5 7
r2 = = 12.25
22
square dh side = 28 cm.
r= 12.25 = 3.5 cm.
NksVs square dh side
1
LM = circle dk diameter
=
2
e
28 2 – 28 j = 7 cm.
square LMNO dk area = 49 cm2
1
=
2
× 28 e j
2 – 1 = 14 × 0.4 = 5.6 cm. vanj okyk square ckgj okys square ds midpoints dks
join dj ds izkIr gqvk gSA blfy, NksVs square dk area cM+s
izR;sd NksVs square dk area = 31.36 sq. cm. square dk vk/k gSA
41. (4)
So, EFGH = 2 × LMNO = 98 cm2

ABCD = 2 EFGH = 2 × 98 = 196 cm2

LM 7
EL = cm.
2 2
AE = EF = 14 cm.
AF = 28 cm. (  EL = EM)
FC = 28 cm. 7
EF = 2 = 7 2 cm.
CO = R, CD = 2R; FD = x cm. 2
2R + x = 28 cm.
OH2 = HF2 + OF2 EF 7 2
(7 + R)2 = 72 + (R + x)2 AE = = = 7 cm.
2 2
(7 + R)2 – (R + x)2 = 49
AB = 2 AE = 14 cm.
(7 + 2R + x) (7 + R – R – x) = 49
(7 + 28) (7 – x) = 49 square ABCD dk area
= (14)2 = 196 sq. cm.
49 7
7–x=
35 5 P C Q
43. (2) B
7 28
x=7– = 5.6 cm. A
5 5
2R + x = 28
2R = 28 – 5.6 = 22.4 S O R
R = 11.2 cm. OA = OR = QR = 7 cm.

BLAM–422
{ks=kfefr
smaller circle dk radius = AB = x cm. BEFG dk area – ar DCG – ar ADE
BC = CQ = x cm. 1 1
= 82 – 62 – ×8×2– ×8×2
BQ = 2 2 2 2
BC CQ
= 64 – 36 – 8 – 8 = 12
= x 2
x 2 = 2x 2
45. (1) A
= 2x cm.
OQR, esa] 2
OR2 + QR2 = OQ2 x
72 + 72 = (OB + BQ)2 B
2 d C
2 × 72 = 7 e x 2x j D

;gk¡ AB = 2 cm
7 x 2x = 7 2
CD = 1 cm
(–ve ugha ysax)s
ekuk BC = x cm
x+ 2x = 7 2 – 7 ATQ,

x e 2 1 =7j e 2 –1 j centre okys circle dk area =


FG 2 x IJ LM r AC OP
2

H 2 K N 2 Q

x=
7e 2 – 1j F 3 x IJ LM R AD OP
2

e 2 1j ckgj okys circle dk area = G


H 2 K N 2 Q

7e 2 – 1j e 2 –1 j L 2 O
lcls vanj okys Circle dk area = 1 M r 2 1P
2
x= N Q
e 2 1j e 2 – 1j
F3 x I 2

7 e 2 –1 j
2 FG 2 x IJ = GH 2 JK
2

= H 2 K 2
2 –1

1F (3 x ) I2
e
= 7 2 1 – 2 2 cm. j (2 + x) = G1
2
2H 4 JK

=7 3–2 e 2 j = e21 – 14 2 j cm. 2 (4 + x2 + 4x) = 4 + 9 + x2 + 6x


8 + 2x2 + 8x = 4 + 9 + x2 + 6x
44. (2) ;gk¡, AB = BC = CD = AD = 8 cm x2 +2x – 5 = 0
D C
–2 22 – 4 1(–5)
G 2
F

–2 24
x=
2
A B
E x 6 –1
EF = EG = GB = EB = 6 cm
BC = 6 – 1 cm
GC = BC – BG
= 8 – 6 = 2 cm 46. (1) ;gk¡,
AE = AB – BE DP = 13 Units
= 8 – 6 = 2 cm CP = 10 Units
Shaded portion dk area BP = 6 Units
AP = ?
= ABCD dk area
BLAM–423
{ks=kfefr
D C 48. (2) P

20

A P B Q R

ABC esa, ;gk¡ PQ = QR


Pythagoras theorem ds }kjk QPR = QRP
2
CP = BC + PB 2 2 PQR = 90°
102 = BC2 + 62 QPR = QRP = 45°
BC2 = 100 – 36 Pythagoras theorem ds }kjk
PQ2 + QR2 = 202
BC = 64 2PQ2 = 400
BC = 8 Units PQ2 = 200
ABCD rectangle gS PQ = 10 2
AD = BC = 8 Units
1
ADP esa Triangle dk area = 2 × PQ × QR
PD2 = AD2 + AP2
132 = 82 + AP2 1
= ×
2
AP = 169 – 64 2 10 2 10 2
= 100 cm2
AP = 105 Units 2nd Method
47. (4) ;gk¡, 1 2 1
area of = h = 20 20 = 100 cm2
r1 + r2 = 91 cm. ... (1) 4 4
Sr12 – Sr22 = 2002

2002 49. (2) P Q


r1 2 – r2 2 =

O
2002 u 7
r1 2 – r 2 2 =
22
= 91 × 7
2 2
r1 – r2 = 637 ...(2) R
S
equation (2) dks (1), ls divide djus ij
;gk¡,
r12 r22 637 PQ = QR = SR = PS = 20 cm
r1 r2 91
PR = 202 202

br1 r2 r1gb r2 g =7 PR = 20 2
br1 r2 g OS = OQ = OP = OR

r1 – r2 = 7 ... (3) 20 2
= = 10 2
equation (1) rFkk (3), dks add djus ij 2
2r1 = 98 4 triangles dk perimeter
r1 = 49
r2 = 42
e
= 4 × 10 2 10 2 20 j
cM+s circle dk radius = 49 cm. = 80 2 + 80 cm

BLAM–424
{ks=kfefr
50. (2) Let the radius of C1 and C2 are r and R. and radius of the semi-circle = r
According to the Question So, In ABC

C3 r2 = a2 +
FG a IJ 2

H 2K
C2
5a 2 5 40
C1 r2 = = [Given a2 = 40]
4 4
= 50
r R
r2
Area of the semi-circle =
2

50
= = 25 cm2
2
6

(62 – R2) = (R2 – r2)


36 – R2 = R2 – r2 a a
54. (3)
36 r 2 a
R2 = a
2
and
(R2 – r2) = r2 a
a
R2 = 2r2
a a
36 r 2
2r 2
2

r = 2 3 cm Side of the square = 2r


51. (1) the perimeter of Given figure
diagonal = 2 2r
= (AB + BC + PC + PQ + QR + RA)
= AB + BC + (CP + QR) + (PQ + RA) diameter of the inner circle = 2 2r 2r
= AB + BC + AB + BC
= 2(AB + BC) = 2r e 2 1 j
= 2 (8 + 4) = 24 cm
52. (1) Area left after laying black tiles
= (20 – 4) × (10 – 4) = 16 × 6 = 96 sq.ft
Radius of the inner circle = e 2 j
1 r

1 55. (3)
Area under white tiles = 96 = 32 sq.ft
3
Area under blue tiles = 96 – 32 = 64 sq. ft
64
no. of blue tiles = 16
2 2

53. (2) A

r a

the length of Rubberband


C a B = (3 × length of rectangle + perimeter of a circle)
2
10
= (3 × 10) + 2 × = 30 + 10 .
Let the side of square = a cm 2

BLAM–425
{ks=kfefr
59. (4)
56. (3)

C2
B
R
C1

O A

R+4
C

let the radius of swimming pool = R


A/q
Given,
11
OA = 12 cm r2 ×
25
br 4g 2
r2
and OB = 3 cm = OC
In ABO 11r 2
= r2 + 8r + 16 – r2
AB = 25
122 32 = 3 15

FG 1 IJ 11r 2
3 15 3 = 9 15 cm2 8r 16
ar (ABOC) = 2 × H2 K 25

57. (3) Required Ratio 11r2 – 200 r – 400 = 0


(r – 20) (11r + 20) = 0
b5g 2
b3g 2
25 9 16 20
= = r = 20, r = (not possible)
b5g 2
25 25 11
60. (1)
58. (4) D 4x C

3
A M N B
K 16 6
C2
C1 K 7x

4 the height of trapezium (h)


2
(4x + 7x) = 2
11

5 Area of the trapazium


radius of C2 (R) =
2 1
2
b
4x 7x g 2x = 176
3 4 5
radius of C1 (r) = = 1 cm 11x2 = 176
2
x=4
required ratio of C1 and C2
In DMB
BD2 = DM2 + MB2
LM OP 2

= 82 + (16 + 6)2
FrI M 1 P2

= G J = MF 5I P =
4
.
= 64 + 484 = 548
H R K M GH 2 JK P 25
N Q BD = 548 = 2 137 cm

BLAM–426
{ks=kfefr
61. (2) A 64. (4) P

45°

2a
a
B
R
A
45° –R
B C R 7 R
a
R
Q R C
P = 2a + 2a In ABQ
R2 + R2 = (7 – R)2
2a + 2a = 10 + 10 2
2R2 = 49 + R2 – 14R
R2 + 14R – 49 = 0
2a e 2 1 j 10 1 e 2 j
14 196 196
10 R=
a= 2
2
R= 7 2 7
10
AC = 2 10 cm
22 2
2 area of circle = R2 =
7
e7 2 7 j
62. (4) Here d = 3, l = 480, b = 160
Area of the path = 2d (l + b + 2d) = 462 – 308 2
= 2 × 3 (480 + 160 + 2 × 3)
= 6 × 646 a a
= 3876 m2 65. (2)
63. (2)
a a
c a
2
E 12 cm D
12

R Q
cm

F C a a
M
Area of the wasted metal
= Area of the square
sheet – Area of four equal circular disks
A P B
= 4a2 – 4 ×
FG a IJ 2

= (4 – ) a2
Given, H 2K
ED = CD = EF = AF = AB = BC = 12 cm
66. (1)
MD = MC = 12 cm = MF A
FC = 12 × 2 = 24 cm
EFCD is a Trapezium
FC ED 24 12 a a
RQ = 18
2 2

3
area of = × 18 × 18 = 81 3 cm2
4 B C
2a

BLAM–427
{ks=kfefr
Let AB = AC = a
A F
BC = 2a
A/q
P = 2a + 2a 70. (4) B O E

e
82 2 j a e2 2 j
= 8 cm
Now, C D

Here, AB = AC = 8, BC = 8 2
F 3a I
Area of Isosceles ABC area of hexagon (ABCDEF) = 6 × GH 4 JK 2

1
=
4
BC 4 AB b g bBCg2 2

3 3 2
1944 3 = a
1 2
= 8 2 4 64 128
4 a2 = 1296
a2 = 36
= 2 2 256 128 the perimeter of hexagon = 6a = 6 × 36 = 216 m
= 2 2 8 2 Cost for fencing = 216 × 11.50 = Rs. 2484
= 32 cm2
5
67. (2) the breadth of rectangle = 5
1
Now,
2 (l + b) = P
2 (l + 5) = 50
l = 25 – 5 = 20. 71. (3) 12
68. (2) let the Perimeter = 6x and length = x
using formula
2 (l + b) = P 24
2 (x + b) = 6x
b = 2x
Now,
Area = 2x × x
total area of figure
288 = 2x2
= (area of Rectangle) + (Area of circle with r = 12)
x = 12 cm.
+ (Area of circle with r = 6)
69. (2) A
b
= 24 12 g LMN 227 12 12
OP LM 22
Q N7 6 6
OP
Q
22
b b = 288 + 180
7
= 288 + 565.2 = 853.2 m2

B 33 C 72. (1) A D

P = 33 + 2b
125 = 33 + 2b
92
b= 46 cm
2
the length of equal sides = 46 cm B C

BLAM–428
{ks=kfefr
ABCD is a rectangle (x + 10) (x + 5) = x (x + 20)
Given, x2 + 15x + 50 = x2 + 20x
Perimeter = 40 m 20x = 15x + 50
let BC = 3x 5x = 50
CD = 2x x = 10 m
A/q Length = x + 20 = 10 + 20 = 30 m
2 (3x + 2x) = 40 [using formula] iQ'kZ dk area
2 × 5x = 40
= 30 × 10 = 300 m2
x=4
Area of ABCD = 3x × 2x 5. (3) gkWy ds pkjksa nhokjksa dk area
= (3 × 4) × (2 × 4) = 96 m2 = 2 × h (l + b)
= 2 × h(15 + 12)
Test–II Total area of floor and ceiling of hall
Questions Based on Area and gkWy osQ iQ'kZ rFkk Nr dk oqQy area
perimeter of four walls of a room = 2 × 15 × 12 = 360 sq.
360 = 2 × h × 27
1. (1) ekuk length= x m, rFkk breadth = 3 x m 360 20
4 h= = m
2 27 3
Rectangular hall dk area = Length × Breadth
Hall dk volume = l × b × h
3 2
= x 768 20
4 = 15 × 12 × = 1200 m3
3
4 u 768
x= 32
3 Test–III
l = 32 m rFkk b = 24 m Questions Based on Surface Area &
vr%] Difference = 32 – 24 = 8 m Volume of Cube and Cuboid
2. (4) length of room = 1517 cm 1. (1) Ratio of volume = 27 : 1
breadth = 902 cm 1 1

iQ'kZ dk area = 1517 × 902 Ratio of the edges b27g :b1g


3 3

= 1368334 cm2 1517 ,oa 902 dk HCF = 41 [ Volume = a3 tgk¡ a = side = 3 : 1]


Square dh izR;sd side dh length = 41 cm 2. (3) Largest rod dh yackbZ

dk area b41g 2

b16g b12g FGH 323 IJK


Square 1681 2 2
2
1024
256 144
minimum no. of tiles 9
1368334
= 814 3600 1024 4624 68 2
1681 22 m
9 9 3 3
3. (3) ekuk breadth = x m
Length = 2x metre 3. (3) Let l = 3 x, b = 2 x
pkjksa nhokjksa dk area and h = x
= 2 × h (l + b) Surface area = 2 (lb + bh + lh)
660 = 2 × 11 (2x + x) = 22 × 3x = 2(6x2 + 2x2 + 3x2) = 2 × 11x2
660 = 88 sq. cm
x 10
66 22 x2 = 88
iQ'kZ dk area = 2x = 2 × 10 = 200 m
2 2 2 x2 = 4
4. (3) ekuk breadth = x m x = 2 cm
Length = (x + 20) metre l = 6 cm, b = 4 cm rFkk h = 2 cm
iQ'kZ dk area = (x + 20) x sq. metre So, volume = lbh
According to 2nd condition, = 6 × 4 × 2 = 48 cm3

BLAM–429
{ks=kfefr
Volume of larger cube 21 × 15 × 100 × x = 630000
4 .(3) No. of Cubes = Volume of smaller cube
630000
x= = 20 cm
15 u 15 u 15 21 15 100
125
3u3u3 = 0.2 metre
5. (1) Volume of all three cubes = (43 + 53 + 63) cu.cm. 13. (3) Perimeter of a face of cube = 20 cm.
= (64 + 125 + 216) cm3 = 405 cm3
20
u, cube dk volume = 405 – 62 = 343 cu.cm. An edge of cube =
4
= 5 cm.

Edge of cube = 3
343 = 7 cm. dk volume = (edge)3 = (5)3 = 125 cu. cm.
Cube
Surface area = 6 × 72 = 294 sq. cm. 14. (2) Bigger cube dk volume = 6 × 6 × 6 = 216 cu.
6. (3) Internal measurement cm.
= (330 – 10) × (260 – 10) (110 – x) Cube dk volume = 1 × 1 × 1 = 1 cu. cm
According to the question, Number of uncoloured cubes = 4 × 4 × 4 = 64
(330 – 10) × (260 – 10) [because edge of uncoloured cube = 4 cm]
(110 – x) = 8000000 ml 15. (2) Volume of rain water = A rea of base × height
320 × 250 × (110 – x) = 8000000
2
110 – x = 100 1000000 = 20000 cu. metre
100
x = 10 cm = 1 dm
Water stored in pool = 10000 cu. metre
7. (4) Number of soap cakes
10000
56 u 35 u 28 ? Required water level = 10 metre
343 1000
8 u5u 4
8. (2) Volume of soil D C
16. (1)
6.5 3
= 48 × 31.5 × m
10
O
ekuk tank dh xgjkbZ = x m
A B
27 × 18.2 × x = 48 × 31.5 × 0.65
.
48
315 0.65
x= =2m
27 18.2
9. (1) Required percentage increment
E

= 50
FG 50
50 50 IJ
% = 125%
H 100 K top face dk diagonal = AC = BD = AB2 BC2
10. (2) Volume of pile = 20 cu. metre
= AB 2 = 8 2 cm.
= 20 × (100)3 cu.cm.
Clearly, AOD, DOC, AOB; BOA right prisms ds
,d brick dk volume = (25 × 12.5 × 8) cu.cm.
base gSa
Required number of bricks
OA = OC = OB = OD
20 100 100 100
= = 8000 = 4 2 cm.
25 12.5 8
izR;sd prism dk perimeter
11. (4) ekuk] ydM+h dh eksVkbZ = x cm = OA + OB + AB
Inner surface dk area = 2 (9 – 2x) (10 – 2x) + 2
=4 2 +4 2 +8
(9 –2x) (7 – 2x) + 2 (7 – 2x) (10 – 2x)
= 262
x = 1, equation dks satisfied djrk gSA
e
= 8 2 j
8 cm.

12. (2) Water level esa c<+kr


s jh = x cm (let) 2 b 2
izR;sd prism ds base dk area = 4 4a – b
According to the question,
2.1 × 1.5 × 10000 × x tgk¡ a = 4 2 cm.
= 630 × 1000 cu. cm.

BLAM–430
{ks=kfefr
b = 8 cm. 18. (3) Volume of prism = Area of base × height

2
rhuksa volumes same gksxk
1
=
4
8 4 e4 2 j – 82

19. (1)
= 2 128 – 64

= 2 64 = 16 sq. cm. 20
x
40
izR;sd part dk surface area = Perimeter of base
× height + 2 × area of base Smallest side = 20 cm
Largest side = 40 cm
e
= 8 2 8 j 8 + 2 × 16 ekuk third side = x cm
Cubiod dk volume = lbh
= e64 2 64 j
32 sq. cm. = 20 × 40 × x
Using option 1
= e96 64 2 sq. cm. j V = 18000
800x = 18000
17. (4) 18000
Y x= = 22.5 cm
X B 800
20 cm rFkk 40 cm ds chp vkrk gSA possible gSA
Z Using option (2)
V = 12000
800x = 12000
XY = YZ = ZX
120
= x= = 15 (Possible ugha gS D;ksafd smallest
82 82 8 2 cm. 8
side 20 cm fn;k gqvk gSA)
3
Area of base = × (side)2 Using option (3)
4
360
3 2 x= = 45 cm (Possible ugha gS D;ksafd Largest
8
=
4
× 8 2 e j
side 40 cm fn;k gqvk gSA)
3 Using option (3)
=
4
e8 2 j × e8 2 j 420
x=
8
= 52.5 cm (Possible ugha gS D;ksafd
= 32 3 sq. cm.
Largest side 40 cm fn;k gS)
pyramid dh slant height
20. (4) ekuk cubes dh la[;k = n
= 82 ( 4 2 )2 = 64 32 n × 1 cube dk volume = cubiod dk volume
n × 23 = 8 × 10 × 12
32 4 2 cm.
Area of lateral 10 12 8
n=
8
1 1
= × perimeter of base × slant height = ×3× n = 120
2 2
Cuboid dk surface area = 2 (lb + bh +hl)
8 2 ×4 2 = 2 (8 × 10 + 10 × 12 + 12 × 8)
= 96 sq. cm. = 2 (80 + 120 + 96)
Total surface area = 592 cm2
= 96 + 32 3 120 cubes dk surface area = 6(side)2 ×120
= 720 × 22
= 32 3 e j
3 sq. cm. = 2880

BLAM–431
{ks=kfefr
Surface area esa percentage increase 3. (3) ekuk] radius = r units
Height = 4 × 2Sr = 8Sr units
FG 2880 – 592 IJ 100
= H 592 K 2Sr = c r=
c
and h = 4c
2
2288
=
592
100 = 386.5% cylinder dk volume = Sr2h
21. (2) Given, the curved surface are of sphere = 1386 c2 c3
= 2
× 4c = cubic units
22 4
4× r2 1386
7
4. (3) Paperdks yackbZ esa fold fd;k x;kA
r2 = 110.25
Base dk circumference = 22 cm,
r = 10.5 cm
Now, Cylinder dh height = 12 cm

4 22 2 S r = 22
Volume = × 10.5 × 10.5 × 10.5
3 7 22
2 r 22
= 4851 cm3 7
22. (2) Given, let the height of cone = 6x
7
5x r cm
and radius = , 2
2
A/q,
Cylinder dh volume = S r2h

1 2 22 7 7
r h 12 = 462 cu.cm.
Volume of the cone = 7 2 2
3
5. (4) Cylinderls fudkys x, ikuh dk volume
2200 1 22 25x 2
6x Reqd. height (water level) esa deh
7 3 7 4
x3 = 8 11000 3
11 cm
22 35 35 7
x=2 u u
7 2 2

b6x g FGH 52x IJK


2

l=
2 13 x {  11 l = 11000 ml rFkk 1 ml = 1 cm
2
6. (2) ekuk cylinders dh radii 2r, 3r rFkk height 5h, 3h
13 2 Volume dk ratio
= 13 cm
2 2
(2r ) .5h 4 u5 20
= 20 : 27
(3r )2 .3h 9u3 27
Test–IV
7. (1) Lateral surface area of the cylinder = 2Srh
Questions Based on Cylinder, Cone,
Sphere, Hemisphere etc. 22 7
= 2 16 = 352 sq.cm.
7 2
1. (4) Right circular cylinder dk volume = Sr2h
8. (1) ekuk radius esa c<+krs jh x cm.
22
= × 5 × 5 × 21 = 1650 cu. cm. then cylinder dk volume = S(10 + x)2 × 4
7
ekuk height esa c<+ksrjh x cm.
2. (3) r12h1 r22h 2 Cylinder dk volume = S ×102 (4 + x)
S (10 + x)2 × 4 = S (10)2 (4 + x )
Fr I 2
h2 2 Ÿ (10 + x)2 = 25 (4 + x)
GH r JK
1
2 h1 1 Ÿ 100 + 20x + x2 = 100 + 25x
Ÿ x2 – 5x = 0
r1 2
2:1 Ÿ x (x – 5) = 0
r2 1 Ÿ x = 5 cm

BLAM–432
{ks=kfefr
9. (4) Curved surface area of cylinder = Area of rect- 13. (2) Volume of water flowing per second = Sr2h
angular tin foil
22 7 u 7 u 5 3
= u m = 0.077 m3
2 rh 16 22 7 10000
22 Volume of tank
Ÿ 2u u r u 16 16 u 22
7 = 3 × 5 × 1.54 m3
7 = 23.1 m3
Ÿr cm
2 231
.
Required time =
Cylinder dk volume = Sr2h 0.077
22 7 7 = 300 seconds = 5 minutes
u u u 16 = 616 cm2 14. (1) Curved surface of cylinder = 2Srh
7 2 2
in other condition
10. (1) Area of the curved surface
1
1 Radius = r ; height = 6h
= × 462 = 154 sq.cm 3
3
? Curved surface
2Srh + 2Sr2 = 462
154 + 2Sr2 = 462 1
2 r 6h (2 rh ) 2
2Sr2 = 462 – 154 = 308 3
308 308 7 ? Increase nqxquh gksxhA
r2 = = = 49 15. (3) Curved surface of cylinder = 2Srh = a
2 2 22

r=
csyu dk oØi`"B = 2Srh = a ...(i)
49 = 7 cm
Base dk area = Sr2 = b ...(ii)
22 2Srh = a
10 10 14
11. (3) Required height 7 On squaring
22
7
e
152 10 2 j 4S2r2h2 = a2

a2
22 4Sbh2 = a2 h2 =
u 10 u 10 u 14 4Sb
7 11 ˜ 2 m [Putting the value from (ii) /(ii)]
22
u 25 u 5
7 a
h= cm
2 b
12. (3) Cylinder dk external radius.
16. (4) cylinder dk radius = r units, height = h units
8
4 cm a
2 2 r=a r= units
2
Internal radius
Cylinder dk volume = r2 h
6
3 cm a2
2 V= × ×h
4 2
8
4 V
6 cm h= units
a2
17. (1) Height = 2 R units
20 cm

Radius = R units
Cylinder dk volume = S R2h
Volume of iron used S (R)2 (2R) = 2 SR3
4
b 4g 2
20 b3g 20 2
18. (2) 2 r 8 r 4 r=

22
20 b16 9g 22 20 7 440 cm3 1 2 1 4 4 112
7 7 V= r h 21 = cu.cm.
3 3

BLAM–433
{ks=kfefr
19. (2) Ratio of the volumes of cone
1 22
45 (282 72 28 7)
1 3 7
r12h
3 F r I FG 3 IJ
1
2 2 9
=
1
r22h
GH r JK H 4 K
2
=
16
= 9 : 16 1 22
45(784 49 196)
3 7
3
1 22
1 2 45 1029
20. (2) r h 1232 3 7
3
= 48510 cu. cm.
1 22
Ÿ r2 24 1232 25. (2) C r1
3 7
O B
1232 3 7 A
Ÿ r2 49 14 cm h
22 24
l = 14 cm
? r = 49 = 7 cm
Semi-circular sheet dh yEckbZ (ACB) = Sr
? Slant height ( l ) h2 r2 22
= 14 44 cm
7
24 2 72 625 = 25 cm
Slant height = 14 cm.
? Curved surface = Srl
Circumference of the base of the cone
22
7 25 = 550 cm2 44
7 = 2Sr1 = r1
7
21. (1) Trick
44
44 r1 r1 7 cm
r2 h1 2 7
2 3
4r h2
h= l2 r12 142 72
h1 2 4 8
=8:3
h2 3 1 3 = 7 3 cm = 7 × 1.732 12 cm
26. (4) Slant height of the tent (l)
22. (2) ekuk radius = 4x cm rFkk slant height (l) = 7x cm
= 122 92
22
Srl = 4x 7 x = 792
7 = 144 81 225 = 15 m
792 7 Curved surface area = Srl
x2 = 9
22 4 7 = (3.14 × 12 × 15) m2
x=3 Total cost
radius = 4 × 3 = 12 cm = (3.14 × 12 × 15 × 120) = 67824
23. (4) Curved surface area = Srl 27. (4) Cylinder dk volume = Sr2h
22
16 l =
2992 = b 20 20 g
9 cm3
7 7
= 3600 S cm3
r1 Cone dk volume = 3600 S cu.cm.
r 1
r S R2 H = 3600 S
3

2992 1
× R2 × 108 = 3600
22 × 16 × l = 2992 l= = 8.5 m 3
22 16
24. (1) Volume of bucket 3600 3
R2 = = 100
108
1
h (r12  r22  r1r2 )
3 R= 100 = 10 cm

BLAM–434
{ks=kfefr
28. (4) Increase in height = 0%
h2 H h
1 2. =
Volume = area of base × height (H h )2 h
3
Percentage increase (H h )3 H h 3
3 =2 = 2
h
F
= Gx y
xy I
J%
h
H 100 K
H H 3
2 –1
3
+1= 2 =
F
= G100 0
100 0 I
J% = 100%
h h 1
H 100 K
h 3
=1: 2 –1
1 2 H
29. (3) Volume of cone = r h
3 31. (4) cus gq, cone dk radius = 9 cm
1 Its height = 12 cm
= 15 15 20 = 1500 cm3
3 1
Smaller cone dk volume Cone dk volume = 3 Sr2h

1 1
= 1.5 1.5 5 = 3.75 cm3 = × S × 9 × 9 × 12 = 324 S cm3
3 3
Number of smaller cones 32. (2) Lateral surface area of cone
1500 = Sr
= = 400 h2 r2
3.75
1 2
30. (4) O Its volume = Sr h
3
According to the question,
A r B
1 2
Sr h2 r 2 = 3 Sr h
B
A R
rh
OA = h unit h2 r2 = 3
AA = H unit Squaring both sides,
AB = R unit
r 2h 2 h2 r2 1
A B = r unit h2 + r2 = 2 2 =
9 r h 9
A B || AB
OA B = OAB
h2 r2 1 1 1 1
OB A = OBA 2 2 + 2 2 = 2 + 2 =
r h r h 9 r h 9
OAB ~ OA B
33. (4) Radius of the base of cone = r units
OA A B h r
= = 1 2
OA AB H +h R r h
Volume =
nksuksa rjiQ square djus ij 3

Curved surface area = r h 2 r2


h2 r2
(H h )2 R2 3 vh3 c2 h2 9v 2

1 2 1 1 2 1 2 1
= 3 r h h 3 – 2r 2 (h 2 r 2 )h 2 9 2 4 2
r h
Sr h = SR2 (H + h) – Sr h 3 9
3 3 3
= S2r2h4– S2r2h4 – S2r4h2 + S2r4h2 = 0
2 2 1
Sr h = SR2 (H + h) r
3 3 34. (3) Radius of the base (r1) = , Slant height = r
4
r2 H h
2 = r2
R2 h Curved surface area = Sr1l =
4

BLAM–435
{ks=kfefr
35. (4) ekuk FQ = r1, DP = r2 36. (4) ekuk sphere dk radius = r1 units
,oa BO = r3 ,oa hemisphere dk radius = r2 units

AQ = QP = PO =
h
4 2 F r1 I 3 1 r1 1
3 Sr 3 = Sr23
3 1 3
GH r2 JK 2 r2
= 3
2
AFQ ,oa ADP ls, 37. (3) Curved surface area of hemisphere = 2Sr2
A
22
= 2 11 11 = 760.57 sq.cm.
7
F G 38. (1) Surface area of sphere
Q
4Sr2 = 8S r2 = 2 r= 2 unit
D E
P sphere dk volume
4 3 4 3
B O C =
3
r
3
e 2j
FQ AQ r1 1
= 8 2
DP AP r2 = 2 = r2 = 2r1 = cubic units
3
AFQ ,oa ABO ls] 39. (3) Radius of sphere = r units
FQ AQ
BO
=
AO Surface area of sphere 4 r2 4
= =4:3
Surface area of hemisphere 3 r2 3
r1 1
r3 = 3 = r3 = 3r1 2 3
40. (4) Volume of hemishpere = Sr
3
V1 : V2 : V3
According to the question,
1 h 1 h 2
=
3
Sr12 ×
3
: S r
3 3 1
e r22 r1 r2 j 2 3
3
r1
6.4 Fr I 3

1 h 2 2 3 = 21.6 GH r JK
1

: S r
3 3 2
e r32 r2 r3 j 3
r2 2

= r12 : r1e
2
4r12 j e
2r12 : 4r12 9r12 6r12 j 64 FG 4 IJ 3
FG 2 IJ 3
= = =
2
= r1 : 7r1 : 19r1 2 2
= 1 : 7 : 19 216 H 6K H 3K
2nd Method r 1 : r2 = 2 : 3
A 4 3
41. (4) sphere dk volume 3
r

1
4 3 88
F G
3
r
21
u 14b g 3

1 Q
88 14 u 14 u 14 u 7 u 3
D
P
E r3 u
21 4 u 22
1
r = 14
B O C The curved surface area of the sphere
= 176 × 14 = 2464 cm2
let OA = 3
42. (4) r1= 3 cm] r2 = 4 cm] r3 = 5 cm
AQ = QP = OP = 1.
Total volume
AQG ~ APE ~ AOC
QG = 1, PE = 2, OC = 3 4
= S(33 + 43 + 53 ) cm3
r1 = 1 r2 = 2 r 3 = 3 3
v1 : v2 : v3 = (12 × 1) : (22 × 2 – 1) : (32 × 3 – 8) 4
= 1 : 7 ; 19 = S (27 + 64 + 125) cm 3
3

BLAM–436
{ks=kfefr
46. (1) Volume of earth : Volume of moon
4 S × 216 = 4 Sr3
3
3
=
4 3 4
r :
FG r IJ 3
= 64 : 1
[r = u, sphere dk radius] 3 3 H 4K
r3 = 216 r = 6 cm 47. (3) Volume of the solid metallic sphere /
diameter 2r = 12 cm 4 3
= r
2nd Rule 3
r1 r2 r3 R D
4 2048
3 4 5 6 12 = (8) 3 = cm3
3 3
6 8 10 12 24
ekuk NksVs sphere dk radius = x cm
diameter (D) = 12 cm
4 3 2048 2048
43. (4) ekuk igys sphere dk radius = r cm 64 × x x3 = 8
3 3 64 u 4
rFkk nwljs sphere dk radius = (r + 2) cm
x= 3
8 2 cm
Now, Difference between surface area
Required ratio
= 4 {(r 2)2 – r 2} = 352
= 4S. (8)2 : 4S (2)2 = 64 : 4 = 16 : 1
22
So, 4 u {(r  2 – r )  (r  2  r )} 352 4
7 48. (4) Sphere dk volume = 3 Sr3
352 u 7
2 × 2 (r + 1) =
4 u 22
nksuksa spheres dk volume
352 u 7 4 4 4
(r + 1) = 7 = S (r13 + r23) = S (13 + 63) = S (1 + 216)
4 u 4 u 22 3 3 3
So, r = 7 – 1 = 6 cm
FG 4 217
IJ
44. (1) Original sphere dk volume =
4
(6)3 = 288S cm3
= H3 K cu. cm.
3
ekuk hollow sphere dk radius = r cm, then
ekuk NksVs spheres dh radii 3x, 4x rFkk 5x cm Volume of the iron of this sphere
4
(3x )  ( 4 x )  (5x )
3 3 3
= 288S 4
3 = S (93 – r3) cu.cm.
3
4 According to the question,
(27x3+64x3+125x3) = 288S
3
4 4
4 = S (93 – r3 ) = × 217
u 216x 3 = 288S 3 3
3
729 – r3 = 217
288 3 r3 = 729 – 217 = 512
x3 = =1 x=1
4 216
r3 = (8)3 r = 8 cm
Required radius = 3 × 1 = 3 cm
Required thickness
45. (3) External radius = 3 cm
Internal radius = (3 – 0·5) = 2·5 cm = 9 – r = 9 – 8 = 1 cm.
49. (1) Inner and outer surface areas of the bowl =
4
Ball dk volume 3
dR 1
3
 r1
3
i 4Sr2

4 22 22
3
u
7
n
(3)3  (2 ˜ 5)3 s =4×
7
× 3.5 × 3.5

4 22 = 154 sq. cm.


u (27 – 15 ˜ 625)
3 7 Cost of painting
4 22 143 2 5
u u 11 ˜ 375 47 cm3 = 154 × = 77
3 7 3 3 10

BLAM–437
{ks=kfefr
50. (4) hemispherical bowl dk internal surface area =
l= r2  r2 r 2
2Sr2
2 Sr
2
Curved surface =
= (2 × 3.14 × 6 × 6) sq. cm
= 226.08 sq. cm (  Curved surface will be Srl = Srl gksxk)
2
Curved surface of hemi-sphere = 2Sr
4 3 4
51. (4) Volume of lead = r = 23
3 3 2 r2
Ratio 2 :1
ekuk gold dh thickness = x cm 2 r2
Gold dk volume 55. (2) ekuk ikuh ds level esa c<+krs jh x cm gksxh
4
=
3
e(2 3
x) 3
2 j cu.cm 2×
4
(3) 3 ( 6) 2 x
3
4 4
(2 x )3 23 =
e 23
j 72S = 36 Sx
3 3
(2 + x)3 – 23 = 23 (2 + x)3 = 8 + 8 = 16 72
x= 2 cm.
36
(2 + x)3 = 23.2
2+x= 2 3
2
56. (4) ekuk cylinder dk radius = r units.
2 + x = 2 × 1.259 = 2.518
x = 2.518 – 2 = 0.518 cm Height = 8r units

2 Its volume = Sr2 × 8r = 8Sr3 cu.units


52. (3) Volume of hemisphere = SR3 cu. units
3 r
Radius of sphere = units
4 3 2
Volume of sphere = Sr cu. units
3
Volume =
4 FG r IJ 3

According to the question, 3 H 2K


2 4 3
SR3 = 4 × Sr r3
3 3 cu.units
R3 = 8r3 R = 2r units 6
Number of spherical balls
R
r units
2 8 r3
6 = 48
1 r3
Radius of sphere × Radius of hemisphere
2
Volume of the cube 363
57. (2) =
8 64 Volume of the sphere 49
53. (1) Ratio of the volumes of spheres =
289 17
x3 363
4 3 =
r1 8 8 8 4 3 49
3 = r
4 3 17 17 17 3
r2
3
x3 363 4 22 121 4 22
= × =
r13 F8I
= G J
3 r1 8 r3 49 3 7 49 7
r23 H 17 K r2 17
x3 11 11 11 2 2 2
54. (2) =
r3 7 7 7

x 11 2 22
= = = 22 : 7
r 7 7
58. (3) Cylinder dk volume = Sr2h
22
= × 10 × 10 × 21 = 6600 cm3
7

BLAM–438
{ks=kfefr
64. (4) ekuk rod dh length = h cm
Volume of cylinder = Volume of 6 spheres
4 3
Sr2h = 6 r
3

4
h= 6 r 8 50 = 400 cm = 4 m
3

22
Cone dk volume 65. (2) 22 × 20 × x = 12 3.5
7
= 6600 – 4400 = 2200 cm3
22 3.5
1 2 2200 x=
2200 = 10 h 2200 = h 7 22 20
3 21
= 0.025 m = 2.5 cm
h = 21 cm
66. (2) S1 = surface area of sphere = 4Sr2
59. (3) Volume of required water = 2 × volume of cone
S2 = curved surface of the circumscribed cylinder
= 2 × 27S = 54Scm3
= 2SRH = 2S (2r) (2r) = 8Sr2
60. (3) Cylindrical vessel dk volume = Sr2h
2 1 S1 4 r 2 1 1
Cone dk volume = 3 r h = S1 = S2
S2 8 r 2 2 2

67. (2) Volume of the material of the hollow cylinder


r 2h
Number of cones = =3
1 2 4 4
3
r h =
3
e
3 3
S R –r =
3
3
j
S 5 –3
3
e j
Volume of cylinder 4 4 98
61. (2)
Volume of cone =
3
b
S 125 – 27 =
3
g
S cm3

r12h1 Fr I Fh I
2
F 3I 2
2 ekuk cylinder dh radius R cm
=
1 2
= 3. GH r JK GH h JK
1
2
1
2
= 3 GH 2 JK 3 8 4 98
3
r2 h 2 rks SR2 × 3 = 3
3 4 98
= 3 3 3: 2 R2 = = 49 R= 49 = 7 cm
2 8
62. (4) Volume of the solid sphere Diameter = 2 R = 2 × 7 = 14 cm
4 3 4 68. (3) Ratio of volumes = cone : cylinder : hemi-sphere
= r = 7 7 7 cm3
3 3
1 2
ekuk cylindrical wire dh yackbZ = h cm = r 2h : S r2 h : S r3
3 3
4
SR2h = 7 7 7 1 3 2 3
3 = Sr : Sr3 : Sr [ r = h ]
3 3
4
7×7×h= 7 7 7 1 2
3 = :1: =1:3:2
3 3
28
h= cm 69. (3) Volume of the two spheres of radius 6 cm. each
3
4 4
Volume of cylinder 3 =2× S r3 = 2 × × S × (6)3 = 576 S cm3
63. (2) = 3 3
Volume of cone 1
According to the question,
r12h 3
r1 = r 2 S × 12 × 12 × h = 576 S
1 2 1
r2 h
3 576
h= = 4 cm
Diameter of cylinder = Diameter of cone 12 12

BLAM–439
{ks=kfefr
70. (1) Whole surface of the remaining solid Height h1=1.6 h
= 2Srh + Sr2 + Srl V1 = S r12 h1
where l = slant height of cone
FG r IJ 2
1.6 h = 0.4 S r2 h
l= r 2
h 2 = 3 2
4 = 9 16
2
H 2K
= 25 = 5 cm Reduced in volume
Required area = S r2 h – 0.4 S r2 h = 0.6 S r2 h
= (2 × S × 4 × 3 + S × 3 × 3 + S × 3 × 5) cm2
= (24S + 9S + 15S) cm2 = 48S cm2 0.6 r 2h
% reduction 100 60
71. (1) Volume of water r 2h

4 1 4 78. (4) Initial volume of cubid


= Sr3 + × Sr3
3 4 3 = x×2x×3x = 6x3 cu. unit

5 5 New volume = 2x × 6x × 9x
= Sr3 = S cm3 = 108x3 cu. unit
3 3
72. (4) Required percentage increase Effective change = 108x3 – 6x3
=102x3 cu. unit
F
= GH x y
xy I
J%
100 K 102 x 3
Increment
6x 3
F
= GH100 100
100 100 I
J % = 300%
100 K = 17 times
79. (4)
73. (2) Percentage decrease
Area of base
10 100 1 = Sr2 = 16 × 5 = 80 ...(i)
= 100 = = 9 %
100 10 11 11
1
74. (4) Single equivalent decrease for 50% and 50% Volume = Sr2h
3
50 50 IJ = 5 × 100 m3 ...(ii)
= (– 50 – 50 +
100
% K equation (ii) dks (i) ls divide djus ij
= (– 100 + 25)% = – 75%
1 r 2h 5 100
Single equivalent percent for – 75% and 50% =
3 r2 80
75 50 IJ
= (– 75 + 50 –
100
% K h
=
25
3 4
= (– 25 – 37.5)% = – 62.5%
Negative sign decrease show djrk gS 75
h=
75. (1) Required percentage increase 4
= 18.75 m
FG
= 8 8
8 8 IJ
% = 16.64%
H 100 K 80. (1)
76. (4) Required percentage increase

FG
= 40 40
40 40
% = 96%
IJ
H 100 K
77. (2) Initial volume of cylinder V
= S r2 h
In 2nd condition,
r
Radius r1 2
2 3
Hemisphere dk volume = 3 r

BLAM–440
{ks=kfefr
tgk¡ r = radius = 7 cm 2 3
83. (4) Hemi-spherical bowl dk volume = r
FG 2 22
7
IJ
7 7 cu.cm.
3
= H3 7 K FG 2 15 15 15 cm3
IJ
Conical part dk volume
= H3 K
1 2
,d bottle dk volume
= r h r h
3 5 5
= SR2h = 6 cm3
2 2
FG 1 22
7 7 7
IJ Number of bottles
= H3 7 K cu.cm.
2 15 15 15
Ice-cream dk volume = = 60
5 5
3 6
2 22 1 22 2 2
= 73 73
3 7 3 7 84. (4) Conical shape dk radius = r cm (let)
22 r
= 73 = 22 × 72 cylinder dk radius = cm
7 3
= 1078 cu.cm. Volume of water = Volume of cone cm3
81. (4) Larger sphere dk radius = R units
1 1
Its volume = S r2 h = S r2 × 24 = 8 S r2 cm3
3 3
4
R 3 cu. units Volume of cylinder = SR2H
=
3
1 =S×
FG r IJ 2
H=
r 2H
cm3
Smaller cone dk volume =
3
R 3 cubic units H 3K 9

Volume of smaller sphere r 2H


= 8Sr2
1 9
= R3
3 H = 72 cm

4 3 1 85. (2) Length of parallelopiped = 12 cm.


r R3
3 3 breadth = 6 cm
height = 6 cm
R3 R
r3 = r= 3 Total surface area
4 4
= 2 (12 × 6 + 6 × 6 + 12 × 6) cm3
Surface area of smaller sphere : Surface area of
larger sphere = 2 (72 + 36 + 72) cm3 = 360 cm3

FG r IJ e1 : 4 j
2
3
2
1: 2
4 86. (2) D C
HRK 3

1 16 m
82. (1) conical vessel dk volume S r2h
3

1 A 30 m B
= × S × 6 × 6 ×12 = 144 S cm3
3
AC = AB 2  BC2 = 30 2  16 2
Sphere dk radius = R cm
2 = 900  256 = 1156 = 34 m
8× S R3 = 144S
3 Distance travelled by elephant
= 34 – 4 = 30 metre
144 3
R3 = =9×3=3×3×3 Speed of elephant
8 2
30
R= 3
3 3 3 = 3 cm = = 2 m/sec
15

BLAM–441
{ks=kfefr

A
87. (2) 88. (1)

E F

C D

B O
CE = AC2 AE 2
OD = 7 cm., OF = 14 cm., = 152 82 225 64
AF = 7 cm.
AD = 14 cm. = 289 = 17 cm.
ACD ~ ABO CAE = CDB = 90°
CD AD CD 14 ACE common gSA
= =
BO AO 9 21 CBD ~ CEA

14 CB CE
CD = 9 = 6 cm.
21 BD AE
AEF ~ ACD 15 r 17
=
AF EF 7 EF r 8
= =
AD CD 14 6 17r = 120 – 8r 25r = 120

7 6 120 24
EF = = 3 cm. r= = cm.
14 25 5
frustum ds lowest part dk] curved surface area 24
Required ratio = 8 : =5:3
5
= (R + r) bR – r g 2
h2
89. (2) izR;sd smaller cylinder dk radius = 3.5 cm.
8 smaller cylinders dk C.S.A.
= (9 + 6) b9 – 6 g 2
72
22
= 15 × = 16 rh = 16 × × 3.5 × 5
9 49 7
= 15 58 sq. cm. = 880 sq. cm.
Area of 8 bases
frustum ds middle part dk curved surface area
22
=8× × 3.5 × 3.5 = 308 sq. cm.
= (6 + 3) b 6–3 g2
72 7
larger cylinder ds cps gq, Hkkx dk area
= 9 58 sq. cm.
topmost part ,d cone gS FG 22 14 14 154
IJ
= H7 K
Its slant height = EF 2 FA 2 = (616 – 154) × 2 = 924 sq. cm.
larger cylinder dk C.S.A. = 1320 sq. cm.
= 32 7 2 = 58 cm.
Required answer
Its curved surface area = rl = (880 + 308 + 924 +1320) sq. cm.
= × 3 58 sq. cm. = 3432 sq. cm.
90. (2) larger cylinder dk total surface area
3 58 sq. cm.
= 2 r (r + h)
Required ratio = 2 × 7 (7 + 18) sq. cm.
= 3 58 9 58 : 15 58 = 350 sq. cm.
=1:3:5 izR;sd smaller cylinder dk total surface area

BLAM–442
{ks=kfefr
= 2 r (r + h1) 144 u 3
= 2 × × 7 (7 + 6) sq. cm. r3 = r3 = 72 × 3
2
= 182 sq. cm.
3
rhuksa NksVs cylinders dk surface area r= 216 r = 6 cm.
= 3 × 182 = 546 sq. cm. 2 2 3
Metal dk volume = R3  r
Increase = 546 – 350 = 196 sq. cm. 3 3
Percentage increase
2
342S = R 3  144
196 3
= × 100 = 56%
350
486 u 3 u
91. (2) Cylinder dk C.S.A = 594 cm2. R3 =
2
Cylinder dk volume = 1336.5 cm3. R= 3
243 3
Cylinder dk C.S.A = 2Srh
R= 3
729
bldk volume = Sr2h
Sr h = 1336.5
2 R = 9 cm
2Srh = 594 ...(i) bldh thickness = R – r
2 =9–6
r h 1336.5
= = 3 cm
2 rh 594
bldk C.S.A = 2SR2
1336.5 u 2 = 2S×92
r=
594 = 162S
2673.0 94. (4) Cone dk original height = 45 cm
r=
594 u, cone dk height = (45 – 15) cm = 30 cm
r 4.5 A
r dh value equation (i) esa j[kus ij
22
2× × 4.5 × h = 594
7
O B 45 cm
594 u 7
h=
44 u 4.5
h = 21 cm
92. (3) Sphere dk radius, R = 14 cm. C R D
cone dk radius, r = 1.75 cm
u, cone dk volume
bldh height, h = 3.5 cm
Cones dh la[;k =n 1 2
= r h
3
Sphere dk volume = n × Cone dk volume
1 22
4 1 2 18480 = u u r 2 u 30
R3 = n u r h 3 7
3 3
4 × 143 = n × (1.75)2 × 3.5 1848 u 7
r2 = = r2 = 84 × 7
22
4 u 14 u 14 u 14
n= n = 1024
1.75 u 175
. u 3.5 r= 12 u 7 u 7 = r = 14 3 cm
2 3 AOB ~ ACD
93. (1) ;gk¡, 3
r 144
AO OB 30 14 3
R AC CD 45 R
r
r = 21 3
1
Cone dk volume = 3
R 2H

BLAM–443
{ks=kfefr
1 22 2 Volume esa percentage increase
= u
3 7
u 21 3 e j u 45
LM FG 102 IJ r . 102h 
2
2
r 2h
OP
1 22
= u u 21 3 u 21 3 u 45
= M
H 100 K 100 PP u 100%
3 7 MM r h 2

PQ
= 22 × 21 × 3 × 45
= 62370 cm3
N
95. (1) As per question,
=
b102g  100 u 100
3 3

=
61208
~ 6.1208 %
Cylinder dk C. S. A 2 100 3
10000
=
Cylinder dk T. S. A 5
97. (4) Cube dh side = 14 cm
2 rh 2 14
b
2 r h r
=
5 g Each hemisphere dk radius = 2
= 7 cm

5h = 2h + 2r
3h = 2r

3h
r
2
cps gq, part dk volume
= cube dk volume – nksuksa hemispheres dk volume
bldk T.S.A = 3080
2Sr (h + r) = 3080 4 3
= (side)3 – r
3
2u
22 3h
u h
3h FG
= 3080
IJ
7 2 2 H K = 14 
3 4 22
u u 73
3 7
22 15h 2 = 2744 – 1437.33
u = 3080
7 2 = 1306.67 cm3
3080 u 14
h2 = 98. (2) A
22 u 15
140 u 14
h2 =
15

28 u 14
h=
2

2
h = 14 cm
3
B C
3h 3 2
r= = u 14
2 2 3
Cone dh height
r = 7 6 cm
h = 24 cm.
bldk volume = Sr2h bldk base area
22 2 2 22 u 7 u 7 u 6 u 14 2 = 154 cm2
=
7
u 7 6 e j u 14
3
=
7 3 r2 = 154
22
= 154 × 2 × 14 6 = 4312 6 cm2 r 2 = 154
7
96. (3)ekuk Cylinder dk radius = r units r2 = 49
bldh height = h units
r= 49 = 7 cm
102r
u;k radius = bldk curved surface area = rl
100

102h = r r2 h2
ubZ height = 100

BLAM–444
{ks=kfefr
22 256 213
= 7 72 24 2 a3 =
7 1000
= 22 × 625
256 u 213 3
= 22 × 25 = 550 cm2 a= 3 = 8.4 4
1000
99. (2) A = 2 r (h + r) + 2 r2
101. (2) Volume of the wall
B = 2 rh
A 3 FG
= 30
30
5
IJ FG 4 25 3 IJ
;gk¡, B 2
H 100 K H 10 100 K
= 45 – 6 = 39 m3
2 r (h r) 2 r2 3
2 rh 2 FG 39 8 IJ
H 9K 13600
h r r 3 no. of Bricks = FG 20 16 8 IJ
h 2 H 100 100 100 K
h 2r 3 102. (4) required volume = Area of pool × depth
h 2
h = 4r
FG 1 b1 3g 20
IJ 10 = 400m3
bldk volume = 4312 cm3
= H2 K
r2h = 4312
22 103. (4) A
r2 4r 4312
7
4312 7
r3 = h
22 4
r3 = 49 × 7
B F
r= 49 7 M 5
r=7
13
nksuksa bases ds areas dk sum
=2 r2 N 5
C E
22
= 2 7 7 = 308 cm2 5
7

3 3 4 4
100. (4) Sphere dk volume = 3 r = 3 21 D
Cube dk volume = sphere ds volume dk 80%
Given,
4 80 32
= 213 = 213 NE = ND = 5 cm = MF
3 100 30
FE = 5 × 2.6 = 13 cm
32 3 80 let AM = h
Hemisphere dk volume = 30 (21) 100 Surface area of the object = 770
rl + 2 rh + 2 r2 = 770
256 213
= 22
300 5 (l + 13 × 2 + 10) = 770
nksuksa spheres dk volume 7
= Hemisphere ds volume dk 80% l + 26 + 10 = 49 l = 13

4 256 80 AM (b) = 132 52 = 12 cm


2u a3 = 213 u
3 300 100 total height of the object
= AM + MN + ND
8a 3 256 u 213 u 8 = 12 + 13 + 5 = 30 cm
=
3 3000 ‡‡‡

BLAM–445
{ks=kfefr
ADVANCE LEVEL QUESTIONS
1. The area of a trapezium whose parallel sides are 4. O is the centre of a circle with AC = 24 cm, AB =
55 cm and 40 cm and non parallel sides are 20 7 cm and BOD = 90°. The area of shaded re-
cm and 25 cm is gion will be
,d leyac prqHkZt dk {ks=kiQy D;k gksxh ftldh lekarj Hkqtk,¡ O ,d o`r dk dsUnz gSA AC = 24 lseh] AB = 7 cm lseh rFkk
55 lseh- ,oa 40 lseh- gSa rFkk vlekarj Hkqtk,¡ 20 lseh rFkk 25 BOD = 90° Nk;kafdr Hkkx dk {ks=kiQy gS &
lseh gSa% A

(1) 900 cm2 (2) 950 cm2


2 C O B
(3) 1000 cm (4) 1050 cm2
2. A kite is in the shape of a square with a diagonal
32 cm and an isosceles triangle of base 8 cm and
sides 6 cm, each is to be made of three different D
shades as shown. How much paper of each side (1) 283.97 cm2 (2) 280 cm2
has been used it? (3) 281.97 cm2 (4) 282.97 cm2
5. Water is flowing at the rate of 5 km/h through a
,d irax ,d oxZ ftldk fod.kZ 32 lseh gS rFkk ,d lef}okgq
pipe of diameter 14 cm into a rectangular tank
f=kHkqt ftldk vk/kj 8 lseh rFkk vU; Hkqtk,¡ 6 lseh gSa ds which is 50 m long and 44 m wide. The time in
vkdkj dk gSa izR;sd Hkkx rhu fofHkUu jax ds gSa tSlk fd fp=k which the level of water in the tank will rise by 7
esa fn[kk;k x;k gSA Hkkx-III dks cukus esa fdruk dkxt yxsxk\ cm is
,d vk;rkdkj Vadh ftldh yackbZ 50 ehVj rFkk pkSM+kbZ 44
A
ehVj gS] esa ,d ikbi ftldk O;kl 14 lseh gS] ls 5fdeh@?kaVk
ds nj ls ikuh Hkjk tk jgk gSa le; fudkysa ftlesa Vadh ds ikuh
I O 32 cm dk Lrj 7 lseh Åij gks tk,xk\
B 32 cm C
(1) 8 hr (2) 7 hr
II (3) 4 hr (4) 5 hr
D 6. A ware house is used as granary. it is in the shape
m

III 6 cm of a cuboid surmounted by a half cylinder. The


6c

E 8 cm F base of the ware house is 6 m × 14 m and its


height is 8 m. The surface area of the non-cuboi-
(1) 17.00 cm2 (2) 17.10 cm2
dal part of the ware house is
2
(3) 17.40 cm (4) 17.92 cm2
,d eky xksnke dks vUukxkj ds :i esa bLrseky fd;k x;k gS
3. A floral design on a floor is made up of 16 tiles, ;g ,d ?kukHk ds Åij v¼Z csyu ds vkdkj esa gSA dk vkèkkj
which are triangular, the sides of the triangle
being 28 cm, 9 cm and 35 cm. The cost of polish-
6 eh × 14 eh rFkk Å¡pkbZ 8 ehVj gSA ds xSj /ukHkh; Hkkx dk
ing the tiles at the rate of 50 paise per cm 2 is i`"B {ks=kiQy gS&
,d lrg ij 16 f=kHkqt kdj VkbYl ls ,d iQwy dh fMtkbu
cuk;h x;h gSA f=kHkq t dh Hkq tk,¡ 28 lseh] 9 lseh rFkk 35
8m
l seh gS
a
A 50 i S l sUVhehVj 2 dh nj ls VkbYl dks ikW fy'k
l s@
djus dk [kpZ gS& 6m 14 m
1122 2 1222 2
m

9 cm (1) m (2) m
c

7 7
28

35 cm
1111 2 1022 2
(3) m (4) m
7 7
7. A small terrace at a hockey ground comprises of
10 steps each of which 20 m long and built of
(1) 700 (2) 656
1
(3) 705.60 (4) 805 solid concrete. Each step has a rise of m and
4

BLAM–446
{ks=kfefr
words on an average. The number of words would
1
a tread of m. The total volume of concrete re- be written using a bottle of ink containing one-
2 fourth of a litre will be
quired to build the terrace is
,d iQkmUVsu isu dk ihik csyukdkj gS ftldh yackbZ 7 lseh
,d gkWdh eSnku dk pcwrjk 10 lhf<+;ksa ls cuk gqvk gS ftlesa rFkk O;kl 0-5 lseh gSA isu ds ihis esa iwjh Hkjh L;kgh vkSlru
izR;sd dh yackbZ 20 ehVj gS tks Bksl daØhV ls cuk gqvk gSA
1
1 1 275 'kCn fy[kus ds fy, bLrseky dh tkrh gSA 4 yhVj okys
izR;sd lh<+h dh ÅWpkbZ ehVj rFkk inpki ehVj gSA pcwrjs
4 2
okys L;kgh ds cksry ls fy[kas tkus okys 'kCnksa dh la[;k gksxh&
dks cukus esa yxs daØhV dk vk;ru gS& (1) 40,000 (2) 20,000
(3) 60,000 (4) 50,000
12. A bucket made up of a metal sheet is in the form
of frustum of a cone of height 16 cm with radii of
its lower and upper ends as 8 cm and 20 cm,
1 m
m
respectively. The cost of bucket, if the cost of metal
20

2 sheet used is 15 per 100 cm2 will be


1 m
4
1 m 3 m /krq ds pknjs esa cuh ,d ckYVh tks fd ,d 'kadq ds fNUud
2 4 ds vk/kj dh ftldh Å¡pkbZ 16 lseh RkFkk fupys rFkk Åijh
Nksjksa dh f=kT;k,¡ Øe'k% 8 lseh rFkk 20 lseh gS ckYVh dk ewY;
(1) 137.5 m3 (2) 135 m3
(3) 140 m3 (4) 144.5 m3 D;k gksxk ;fn /krq ds 100 lseh2 pnjs dh dher 15 #i, gS\
8. Two cones with same base radius 8 cm and height (1) 290 (2) 390
15 cm are joined together along their bases. The (3) 293.90 (4) 299
surface area of the shape so formed is 13. A drinking glass is in the shape of a frustum of a
nks 'kadq ftudh vk/kj f=kT;k 8 lseh rFkk Å¡pkbZ 15 lseh gS dks cone of height 14 cm. The diameters of its two
muds vk/kj ds vksj ls tksM+ fn;k x;k gSA bl izdkj cus vkÑfr circular ends are 4 cm and 2 cm. The capacity of
the glass is
dk i`"B {ks=kiQy gS&
(1) 840 cm2 (2) 820 cm2 ,d ihusokyk fxykl ,d 'kadq ds fNUud ds vkdkj dk gS
(3) 855 cm 2
(4) 810 cm2 ftldh Å¡pkbZ 14 lseh gSA nksuksa o`rh; Nksjksa ds O;kl 4 lseh rFkk
9. A cone is divided into two equal parts by drawing 2 lseh gSA fxykl dh /kfjrk D;k gS\
plane through the mid-point of its axis, parallel
to its base. The ratio of volumes of two parts is 1 2
(1) 103 cm3 (2) 102 cm3
,d 'kadq dks mlds v{k ds eè; foUnq ls mlds vk/kj ds 4 3
lekarj nks cjkcj Hkkx esa ckaVk x;k gSA nksuksa Hkkxksa ds vk;ruksa 1 1
(3) 102 cm3 (4) 105 cm3
dk vuqikr gS& 3 3
(1) 2 : 7 (2) 1 : 7 14. A fez, the cap used by the Turks is shaped like
(3) 7 : 1 (4) 3 : 7 the frustum of a cone. If its radius on the open
10. A hollow cube of internal edge 22 cm. is filled side is 10 cm radius at the upper base is 4 cm
with spherical marbles of diameter 0.5 cm. and and its slant height is 15 cm, then the area of
1 material used for making it will be
it is assumed that space of the cube remains
8 rqdksZ± }kjk iguh tkus okyh ,d Vksih ,d 'kadq ds fNUud ds
unfilled. Then, the number of marbles that the vkdkj dh gSA ;fn [kqys Hkkx dh f=kT;k 10 lseh gS vkSj Åijh
cube can accommodate is vk/kj 4 lseh gS rFkk bldh frjNh Å¡pkbZ 15 lseh gSa bls cukus
,d [kks[kys ?ku ftldh vkarfjd Hkqtk 22 lseh gS] 0-5 lseh esa yxh lkexzh dk {ks=kiQy&
O;kl okys xksykdkj ekcZy ls Hkjk x;k gSA ;g ekurs gq, fd
2 2
1 (1) 210 cm2 (2) 205 cm2
?ku dk Hkkx [kkyh gSa ekcZy dh la[;k tks ?ku lekfgr dj 7 7
8
ldrk gS& (3) 200
1
cm2
1
(4) 200 cm2
(1) 142244 (2) 142200 7 7
(3) 140044 (4) 142044 15. The slant height of a frustum of a cone is 4 cm
11. The barrel of a fountain pen, cylindrical in shape, and the perimeters (circumference) of its circular
is 7 cm. long and 0.5 cm. in diameter. A full bar- ends are 18 cm. and 6 cm. The curved surface
rel of ink in the pen can be used for writing 275 area of frustum will be

BLAM–447
{ks=kfefr
,d 'kadq ds fNUud dh frjNh Å¡pkbZ 4 lseh gS rFkk mlds FG 2 – IJ cm FG – 2IJ cm
2 2
o`rh; Nksjksa dh ifjf/;k¡ 18 lseh rFkk 6 lseh gSaA fNUud ds oØ (1) (2)
H 2K H2 K
i`"B dk {ks=kiQy gksxk &
(1) 48 cm2 (2) 40 cm2 (3)
FG 4 – IJ cm
2
(4) – 4 cm2
(3) 20 cm 2
(4) 25 cm2 H 2K 2
16. A metallic right circular cone 20 cm high and 20. Side of a square park is x metre. As given in the
whose vertical angle is 60°, is cut into two parts figure its upper side is decreased by 1m and right
at the middle of its height by a plane parallel to side is increased by 1 m. The effect in its area
its base. If the frustum so obtained be drawn into will be.

a wire of diameter
1
cm, then the length of wire
,d oxkZdkj ikdZ dh Hkqtk x ehVj gSA mijh fn'kk esa Hkqtk 1
16 eh ?kVk nh tkrh gs vkSj mlds nkfgus dks.k okyh Hkqtk 1 eh c<+k
will be nh tkrh {ks=kiQy D;k gksxk\
,d /kfRod yac o`rh; 'kadq ftldh Å¡pkbZ 20 lseh gS rFkk ftldk
mèokZ/j dks.k 60° gS dks mlds vk/kj ds lekarj ry }kjk mlds 1m
Å¡pkbZ ls chp esa nks Hkkxksa esa dkVk tkrk gSA bl rjg izkIr fNUud
dks rkj ds :i esa [khapk tkrk gS rks rkj dh yackbZ gksxh &
(1) 7900 cm (2) 7250 cm
(3) 7964.44 cm (4) 7900.15 cm
17. How many silver coins, 1.75 cm in diameter and
1m
of thickness 2 mm, must be melted to form a
cuboid of dimensions 5.5 cm × 10 cm × 3.5 cm
1
1.75 lseh O;kl rFkk 2 feeh eksVkbZ okys fdrus pk¡nh ds flDdksa (1) remain same (2) increase m2
x2
dks fi?kyk;k tk;s rkfd 5.5 lseh × 10 lseh × 3.5 lseh dk
?kukHk cuk;k tk lds\ 1
(3) decrease by m2 (4) decrease by 1 m2
(1) 400 (2) 200 x2
(3) 390 (4) 280
18. The height of the cone is 30 cm. A small cone is 21. The ratio of height and half perimeter of a room
cut off at the top by a plane parallel to its base. is 2 : 5. Except 15 m2 area of doors and windows
the cost of plastering 15 cm wide paper at the
1
If its volume is of the volume of the cone, at rate of Rs. 2 per metre on the four walls is
27 Rs. 260. The height of room is
what height, above the base, is the section made?
,d dejs dh Å¡pkbZ rFkk blds v¼Z&ifjeki dk vuqikr 2 % 5
fdlh 'kadq dh mQapkbZ 30 lseh- gSA 'kadq ds vk/kj ds lekUrj
gS njoktksa ,oa f[kM+fd;ksa ds 15 oxZ ehVj dks NksM+dj pkj
,d lery }kjk 'kadq ds mQijh Hkkx ls ,d NksVk 'kadq dkVk x;k
nhokjksa ij 50 lseh pkSMk+ dkxt 2 #i, izfr ehVj dh nj ls
1
gSA ;fn bldk vk;ru 'kadq ds vk;ru dk 27 gks] rks vk/kj yxkus dk dqy [kpZ 260 #i, gSA bl dejs dh Å¡pkbZ fdruh
gS\
ls fdruh m¡QpkbZ ij 'kadq dks dkVk x;k gS \
(1) 2-6 m (2) 3-9 m
(1) 6 cm (2) 8 cm
(3) 10 cm (4) 20 cm (3) 4 m (4) 4-2 m
19. AB is a tangent to a circle. The radius of circle is 22. The difference of areas of two squares on differ-
2 cm. The area of shaded portion is ent line segments is found to be 32 cm2. If the
o`r ij ,d Li'kZ js[kk AB gSA o`Ùk dh f=kT;k 2 lseh gSA length of bigger line segment is 2 cm more than
that of smaller line segment; the length of bigger
Nk;kafdr Hkkx dk {ks=kiQy
line segment will be.
A B nks fHkUu&fHkUu yEckb;ksa ds js[kk[k.Mksa ij [khaps x, oxksZ± ds
{ks=kiQyksa dk vUrj 32 lseh2 gSA ;fn cM+s js[kk[k.M dh yEckbZ]
90° NksVs js[kk[k.M ls 2 lseh vf/d gks] rks cM+s js[kk[k.M dh
45° yEckbZ gksxhA
(1) 7 cm (2) 9 cm
(3) 11 cm (4) 16 cm
O 23. On increasing the length and breadth of a rect-
angle by 1 m, its area increases by 21 m2. If its
length increases by 1 m but its breadth decrease
by 1m, the area of floor decrease by 5 m2. Perim-
eter of floor is.

BLAM–448
{ks=kfefr
,d vk;rkdkj dejs dh yEckbZ rFkk pkSM+kbZ esa ls izR;sd esa 1
A B
ehVj dh o`f¼ dh tkrh gS rks {ks=kiQy 21 oxZ ehVj c<+ tkrk
gSA ;fn yEckbZ esa 1 ehVj dh o`f¼ dj nsa rFkk pkSM+kbZ 1 ehVj
de dj ns]a rks iQ'kZ dk {ks=kiQy 5 oxZ ehVj de gks tkrk gSA O
iQ'kZ dh ifjfefr fdruh gSA
(1) 30 m (2) 32 m D C
(3) 36 m (4) 40 m
24. On a square field of side 63 m four hourses are (1) 8 – 16 cm2 (2) 8 ( + 6)
tied at the four corners in such a way that they (3) 8 ( – 6) (4) 16 – 8 cm2
do not reach at one another. The area of portion 28. In the adjoining figure, ABCD is a square and
they do not cover is how much ? EDC is an equilateral triangle where side of
63 ehVj Hkqtk okys oxkZdkj Hkw[k.M ds pkjksa 'kh"kksZ± ij pkj ?kksM+s
square is 12 cm and O is the centre, GC = 32
bl izdkj cka/s x;s gSa fd os Bhd ,d nwljs rd ugha igq¡p ikrsA
cm, then find the area of smaller circle (in cm2)
ftl {ks=kiQy dks ;s ?kksM+s u pj ik;s gksa] og fdruk gS\
nh x;h vkÑfr esa ABCD ,d oxZ gS rFkk EDC ,d leckgq
(1) 675-5 m2 (2) 780-6 m2
f=kHkqt gS] tgk¡ oxZ dh Hkqtk 12 gS rFkk O o`Ùk dk osaQUnz gSA GC
(3) 758-8 m 2
(4) 850-5 m2
25. In the figure, ABCD is a rectangle and area of = 32 cm rks NksVs o`Ùk dk {ks=kiQy Kkr djsa
ABE = 20 cm2, if EC = 3BE, then find the area
of rectangle in cm2 is–
E B
fp=k esa ABCD ,d vk;r gS rFkk ABE dk {ks=kiQy = 20 A
lseh-2, ;fn EC = 3BE rks vk;r dk {ks=kiQy (lseh-2 esa) Kkr
djsa &
A D 12 cm 12 cm

O
F
G
B C D C
E 12 cm
(1) 120 (2) 80
(3) 160 (4) 40 (1) (5 2 6 ) cm
2
(2) ( 2 3)
26. In the figure, ABCD is a square with side 20.
BFD and BGD is an are of circle with centre C (3) (5 2 6 ) cm
2
(4) ( 2 3)
and A, then what is the area of shaded region?
29. Sides of triangle are 12, 16 and 20 cm respec-
fp=k esa ABCD ,d oxZ gS ftldh Hkqtk 20 gSA BFD rFkk tively, At each corner there is a circle drawn whose
BGD o`Ùk osQ pki gSa tks osQUnz C rFkk A ls [khaps x, gSa rks radius is 2.5 cm, find the area of triangle which
Nk;kafdr {ks=k dk {ks=kiQy D;k gS \ is not enclosed by circle?
A B f=kHkqt dh Hkqtk,¡ Øe'k% 12, 16 rFkk 20 cm gS izR;sd dksus
ij ,d o`Ùk cuk;k x;k gS ftldh f=kT;k 2.5 cm gSA f=kHkqt
F
osQ mi{ks=k dks {ks=kiQy Kkr djsa tks o`Ùk ls f?kjk gqvk ugha gSA
G (1) 75.75 cm2 (2) 76.36 cm2
D C (3) 78.25 cm2 (4) 95.25 cm2
30. In the given figure, PQRS is a rectangle and a
(1) 200 – 800 (2) 200 – 400
semicircle with SR as diameter is drawn. Two iden-
(3) 800 – 200 (4) 400 – 200 tical circle drawn as shown in figure. If QR = 9
27. In the adjoining figure, ABCD is a square where cm, then what is the ratio of area of bigger semi-
DB is the diameter of the semicircle and O is the circle to smaller circle?
centre, Each side of square be 8 cm, then find
the area of unshaked portion nh x;h vkÑfr esa] PQRS ,d vk;r gS rFkk SR O;kl osQ lkFk
nh x;h vkÑfr esa] ABCD ,d oxZ gSA DB v¼Zo`r dk O;kl ,d v¼Zo`Ùk [khapk x;k gSA nks ,d leku o`Ùk [khaps x;s gSa tSlk
gS rFkk O o`Ùk dk osQUnz gSA oxZ dh izR;sd Hkqtk 8 lseh gS rks fd vkÑfr esa fn[kk;k x;k gSA ;fn QR = 9 lseh- rks cM+s
fcuk Nk;kafdr Hkkx dk {ks=kiQy Kkr djsa \ v¼Zo`Ùk rFkk NksVs o`Ùk osQ {ks=kiQy dk vuqikr D;k gS \

BLAM–449
{ks=kfefr
its base so as to divide it into two parts. The
P Q volume of the frustrum (ie the lower part) of the
cone is 88 cm3. The radius of the circular surface
FG 22 IJ
of the frustum H 7 K is :

,d yEc o`Ùkh; 'kaoqQ dh Å¡pkbZ rFkk f=kT;k Øe'k% 15 lseh-


S R 15 lseh- rFkk 5 lseh- gSA 'kaoqQ vk/kj osQ lekarj ry }kjk nks
Hkkx esa dkVk tkrk gSA 'kaoqQ osQ fNUud dk vk;ru (fupyk
(1) 27 : 2 (2) 2 : 27
Hkkx) 88 lseh-3 gSA o`Ùkh; i`"B dh f=kT;k gSA
(3) 27 : 4 (4) 4 : 27
(1) 3 (2) 3
97 88
31. AB and AC is a tangent to a circle. The radius of
(3) 3 28 (4) 3 83
circle is 2 cm and CB = 6cm, find the area of
shaded portion from the figure (in cm2) 34. A sector of a circle of radius 12 cm has the angle
AB rFkk AC o`Ùk dh Li'kZ js[kk,¡ gSaA o`Ùk dh f=kT;k 2 lseh- 60°. It is rolled up so that two bounding radii are
joined together to form a cone. The volume of
gSa rFkk CB = 6 lseh- rks Nk;kafdr Hkkx dk {ks=kiQy (lseh-2 esa) the cone is
Kkr djsa \ 12 lseh- f=kT;k okys ,d o`Ùk [kaM dk dks"k 60° gSA bls bl
A izdkj ?kqek;k tkrk gS fd nks lhek f=kT;k,¡ feydj ,d 'kaoqQ
C B cuk;k tkrk gSA 'kaoqQ dk vk;ru gS&
90°
E
60° 45° (1) (4 35 ) / 3 cm 3 (2) (2 35 ) / 3 cm 3

O (3) (8 35 ) / 3 cm3 (4) None of these


35. From a circular sheet of paper of radius 5 cm, a
sector of area 60% is removed. If the remaining
part is used to make a conical surface, then the
ratio of radius and height of the cone will be
FG 6 7 IJ cm
2
FG 7 IJ
6 cm2 5 lseh- f=kT;k okys ,d o`Ùkkdkj dkxt osQ lhV ls 60%
(1) H K (2) H K {ks=kiQy dk ,d o`Ùk[kaM gVk fn;k x;kA ;fn 'ks"k Hkkx ls ,d
FG12 7 IJ cm FG 7 IJ 'kaDokdkj lrg cuk;h tkrh gS rks 'kaoqQ osQ f=kT;k rFkk Å¡pkbZ dk
2 12 cm2 vuqikr D;k gksxk \
(3) H K (4) H K
32. A cylinder has a radius 8 cm and height 10 cm. It (1) 23 : 2 (2) 21 : 2
is melted to form a cone. 80% material is wasted
(3) 2 : 21 (4) 2 : 23
in this process.The cone is melted to form
frustrum. In this process 80% material is wasted. 36. The height and radius of the base of a right cir-
The frustrum is melted to form two equal cones cular cone are 15 cm and 5 cm respectively. The
of equal radius and height, whose height is 3 radius of the circular cross section of the cone
cm. In this process 80% material was wasted. cut by a plane parallel to its base at a distance of
What is the radius of each new cones? 6 cm from the base, then final ratio of curved
,d csyu dh f=kT;k 8 lseh- rFkk Å¡pkbZ 10 lseh- gSA bls surface area of cone to the frustum is:
fi?kykdj ,d 'kaoqQ cuk;k x;k gSA bl izfØ;k esa 80% lkexzh ,d yEc o`Ùkh; 'kaoqQ dh Å¡pkbZ rFkk vk/kj dh fØ;k Øe'k5
u"V gks tkrh gS 'kaoqQ dks fi?kykdj ,d fNUud cuk;k tkrk gSA 15 lseh- rFkk 5 lseh- gSA blosQ vk/kj ls 6 lseh- dh nwjh ij
bl izfØ;k esa 80% lkexzh u"V gks tkrh gSA fNUud dks vk/kj osQ lekarj ry }kjk f=kT;k dks dkVk x;k gS rks dks.k osQ
fi?kykdj nks leku f=kT;k rFkk Å¡pkbZ osQ 'kaoqQ cuk;s tkrs gSa oØ i`"B rFkk fNUud osQ oØ i`"B dk vuqikr Kkr djsa \
ftudh Å¡pkbZ 3 lseh- gS bl izfØ;k esa 80% lkexzh u"V gks (1) 16 : 9 (2) 14 : 9
(3) 9 : 16 (4) 9 : 14
x;kA izR;sd u;s 'kaoqQ dh f=kT;k D;k gS \
(1) 3.2 cm (2) 1.6 cm 6
37. A cylinder is filled to th of volume. It is then
9
(3) 1.28 cm (4) 8 cm
tilted so that the level of water coincides with
33. The height of a right circular cone and the ra- one half edge of its bottom and top edge of the
dius of its circular base are respectively 15 cm opposite side. In this process, 60 litre of the wa-
and 5 cm. The cone is cut by a plane parallel to ter is spilled. What is the volume of the cylinder?

BLAM–450
{ks=kfefr
6 P
,d csyu dk 9 Hkkx Hkjk gqvk gSA bls bl izdkj >qdk;k tkrk Q

gS fd ty dk Lrj blosQ vk/s Hkkx osQ fupys lrg dks rFkk


foijhr okyk Hkkx Åijh fdukjs dks Nwrk jgsA 60 yhVj ikuh fxj
x;kA csyu dk vk;ru D;k gS \
(1) 140 litre (2) 240 litre
(3) 180 litre (4) 360 litre T S R U
38. Water flows through a cylindrical pipe whose ra-
dius is 4 cm at 50 metres per second. The time it (1) 200 (2) 360
take to fill the empty frustum with height 24 (3) 180 (4) 240
metres, R = 8 m, r = 6 metre 43. Find the length of MN from the given figure :
4 lseh- f=kT;k okys csyukdkj ikbi ls ikuh 50 eh@lsdaM dh xfr uhps fn, x, fp=k esa] MN dh yackbZ Kkr djsa %
ls cg jgk gSA ml [kkyh fNUud ftldh Å¡pkbZ 24 lseh-] R = M N
A D
8 lseh-] r = 6 eh- gS dks Hkjus esa yxk le; gS 3 cm
(1) 17.72 s (2) 18.66 s
(3) 16.67 s (4) 19.20 s
8 cm 6 cm
39. The base of a right prism is an equilateral tri- B C
angle. If its height is one-fifth and each side of
the base is 8 times, then the ratio of the volume Radius of bigger circle is 8 cm medium circle is 6
of the old to the new prism is : cm & small circle is 3 cm.
,d ledks.k fizT;k dk vk/kj ,d leckgq f=kHkqt gSA ;fn cM+s o`Ùk dh f=kT;k 8 lseh- eè;e o`Ùk dh f=kt;k 6 lseh- rFkk
1
NksVs o`Ùk dh f=kT;k 3 lseh- gSA
bldh Å¡pkbZ 5 rFkk izR;sd Hkqtk vk/kj osQ 8 xquk gS rks iqjkus
(1) 8(2 3 2 ) cm (2) 3(2 2 3 ) cm
rFkk u;s fizTe osQ vk;ru dk vuqikr dk gS \
(1) 5 : 64 (2) 64 : 5 (3) 4(2 3 2 ) cm (4) (2 4 3 ) cm
(3) 3 : 5 (4) 5 : 3 44. In a triangle PQR, QR = RS = SP and PQ = PR,
40. A sphere is inserted into the cone completely, find the angle PQR?
where radius and slant height of the cone is 8 PQR esa QR = RS = SP rFkk PQ = PR PQR Kkr
cm and 17 cm respectively, then find the total djs a \
surface area of sphere?
,d xksys ,d 'kaoqQ esa iw.kZr% lekfgr gS tgk¡ 'kaoqQ dh f=kT;k P
rFkk frjNh Å¡pkbZ Øe'k% 8 lseh- rFkk 17 lseh- gSa rks xksys osQ
lEiw.kZ i`"B dk {ks=kiQy Kkr djsa \
(1) 310.75 cm2 (2) 192.16 cm2 S
(3) 250.49 cm2 (4) 289.65 cm2
41. ,d vk;rkdkj {ks=k 60 ehVj yack rFkk 15 ehVj pkSM+k gSa blosQ
Q R
,d dksus esa 15 ehVj yack] 4 ehVj pkSM+k 2 ehVj xgjk ,d
[kkbZ [kksnh x;h gS rFkk blls fudkyh x;h feV~Vh 'ks"k Hkkx ij (1) 18° (2) 45°
leku :i ls iSQyk;h x;h gS rks {ks=k osQ Lrj esa o`f¼ Kkr djsa (3) 72° (4) 36°
(1) 15.38 cm2 (2) 14.28 cm2 45. In the given triangle PQR show below PU, QT and
2 RS are medians which intersects each other at
(3) 12 cm (4) 17.52 cm2
common point O. QR = 18 cm, PR = 12 cm, the
42. In the following figure find the area of trapezium?
length of the median RS = 15 cm. Find the length
If PT = QU of PS.
uhps fn, x, fp=k esa leyac prqHkqZt dk {ks=kiqy Kkr djsa ;fn fn, x, PQR esa] PU, QT rFkk RS ekfè;dk,¡ ,d nwljs dks
PT = QU O ij dkVrh gSa QR = 18 cm, PR = 12 cm ekfè;dk RS
PU = 41 cm dh yackbZ 15 cm gS PS dh yackbZ Kkr djsa \
PS = 9 cm

BLAM–451
{ks=kfefr
(1) 18 3 (2) 27 3
P
(3) 32 3 (4) 36 3
49. In the given figure. ABC is an equilateral triangle.
S T If the area of bigger Circle is 1386 cm2, then what
O
is the area (in cm2) of smaller circle?
nh xbZ vkÑfr esa ABC ,d leckgq f=kHkqt gSA ;fn cM+s o`Ùk dk
Q
U R {ks=kiQy 1386 cm2 gSA rks NksVs o`Ùk dk {ks=kiQy (cm2) D;k gS\
(1) 1 (2) 154
(1) 5.2 cm (2) 3 cm (3) 288 (4) 462
(3) 4 cm (4) 6 cm 50. The base angle of an isoseles trapezium is 45°. If
46. The Cross-Section of a Canal is in the Shape of the shorter side and both the equal sides are 10
an isosceles trapezium which is 3m wide at the cm each. What is the area of the trapezium?
bottom and 5 m wide at the top. If the depth of
the canal is 2m and it is 110 m long, what is the
,d lef¼ckgq leysc osQ vk/kj dk dks.k 45° gS] ;fn NksVh
maximum capacity of this canal? Hkqtk vkSj nks cjkcj Hkqtk,¡ izR;sd 10 cm gS rks ml leyac dk
{ks=kiQy D;k gksxk\
LM Take 22 OP
N 7 Q e
(1) 50 2 j
50 sq cm
,d ugj dh vuqizLFk dkV ,d lef}ckgq leyEc osQ vkdkj
esa gSa] ftlosQ ry dh pkSM+kbZ 3 eh vkSj Åij osQ fljs dh pkSM+kbZ e
(2) 50 2 100 sq cmj
5 eh gSA vkSj ;fn ml ugj dh xgjkbZ 2 ehñ gS yackbZ 110 eh-
gS] rks bl ugj dh vf/dre /kfjrk D;k gSA e
(3) 100 2 50 sq cmj
LM 22
ysuk gS OP e
(4) 100 2 100 sq cm j
N 7 Q
(1) 1760 cubic metres (2) 1650 cubic metres 51. Two sides of a plot measuring 32 m and 24 m
(3) 1056 cubic metres (4) 880 cubic metres and the angle between them is a perfect right
47. A farmer’s land is in the shape of a trapezium angle. The other two sides measure 25 m each
which has its parallel sides measuring 2.56 yards and the other three angles are not right angles.
and 3.44 yards and the distance between the par- The area of the plot in m2 is—
allel sides in 1.44 yards. fdlh eSnku dh Hkqtk,¡ 32 m rFkk 24 m gS rFkk osQ iw.kZr%
The cost of ploughing the land is Rs. 1800 per ledks.kh; gS] vU; nks Hkqtk,¡ izR;sd 25 m gS rFkk vU; 3 dks.k
square yards. What amount will (in Rs.) have to
90° ugha gSA eSnku dk {ks=kiQy m2 esa Kkr djsa\
be spent in order to plough the entire land?
(1) 768 (2) 534
,d fdlku osQ ikl leyEc vkdkj dh tehu gS] ftldh
(3) 696.5 (4) 684
lekukrj Hkqtkvksa dh yEckbZ 2-56 xt rFkk 3-44 gS] rFkk
lekukarj Hkqtkvksa esa nwjh 1-44 xt gSA tehu dks tksrus dk [kpkZ SHORT ANSWERS
1800 #i, izfr oxZ xt gSA iwjh tehu dks tksrus osQ fy, fdruh
jkf'k (#i, esa) [kpZ gksxh\ 1. (2) 2. (4) 3. (3) 4. (1) 5. (2)
(1) 3672 (2) 6732 6. (1) 7. (1) 8. (3) 9. (2) 10. (1)
(3) 7776 (4) 8214 11. (4) 12. (3) 13. (2) 14. (1) 15. (1)
48. Smaller diagonal of a rhombus is equal to length 16. (3) 17. (1) 18. (4) 19. (1) 20. (4)
of its sides. If length of each side is 6 cm, then
21. (3) 22. (2) 23. (4) 24. (4) 25. (3)
what is the area (in cmt) of equilateral triangle
whose side is equal to the bigger diagonal of the 26. (3) 27. (3) 28. (3) 29. (2) 30. (2)
rhombus? 31. (1) 32. (2) 33. (1) 34. (3) 35. (3)
,d leprqHkqt Z dk NksVk fod.kZ mldh Hkqtkvksa dh yEckbZ osQ 36. (3) 37. (4) 38. (3) 39. (1) 40. (4)
cjkcj gSA ;fn izR;sd Hkqtk dh yEckbZ 6 lseh gS] rks ml leckgq 41. (2) 42. (2) 43. (3) 44. (3) 45. (2)
f=kHkqt dk {ks=kiQy (lseh2) D;k gS] ftldh Hkqtk leprqHkqZt osQ 45. (2) 46. (4) 47. (3) 48. (2) 49. (2)
cM+s fod.kZ osq cjkcj gS\ 50. (4) 51. (4)

BLAM–452
{ks=kfefr
DEF dk area
EXPLANATIONS
1. (2) ABCD fn;k gqvk trapezium
= SS b a S gb b S gb c g
AB = 55 cm
CD = 40 cm
= 10 10 b gb
6 10 8 gb10 6 g
AD = 20 cm
= 10 4 2 4
BC = 25 cm
CE || DA cuk,¡ rFkk 11gm AECD dks iwjk djsa Part III dk area = 8 5 = 8 × 2.24 = 17.92 cm2
3. (3) ekuk a = 28 cm, b = 9 cm rFkk c = 35 cm
D 40 cm C
a b c 28 9 35 72
S= = = = 36 cm
20 cm 25 cm 2 2 2
cm
20 Triangular tile dk Area

A 40 cm E F B
= SS b a S gb gb c g
b S
15 cm

CE = 20 cm rFkk AE = 40 cm
= 36 b36 28 gb 36 9 gb 36 35 g
Also, BE = AB – AE = 36 8 27 1
BE = 55 – 40 = 15 cm 2
= 36 6 cm
CF AB rFkk CF = h cm
EBC esa,
16 triangular tiles dk
a = 20 cm, b = 25 cm and c = 15 cm Area = 16 × 36 6 cm2
a b c 20 25 15 60 = 576 6 = 576 × 2.45 = 1411.2 cm2
S= = = = 30 cm
2 2 2
EBC dk area 1
cost of polising = 1411.2 ×
2
= b
SS a S gb gb c g
b S = 705.60
4. (1) AC = 24 cm, AB = 7cm rFkk BOD = 90°
= 30b30 20gb30 gb
25 30 15 g ABC ,d right angled gSA
= BC 2 = AB2 + AC2
30 10 5 15
BC 2 = 72 + 242
= 10 × 3 × 5 = 150 cm2
BC 2 = 49 + 576
also EBC dk area
BC = 625 = 25 cm
1
= ×b×h Area of shaded region
2
= Area of circle – Area of quadrant COD – Area of
1 BAC
150 =
× 15 × h
2
h = 20 cm r2 1
= r2 – – × AB × AC
Trapezium ABCD dk area 4 2

1 3 r2 1
= × (sum of parallel sides) × Height = – × AB × AC
2 4 2
1 3 1
= × (40 + 55) × 20 = × 3.14 × (12.5)2 – × 7 × 24
2 4 2
= 95 × 10 = 367.97 – 84 = 283.97 cm2
= 950 cm2 5. (2) ekuk x hrs. esa ikuh dk level x cm c<+ tk,xk
2. (4) DEF esa] flow dh speed = 5 km/hr
a = 6 cm, b = 8 cm rFkk c = 6 cm
flow dh yackbZ
a b c 6 8 6 = 5x km
S= = = 10 cm
2 2 = 5000x m esa x hrs.

BLAM–453
{ks=kfefr
1 1
nwljs Step dh height = 2 × = m
4 2
1 3
rhljs Step dk height = 3 × = m
4 4

7cm. 1 10
44
nlosa Step dh height = 10 × = m
4 4
50
daØhV dk dqy Volume
Cylindrical pipe dk diameter = 14 cm.
1 1 1 2 1 3
14 7 = 20 × + 20 × + 20 × + ...... +
Radius, r = = 7 cm. = m 2 4 2 4 2 4
2 100
1 10
Volume of water flowing through pipe in x h 20 ×
2 4

r2h =
22 FG 7 IJ 2
5000x 1 1
7 H 100K = 20 ×
2 4
[1 + 2 + ........ 10]

= 77 x m3 1 1 10 (10 1)
Volume of water = p × b × h = 20 × ×
2 4 2
7 = 137.5 m3
= 50 × 44 × = 154 m3
100 8. (3) cones dks join djosQ ,slk figure cusxk
77 x = 154
x=2
water rises by 7 cm. in 2h. 8 cm.
6. (1) ekuk 16 cm.
r = radius of cylinder 15 cm. 8 cm.
h = height of cylinder
;gk¡, r = 8 cm.
l = 14 m, b = 6 m, h = 8 m h = 15 cm.
S.A. of shape = C.S.A of Ist cone + C.S.A. of 2nd
Cuboid dh pkSM+k bZ
Cylinder dh radius = Cone
2
= 2 × rl
6 22
= =3m =2× ×r×
2 7 r2 h2
Cylinder dh height = cuboid dh yackbZ = 14 m
22
S.A. of non cuboidal part = C.S.A of half cylinder =2× ×8× 8 2 152
7
+ 2 × Area of one semi-circle
44 8 289
2 rh r2 =
rh + r2 = 2 7
2 2
44 8 17
22 =
r(h + r) = × 3(14 + 3) 7
7
5984
22 = = 854.85 cm.2
S.A. of non cuboidal part = × 3 × 17 7
7 855 cm.2
1122 2
= m 9. (2) A
7
h
1
7. (1) l = 20 m rFkk width = m izR;sd step dk B r C
2h
2
1
igys Step dh height = m
C'
4 B' R

BLAM–454
{ks=kfefr
ABC AB'C' 1
Volume of ink = l.
BC AB 4
=
B' C' AB' 22
cm.3 = 275 words
r h 16
R 2h 275 16
1cm3. =
R = 2r 22
vc, 1 275 16 1
× 1000 cm.3 = × 1000
1 4 22 4
mQijh Hkkx dk Volume r 2h 1
= 3 = 275 16 1000
uhpys Hkkx dk Volume 1 h 4r 2 r 2 2r 2 7 = = 25 × 2 × 1000
3
e j 22 4
= 50,000 words
nksuksa parts dk ratio 1 : 7
10. (1) Cube dk fdukjk = 22 cm. 12. (3)
Cube dk volume = (22)3 = 10648 cm.3
20 cm.
Marble dk diameter = 0.5 cm.
0.5 16 cm. l
r= = 0.25 cm.
2
8 cm.
8 cm.
4 22
Marble dk Volume = × (0.25)3
3 7 ;gk¡,
88 1375
. R = 20 cm.
= × 0.015625 = = 0.0655 cm.3 r = 8 cm.
21 21
h = 16 cm.
Marble ds }kjk cube dk Hkjk x;k space = Volume of
1 l = h2 (R r)2
cube – × volume of cube
8
= 16 2 (20 8)2
1
= 10648 – × 10648 =
8 256 144
= 10648 – 1331 = 9317 cm.3 =400
Required no. of marbles
= 20 cm.
space filled by marbles 9317 Bucket dk T.S.A.
= = = 142244
Volume of one marble 0.0655 = C.S.A of frustum + Area of base
11. (4) Cylindrical pen dh height = 7 cm. = l (R + r) + r2
0.5 = [l (R + r) + r2]
Radius = cm. = 3.14 [20 (20 + 8) + 82]
2
= 3.14 [20 × 28 + 64]
Fountain pen ds barrel dk volume = 3.14 × 624
= r2 h = 1959.36 cm.2
100 cm.2 dk cost = 15
=
22 FG 0.5 IJ 2
7
7 H 2K cost of 1959.36 cm.2 dk cost =
1959.36 15
100
22
= × 0.25 × 0.25 × 7 = 293.90
7 p
13. (2)
22 25 25 22 D C
= cm.3
10000 16
22 2 cm
cm.3 ink dk bLrseky djosQ pen ds }kjk 275 words A B
16
fy[ks tk ldrs gSaA p'
4 cm

BLAM–455
{ks=kfefr
h = 14 cm Conedh height = 20 cm
4 2 Frustum dh height = 10 cm
r1 = = 2 cm & r2 = = 1 cm
2 2 AOE es,a
h 2 OE r1
Drinking glass dk volume = r1 r 22 r1 r 2 cot 60° = =
3 OA 10
1 22 1 r1 10
= 14 12 22 1 2 r1 = cm
3 7 3 10 3

=
44
1 4 2 =
308
102
2
cm2 AO'C esa ,
3 3 3
O'C r2
cot 60° = =
AO' 20
14. (1)
1 r2 20
r2 = cm
3 20 3

h 2
Frustum dk volume = r1 r22 r1 r2
3
;gk¡,
u 10
FG 100  400  200 IJ
l = 15 cm =
3 H3 3 3K
r1 = 4 cm rFkk
7000
r2 = 10 cm = cm3
9
Total curved surface area
= (r 1+r 2)l + r12 ekuk wire dh yackbZ = h cm
= (4 + 10) 15 + 42] 1
bldk diameter = 1 6 cm
22
= [210 + 16]
7 1
Wire dk radius = cm
22 226 4972 2 32
= 210 cm2
7 7 7
15. (1) ;gk¡, Wire dk volume = r2h =
FG 1 IJ 2
×h
l = 4 cm
H 32 K
r1, r2 nksuksa ends ds radii Volume of wire = Volume of frustum
2 r 1 = 18 h 7000
9 32 32 9
r1 = cm
7000 32 32
h= cm
2 r2 = 6 9
3 h = 7964.44 cm
r2 = cm 17. (1) coin dk diameter = 1.75 cm
C.S.A of frustum = (r1 + r2)l 175
.
r1 =
9 3 2
= ( + )×4 h1 = 2 mm = 0.2 cm
= 12 × 4 = 48 cm2 Cylindrical coin dk volume = r12 h
16. (3) A 22 175
. 175
.
= 0.2
7 2 2

°
30

3 13.475
O E = cm3
28
r1 Cuboid dk volume
10 cm
r2 =l×b×h
60° = 5.5 × 10 × 3.5
B C
O' = 192.5 cm3

BLAM–456
{ks=kfefr
ekuk coins dh la[;k n EF AF
n × volume of 1 coin = volume of cuboid BG AG
13.475
n× = 192.5 r1 30 – x
28
r2 30
192.5 28
n=
13.475
= 400
r1 b30 – xg eq. (i) esa j[kus ij
r2 30
18. (4)
9
b
r12 30 – x g 10
r22

cm 9
FG 30 – x IJ b30 – xg 10
2

H 30 K
9
b30 – xg b30 – xg 10
2

30 30

b30 – xg 3
10
10 10
(30 – x)3 = 10 × 10 × 10
Figure ds vuqlkj ABC right circular conegS , height
(30 – x)3 = 103
GF = x cm, ,d NksVk AED dkVk tkrk gSA
30 – x = 10
ekuk radius of small cone is r1 rFkk radius of bigger
cone is r2. x = 20 cm

Volume of smaller cone smaller cone dks 10 cm m¡QpkbZ ls dkVk x;k gSA
2nd Method
1 2 1 2
3
r h
3
b
r1 30 – x g
h 3
1
Volume of bigger cone H 27
1 2 1 2
r h r2 30 10 r22 h 1
3 3 h = 10 cm
30 3
Volume of small cone
height from Base = 30 – 10 = 20 cm
1
u Volume of bigger cone 1
27 19. (1) AOB dk area = × OA × AB [  OA = AB]
2
1 2 1
3
b
r1 30 – x g 27
10 r22 1
= × 2 × 2 = 2 cm2
2
1 2 1
r1 (30 x) = 10 r22
3 27 r2
Area of sector [T = 45°]
360
1 2
27
3
r1 ( 30 x) 10 r22
(2)2 45
= = cm2
r22 360 2
9 r12 ( 30 x) 10

9r12 (30 x) 10r22 Shaded dk area = 2  2 cm2


FG IJ
H K
b
9 r12 30 – x g = 10 ...(i) 20. (4) Park dk area = x2
r22 New area = (x +1) (x – 1) = x2 – 1
AEF ~ ABG area 1 m2 oxZ ?kV tk,xkA

BLAM–457
{ks=kfefr
21. (3) ekuk height rFkk semiperimeter% 2x rFkk 5x gSA 25. (3) EC = 3x
vc] Length + breadth = 5x BE = x
Area of ABE = 20 cm2
pkjksa nhokjksa dk area = 2 × height (l + b)
1
= 2 × 2x × 5x = 20x2 × AB × BE = 20
2
50
dkxt dk area = 130 × 100 = 65 m2 1
× AB × x = 20
2
 pkjksa nhokjksa dk area = (65 + 15) = 80 cm2
AB x = 40 cm2
vr% 20x2 = 80 Area of Rectangle
= AB × BC
80
x= = 2 metre = AB × 4x
20 = 40 × 4 = 160 cm2
Required height = 2x = 2 × 2 = 4 metre 26. (3) Let we join D to B
22. (2) ekuk igys line segment dh length = x cm region DEBG dk area
nwljs line segment dh length = (x + 2) cm 1
= × 20 × 20 = 200
According to the question] 2
(x – 2)2 – x2 = 32 Area of region DFBG
(x + 2 – x) (x + 2 + x) = 32 Fr 2
Triangle dk area
I
(2x + 2) = 16 GH 360 JK 2
2x = 14
x=7
cM+s line segment dh length = 7 + 2 = 9 cm
F (20) 2 I
GH 4 200 JK 2
23. (4) ekuk iQ'kZ dh length = x metre rFkk breadth =
y metre rc] area = xy m2 400
–4
(x + 1) (y + 1) – xy = 21
x + y = 20 ...(i) = 200 – 400
xy – (x + 1) (y – 1) = 5 Shaded region dk area
x–y=4 ...(ii)
= Square dk area – region DFGB dk area
(i) rFkk (ii) dks solve djus ij] x = 12 rFkk y = 8 = (20)2 – (200 – 400)
dejs dk perimeter = 2(12 + 8) m = 400 – 200 + 400
= 40 metre = 800 – 200
24. (4) Required area 27. (3) Let we join O to E to find the area of sector
DOE
FG 63 63 – 4
1 22 63 63 IJ m2 O
H 4 7 2 2 K
= 850.5 m2 4

D E
4

r 90 nr 2 4 4
63 m =4
360 4
DO rFkk OE ls f?kjs shaded portion dk area
FG 4 1
4 4
IJ 2
Trick :
= H 2 K
= 8 – 16
shaded Hkkx dk area FG IJ
6 a
2

=
6 63 63 DB2 = DC2 + BC2
7 2H K 7 2 2 = 82 + 82
= 850.5m2 DB = 64 64 128 8 2

BLAM–458
{ks=kfefr
8 2 29. (2) A
OB = radius = 4 2
2
semicircle dk area 12 16
2 2
r ( 4 2) 4 4 2 30° 60°
2 B C
20
16
AB 2 + AC2 = BC2
unshaded portion dk area 122 + 162 = 202
1 144 + 256 = 400
= 16 – (8 – 16) + ×8×8 400 = 400
2
= 8 + 16 + 32 (;g ,d right triangle gS)
= 8 + 48 = 8 ( + 6) 1
28. (3) Equilateral triangle dk in-radius Right ABC dk area = 2 × 12 × 16 = 96 cm2

side 12 6 3
ml triangle dk area tks circle osQ vanj ugha gS
OF = 2 3 cm
2 3 2 3 3 3 F r 90 r 30
2 2
r 2 60 I
 (ABCD ,d square gS AB = BC = CD = AD =
= 96 GH 360 360 360 JK
ED = EC = 12 cm)
Fr r rI
2 2 2

A E B
= 96 GH 4 12 6 JK
F3 r r 2 r I
2 2 2
= 96 GH 12 JK
O I 6 r2
F = 96
12
G
O1
D C r2
H = 96
2
Let we join O to H and O to I
2.5 2.5 22
OHCI ,d square gS = 96
2 7
OH = HC = CI = OI = 6 cm
= 96 – 19.64 = 76.36 cm2
OC = 2 2
6 6 72 6 2
30. (2) P Q
ekuk NksVs circle dk radius = r r r
2

OC = OF + FG + GC r
r
6 2 2 3 2r 32 9 cm T
cm
9
=

6 2 4 2 2 3
r
R

2
S R
O
r 2 3 18 cm
NksVs circle dk area
= r2 ekuk cM+s circle dk radius = R
NksVs circle dk radius = r
= ( 2 3 )2
OQ = R 2, and CQ = r 2
= (2 3 2 6)
(phythagorous theorem ls)
= (5 2 6) cm2 OQ = OT + TC + CQ

BLAM–459
{ks=kfefr
R 2 R r r 2
Cylinder dk Volume = r2h
= × 8 × 8 × 10
R( 2 1) 2 1 Cone dk volume = 20% of volume of cylinder
r 9 (2 1 2 2 )
2 1 2 1 20
= × × 64 × 10 = 128
(  R = 9 cm) 100
= 27 – 18 2 Frustum dk volume = 20% of volume of cone

nks NksVs circles dk area 20


=
× 128 =
128
2 100 5
=2× r
Volume of 2 cones = 20% of volume of frustrum
=2× (27 – 18 2 )2
1 2 20 128
2 r h
= 2 (729 + 648 – 972 2 ) 3 100 5
= 2 (1377 – 972 2 ) 1 2 128
2 r 3
cM+s semicircle dk area 3 25

R2 9 9 81 r
64 8
= 1.6 cm
2 2 2 25 5
33. (1)
81 1
Required Ratio =
2 2 (1377 972 2 ) A

81
= 27 : 4=
4 3
31. (1) AOB = ABO = 45° O'
D E 15 cm
AO = AB = 2 cm
1
AOB dk area = 2 × 2 × 2 = cm2
O
2 B C
r 2 2 45
Sector AOF dk area =
360 360 2 5 cm
FG 2 IJ cm2
1
Shaded portion AFB = H 2K Cone dk volume = 3 R2h
 CB = 6 cm 1
CA = 6 – 2 = 4 cm = × 5 × 5 × 15
3
1 = 125
AOC dk area = 2 × 4 × 2 = 4 cm2 AOC ~ AO'E
AO' O' E
r 2 60 2 2 2
Sector AOE dk area = =
6
=
3
AO OC
360
h r
F
Shaded portion AEC = GH 4
2 IJ cm 2 15 5
3 K h = 3r
Frustum osQ nwljs part dk volume
2
Total shaded portion = 2 4 88
3 7 = 28
22
7
= 6 1
6 Cone dk volume = 3 r2h
32. (2)
1 2
(125 – 28) = r × 3r
3
97 = r3
Cylinder Cone Frustrum Cones r= 3
97

BLAM–460
{ks=kfefr
34. (3)
36. (3) A

cm
12
h l h
60°
C1 15 cm
R = 12 cm r O 1

6 cm 1
Sectordk radius = Cone dk slant height = 12 cm h
O C
Base sector dk Perimeter = Cone osQ vk/kj dk Pe- B
rimeter 5 cm

2 (12)60 AO' = 15 – 6 = 9 cm
=2 r
360
AO' O' C'
r = 2 cm AO OC
l = 12 cm
9 O' C'
Cone dh height = l 2 r 2 15 5
O'C' = r = 3 cm
= (12)2 (2)2 Frustum dk curved surface area

= 144 4 140 2 35 (R r ) h '2 (R r )2

1 (5 3) 6 2 (5 3) 2
Cone dk Volume = 3 r2h
8 40
1
= (2)2 2 35
3 8 2 10

L (8 OP cm Cone dk curved surface area = r h2 r2


35 )
= M 3
MN 3 PQ = 3 92 32
35. (3) = 3 90
Curved surface area of cone
Required ratio =
Curved surface area of frustrum
h l
3 90 3 3 10
=
8 10 8 2 10
r 9 : 16
37. (4) Let the total volume of the cylinder is 9x litre
Sector dk radius = Cone dk slant height = 5 cm
6
Volume of water = × 9x = 6x litre
40 9
× (5)2 × = r(5)
100
9x
r = 2 cm After tilting volume of water =
2
l = 5 cm
9x
Spilled water = 6 x
height = (5)2 (2)2 2

= 3x
25 4 60
2
= 21 x = 40 litre
Cylinder dk volume of cylinder = 9x
Required Ratio = 2 : 21
= 9 × 40 = 360 litre

BLAM–461
{ks=kfefr
38. (3)
1
Triangle dk area 2R H
50 m/s In radius, r = 2
Semiperimeter
l R
4 cm
1
R = 8m 2 8 15
2 120 24 cm
17 8 25 5
24 metre T.S.A of sphere = 4 r2
22 24 24
r = 8m =4 = 289.65 cm2
7 5 5
41. (2) Region dk area tgk¡ soil dks iSQykuk gS
Cone dk Volume
Required time = Cylinder dk cylinder = Area of field – Area of pit
= (60 × 15 – 15 × 4) m2
= 900 – 60 = 840 m2
1
(R 2 r 2 )h
3 60 m
r12h1
15 m
1 4m
(82 6 2 ) 24
3
(0.04)2 50 15 m
= 1000 second = 16.67 min.
39. ( ) Old New
Side a 8a
Height 5h h
Old ekuk] field dh Å¡pkbZ = h
Required volume ratio
New
field esa fcNk;s x;s Soil dk Volume = volume of
soil taken out from it
3
a 2 5h 15 × 4 × 2 = 840 × h
= 4 = 5 : 64 15 4 2
3
(8a )2 h h=
840
= 14.28 m
4
42. (2)

40. (4) P Q
M
41
c m
l = 17 cm 9 cm
H
r
T
S R U
40 cm
R = 8 cm ge tkurs gSa fd PST right angled triangle gSA
TS = RU, PT = QU & PST = QRU, So PST
fn;k x;k gS QRU.
R = 8 cm We are adjusting PST at QRU.
l = 17 cm Now, PSUM is rectangle.
H= (17)2 (8)2 1
Required area = 2 × × Area of PSU
2
= 289 64
1
= 225 = 15 cm =2× × 9 × 40
2
= 360 cm2

BLAM–462
{ks=kfefr
43. (3) A M O N D P
3 3
Y R
P 14 16 cm T
S
X O
Q
8
6
B C R
U
PXQ esa Here,
P
RO QO PO 2
OS OT OU 1
14
2 Here, Abollonius theorem ls]
QR2 + PR2 = 2(RS2 + SP2)
X Q (18)2 + (12)2 = 2(152 + SP2)
192 324 + 144 = 2(225 + SP2)
YQR esa 234 = 225 + SP2
SP2 = 9
R
SP = 3 cm.

9 46. (4) Max. Capacity of the Canal


32 = Area of Canal × depth
1 1
=
2
b3 5 g 2 110 =
2
8 2 110 = 880 m2
Y Q
7 47. (3)
Required distance = MN
= OM + ON D 2.56 C
= QX + YR
= 192 32 = 8 3 4 2
1.44
= 4(2 3 2 ) cm.
44. (3) ekuk P=x
A 3.44 B
P Area of trapezium (ABCD)
x 1 1
=
2
ABb CD g h=
2
b
2.56 3.44 g 1.44
S
2x 1
= 6 1.44 = 4.32 sq. yards.
2
2x 2x x total cost plough the land
Q R
432
 PQ = PR = 2x = 1800 × = Rs. 7776
100
2x + 2x + x = 180°
5x = 180° A D
48. (2) 6
x 36
PQR = 2x
6 O
= 2 × 36° = 72° 6
45. (2) The point where all three medians of triangle
meet is known as centroid of the triangle. The
centroid divides the median internally in the ra-
tio 2 : 1. B 6 C

BLAM–463
{ks=kfefr
In Rhombus, 50. (4)
Small diagonal = a = 6
D 10 C
then large diagonal = 3a 6 3
Now,
Area of equilateral triangle 10 5 2 10

3
= 6 3 6 3 45°
4
E A 10 B F
5 2
= 27 3 cm2
49. (2) In DAE,
DA = EA = 5 2

A EF = 5 2 e 2 j 10 10 2 10

1
P Area of trapezium =
2
e10 2 10 10 j 5 2
F E
Q 1
=
2
10 2e 2 j 5 2

O = 25 2 2 e 2j
= 50 50 2
B D C
51. (4) A 25 D

ABC is equilateral triangle area of large circle


22 2 22 2
= r 1386 r
7 7
r = 21 cm 25
32 40
In ABC
a
r=
2 3

a B 24 C
21 =
2 3 By pythagorean Triplet
24, 32, 40
a = 42 3
AC = 40 m
3 1
h=
2
42 3 63 cm AD b g ar of ABC =
2
24 40 = 384 cm2

OA : OD = 2 : 1 = 42 : 21 Now, In ACD
In AFE 25 25 40
S= 45 m
AQ = 42 – 21 = 21 2
Now,
AP : PQ = 2 : 1 area of ADC = b
Ss a s gb b s gb c g
PQ = 7
= 45 20 20 5
Area of smaller circle
= 300 m2
22 Area of the plot = 384 + 300 = 684 m2
‡‡‡
= 7 7 = 154 cm2
7

BLAM–464

You might also like